Está en la página 1de 198

ANTAMOK GOLDFIELDS MINING COMPANY vs. COURT OF intervino el Departamento del Trabajo y por la mediacion de Eladio C.

Leaño
INDUSTRIAL RELATIONS los obreros volvieron al trabajo en la noche del 6 de enero de 1939 en que los
G.R. No. 46892 | 1940-06-28 trabajos de mina se reanudaron paulatinamente.
IMPERIAL, M:
El 9 de enero de 1939 el Departamento del Trabajo endoso la disputa al
Esta es una apelacion mediante certiorari interpuesta por la recurrente contra Tribunal de Relaciones Industriales de conformidad con el articulo 4 de la
la orden dictada por el Tribunal de Relaciones Industriales el 6 de mayo de Ley No. 103 del Commonwealth y dicho Tribunal celebro la primera vista
1939 que le obligo a que reponga en sus anteriores trabajos o en otros del asunto el 13 del mismo mes en la Ciudad de Baguio. En esta vista se
substancialmente equivalentes a los 45 obreros enumerados en la peticion del discutieron una por una las 21 reclamaciones de la recurrida National Labor
31 de marzo de 1939 y a los 10 obreros encabezados por A. Haber que fueron Union, Inc., y se llego por las partes a un acuerdo sobre algunas de ellas, se
excluidos indefinidamente, dentro de 10 dias desde que reciba copia de la sometieron otras a la decision del Tribunal y las demas se dejaron pendientes
orden; que pague a estos 55 obreros los jornales que debieron haber percibido para ser vistas y resueltas mas tarde.
desde la fecha de su suspension o separacion hasta la de su reposicion; y que
pendiente de resolucion las otras cuestiones que las partes han sometido, la El 31 de marzo de 1939, hallandose pendiente aun de decision la mayor parte
recurrente se abstenga, bajo pena de desacato, de despedir o excluir, sin de las reclamaciones antes mencionadas, la recurrida National Labor Union,
permiso previo del tribunal, a cualquier obrero o empleado que se hallaba Inc., presento una mocion en que alego que el capataz A. Haber y otros 9
bajo su servicio en la epoca en que surgio la disputa que este actualmente obreros de la recurruente habian sido indefinidamente suspendidos el 29 del
trabajando en las minas o que sea repuesto en su trabajo de conformidad con mismo mes; que estos obreros habian sido transferidos anteriormente a
la orden; y contra la resolucion del mismo tribunal del 17 de agosto de 1939 trabajos exteriores con el fin de proporcionar a la recurrente una excusa para
que denego la mocioon de reconsideracion de la recurrente presentada el 26 separarles mas tarde del servicio; que otro grupo de cerca de 30 obreros
de mayo de 1939. fueron despedidos por la compañia sin motivo alguno y sin autorizacion del
tribunal; y que las suspensiones y separaciones que asi se hicieron eran actos
El 12 de diciembre de 1938 la recurrida National Labor Union, Inc., en de venganza y discriminatorios para los obreros, por cuya razon se pidio que
representacion de los obreros y empleados de la recurrente que eran los funcionarios de la recurrente responsables de dichos actos sean castigados
miembros de dicha union obrera, dirigio una carta a la recurrente solicitando por desacato y que la recurrente sea obligada a reponer a los obreros en sus
21 reclamaciones en favor de sus afiliados. La carta fue recibida por la oficina primitivos trabajos dentro de las minas y a pagarles sus salarios
de la recurrente en Manila en un sobre timbrado por la estafeta de Baguio el correspondientes al periodo en que fueron separados del servicio. La
30 del mismo mes. Los funcionarios de la recurrente convocaron a un recurrente contesto la mocion negando los hechos imputados y alego que
meeting a sus empleados el 2 de enero de 1939 y en el informaron a todos Haber y sus 9 companeros fueron suspendidos por su continua holgazaneria
sus obreros que algunas de las demandas se habian aceptado y se habian durante las horas de trabajo y por haberse negado constantemente a trabajar,
puesto ya en practica, otras serian consideradas y las restantes iban a ser y que los 45 obreros encabezados por el capataz Victoriano Madayag fueron
rechazadas por ser irrazonables, y se les aconsejo que no recurrieran a la despedidos por haber rehusado senalar a los responsables del maltrato del
violencia y observaran metodos legales en el arreglo de sus diferencias con capataz Juan Moldero en la manana del 30 de marzo de 1939. La mocion se
la recurrente. En la noche del mismo dia los obreros y empleados de la vio el 3 de abril de 1939 y en la vista las partes presentaron sus testigos. El
recurrente se declararon en huelga y abandonaron sus trabajos. La recurrente tribunal designo a uno de sus agentes especiales para que se constituya en las
dio cuenta inmediatamente de esta huelga al Departamento del Trabajo y minas de la recurrente y practicara una investigacion con el fin de
solicito su intervencion con el fin de solucionarla. El Secretario del Trabajo suplementar los hechos que se durante la vista. Despues de considerar las
designo a Adolfo Umengan, Investigador Especial del Departamento, y a pruebas presentadas ante el y los hechos hallados por el comisionado
Eladio C. Leaño, Defensor Publico de la Provincia Montanosa, para que nombrado, el tribunal en su orden del 6 de mayo de 1939 declaro probados
intervinieran y vieran la manera de solucionar la huelga. Estos funcionarios los hechos siguientes:
convocaron una conferencia a la que acudieron funcionarios de la recurrente,
representante de los huelguistas y Luis Lardizabal, Jefe de la Baguio "1. The discharges and indefinite suspensions alleged in the motion were
Federation of Labor, una organizacion obrera afiliada a National Labor made by the respondent without first securing the consent of the Court in
Union, Inc. Como resultado de la conferencia las partes convinieron en el violation of the order of this Court of January 23, 1939."
siguiente arreglo amistoso:
"2. The discharges and indefinite suspensions were made by the respondent
"AMICABLE SETTLEMENT without just cause."

"In order to have the present strike of the contractors and laborers of the En la misma orden el Tribunal de Relaciones Industriales hace las siguientes
respondent company who staged a walk-out on January 3, 1939, amicably consideraciones que apoyan las conclusiones a que ha llegado:
settled, the parties hereby mutually agree to end the said strike under the
condition that all laborers will be readmitted upon the execution of this "In the order of January 23, 1939, the respondent was enjoined to refrain from
agreement; provided, that all laborers whose services should be dispensed discharging any laborer involved in the dispute without just cause and
with due to lack of work in those tunnels where they are no longer needed without previous authority of the Court. It appears and no denial of the fact
will be given not less than fifteen days employment from the date of this is made by the respondent that the dismissal in one case and the alleged
settlement or resumption of work, and provided, further, that as soon as the suspension for an indefinite time in the other, which has all the effects of a
stopes in 1360 and 1460 levels are opened and the services of men are discharge, were made without seeking the authority of the Court."
needed, the company will give preference to efficient laborers when reducing
the personnel as above mentioned in those working places and may transfer "The charge that Haber and the group of nine laborers were indefinitely
them to other divisions to replace inefficient men." suspended because of continuous loafing and refusal to work was not
established. The real motive behind the lay off was the completion of their
"In witness hereof, the laborers represented by a committee composed of work 'outside.' Under the circumstances, the provision of the order of March
Messrs. Luis Lardizabal, Tomas Dirige, Victoriano Madayag, Maximo 21, to the effect that these men should be returned to their work underground
Conaoi, Daniel Lambinicio, and Juan Cerilo and the Antamok Goldfields after the completion of their work 'outside' should have been observed. The
Mining Co. as represented by its President, Mr. Andres Soriano, have respondent instead of complying with the order laid off the men."
hereunto placed their signatures this 4th day of January, 1939."
"The discharge of Victoriano Madayag and his forty-four companions as a
El convenio fue firmado por las partes el 4 de enero de 1939, pero los obreros result of the Moldero incident also lacks justification. In the case of Madayag,
no se presentaron sino a las 9 de la mañana del 6 del mismo mes. La gerencia although he was present with Haber when Moldero was attacked, neither one
de la recurrente no permitio, sin embargo, a ningun obrero que entrara en la is accused of the aggression. The two of them were conversing with Moldero
seccion subterranea conocida como "830 level" por la razon de que el aire se when the latter was stoned from behind without anybody apparently being
habia viciado con motivo de la huelga y era necesario renovarlo con aire puro able to point out the aggressor. Less justification can be found for the
con el fin de evitar desgracias personales. Esta precaucion la tomaron los discharge of the forty-four men as a result of the incident. The investigation
obreros como una negativa de la recurrente a que ellos trabajaran de nuevo, disclosed that at the time of the assault, they were at the Creek busy with their
por lo que se declararon otra vez en huelga. A los huelguistas se unieron por work. Both the distance and the topographical situation of the place where
simpatia los obreros que trabajaban en la mina denominada "680 division," the men were working, which is far and well below the bank of the place of
que es otra mina separada y situada a 3 kilometros de la fabrica. Otra vez the incident, precluded their hearing or seeing clearly what transpired above
them in the place where Moldero was assaulted. An ocular inspection of the majority of the workers joined openly the strike. It would not have been
premises made by the investigator confirmed this view. So far as is known, difficult for the respondent, with the means at its command, to find for itself
despite the investigations conducted by the officials of the company and the the employees and laborers who remained loyal to the company and to
policemen of the camp and by the constabulary authorities in Baguio, the consider those who struck as either members of the union or its
person or persons responsible for the stoning has not been determined. The sympathizers."
precipitate and unwarranted dismissal of the forty-five men after the incident
seems to have been spurred by an over anxious desire on the part of the "The respondent's claim as to the motive for the suspension and discharges
company to get rid of these men." lacks substance and support in the evidence and the inferences to be drawn
from it. From all what appears, it is inferred that the respondent desires to
"As previously found, in the order of this Court of March 21, 1939, about discourage membership in the union and to rout it if possible. The wholesale
134 underground laborers of the respondent were transferred and made to discharges were the expression of such desire. The acts in the mind of the
work 'outside of the mines' or surface work. The majority of these men were Court, are calculated to have two effects. They will not only immediately
muckers, miners, timbermen, trammers, and mine helpers and had to their affect the discharged laborers but would also discourage other laborers from
favor from 6 months to 5 years service in the mines of the company and not joining or remaining members of the union."
a few of them have done underground work in several capacities and in
different tunnels and divisions of the mine. Among them are found leaders "The allegation that it has always been the policy to consider the laborer's
of the movement of the laborers for higher pay and better working conditions connection with the company terminated upon termination of the working
which culminated in the strike called on January 3, 1939. These leaders have place in which he is employed is not supported by the facts. It has been shown
been prominent in the formation of the union and its activities and in that as a general rule when work in a place is completed, workers are
connection with the strike. The temporary transfer of these men to 'outside' transferred to another working place in one level or to another level, although
work was authorized by the Court in said order on the strength of the in some instances days may elapse before all the men in a bunch can be
assurance of the respondent that no more work suited for them inside the absorbed in different levels."
mines existed. It was directed, however, in the aforesaid order that as soon
as their work outside was completed the laborers should be immediately "It is alleged that mining operations in the property vary and involve several
returned to their respective work inside the mines. Subsequent events and types, and that a miner, for example, may be good in one type, but that it does
acts of the officials of the respondent in charge of the mines have convinced not necessarily follow that he can do good work in another type. And that the
the Court that work existed and exists for the men inside the tunnels and their employment of men in particular jobs not suitable for them increased the cost
transfers were made to provide an opportunity to the company to dispense of production as a result of lower output. Consequently; the respondent
with their services as soon as the work outside is completed. The unwarranted vehemently insists in its right of selecting the men that it should employ and
discharges of Haber and nine others and those of Victoriano Madayag and that in the exercise of this right it should not be restrained or interfered with
his forty-four companions amply demonstrated this conclusion. Upon the by the Court. It contends that as to the fitness of a laborer to do a particular
company's own admission, as shown in its reports in the records and upon type of work the opinion of the management or its technical men should be
the findings of the investigator of the Court, more than four hundred (400) respected. But all these arguments are meaningless in the face of the finding
workers of different classes among them, muckers, miners, timbermen, of the Court that the underground laborers transferred to the 'outside' work
trammers and capataces coming from different mines in the region have been are not wanting in experience, efficiency and other conditions alleged to be
employed by the respondent as fresh laborers. Almost all, if not all, of these found among the fresh laborers. The special qualifications to do particular
men are not members of the petitioner, the National Labor Union, Inc." work can not rightly be invoked in favor of the employment of new laborers
most especially in those cases of common or unskilled labor like muckers,
"At the same time the work in different tunnels and divisions in the mines trammers, helpers, etc."
are allegedly being completed, the old workers are being laid off. Although
a small number of the men found transfer to other divisions being operated, "Under normal circumstances, the exercise of Judgment of the employer in
the majority are being left without work. Instead of laying hands on the old selecting men he is to employ should not be interfered with. But when such
men laid off and making them work in the tunnels needing hands and judgment is arbitrarily exercised to the prejudice of members of a labor union
reinstating in the tunnel work those laborers transferred to the 'outside' whose rights should be safeguarded in consonance with the policies of the
department, the respondent preferred to take in and hire other workers law, the Court not only feels it justified but rightly its duty to interfere to
coming from different places because evidently they are not members of the afford protection to the laborers affected."
union."
La recurrente presento una extensa mocion de reconsideracion de la indicada
"There is no doubt in the mind of the Court that a good number of the orden, mocion que fue denegada por la resolucion del 17 de agosto de 1939.
positions given of the men who were employed after the strike numbering La orden del 6 de mayo de 1939 y la resolucion del 17 de agosto del mismo
more than four hundred to date could have been offered to the strikers who ano son las que dieron lugar a la apelacion interpuesta por la recurrente.
are now doing work 'outside' and other who have been laid off on the
allegation that the underground work in which they were engaged had been La recurrente sostiene que la Ley No. 103 del Commonwealth, conforme ha
completed. To believe that not a single man or say a few among the latter sido enmendada por las leyes Nos. 254 y 355, es anticonstitucional (1) porque
could have met the requirements set by the technical men of the company to infringe el principio de separacion de poderes; (2) porque por ella la
perform the different classes of work for which the fresh men were engaged Asamblea Nacional abdico de su facultad legislativa violando la doctrina
because they lack the required efficiency, experience, physique, intelligence sobre delegacion de poderes; (3) porque las facultades judiciales que la ley
and skill of the four hundred fresh laborers would be shutting the eyes of the confiere al Tribunal de Relaciones Industriales, consideradas separadamente,
court to realities. These men prior to the occurrence of the dispute, had son arbitrarias e irrazonables y permiten la privacion de la libertad y
worked for months and many for years in the mines of the respondent and it propiedad sin el debido proceso de ley; y (4) porque suponiendo que la ley
can not be easily accepted that their experience gained in their particular lines es valida y constitucional en su totalidad, la porcion, por lo menos, del
in the very property of the respondent would be inferior to that attained by articulo 20 que dispone que el Tribunal de Relaciones Industriales "adoptara
the other workmen in other mines in the district for an equal period of time. sus reglamentos de procedimiento" debe declararse nula e invalida porque
Their inefficiency as a whole group can not be successfully sustained now infringe el articulo 13 del Titulo VIII de la Constitucion de Filipinas que
because they were not transferred to surface work for this reason but because obliga al Tribunal de Relaciones Industriales a observar las reglas generales
of the alleged lack of work or completion of their work underground. Had de procedimiento aplicables a los tribunales de justicia. La recurrente alega
any of them been inefficient in the past, it can not be explained why such en este respecto que como a ella se le ha sometido a un procedimiento
laborer continued in the service as the records of the company abound with arbitrario y distinto del que se aplica a los demas litigantes en 105 tribunales
instances of discharges made in the past of laborers who were found either de Filipinas, se le ha negado el debido proceso de ley y el principio de igual
inefficient or incompentent or whose services were unsatisfactory." proteccion ante las leyes.

"The company asserts ignorance of the union affiliations of the men in the La Ley No. 103 del Commonwealth que, como su titulo indica, provee a la
mine but the evidence stands uncontradicted that before the strike was called proteccion del obrero, creando un Tribunal de Relaciones Industriales
a petition was presented by the men to the management carrying the facultado para fijar un jornal minimo para los obreros y la renta maxima que
signatures of about eight hundred (800) workers demanding higher pay and se ha de pagar por los inquilinos; para poner en vigor el arbitraje obligatorio
better working conditions. When the men struck, the operation of the mine entre patronos o propietarios y empleados o inquilinos, respectivamente, y
was completely paralyzed and there is a strong indication that a great prescribe penas por la infraccion de sus decretos, se ha promulgado por la
Asamblea Nacional en virtud de los preceptos contenidos en el articulo 5, propias reglas de procedimiento, lo cual contraviene el articulo 13, Titulo
Titulo II; articulo 6, Titulo XIII; y articulos 1 y 2, Titulo VIII, de la VIII, de la Constitucion que prescribe que el Tribunal Supremo dictara reglas
Constitucion de Filipinas que disponen: concernientes a los escritos de alegaciones, practica y procedimiento
uniformes para todos los tribunales de la misma categoria.
"ART. 5. El Estado cuidara de promover la justicia social a fin de asegurar
el bienestar y la establidad economica de todo el pueblo." El articulo 20 de la Ley No. 103 se lee asi:

"ART. 6. El Estado debera proteger a todos los trabajadores, especialmente "ART. 20. Reglamentos del Tribunal. El Tribunal de Relaciones Industriales
a las mujeres y a los menores de edad, y debera regular las relaciones entre promulgara sus reglas de procedimiento y tendra las demas atribuciones que
propietarios e inquilinos, y entre el trabajo y el capital en la industria y la en general corresponden a un tribunal de justicia: Entendiendose, sin
agricultura. El Estado podra establecer el arbitraje obligatorio." embargo, Que en la vista, investigacion y resolucion de cualquier cuestion o
conflicto, y en el ejercicio de cualquiera de sus deberes y facultades en virtud
"ART. 1. El Poder Judicial estara investido en un Tribunal Supremo y en de esta Ley, el Tribunal actuara de acuerdo con la justicia y la equidad y los
otros tribunales inferiores que se establezcan por ley." meritos'substanciales de la causa, sin consideracion a los tecnicismos o
formulismos legales, y no estara sujeto a cualesquiera reglas, tecnicas de
"ART. 2. La Asamblea Nacional tendra la facultad de definir, prescribir y prueba legal, sino que formara juicio de la manera que crea justo y
distribuir la jurisdiccion de los varios tribunales, . . ." equitativo."

En cumplimiento de los preceptos constitucionales transcritos, la Asamblea Una simple lectura de dicho articulo demuestra que la ley no ha facultado al
Nacional promulgo la Ley No. 103 del Commonwealth que crea el Tribunal Tribunal de Relaciones Industriales a investigar y resolver las cuestiones y
de Relaciones Industriales que es un tribunal especial con facultades conflictos entre obreros y patronos, e inquilinos y propietarios, de una
judiciales (Pambusco Employees Union vs. Court of Industrial Relations et manera arbitraria y caprichosa sin someterse a una norma de conducta
al., G. R. No. 46727; Ang Tibay et al. vs. Court of Industrial Relations et al., determinada. El articulo dispone claramente que las reglas de procedimiento
G. R. No. 46496, opinion concurrente del Magistrado Jose P. Laurel). El que adopte, a las cuales debera ajustarse el tribunal, deberan inspirarse en la
articulo 1 de dicha ley provee que el Tribunal de Relaciones Industriales justicia y la equidad, y prescribe que el criterio que se forma debera fundarse
ejercera jurisdiccion para considerar, investigar, decidir y zanjar toda en los meritos substanciales de la causa sin consideracion a los tecnicismos
cuestion, asunto, conflicto o disputa que afecte o surja entre patronos y o formulismos legales. La Ley No. 103 que crea un tribunal especial
empleados u obreros, y entre propietarios e inquilinos o aparceros, y para denominado Tribunal de Relaciones Industriales con facultad para dictar sus
regular las relaciones entre los mismos, con arreglo y sujecion a las propios reglamentos y para resolver y decidir los conflictos agrarios e
disposiciones de la ley. El articulo 4 dispone que el tribunal tomara industriales de acuerdo con los dictados de la justicia y equidad, no puede ser
conocimiento, para fines de prevencion, arbitraje, decision y ajuste, de impugnada bajo el fundamento de que autoriza la privacion de la libertad y
cualquier conflicto agrario o industrial que motive o de lugar a una huelga o propiedad sin el debido proceso de ley; ni pugna con el precepto del articulo
paro a causa de diferencias que surjan en la cuestion de jornales, participacion 13, Titulo VIII, de la Constitucion porque el Tribunal de Relaciones
o compensacion, horas de trabajo o condiciones de aparceria o empleo, entre Industriales no es de la misma categoria que los juzgados municipales,
patronos y empleados u obreros, y entre propietarios e inquilinos o aparceros, juzgados de paz y juzgados de primera instancia para los cuales se han
siempre que el numero de empleados, obreros, inquilinos o aparceros dictado los reglamentos de los tribunales por el Tribunal Supremo.
afectados exceda de treinta, y que el conflicto agrario o industrial se someta
al tribunal por el Secretario del Trabajo, o por una o ambas partes interesadas, En relacion con la validez y constitucionalidad de la Ley No. 103 Y SUS
cuando el referido Secretario del Trabajo certifique en cuanto a su existencia enmiendas, insertamos a continuacion la opinion concurrente del Magistrado
y la conveniencia de la intervencion del tribunal en bien del interes publico. Laurel en el asunto de Ang Tibay, supra, cuyas observaciones serviran para
Y el articulo 20 preceptua que en la vista, investigacion y resolucion de rebustecer la proposicion sentada de que la referida ley y sus enmiendas es
cualquier cuestion o conflicto, y en el ejercicio de cualquiera de sus deberes valida y no infringe la Constitucion.
y facultades, el tribunal actuara de acuerdo con la justicia y la equidad y los
meritos substanciales de la causa, sin consideracion a los tecnicismos o "It should be observed at the outset that our Constitution was adopted in the
formulismos legales, y no estara sujeto a cualesquier reglas tecnicas de midst of surging unrest and dissatisfaction resulting from economic and
prueba legal, sino que formara juicio de la manera que crea justo y equitativo. social distress which was threatening the stability of governments the world
La Ley No. 103 confiere al Tribunal de Relaciones Industriales plena facultad over. Alive to the social and economic forces at work, the framers of our
discrecional para resolver y decidir las disputas agrarias e industriales de la Constitution boldly met the problems and difficulties which faced them and
manera que crea justo y equitativo, prescindiendo de los tecnicismos y endeavored to crystalize, with more or less fidelity, the political, social and
formulismos legales, y la facultad asi concedida es judicial y no legislativa, economic propositions of their age, and this they did, with the consciousness
por lo que no infringe el principio de separacion de poderes, la prohibicion that the political and philosophical aphorism of their generation will, in the
sobre delegacion de facultades legislativas ni la proteccion iqualitaria ante la language of a great jurist, 'be doubted by the next and perhaps entirely
ley. Como se ha dicho en el asunto de Cincinnati, W. & Z. R. Co. vs. discarded by the third.' (Chief Justice Winslow in Gorgnis v. Falk Co., 147
Comm'rs, of Clinton County '1852), 1 Ohio St., 88, citado en el asunto de WiS., 327; 133 N. W., 209.) Embodying the spirit of the present epoch,
Rubi et al contra La Junta Provincial de Mindoro, 39 Jur. Fil., 675, "Existe general provisions were inserted in the Constitution which are intended to
una verdadera diferencia entre delegar la facultad para dictar leyes, lo cual bring about the needed social and economic equilibrium between component
supone necesariamente discrecion en cuanto a lo que hayan de ser aquellas, elements of society through the application of what may be termed as the
y conferir tribucion o discrecion para hacerlas cumplir, discrecion que debe justitia communis advocated by Grotius and Leibnits many years ago to be
ejecitarse con arreglo a la ley. La primera no puede hacerse en modo alguno; secured through the counterbalancing of economic and social forces and
contra la segunda no cabe interponer objecion alguna." opportunities which should be regulated, if not controlled, by the State or
placed, as it were, in custodia societatis. 'The promotion of social justice to
Para reforzar los argumentos en favor de la anticonstitucionalidad de la Ley in sure the well-being and economic security of all the people' was thus
No. 103 la recurrente hace hincapie en lo resuelto en el asunto de Schechter inserted as vital principle in our Constitution. (Sec. 5, Art. II, Constitution.)
vs. United States (1935), 295 U. S., 496, 79 Law. ed. 270, en que el Tribunal And in order that this declaration of principle may not just be an empty
Supremo de los Estados Unidos declaro anticonstitucional la National medley of words, the Constitution in various sections thereof has provided
Recovery Act. Existe, sin embargo, una marcada diferencia entre dicho the means towards its realization. For instance, section 6 of Article XIII
asunto y el que se considera porque la National Recovery Act en vez de crear declares that the State 'shall afford protection to labor, especially to working
un tribunal de justicia, creo juntas con facultades legislativas y autorizo al women and minors, and shall regulate the relations between landowner and
Presidente de los Estados Unidos a promulgar codigos que prescriban las tenant, and between labor and capital in industry and in agriculture.' The
reglas de precedimiento con el fin de realizar los propositos de la ley. same section also states that 'the State may provide for compulsory
arbitration.' In extraordinary cases mentioned in section 16, Article VI, of the
El ultimo fundamento que se alega en contra de la validez de la Ley No. 103 Constitution, the President of the Philippines may be authorized by law, for
se hace consistir en que las facultades judiciales que concede al Tribunal de a limited period and subject to such restrictions as the National Assembly
Relaciones Industria!es son tan arbitrarias e irrazonables que permiten la may prescribe, to 'promulgate rules and regulations to carry out a declared
privacion de la libertad y la propiedad sin el debido proceso de ley; y que su national policy.' Albeit, almost at the same time the Congress of the United
articulo 20, por lo menos, adolece de este defecto fundamental porque States approved the National Labor Regulations Act (49 Stat., 449) on July
confiere al Tribunal de Relaciones Industriales la facultad de dictar sus 5, 1935, commonly known as the Wagner Act, we were in the Philippines
headway towards the adoption of our fundamental law, pursuant to has had a similar development, although of course to a much smaller degree
congressional authority given in the Tydings-McDuffie Independence Act, and of different adaptation giving rise to several attempts at meeting and
approved March 24, 1934. In our Bill of Rights we now find the following solving our peculiar social and economic problems. (See Commonwealth
provision 'The right to form associations or societies for purposes not Acts Nos. 37, 104, 139, 211; Presidential Message to the National Assembly,
contrary to law shall not be abridged.' (Par. 6, section 1, art. III, Constitution.) September 2, 1936; Executive Order No. 49, S. 1936). The system of
What was an agitation in the United States which brought about the voluntary arbitration devised by Act No. 4055 of the defunct Philippine
recommendation by the Commission on Industrial Relations created by an Legislature has apparently been abandoned by the enactment of the
Act of Congress in 1912 for the adoption of a Labor Bill of Rights as an aforementioned Commonwealth Acts Nos. 103 and 213. In the midst of
amendment to the United States Constitution is, in our case, virtually an changes that have taken place, it may likewise be doubted if the
accepted principle, which may be expanded and vitalized by legislation to pronouncement made by this court in the case of People vs. Pomar (46 Phil.,
keep pace with the development of time and circumstances. 440) also relied upon by the petitioner in its printed memorandum still retains
its virtuality as a living principle. The policy of laissez faire has to some
"By and large, these provisions in our Constitution all evince and express the extent given way to the assumption by the government of the right of
need of shifting emphasis to community interest with a view to affirmative intervention even in contractual relations affected with public interests."
enhancement of human n conformity with the constitutional objective and
cognizant of the historical fact that industrial and agricultural disputes had xxx xxx xxx
given rise to disquietude, bloodshed and revolution in our country, the
National Assembly enacted Commonwealth Act No. 103, entitled 'An Act to "In Commonwealth Act No. 103, and by it, our Government no longer
afford protection of labor by creating a Court of Industrial Relations performs the role of a mere mediator or intervenor but that of the supreme
empowered to fix minimum wages for laborers and maximum rental to be arbiter."
paid by tenants, and to enforce compulsory arbitration between employers or
landlords, and employees or tenants, respectively; and by prescribing En su siguiente senalamiento de error la recurrente alega que la conducta del
penalties for the violation of the orders' and, later, Commonwealth Act No. investigador, la investigacion que practico y la manera como conocio del
213, entitled, 'An Act to define and regulate legitimate labor organizations.' asunto el Tribunal de Relaciones Industriales le privaron de una vista
(As to this last act, vide 'findings and policy,' preamble [sec. 1] of the Wagner imparcial y justa, y constituyen privacion de su propiedad sin el debido
Act [49 Sta., 449]). proceso de ley. Para demostrar la carencia de fundamento del senalamiento
de error, creemos suficiente reproducir a continuacion la forma como se
"Commonwealth Act No. 103, .approved October 29, 1936, was originally practico la investigacion por el comisionado nombrado por el Tribunal de
Bill No. 700 of the National Assembly. More light is shed by the explanatory Relaciones Industriales y la manera como se celebro la vista por dicho
statement of the Bill than by what transpired in the course of the deliberation tribunal, tal como se expone en la orden del 6 de mayo de 1939.
of the measure in the legislative chamber. 'El presente proyecto de ley,' thus
begins the explanatory statement of Bill No. 700, 'crea una Junta de "Hearing was held on April 3, 1939, where witnesses for both the petitioners
Relaciones Industriales . . . y provee el arbitraje obligatorio . . . de acuerdo and the respondent testified. To supplement the facts brought out at the
con el Articulo 6, Titulo XIII de la Constitucion, el cual provee que 'El Estado hearing, the Court ordered one of its Special Agents to proceed to the
podra establecer el arbitraje obligatorio." "Incorporating the conclusion premises of the mines to conduct a further investigation."
reached by a committee appointed, a year or so before it was observed that
'bajo la legislacion actual'" evidently referring to Act No. 4055 'no existe El comisionado fue nombrado por el Tribunal de Relaciones Industriales en
instrumento adecuado para evitar las huelgas. El Departamento de Trabajo el ejercicio de su facultad conferida por el articulo 10 de la Ley No. 103 del
desempena meramente el papel de pacificador entre las partes en controversia Commonwealth y en la inspeccion y vistas que celebraron el comisionado y
y sus decisiones no son obligatorias ni para los patronos ni para los obreros. el tribunal, respectivamente, las partes estuvieron representadas
El pueblo ha llegado a un grado de desarrollo industrial, que hace imperiosa debidamente, fueron oidas y presentaron las pruebas que tenian disponibles
el que la intervencion del gobierno en estos conflictos sea mas efectiva . . .' y creyeron conveniente ofrecer. Tales inspeccion y vistas tenian el caracter
The creation of a Court of Industrial Relations was thus proposed, endowed de una vista judicial imparcial y justa y constituyen el debido proceso de ley
"no solamente del poder de arbitrar sino tambien del deber de investigar, que garantiza la Constitucion.
decidir, y hacer recomendaciones sobre las cuestiones en conflicto y los
problemas que afectan al Capital y al Trabajo en la Industria y la Agricultura Sostiene igualmente la recurrente que la orden del 6 de mayo de 1939 es
bajo la direccion del Presidente de la Mancomunidad de Filipinas o a peticion arbitraria porque no existen pruebas substanciales ni competentes que la
del Secretario del Trabajo.' sostengan. Sobre este extremo, las conclusiones de hecho que ha sentado el
Tribunal de Relaciones Industriales demuestran que la orden impugnada esta
"From what has been stated, it appears that the legislation which are now sostenida por el resultado de la investigacion practicada por el comisionado
called upon to construe was enacted in pursuance of what appears to be the y las pruebas que las partes presentaron directamente ante el Tribunal. En
deliberate embodiment of a new social policy, founded on the conception of dichas conclusiones se han considerado y analizado por el Tribunal de
a society integrated not by independent individuals dealing at arms' length, Relaciones Industriales todas las pruebas que las partes presentaron y resulta
but by interdependent members of a consolidated whole whose interests must inevitable la conclusion de que la orden no es arbitraria y esta justificada y
be protected against mutual aggression and warfare among and between sostenida por los hechos probados.
divers and diverse units which are impelled by countervailing and opposite
individual and group interests, and this is particularly true in the relationship El ultimo seiialamiento de error guarda relacion con la parte de la orden del
between labor and capital. Social and industrial disturbances which fifty 6 de mayo de 1939 que dispone que la recurrente pague a los 55 obreros
years ago were feudal-like and of isolated importance may now well result repuestos los jornales que dejaron de percibir durante su separacion del
in a serious strain upon the entire economic organism of the nation. In the servicio. La recurrente sostiene que esta parte de la orden equivale a una
United States labor legislation has undergone a long process of development sentencia por danos y perjuicios que el Tribunal de Relaciones Industriales
too long to narrate here, culminating in the enactments of what were no puede pronunciar por carecer de jurisdiccion. La pretension no es
commonly known as the Clayton Act, the Norris-La Guardia Act, and finally, meritoria. El Tribunal de Relaciones Industriales, conforme ya se ha dicho,
the Wagner Act and the Fair Labor Standards Act of 1938. The Wagner Act es un tribunal especial y como tal tiene facultad para disponer que la
created the National Labor Relations Board as an instrumentality of the recurrente pague los jornales de sus empleados y obreros que han sido
Federal Government in the settlement of labor disputes, which device is repuestos. Los articulos 1 y 4 de la Ley No. 103 del Commonwealth, segun
aimed at the avoidance of unnecessary friction between labor and capital and ha sido enmendado el primero por el articulo 1 de la Ley No. 254, confieren
the establishment of industrial peace. Scrutiny of legislation in that country facultad y jurisdiccion al Tribunal de Relaciones Industriales para conocer,
and of pronouncements made by its Supreme Court reveals a continuous resolver y decidir todas las cuestiones, controversias y disputas entre
renovation and change made necessary by the impact of changing needs and patronos y obreros y propietarios y terratenientes, y los jornales de los
economic pressure brought about by the irrisistible momentum of new social obreros repuestos, durante el tiempo en que fueron separados del servicio,
and economic forces developed there. In the light of changes that have estaban incluidos en las controversias y disputas sometidas al Departamento
occurred, it is doubted if the pronouncements made by the said Supreme del Trabajo y certificadas por este al Tribunal de Relaciones Industriales.
Court in 1905 (Lochner v. New York, 198, U. S., 45) or in 1908 (Adair v. U.
S., 52 Law. ed. 430, 208 U. S., 161, and Coppage v. Kansas, 236 U. S., 1) Se deniega el recurso de certiorari y se confirman la orden del 6 de mayo de
cases which are relied upon by the petitioner in its printed memorandum still 1939 y la resolucion del 17 de agosto del mismo ano, con las costas a la
retain their virtuality at the present time. In the Philippines, social legislation recurrente. Asi se ordena.
UNIVERSITY OF IMMACULATE, CONCEPCION, INC. vs THE 5. Melanie de la Rosa – Secretary to [the] Dean of [the] College of
HONORABLE SECRETARY OF LABOR Pharmacy/ Academic Affairs/ Dean of College
G.R. No. 151379 | 2005-01-14 6. Zenaida Canoy – Secretary to [the] Vice President for Academic
AZCUNA, J.: Affairs/ Dean of College
7. Alma Villacarlos – Guidance Counselor (College)
This is a petition for review of a decision of the Court of Appeals and the 8. Josie Boston – Grade School Psychometrician
resolution denying reconsideration thereof. The principal issue to be resolved 9. Paulina Palma Gil – Cashier
in this recourse is whether or not the Secretary of Labor, after assuming 10. Gemma Galope – High School Registrar
jurisdiction over a labor dispute involving an employer and the certified 11. Leah Cruza – Guidance Counselor (College)
bargaining agent of a group of employees in the workplace, may legally order 12. Delfa Diapuez – High School Psychometrician [4]
said employer to reinstate employees terminated by the employer even if
those terminated employees are not part of the bargaining unit. Thereafter, the UNIVERSITY gave the abovementioned individual
respondents two choices: to resign from the UNION and remain employed
This case stemmed from the collective bargaining negotiations between as confidential employees or resign from their confidential positions and
petitioner University of Immaculate Concepcion, Inc. (UNIVERSITY) and remain members of the UNION. The UNIVERSITY relayed to these
respondent The UIC Teaching and Non-Teaching Personnel and Employees employees that they could not remain as confidential employees and at the
Union (UNION). The UNION, as the certified bargaining agent of all rank same time as members or officers of the Union. However, the individual
and file employees of the UNIVERSITY, submitted its collective bargaining respondents remained steadfast in their claim that they could still retain their
proposals to the latter on February 16, 1994. However, one item was left confidential positions while being members or officers of the Union. Hence,
unresolved and this was the inclusion or exclusion of the following positions on February 21, 1995, the UNIVERSITY sent notices of termination to the
in the scope of the bargaining unit: individual respondents.

a. Secretaries On March 10, 1995, the UNION filed another notice of strike, this time citing
b. Registrars as a reason the UNIVERSITY’s termination of the individual respondents.
c. Accounting Personnel The UNION alleged that the UNIVERSITY’s act of terminating the
d. Guidance Counselors [1] individual respondents is in violation of the Order of the Secretary of Labor
dated January 23, 1995.
This matter was submitted for voluntary arbitration. On November 8, 1994,
the panel of voluntary arbitrators rendered a decision, the dispositive portion On March 28, 1995, the Secretary of Labor issued another Order reiterating
of which states: the directives contained in the January 23, 1995 Order. The Secretary also
stated therein that the effects of the termination from employment of these
WHEREFORE, premises considered, the Panel hereby resolves to exclude individual respondents be suspended pending the determination of the
the above-mentioned secretaries, registrars, chief of the accounting legality thereof. Hence, the UNIVERSITY was directed to reinstate the
department, cashiers and guidance counselors from the coverage of the individual respondents under the same terms and conditions prevailing prior
bargaining unit. The accounting clerks and the accounting staff member are to the labor dispute.
hereby ordered included in the bargaining unit.[2]
The UNIVERSITY, thereafter, moved to reconsider the aforesaid Order on
The UNION moved for the reconsideration of the above decision. Pending, March 28, 1995. It argued that the Secretary’s Order directing the
however, the resolution of its motion, on December 9, 1994, it filed a notice reinstatement of the individual respondents would render nugatory the
of strike with the National Conciliation and Mediation Board (NCMB) of decision of the panel of voluntary arbitrators to exclude them from the
Davao City, on the grounds of bargaining deadlock and unfair labor practice. collective bargaining unit. The UNIVERSITY’s motion was denied by the
During the thirty (30) day cooling-off period, two union members were Secretary in an Order dated June 16, 1995, wherein the Secretary declared
dismissed by petitioner. Consequently, the UNION went on strike on that the decision of the panel of voluntary arbitrators to exclude the individual
January 20, 1995. respondents from the collective bargaining unit did not authorize the
UNIVERSITY to terminate their employment. The UNIVERSITY filed a
On January 23, 1995, the then Secretary of Labor, Ma. Nieves R. Confessor, second motion for reconsideration, which was again denied in an Order dated
issued an Order assuming jurisdiction over the labor dispute. The dispositive July 19, 1995. Undeterred, the UNIVERSITY filed a third motion for
portion of the said Order states: reconsideration. In the Order dated August 18, 1995, then Acting Secretary
Jose S. Brilliantes denied the third motion for reconsideration, but modified
WHEREFORE, ABOVE PREMISES CONSIDERED, and pursuant to the two previous Orders by adding:
Article 263 (g) of the Labor Code, as amended, this Office hereby assumes
jurisdiction over the entire labor dispute at the University of the Immaculate xxx
Concepcion College.
Anent the Union’s Motion, we find that superseding circumstances would
Accordingly, all workers are directed to return to work within twenty-four not warrant the physical reinstatement of the twelve (12) terminated
(24) hours upon receipt of this Order and for Management to accept them employees. Hence, they are hereby ordered placed under payroll
back under the same terms and conditions prevailing prior to the strike. reinstatement until the validity of their termination is finally resolved.[5]

Parties are further directed to cease and desist from committing any or all x x x
acts that might exacerbate the situation.
Still unsatisfied with the Order of the Secretary of Labor, the UNIVERSITY
Finally, the parties are hereby directed to submit their respective position filed a petition for certiorari with this Court on September 15, 1995.
papers within ten (10) days from receipt hereof. However, its petition was referred to the Court of Appeals, following the
ruling in St. Martin Funeral Homes v. Court of Appeals. [6]
SO ORDERED.[3]
On October 8, 2001, the Court of Appeals promulgated its Decision,
On February 8, 1995, the panel of voluntary arbitrators denied the motion for affirming the questioned Orders of the Secretary of Labor. The dispositive
reconsideration filed by the UNION. The UNIVERSITY then furnished portion of the Decision states:
copies of the panel’s denial of the motion for reconsideration and the
Decision dated November 8, 1995 to the individual respondents herein: WHEREFORE, the instant petition is DISMISSED for lack of merit.[7]

1. Lelian Concon – Grade School Guidance Counselor The UNIVERSITY then moved for the reconsideration of the
2. Mary Ann de Ramos – High School Guidance Counselor abovementioned Decision,[8] but on January 10, 2002, the Court of Appeals
3. Jovita Mamburam – Secretary to [the] Vice President for Academic denied the motion on the ground that no new matters were raised therein that
Affairs/ Dean of College would warrant a reconsideration.[9]
4. Angelina Abadilla – Secretary to [the] Vice President for Academic
Affairs/ Dean of College Hence, this petition.
Indeed, it is clear that the act of the UNIVERSITY of dismissing the
The UNIVERSITY assigns the following error: individual respondents from their employment became the impetus for the
UNION to declare a second notice of strike. It is not a question anymore of
THE COURT OF APPEALS COMMITTED REVERSIBLE ERROR IN whether or not the terminated employees, the individual respondents herein,
AFFIRMING THE ORDERS OF THE SECRETARY OF LABOR THAT are part of the bargaining unit. Any act committed during the pendency of
SUSPENDED THE EFFECTS OF THE TERMINATION OF TWELVE the dispute that tends to give rise to further contentious issues or increase the
EMPLOYEES WHO WERE NOT PART OF THE BARGAINING UNIT tensions between the parties should be considered an act of exacerbation and
INVOLVED IN A LABOR DISPUTE OVER WHICH THE SECRETARY should not be allowed.
OF LABOR ASSUMED JURISDICTION.[10]
With respect to the Secretary’s Order allowing payroll reinstatement instead
The Court of Appeals relied upon the doctrine in St. Scholastica’s College v. of actual reinstatement for the individual respondents herein, an amendment
Torres.[11] In the case therein, this Court, citing International to the previous Orders issued by her office, the same is usually not allowed.
Pharmaceuticals Incorporated v. the Secretary of Labor,[12] declared that: Article 263(g) of the Labor Code aforementioned states that all workers must
immediately return to work and all employers must readmit all of them under
x x x [T]he Secretary was explicitly granted by Article 263(g) of the Labor the same terms and conditions prevailing before the strike or lockout. The
Code the authority to assume jurisdiction over a labor dispute causing or phrase “under the same terms and conditions” makes it clear that the norm is
likely to cause a strike or lockout in an industry indispensable to the national actual reinstatement. This is consistent with the idea that any work stoppage
interest, and decide the same accordingly. Necessarily, the authority to or slowdown in that particular industry can be detrimental to the national
assume jurisdiction over the said labor dispute must include and extend to all interest.
questions and controversies arising therefrom, including cases over which
the Labor Arbiter has exclusive jurisdiction. In ordering payroll reinstatement in lieu of actual reinstatement, then Acting
Secretary of Labor Jose S. Brillantes said:
The UNIVERSITY contends that the Secretary cannot take cognizance of an
issue involving employees who are not part of the bargaining unit. It insists Anent the Union’s Motion, we find that superseding circumstances would
that since the individual respondents had already been excluded from the not warrant the physical reinstatement of the twelve (12) terminated
bargaining unit by a final and executory order by the panel of voluntary employees. Hence, they are hereby ordered placed under payroll
arbitrators, then they cannot be covered by the Secretary’s assumption order. reinstatement until the validity of their termination is finally resolved.[18]

This Court finds no merit in the UNIVERSITY’s contention. In Metrolab As an exception to the rule, payroll reinstatement must rest on special
Industries, Inc. v. Roldan-Confessor,[13] this Court declared that it circumstances that render actual reinstatement impracticable or otherwise not
recognizes the exercise of management prerogatives and it often declines to conducive to attaining the purposes of the law.[19]
interfere with the legitimate business decisions of the employer. This is in
keeping with the general principle embodied in Article XIII, Section 3 of the The “superseding circumstances” mentioned by the Acting Secretary of
Constitution,[14] which is further echoed in Article 211 of the Labor Labor no doubt refer to the final decision of the panel of arbitrators as to the
Code.[15] However, as expressed in PAL v. National Labor Relations confidential nature of the positions of the twelve private respondents, thereby
Commission,[16] this privilege is not absolute, but subject to exceptions. rendering their actual and physical reinstatement impracticable and more
One of these exceptions is when the Secretary of Labor assumes jurisdiction likely to exacerbate the situation. The payroll reinstatement in lieu of actual
over labor disputes involving industries indispensable to the national interest reinstatement ordered in these cases, therefore, appears justified as an
under Article 263(g) of the Labor Code. This provision states: exception to the rule until the validity of their termination is finally resolved.
This Court sees no grave abuse of discretion on the part of the Acting
(g) When, in his opinion, there exists a labor dispute causing or likely to Secretary of Labor in ordering the same. Furthermore, the issue has not been
cause a strike or lockout in an industry indispensable to the national interest, raised by any party in this case.
the Secretary of Labor and Employment may assume jurisdiction over the
dispute and decide it or certify the same to the Commission for compulsory WHEREFORE, the Decision of the Court of Appeals dated October 8, 2001
arbitration. Such assumption or certification shall have the effect of and its Resolution dated January 10, 2002 in CA-G.R. SP No. 61693 are
automatically enjoining the intended or impending strike or lockout as AFFIRMED.
specified in the assumption or certification order. If one has already taken
place at the time of assumption or certification, all striking or locked out No costs.
employees shall immediately return to work and the employer shall
immediately resume operations and readmit all workers under the same terms SO ORDERED.
and conditions prevailing before the strike or lockout. x x x

When the Secretary of Labor ordered the UNIVERSITY to suspend the effect
of the termination of the individual respondents, the Secretary did not exceed
her jurisdiction, nor did the Secretary gravely abuse the same. It must be
pointed out that one of the substantive evils which Article 263(g) of the Labor
Code seeks to curb is the exacerbation of a labor dispute to the further
detriment of the national interest. In her Order dated March 28, 1995, the
Secretary of Labor rightly held:

It is well to remind both parties herein that the main reason or rationale for
the exercise of the Secretary of Labor and Employment’s power under
Article 263(g) of the Labor Code, as amended, is the maintenance and
upholding of the status quo while the dispute is being adjudicated. Hence,
the directive to the parties to refrain from performing acts that will exacerbate
the situation is intended to ensure that the dispute does not get out of hand,
thereby negating the direct intervention of this office.

The University’s act of suspending and terminating union members and the
Union’s act of filing another Notice of Strike after this Office has assumed
jurisdiction are certainly in conflict with the status quo ante. By any
standards[,] these acts will not in any way help in the early resolution of the
labor dispute. It is clear that the actions of both parties merely served to
complicate and aggravate the already strained labor-management
relations.[17]
VICTORIA vs. INCIONG The decision of the arbitrator was successively appealed to the ad hoc
G.R. No. L-49046 | 1988-01-26 National Labor Relations Commission, the Secretary of Labor and the Office
FERNAN, J.: of the President of the Philippines, and was affirmed in all instances.

Petition for review of the Order of the then Acting Secretary of Labor Amado On April 23, 1975, the Court of First Instance of Bulacan rendered judgment,
G. Inciong dated June 6, 1978, in NLRC Case No. RB-1764-75, reversing to wit:
the decision of the National Labor Relations Commission dated November
17, 1976 and holding that, under the law and facts of the case, there was no "WHEREFORE, judgment is hereby rendered:
necessity for private respondent to obtain a clearance for the termination of
petitioner's employment under Article 257 [b] of the Labor Code, as 1. Making injunction against defendants permanent;
amended, and that a mere report of such termination was sufficient, under
Section 11 [f]. Rule XIV of the Rules and Regulations implementing said 2. Declaring that this Court has jurisdiction to try and hear the instant case
Code. despite Section 2 of Presidential Decree No. 2;

Petitioner Saturno Victoria was employed on March 17, 1956 by private 3. Declaring that plaintiff Far East Broadcasting Company is a non-profit
respondent Far East Broadcasting Company, Incorporated as a radio organization since it does not declare dividends;
transmitter operator. Sometime in July 1971, he and his co-workers
organized the Far East Broadcasting Company Employees Association. After 4. Declaring that the strike admitted by the defendants to have been declared
registering their association with the then Department of Labor, they by them is illegal inasmuch as it was for the purpose of compelling the
demanded recognition of said association by the company but the latter plaintiff-company to recognize their labor union which could not be legally
refused on the ground that being a non-profit, non-stock, non-commercial done because the plaintiffs were not covered by Republic Act 875;
and religious corporation, it is not covered by Republic Act 875, otherwise
known as the Industrial Peace Act, the labor law enforced at that time. 5. Declaring that the evidence presented is insufficient to show that
defendants caused the damage to the plaintiff consequent on the destruction
Several conciliation meetings were held at the Department of Labor and in of its relays and its antennas as well as its transmission lines.
those meetings, the Director of Labor Relations Edmundo Cabal advised the
union members that the company could not be forced to recognize them or to SO ORDERED." 2
bargain collectively with them because it is a non-profit, non-commercial and
religious organization. Notwithstanding such advice, the union members led On April 24, 1975, by virtue of the above decision, the company notified
by Saturno Victoria as its president, declared a strike and picketed the Saturno Victoria that he is dismissed effective April 26, 1975. Thereupon, he
company's premises on September 6, 1972 for the purpose of seeking filed Case No. RB-IV-1764 before the National Labor Relations
recognition of the labor union. Commission, Regional Branch IV against the company alleging violation of
Article 267 of the Labor Code which requires clearance from the Secretary
As a countermeasure, the company filed a case for damages with preliminary of Labor for every shutdown of business establishments or dismissal of
injunction against the strikers before the then Court of First Instance of employees. On February 27, 1976, Labor Arbiter Manuel B. Lorenzo
Bulacan docketed as Civil Case No. 750-V. Said court issued an injunction rendered a decision in petitioner's favor declaring the dismissal to be illegal,
enjoining the three-day-old strike staged against the company. The complaint thereby ordering reinstatement with full backwages. On appeal, the arbiter's
was later amended seeking to declare the strike illegal. decision was affirmed by the National Labor Relations Commission. But
when the commission's decision was in turn appealed to the Secretary of
Upon the declaration of martial law on September 21, 1972 and the Labor, it was set aside and in lieu thereof the questioned Order dated June 6,
promulgation of Presidential Decree No. 21 creating the National Labor 1978 was issued.
Relations Commission, the ad hoc National Labor Relations Commission
took cognizance of the strike through NLRC Case No. 0021 entitled "Far In view of its brevity and for a better understanding of the reasons behind it,
East Broadcasting Company Employees Association, complainant versus Far We quote the disputed Order in full:
East Broadcasting Company, respondent" and NLRC Case No. 0285 entitled
"Generoso Serino, complainant, versus Far East Broadcasting Company, "O R D E R
respondent", both cases for reinstatement due to the company's refusal to
accept the union's offer to return to work during the pendency of the case in This is an appeal by respondent from the Decision of the National Labor
the Court of First Instance. Relations Commission, dated November 17, 1976.

On December 28, 1972, Arbitrator Flavio Aguas rendered a joint decision in The Commission upheld the Decision of the labor arbiter dated February 27,
the two cases mentioned above recognizing the jurisdiction of the Court of 1976 ordering respondent to reinstate with full backwages herein
First Instance of Bulacan, the dispositive portion reading as follows: complainant Saturno A. Victoria based on the finding that respondent did not
file any application for clearance to terminate the services of complainant
"IN VIEW WHEREOF, and in the interest of justice and equity, it is hereby before dismissing him from his employment.
directed that:
Briefly the facts of this case are as follows:
1. That striking members of the Far East Broadcasting Company Employees
Association return to their respective positions in the corporation; Complainant Saturno Victoria is the president of the Far East Broadcasting
Company Employees Union. On September 8, 1972, the said union declared
2. The respondent Far East Broadcasting Company Incorporated to accept a strike against respondent company. On September 11, 1972, respondent
back the returning strikers without loss in rank seniority or status; filed with the Court of First Instance of Bulacan, Civil Case No. 750-V, for
the issuance of an injunction and a prayer that the strike be declared illegal.
3. The workers shall return to work within ten [10] days from receipt of this
resolution otherwise they shall be deemed to have forfeited such right; On October 24, 1972, complainant together with the other strikers filed with
the ad hoc National Labor Relations Commission Case Nos. 0021 and 0285
4. The respondent shall report compliance with this decision within fifteen for reinstatement. The Arbitrator rendered a decision in said case on
[15] days from receipt hereof. December 28, 1972, wherein he ordered respondent to reinstate complainants
subject to the following condition:
This Order shall, however, be without prejudice to whatever decision the
Court of First Instance of Bulacan may promulgate in Civil Case No. 750-V 'This Order shall, however, be without prejudice to whatever decision the
and to the requirements the existing order may need of people working with Court of First Instance may promulgate on Civil Case No. 750-V and to the
the mass media of communications. requirements the existing order may need of people working with the mass
media of communications.'
IT IS SO ORDERED." 1
Since said decision was affirmed by the NLRC, the Secretary of Labor, and requiring a prior clearance from the Secretary of Labor in cases of shutdown
the Office of the President of the Philippines, complainants were reinstated or dismissal of employees, is to afford the Secretary ample opportunity to
pursuant thereto. examine and determine the reasonableness of the request.

In a Decision dated April 23, 1975, in Civil Case No. 750-V, promulgated by The Solicitor General, in relation to said pronouncement and in justification
the Court of First Instance of Bulacan, the strike staged by herein of the Acting Labor Secretary's decision makes the following observations:
complainant and the other strikers was declared illegal. Based on said
Decision, respondent dismissed complainant from his employment. Hence, "It is true that Article 267 [b] of the Labor Code requires that before any
complainant filed the instant complaint for illegal dismissal. business establishment is shutdown or any employee is dismissed, written
clearance from the Secretary of Labor must first be obtained. It is likewise
Under the aforecited facts, we do not agree with the ruling of the Commission true that in the case of petitioner, there was no written clearance in the usual
now subject of this appeal that an application for clearance to terminate form. But while there may not have been strict compliance with Article 267
herein complainant is mandatory on the part of respondent before terminating there was substantial compliance. The Secretary of Labor twice manifested
complainant's services. We believe that what would have been necessary was his conformity to petitioner's dismissal.
a report as provided for under Section 11 [f], Rule XIV, Book V of the Rules
and Regulations Implementing the Labor Code. Moreover, even if an "The first manifestation of acquiescence by the Secretary of Labor to the
application for clearance was filed, this Office would have treated the same dismissal of petitioner was his affirmance of the decision of the arbitrator in
as a report. Otherwise, it would render nugatory the Decision of the NLRC Case Nos. 0021 and 0285. The arbitrator ordered the reinstatement of
Arbitrator dated December 28, 1972 in Case Nos. 0021 and 0285 which was the strikers but subject to the decision of the CFI of Bulacan in Civil Case
affirmed by the Commission, the Secretary of Labor and the Office of the No. 750-V. The Secretary of Labor affirmed the decision of the arbitrator. In
President of the Philippines, ordering his temporary reinstatement, subject to effect, therefore, the Secretary of Labor issued a carte blanche to the CFI of
whatever Decision the CFI of Bulacan may promulgate in Civil Case No. Bulacan to either dismiss or retain petitioner.
750-V. It could be clearly inferred from said CFI Decisions that if the strike
is declared illegal, the strikers will be considered to have lost their "The second manifestation was his decision in NLRC Case No. RB-IV-1764-
employment status under the then existing laws and jurisprudence, otherwise 65 wherein he said that clearance for the dismissal of petitioner was not
strikers could stage illegal strike with impunity. Since the strike was declared required, but only a report; that even if an application for clearance was filed,
illegal, respondent acted in good faith when it dispensed with the services of he would have treated it as a mere report. While this is not prior clearance in
herein complainant. the contemplation of Article 267, it is at least a ratification of the dismissal
of petitioner." 6
For failure of respondent to file the necessary report and based on equitable
considerations, complainant should be granted separation pay equivalent to We agree with the Solicitor General. Technically speaking, no clearance was
one-half month salary for every year of service. obtained by private respondent from the then Secretary of Labor, the last step
towards full compliance with the requirements of law on the matter of
WHEREFORE, let the decision of the National Labor Relations Commission dismissal of employees. However, the rationale behind the clearance
dated November 17, 1976 be, as it is hereby, set aside and a new judgment is requirement was fully met. The Secretary of Labor was apprised of private
entered, ordering respondent to give complainant separation pay equivalent respondent's intention to terminate the services of petitioner. This in effect is
to one-half month salary for every year of service. an application for clearance to dismiss petitioner from employment. The
affirmance of the restrictive condition in the dispositive portion of the labor
SO ORDERED." 3 arbiter's decision in NLRC Case Nos. 0021 and 0285 by the Secretary of
Labor and the Office of the President of the Philippines, signifies a grant of
Petitioner elevates to Us for review on Certiorari the aforequoted Order authority to dismiss petitioner in case the strike is declared illegal by the
seeking to persuade this Court that then Acting Secretary of Labor Amado Court of First Instance of Bulacan. Consequently and as correctly stated by
G. Inciong committed reversible error in holding that, under the law and facts the Solicitor General, private respondent acted in good faith when it
of this case, a mere report of the termination of the services of said petitioner terminated the employment of petitioner upon a declaration of illegality of
was sufficient. Petitioner assigns the following errors: the strike by the Court of First Instance of Bulacan. Moreover, the then
Secretary of Labor manifested his conformity to the dismissal, not once, but
I twice. In this regard, the mandatory rule on clearance need not be applied.
WHETHER OR NOT A CLEARANCE FROM THE SECRETARY OF
LABOR IS STILL NECESSARY BEFORE THE PETITIONER HEREIN The strike staged by the union in 1972 was a futile move. The law then
COULD BE DISMISSED CONSIDERING THE RESTRICTIVE enforced, Republic Act 875 specifically excluded respondent company from
CONDITION IN THE DECISION OF THE COMPULSORY its coverage. Even if the parties had gone to court to compel recognition, no
ARBITRATOR IN NLRC CASE NOS. 0021 AND 0285. positive relief could have been obtained since the same was not sanctioned
by law. Because of this, there was no necessity on the part of private
II respondent to show specific acts of petitioner during the strike to justify his
WHETHER OR NOT THE DECISION OF THE COURT OF FIRST dismissal.
INSTANCE OF BULACAN IN CIVIL CASE NO. 750-V IPSO FACTO
GAVE THE RESPONDENT COMPANY AUTHORITY TO DISMISS This is a matter of responsibility and of answerability. Petitioner as a union
HEREIN PETITIONER WITHOUT ANY CLEARANCE FROM THE leader, must see to it that the policies and activities of the union in the conduct
SECRETARY OF LABOR. 4 of labor relations are within the precepts of law and any deviation from the
legal boundaries shall be imputable to the leader. He bears the responsibility
The substantive law on the matter enforced during the time of petitioner's of guiding the union along the path of law and to cause the union to demand
dismissal was Article 267 [b] of the Labor Code [in conjunction with the what is not legally demandable, would foment anarchy which is a prelude to
rules and regulations implementing said substantive law.] Article 267 reads: chaos.

"No employer that has no collective bargaining agreement may shut down Petitioner should have known and it was his duty to impart this imputed
his establishment or dismiss or terminate the service of regular employees knowledge to the members of the union that employees and laborers in non-
with at least one [1] year of service except managerial employees as defined profit organizations are not covered by the provisions of the Industrial Peace
in this book without previous written clearance from the Secretary of Labor." Act and the Court of Industrial Relations [in the case at bar, the Court of First
Instance] has no jurisdiction to entertain petitions of labor unions or
Petitioner maintains that the abovecited provision is very clear. It does not organizations of said non-profit organizations for certification as the
make any distinction as to the ground for dismissal. Whether or not the exclusive bargaining representatives of said employees and laborers. 7
dismissal sought by the employer company is for cause, it is imperative that
the company must apply for a clearance from the Secretary of Labor." As a strike is an economic weapon at war with the policy of the Constitution
and the law at that time, a resort thereto by laborers shall be deemed to be a
In a recent case 5 penned by Justice Abraham F. Sarmiento promulgated on choice of remedy peculiarly their own, and outside of the statute, and as such,
June 30, 1987, we had occasion to rule in agreement with the findings of then the strikers must accept all the risks attendant upon their choice. If they
Presidential Assistant for Legal Affairs Ronaldo Zamora that the purpose in succeed and the employer succumbs, the law will not stand in their way in
the enjoyment of the lawful fruits of their victory. But if they fail, they cannot
thereafter invoke the protection of the law for the consequences of their
conduct unless the right they wished vindicated is one which the law will, by
all means, protect and enforce. 8

We further agree with the Acting Secretary of Labor that what was required
in the case of petitioner's dismissal was only a report as provided under
Section ll [f] of Rule XIV of the Rules and Regulations implementing the
Labor Code which provides:

"Every employer shall submit a report to the Regional Office in accordance


with the form presented by the Department on the following instances of
termination of employment, suspension, lay-off or shutdown which may be
effected by the employer without prior clearance within five [5] days
thereafter:

xxx xxx xxx

[f] All other terminations of employment, suspension, lay-offs or shutdowns,


not otherwise specified in this and in the immediately preceding sections."

To hold otherwise would render nugatory the conditions set forth in the
decision of Labor Arbiter Aguas on the basis of which petitioner was
temporarily reinstated.

Inasmuch as there was a valid and reasonable ground to dismiss petitioner


but no report as required by the implementing rules and regulations of the
Labor Code was filed by respondent Company with the then Department of
Labor, petitioner as held by the Acting Secretary of Labor, is entitled to
separation pay equivalent to one-half month salary for every year of service.

WHEREFORE, the petition is dismissed. The decision of the acting


Secretary of Labor is AFFIRMED in toto.

SO ORDERED.
ESTATE OF NELSON R. DULAY vs. ABOITIZ JEBSEN MARITIME, Herein respondents then filed a special civil action for certiorari with the CA
INC. contending that the NLRC committed grave abuse of discretion in affirming
G.R. No. 172642 June 13, 2012 the jurisdiction of the NLRC over the case; in ruling that a different provision
PERALTA, J.: of the CBA covers the death claim; in reversing the findings of the Labor
Arbiter that the cause of death is not work-related; and, in setting aside the
Before the Court is a petition for review on certiorari under Rule 45 of the release and quitclaim executed by the attorney-in-fact and not considering
Rules of Court seeking to reverse and set aside the Decision1 and the P20,000.00 already received by Merridy Jane through her attorney-in-
Resolution2 dated July 11, 2005 and April 18, 2006 of the Court of Appeals fact.
(CA) in CA-G.R. SP No. 76489.
On July 11, 2005, the CA promulgated its assailed Decision, the dispositive
The factual and procedural antecedents of the case, as summarized by the portion of which reads as follows:
CA, are as follows:
WHEREFORE, in view of the foregoing, the petition is hereby GRANTED
Nelson R. Dulay (Nelson, for brevity) was employed by [herein respondent] and the case is REFERRED to the National Conciliation and Mediation
General Charterers Inc. (GCI), a subsidiary of co-petitioner [herein co- Board for the designation of the Voluntary Arbitrator or the constitution of a
respondent] Aboitiz Jebsen Maritime Inc. since 1986. He initially worked as panel of Voluntary Arbitrators for the appropriate resolution of the issue on
an ordinary seaman and later as bosun on a contractual basis. From the matter of the applicable CBA provision.
September 3, 1999 up to July 19, 2000, Nelson was detailed in petitioners’
vessel, the MV Kickapoo Belle. SO ORDERED.4

On August 13, 2000, or 25 days after the completion of his employment The CA ruled that while the suit filed by Merridy Jane is a money claim, the
contract, Nelson died due to acute renal failure secondary to septicemia. At same basically involves the interpretation and application of the provisions
the time of his death, Nelson was a bona fide member of the Associated in the subject CBA. As such, jurisdiction belongs to the voluntary arbitrator
Marine Officers and Seaman’s Union of the Philippines (AMOSUP), GCI’s and not the labor arbiter.
collective bargaining agent. Nelson’s widow, Merridy Jane, thereafter
claimed for death benefits through the grievance procedure of the Collective Petitioner filed a Motion for Reconsideration but the CA denied it in its
Bargaining Agreement (CBA) between AMOSUP and GCI. However, on Resolution of April 18, 2006.
January 29, 2001, the grievance procedure was "declared deadlocked" as
petitioners refused to grant the benefits sought by the widow. Hence, the instant petition raising the sole issue of whether or not the CA
committed error in ruling that the Labor Arbiter has no jurisdiction over the
On March 5, 2001, Merridy Jane filed a complaint with the NLRC Sub- case.
Regional Arbitration Board in General Santos City against GCI for death and
medical benefits and damages. Petitioner contends that Section 10 of Republic Act (R.A.) 8042, otherwise
known as the Migrant Workers and Overseas Filipinos Act of 1995, vests
On March 8, 2001, Joven Mar, Nelson’s brother, received ₱20,000.00 from jurisdiction on the appropriate branches of the NLRC to entertain disputes
[respondents] pursuant to article 20(A)2 of the CBA and signed a regarding the interpretation of a collective bargaining agreement involving
"Certification" acknowledging receipt of the amount and releasing AMOSUP migrant or overseas Filipino workers. Petitioner argues that the
from further liability. Merridy Jane contended that she is entitled to the abovementioned Section amended Article 217 (c) of the Labor Code which,
aggregate sum of Ninety Thousand Dollars ($90,000.00) pursuant to in turn, confers jurisdiction upon voluntary arbitrators over interpretation or
[A]rticle 20 (A)1 of the CBA x x x implementation of collective bargaining agreements and interpretation or
enforcement of company personnel policies.
xxxx
The pertinent provisions of Section 10 of R.A. 8042 provide as follows:
Merridy Jane averred that the P20,000.00 already received by Joven Mar
should be considered advance payment of the total claim of US$90,000.[00]. SEC. 10. Money Claims. - Notwithstanding any provision of law to the
contrary, the Labor Arbiters of the National Labor Relations Commission
[Herein respondents], on the other hand, asserted that the NLRC had no (NLRC) shall have the original and exclusive jurisdiction to hear and decide,
jurisdiction over the action on account of the absence of employer-employee within ninety (90) calendar days after filing of the complaint, the claims
relationship between GCI and Nelson at the time of the latter’s death. Nelson arising out of an employer-employee relationship or by virtue of any law or
also had no claims against petitioners for sick leave allowance/medical contract involving Filipino workers for overseas deployment including
benefit by reason of the completion of his contract with GCI. They further claims for actual, moral, exemplary and other forms of damages.
alleged that private respondent is not entitled to death benefits because
petitioners are only liable for such "in case of death of the seafarer during the Article 217(c) of the Labor Code, on the other hand, states that:
term of his contract pursuant to the POEA contract" and the cause of his death
is not work-related. Petitioners admitted liability only with respect to article xxxx
20(A)2 [of the CBA]. x x x
(c) Cases arising from the interpretation or implementation of collective
xxxx bargaining agreements and those arising from the interpretation or
enforcement of company personnel policies shall be disposed by the Labor
However, as petitioners stressed, the same was already discharged. Arbiter by referring the same to the grievance machinery and voluntary
arbitration as may be provided in said agreements.
The Labor Arbiter ruled in favor of private respondent. It took cognizance of
the case by virtue of Article 217 (a), paragraph 6 of the Labor Code and the On their part, respondents insist that in the present case, Article 217,
existence of a reasonable causal connection between the employer-employee paragraph (c) as well as Article 261 of the Labor Code remain to be the
relationship and the claim asserted. It ordered the petitioner to pay governing provisions of law with respect to unresolved grievances arising
₱4,621,300.00, the equivalent of US$90,000.00 less ₱20,000.00, at the time from the interpretation and implementation of collective bargaining
of judgment x x x agreements. Under these provisions of law, jurisdiction remains with
voluntary arbitrators.
xxxx
Article 261 of the Labor Code reads, thus:
The Labor Arbiter also ruled that the proximate cause of Nelson’s death was
not work-related. ARTICLE 261. Jurisdiction of Voluntary Arbitrators or panel of Voluntary
Arbitrators. – The Voluntary Arbitrator or panel of Voluntary Arbitrators
On appeal, [the NLRC] affirmed the Labor Arbiter’s decision as to the grant shall have original and exclusive jurisdiction to hear and decide all
of death benefits under the CBA but reversed the latter’s ruling as to the unresolved grievances arising from the interpretation or implementation of
proximate cause of Nelson’s death.3 the Collective Bargaining Agreement and those arising from the
interpretation or enforcement of company personnel policies referred to in
the immediately preceding article. Accordingly, violations of a Collective Section 29. Dispute Settlement Procedures. − In cases of claims and disputes
Bargaining Agreement, except those which are gross in character, shall no arising from this employment, the parties covered by a collective bargaining
longer be treated as unfair labor practice and shall be resolved as grievances agreement shall submit the claim or dispute to the original and exclusive
under the Collective Bargaining Agreement. For purposes of this article, jurisdiction of the voluntary arbitrator or panel of arbitrators. If the parties
gross violations of Collective Bargaining Agreement shall mean flagrant are not covered by a collective bargaining agreement, the parties may at their
and/or malicious refusal to comply with the economic provisions of such option submit the claim or dispute to either the original and exclusive
agreement. jurisdiction of the National Labor Relations Commission (NLRC), pursuant
to Republic Act (RA) 8042, otherwise known as the Migrant Workers and
The Commission, its Regional Offices and the Regional Directors of the Overseas Filipinos Act of 1995 or to the original and exclusive jurisdiction
Department of Labor and Employment shall not entertain disputes, of the voluntary arbitrator or panel of arbitrators. If there is no provision as
grievances or matters under the exclusive and original jurisdiction of the to the voluntary arbitrators to be appointed by the parties, the same shall be
Voluntary Arbitrator or panel of Voluntary Arbitrators and shall immediately appointed from the accredited voluntary arbitrators of the National
dispose and refer the same to the Grievance Machinery or Voluntary Conciliation and Mediation Board of the Department of Labor and
Arbitration provided in the Collective Bargaining Agreement. Employment.

The petition is without merit. The Philippine Overseas Employment Administration (POEA) shall exercise
original and exclusive jurisdiction to hear and decide disciplinary action on
It is true that R.A. 8042 is a special law governing overseas Filipino workers. cases, which are administrative in character, involving or arising out of
However, a careful reading of this special law would readily show that there violations of recruitment laws, rules and regulations involving employers,
is no specific provision thereunder which provides for jurisdiction over principals, contracting partners and Filipino seafarers. (Emphasis supplied)
disputes or unresolved grievances regarding the interpretation or
implementation of a CBA. Section 10 of R.A. 8042, which is cited by It is clear from the above that the interpretation of the DOLE, in consultation
petitioner, simply speaks, in general, of "claims arising out of an employer- with their counterparts in the respective committees of the Senate and the
employee relationship or by virtue of any law or contract involving Filipino House of Representatives, as well as the DFA and the POEA is that with
workers for overseas deployment including claims for actual, moral, respect to disputes involving claims of Filipino seafarers wherein the parties
exemplary and other forms of damages." On the other hand, Articles 217(c) are covered by a collective bargaining agreement, the dispute or claim should
and 261 of the Labor Code are very specific in stating that voluntary be submitted to the jurisdiction of a voluntary arbitrator or panel of
arbitrators have jurisdiction over cases arising from the interpretation or arbitrators. It is only in the absence of a collective bargaining agreement that
implementation of collective bargaining agreements. Stated differently, the parties may opt to submit the dispute to either the NLRC or to voluntary
instant case involves a situation where the special statute (R.A. 8042) refers arbitration. It is elementary that rules and regulations issued by
to a subject in general, which the general statute (Labor Code) treats in administrative bodies to interpret the law which they are entrusted to enforce,
particular.5 In the present case, the basic issue raised by Merridy Jane in her have the force of law, and are entitled to great respect.8 Such rules and
complaint filed with the NLRC is: which provision of the subject CBA regulations partake of the nature of a statute and are just as binding as if they
applies insofar as death benefits due to the heirs of Nelson are concerned. have been written in the statute itself.9 In the instant case, the Court finds no
The Court agrees with the CA in holding that this issue clearly involves the cogent reason to depart from this rule.1âwphi1
interpretation or implementation of the said CBA. Thus, the specific or
special provisions of the Labor Code govern. The above interpretation of the DOLE, DFA and POEA is also in consonance
with the policy of the state to promote voluntary arbitration as a mode of
In any case, the Court agrees with petitioner's contention that the CBA is the settling labor disputes.10
law or contract between the parties. Article 13.1 of the CBA entered into by
and between respondent GCI and AMOSUP, the union to which petitioner No less than the Philippine Constitution provides, under the third paragraph,
belongs, provides as follows: Section 3, Article XIII, thereof that "[t]he State shall promote the principle
of shared responsibility between workers and employers and the preferential
The Company and the Union agree that in case of dispute or conflict in the use of voluntary modes in settling disputes, including conciliation, and shall
interpretation or application of any of the provisions of this Agreement, or enforce their mutual compliance therewith to foster industrial peace."
enforcement of Company policies, the same shall be settled through
negotiation, conciliation or voluntary arbitration. The Company and the Consistent with this constitutional provision, Article 211 of the Labor Code
Union further agree that they will use their best endeavor to ensure that any provides the declared policy of the State "[t]o promote and emphasize the
dispute will be discussed, resolved and settled amicably by the parties hereof primacy of free collective bargaining and negotiations, including voluntary
within ninety (90) days from the date of filing of the dispute or conflict and arbitration, mediation and conciliation, as modes of settling labor or
in case of failure to settle thereof any of the parties retain their freedom to industrial disputes."
take appropriate action.6 (Emphasis supplied)
On the basis of the foregoing, the Court finds no error in the ruling of the CA
From the foregoing, it is clear that the parties, in the first place, really that the voluntary arbitrator has jurisdiction over the instant case.
intended to bring to conciliation or voluntary arbitration any dispute or
conflict in the interpretation or application of the provisions of their CBA. It WHEREFORE, the petition is DENIED. The Decision and Resolution of the
is settled that when the parties have validly agreed on a procedure for Court of Appeals in CA-G.R. SP No. 76489 dated July 11, 2005 and April
resolving grievances and to submit a dispute to voluntary arbitration then that 18, 2006, respectively, are AFFIRMED.
procedure should be strictly observed.7
SO ORDERED.
It may not be amiss to point out that the abovequoted provisions of the CBA
are in consonance with Rule VII, Section 7 of the present Omnibus Rules and
Regulations Implementing the Migrant Workers and Overseas Filipinos Act
of 1995, as amended by Republic Act No. 10022, which states that "[f]or
OFWs with collective bargaining agreements, the case shall be submitted for
voluntary arbitration in accordance with Articles 261 and 262 of the Labor
Code." The Court notes that the said Omnibus Rules and Regulations were
promulgated by the Department of Labor and Employment (DOLE) and the
Department of Foreign Affairs (DFA) and that these departments were
mandated to consult with the Senate Committee on Labor and Employment
and the House of Representatives Committee on Overseas Workers Affairs.

In the same manner, Section 29 of the prevailing Standard Terms and


Conditions Governing the Employment of Filipino Seafarers on Board Ocean
Going Vessels, promulgated by the Philippine Overseas Employment
Administration (POEA), provides as follows:
ST. MARTIN FUNERAL HOME vs. NATIONAL LABOR said provision and abolished such appeals. No appellate review has since then
RELATIONS COMMISSION been provided for.
G.R. No. 130866 | 1998-09-16
REGALADO, J.: Thus, to repeat, under the present state of the law, there is no provision for
appeals from the decision of the NLRC.10 [While Art. 223 bears the epigraph
The present petition for certiorari stemmed from a complaint for illegal of "Appeal," it actually refers only to decisions, awards, or orders of the labor
dismissal filed by herein private respondent before the National Labor arbiter which shall be final and executory unless appealed to the NLRC by
Relations Commission (NLRC), Regional Arbitration Branch No. III, in San any or both parties within ten calendar days from receipt thereof.] The present
Fernando, Pampanga. Private respondent alleges that he started working as Section 223, as last amended by Section 12 of R.A. No. 6715, instead merely
Operations Manager of petitioner St. Martin Funeral Home on February 6, provides that the Commission shall decide all cases within twenty days from
1995. However, there was no contract of employment executed between him receipt of the answer of the appellee, and that such decision shall be final and
and petitioner nor was his name included in the semi-monthly payroll. On executory after ten calendar days from receipt thereof by the parties.
January 22, 1996, he was dismissed from his employment for allegedly
misappropriating P38,000.00 which was intended for payment by petitioner When the issue was raised in an early case on the argument that this Court
of its value added tax (VAT) to the Bureau of Internal Revenue (BIR).1 has no jurisdiction to review the decisions of the NLRC, and formerly of the
[Rollo, 17.] Secretary of Labor, since there is no legal provision for appellate review
thereof, the Court nevertheless rejected that thesis. It held that there is an
Petitioner on the other hand claims that private respondent was not its underlying power of the courts to scrutinize the acts of such agencies on
employee but only the uncle of Amelita Malabed, the owner of petitioner St. questions of law and jurisdiction even though no right of review is given by
Martin's Funeral Home. Sometime in 1995, private respondent, who was statute; that the purpose of judicial review is to keep the administrative
formerly working as an overseas contract worker, asked for financial agency within its jurisdiction and protect the substantial rights of the parties;
assistance from the mother of Amelita. Since then, as an indication of and that it is that part of the checks and balances which restricts the separation
gratitude, private respondent voluntarily helped the mother of Amelita in of powers and forestalls arbitrary and unjust adjudications.11 [San Miguel
overseeing the business. Corporation vs. Secretary of Labor, et al., G.R. No. L-39195, May 15, 1975,
64 SCRA 56; Scott vs. Inciong, et al. G.R. No. L-38868, December 29, 1975,
In January 1996, the mother of Amelita passed away, so the latter she took 68 SCRA 473; Bordeos, et al., vs. NLRC, et al., G.R. Nos. 115314-23,
over the management of the business. She then discovered that there were September 26, 1996, 262 SCRA 424.]
arrears in the payment of taxes and other government fees, although the
records purported to show that the same were already paid. Amelita then Pursuant to such ruling, and as sanctioned by subsequent decisions of this
made some changes in the business operation and private respondent and his Court, the remedy of the aggrieved party is to timely file a motion for
wife were no longer allowed to participate in the management thereof. As a reconsideration as a precondition for any further or subsequent remedy,12
consequence, the latter filed a complaint charging that petitioner had illegally [Zapata vs. NLRC, et al., G.R. No. 77827, July 5, 1989, 175 SCRA 56.] and
terminated his employment.2 [Ibid., 18-19.] then seasonably avail of the special civil action of certiorari under Rule 65,
13 [See, for instance, Pure Foods Corporation vs. NLRC, et al., G.R. No.
Based on the position papers of the parties, the labor arbiter rendered a 78591, March 21, 1989, 171 SCRA 415.] for which said Rule has now fixed
decision in favor of petitioner on October 25, 1996 declaring that no the reglementary period of sixty days from notice of the decision. Curiously,
employer-employee relationship existed between the parties and, therefore, although the 10-day period for finality of the decision of the NLRC may
his office had no jurisdiction over the case.3 [Ibid., 19.] already have lapsed as contemplated in Section 223 of the Labor Code, it has
been held that this Court may still take cognizance of the petition for
Not satisfied with the said decision, private respondent appealed to the NLRC certiorari on jurisdictional and due process considerations if filed within the
contending that the labor arbiter erred (1) in not giving credence to the reglementary period under Rule 65.14 [Mantrade, etc. vs. Bacungan, et al.,
evidence submitted by him; (2) in holding that he worked as a "volunteer" G.R. No. L-48437, September 30, 1986, 144 SCRA 511.]
and not as an employee of St. Martin Funeral Home from February 6, 1995
to January 23, 1996, or a period of about one year; and (3) in ruling that there Turning now to the matter of judicial review of NLRC decisions, B.P. No.
was no employer-employee relationship between him and petitioner.4 [Ibid., 129 originally provided as follows:
16.] SEC. 9. Jurisdiction. - The Intermediate Appellate Court shall exercise:

On June 13, 1997, the NLRC rendered a resolution setting aside the (1) Original jurisdiction to issue writs of mandamus, prohibition, certiorari,
questioned decision and remanding the case to the labor arbiter for immediate habeas corpus, and quo warranto, and auxiliary writs or processes, whether
appropriate proceedings.5 [Ibid., 21.] Petitioner then filed a motion for or not in aid of its appellate jurisdiction;
reconsideration which was denied by the NLRC in its resolution dated
August 18, 1997 for lack of merit,6 [Ibid., 23-24.] hence the present petition (2) Exclusive original jurisdiction over actions for annulment of judgments
alleging that the NLRC committed grave abuse of discretion.7 [Ibid., 6.] of Regional Trial Courts; and

Before proceeding further into the merits of the case at bar, the Court feels (3) Exclusive appellate jurisdiction over all final judgments, decisions,
that it is now exigent and opportune to reexamine the functional validity and resolutions, orders, or awards of Regional Trial Courts and quasi-judicial
systemic practicability of the mode of judicial review it has long adopted and agencies, instrumentalities, boards, or commissions, except those falling
still follows with respect to decisions of the NLRC. The increasing number within the appellate jurisdiction of the Supreme Court in accordance with the
of labor disputes that find their way to this Court and the legislative changes Constitution, the provisions of this Act, and of subparagraph (1) of the third
introduced over the years into the provisions of Presidential Decree (P.D.) paragraph and subparagraph (4) of the fourth paragraph of Section 17 of the
No. 442 (The Labor Code of the Philippines and Batas Pambansa Blg. (B.P. Judiciary Act of 1948.
No.) 129 (The Judiciary Reorganization Act of 1980) now stridently call for
and warrant a reassessment of that procedural aspect. The Intermediate Appellate Court shall have the power to try cases and
conduct hearings, receive evidence and perform any and all acts necessary to
We prefatorily delve into the legal history of the NLRC. It was first resolve factual issues raised in cases falling within its original and appellate
established in the Department of Labor by P.D. No. 21 on October 14, 1972, jurisdiction, including the power to grant and conduct new trials or further
and its decisions were expressly declared to be appealable to the Secretary of proceedings.
Labor and, ultimately, to the President of the Philippines.
These provisions shall not apply to decisions and interlocutory orders issued
On May 1, 1974, P.D. No. 442 enacted the Labor Code of the Philippines, under the Labor Code of the Philippines and by the Central Board of
the same to take effect six months after its promulgation. 8 [Article 2.] Assessment Appeals.15 [75 O.G. 4781, August 29, 1983.]
Created and regulated therein is the present NLRC which was attached to the
Department of Labor and Employment for program and policy coordination Subsequently, and as it presently reads, this provision was amended by R.A.
only. 9 [Article 213.] Initially, Article 302 (now, Article 223) thereof also No. 7902 effective March 18, 1995, to wit:
granted an aggrieved party the remedy of appeal from the decision of the
NLRC to the Secretary of Labor, but P.D. No. 1391 subsequently amended SEC. 9. Jurisdiction. - The Court of Appeals shall exercise:
(1) Original jurisdiction to issue writs of mandamus, prohibition, certiorari, Constitution, the provisions of B.P. No. 129, and those specified cases in
habeas corpus, and quo warranto, and auxiliary writs or processes, whether Section 17 of the Judiciary Act of 1948. These cases can, of course, be
or not in aid of its appellate jurisdiction; properly excluded from the exclusive appellate jurisdiction of the Court of
Appeals. However, because of the aforementioned amendment by
(2) Exclusive original jurisdiction over actions for annulment of judgments transposition, also supposedly excluded are cases falling within the appellate
of Regional Trial Courts; and jurisdiction of the Supreme Court in accordance with the Labor Code. This
is illogical and impracticable, and Congress could not have intended that
(3) Exclusive appellate jurisdiction over all final judgments, decisions, procedural gaffe, since there are no cases in the Labor Code the decisions,
resolutions, orders or awards of Regional Trial Courts and quasi-judicial resolutions, orders or awards wherein are within the appellate jurisdiction of
agencies, instrumentalities, boards or commissions, including the Securities the Supreme Court or of any other court for that matter.
and Exchange Commission, the Social Security Commission, the Employees
Compensation Commission and the Civil Service Commission, except those A review of the legislative records on the antecedents of R.A. No. 7902
falling within the appellate jurisdiction of the Supreme Court in accordance persuades us that there may have been an oversight in the course of the
with the Constitution, the Labor Code of the Philippines under Presidential deliberations on the said Act or an imprecision in the terminology used
Decree No. 442, as amended, the provisions of this Act, and of subparagraph therein. In fine, Congress did intend to provide for judicial review of the
(1) of the third paragraph and subparagraph (4) of the fourth paragraph of adjudications of the NLRC in labor cases by the Supreme Court, but there
Section 17 of the Judiciary Act of 1948. was an inaccuracy in the term used for the intended mode of review. This
conclusion which we have reluctantly but prudently arrived at has been
The Court of Appeals shall have the power to try cases and conduct hearings, drawn from the considerations extant in the records of Congress, more
receive evidence and perform any and all acts necessary to resolve factual particularly on Senate Bill No. 1495 and the Reference Committee Report on
issues raised in cases falling within its original and appellate jurisdiction, S. No. 1495/H. No. 10452.18 [An Act Expanding the Jurisdiction of the
including the power to grant and conduct new trials or further proceedings. Court of Appeals, Amending for the Purpose Section 9 of Batas Pambansa
Trials or hearings in the Court of Appeals must be continuous and must be Blg. 129, known as the Judiciary Reorganization Act of 1980.]
completed within, three (3) months, unless extended by the Chief Justice."
In sponsoring Senate Bill No. 1495, Senator Raul S. Roco delivered his
It will readily be observed that, aside from the change in the name of the sponsorship speech19 [Transcript of Session Proceedings (TSP), S. No.
lower appellate court,16 [Executive Order No. 33 restored the name of the 1495, February 8, 1995, 31-36.] from which we reproduce the following
Court of Appeals, in lieu of the Intermediate Appellate Court, effective July excerpts:
28, 1986.] the following amendments of the original provisions of Section 9
of B.P. No. 129 were effected by R.A. No. 7902, viz.: The Judiciary Reorganization Act, Mr. President, Batas Pambansa Blg. 129,
reorganized the Court of Appeals and at the same time expanded its
1. The last paragraph which excluded its application to the Labor Code of the jurisdiction and powers. Among others, its appellate jurisdiction was
Philippines and the Central Board of Assessment Appeals was deleted and expanded to cover not only final judgment of Regional Trial Courts, but also
replaced by a new paragraph granting the Court of Appeals limited powers all final judgment(s), decisions, resolutions, orders or awards of quasi-
to conduct trials and hearings in cases within its jurisdiction. judicial agencies, instrumentalities, boards and commissions, except those
falling within the appellate jurisdiction of the Supreme Court in accordance
2. The reference to the Labor Code in that last paragraph was transposed to with the Constitution, the provisions of BP Blg. 129 and of subparagraph 1
paragraph (3) of the section, such that the original exclusionary clause therein of the third paragraph and subparagraph 4 of Section 17 of the Judiciary Act
now provides "except those falling within the appellate jurisdiction of the of 1948.
Supreme Court in accordance with the Constitution, the Labor Code of the
Philippines under Presidential Decree No. 442, as amended, the provisions Mr. President, the purpose of the law is to ease the workload of the Supreme
of this Act, and of subparagraph (1) of the third paragraph and subparagraph Court by the transfer of some of its burden of review of factual issues to the
(4) of the fourth paragraph of Section 17 of the Judiciary Act of 1948." Court of Appeals. However, whatever benefits that can be derived from the
(Italics supplied) expansion of the appellate jurisdiction of the Court of Appeals was cut short
3. Contrarily, however, specifically added to and included among the quasi- by the last paragraph of Section 9 of Batas Pambansa Blg. 129 which
judicial agencies over which the Court of Appeals shall have exclusive excludes from its coverage the "decisions and interlocutory orders issued
appellate jurisdiction are the Securities and Exchange Commission, the under the Labor Code of the Philippines and by the Central Board of
Social Security Commission, the Employees Compensation Commission and Assessment Appeals."
the Civil Service Commission. Among the highest number of cases that are brought up to the Supreme Court
are labor cases. Hence, Senate Bill No. 1495 seeks to eliminate the exceptions
This, then, brings us to a somewhat perplexing impasse, both in point of enumerated in Section 9 and, additionally, extends the coverage of appellate
purpose and terminology. As earlier explained, our mode of judicial review review of the Court of Appeals in the decision(s) of the Securities and
over decisions of the NLRC has for some time now been understood to be by Exchange Commission, the Social Security Commission, and the Employees
a petition for certiorari under Rule 65 of the Rules of Court. This is, of course, Compensation Commission to reduce the number of cases elevated to the
a special original action limited to the resolution of jurisdictional issues, that Supreme Court.
is, lack or excess of jurisdiction and, in almost all cases that have been
brought to us, grave abuse of discretion amounting to lack of jurisdiction. xxx

It will, however, be noted that paragraph (3), Section 9 of B.P. No. 129 now Senate Bill No. 1495 authored by our distinguished Colleague from Laguna
grants exclusive appellate jurisdiction to the Court of Appeals over all final provides the ideal situation of drastically reducing the workload of the
adjudications of the Regional Trial Courts and the quasi-judicial agencies Supreme Court without depriving the litigants of the privilege of review by
generally or specifically referred to therein except, among others, "those an appellate tribunal.
falling within the appellate jurisdiction of the Supreme Court in accordance
with x x x the Labor Code of the Philippines under Presidential Decree No. In closing, allow me to quote the observations of former Chief Justice
442, as amended, x x x." This would necessarily contradict what has been Teehankee in 1986 in the Annual Report of the Supreme Court:
ruled and said all along that appeal does not lie from decisions of the x x x Amendatory legislation is suggested so as to relieve the Supreme Court
NLRC.17 [The different modes of appeal, that is, by writ of error (Rule 41), of the burden of reviewing these cases which present no important issues
petition for review (Rules 42 and 43), and petition for review on certiorari involved beyond the particular fact and the parties involved, so that the
(Rule 45) obviously cannot be availed of because there is no provision for Supreme Court may wholly devote its time to cases of public interest in the
appellate review of NLRC decisions in P.D. No. 442, as amended.] Yet, discharge of its mandated task as the guardian of the Constitution and the
under such excepting clause literally construed, the appeal from the NLRC guarantor of the people's basic rights and additional task expressly vested on
cannot be brought to the Court of Appeals, but to this Court by necessary it now "to determine whether or not there has been a grave abuse of discretion
implication. amounting to lack of jurisdiction on the part of any branch or instrumentality
of the Government."
The same exceptive clause further confuses the situation by declaring that the We used to have 500,000 cases pending all over the land, Mr. President. It
Court of Appeals has no appellate jurisdiction over decisions falling within has been cut down to 300,000 cases some five years ago. I understand we are
the appellate jurisdiction of the Supreme Court in accordance with the
now back to 400,000 cases. Unless we distribute the work of the appellate conclusions of said bodies are correspondingly affirmed, modified or
courts, we shall continue to mount and add to the number of cases pending. reversed.
Under such guarantee, the Supreme Court can then apply strictly the axiom
In view of the foregoing, Mr. President, and by virtue of all the reasons we that factual findings of the Court of Appeals are final and may not be reversed
have submitted, the Committee on Justice and Human Rights requests the on appeal to the Supreme Court. A perusal of the records will reveal appeals
support and collegial approval of our Chamber. which are factual in nature and may, therefore, be dismissed outright by
minute resolutions.24 [TSP, S. No. 1495, February 8, 1995, pp. 32-33.]
xxx
While we do not wish to intrude into the Congressional sphere on the matter
Surprisingly, however, in a subsequent session, the following Committee of the wisdom of a law, on this score we add the further observations that
Amendment was introduced by the said sponsor and the following there is a growing number of labor cases being elevated to this Court which,
proceedings transpired:20 [TSP, id., February 15, 1995, 18-19.] not being a trier of fact, has at times been constrained to remand the case to
the NLRC for resolution of unclear or ambiguous factual findings; that the
Senator Roco. On page 2, line 5, after the line "Supreme Court in accordance Court of Appeals is procedurally equipped for that purpose, aside from the
with the Constitution," add the phrase "THE LABOR CODE OF THE increased number of its component divisions; and that there is undeniably an
PHILIPPINES UNDER P.D. 442, AS AMENDED." So that it becomes clear, imperative need for expeditious action on labor cases as a major aspect of
Mr. President, that issues arising from the Labor Code will still be appealable constitutional protection to labor.
to the Supreme Court.
Therefore, all references in the amended Section 9 of B.P. No. 129 to
The President. Is there any objection? (Silence) Hearing none, the supposed appeals from the NLRC to the Supreme Court are interpreted and
amendment is approved. hereby declared to mean and refer to petitions for certiorari under Rule 65.
Consequently, all such petitions should henceforth be initially filed in the
Senator Roco. On the same page, we move that lines 25 to 30 be deleted. Court of Appeals in strict observance of the doctrine on the hierarchy of
This was also discussed with our Colleagues in the House of Representatives courts as the appropriate forum for the relief desired.
and as we understand it, as approved in the House, this was also deleted, Mr.
President. Apropos to this directive that resort to the higher courts should be made in
accordance with their hierarchical order, this pronouncement in Santiago vs.
The President. Is there any objection? (Silence) Hearing none, the Vasquez, et al.25 [G.R. Nos. 99289-90, January 27, 1993, 217 SCRA 633.
amendment is approved. See also Tano, et al. vs. Socrates, et al., G.R. No. 110249, August 21, 1997,
278 SCRA 155.] should be taken into account:
Senator Roco. There are no further Committee amendments, Mr. President.
One final observation. We discern in the proceedings in this case a propensity
Senator Romulo. Mr. President, I move that we close the period of on the part of petitioner, and, for that matter, the same may be said of a
Committee amendments. number of litigants who initiate recourses before us, to disregard the
hierarchy of courts in our judicial system by seeking relief directly from this
The President. Is there any objection? (Silence) Hearing none, the Court despite the fact that the same is available in the lower courts in the
amendment is approved. exercise of their original or concurrent jurisdiction, or is even mandated by
law to be sought therein. This practice must be stopped, not only because of
xxx the imposition upon the precious time of this Court but also because of the
inevitable and resultant delay, intended or otherwise, in the adjudication of
Thereafter, since there were no individual amendments, Senate Bill No. 1495 the case which often has to be remanded or referred to the lower court as the
was passed on second reading and being a certified bill, its unanimous proper forum under the rules of procedure, or as better equipped to resolve
approval on third reading followed.21 [TSP, id., id., 19-21; Record of the the issues since this Court is not a trier of facts. We, therefore, reiterate the
Senate, Vol. V, No. 63, pp. 180-181.] The Conference Committee Report on judicial policy that this Court will not entertain direct resort to it unless the
Senate Bill No. 1495 and House Bill No. 10452, having theretofore been redress desired cannot be obtained in the appropriate courts or where
approved by the House of Representatives, the same was likewise approved exceptional and compelling circumstances justify availment of a remedy
by the Senate on February 20, 1995,22 [TSP, id., February 20, 1995, pp. 42- within and calling for the exercise of our primary jurisdiction.
43.] inclusive of the dubious formulation on appeals to the Supreme Court
earlier discussed. WHEREFORE, under the foregoing premises, the instant petition for
certiorari is hereby REMANDED, and all pertinent records thereof ordered
The Court is, therefore, of the considered opinion that ever since appeals to be FORWARDED, to the Court of Appeals for appropriate action and
from the NLRC to the Supreme Court were eliminated, the legislative disposition consistent with the views and ruling herein set forth, without
intendment was that the special civil action of certiorari was and still is the pronouncement as to costs.
proper vehicle for judicial review of decisions of the NLRC. The use of the
word "appeal" in relation thereto and in the instances we have noted could SO ORDERED.
have been a lapsus plumae because appeals by certiorari and the original
action for certiorari are both modes of judicial review addressed to the
appellate courts. The important distinction between them, however, and with
which the Court is particularly concerned here is that the special civil action
of certiorari is within the concurrent original jurisdiction of this Court and
the Court of Appeals;23 [The Regional Trial Court also shares that
concurrent jurisdiction but that cannot be considered with regard to the
NLRC since they are of the same rank.] whereas to indulge in the assumption
that appeals by certiorari to the Supreme Court are allowed would not
subserve, but would subvert, the intention of Congress as expressed in the
sponsorship speech on Senate Bill No. 1495.

Incidentally, it was noted by the sponsor therein that some quarters were of
the opinion that recourse from the NLRC to the Court of Appeals as an initial
step in the process of judicial review would be circuitous and would prolong
the proceedings. On the contrary, as he commendably and realistically
emphasized, that procedure would be advantageous to the aggrieved party on
this reasoning:

On the other hand, Mr. President, to allow these cases to be appealed to the
Court of Appeals would give litigants the advantage to have all the evidence
on record be reexamined and reweighed after which the findings of facts and
PHILIPPINE AIRLINES, INC. VS. AIRLINE PILOTS prescriptive period set forth in Article 291 of the Labor Code. The LA
ASSOCIATION OF THE PHILIPPINES suggested, however, that PAL's cause of action may be treated as an
G.R. No. 200088 | 2018-02-26 independent civil action in another forum. The dispositive portion reads:
MARTIRES, J.:
WHEREFORE, the complaint is DISMISSED for lack of merit.
This is a petition for review on certiorari under Rule 45 of the Rules of Court
seeking the reversal of the 26 August 2011 Decision[1] and 05 January 2012 SO ORDERED.[13]
Resolution[2] of the Court of Appeals (CA) in CA-G.R. SP No. 113985,
which affirmed with modification the 27 April 2009[3] and 26 February Aggrieved, PAL elevated an appeal to the NLRC.
2010[4] Resolutions of the National Labor Relations Commission (NLRC)
in NLRC LAC No. 036558-03 (RA-10-08), which likewise affirmed with The NLRC Ruling
modification the 22 April 2008 Decision[5] of the Labor Arbiter (LA) in
NLRC NCR No. 04-04906-03. In its resolution, dated 27 April 2009, the NLRC affirmed with modification
the LA's 22 April 2008 decision. It ruled that labor tribunals have no
THE FACTS jurisdiction over the claims interposed by PAL. It opined that the reliefs
prayed for by PAL should have been ventilated before the regular courts
The present case arose from a labor dispute between petitioner Philippine considering that they are based on the tortuous acts allegedly committed by
Airlines, Inc. (PAL) and respondent Airline Pilots' Association of the the respondents. It explained that the airline pilots' refusal to fly their
Philippines (ALPAP), a duly registered labor organization and the exclusive assigned aircrafts constitutes breach of contractual obligation which is
bargaining agent of all commercial pilots of PAL. On 9 December 1997, intrinsically a civil dispute. The dispositive portion of the resolution states:
ALPAP filed with the Department of Labor and Employment (DOLE) a
notice of strike alleging that PAL committed unfair labor practice. On 23 WHEREFORE, except for the MODIFICATION that the phrase "for lack of
December 1997, the Secretary of DOLE (SOLE) assumed jurisdiction over merit" in the dispositive portion is deleted therefrom, the appealed Decision
the dispute and thereafter prohibited ALPAP from staging a strike and is hereby AFFIRMED.
committing any act that could exacerbate the dispute.[6]
SO ORDERED.[14]
Despite the prohibition by the SOLE, ALPAP staged a strike on 5 June 1998.
A return-to-work order[7] was issued by the SOLE on 7 June 1998, but PAL moved for reconsideration, but the same was denied by the NLRC in its
ALPAP defied the same and went on with their strike. Consequently, on 1 resolution, dated 26 February 2010.
June 1999, the SOLE issued a resolution[8] which declared the illegality of
the strike staged by ALPAP and the loss of employment status of the officers Unconvinced, PAL filed a petition for certiorari under Rule 65 of the Rules
who participated in the strike. of Court before the CA.

The SOLE's resolution was upheld by the CA in CA-G.R. SP No. 54880.[9] The CA Ruling
The matter was eventually elevated to this Court in G.R. No. 152306. In a
Resolution,[10] dated 10 April 2002, the Court dismissed ALPAP's petition In its assailed Decision, dated 26 August 2011, the CA partially granted
for failure to show that the CA committed grave abuse of discretion or a PAL's petition. It ruled that while the NLRC correctly sustained the LA's
reversible error. The resolution attained finality on 29 August 2002.[11] dismissal of the complaint for lack of jurisdiction, it declared that the NLRC
gravely abused its discretion when it affirmed the LA's pronouncement that
On 22 April 2003, or almost eight (8) months :from the finality of the Court's PAL's cause of action had already prescribed.
10 April 2002 Resolution, PAL filed before the LA a complaint[12] for
damages against ALPAP, as well as some of its officers and members. The appellate court concurred with the NLRC's opinion that exclusive
jurisdiction over PAL's claim for damages lies with the regular courts and
PAL alleged, among others, that on 6 June 1998, the second day of the illegal not with the SOLE. It ratiocinated that while Article 263(g) of the Labor
strike conducted by ALPAP, its striking pilots abandoned three (3) PAL Code vests in the SOLE the authority to resolve all questions and
aircraft, as follows: (i) PR 730 bound for Paris, France, at Bangkok, Thailand; controversies arising from a labor dispute over which it assumed jurisdiction,
(ii) PR 741 bound for Manila, at Bangkok, Thailand; and (iii) PR 104 bound said authority must be interpreted to cover only those causes of action which
for Manila, at San Francisco, California, U.S.A. Because of the deliberate are based on labor laws. Stated differently, causes of action based on an
and malicious abandonment of the said flights, its passengers were stranded, obligation or duty not provided under the labor laws are beyond the SOLE's
and rendered PAL liable for violation of its contract of carriage. Thus, PAL jurisdiction. It continued that only those issues that arise from the assumed
was compelled to incur expenses by way of hotel accommodations, meals for labor dispute, which has a direct causal connection to the employer-employee
the stranded passengers, airport parking fees, and other operational expenses. relationship between the parties, will fall under the jurisdiction of the SOLE.
PAL further alleged that its operation was crippled by the illegal strike It pointed out that the damages caused by the wilful acts of the striking pilots
resulting in several losses from ticket refunds, extraordinary expenses to cope in abandoning their aircraft are recoverable under civil law and are thus
with the shutdown situation, and lost income from the cancelled domestic within the jurisdiction of the regular courts.
and international flights. PAL claimed that, as a result of the illegal strike, it
suffered actual damages in the amount of P731,078,988.59. PAL further Further, the appellate court held that PAL's cause of action accrued only on
prayed that it be awarded P300,000,000.00 and P3,000,000.00 as exemplary 29 August 2002, the date when this Court's resolution sustaining the finding
damages and attorney's fees, respectively. of the strike's illegality had attained finality. The dispositive portion of the
assailed decision reads:
The LA Ruling
WHEREFORE, premises considered, the Petition for Certiorari is
In its decision, dated 22 April 2008, the LA dismissed PAL's complaint. It PARTIALLY GRANTED. The April 27, 2009 and February 26, 2010 NLRC
ruled that it had no jurisdiction to resolve the issue on damages. It noted that Resolutions are MODIFIED as follows:
the SOLE did not certify the controversy for compulsory arbitration to the
NLRC nor in any occasion did the parties agree to refer the same to voluntary 1) The complaint for damages arising from the illegal strike claimed by the
arbitration under Article 263(h) of the Labor Code. Hence, jurisdiction to petitioner lies not within the jurisdiction of the DOLE Secretary or the Labor
resolve all issues arising from the labor dispute, including the claim for Arbiter but with the regular courts; and
damages arising from the illegal strike, was left with the SOLE to the
exclusion of all other fora. 2) Petitioner's cause of action for damages has not yet prescribed.

The LA further ruled that PAL's cause of action had already been barred by No costs.
prescription. It opined that since the complaint was premised on the illegality
of the strike held by the respondents, the accrual of PAL's cause of action SO ORDERED.[15]
should be reckoned either on 5 June 1998, the first day of the strike, or on 7
June 1998, when the respondents defied the SOLE's return-to-work order. PAL moved for partial reconsideration but the same was denied by the CA
Hence, PAL's 22 April 2003 complaint was filed beyond the 3-year in its assailed Resolution, dated 5 January 2012.
Hence, this petition. The appellate court is mistaken.

THE ISSUE The Court agrees with PAL that its claim for damages has reasonable
connection with its employer-employee relationship with the respondents.
WHETHER THE NLRC AND THE LABOR ARBITER HAVE Contrary to the pronouncements made by the appellate court, PAL's cause of
JURISDICTION OVER PAL'S CLAIMS AGAINST THE RESPONDENTS action is not grounded on mere acts of quasi-delict. The claimed damages
FOR DAMAGES INCURRED AS A CONSEQUENCE OF THE arose from the illegal strike and acts committed during the same which were
LATTER'S ACTIONS DURING THE ILLEGAL STRIKE. in tum closely related and intertwined with the respondents' allegations of
unfair labor practices against PAL. This could not even be disputed as even
THE COURT'S RULING the appellate court recognized this fact. In its 26 August 2011 Decision, the
CA made the following statements:
The petition is partially meritorious.
The damages caused by the willful act of the striking pilots in abandoning
Labor tribunals have jurisdiction their aircrafts, together with the passengers and cargo, which resulted in
over actions for damages arising injury to petitioner's business is recoverable under civil law.[19] [emphasis
from a labor strike. supplied]

Under Article 217 [now Article 224] of the Labor Code, as amended by xxx
Section 9 of R.A. No. 6715, the LA and the NLRC have jurisdiction to
resolve cases involving claims for damages arising from employer-employee 1) The complaint for damages arising from the illegal strike claimed by
relationship, to wit: petitioner lies not within the jurisdiction of the DOLE Secretary or the Labor
Arbiter but with the regular courts; xxx[20] [emphasis supplied]
ART. 217. Jurisdiction of Labor Arbiters and the Commission-- (a) Except
as otherwise provided under this Code, the Labor Arbiters shall have original Since the loss and injury from which PAL seeks compensation have
and exclusive jurisdiction to hear and decide, within thirty (30) calendar days reasonable causal connection with the alleged acts of unfair labor practice, a
after the submission of the case by the parties for decision without extension, claim provided for in Article 217 of the Labor Code, the question of damages
even in the absence of stenographic notes, the following cases involving all becomes a labor controversy and is therefore an employment relationship
workers, whether agricultural or nonagricultural: dispute.

1. Unfair labor practice cases; This issue is not novel. It has been previously decided by the Court in several
cases.
2. Termination disputes;
In Goodrich Employees Association v. Hon. Flores,[21] the Court stressed
3. If accompanied with a claim for reinstatement, those cases that workers the rule that cases involving unfair labor practices are within the jurisdiction
may file involving wages, rates of pay, hours of work and other terms and of the Court of Industrial Relations (CIR), the labor tribunal at that time. The
conditions of employment Court further emphasized that where the subject matter is within the
exclusive jurisdiction of the CIR, it must be deemed to have jurisdiction over
4. Claims for actual, moral, exemplary and other forms of damages arising all incidental matters connected to the main issue.
from employer-employee relations;
Thus, in Holganza v. Hon. Apostol,[22] the Court reaffirmed the exclusive
5. Cases arising from any violation of Article 264 of this Code including jurisdiction of the labor tribunal over actions for damages arising from labor
questions involving the legality of strikes and lockouts; and controversies. In the said case, the Social Security System (SSS) filed with
the then Court of First Instance (CFI) of Rizal a complaint for damages with
6. Except claims for Employees Compensation, Social Security, Medicare writ of preliminary attachment against several of its employees. It alleged
and maternity benefits, all other claims, arising from employer-employee that it sustained damages as a consequence of the picketing carried on by its
relations, including those of persons in domestic or household service, striking employees during a strike held against it. The striking employees
involving an amount exceeding five thousand pesos (P5,000.00) regardless moved for the dismissal of the complaint on the ground of lack of
of whether accompanied with a claim for reinstatement. jurisdiction, but the trial court denied the same. Eventually, the issue reached
this Court which opined that the trial court is devoid of any jurisdiction to
[emphases supplied] entertain the said complaint for damages. In so ruling, the Court declared that
exclusive jurisdiction over disputes of this character belonged to the then
It is settled, however, that not every controversy or money claim by an CIR. To hold otherwise would be to sanction split jurisdiction which is
employee against the employer or vice-versa falls within the jurisdiction of obnoxious to the orderly administration of justice.
the labor arbiter.[16] Intrinsically, civil disputes, although involving the
claim of an employer against its employees, are cognizable by regular A similar controversy arose in Philippine Long Distance Telephone
courts.[17] Company v. Free Telephone Workers Union.[23] The Court reiterated the
rule that regular courts are devoid of any jurisdiction over claims for damages
To determine whether a claim for damages under paragraph 4 of Article 217 arising from a labor strike, thus:
is properly cognizable by the labor arbiter, jurisprudence has evolved the
"reasonable connection rule" which essentially states that the claim for It is clear from the records that the subject complaint for damages is
damages must have reasonable causal connection with any of the claims intertwined with or deeply rooted from the 1964 certified labor dispute
provided for in that article. A money claim by a worker against the employer between appellant and appellees. As can be gleaned from the aforesaid
or vice-versa is within the exclusive jurisdiction of the labor arbiter only if complaint, appellant is claiming against appellees damages it allegedly
there is a "reasonable causal connection" between the claim asserted and sustained as a consequence of the strikes declared by the appellees. It is
employee-employer relations. Only if there is such a connection with the therefore obvious in the light of the established jurisprudence as aforestated
other claims can the claim for damages be considered as arising from that the lower court, Court of First Instance of Manila, Branch XII, did not
employer-employee relations.[18] Absent such a link, the complaint will be have jurisdiction over the aforesaid complaint for damages; hence, all the
cognizable by the regular courts. proceedings taken therein are void for lack of jurisdiction.[24]

The appellate court was of the opinion that, applying the reasonable The rule stands even if the strike is illegal. In Antipolo Highway Lines
connection rule, PAL's claims for damages have no relevant connection Employees Union v. Hon. Aquino.[25] Francisco De Jesus, the owner of
whatsoever to the employer-employee relationship between the parties. Thus, Antipolo Highway Lines (AHL), instituted a complaint for damages with
the claim is within the exclusive jurisdiction of the regular courts. It injunction against AHL Employees Union (AHLEU) and its officers before
explained that Article 217 of the Labor Code does not include a claim for the CFI of Rizal. De Jesus alleged that AHLEU staged a strike and posted
damages wherein the employer-employee relation is merely incidental, and picket lines along AHL's compound, thereby preventing its employees from
where the claim is largely civil in character. performing their work and causing it to suffer losses and damages from the
non-operation of its buses. The Court ruled that the trial court lacked
jurisdiction over the complaints for damages and injunction because the While there is merit in the contention that regular courts do not have
illegal strike and picket which allegedly caused damages to De Jesus were jurisdiction over claims for damages arising from a labor controversy, the
mere incidents of the labor dispute between the parties, to wit: Court opines that PAL could no longer recover the alleged damages.

Although it was artfully made to appear that the suit was one for damages It must be recalled that the SOLE assumed jurisdiction over the labor dispute
that did not divest the Court of Industrial Relations of its jurisdiction. The between PAL and the respondents on 23 December 1997. In this regard, it is
Complaint itself, in paragraph 5, adverted to an "illegal strike" and "picket settled that the authority of the SOLE to assume jurisdiction over a labor
lines," which are but mere incidents or consequences of the unfair labor dispute causing or likely to cause a strike or lockout in an industry
practice complained against by petitioner Union. In other words, it is clear indispensable to national interest includes and extends to all questions and
that the cause of action for damages "arose out of or was necessarily controversies arising therefrom.[31] It has also been opined that when the
intertwined with" an alleged unfair labor practice committed by DE JESUS very reason for the SOLE's assumption of jurisdiction is the declaration of
in refusing to sit at the bargaining table. It is still the labor court: therefore, strike, any issue regarding the strike is not merely incidental to but is
that has jurisdiction, particularly under the principle that split jurisdiction is essentially involved in the labor dispute itself. [32]
not to be countenanced for being "obnoxious to the orderly administration of
justice."[26] It bears emphasis, even at the risk of being repetitious, that it is beyond
question that the issue on damages is a controversy which arose from the
Indeed, the aforecited cases were decided by this Court under R.A. No. 875 labor dispute between the parties herein. Consequently, when the SOLE
or the Industrial Peace Act. The Court is also not unmindful of the fact that assumed jurisdiction over the labor dispute, the claim for damages was
R.A. No. 875 had been completely superseded in 1974 by Presidential Decree deemed included therein. Thus, the issue on damages was also deemed
(P.D.) No. 442 or the Labor Code of the Philippines. Nevertheless, it could resolved when the SOLE decided the main controversy in its 1 June 1999
not be denied that the underlying rationale for the rule finds application even resolution declaring the illegality of the strike and the loss of employment
with the effectivity of the Labor Code. As in the Industrial Peace Act, status of the striking officers of ALPAP, as well as when the case was finally
splitting of jurisdiction is abhorred under the Labor Code.[27] settled by this Court in its 10 April 2002 Resolution in G.R. No. 152306. This
is true even if the respective resolutions of the SOLE, CA, and this Court
A case in point is National Federation of Labor v. Hon. Eisma,[28] decided were silent with respect to the damages.
by the Court under the provisions of the Labor Code. In case, as in those
cited, the employer, Zamboanga Wood Products, Inc., filed, before the CFI To insist that PAL may recover the alleged damages through its complaint
of Zamboanga City, a complaint for damages against the officers and before the LA would be to sanction a relitigation of the issue of damages
members of the labor union. The employer alleged that it incurred damages separately from the main issue of the legality of the strike from which it is
because the union officers and members blockaded the road leading to its intertwined. This runs counter to the proscription against split jurisdiction –
manufacturing division, thus preventing customers and suppliers free ingress the very principle invoked by PAL.
to or egress from their premises. The labor union, however, contended that
jurisdiction over the controversy belongs to the labor arbiter because the acts Likewise, PAL's claim for damages is barred under the doctrine of
complained of were incidents of picketing by the defendants who were then immutability of final judgment. Under the said doctrine, a decision that has
on strike against the employer. acquired finality becomes immutable and unalterable, and may no longer be
modified in any respect, even if the modification is meant to correct
The Court ruled in favor of the labor union and nullified the proceedings erroneous conclusions of fact and law, and whether it is made by the court
before the trial court. The Court opined that the complaint for damages is that rendered it or by the Highest Court of the land. Any act which violates
deeply rooted in the labor dispute between the parties and thus should be this principle must immediately be struck down.[33]
dismissed by the regular court for lack of jurisdiction. The Court stressed that
the wordings of Article 217 of the Labor Code is explicit and clear enough Whether the damages claimed by PAL are recoverable and to what extent
to mean that exclusive jurisdiction over suits for damages arising from a would depend on the evidence in the illegal strike case which had long
strike belongs to the labor arbiter, thus: attained finality.[34] PAL's recovery, therefore, would entail a relitigation
of the illegal strike case. The subject claim for damages would ultimately
Article 217 is to be applied the way it is worded. The exclusive original require the modification of a final judgment. This cannot be done. The
jurisdiction of a labor arbiter is therein provided for explicitly. It means, it dismissal of the present petition as well as the complaint for damages is
can only mean, that a court of first instance judge then, a regional trial court therefore in order.
judge now, certainly acts beyond the scope of the authority conferred on him
by law when he entertained the suit for damages, arising from picketing that In any event, PAL only has itself to blame for this blunder. It was already
accompanied a strike. That was squarely within the express terms of the law. aware that it had sustained damages even before the SOLE issued its
Any deviation cannot therefore be tolerated. So it has been the constant ruling resolution. It must be remembered that the damages allegedly sustained by
of this Court even prior to Lizarraga Hermanos v. Yap Tico, a 1913 decision. PAL were incurred as a consequence of the acts committed by the
The ringing words of the ponencia of Justice Moreland still call for respondents on the second day of the strike on 6 June 1998, or almost a year
obedience. Thus, "The first and fundamental duty of courts, in our judgment, prior to the issuance of the SOLE's resolution. However, PAL did not assert
is to apply the law. Construction and interpretation come only after it has its claim during the proceedings before the SOLE and, instead, acted on it
been demonstrated that application is impossible or inadequate without only after the decision on the main case attained finality. This is a grave error
them." It is so even after the lapse of sixty years.[29] [Citations omitted] on the part of PAL.

Jurisprudence dictates that where the plaintiffs cause of action for damages The proper recourse for PAL should have been to assert its claim for damages
arose out of or was necessarily intertwined with an alleged unfair labor before the SOLE and, as aptly stated by the LA, to elevate the case to the CA
practice, the jurisdiction is exclusively with the labor tribunal. Likewise, when the SOLE failed to rule on the matter of damages. The 22 April 2008
where the damages separately claimed by the employer were allegedly LA decision, therefore, deserves reinstatement insofar as it dismissed PAL's
incurred as a consequence of strike or picketing of the union, such complaint 22 April 2003 complaint for lack of jurisdiction for the reason that the SOLE
for damages is deeply rooted in the labor dispute between the parties and has exclusive jurisdiction over the same. Thus, the Court quotes with
within the exclusive jurisdiction of the labor arbiter. Consequently, the same approval the following pronouncements by the LA:
should be dismissed by ordinary courts for lack of jurisdiction.[30]
The respondents maintain that the complainant simply slept on its rights
From the foregoing, it is clear that the regular courts do not have jurisdiction when it failed to elevate the matter of damages to the Court of Appeals. In
over PAL's claim of damages, the same being intertwined with its labor this regard, we find the argument of the respondents availing considering that
dispute with the respondents over which the SOLE had assumed jurisdiction. upon the assumption of jurisdiction of the Secretary of Labor over the labor
It is erroneous, therefore, for the CA to even suggest that PAL's complaint disputes at PAL, all other issues had been subsumed therein including the
should have been ventilated before the trial court. claim for damages arising from the strike. This is clear from the language of
Article 263(g) of the Labor Code granting the Secretary to order the
A separate complaint for damages "dismissal or loss of employment status or payment by the locking-out
runs counter to the rule against employer of back wages, damages and other affirmative relief even criminal
split jurisdiction. prosecution against either or both."
xxx

There is no quarrel regarding the jurisdiction of labor arbiters to rule on the


legality of strikes and lock-outs under Article 217(a)(4) but this refers to
strikes or lock-outs in establishments that are not indispensable to national
interest. However, if in his opinion the dispute affects industries imbued with
national interest, the Secretary of Labor who has the authority, may assume
jurisdiction over the dispute and may opt to hear the same until its final
disposition as is obtaining at bar, or to certify the same for compulsory
arbitration to the NLRC, where it is the Commission that will hear and
dispose of the certified cases under Rule VIII of the Revised Rules of the
NLRC. Even in voluntary arbitration, should the disputants agree to submit
the dispute to voluntary arbitration, the Voluntary Arbitrator is not precluded
from awarding damages.

As the issue on the illegality of the strikes of June 5, 1998 has already been
passed upon by the Secretary of Labor when he assumed jurisdiction to the
exclusion of all others, all incidents arising from the main issue of the legality
of the strike are presumed to have been ruled upon because they are deemed
subsumed by the assumption by the Secretary of Labor.[35]

In sum, the Court finds meritorious PAL's claim that the CA erred in its
decision. Indeed, the CA erred when it ruled that regular courts have
jurisdiction to entertain claims for damages arising from strike as the same
violates the proscription against splitting of jurisdiction. The Court, however,
also finds that the LA was already divested of its jurisdiction to entertain
PAL's claim for damages as such issue was deemed included in the issue of
legality of strike of which the SOLE had assumed jurisdiction, pursuant to
the rule against splitting of jurisdiction. Unfortunately, for PAL's failure to
raise the claim during the pendency of the illegal strike case before the SOLE,
the same is deemed waived.

WHEREFORE, the 26 August 2011 Decision and 5 January 2012 Resolution


of the Court of Appeals (CA) in CA-G.R. SP No. 113985 are SET ASIDE.
The 22 April 2008 Decision of the Labor Arbiter is REINSTATED insofar
as it dismissed the 22 April 2003 Complaint filed by Philippine Airlines, Inc.
in NLRC NCR No. 04-04906-03 for lack of jurisdiction.

SO ORDERED.
PHILIPPINE TRANSMARINE CARRIERS, INC. vs. LEGASPI
G.R. No. 202791 | 2013-06-10 4. That complainant’s counsel was amenable to the arrangement and
MENDOZA, J.: accepted the offer. NOW THEREFORE complainant and his counsel hereby
acknowledge RECEIPT of the sum of EIGHTY-ONE THOUSAND THREE
This is a petition for review on certiorari under Rule 45 of the Rules of Court HUNDRED TWENTY AND 0/100 (US$81,320.00) covered by CITIBANK
assailing the January 5, 2012 Resolution 1 and July 20, 20 I 2 Resolution2 of CHECK with No. 1000001161 dated October 21, 2010 payable to the order
the Court of Appeals (CA), in CA-G.R. SP No. I I 6686, which denied the of LEANDRO V. LEGASPI and UNDERTAKES to RETURN the entire
petitioner's motion to amend the dispositive portion of the June 29, 201 I CA amount to respondent PHILIPPINE TRANSMARINE CARRIERS,INC. in
Decision. the event that the Petition for Certiorari is granted without prejudice to
complainant’s right to appeal. Such undertaking shall be ENFORCEABLE
The Factual and Procedural Antecedents by mere motion before this Honorable office without need of separate
action.5 [Emphases and underscoring supplied]
Respondent Leandro Legaspi (respondent) was employed as Utility Pastry
on board the vessel “Azamara Journey” under the employment of petitioner On November 8, 2010, petitioner timely filed a petition for certiorari with the
Philippine Transmarine Carriers, Inc. (petitioner). Respondent’s employment CA.6
was covered by a Collective Bargaining Agreement (CBA) wherein it was
agreed that the company shall pay a maximum disability compensation of up In the meantime, on March 2, 2011, the LA issued a writ of execution which
to US$60,000.00 only. noted petitioner’s payment of the amount of US$81,320.00. On March 16,
2011, in compliance with the said writ, petitioner tendered to the NLRC
While on board the vessel, respondent suffered “Cardiac Arrest S/P ICD Cashier the additional amounts of US$8,132.00 as attorney’s fees and
Insertation.” He was checked by the ship’s doctor and was prescribed P3,042.95 as execution fee. In its Order, dated March 31, 2011, the LA
medications. On November 14, 2008, respondent was repatriated to receive ordered the release of the aforementioned amounts to respondent.
further medical treatment and examination. On May 23, 2009, the
companydesignated physician assessed his condition to be Disability Grade The CA’s Ruling
2.
Unaware of a) the September 5, 2010 entry of judgment of the NLRC, b) the
Not satisfied, respondent filed a complaint for full and permanent disability October 22, 2010 payment of US$81,320.00, and c) the writ of execution
compensation against petitioner before the Labor Arbiter (LA). issued by the LA, the CA rendered its Decision, dated June 29, 2011. The
CA partially granted the petition for certiorari and modified the assailed
The Labor Arbiter’s Ruling resolutions of the NLRC, awarding only US$60,000.00 pursuant to the CBA
between Celebrity Cruise Lines and Federazione Italianaa Transporti CISL.
In its January 25, 2010 Decision,3 the LA ruled in favor of respondent, the
dispositive portion of which reads: Petitioner then filed its Manifestation with Motion to Amend the Dispositive
Portion, submitting to the CA the writ of execution issued by the LA in
WHEREFORE, respondents (now petitioner) are hereby ordered to pay support of its motion. Petitioner contended that since it had already paid the
complainant jointly and severally, the following: total amount of US$89,452.00, it was entitled to the return of the excess
payment in the amount of US$29,452.00.
1. US$80,000.00 or its peso equivalent at the time of payment as permanent
disability compensation; In its assailed January 5, 2012 Resolution, the CA denied the motion and
ruled that the petition should have been dismissed for being moot and
2. US$1,320.00 or its peso equivalent as sick wages; academic not only because the assailed decision of the NLRC had become
final and executory on September 5, 2010, but also because the said judgment
3. Attorney’s fees equivalent to 10% of the total award. had been satisfied on October 22, 2010, even before the filing of the petition
for certiorari on November 8, 2010. In so ruling, the CA cited
SO ORDERED. the pronouncement in Career Philippines Ship Management v. Geronimo
Madjus7 where it was stated that the satisfaction of the monetary award
Notably, the LA awarded US$80,000.00 based on the ITF Cruise Ship Model rendered the petition for certiorari moot.
Agreement for Catering Personnel, not on the CBA.
Petitioner filed a motion for reconsideration but it was denied by the CA in
Not satisfied, petitioner appealed the LA decision before the National Labor its assailed July 20, 2012 Resolution.
Relations Commission (NLRC).
Hence, this petition.
The NLRC’s Ruling
ISSUES
In its May 28, 2010 Decision, the NLRC affirmed the decision of the LA.
Petitioner timely filed its motion for reconsideration but it was denied by the I. WHETHER THE COURT OF APPEALS COMMITTED SERIOUS
NLRC in its July 30, 2010 Resolution. On September 5, 2010, the NLRC REVERSIBLE ERROR OF LAW IN RULING THAT PETITIONER IS
issued the Entry of Judgment stating that its resolution affirming the LA ESTOPPED IN COLLECTING THE EXCESS PAYMENT IT MADE TO
decision had become final and executory. THE RESPONDENT NOTWITHSTANDING THE RECEIPT OF
JUDGMENT AWARD SIGNED BY THE RESPONDENT
On October 22, 2010, during the hearing on the motion for execution before
the NLRC, petitioner agreed to pay respondent US$81,320.00. The terms and II. WHETHER THE COURT OF APPEALS COMMITTED SERIOUS
conditions of said payment were embodied in the Receipt of Judgment REVERSIBLE ERROR IN INVOKING THE RULING OF CAREER V.
Award with Undertaking,4 wherein respondent acknowledged receipt of the MADJUS
said amount and undertook to return it to petitioner in the event the latter’s
petition for certiorari would be granted, without prejudice to respondent’s Petitioner argues that it clearly filed its petition for certiorari within the 60-
right to appeal. It was also agreed upon that the remaining balance would be day reglementary period and, thus, the NLRC resolutions could not have
given on the next scheduled conference. Pertinent portions of the said attained finality. Citing Delima v. Gois,8 petitioner avers that the NLRC
undertaking provide: cannot declare that a decision has become final and executory because the
period to file the petition has not yet expired. Petitioner, thus, contends that
xxxx the finality of the NLRC judgment did not render the petition moot and
academic because such is null and void ab initio.
3. That counsel (of the petitioner) manifested their willingness to tender the
judgment award without prejudice to the respondent’s (now petitioner) right Petitioner also argues that the Receipt of the Judgment Award with
to file a Petition for Certiorari and provided, complainant (now respondent) Undertaking, which was never refuted by respondent, clearly stated that the
undertakes to return the full amount without need of demand or a separate payment of the judgment award was without prejudice to its right to file a
action in the event that the Petition for Certiorari is granted; petition for certiorari with the CA. Petitioner asserts that the case relied upon
by the CA, Career Philippines, is not applicable as it is not on all fours with
this case. Instead, it asserts that the applicable case should be Leonis Stated differently, the Court ruled against the employer because the
Navigation Co., Inc. v. Villamater,9 where it was held that the satisfaction of conditional satisfaction of judgment signed by the parties was highly
the monetary award by the employer does not render the petition for certiorari prejudicial to the employee. The agreement stated that the payment of the
moot before the CA. monetary award was without prejudice to the right of the employer to file a
petition for certiorari and appeal, while the employee agreed that she would
On the other hand, respondent reiterates the CA ruling, asserting that the no longer file any complaint or prosecute any suit of action against the
voluntary satisfaction by petitioner of the full judgment award rendered the employer after receiving the payment.
case moot, and insists that it was a clear indication that it had already been
persuaded by the judiciousness and merits of the award for disability In contrast, in Leonis Navigation, after the NLRC resolution awarding
compensation. He also avers that this petition is merely pro-forma as it is a disability benefits became final and executory, the employer paid the
reiteration of petitioner’s previous issues and arguments already resolved by monetary award to the employee. The CA dismissed the employer’s petition
the CA. for certiorari, ruling that the final and executory decisions or resolutions of
the NLRC rendered appeals to superior courts moot and academic. This
The Court’s Ruling Court disagreed with the CA and held that final and executed decisions of
the NLRC did not prevent the CA from reviewing the same under Rule 65 of
Petition for Certiorari, Not Moot the Rules of Court. It was further ruled that the employee was estopped from
claiming that the case was closed and terminated, considering that the
Section 14, Rule VII of the 2011 NLRC Rules of Procedure provides that employee’s Acknowledgment Receipt stated that such was without prejudice
decisions, resolutions or orders of the NLRC shall become final and to the final outcome of the petition for certiorari pending before the CA.
executory after ten (10) calendar days from receipt thereof by the parties, and
entry of judgment shall be made upon the expiration of the said period.10In In the present case, the Receipt of the Judgment Award with Undertaking
St. Martin Funeral Home v. NLRC, was fair to both the employer and the employee. As in Leonis Navigation,
the said agreement stipulated that respondent should return the amount to
11 however, it was ruled that judicial review of decisions of the NLRC may petitioner if the petition for certiorari would be granted but without prejudice
be sought via a petition for certiorari before the CA under Rule 65 of the to respondent’s right to appeal. The agreement, thus, provided available
Rules of Court; and under Section 4 thereof, petitioners are allowed sixty remedies to both parties.
(60) days from notice of the assailed order or resolution within which to file
the petition. Hence, in cases where a petition for certiorari is filed after the It is clear that petitioner paid respondent subject to the terms and conditions
expiration of the 10-day period under the 2011 NLRC Rules of Procedure stated in the Receipt of the Judgment Award with Undertaking.17 Both
but within the 60-day period under Rule 65 of the Rules of Court, the CA can parties signed the agreement. Respondent neither refuted the agreement nor
grant the petition and modify, nullify and reverse a decision or resolution of claimed that he was forced to sign it against his will. Therefore, the petition
the NLRC. for certiorari was not rendered moot despite petitioner’s satisfaction of the
judgment award, as the respondent had obliged himself to return the payment
Accordingly, in this case, although the petition for certiorari was not filed if the petition would be granted.
within the 10-day period, petitioner timely filed it before the CA within the
60-day reglementary period under Rule 65. It has, thus, been held that the Return of Excess Payment
CA’s review of the decisions or resolutions of the NLRC under Rule 65,
particularly those which have already been executed, does not affect their As the agreement was voluntarily entered into and represented a reasonable
statutory finality, considering that Section 4,12 Rule XI of the 2011 NLRC settlement, it is binding on the parties and may not later be disowned simply
Rules of Procedure, provides that a petition for certiorari filed with the CA because of a change of mind.18 Respondent agreed to the stipulation that he
shall not stay the execution of the assailed decision unless a restraining order would return the amount paid to him in the event that the petition for
is issued. In Leonis Navigation, it was further written: certiorari would be granted. Since the petition was indeed granted by the CA,
albeit partially, respondent must comply with the condition to return the
The CA, therefore, could grant the petition for certiorari if it finds that the excess amount.
NLRC, in its assailed decision or resolution, committed grave abuse of
discretion by capriciously, whimsically, or arbitrarily disregarding evidence The Court finds that the Receipt of the Judgment Award with Undertaking
that is material to or decisive of the controversy; and it cannot make this was a fair and binding agreement. It was executed by the parties subject to
determination without looking into the evidence of the parties. Necessarily, outcome of the petition. To allow now respondent to retain the excess money
the appellate court can only evaluate the materiality or significance of the judgment would amount to his unjust enrichment to the prejudice of
evidence, which is alleged to have been capriciously, whimsically, or petitioner.
arbitrarily disregarded by the NLRC, in relation to all other evidence on
record.13 Notably, if the CA grants the petition and nullifies the decision or Unjust enrichment is a term used to depict result or effect of failure to make
resolution of the NLRC on the ground of grave abuse of discretion amounting remuneration of or for property or benefits received under circumstances that
to excess or lack of jurisdiction, the decision or resolution of the NLRC is, in give rise to legal or equitable obligation to account for them. To be entitled
contemplation of law, null and void ab initio; hence, the decision or to remuneration, one must confer benefit by mistake,fraud, coercion, or
resolution never became final and executory.14 request. Unjust enrichment is not itself a theory of reconveyance. Rather, it
is a prerequisite for the enforcement of the doctrine of restitution.19 There is
Career Philippines not applicable unjust enrichment when:

In Career Philippines, believing that the execution of the LA Decision was 1. A person is unjustly benefited; and
imminent after its petition for injunctive relief was denied, the employer filed
before the LA a pleading embodying a conditional satisfaction of judgment 2. Such benefit is derived at the expense of or with damages to another.20
before the CA and, accordingly, paid the employee the monetary award in
the LA decision. In the said pleading, the employer stated that the conditional In the case at bench, petitioner paid respondent US$81,320.00 in the pre-
satisfaction of the judgment award was without prejudice to its pending execution conference plus attorney’s fees of US$8,132.00 pursuant to the
appeal before the CA and that it was being made only to prevent the imminent writ of execution. The June 29, 2011 CA Decision, however, modified the
execution.15 final resolution of the NLRC and awarded only US$60,000.00 to respondent.
If allowed to return the excess, the respondent would have been unjustly
The CA later dismissed the employer’s petition for being moot and academic, benefited to the prejudice and expense of petitioner.
noting that the decision of the LA had attained finality with the satisfaction
of the judgment award. This Court affirmed the ruling of the CA, interpreting Petitioner's claim of excess payment is further buttressed by, and in 1 line
the “conditional settlement” to be tantamount to an amicable settlement of with, Section 14, Rule XI of the 20 II NLRC Rules of Procedure which
the case resulting in the mootness of the petition for certiorari, considering provides:
(i) that the employee could no longer pursue other claims,16 and (ii) that the
employer could not have been compelled to immediately pay because it had EFFECT OF REVERSAL OF EXECUTED JUDGMENT. -Where the
filed an appeal bond to ensure payment to the employee. executed judgment is totally or partially reversed or annulled by the Court of
Appeals or the Supreme Court, the Labor Arbiter shall, on motion, issue such
orders of restitution of the executed award, except wages paid during
reinstatement pending appeal. [Emphases supplied]

Although the Court has, more often than not, been inclined towards the plight
of the workers and has upheld their cause in their conflicts with the
employers, such inclination has not blinded it to the rule that justice is in
every case for the deserving, to be dispensed in the light of the established
facts and applicable law and doctrine.21

WHEREFORE, the petition is GRANTED. The Court of Appeals


Resolutions, dated January 5, 2012 and July 20, 2012, are hereby
REVERSED and SET ASIDE. Respondent Leandro Legaspi is ORDERED
to return the excess amount of payment in the sum of US$29,452.00 to
petitioner Philippine Transmarine Carriers, Inc. The amount shall earn
interest at the rate of 12% per annum from the finality of this judgment.

SO ORDERED.
ADTEL, INC. VS. VALDEZ 1. P283,000.00 - representing her separation pay for her almost ten years of
G.R. No. 189942 | 2017-08-09 service to the company;
CARPIO, J.:
2. P684,600.58 - representing her backwages from May 29, 2006 up to the
The Case date of this Decision;

Before the Court is a petition for review on certiorari[1] assailing the 28 May Plus ten percent (10%) of the total monetary awards, as and for attorney's
2009 Resolution[2] and the 8 October 2009 Resolution[3] of the Court of fees.
Appeals (CA) in CA-G.R. SP No. 108169.
Other claims and charges are dismissed for lack of merit.
The Facts
SO ORDERED.[16]
Adtel, Inc. (Adtel) is a domestic corporation engaged in the distribution of
telephone units, gadgets, equipment, and allied products. On 9 September Adtel filed a Motion for Reconsideration which was denied by the NLRC on
1996, Adtel hired Marijoy A. Valdez (respondent) to work as an accountant 24 December 2008. Adtel received the NLRC Resolution on 5 February
for the company. Adtel promoted respondent as the company's purchasing 2009. On 7 April 2009, the last day for filing its petition for certiorari with
and logistics supervisor.[4] Adtel then entered into a dealership agreement the CA, Adtel filed a motion for extension of time with the CA. On 22 April
with respondent's husband, Angel Valdez (Mr. Valdez), to distribute Adtel's 2009, fifteen (15) days after the last day for filing or the 75th day, Adtel filed
wideband VHF-UHF television antennas. The dealership agreement was for its petition for certiorari with the CA.[17]
twelve (12) months and the agreement was extended for another three (3)
months.[5] On 3 February 2006, Mr. Valdez filed a civil case against Adtel The Decision of the CA
for specific performance and damages for the execution of the terms of the
dealership agreement.[6] On 10 May 2006, Mr. Valdez also instituted a On 28 May 2009, the CA denied the motion for extension and dismissed
criminal complaint for libel against Adtel's chairman, president, and Adtel's petition for certiorari for being filed beyond the reglementary period.
officers.[7] The CA ruled that Adtel had until 7 April 2009 to file its petition for
certiorari. Instead of filing the petition for certiorari, Adtel filed a motion for
On 22 May 2006, Adtel issued a memorandum[8] directing respondent to extension of time on 7 April 2009 and subsequently filed its petition for
show cause in writing why she should not be terminated for conflict of certiorari on 22 April 2009, the last day of the extended period prayed for by
interest and/or serious breach of trust and confidence.[9] The memorandum Adtel. The CA held that the reglementary period to file a petition for
stated that the filing of cases by respondent's husband created a conflict of certiorari can no.longer be extended pursuant to A.M. No. 07-7-12-SC which
interest since respondent had access to vital information that can be used amended Section 4, Rule 65 of the Rules of Court.[18]
against Adtel.[10] Respondent was placed under preventive suspension by
Adtel. On 23 May 2006, respondent denied the charges of Adtel. Respondent The dispositive portion of the CA's Resolution states:
contended that the cases had nothing to do with her being an employee of
Adtel and had not affected her performance in the company.[11] WHEREFORE, the Motion is DENIED. Instead, the petition is DISMISSED
for being filed beyond the reglementary period.
On 29 May 2006, Adtel terminated respondent from the company.
Respondent filed a complaint for illegal dismissal with the Labor Arbiter. In SO ORDERED.[19]
her Position Paper,[12] respondent alleged that she did not violate any
company rule or policy; neither was she guilty of fraud, nor willful breach of Adtel filed a motion for reconsideration which was denied on 8 October
trust. Respondent contended that she was illegally dismissed without just 2009.[20]
cause and was entitled to separation pay, backwages, and damages.
The Issues
The Decision of the Labor Arbiter
Adtel presented the following issues in this petition:
In a Decision[13] dated 24 May 2007, the Labor Arbiter dismissed
respondent's complaint for illegal dismissal. The Labor Arbiter found that A. The Court of Appeals committed a reversible error in denying the
there existed a conflict of interest between respondent and Adtel. The Labor petitioners' motion for reconsideration and in dismissing the petition for
Arbiter ruled that respondent was not an ordinary rank-and-file employee but certiorari on the sole basis of technicality.
a managerial employee with a fiduciary duty to protect the interest of Adtel.
The Labor Arbiter held that the civil and criminal cases initiated by B. Technicalities should give way to a judgment on the merits considering
respondent's husband indubitably created a conflict of interest that was a just that the Labor Arbiter justly and correctly ruled that the complaint for illegal
cause for her dismissal by Adtel. dismissal against petitioner was baseless and unmeritorious only to be later
reversed by the NLRC upon respondent's appeal.[21]
The dispositive portion of the Labor Arbiter's decision reads:
The Decision of this Court
WHEREFORE, premises considered, judgment is hereby rendered
DISMISSING the instant complaint for utter lack of merit. We deny the petition.

SO ORDERED.[14] A.M. No. 07-7-12-SC which amended Section 4, Rule 65 of the Rules of
Court states:
The Decision of the National Labor Relations Commission
Sec. 4. When and where to file the petition. — The petition shall be filed not
In a Decision[15] dated 21 May 2008, the National Labor Relations later than sixty (60) days from notice of the judgment, order or resolution. In
Commission (NLRC) reversed the decision of the Labor Arbiter. The NLRC case a motion for reconsideration or new trial is timely filed, whether such
ruled that Adtel illegally dismissed respondent. The NLRC held that Adtel motion is required or not, the petition shall be filed not later than sixty (60)
failed to substantially prove the existence of an act or omission personally days counted from the notice of the denial of the motion.
attributable to the respondent to serve as a just cause to terminate her
employment. If the petition relates to an act or an omission of a municipal trial court or of
a corporation, a board, an officer or a person, it shall be filed with the
The dispositive portion of the NLRC's decision states: Regional Trial Court exercising jurisdiction over the territorial area as
defined by the Supreme Court. It may also be filed with the Court of Appeals
WHEREFORE, the appeal is GRANTED and the assailed Decision is hereby or with the Sandiganbayan, whether or not the same is in aid of the court's
REVERSED and SET ASIDE. A new one is hereby rendered ordering the appellate jurisdiction. If the petition involves an act or an omission of a quasi-
respondent company to pay to the complainant the following amounts: judicial agency, unless otherwise provided by law or these rules, the petition
shall be filed with and be cognizable only by the Court of Appeals.
In election cases involving an act or an omission of a municipal or a regional Petitioner prays for such other relief which may be deemed just and equitable
trial court, the petition shall be filed exclusively with the Commission on under the circumstances.[29] (Boldfacing and underscoring supplied)
Elections, in aid of its appellate jurisdiction.
In Yutingco v. Court of Appeals,[30] this Court held that the circumstance
A.M. No. 07-7-12-SC states that in cases where a motion for reconsideration of heavy workload alone, absent a compelling or special reason, is not a
was timely filed, the filing of a petition for certiorari questioning the sufficient justification to allow an extension of the 60-day period to file a
resolution denying the motion for reconsideration must be made not later petition for certiorari, to wit:
than sixty (60) days from the notice of the denial of the motion. In Laguna
Metts Corporation v. Court of Appeals,[22] this Court held that following Heavy workload, which is relative and often self serving, ought to be coupled
A.M. No. 07-7-12-SC, petitions for certiorari must be filed strictly within 60 with more compelling reasons such as illness of counsel or other emergencies
days from the notice of judgment or from the order denying a motion for that could be substantiated by affidavits of merit. Standing alone, heavy
reconsideration. In Laguna Metts Corporation, this Court stated the rationale workload is not sufficient reason to deviate from the 60-day rule. Thus, we
for the strict observance of the 60-day period to file a petition for certiorari, are constrained to state that the Court of Appeals did not err in dismissing the
to wit: petition for having been filed late.[31]
The 60-day period is deemed reasonable and sufficient time for a party to
mull over and to prepare a petition asserting grave abuse of discretion by a In Thenamaris Philippines, Inc. v. Court of Appeals,[32] this Court held that
lower court. The period was specifically set to avoid any unreasonable delay the heavy workload of counsel is hardly a compelling or meritorious reason
that would violate the constitutional rights of the parties to a speedy for availing a motion for extension of time to file a petition for certiorari.
disposition of their case.[23] Similarly, in Mid-Islands Power, this Court ruled that the heavy workload
and the resignation of the lawyer handling the case are insufficient reasons
In Laguna Metts Corporation, this Court ruled that the 60-day period was to justify the relaxation of the procedural rules under Rule 65. In both
non-extendible and the CA no longer had the authority to grant the motion Thenamaris and Mid-Islands Power, this Court denied the motions for
for extension in view of A.M. No. 07-7-12-SC which amended Section 4 of extension of time to file a petition for certiorari and held that the heavy
Rule 65. workload of counsel was not a compelling reason contemplated by the Rules
of Court.
However, in Domdom v. Third and Fifth Divisions of the Sandiganbayan[24]
this Court held that the strict observance of the 60-day period to file a petition As previously stated in Labao,[33] there should be an effort on the part of the
for certiorari is not absolute. This Court ruled that absent any express party invoking liberality to advance a reasonable or meritorious explanation
prohibition under Rule 65, a motion for extension is still permitted, subject for his or her failure to comply with Rule 65. Accordingly, in the absence of
to the Court's sound discretion. Similarly, in Labao v. Flores,[25] this Court a more compelling reason cited in the motion for extension of time other than
recognized that the extension of the 60-day period may be granted by the the "undersigned counsel's heavy volume of work," the CA did not commit
Court in the presence of special or compelling circumstances provided that a reversible error in dismissing the petition for certiorari.
there should be an effort on the part of the party invoking liberality to
advance a reasonable or meritorious explanation for his or her failure to WHEREFORE, we DENY the petition. We AFFIRM the Resolutions of the
comply with the rules. Likewise, in Mid-Islands Power Generation v. Court Court of Appeals dated 28 May 2009 and 8 October 2009 in CA-G.R. SP No.
of Appeals[26] this Court held that a motion for extension was allowed in 108169.
petitions for certiorari under Rule 65 subject to the Court's sound discretion
and only under exceptional or meritorious cases. SO ORDERED.

The exception to the 60-day rule to file a petition for certiorari under Rule 65
was also applied by this Court in a more recent case in Republic of the
Philippines v. St. Vincent de Paul Colleges, Inc.,[27] to wit: "[u]nder
exceptional circumstances, however, and subject to the sound discretion of
the Court, [the] said period may be extended pursuant to [the] Domdom,
Labao and Mid-Islands Power cases."[28]

Therefore, the rule is that in filing petitions for certiorari under Rule 65, a
motion for extension is a prohibited pleading. However in exceptional or
meritorious cases, the Court may grant an extension anchored on special or
compelling reasons.

Adtel's motion for extension filed with the CA on 7 April 2009 reads:

MOTION FOR EXTENSION OF TIME TO FILE PETITION FOR


CERTIORARI

1. Petitioner's Petition for Certiorari was due for filing yesterday, 06 April
2009 or sixty (60) days from 05 February 2009, the date of receipt of the
Resolution dated 24 December 2008 issued by the National Labor Relations
Commission (NLRC). Considering that yesterday was a holiday, the petition
in effect is due today, 07 April 2009.

2. While a draft of the pleading had already been prepared, final revisions
have yet to be completed. However, due to the undersigned counsel's heavy
volume of work, petitioner is constrained to request for an additional period
of fifteen (15) days from today or up to 22 April 2009 within which to file
the Petition for Certiorari.

3. This motion is not intended to delay the proceedings but is prompted solely
by the above-stated reason.

PRAYER

WHEREFORE, petitioner respectfully prays for an extension of fifteen (15)


days from 07 April 2009 or up to 22 April 2009 within which to file its
Petition for Certiorari.
ABBOTT LABORATORIES PHILIPPINES, INC. vs. ABBOTT WHEREFORE, the appeal is GRANTED and the decision of the Regional
LABORATORIES EMPLOYEES UNION Director dated 21 June 1996 is hereby REVERSED. Abbott Laboratories
G.R. No.131374 | 2000-01-26 Employees Union shall remain in the roster of legitimate labor organizations,
DAVIDE, JR., C.J.: with all the rights, privileges and obligations appurtenant thereto.12 [Id.,
225.]
This special civil action for certiorari and mandamus assails the action of the
then Acting Secretary of Labor and Employment Cresenciano. B. Trajano It gave the following reasons to justify the reversal: ( 1) Article 234 of the
contained in its letter dated 19 September 1997,1 [Annex "1" of Petition; Labor Code does not require an applicant union to show proof of the
Rollo, 39; Original Record, vol. I (hereafter I OR), 344.] informing petitioner "desirability of more than one Ibargaining unit within an employer unit," and
Abbott Laboratories Philippines, Inc. (hereafter ABBOTT), thru its counsel the absence of such proof is not a ground for the cancellation of a union's
that the Office of the Secretary of Labor cannot act on ABBOTT's appeal registration pursuant to Article 239 of Book V, Rule II of the implementing
from the decision of 31 March 19972 [Annex "2" of Petition; Ibid., 40-50; rules of the Labor Code; (2) the issue pertaining to the appropriateness of a
Ibid., 235-225.] and the Order of 9 July 19973 [Annex "3" of Petition; Id, 51- bargaining unit cannot be raised in a cancellation proceeding but may be
53; Id, 337 -335.] of the Bureau of Labor Relations, for lack of appellate threshed out in the exclusion-inclusion process during a certification
jurisdiction. election; and (3) the "one-bargaining unit, one-employer unit policy" must
not be interpreted in a manner that shall derogate the right of the employees
ABBOTT is a corporation engaged in the manufacture and distribution of to self-organization and freedom of association as guaranteed by Article III,
pharmaceutical drugs. On 22 February 1996,4 [I OR, 56-51.] the Abbott Section 8 of the 1987 Constitution and Article II of the International Labor
Laboratories Employees Union (hereafter ALEU) represented by its Organization's Convention No.87.
president, Alvin B. Buerano, filed an application for union registration in the
Department of Labor and Employment. ALEU alleged in the application that Its motion to reconsider the 31 March 1997 decision of the Bureau of Labor
it is a labor organization with members consisting of 30 rank-and-file Relations having been denied for lack of merit in the Order13 [IOR, 337-
employees in the manufacturing unit of ABBOTT and that there was no 335.] of 9 July 1997, ABBOTT appealed to the Secretary of Labor and
certified bargaining agent in the unit it sought to represent, namely, the Employment. However, in its letter dated 19 September 1997,14 [Id., 344.]
manufacturing unit. addressed to ABBOTT's counsel, the Secretary of Labor and Employment
refused to act on ABBOTT's appeal on the ground that it has no jurisdiction
On 28 February 1996,5 [31 March 1997 Decision in BLR-A 10-25-96, Rollo, to review the decision of the Bureau of Labor Relations on iappeals in
40.] ALEU's application was approved by the Bureau of Labor Relations, cancellation cases emanating from the Regional Offices. The decision of the
which in due course issued Certificate of Registration No. NCR-UR-2-1638- Bureau of Labor Relations therein is final and executory under Section 4,
96. Consequently, ALEU became a legitimate labor organization. Rule III, Book V of the Rules and Regulations Implementing thc Labor Code,
as amended by Department Order No. 09, s. of 1997. Finally, the Secretary
On 2 April 1996,6 [Id, 69-59.] ABBOTT filed a petition for cancellation of stated:
the Certificate of Registration No. NCR-UR-2-1638-96 in the Regional
Office of the Bureau of Labor Relations. This case was docketed as Case No. It has always been the policy of this Office that pleadings denominated as
OD-M-9604-006. ABBOTT assailed the certificate of registration since appeal thereto over decisions of the BLR in cancellation cases coming from
ALEU's application was not signed by at least 20% of the total 286 rank-and- the Regional Offices are referred back to the BLR, so that the same may be
file employees of the entire employer unit; and that it omitted to submit treated as motions for reconsideration and disposed of accordingly. However,
copies of its books of account. since your office has already filed a motion for reconsideration with the BLR
which has been denied in its Order dated 09 July 1997, your recourse should
On 21 June 1996,7 [I OR, 131-123. Per Regional Director Romeo A. Young.] have been a special civil action for certiorari with the Supreme Court.
the Regional Director of the Bureau of Labor Relations decreed the
cancellation of ALEU's registration certificate No. NCR-UR-II-1585-95.8 In view of the foregoing, please be informed that the Office of the Secretary
[What was actually sought to be cancelled was Registration Certificate No. cannot act upon your Appeal, except to cause the BLR to include it in the
NCR-UR-2-1638-96. Apparently, the Regional Director in NCR-OD-M- records of the case.
9604-006 unwittingly erred in ordering the cancellation of Registration
Certificate No. NCR-UR-11-1585-95 since this refer to the certificate of Hence, this petition. ABBOTT premised its argument on the authority of the
registration ssued to ALEU on 4 November 1995 which later became the Secretary of Labor and Employment to review the decision of the Bureau of
subject matter of Case No. NCR-OD-M-9512-038 entitled "Ma. Luisa Labor Relations and at the same time raised the issue on the validity of
Mariazeta, et al. v. Abbott Laboratories (Phils.) Employees Union." The ALEU's certificate of registration.
Registration Certificate No. NCR-UR-11-1585-95 was cancelled in view of
the decision dated 28 February 1996 of the Bureau of Labor Relations, which We find no merit in this petition.
became final since the ALEU did not appeal therefrom. See Petition for
cancellation of union registration, I OR, 69-61.] In its decision, the Regional At the outset, it is wortl1y to note that the present petition assails only the
Director adopted the 13 June 19969 [Id., 120-110. Per Med-Arbiter Renato letter of the then Secretary of Labor & Employment refusing to take
D. Parungo.] findings and recommendations of the Med-Arbiter. It ruled that cognizance of ABBOTT's appeal for lack of appellate jurisdiction. Hence, in
the union has failed to sliow that the rank-and-file employees in the the resolution of the present petition, it is just appropriate to limit the issue
manufacturing unit of ABBOTT were bound by a common interest to justify on the power of the Secretary of Labor and Employment to review the
the formation of a bargaining unit separate from those belonging to the sales decisions of the Bureau of Labor Relations rendered in the exercise of its
and office staff units. There was, therefore, sufficient reason to assume that appellate jurisdiction over decisions of the Regional Director in cases
the entire membership of the rank-and-file consisting of 286 employees or involving cancellations of certificates of registration of labor unions. The
the "employer unit" make up the appropriate bargaining unit. However, it issue anent the validity of ALEU's certificate of registration is subject of the
was clear on the record that the union's application for registration was Bureau of Labor Relations decision dated 31 March 1997. However, said
supported by 30 signatures of its members or barely constituting 10% of the decision is not being assailed in the present petition; hence, we are not at
entire rank-and-file employees of ABBOTT. Thus the Regional Director liberty to review the same.
found that for ALEU's failure to satisfy the requirements of union registration
under Article 234 of the Labor Code; the cancellation of its certificate of Contrary to ABBOTT's contention, there has been no grave abuse of
registration was in order. discretion on the part of the Secretary of Labor and Employment. Its refusal
to take cognizance of ALEU's appeal from the decision of the Bureau of
Forthwith, on 19 August 11996,10 [I OR, 198-177.] ALEU appealed said Labor Relations is in accordance with the provisions of Rule VIII, Book V
cancellation to the Office of the Secretary of Labor and Employment, which of the Omnibus Rules Implementing the Labor Code as amended by
referred the same to the Director of the Bureau of Labor Relations. The said Department Order No. 09.15 [Department Order No. 09 Series of 1997,
appeal was docketed as Case No. BLR-A-10-25-96. which took effect on 21 June 1997, amends and renumbers numerous
provisions of Book Vof the .Rules Implementing the Labor Code.] The rule
On 31 March 1997,11 [Id., 235-225. Per Director Benedicto Ernesto R. governing petitions for cancellation of registration of any legitimate labor
Bitonio, Jr.] the Bureau of Labor Relations rendered judgment reversing the organization or worker association, as it now stands, provides:
21 June 1996 decision of the Regional Director, thus:
SECTION 1. Venue of Action --If the respondent to the petition is a
local/chapter, affiliate, or a workers' association with operations limited to Respondent Acting Labor Secretary's ruling --that the BLR's decision
one region, the petition shall be filed with the Regional Office having upholding the validity of respondent union's certificate of registration is final
jurisdiction over the place where the respondent principally operates. and inappealable --is thus in accordance with aforequoted Omnibus Rules
Petitions filed against federations, national or industry unions, trade union because the petition for cancellation of union registration was filed by
centers, or workers' associations operating in more than one regional petitioner with a Regional Office, specifically, with the Regional Office of
jurisdiction, shall be filed with the Bureau. the BLR, National Capital Region (vide pp.1-2, Annex 2, Petition). The
cancellation proceedings initiated by petitioner before the Regional Office is
SECTION 3. Cancellation of registration;. nature and grounds. -- Subject to covered by the first situation contemplated by Sections 7 to 9 of the Omnibus
the requirements of notice and due process, the registration of any legitimate Rules. Hence, an appeal from the decision of the Regional Office may be
labor organization or worker's association may be cancelled by the Bureau or brought to the BLR whose decision on the matter is final and inappealable.
the Regional Office upon the filing of an independent petition for
cancellation based on any of the following grounds: In the instant case, upon the cancellation of respondent union's registration
by the Regional Office, respondent union incorrectly appealed said decision
(a) Failure to comply with any of the requirements prescribed under Articles to the Office of the Secretary. Nevertheless, this situation was immediately
234, 237 and 238 of the Code; rectified when the Office of the Secretary motu proprio referred the appeal
to the BLR However, upon reversal by the BLR of the decision of the
(b) Violation of any of the provisions of Article 239 of the Code; Regional Office cancelling registration, petitioner should have immediately
elevated the BLR decision to the Supreme Court in a special civil action for
(b) Commission of any of the acts enumerated under Article 241 of the Code; certiorari under Rule 65 of the Rules of Court.
provided, that no petition for cancellation based on this ground may be
granted unless supported by at least thirty percent (30%) of all the members Under Sections 3 and 4, Rule VIII of Book V of the Rules and Regulations
of the respondent labor organization or workers' association. implementing the Labor Code, as amended by Department Order No. 09,
petitions for cancellation of union registration may be filed with a Regional
Section 4. Action on the petition; appeals -- The Regional or Bureau Director, office, or directly, with the Bureau of Labor Relations. Appeals from the
as the case may be, shall have thirty (30) days from submission of the case decision of a Regional Director may be filed with the BLR Director whose
for resolution within which to resolve the petition. The decision of the decision shall be final and executory. On the other hand, appeals from the
Regional or Bureau Director may be appealed to the Bureau or the Secretary, decisions of the BLR may be filed with the Secretary of Labor whose
as the case may be, within ten (10) days from receipt thereof by the aggrieved decision shall be final and executory .
party on the ground of grave abuse of discretion or any violation of these
Rules. Thus, under Sections 7 to 9 of the Omnibus Rules and under Sections 3 and
4 of the Implementing Rules (as amended by Department Order No. 09), the
The Bureau or the Secretary shall have fifteen ( 15) days from receipt of the finality of the BLR decision is dependent on whether or not the petition for
records of the case within which to decide the appeal. The decision of the cancellation was filed with the BLR directly. Under said Rules, if the petition
Bureau or the Secretary shall be final and executory. for cancellation is directly filed with the BLR, its decision cancelling union
registration is not yet final and executory as it may still be appealed to the
Clearly, the Secretary of Labor and Employment has no jurisdiction to Office of the Secretary. However, if the petition for cancellation was filed
entertain the appeal of ABBOTT. The appellate jurisdiction of the Secretary with the Regional Office, the decision of the BLR resolving an appeal of the
.of Labor and Employment is limited only to a review of cancellation decision of said Regional Office is final and executory.18 [Rollo, 144-146;
proceedings decided by the Bureau of Labor Relations in the exercise of its 279-281.]
exclusive and original jurisdiction. The Secretary of Labor and Employment
has no jurisdiction over decisions of the Bureau of Labor Relations rendered It is clear then that the Secretary of Labor and Employment did not commit
in the exercise of its appellate power to review the decision of the Regional grave abuse of discretion in not acting on ABBOTT's appeal. The decisions
Director in a petition to cancel the union's certificate of registration, said of the Bureau of Labor Relations on cases brought before it on appeal from
decisions being final and inappealable.16 [Even under the old rule, decisions the Regional Director are final and executory. Hence, the remedy of the
of the Bureau of Labor Relations and the Secretary are final and inappealable. aggrieved party is to seasonably avail of the special civil action of certiorari
Section 9, Rule II, Book V of the Omnibus Rule provides that: The labor under Rule 65 of the Rules of Court.19 [Bordeos, et al. v. NLRC, et al., 262
organization may, unless the law provides otherwise, within fifteen (15) SCRA 424 (1996); See also St. Martin Funeral Homes v. NLRC, et al., 295
calendar days from receipt of the decision cancelling or revoking its SCRA 494 (1998)]
certificate of registration, file an appeal to the Bureau, or in case of
cancellation by the Bureau, to the Secretary, on any of the following grounds: Even if we relaxed the rule and consider the present petition as a petition for
certiorari not only of the letter of the Secretary of Labor and Employment but
a) grave abuse of discretion; and also of the decision of the Bureau of the Labor Relations which overruled the
order of cancellation of ALEU's certificate of registration, the same would
b) gross incompetence. still be dismissable for being time-barred. Under Sec. 4 of Rule 65 of the
1997 Revised Rules of Court the special civil action for certiorari should be
The Bureau/Secretary shall have fifteen (15) calendar days from receipt of instituted within a period of sixty (60) days from notice of the judgment,
the records of the case within which to decide the appeal. The decision shall order or resolution sought to be assailed. ABBOTT received the decision of
be final and inappealable.] We sustain the analysis and interpretation of the the Bureau of Labor Relations on 14 April 1997 and the order denying its
OSG on this matter, to wit: motion for reconsideration of the said decision on 16 July 1997. The present
petition was only filed on 28 November 1997, after the laps of more than four
From the foregoing, the Office of the Secretary correctly maintained that it months. Thus, for failure to avail of the correct remd4y within the period
cannot take cognizance of petitioner's appeal from the decision of BLR provided by law, the decision of the Bureau of Labor Relations has become
Director Bitonio. Sections 7 to 917 [Now Rule VIII, Implementing Rules of final and executory.
Book V, specifically sections 1, 3 and 4.] (of the Implementing Rules of the
Labor Code) thus provide for two situations: WHEREFORE, the Petition is DENIED. The challenged order in BLR-A-
10-25-96 of the Secretary of Labor and Employment embodied in its 19
(1) The first situation involves a petition for cancellation of union registration September letter is hereby AFFIRMED.
which is filed with a Regional Office. A decision of a Regional Office
cancelling a union's certificate of registration may be appealed to the BLR SO ORDERED.
whose decision on the matter shall be final and inappealable.

(2) The second situation involves a petition for cancellation of certificate of


union registration which is filed directly with the BLR. A decision of the
BLR cancelling a union's certificate of registration may be appealed to the
Secretary of Labor whose decision on the matter shall be final and
inappealable.
GUAGUA NATIONAL COLLEGES VS. COURT OF APPEALS Shortly thereafter, and in order to save the depleting funds of the petitioner's
G.R. No. 188492 | 2018-08-28 Retirement Plan, its Board of Trustees approved the funding of the retirement
BERSAMIN, J.: program out of the 70% net incremental proceeds arising from the tuition fee
increases.[5] Respondents GNC-Faculty Labor Union and GNC Non-
This case focuses on the correct period for appealing the decision or award Teaching Maintenance Labor Union challenged the petitioner's unilateral
of the Voluntary Arbitrator or Panel of Arbitrators. The issue arises because decision by claiming that the increase violated Section 5(2) of R.A. No. 6728.
the decision or award of the Voluntary Arbitrator or Panel of Arbitrators is
appealable to the Court of Appeals (CA) by petition for review under Rule The parties referred the matter to voluntary arbitration after failing to settle
43 of the Rules of Court, which provides a period of 15 days from notice of the controversy by themselves.[6]
the decision or award within which to file the petition for review. On the
other hand, Article 262-A (now Article 276)[1] of the Labor Code sets 10 Decision of the Voluntary Arbitrator
days as the period within which the appeal is to be made.
After hearing the parties, Voluntary Arbitrator Froilan M. Bacungan
The Case rendered his decision dated June 16, 2008 in favor of GNC,[7] holding that
retirement benefits fell within the category of "other benefits" that could be
Petitioner Guagua National Colleges (GNC) hereby assails by petition for charged against the 70% net incremental proceeds pursuant to Section 5(2)
certiorari the resolution promulgated on December 15, 2008,[2] whereby the of R.A. No. 6728.
Court of Appeals (CA) denied its Motion to Dismiss filed vis-à-vis the
respondents' petition for certiorari in the following manner: After receiving a copy of the decision on June 16, 2008, the respondents filed
an Urgent Motion for Extensionpraying that the CA grant them an extension
This Court resolves: of 15 days from July 1, 2008, or until July 16, 2008, within which to file their
petition for review.[8]
1. x x x
Ruling of the CA
2. To Deny:
On July 2, 2008, the CA issued a resolution granting the Urgent Motion for
a) respondent's Motion to Dismiss dated 22 July 2008. While it is true that Extension.[9] The respondents filed the petition for review[10] on July 16,
Coca-Cola Bottlers Philippines, Inc., Sales Force Union-PTGWO-Balais vs. 2008.[11]
Coca-Cola Bottlers Philippines, Inc. held in part:
Subsequently, the petitioner filed its Motion to Dismiss,[12] asserting that
"x x x [U]nder Section 6, Rule VII of the same guidelines implementing the decision of the Voluntary Arbitrator had already become final and
Article 262-A of the Labor Code, this Decision, as a matter of course, would executory pursuant to Article 276 of the Labor Code and in accordance with
become final and executory after ten (10) calendar days from receipt of the ruling in Coca-Cola Bottlers Philippines, Inc. Sales Force Union-
copies of the decision by the parties x xx unless, in the meantime, a motion PTGWO-Balais v. Coca-Cola Bottlers Philippines, Inc.[13]
for reconsideration or a petition for review to the Court of Appeals under
Rule 43 of the Rules of Court is filed within the same 10-day period. x xx;", The CA acted on the Motion to Dismiss on December 15, 2008 through the
We, more importantly recognize the pronouncement of the Supreme Court in now assailed resolution denying theMotion to Dismiss.[14]
Manila Midtown vs. Borromeowhich reads in part:
The petitioner sought reconsideration,[15] but the CA denied the motion for
"Upon receipt of a copy of the Voluntary Arbitrator's Decision, petitioner reconsideration on January 30, 2009.[16]
should have filed with the Court of Appeals, within the 15-day reglementary
period, a petition for review x xx" Hence, the petitioner instituted its petition for certiorari.

Coca-Cola Bottlers is not in direct conflict with Manila Midtown as there is Issue
no categorical ruling in the former that the petition for review under Rule 43
of the Rules of Court assailing the decision of the Voluntary Arbitrator The petitioner submits the lone issue that—
should be filed within ten (10) days from receipt thereof and not the
customary reglementary period of fifteen (15) days. Likewise, Leyte IV THE COURT OF APPEALS, WITH ALL DUE RESPECT, IS ACTING
Electric Cooperative, Inc. vs. LEYECO IV Employees Unio-ALU, WITHOUT OR IN EXCESS OF ITS JURISDICTION IN CA-G.R. SP NO.
reiterating the landmark Case of Luzon Development Bank vs. Association 104109 CONSIDERING THAT THE DECISION OF THE VOLUNTARY
of Luzon Development Bank Employees, declared that the proper remedy ARBITRATOR IN AC-025-RB3-04-01-03-2007, FOLLOWING RULE
from the award of a voluntary arbitrator is a petition for review to the CA, [276] OF THE LABOR CODE AND THE DECISION OF THE
following Revised Administrative Circular No. 1-95, which in turn provides HONORABLE COURT IN COCA-COLA BOTTLERS PHILIPPINES,
for a reglementary period of fifteen (15) days within which to appeal. INC. SALES FORCE UNION-PTGWO- BALAIS v. COCA-COLA
BOTTLERS PHILIPPINES, INC. XXXX, HAD ALREADY BECOME
Keeping in mind Article 4 of the Labor Code which mandates that all doubts FINAL AND EXECUTORY, HENCE UNCHALLENGEABLE SINCE
in the implementation and interpretation of its provisions, including its THE "URGENT MOTION FOR EXTENSION" DATED 30 JUNE 2008
implementing rules and regulations, should be resolved in favor of labor and AND 16 JULY 2008 RESPECTIVELY, OR TEN (10) DAYS AFTER THE
considering that technicalities are not supposed to stand in the way of UNIONS AND THEIR COUNSEL OF RECORD WERE PERSONALLY
equitably and completely resolving the rights and obligations of labor arid SERVED THE VOLUNTARY ARBITRATOR'S DECISION ON 16 JUNE
capital, We rule that the Petition for Review was seasonably filed. Moreso 2008.[17]
that We have already granted petitioners' Urgent Motion for Extension.
The petitioner argues that the CA went beyond its jurisdiction when it denied
3. x x x the Motion to Dismiss despite the finality of the decision of the Voluntary
Arbitrator pursuant to Article 276 of the Labor Code; that following the
SO ORDERED. pronouncement in Coca-Cola Bottlers Philippines, Inc. Sales Force Union-
PTGWO-Balais v. Coca-Cola Bottlers Philippines, Inc.[18] the CA was no
Antecedents longer authorized to exercise its appellate jurisdiction;[19] that the CA's
reliance on the rulings in Manila Midtown Hotel v. Borromeo[20] and Leyte
Under Section 5(2)[3] of Republic Act No. 6728 (Government Assistance To IV Electric Cooperative, Inc. v. Leyeco IV Employees Union-ALU[21] was
Students and Teachers In Private Education Act), 70% of the increase in misplaced because said rulings did not define the reglementary period to
tuition fees shall go to the payment of salaries, wages, allowances and other appeal the decision or award of the Voluntary Arbitrator;[22] and that the CA
benefits of the teaching and non-teaching personnel. Pursuant to this misapplied the rule on equity in the absence of strong or compelling reasons
provision, the petitioner imposed a 7% increase of its tuition fees for school to suspend the rules of procedure.[23]
year 2006-2007.[4]
The petitioner emphasizes the need to harmonize Rule 43 of the Rules of
Court with Article 276 of the Labor Code in view of their conflicting
provisions on the period for the appeal from the decision of the Voluntary however, the Court pronounced in Oceanic Bic Division (FFW) v.
Arbitrator. It maintains that unless Congress amends Article 276 of the Labor Romero[29]that the decisions or awards of the Voluntary Arbitrators
Code, the reglementary period within which to appeal the decision or award involving interpretations of law were within the scope of the Court's power
of the Voluntary Arbitrator is 10 days following the ruling in Coca-Cola of review. The Court explained:
Bottlers Philippines, Inc. Sales Force Union-PTGWO-Balais v. Coca--Cola
Bottlers Philippines, Inc., instead of 15 days under Rule 43 of the Rules of x x x x We agree with the petitioner that the decisions of voluntary arbitrators
Court. must be given the highest respect and as a general rule must be accorded a
certain measure of finality. This is especially true where the arbitrator chosen
In contrast, the respondents insist that they have a meritorious case because by the parties [enjoys] the first rate credentials of Professor Flerida Ruth
the controversy involves the interpretation of Section 5(2) of R.A. No. 6728 Pineda Romero, Director of the U.P. Law Center and an academician of
on the disposition of the tuition fee increase;[24] that the CA did not abuse unquestioned expertise in the field of Labor Law. It is not correct, however,
its discretion given the rule on the liberal application of rules of procedure to that this respect precludes the exercise of judicial review over their decisions.
achieve substantial justice, and the policy on the liberal construction of laws Article 262 of the Labor Code making voluntary arbitration awards final,
in favor of labor;[25] that a long line of jurisprudence[26] set the remedy of inappealable, and executory except where the money claims exceed
appeal under Rule 43 of the Rules of Court as applicable in challenging the P100,000.00 or 40% of paid-up capital of the employer or where there is
decisions or awards of the Voluntary Arbitrator. abuse of discretion or gross incompetence refers to appeals to the National
Labor Relations Commission and not to judicial review.
Did the CA gravely abuse its discretion in denying the petitioner's Motion to
Dismiss despite the finality of the decision of the Voluntary Arbitrator Inspite of statutory provisions making '"final" the decisions of certain
pursuant to Article 276 of the Labor Code? administrative agencies, we have taken cognizance of petitions questioning
these decisions where want of jurisdiction, grave abuse of discretion,
Ruling of the Court violation of due process, denial of substantial justice, or erroneous
interpretation of the law were brought to our attention. There is no provision
We dismiss the petition for certiorari. for appeal in the statute creating the Sandiganbayan but this has not precluded
us from examining decisions of this special court brought to us in proper
I petitions. Thus, we have ruled:
The petition for review shall be filed within 15 days
pursuant to Section 4, Rules 43 of the Rules of Court; "Yanglay raised a jurisdictional question which was not brought up by
the 10-day period under Article 276 of the Labor Code respondent public officials. He contends that this Court has no jurisdiction to
refers to the filing of a motion for reconsideration review the decisions of the NLRC and the Secretary of Labor 'under the
vis-à-vis the Voluntary Arbitrator's decision or award principle of separation of powers' and that judicial review is not provided for
in Presidential Decree No. 21.
In resolving whether or not the CA committed grave abuse of discretion, the
Court has first to determine which between the two periods found in Article "That contention is a flagrant error, it is generally understood that as to
276 of the Labor Code and Section 4 of Rule 43 of the Rules of Courtgoverns administrative agencies exercising quasi- judicial or legislative power there
the appeal from the decision or award by the Voluntary Arbitrator or Panel is an underlying power in the courts to scrutinize the acts of such agencies
of Arbitrators. on questions of law and jurisdiction even though no right of review is given
by statute' (73 C.J.S. 506, note 56).
The petitioner posits that the appeal from the decision or award of the
Voluntary Arbitrator should be filed within 10 days in view of Article 276 of "The purpose of judicial review is to keep the administrative agency within
the Labor Code which reads in full: its jurisdiction and protect substantial rights of parties affected by its
decisions' (73 C.J.S. 507, Sec. 165). It is part of the system of checks and
Article 276. Procedures. – The Voluntary Arbitrator or panel of Voluntary balances which restricts the separation of powers and forestalls arbitrary and
Arbitrators shall have the power to hold hearings, receive evidences and take unjust adjudications.
whatever action is necessary to resolve the issue or issues subject of the
dispute, including efforts to effect a voluntary settlement between parties. "Judicial review is proper in case of lack of jurisdiction, grave abuse of
discretion, error of law, fraud or collusion (Timbancaya vs. Vicente, 62 O.G.
All parties to the dispute shall be entitled to attend the arbitration 9424; Macatangay vs. Secretary of Public Works and Communications, 63
proceedings. The attendance of any third party or the exclusion of any O.G. 11236; Ortua vs. Singson Encarnacion, 59 Phil. 440).
witness from the proceedings shall be determined by the Voluntary Arbitrator
or panel of Voluntary Arbitrators. Hearings may be adjourned for cause or "'The courts may declare an action or resolution of an administrative
upon agreement by the parties. authority to be illegal (1) because it violates or fails to comply with some
mandatory provision of the law or (2) because it is corrupt, arbitrary or
Unless the parties agree otherwise, it shall be mandatory for the Voluntary capricious' (Borromeo vs. City of Manila and Rodriguez Lanuza, 62 Phil.
Arbitrator or panel of Voluntary Arbitrators to render an award or decision 512, 516; Villegas vs. Auditor General, L-21352, November 29, 1966, 18
within twenty (20) calendar days from the date of submission of the dispute SCRA 877, 891). [San Miguel Corporation v. Secretary of Labor, 64 SCRA
to voluntary arbitration. 60].

The award or decision of the Voluntary Arbitrator or panel of Voluntary xxx xxx xxx
Arbitrators shall contain the facts and the law on which it is based. It shall be
final and executory after ten (10) calendar days from receipt of the copy of "It is now settled rule that under the present Labor Code, (Presidential Decree
the award or decision by the parties. No. 442, as amended [1974] if lack of power or arbitrary or improvident
exercise of authority be shown, thus giving rise to a jurisdictional question,
Upon motion of any interested party, the Voluntary Arbitrator or panel of this Court may, in appropriate certiorari proceedings, pass upon the validity
Voluntary Arbitrators or the Labor Arbiter in the region where the movant of the decisions reached by officials or administrative agencies in labor
resides, in case of the absence or incapacity of the Voluntary Arbitrator or controversies. So it was assumed in Maglasang v. Ople, (L-38813, April 29,
panel of Voluntary Arbitrators, for any reason, may issue a writ of execution 1975, 63 SCRA 508). It was explicitly announced in San Miguel Corporation
requiring either the sheriff of the Commission or regular courts or any public v. Secretary of Labor, (L-39195, May 16, 1975, 64 SCRA 56) the opinion
official whom the parties may designate in the submission agreement to being penned by Justice Aquino. Accordingly, cases of that character
execute the final decision, order or award. (Bold underscoring supplied for continue to find a place in our docket. (Cf. United Employees Union of
emphasis) Gelmart Industries v. Noriel, L-40810, Oct. 3, 1975, 67 SCRA 267) The
present suit is of that category. [Kapisanan ng mga Manggagawa sa La
Article 276 is an amendment introduced by R.A. No. 6715.[27] Prior to the Suerte-Foitaf vs. Noriel, 77 SCRA 415-416].
effectivity of the amendment on March 21, 1989,[28] Article 262 (the
predecessor provision) stated that voluntary arbitration decisions or awards A voluntary arbitrator by the nature of her functions acts in a quasi-judicial
would be final, unappealable and executory. Despite such immediately capacity. There is no reason why her decisions involving interpretation of
executory nature of the decisions and awards of the Voluntary Arbitrators, law should be beyond this Court's review. Administrative officials are
presumed to act in accordance with law and yet we do not hesitate to pass An "instrumentality" is anything used as a means or agency. Thus, the terms
upon their work where a question of law is involved or where a showing of governmental "agency" or "instrumentality" are synonymous in the sense that
abuse of authority or discretion in their official acts is properly raised in either of them is a means by which a government acts, or by which a certain
petitions for certiorari.[30] government act or function is performed. The word "instrumentality," with
respect to a state, contemplates an authority to which the state delegates
Accordingly, the decisions and awards of Voluntary Arbitrators, albeit governmental power for the performance of a state function. An individual
immediately final and executory, remained subject to judicial review in person, like an administrator or executor, is a judicial instrumentality in the
appropriate cases through petitions for certiorari.[31] settling of an estate, in the same manner that a sub-agent appointed by a
bankruptcy court is an instrumentality of the court, and a trustee in
Such was the state of things until the promulgation in 1995 of the ruling in bankruptcy of a defunct corporation is an instrumentality of the state.
Luzon Development Bank v. Association of Luzon Development Bank
Employees.[32] Therein, the Court noted the silence of R.A. No. 6715 on the The voluntary arbitrator no less performs a state function pursuant to a
availability of appeal from the decisions or awards of the Voluntary governmental power delegated to him under the provisions therefor in the
Arbitrators. In declaring the Voluntary Arbitrators or Panels of Voluntary Labor Code and he falls, therefore, within the contemplation of the term
Arbitrators as quasi-judicial instrumentalities, Luzon Development Bank v. "instrumentality" in the aforequoted Sec. 9 of B.P. 129. The fact that his
Association of Luzon Development Bank Employees pronounced the functions and powers are provided for in the Labor Code does not place him
decisions or awards of the Voluntary Arbitrators to be appealable to the CA, within the exceptions to said Sec.. 9 since he is a quasi-judicial
viz.: instrumentality as contemplated therein. It will be noted that, although the
Employees' Compensation Commission is also provided for in the Labor
It will thus be noted that the jurisdiction conferred by law on a voluntary Code, Circular No. 1-91, which is the forerunner of the present Revised
arbitrator or a panel of such arbitrators is quite limited compared to the Administrative Circular No. 1-95, laid down the procedure for the
original jurisdiction of the labor arbiter and the appellate jurisdiction of the appealability of its decisions to the Court of Appeals under the foregoing
National Labor Relations Commission (NLRC) for that matter. The state of rationalization, and this was later adopted by Republic Act No. 7902 in
our present law relating to voluntary arbitration provides that "(t)he award or amending Sec. 9 of B.P. 129.
decision of the Voluntary Arbitrator x x x shall be final and executory after
ten (10) calendar days from receipt of the copy of the award or decision by A fortiori, the decision or award of the voluntary arbitrator or panel of
the parties," while the "(d)ecision, awards, or orders of the Labor Arbiter are arbitrators should likewise be appealable to the Court of Appeals, in line with
final and executory unless appealed to the Commission by any or both parties the procedure outlined in Revised Administrative Circular No. 1-95, just like
within ten (10) calendar days from receipt of such decisions, awards, or those of the quasi-judicial agencies, boards and commissions enumerated
orders." Hence, while there is an express mode of appeal from the decision therein.
of a labor arbiter, Republic Act No. 6715 is silent with respect to an appeal
from the decision of a voluntary arbitrator. This would be in furtherance of, and consistent with, the original purpose of
Circular No. 1-91 to provide a uniform procedure for the appellate review of
Yet, past practice shows that a decision or award of a voluntary arbitrator is, adjudications of all quasi-judicial entities not expressly excepted from the
more often than not, elevated to the Supreme Court itself on a petition for coverage of Sec. 9 of B.P. 129 by either the Constitution or another statute.
certiorari, in effect equating the voluntary arbitrator with the NLRC or the Nor will it run counter to the legislative intendment that decisions of the
Court of Appeals. In the view of the Court, this is illogical and imposes an NLRC be reviewable directly by the Supreme Court since, precisely, the
unnecessary burden upon it. cases within the adjudicative competence of the voluntary arbitrator are
excluded from the jurisdiction of the NLRC or the labor arbiter.[33]
In Volkschel Labor Union, et al. v. NLRC, et al., on the settled premise that
the judgments of courts and awards of [quasi-judicial] agencies must become In other words, the remedy of appeal by petition for review under Rule 43 of
final at some definite time, this Court ruled that the awards of voluntary the Rules of Court became available to the parties aggrieved by the decisions
arbitrators determine the rights of parties; hence, their decisions have the or awards of the Voluntary Arbitrators or Panels of Arbitrators.
same legal effect as judgments of a court. In Oceanic Bic Division (FFW), et
al. v. Romero, et al., this Court ruled that "a voluntary arbitrator by the nature In the 2004 ruling in Sevilla Trading Company v. Semana,[34] the Court
of her functions acts in a quasi-judicial capacity." Under these rulings, it ruled that the decision of the Voluntary Arbitrator became final and
follows that the voluntary arbitrator, whether acting solely or in a panel, executory after the expiration of the 15-day reglementary period within
enjoys in law the status of a quasi-judicial agencybut independent of, and which to file the petition for review under Rule 43. Manila Midtown Hotel
apart from, the NLRC since his decisions are not appealable to the latter. v. Borromeo[35] also ruled so. The 15-day period was likewise adverted to
in the ruling in Nippon Paint Employees Union-Olalia v. Court of
Section 9 of B.P. Blg. 129, as amended by Republic Act No. 7902, provides Appeals,[36]promulgated in November 2004.
that the Court of Appeals shall exercise:
In 2005, the Court promulgated the decision in Coca-Cola Bottlers
"xxx xxx xxx (3) Exclusive appellate jurisdiction over all final judgments, Philippines, Inc., Sales Force Union-PTGWO-Balais v. Coca-Cola Bottlers
decisions, resolutions, orders or awards of Regional Trial Courts and quasi- Philippines, Inc.,[37] wherein it made reference for the first time to the 10-
judicial agencies, instrumentalities, boards or commissions, including the day period for the filing of the petition for review vis-a-vis decisions or
Securities and Exchange Commission, the Employees' Compensation awards of the Voluntary Arbitrator provided in Article 262-A (now Article
Commission and the Civil Service Commission, except those falling within 276).[38] Within the same year, Philex Gold Philippines, Inc. v. Philex
the appellate jurisdiction of the Supreme Court in accordance with the Bulawan Supervisors Union[39] applied the period of 10 days in declaring
Constitution, the Labor Code of the Philippines under Presidential Decree the appeal to have been timely filed.
No. 442, as amended, the provisions of this Act, and of subparagraph (1) of
the third paragraph and subparagraph (4) of the fourth paragraph of Section Thereafter, the Court has variantly applied either the 15-day or the 10- day
17 of the Judiciary Act of 1948. period as the time within which to appeal the decisions or awards of the
Voluntary Arbitrators or Panels of Arbitrators. Thus, in the 2007 ruling
xxx xxx xxx" inLeyte IV Electric Cooperative, Inc. v. Leyeco IV Employees Union-
ALU,[40] the Court recognized the 15-day reglementary period under Rule
Assuming arguendo that the voluntary arbitrator or the panel of voluntary 43. This was reiterated in AMA Computer College-Santiago City, Inc. v.
arbitrators may not strictly be considered as a [quasi-judicial] agency, board Nacino(2008),[41] Mora v. Avesco Marketing Corporation[42] (2008),
or commission, still both he and the panel are comprehended within the Samahan Ng Mga Manggagawa sa Hyatt-NUWHRAIN-APL v. Bacungan
concept of a "quasi-judicial instrumentality." It may even be stated that it was (2009),[43] Saint Luis University, Inc. v. Cobarrubias[44] (2010), Samahan
to meet the very situation presented by the quasi-judicial functions of the ng mga Manggagawa sa Hyatt (SAMASAH-NUWHRAIN) v. Magsalin[45]
voluntary arbitrators here, as well as the subsequent arbitrator/arbitral (2011) and Royal Plant Workers Union v. Coca Cola Bottlers Philippines,
tribunal operating under the Construction Industry Arbitration Commission, Inc.-Cebu Plant (2013). [46]
that the broader term "instrumentalities" was purposely included in the
above-quoted provision. But in Philippine Electric Corporation (PHILEC) v. Court of Appeals[47]
(2014), Baronda v. Court of Appeals[48](2015), and NYK-FIL Ship
Management, Inc. v. Dabu[49] (2017), the Court, citing Article 276 of the
Labor Code, applied the 10-day period. Notably, the Court opined in The requirement that administrative remedies be exhausted is based on the
Philippine Electric Corporation (PHILEC) v. Court of Appeals that despite doctrine that in providing for a remedy before an administrative agency,
the period provided in Rule 43, the 10-day period should apply in every opportunity must be given to the agency to resolve the matter and to
determining the timeliness of appealing the decision or award of the exhaust all opportunities for a resolution under the given remedy before
Voluntary Arbitrator or Panel of Arbitrators, to wit: bringing an action in, or resorting to, the courts of justice. Where Congress
has not clearly required exhaustion, sound judicial discretion governs, guided
Despite Rule 43 providing for a 15-day period to appeal, we rule that the by congressional intent.
Voluntary Arbitrator's decision must be appealed before the Court of Appeals
within 10 calendar days from receipt of the decision as provided in the Labor By disallowing reconsideration of the VA's decision, Section 7, Rule XIX of
Code. DO 40-03 and Section 7 of the 2005 Procedural Guidelines went directly
against the legislative intent behind Article 262-A of the Labor Code. These
Appeal is a "statutory privilege," which may be exercised "only in the manner rules deny the VA the chance to correct himself and compel the courts of
and in accordance with the provisions of the law." "Perfection of an appeal justice to prematurely intervene with the action of an administrative agency
within the reglementary period is not only mandatory but also jurisdictional entrusted with the adjudication of controversies coming under its special
so that failure to do so rendered the decision final and executory, and deprives knowledge, training and specific field of expertise. In this era of clogged
the appellate court of jurisdiction to alter the final judgment much less to court dockets, the need for specialized administrative agencies with the
entertain the appeal." special knowledge, experience and capability to hear and determine promptly
disputes on technical matters or intricate questions of facts, subject to judicial
We ruled that Article 262-A of the Labor Code allows the appeal of decisions review, is indispensable. In Industrial Enterprises, Inc. v. Court of Appeals,
rendered by Voluntary Arbitrators. Statute provides that the Voluntary we ruled that relief must first be obtained in an administrative proceeding
Arbitrator's decision "shall be final and executory after ten (10) calendar days before a remedy will be supplied by the courts even though the matter is
from receipt of the copy of the award or decision by the parties." Being within the proper jurisdiction of a court.[52] (Emphasis supplied)
provided in the statute, this 10-day period must be complied with; otherwise,
no appellate court will have jurisdiction over the appeal. This absurd situation Hence, the 10-day period stated in Article 276 should be understood as the
occurs when the decision is appealed on the 11th to 15th day from receipt as period within which the party adversely affected by the ruling of the
allowed under the Rules, but which decision, under the law, has already Voluntary Arbitrators or Panel of Arbitrators may file a motion for
become final and executory. reconsideration. Only after the resolution of the motion for reconsideration
may the aggrieved party appeal to the CA by filing the petition for review
Furthermore, under Article VIII, Section 5 (5) of the Constitution, this court under Rule 43 of the Rules of Court within 15 days from notice pursuant to
"shall not diminish, increase, or modify substantive rights" in promulgating Section 4 of Rule 43.
rules of procedure in courts. The 10-day period to appeal under the Labor
Code being a substantive right, this period cannot be diminished, increased, The Court notes that despite the clarification made in Teng v. Pagahac, the
or modified through the Rules of Court. Department of Labor and Employment (DOLE) and the National
Conciliation and Mediation Board (NCMB) have not revised or amended the
In Shioji v. Harvey, this Court held that the "rules of court, promulgated by Revised Procedural Guidelines in the Conduct of Voluntary Arbitration
authority of law, have the force and effect of law, if not in conflict with Proceedings insofar as its Section 7 of Rule VII[53] is concerned. This
positive law." Rules of Court are "subordinate to the statute." In case of inaction has obviously sown confusion, particularly in regard to the filing of
conflict between the law and the Rules of Court, "the statute will prevail." the motion for reconsideration as a condition precedent to the filing of the
petition for review in the CA. Consequently, we need to direct the DOLE and
The rule, therefore, is that a Voluntary Arbitrator's award or decision shall be the NCMB to cause the revision or amendment of Section 7 of Rule VII of
appealed before the Court of Appeals within 10 days from receipt of the the Revised Procedural Guidelines in the Conduct of Voluntary Arbitration
award or decision. Should the aggrieved party choose to file a motion for Proceedings in order to allow the filing of motions for reconsideration in line
reconsideration with the Voluntary Arbitrator, the motion must be filed with Article 276 of the Labor Code.
within the same 10-day period since a motion for reconsideration is filed
"within the period for taking an appeal."[50] II
Certiorari does not lie in assailing
The ratiocination in Philippine Electric Corporation (PHILEC) v. Court of the CA's denial of a motion to dismiss
Appeals backstopped the ruling in NYK-FIL Ship Management, Inc. v.
Dabu. Generally, the denial of a motion to dismiss cannot be assailed by petition
for certiorari. As we indicated inBiñan Rural Bank v. Carlos:[54]
Given the variable rulings of the Court, what should now be the period to be
followed in appealing the decisions or awards of the Voluntary Arbitrators The denial of a motion to dismiss generally cannot be questioned in a special
or Panel of Arbitrators? civil action for certiorari, as this remedy is designed to correct only errors of
jurisdiction and not errors of judgment. Neither can a denial of a motion to
In the 2010 ruling in Teng v. Pagahac,[51] the Court clarified that the 10-day dismiss be the subject of an appeal which is available only after a judgment
period set in Article 276 of theLabor Code gave the aggrieved parties the or order on the merits has been rendered. Only when the denial of the motion
opportunity to file their motion for reconsideration, which was more in to dismiss is tainted with grave abuse of discretion can the grant of the
keeping with the principle of exhaustion of administrative remedies, holding extraordinary remedy of certiorari be justified.
thusly:
Although it admits being aware of this rule, the petitioner insists on the
In the exercise of its power to promulgate implementing rules and propriety of its petition for certioraribased on its belief that the CA had
regulations, an implementing agency, such as the Department of Labor, is gravely abused its discretion in assuming jurisdiction over the respondents'
restricted from going beyond the terms of the law it seeks to implement; it petition. It argues that the decision rendered by Voluntary Arbitrator
should neither modify nor improve the law. The agency formulating the rules Bacungan had already become final pursuant to Article 276 of the Labor
and guidelines cannot exceed the statutory authority granted to it by the Code, and, accordingly, the CA could no longer exercise its appellate
legislature. jurisdiction.

By allowing a 10-day period, the obvious intent of Congress in amending The petitioner is mistaken.
Article 263 to Article 262-A is to provide an opportunity for the party
adversely affected by the VA's decision to seek recourse via a motion for Grave abuse of discretion means either that the judicial or quasi-judicial
reconsideration or a petition for review under Rule 43 of the Rules of Court power was exercised in an arbitrary or despotic manner by reason of passion
filed with the CA. Indeed, a motion for reconsideration is the more or personal hostility, or that the respondent judge, tribunal or board evaded a
appropriate remedy in line with the doctrine of exhaustion. of administrative positive duty, or virtually refused to perform the duty enjoined or to act in
remedies. For this reason, an appeal from administrative agencies to the CA contemplation of law, such as when such judge, tribunal or board exercising
via Rule 43 of the Rules of Court requires exhaustion of available remedies judicial or quasi-judicial powers acted in a capricious or whimsical manner
as a condition precedent to a petition under that Rule. as to be equivalent to lack of jurisdiction.[55]
Here, the CA did not act arbitrarily in denying the petitioner's Motion to
Dismiss. It correctly noted that Coca-Cola Bottlers Philippines, Inc. Sales
Force Union-PTGWO-Balais v. Coca-Cola Bottlers Philippines, Inc. did not
make a definitive ruling on the correct reglementary period for the filing of
the petition for review. Given the varying applications of the periods defined
in Article 276 and Section 4 of Rule 43, the CA could not be objectively held
to be guilty of grave abuse of discretion in applying the equitable rule on
construction in favor of labor. To be underscored is that the underlying aim
for the requirement of strict adherence to procedural rules, particularly on
appeals, should always be the prevention of needless delays that could enable
the unscrupulous employers to wear out the efforts and meager resources of
their workers to the point that the latter would be constrained to settle for less
than what were due to them.[56]

ACCORDINGLY, the Court DISMISSES the unmeritorious petition for


certiorari; AFFIRMS the decision promulgated on December 15, 2008 by the
Court of Appeals; and DIRECTS the Department of Labor and Employment
and the National Conciliation and Mediation Board to revise or amend the
Revised Procedural Guidelines in the Conduct of Voluntary Arbitration
Proceedings to amend the Revised Procedural Guidelines in the Conduct of
Voluntary Arbitration Proceedings to reflect the foregoing ruling herein.

No pronouncement on costs of suit.

SO ORDERED.
KAPATIRAN SA MEAT AND CANNING DIVISION vs. CALLEJA
G.R. No. 82914 | 1988-06-20 The fact that TUPAS was able to negotiate a new CBA with ROBINA within
GRIá'O-AQUINO, J.: the 60-day freedom period of the existing CBA, does not foreclose the right
of the rival union, NEW ULO, to challenge TUPAS' claim to majority status,
The petitioner, Kapatiran sa Meat and Canning Division (TUPAS Local by filing a timely petition for certification election on October 13, 1987
Chapter No. 1027) hereinafter referred to as "TUPAS," seeks a review of the before TUPAS' old CBA expired on November 15, 1987 and before it signed
resolution dated January 27, 1988 (Annex D) of Public respondent Pura a new CBA with the company on December 3, 1987. As pointed out by Med-
Ferrer-Calleja, Director of the Bureau of Labor Relations, dismissing its Arbiter Abdullah, a "certification election is the best forum in ascertaining
appeal from the Order dated November 17, 1987 (Annex C) of the Med- the majority status of the contending unions wherein the workers themselves
Arbiter Rasidali C. Abdullah ordering a certification election to be conducted can freely choose their bargaining representative thru secret ballot." Since it
among the regular daily paid rank and file employees/workers of Universal has not been shown that this order is tainted with unfairness, this Court will
Robina Corporation-Meat and Canning Division to determine which of the not thwart the holding of a certification election (Associated Trade Unions
contending unions: [ATU] vs. Noriel, 88 SCRA 96).

a) Kapatiran sa Meat and Canning Division TUPAS Local Chapter No. 1027 WHEREFORE, the petition for certiorari is denied, with costs against the
(or "TUPAS" for brevity); petitioner.

b) Meat and Canning Division New Employees and Workers United Labor SO ORDERED.
Organization (or "NEW ULO" for brevity);

c) No Union.

shall be the bargaining unit of the daily wage rank and file employees in the
Meat and Canning Division of the company.

From 1984 to 1987 TUPAS was the sole and exclusive collective bargaining
representative of the workers in the Meat and Canning Division of the
Universal Robina Corporation, with a 3-year collective bargaining agreement
(CBA) which was to expire on November 15, 1987.

Within the freedom period of 60 days prior to the expiration of its CBA,
TUPAS filed an amended notice of strike on September 28, 1987 as a means
of pressuring the company to extend, renew, or negotiate a new CBA with it.

On October 8, 1987, the NEW ULO, composed mostly of workers belonging


to the IGLESIA NI KRISTO sect, registered as a labor union.

On October 12, 1987, the TUPAS staged a strike. ROBINA obtained an


injunction against the strike, resulting in an agreement to return to work and
for the parties to negotiate a new CBA.

The next day, October 13, 1987, NEW ULO, claiming that it has "the
majority of the daily wage rank and file employees numbering 191," filed a
petition for a certification election at the Bureau of Labor Relations (Annex
A).

TUPAS moved to dismiss the petition for being defective in form and that
the members of the NEW ULO were mostly members of the Iglesia ni Kristo
sect which three (3) years previous refused to affiliate with any labor union.
It also accused the company of using the NEW ULO to defeat TUPAS'
bargaining rights (Annex B).

On November 17, 1987, the Med-Arbiter ordered the holding of a


certification election within 20 days (Annex C).

TUPAS appealed to the Bureau of Labor Relations (BLR). In the meantime,


it was able to negotiate a new 3-year CBA with ROBINA, which was signed
on December 3, 1987 and to expire on November 15, 1990.

On January 27, 1988, respondent BLB Director Calleja dismissed the appeal
(Annex D).

TUPAS' motion for reconsideration (Annex E) was denied on March 17,


1988 (Annex F). On April 30, 1988, it filed this petition alleging that the
public respondent acted in excess of her jurisdiction and with grave abuse of
discretion in affirming the Med-Arbiter's order for a certification election.

After deliberating on the petition and the documents annexed thereto, We


find no merit in the petition. The public respondent did not err in dismissing
the petitioner's appeal in BLR Case No. A-12-389-87. This Court's decision
in Victoriano vs. Elizalde Rope Workers' Union, 59 SCRA 54, upholding the
right of members of the IGLESIA NI KRISTO sect not to join a labor union
for being contrary to their religious beliefs, does not bar the members of that
sect from forming their own union. The public respondent correctly observed
that the "recognition of the tenets of the sect . . . should not infringe on the
basic right of self-organization granted by the constitution to workers,
regardless of religious affiliation."
REYES vs. TRAJANO elections in the company for the reason that their religious beliefs do not
G.R. No. 84433 | 1992-06-02 allow them to form, join or assist labor organizations."
NARVASA, C.J.:
It is this Decision of July 22, 1988 that the petitioners would have this Court
The officer-in-charge of the Bureau of Labor Relations (Hon. Cresenciano annul and set aside in the present special civil action of certiorari.
Trajano) sustained the denial by the Med Arbiter of the right to vote of one
hundred forty-one (141) members of the "Iglesia ni Kristo" (INK), all The Solicitor General having expressed concurrence with the position taken
employed in the same company, at a certification election at which two (2) by the petitioners, public respondent NLRC was consequently required to
labor organizations were contesting the right to be the exclusive file, and did thereafter file, its own comment on the petition. In that comment
representative of the employees in the bargaining unit. That denial is assailed it insists that "if the workers who are members of the Iglesia ni Kristo in the
as having been done with grave abuse of discretion in the special civil action exercise of their religious belief opted not to join any labor organization as a
of certiorari at bar, commenced by the INK members adversely affected consequence of which they themselves can not have a bargaining
thereby. representative, then right to be represented by a bargaining agent should not
be denied to other members of the bargaining unit."
The certification election was authorized to be conducted by the Bureau of
Labor Relations among the employees of Tri-Union Industries Corporation Guaranteed to all employees or workers is the "right to self-organization and
on October 20, 1987. The competing unions were the Tri-Union Employees to form, join, or assist labor organizations of their own choosing for purposes
Union-Organized Labor Association in Line Industries and Agriculture of collective bargaining." This is made plain by no less than three provisions
(TUEU-OLALIA), and Trade Union of the Philippines and Allied Services of the Labor Code of the Philippines. 2 Article 243 of the Code provides as
(TUPAS). Of the 348 workers initially deemed to be qualified voters, only follows: 3
240 actually took part in the election, conducted under the supervision of the
Bureau of Labor Relations. Among the 240 employees who cast their votes ART. 243. Coverage and employees right to self-organization. All persons
were 141 members of the INK. employed in commercial, industrial and agricultural enterprises and in
religious, charitable, medical, or educational institutions whether operating
The ballots provided for three (3) choices. They provided for votes to be cast, for profit or not, shall have the right to self-organization and to form, join, or
of course, for either of the two (2) contending labor organizations, (a) assist labor organizations of their own choosing for purposes of collective
TUPAS and (b) TUEU-OLALIA; and, conformably with established rule and bargaining.
practice, 1 for (c) a third choice: "NO UNION." Ambulant, intermittent and itinerant workers, self-employed people, rural
workers and those without any definite employers may form labor
The final tally of the votes showed the following results: organizations for their mutual aid and protection.

TUPAS 1 Article 248 (a) declares it to be an unfair labor practice for an employer,
TUEU-OLALIA 95 among others, to "interfere with, restrain or coerce employees in the exercise
NO UNION 1 of their right to self-organization." Similarly, Article 249 (a) makes it an
SPOILED 1 unfair labor practice for a labor organization to "restrain or coerce employees
CHALLENGED 141 in the exercise of their rights to self-organization . . . ."

The challenged votes were those cast by the 141 INK members. They were The same legal proposition is set out in the Omnibus Rules Implementing the
segregated and excluded from the final count in virtue of an agreement Labor Code, as amended, as might be expected. Section 1, Rule II
between the competing unions, reached at the pre-election conference, that (Registration of Unions), Book V (Labor Relations) of the Omnibus Rules
the INK members should not be allowed to vote "because they are not provides as follows: 4
members of any union and refused to participate in the previous certification
elections." "SEC. 1. Who may join unions; exception. All persons employed in
commercial, industrial and agricultural enterprises, including employees of
The INK employees promptly made known their protest to the exclusion of government corporations established under the Corporation Code as well as
their votes. They filed a petition to cancel the election alleging that it "was employees of religious, medical or educational institutions, whether
not fair" and the result thereof did "not reflect the true sentiments of the operating for profit or not, except managerial employees, shall have the right
majority of the employees." TUEU-OLALIA opposed the petition. It to self-organization and to form, join or assist labor organizations for
contended that the petitioners " do not have legal personality to protest the purposes of collective bargaining. Ambulant, intermittent and itinerant
results of the election," because "they are not members of either contending workers, self-employed people, rural workers and those without any definite
unit, but . . . of the INK" which prohibits its followers, on religious grounds, employers may form labor organizations for their mutual aid and protection.
from joining or forming any labor organization . . . ."
xxx xxx xxx"
The Med-Arbiter saw no merit in the INK employees' petition. By Order
dated December 21, 1987, he certified the TUEU-OLALlA as the sole and The right of self-organization includes the right to organize or affiliate with
exclusive bargaining agent of the rank-and-file employees. In that Order he a labor union or determine which of two or more unions in an establishment
decried the fact that "religious belief was (being) utilized to render to join, and to engage in concerted activities with co-workers for purposes of
meaningless the rights of the non-members of the Iglesia ni Kristo to exercise collective bargaining through representatives of their own choosing, or for
the rights to be represented by a labor organization as the bargaining agent," their mutual aid and protection, i.e., the protection, promotion, or
and declared the petitioners as "not possessed of any legal personality to enhancement of their rights and interests. 5
institute this present cause of action" since they were not parties to the
petition for certification election. Logically, the right NOT to join, affiliate with, or assist any union, and to
disaffiliate or resign from a labor organization, is subsumed in the right to
The petitioners brought the matter up on appeal to the Bureau of Labor join, affiliate with, or assist any union, and to maintain membership therein.
Relations. There they argued that the Med-Arbiter had "practically The right to form or join a labor organization necessarily includes the right
disenfranchised petitioners who had an overwhelming majority," and "the to refuse or refrain from exercising said right. It is self-evident that just as no
TUEU-OLALIA certified union cannot be legally said to have been the result one should be denied the exercise of a right granted by law, so also, no one
of a valid election where at least fifty-one percent of all eligible voters in the should be compelled to exercise such a conferred right. The fact that a person
appropriate bargaining unit shall have cast their votes." Assistant Labor has opted to acquire membership in a labor union does not preclude his
Secretary Cresenciano B. Trajano, then Officer-in-Charge of the Bureau of subsequently opting to renounce such membership. 6
Labor Relations, denied the appeal in his Decision of July 22, 1988. He
opined that the petitioners are "bereft of legal personality to protest their As early as 1974 this Court had occasion to expatiate on these self-evident
alleged disenfranchisement" since they "are not constituted into a duly propositions in Victoriano v. Elizalde Rope Workers' Union, et al., 7 viz.:
organized labor union, hence, not one of the unions which vied for
certification as sole and exclusive bargaining representative." He also ". . . What the Constitution and Industrial Peace Act recognize and guarantee
pointed out that the petitioners "did not participate in previous certification is the 'right' to form or join associations. Notwithstanding the different
theories propounded by the different schools of jurisprudence regarding the
nature and contents of a 'right,' it can be safely said that whatever theory one the plea that they, the minority workers, are being denied the right of self-
subscribes to, a right comprehends at least two broad notions, namely: first, organization and collective bargaining. As repeatedly stated, the right of self-
liberty or freedom, i.e., the absence of legal restraint, whereby an employee organization embraces not only the right to form, join or assist labor
may act for himself without being prevented by law; second, power, whereby organizations, but the concomitant, converse right NOT to form, join or assist
an employee may, as he pleases, join or refrain from joining an association. any labor union.
It is therefore the employee who should decide for himself whether he should
join or not an association; and should he choose to join, he himself makes up That the INK employees, as employees in the same bargaining unit in the
his mind as to which association he would join; and even after he has joined, true sense of the term, do have the right of self-organization, is also in truth
he still retains the liberty and the power to leave and cancel his membership beyond question, as well as the fact that when they voted that the employees
with said organization at any time (Pagkakaisa Samahang Manggagawa ng in their bargaining unit should be represented by "NO UNION," they were
San Miguel Brewery vs. Enriquez, et al., 108 Phil. 1010, 1019). It is clear, simply exercising that right of self-organization, albeit in its negative aspect.
therefore, that the right to join a union includes the right to abstain from
joining any union (Abo, et al. vs. PHILAME [KG] Employees Union, et al., The respondents' argument that the petitioners are disqualified to vote
L-19912, January 20, 1965, 13 SCRA 120, 123, quoting Rothenberg, Labor because they "are not constituted into a duly organized labor union" "but
Relations). members of the INK which prohibits its followers, on religious grounds, from
Inasmuch as what both the Constitution and the Industrial Peace Act have joining or forming any labor organization" and "hence, not one of the unions
recognized, and guaranteed to the employee, is the 'right' to join associations which vied for certification as sole and exclusive bargaining representative,"
of his choice, it would be absurd to say that the law also imposes, in the same is specious. Neither law, administrative rule nor jurisprudence requires that
breath, upon the employee the duty to join associations. The law does not only employees affiliated with any labor organization may take part in a
enjoin an employee to sign up with any association." certification election. On the contrary, the plainly discernible intendment of
the law is to grant the right to vote to all bona fide employees in the
The right to refuse to join or be represented by any labor organization is bargaining unit, whether they are members of a labor organization or not. As
recognized not only by the law but also in the rules drawn up for held in Airtime Specialists, Inc. v. Ferrer-Calleja: 9
implementation thereof. The original Rules on Certification promulgated by
the defunct Court of Industrial Relations required that the ballots to be used "In a certification election all rank-and-file employees in the appropriate
at a certification election to determine which of two or more competing labor bargaining unit are entitled to vote. This principle is clearly stated in Art. 255
unions would represent the employees in the appropriate bargaining unit of the Labor Code which states that the 'labor organization designated or
should contain, aside from the names of each union, an alternative choice of selected by the majority of the employees in an appropriate bargaining unit
the employee voting, to the effect that he desires not to be represented by any shall be the exclusive representative of the employees in such unit for the
union. 8 And where only one union was involved, the ballots were required purpose of collective bargaining.' Collective bargaining covers all aspects of
to state the question "Do you desire to be represented by said union?" as the employment relation and the resultant CBA negotiated by the certified
regards which the employees voting would mark an appropriate square, one union binds all employees in the bargaining unit. Hence, all rank-and-file
indicating the answer, "Yes," the other, "No." employees, probationary or permanent, have a substantial interest in the
selection of the bargaining representative. The Code makes no distinction as
To be sure, the present implementing rules no longer explicitly impose the to their employment status as basis for eligibility in supporting the petition
requirement that the ballots at a certification election include a choice for for certification election. The law refers to `all' the employees in the
"NO UNION." Section 8 (Rule VI, Book V of the Omnibus Rules) entitled bargaining unit. All they need to be eligible to support the petition is to
"Marking and canvassing of votes," pertinently provides that: belong to the 'bargaining unit.'"

". . . (a) The voter must write a cross (X) or a check (/) in the square opposite Neither does the contention that petitioners should be denied the right to vote
the union of his choice. If only one union is involved, the voter shall make because they "did not participate in previous certification elections in the
his cross or check in the square indicating 'YES' or 'NO.' company for the reason that their religious beliefs do not allow them to form,
join or assist labor organizations," persuade acceptance. No law,
xxx xxx xxx" administrative rule or precedent prescribes forfeiture of the right to vote by
reason of neglect to exercise the right in past certification elections. In
Withal, neither the quoted provision nor any other in the Omnibus denying the petitioners' right to vote upon these egregiously fallacious
Implementing Rules expressly bars the inclusion of that choice of "NO grounds, the public respondents exercised their discretion whimsically,
UNION" in the ballots. Indeed, it is doubtful if the employee's alternative capriciously and oppressively and gravely abused the same.
right NOT to form, join or assist any labor organization or withdraw or resign
from one may be validly eliminated and he be consequently coerced to vote WHEREFORE, the petition for certiorari is GRANTED; the Decision of the
for one or another of the competing unions and be represented by one of then Officer-in-Charge of the Bureau of Labor Relations dated December 21,
them. 1987 (affirming the Order of the Med-Arbiter dated July 22, 1988) is
Besides, the statement in the quoted provision that "(i)f only one union is ANNULLED and SET ASIDE; and the petitioners are DECLARED to have
involved, the voter shall make his cross or check in the square indicating legally exercised their right to vote, and their ballots should be canvassed
'YES' or 'NO,' is quite clear acknowledgment of the alternative possibility and, if validly and properly made out, counted and tallied for the choices
that the "NO" votes may outnumber the "YES" votes indicating that the written therein. Costs against private respondents.
majority of the employees in the company do not wish to be represented by
any union in which case, no union can represent the employees in collective SO ORDERED.
bargaining.
And whether the prevailing "NO" votes are inspired by considerations of
religious belief or discipline or not is beside the point, and may not be
inquired into at all.

The purpose of a certification election is precisely the ascertainment of the


wishes of the majority of the employees in the appropriate bargaining unit:
to be or not to be represented by a labor organization, and in the affirmative
case, by which particular labor organization. If the results of the election
should disclose that the majority of the workers do not wish to be represented
by any union, then their wishes must be respected, and no union may properly
be certified as the exclusive representative of the workers in the bargaining
unit in dealing with the employer regarding wages, hours and other terms and
conditions of employment. The minority employees who wish to have a
union represent them in collective bargaining can do nothing but wait for
another suitable occasion to petition for a certification election and hope that
the results will be different. They may not and should not be permitted,
however, to impose their will on the majority who do not desire to have a
union certified as the exclusive workers' benefit in the bargaining unit - upon
FEU-DR. NICANOR REYES MEDICAL FOUNDATION, INC. vs. Ambulant intermittent and itinerant workers, self-employed people, rural
TRAJANO workers and those without any definite employers may form labor
G.R. No. 76273 | 1987-07-31 organizations for the purpose of enhancing and defending their interests and
PARAS, J.: for their mutual aid and protection." (emphasis supplied).

This is a petition for certiorari seeking to annul and set aside the decision of Under the aforequoted provision, there is no doubt that rank and file
the respondent Director which affirmed the Order of the Med-Arbiter in the employees of non-profit medical institutions (as herein petitioner) are now
petition for certification election (NCR-LRD-N-2-050-86) filed by private permitted to form, organize or join labor unions of their choice for purposes
respondent, thus ordering the holding of a certification election among the of collective bargaining. Since private respondent had complied with the
rank and file employees of the herein petitioner. requisites provided by law for calling a certification election (p. 15, Rollo),
it was incumbent upon respondent Director to conduct such certification
The facts of the case are as follows: election to ascertain the bargaining representative of petitioner's employees
(Samahang Manggagawa Ng Pacific Mills, Inc. vs. Noriel, 134 SCRA 152).
The petitioner, Far Eastern University-Dr. Nicanor Reyes Memorial
Foundation, Inc., has a work force of about 350 rank and file employees, As held in Quimpo v. Dela Victoria, 46 SCRA 139, in order that the pendency
majority of whom are members of private respondent Alliance of Filipino of another action between the same parties for the same cause may be availed
Workers. of as a ground to dismiss a case, there must be, between the action under
consideration and the other action: (1) identity of parties, or at least such as
On February 13, 1986, private respondent filed a Petition for Consent and/or representing the same interest in both actions; (2) identity of rights asserted
Certification Election with The Ministry of Labor and Employment. The and relief prayed for, the relief being founded on the same facts; and (3) the
petitioner opposed the petition on the ground that a similar petition involving identity on the two preceding particulars should be such that any judgment
the same issues and the same parties is pending resolution before the which may be rendered on the other action will, regardless of which party is
Supreme Court, docketed as G.R. No. L-49771. successful, amount to res judicata in the action under consideration.

In its position paper, private respondent admitted: that as early as May 10, In the instant case, any judgment which may be rendered in the petition for
1976, private respondent filed a similar petition for certification election with certiorari pending before the Supreme Court (G.R. No. L-49771) will not
the Ministry of Labor and Employment but the petition was denied by the constitute res judicata in the petition for certification election under
MED Arbiter and the Secretary of Labor on appeal, on the ground that the consideration, for while in the former, private respondent questioned the
petitioner was a non-stock, non-profit medical institution, therefore, its constitutionality of Article 244 of the Labor Code before its amendment, in
employees may not form, join, or organize a union pursuant to Article 244 of the latter, private respondent invokes the same article as already amended.
the Labor Code; that private respondent filed a petition for certiorari with the
Supreme Court (docketed as G.R. No. L-49771) assailing the Petitioner, however, has pointed out that respondent Director should not have
constitutionality of Article 244 of the Labor Code; that pending resolution of arrogated upon himself the power to declare the aforesaid petition for
the aforesaid petition, or on May 1, 1980, Batas Pambansa Bilang 70 was certiorari (G.R. No. L-49771) moot and academic, as the same is sub-judice
enacted amending Article 244 of the Labor Code, thus granting even and only the Supreme Court can decide the matter. The Director cannot be
employees of non-stock, non-profit institutions the right to form, join and faulted for he had to make a decision.
organize labor unions of their choice; and that in the exercise of such right,
private respondent filed another petition for certification election with the WHEREFORE, this petition is DISMISSED, and the decision appealed from
Ministry of Labor and Employment (NCR-LRD-N-2-050-86). is hereby AFFIRMED.

On April 17, 1986, the Med Arbiter issued an Order granting the petition, SO ORDERED.
declaring that a certification election be conducted to determine the exclusive
bargaining representative of all the rank and file employees of the petitioner
(p. 4, Rollo)

Respondent Director affirmed said Order on appeal. In dismissing the appeal,


however, respondent Director said that:

". . . respondent's (petitioner herein, reliance on the petition with the Supreme
Court involving as it does the provisions of Article 244 of the Labor Code
vis-a-vis the character of the hospital, which has been alleged as a non-profit
medical foundation, has been rendered moot and academic by virtue of the
amendatory BP #70, which allows employees of non-profit medical
institutions to unionize.

Whatever doubt there may be on the right of the workers in a medical


institution has been laid to rest by BP#70.

WHEREFORE, premises considered, the present appeal is hereby dismissed


for lack of merit and the Order of the Med-Arbiter dated 17 April 1986
affirmed . . ." (p. 19, Rollo)

Hence, this petition, raising the issue of whether or not respondent Director
gravely abused his discretion in granting the petition for certification
election, despite the pendency of a similar petition before the Supreme Court
(G.R. No. 49771) which involves the same parties for the same cause.

The Petition is devoid of merit.

At the time private respondent filed its petition for certification election on
February 13, 1986, Article 244 of the Labor Code was already amended by
Batas Pambansa Bilang 70, to wit:

"Art. 244. Coverage and employees' right to self-organization. All persons


employed in commercial, industrial and charitable, medical, or educational
institutions whether operating for profit or not, shall have the right to self-
organizations of their own choosing for purposes of collective bargaining.
TAGAYTAY HIGHLANDS INTERNATIONAL GOLF CLUB local/chapter. Its due reporting through the submission of all the
INCORPORATED vs. TAGAYTAY HIGHLANDS EMPLOYEES requirements for registration of a local/chapter is a clear showing that it was
UNION-PGTWO already included in the roster of legitimate labor organizations in this Office
G.R. No. 142000 | 2003-01-22 pursuant to Department Order No. 9 Series of 1997 with all the legal right
CARPIO-MORALES, J.: and personality to institute this instant petition. Pursuant therefore to the
provisions of Article 257 of the Labor Code, as amended, and its
Before this Court on certiorari under Rule 45 is the petition of the Tagaytay Implementing Rules as amended by Department Order No. 9, since the
Highlands International Golf Club Incorporated (THIGCI) assailing the respondent's establishment is unorganized, the holding of a certification
February 15, 2002 decision of the Court of Appeals denying its petition to election is mandatory for it was clearly established that petitioner is a
annul the Department of Labor and Employment (DOLE) Resolutions of legitimate labor organization. Giving due course to this petition is therefore
November 12, 1998 and December 29, 1998. proper and appropriate.[9]

On October 16, 1997, the Tagaytay Highlands Employees Union (THEU) . . Passing on THIGCI's allegation that some of the union members are
. Philippine Transport and General Workers Organization (PTGWO), Local supervisory, resigned and AWOL employees or employees of a separate and
Chapter No. 776, a legitimate labor organization said to represent majority distinct corporation, the Med-Arbiter held that the same should be properly
of the rank-and-file employees of THIGCI, filed a petition for certification raised in the exclusion-inclusion proceedings at the pre-election conference.
election before the DOLE Mediation-Arbitration Unit, Regional Branch No. As for the allegation that some of the signatures were secured through
IV. fraudulent and deceitful means, he held that it should be coursed through an
independent petition for cancellation of union registration which is within the
THIGCI, in its Comment[1] filed on November 27, 1997, opposed THEU's jurisdiction of the DOLE Regional Director. In any event, the Med-Arbiter
petition for certification election on the ground that the list of union members held that THIGCI failed to submit the job descriptions of the questioned
submitted by it was defective and fatally flawed as it included the names and employees and other supporting documents to bolster its claim that they are
signatures of supervisors, resigned, terminated and absent without leave disqualified from joining THEU.
(AWOL) employees, as well as employees of The Country Club, Inc., a
corporation distinct and separate from THIGCI; and that out of the 192 THIGCI appealed to the Office of the DOLE Secretary which, by Resolution
signatories to the petition, only 71 were actual rank-and-file employees of of June 4, 1998, set aside the said Med-Arbiter's Order and accordingly
THIGCI. dismissed the petition for certification election on the ground that there is a
"clear absence of community or mutuality of interests," it finding that THEU
THIGCI thus submitted a list of the names of its 71 actual rank-and-file sought to represent two separate bargaining units (supervisory employees
employees which it annexed[2] to its Comment to the petition for and rank-and-file employees) as well as employees of two separate and
certification election. And it therein incorporated the following tabulation[3] distinct corporate entities.
showing the number of signatories to said petition whose membership in the
union was being questioned as disqualified and the reasons for Upon Motion for Reconsideration by THEU, DOLE Undersecretary
disqualification: Rosalinda Dimalipis-Baldoz, by authority of the DOLE Secretary, issued
DOLE Resolution of November 12, 1998[10] setting aside the June 4, 1998
# of Signatures Reasons for Disqualification Resolution dismissing the petition for certification election. In the November
13 Supervisors of THIGCI 12, 1998 Resolution, Undersecretary Dimapilis-Baldoz held that since THEU
6 Resigned employees of THIGCI is a local chapter, the twenty percent (20%) membership requirement is not
2 AWOL employees of THIGCI necessary for it to acquire legitimate status, hence, "the alleged retraction and
53 Rank-and-file employees of The Country Club at Tagaytay Highlands, withdrawal of support by 45 of the 70 remaining rank-and-file members . . .
Inc. cannot negate the legitimacy it has already acquired before the petition;" that
14 Supervisors of The Country Club at Tagaytay Highlands, Inc. rather than disregard the legitimate status already conferred on THEU by the
6 Resigned employees of The Country Club at Tagaytay Highlands, Inc. Bureau of Labor Relations, the names of alleged disqualified supervisory
3 Terminated employees of The Country Club at Tagaytay Highlands, Inc. employees and employees of the Country Club, Inc., a separate and distinct
1 AWOL employees of The Country Club at Tagaytay Highlands, Inc. corporation, should simply be removed from the THEU's roster of
4 Signatures that cannot be deciphered membership; and that regarding the participation of alleged resigned and
16 Names in list that were erased AWOL employees and those whose signatures are illegible, the issue can be
2 Names with first names only resolved during the inclusion-exclusion proceedings at the pre-election stage.
The records of the case were thus ordered remanded to the Office of the Med-
THIGCI also alleged that some of the signatures in the list of union members Arbiter for the conduct of certification election.
were secured through fraudulent and deceitful means, and submitted copies
of the handwritten denial and withdrawal of some of its employees from THIGCI's Motion for Reconsideration of the November 12, 1998 Resolution
participating in the petition.[4] having been denied by the DOLE Undersecretary by Resolution of December
29, 1998,[11] it filed a petition for certiorari before this Court which, by
Replying to THIGCI's Comment, THEU asserted that it had complied with Resolution of April 14, 1999,[12] referred it to the Court of Appeals in line
all the requirements for valid affiliation and inclusion in the roster of with its pronouncement in National Federation of Labor (NFL) v. Hon.
legitimate labor organizations pursuant to DOLE Department Order No. 9, Bienvenido E. Laguesma, et al.,[13] and in strict observance of the hierarchy
series of 1997,[5] on account of which it was duly granted a Certification of of courts, as emphasized in the case of St. Martin Funeral Home v. National
Affiliation by DOLE on October 10, 1997;[6] and that Section 5, Rule V of Labor Relations Commission.[14]
said Department Order provides that the legitimacy of its registration cannot
be subject to collateral attack, and for as long as there is no final order of By Decision of February 15, 2000,[15] the Court of Appeals denied
cancellation, it continues to enjoy the rights accorded to a legitimate THIGCI's Petition for Certiorari and affirmed the DOLE Resolution dated
organization. November 12, 1998. It held that while a petition for certification election is
an exception to the innocent bystander rule, hence, the employer may pray
THEU thus concluded in its Reply[7] that under the circumstances, the Med- for the dismissal of such petition on the basis of lack of mutuality of interests
Arbiter should, pursuant to Article 257 of the Labor Code and Section 11, of the members of the union as well as lack of employer-employee
Rule XI of DOLE Department Order No. 09, automatically order the conduct relationship following this Court's ruling in Toyota Motor Philippines
of a certification election. Corporation v. Toyota Motor Philippines Corporation Labor Union et al[16]
and Dunlop Slazenger [Phils.] v. Hon. Secretary of Labor and Employment
By Order of January 28, 1998, [8] DOLE Med-Arbiter Anastacio Bactin et al,[17] petitioner failed to adduce substantial evidence to support its
ordered the holding of a certification election among the rank-and-file allegations.
employees of THIGCI in this wise, quoted verbatim: Hence, the present petition for certiorari, raising the following

We evaluated carefully this instant petition and we are of the opinion that it "ISSUES/ASSIGNMENT OF ERRORS:
is complete in form and substance. In addition thereto, the accompanying
documents show that indeed petitioner union is a legitimate labor federation THE COURT OF APPEALS GRIEVOUSLY ERRED IN AFFIRMING
and its local/chapter was duly reported to this Office as one of its affiliate THE RESOLUTION DATED 12 NOVEMER 1998 HOLDING THAT
SUPERVISORY EMPLOYEES AND NON-EMPLOYEES COULD certification election for the purpose of collective bargaining. It becomes
SIMPLY BE REMOVED FROM APPELLEES ROSTER OF RANK-AND- necessary, therefore, anterior to the granting of an order allowing a
FILE MEMBERSHIP INSTEAD OF RESOLVING THE LEGITIMACY certification election, to inquire into the composition of any labor
OF RESPONDENT UNION'S STATUS organization whenever the status of the labor organization is challenged on
the basis of Article 245 of the Labor Code." (Emphasis by petitioner)
THE COURT OF APPEALS GRIEVOUSLY ERRED IN AFFIRMING (Dunlop Slazenger (Phils.), v. Secretary of Labor, 300 SCRA 120 [1998];
THE RESOLUTION DATED 12 NOVEMBER 1998 HOLDING THAT Underscoring and emphasis supplied by petitioner.)
THE DISQUALIFIED EMPLOYEES' STATUS COULD READILY BE
RESOLVED DURING THE INCLUSION AND EXCLUSION The petition fails. After a certificate of registration is issued to a union, its
PROCEEDINGS legal personality cannot be subject to collateral attack. It may be questioned
THE COURT OF APPEALS GRIEVOUSLY ERRED IN NOT HOLDING only in an independent petition for cancellation in accordance with Section 5
THAT THE ALLEGATIONS OF PETITIONER HAD BEEN DULY of Rule V, Book IV of the "Rules to Implement the Labor Code"
PROVEN BY FAILURE OF RESPONDENT UNION TO DENY THE (Implementing Rules) which section reads:
SAME AND BY THE SHEER WEIGHT OF EVIDENCE INTRODUCED
BY PETITIONER AND CONTAINED IN THE RECORDS OF THE Sec. 5. Effect of registration. The labor organization or workers' association
CASE"[18] shall be deemed registered and vested with legal personality on the date of
issuance of its certificate of registration. Such legal personality cannot
The statutory authority for the exclusion of supervisory employees in a rank- thereafter be subject to collateral attack, but may be questioned only in an
and-file union, and vice-versa, is Article 245 of the Labor Code, to wit: independent petition for cancellation in accordance with these Rules.
Article 245. Ineligibility of managerial employees to join any labor (Emphasis supplied)
organization; right of supervisory employees. - Managerial employees are
not eligible to join, assist or form any labor organization. Supervisory The grounds for cancellation of union registration are provided for under
employees shall not be eligible for membership in a labor organization of the Article 239 of the Labor Code, as follows:
rank-and-file employees but may join, assist or form separate labor
organizations of their own. Art. 239. Grounds for cancellation of union registration. The following shall
constitute grounds for cancellation of union registration:
While above-quoted Article 245 expressly prohibits supervisory employees
from joining a rank-and-file union, it does not provide what would be the (a) Misrepresentation, false statement or fraud in connection with the
effect if a rank-and-file union counts supervisory employees among its adoption or ratification of the constitution and by-laws or amendments
members, or vice-versa. thereto, the minutes of ratification, and the list of members who took part in
the ratification;
Citing Toyota[19] which held that "a labor organization composed of both
rank-and-file and supervisory employees is no labor organization at all," and (b) Failure to submit the documents mentioned in the preceding paragraph
the subsequent case of Progressive Development Corp. " Pizza Hut v. within thirty (30) days from adoption or ratification of the constitution and
Ledesma[20] which held that: by-laws or amendments thereto;

"The Labor Code requires that in organized and unorganized establishments, (c) Misrepresentation, false statements or fraud in connection with the
a petition for certification election must be filed by a legitimate labor election of officers, minutes of the election of officers, the list of voters, or
organization. The acquisition of rights by any union or labor organization, failure to subject these documents together with the list of the newly
particularly the right to file a petition for certification election, first and elected/appointed officers and their postal addresses within thirty (30) days
foremost, depends on whether or not the labor organization has attained the from election;
status of a legitimate labor organization.
(d) Failure to submit the annual financial report to the Bureau within thirty
In the case before us, the Med-Arbiter summarily disregarded the petitioner's (30) days after the losing of every fiscal year and misrepresentation, false
prayer that the former look into the legitimacy of the respondent Union by a entries or fraud in the preparation of the financial report itself;
sweeping declaration that the union was in the possession of a charter
certificate so that 'for all intents and purposes, Sumasaklaw sa Manggagawa (e) Acting as a labor contractor or engaging in the "cabo" system, or
sa Pizza Hut (was) a legitimate organization,'"[21] otherwise engaging in any activity prohibited by law;

petitioner contends that, quoting Toyota, "[i]t becomes necessary . . ., anterior (f) Entering into collective bargaining agreements which provide terms and
to the granting of an order allowing a certification election, to inquire into conditions of employment below minimum standards established by law;
the composition of any labor organization whenever the status of the labor
organization is challenged on the basis of Article 245 of the Labor Code."[22] (g) Asking for or accepting attorney's fees or negotiation fees from
Continuing, petitioner argues that without resolving the status of THEU, the employers;
DOLE Undersecretary "conveniently deferred the resolution on the serious
infirmity in the membership of [THEU] and ordered the holding of the (h) Other than for mandatory activities under this Code, checking off special
certification election" which is frowned upon as the following ruling of this assessments or any other fees without duly signed individual written
Court shows: authorizations of the members;

We also do not agree with the ruling of the respondent Secretary of Labor (i) Failure to submit list of individual members to the Bureau once a year or
that the infirmity in the membership of the respondent union can be remedied whenever required by the Bureau; and
in "the pre-election conference thru the exclusion-inclusion proceedings
wherein those employees who are occupying rank-and-file positions will be (j) Failure to comply with the requirements under Articles 237 and 238,
excluded from the list of eligible voters." Public respondent gravely (Emphasis supplied),
misappreciated the basic antipathy between the interest of supervisors and
the interest of rank-and-file employees. Due to the irreconcilability of their while the procedure for cancellation of registration is provided for in Rule
interest we held in Toyota Motor Philippines v. Toyota Motors Philippines VIII, Book V of the Implementing Rules.
Corporation Labor Union, viz:
The inclusion in a union of disqualified employees is not among the grounds
'x x x for cancellation, unless such inclusion is due to misrepresentation, false
statement or fraud under the circumstances enumerated in Sections (a) and
"Clearly, based on this provision [Article 245], a labor organization (c) of Article 239 of above-quoted Article 239 of the Labor Code.
composed of both rank-and-file and supervisory employees is no labor THEU, having been validly issued a certificate of registration, should be
organization at all. It cannot, for any guise or purpose, be a legitimate labor considered to have already acquired juridical personality which may not be
organization. Not being one, an organization which carries a mixture of rank- assailed collaterally.
and-file and supervisory employees cannot posses any of the rights of a
legitimate labor organization, including the right to file a petition for
As for petitioner's allegation that some of the signatures in the petition for
certification election were obtained through fraud, false statement and
misrepresentation, the proper procedure is, as reflected above, for it to file a
petition for cancellation of the certificate of registration, and not to intervene
in a petition for certification election.

Regarding the alleged withdrawal of union members from participating in


the certification election, this Court's following ruling is instructive:

"'[T]he best forum for determining whether there were indeed retractions
from some of the laborers is in the certification election itself wherein the
workers can freely express their choice in a secret ballot.' Suffice it to say
that the will of the rank-and-file employees should in every possible instance
be determined by secret ballot rather than by administrative or quasi-judicial
inquiry. Such representation and certification election cases are not to be
taken as contentious litigations for suits but as mere investigations of a non-
adversary, fact-finding character as to which of the competing unions
represents the genuine choice of the workers to be their sole and exclusive
collective bargaining representative with their employer."[23]

As for the lack of mutuality of interest argument of petitioner, it, at all events,
does not lie given, as found by the court a quo, its failure to present
substantial evidence that the assailed employees are actually occupying
supervisory positions.

While petitioner submitted a list of its employees with their corresponding


job titles and ranks,[24] there is nothing mentioned about the supervisors'
respective duties, powers and prerogatives that would show that they can
effectively recommend managerial actions which require the use of
independent judgment.[25]

As this Court put it in Pepsi-Cola Products Philippines, Inc. v. Secretary of


Labor:[26]

Designation should be reconciled with the actual job description of subject


employees x x x The mere fact that an employee is designated manager does
not necessarily make him one. Otherwise, there would be an absurd situation
where one can be given the title just to be deprived of the right to be a member
of a union. In the case of National Steel Corporation vs. Laguesma (G. R.
No. 103743, January 29, 1996), it was stressed that:

What is essential is the nature of the employee's function and not the
nomenclature or title given to the job which determines whether the
employee has rank-and-file or managerial status or whether he is a
supervisory employee.[27]

WHEREFORE, the petition is hereby DENIED. Let the records of the case
be remanded to the office of origin, the Mediation-Arbitration Unit, Regional
Branch No. IV, for the immediate conduct of a certification election subject
to the usual pre-election conference.

SO ORDERED.
PEPSI - COLA PRODUCTS PHILIPPINES, INC. vs. HONORABLE This provision of law does not prohibit a local union composed of
SECRETARY OF LABOR supervisory employees from being affiliated to a federation which has local
G.R. No. 96663 | 1999-08-10 unions with rank-and-file members as affiliates.
PURISIMA, J.:
xxx xxx xxx
These are petitions for certiorari relating to three (3) cases filed with the Med-
Arbiter, to wit: MED ARB ROX Case No. R100-9101-RU-002 for xxx the Petition to Cancel, Revoke or Set Aside the Charter Certificate of the
Certification Election filed by Pepsi Cola Supervisors Union-UOEF (Union), private respondent is anchored on the alleged ground that certain managerial
MED ARB Case No. R1000-9102-RU-008, Re: Petition to Set Aside, Cancel employees are included as members thereof. The grounds for the cancellation
and/ or Revoke the Charter Affiliation of the Union, and MED-ARB ROX of the registration certificate of a labor organization are provided in Section
Case No. R1000-9104-RU-012, for Cancellation of Registration Certificate 7 of Rule II, Book V of the Omnibus Rules Implementing the Labor Code,
No. 11492-LC in favor of the Union. and the inclusion of managerial employees is not one of the grounds. xxx (in
this case, the private respondent herein) remains to be a legitimate labor
G. R. No. 96663 organization."1 [Pepsi - Cola Supervisory Employees Organization - UOEF,
Comment, pp. 4-6, Rollo, pp. 71-73.]
The facts that matter can be culled as follows:
On April 8, 1991, the Secretary of Labor and Employment, through the
Sometime in June 1990, the Pepsi-Cola Employees Organization-UOEF Office of the Solicitor General, sent in a Comment, alleging inter alia, that:
(Union) filed a petition for certification election with the Med-Arbiter
seeking to be the exclusive bargaining agent of supervisors of Pepsi-Cola "xxx under Article 259 of the New Labor Code, only orders of the Med-
Philippines, Inc. (PEPSI). Arbiter can be appealed through the Secretary of Labor and only on the
ground that the rules and regulations for the conduct of the certification
On July 12, 1990, the Med-Arbiter granted the Petition, with the explicit election have been violated. The Order of the Representation Officer is
statement that it was an affiliate of Union de Obreros Estivadores de Filipinas "interlocutory" and not appealable.
(federation) together with two (2) rank and file unions, Pepsi-Cola Labor
Unity (PCLU) and Pepsi-Cola Employees Union of the Philippines (PEUP). xxx until and unless there is a final order cancelling its certificate of
registration or charter certificate, a labor organization remains to be a
On July 23, 1990, PEPSI filed with the Bureau of Labor Relations a petition legitimate labor organization entitled to exercise all the rights and duties
to Set Aside, Cancel and/or Revoke Charter Affiliation of the Union, entitled accorded to it by the Labor Code including the right to be certified as a
PCPPI v. PCEU-UOEF and docketed as Case No. 725-90, on the grounds bargaining representative.
that (a) the members of the Union were managers and (b) a supervisors' union
can not affiliate with a federation whose members include the rank and file xxx Public respondent cannot be deemed to have committed grave abuse of
union of the same company. discretion with respect to an issue that was never presented before it for
resolution. xxx
On August 29,1990, PEPSI presented a motion to re-open the case since it
was not furnished with a copy of the Petition for Certification Election. Article 245 of the New Labor Code does not preclude the supervisor's union
and the rank-and-file union from being affiliated with the same federation.
On September 4, 1990, PEPSI submitted its position paper to the BLR in
Case No. 725-90. xxx xxx xxx

On September 21, 1990, PEPSI received summons to appear at the pre-trial A federation of local union is not the labor organization referred to in Article
conference set on September 25, 1990 but which the hearing officer 245 but only becomes entitled to all the rights enjoyed by the labor
rescheduled on October 21, 1990. organization (at the company level) when it has complied with the
registration requirements found in Articles 234 and 237. Hence, what is
On October 12, 1990, PEPSI filed a Notice of Appeal and Memorandum of prohibited by Article 245 is membership of supervisory employees in a labor
Appeal with the Secretary of Labor, questioning the setting of the union (at the company level) of the rank and file. xxx
certification election on the said date and five (5) days after. It also presented
an urgent Ex-Parte Motion to Suspend the Certification Election, which xxx In other words, the affiliation of the supervisory employee's union with
motion was granted on October 18, 1990. the same federation with which the rank and file employees union is affiliated
did not make the supervisory employees members of the rank and file
On November 12, 1990, the Secretary of Labor denied the appeal and Motion employee's union and vice versa."2 [Rollo, pp. 86-89, 92.]
for Reconsideration. Even as the Petition to Cancel, Revoke and Suspend
Union Charter Certificate was pending before the BLR, PEPSI found its way PEPSI, in its Reply dated May 7, 1991, asserted:
to this Court via the present petition for certiorari.
"It is our humble contention that a final determination of the Petition to Set-
On February 6, 1991, the Court granted the prayer for temporary restraining Aside, Cancel, Revoke Charter Union Affiliation should first be disposed of
order and/or preliminary injunction. before granting the Petition for the Conduct of Certification Election. To
allow the conduct of the certification election to proceed would make any
The pivot of inquiry here is: whether or not a supervisors' union can affiliate decision arrived at by the Bureau of Labor Relations useless inasmuch as the
with the same Federation of which two (2) rank and file unions are likewise same would necessarily be rendered moot and academic."3 [Rollo, p. 104.]
members, without violating Article 245 of the Labor Code (PD 442), as
amended, by Republic Act 6715, which provides: On June 7, 1991, petitioner again filed a Supplemental Reply stressing:

"Art. 245. Ineligibility of managerial employees to join any labor "It is likewise stressed that officials of both the PCLU and PEUP are top
organization; right of supervisory employees.- Managerial employees are not ranking officers of UOEF, the federation of supervisors' union, to wit:
eligible to join, assist or form any labor organization. Supervisory employees
shall not be eligible for membership in a labor organization of the rank-and- POSITION IN RANK AND FILE UNION POSITION IN FEDERATION
file employees but may join, assist or form separate labor organizations of
their own." 1. Rogelio de la Cruz PCLU -President General Vice President
2. Felix Gatela PEUP - President General Treasurer
In its Comment dated March 19, 1991, the Federation argued that: 3. Carlito Epino PCLU Board Member Educational Research Director

"The pertinent portion of Article 245 of the Labor Code states that. xxx xxx xxx
"Supervisory employees shall not be eligible for membership in a labor
organization of the rank and file employees but may join, assist or form The respondent supervisory union could do indirectly what it could not do
separate labor organization of their own." directly as the simple expedient of affiliating with UOEF would negate the
manifest intent and letter of the law that supervisory employees can only
"join, assist or form separate labor organizations of their own" and cannot the aforementioned payroll during the pre-election conference for
"be eligible for membership in a labor organization of the rank and file verification purposes."6 [OSG Comment, pp. 3 -4, Rollo, pp. 145 - 146.]
employees."4 [Rollo, p.110.]
The supervisory employees of the Union are:
On August 6, 1991, the Secretary of Labor and Employment filed a ÂÂ
Rejoinder, claiming thus: POSITION

"xxx an employer has no legal standing to question the validity of a 1. Felipe Valdehueza Route Manager
certification election.
2. Gerberto Vertudazo C & C Manager
xxx For this reason, the Supreme Court has consistently held that, as a rule,
a certification election is the sole and exclusive concern of the employees 3. Paul Mendoza Sales Service Department Manager
and that the employer is definitely an intruder or a mere bystander
(Consolidated Farms vs. Noriel, L-47752, July 31, 1978, 84 SCRA 469; 4. Gilberto Emano, Jr. Route Manager
Filipino Metals Corporation vs. Ople, L- 43861, September 4, 1981, 107
SCRA 211; Trade Unions of the Philippines and Allied Services (TUPAS) 5. Jaime Huliganga Chief Checker
vs. Trajano No. L-61153, January 17, 1983, 120 SCRA 64].
6. Elias Edgama, Sr. Accounting Manager
xxx xxx xxx
7. Romanico Ramos Route Manager
In Adamson & Adamson, Inc. vs. CIR No. L-35120, January 31, 1984, 127
SCRA 268, the Supreme Court (then dealing with the interpretation of 8. Raul Yacapin Route Manager
Section 3 of the Industrial Peace Act, from which Section 245 of the Labor
Code was derived) grappled with the issue in the case at bar. It held that, 9. Jovenal Albaque Route Manager

'There is nothing in the provisions of the Industrial Peace Act which provides 10. Fulvio Narciso Route Manager
that a duly registered local union affiliating with a national union or
federation loses its legal personality, or its independence.'
11. Apolinario Opiniano Route Manager
xxx xxx xxx
12. Alfredo Panas Route Manager
However, there is absolutely nothing in the Labor Code that prohibits a
federation from representing or exercising influence over its affiliates. On the 13. Simplicio Nelie Route Manager
contrary, this is precisely the reason why federations are formed and are
allowed by law to exist."5 [Rejoinder, pp. 2,3,10,14; Rollo, pp. 125,126, 14. Arthur Rodriguez Route Manager
133,137.]
15. Marco Ilano Warehouse Operations Manager and
On November 8, 1991, the Union also filed a Rejoinder.
16. Deodoro Ramos Maintenance Manager
On December 9, 1991, the Court resolved to DISMISS the case for "failure
to sufficiently show that the questioned judgment is tainted with grave abuse
of discretion." On June 6, 1991, PEPSI appealed the said Order to the Secretary of Labor
and Employment on the ground of grave abuse of discretion, docketed as
In a Resolution dated March 2, 1992, the Second Division of the Court Case No. OS-A-232-91.
resolved to grant the motion for reconsideration interposed on January 28,
1992. On October 4, 1991, the Secretary modified the appealed decision, ruling
thus:
G.R. No. 103300
"WHEREFORE, the Order of the Med-Arbiter dated 23 May 1991 is hereby
What are assailed in this case is Med-Arbiter Order dated May 23, 1991 and modified to the effect that MED ARB ROX Case No. R1000-9104-RU-012
the Decision and Order of the Secretary of Labor and Employment, dated and R1000-9102-RU-008 are hereby referred to the Office of the Regional
October 4, 1991 and December 12, 1991, respectively. Director which has jurisdiction over these cases. The call for certification
election among the supervisory workers of the Pepsi-Cola Products
The decretal portion of the Med-Arbiter Order under attack, reads: Philippines, Inc. at its plant at Tin-ao, Cagayan de Oro City is hereby
sustained."7 [OSG Comment, p. 5, Rollo, p. 147.]
"WHEREFORE, premises considered, an order is hereby issued:
On October 19, 1991, PEPSI presented a motion for reconsideration of the
1. Dismissing MED ARB ROX CASE NO. R1000-919104-RU-012 and aforesaid Order but the same was denied on December 12, 1991.
R1000-9102-RU-008 for lack of merit; and
Meanwhile, the BLR issued Registration Certificate No. 11492-LC in favor
2. Ordering the conduct of a Certification Election to be participated by and of the Union. Dissatisfied therewith, PEPSI brought the instant petition for
among the supervisory workers of the respondent company, Pepsi-Cola certiorari, contending that:
Products Philippines, Inc. at its plant at Tin-ao, Cagayan de Oro City,
including all the satellite warehouse within the territorial coverage and "PUBLIC RESPONDENT COMMITTED GRAVE ABUSE OF
control of the Cagayan de Oro Pepsi-Cola Plant. The choices are as follows: DISCRETION IN RULING THAT PRIVATE RESPONDENT'S
OFFICERS AND MEMBERS ARE NOT MANAGERIAL EMPLOYEES;
1. Cagayan de Oro Pepsi-Cola Supervisors Union (U.O.E.P.)
PRIVATE RESPONDENT IS PROHIBITED FROM AFFILIATING
2. No union ITSELF WITH A FEDERATION ALREADY AFFILIATED WITH THE
RANK AND FILE UNION;
The parties are directed to attend a pre-election conference on June 10, 1991,
2:30 p.m. at the Regional Office to determine the qualification of the voters PUBLIC RESPONDENT COMMITTED GRAVE OF (SIC) ABUSE OF
and to thresh out the mechanics of the election. Respondent/employer is DISCRETION IN RULING THAT THE INSTITUTION OF A PETITION
directed to submit five (5) copies of the names of the rank and file workers FOR CANCELLATION OF UNION REGISTRATION DOES NOT
taken from the payroll on October 1-31, 1991, alphabetically arranged (sic) CONSTITUTE A PREJUDICIAL QUESTION TO A PETITION
indicating their names and positions and dates of employment and to bring CERTIFICATION ELECTION."8 [Petition, pp. 8, 13, 14; Rollo, pp. 9, 14,
15.]
examination of the sufficiency of the petition as well as a careful look into
The petitions must fail for want of merit. the arguments contained in the position papers and other documents.

At the outset, it must be stressed that on September 1, 1992, there was a "At any rate, the Court applies the established rule correctly followed by the
Resolution of the Union withdrawing from the Federation, to wit: public respondent that an order to hold a certification election is proper
despite the pendency of the petition for cancellation of the registration
"BE IT RESOLVED, as it is hereby RESOLVED, that this UNION certificate of the respondent union. The rationale for this is that at the time
WITHDRAW, as it hereby WITHDRAWS its affiliation from the Union de the respondent union filed its petition, it still had the legal personality to
Obreros Estivadores de Filipinas, and at the same time, give our thanks to the perform such act absent an order directing the cancellation.
said federation for its help and guidance rendered to this Union in the past."9
[Annex I, Rollo, p. 213.] xxx xxx xxx"

The issue in G.R. No. 96663, whether or not the supervisors union can be As regards the issue of whether or not confidential employees can join the
affiliated with a Federation with two (2) rank and file unions directly under labor union of the rank and file, what was held in the case of National
the supervision of the former, has thus become moot and academic in view Association of Trade Unions (NATU) - Republic Planters Bank Supervisors
of the Union's withdrawal from the federation. Chapter vs. Hon. R. D. Torres, et. al., G.R. No. 93468, December 29, 1994,
applies to this case. Citing Bulletin Publishing Corporation vs. Sanchez, 144
In a long line of cases (Narciso Nakpil, et. al., vs. Hon. Crisanto Aragon, et. SCRA 628,635, Golden Farms vs. NLRC, 175 SCRA 471, and Pier 8
al.,, G. R. No. L - 24087, January 22, 1980, 95 SCRA 85; Toribio v. Bidin, Arrastre and Stevedoring Services, Inc. vs. Hon. Nieves Roldan-Confessor
et. al., G.R. No. L-37960, February 28, 1980, 96 SCRA 361; Gumaua v. et al., G.R. No. 110854, February 14, 1995, the Court ruled:
Espino, G.R. No. L- 36188 - 37586 February 29, 1980, 96 SCRA 402), the
Court dismissed the petition for being moot and academic. In the case of F. "xxx A confidential employee is one entrusted with confidence on delicate
C. Fisher v. Yangco Steamship Co., March 31, 1915, the Court held: matters, or with the custody, handling, or care and protection of the
employer's property. While Art. 245 of the Labor Code singles out
"It is unnecessary, however to indulge in academic discussion of a moot managerial employee as ineligible to join, assist or form any labor
question. organization, under the doctrine of necessary implication, confidential
employees are similarly disqualified. This doctrine states that what is implied
xxx The action would have been dismissed at any time on a showing of the in a statute is as much a part thereof as that which is expressed, as elucidated
facts as they were . The question left for the court was a moot one. Its in several case; the latest of which is Chua v. Civil Service Commission
Resolution would have been useless. Its judgment would have been where we said:
impossible of execution xxx."
No statute can be enacted that can provide all the details involved in its
However, in the case of University of San Agustin, Inc., et al. vs. Court of application. There is always an omission that may not meet a particular
Appeals, et al., the court resolved the case, ruling that "even if a case were situation. What is thought, at the time of the enactment, to be an all embracing
moot and academic, a statement of the governing principle is appropriate in legislation maybe inadequate to provide for the unfolding events of the
the resolution of dismissal for the guidance not only of the parties but of future. So-called gaps in the law develop as the law is enforced. One of the
others similarly situated. xxx"10 [230 SCRA 761, 770, citing Eastern rules of statutory construction used to fill in the gap is the doctrine of
Broadcasting Corporation (DYRE) vs. Dans, etc, et al., 137 SCRA 628.] necessary implication xxx, Every statute is understood, by implication, to
contain all such provisions as may be necessary to effectuate its object and
In Atlas Lithographic Services, Inc. v. Laguesma, 205 SCRA 12, [1992] purpose, or to make effective rights, powers, privileges or jurisdiction which
decided by the Third Division with J. Gutierrez, Jr., as ponente and JJ. it grants, including all such collateral and subsidiary consequences as may be
Feliciano, Bidin, Romero and now Chief Justice Davide, Jr., as members it fairly and logically inferred from its terms.
was ratiocinated:
In applying the doctrine of necessary implication, we took into consideration
" xxx xxx xxx the rationale behind the disqualification of managerial employees expressed
in Bulletin Publishing Corporation v. Sanchez, thus "xxx if these managerial
Thus, if the intent of the law is to avoid a situation where supervisors would employees would belong to or be affiliated with a Union, the latter might not
merge with the rank-and-file or where the supervisors' labor organization be assured of their loyalty to the Union in view of evident conflict of interests.
would represent conflicting interests, then a local supervisors' union should The Union can also become company - dominated with the presence of
not be allowed to affiliate with the national federation of union of rank-and- managerial employees in Union membership." Stated differently, in the
file employees where that federation actively participates in union activity in collective bargaining process, managerial employees are supposed to be on
the company. the side of the employer, to act as its representatives, and to see to it that its
interest are well protected. The employer is not assured of such protection if
xxx xxx xxx these employees themselves are union members. Collective bargaining in
such a situation can become one-sided. It is the same reason that impelled
The prohibition against a supervisors' union joining a local union of rank and this Court to consider the position of confidential employees as included in
file is replete with jurisprudence. The Court emphasizes that the limitation is the disqualification found in Art. 245 as if the disqualification of confidential
not confined to a case of supervisors' wanting to join a rank-and-file union. employees were written in the provision. If confidential employees could
The prohibition extends to a supervisors' local union applying for unionize in order to bargain for advantages for themselves, then they could
membership in a national federation the members of which include local be governed by their own motives rather than the interest of the employers.
unions of rank and file employees. The intent of the law is clear especially Moreover, unionization of confidential employees for the purpose of
where, as in this case at bar, the supervisors will be co-mingling with those collective bargaining would mean the extension of the law to persons or
employees whom they directly supervise in their own bargaining unit." individuals who are supposed to act "in the interest of" the employers. It is
not farfetched that in the course of collective bargaining, they might
Anent the issue of whether or not the Petition to cancel/revoke registration is jeopardize that interest which they are duty bound to protect. Along the same
a prejudicial question to the petition for certification election, the following line of reasoning we held in Golden Farms, Inc. vs. Ferrer-Calleja reiterated
ruling in the case of Association of the Court of Appeals Employees (ACAE) in Philips Industrial Development, Inc., NLRC, that "confidential employees
vs. Hon. Pura Ferrer-Calleja, in her capacity as Director, Bureau of Labor such as accounting personnel, radio and telegraph operators who, having
Relations et. Al., 203 ACRA 597, 598, [1991], is in point, to wit: access to confidential information, may become the source of undue
advantage. Said employee(s) may act as spy or spies of either party to a
xxx It is a well-settled rule that "a certification proceedings is not a litigation collective bargaining agreement."
in the sense that the term is ordinarily understood, but an investigation of a
non-adversarial and fact finding character." (Associated Labor Unions The Court finds merit in the submission of the OSG that Route Managers,
(ALU) v. Ferrer-Calleja, 179 SCRA 127 [1989]; Philippine Telegraph and Chief Checkers and Warehouse Operations Managers are supervisors while
Telephone Corporation v. NLRC, 183 SCRA 451 [1990]. Thus, the technical Credit & Collection Managers and Accounting Managers are highly
rules of evidence do not apply if the decision to grant it proceeds from an confidential employees. Designation should be reconciled with the actual job
description of subject employees. A careful scrutiny of their job description
indicates that they don't lay down company policies. Theirs is not a final
determination of the company policies since they have to report to their
respective superior. The mere fact that an employee is designated manager
does not necessarily make him one. Otherwise, there would be an absurd
situation where one can be given the title just to be deprived of the right to
be a member of a union. In the case of National Steel Corporation v.
Laguesma, G. R. No. 103743, January 29,1996, it was stressed that:

"What is essential is the nature of the employee's function and not the
nomenclature or title given to the job which determines whether the
employee has rank and file or managerial status, or whether he is a
supervisory employee."

WHEREFORE, the petitions under consideration are DISMISSED but


subject Decision, dated October 4, 1991, of the Secretary of Labor and
Employment is MODIFIED in that Credit and Collection Managers and
Accounting Managers are highly confidential employees not eligible for
membership in a supervisors' union. No pronouncement as to costs.

SO ORDERED.
MANILA ELECTRIC COMPANY vs. THE HON. SECRETARY OF
LABOR AND EMPLOYMENT ". . ." (p. 19, Rollo).
G.R. No. 91902 | 1991-05-20
MEDIALDEA, J.: MERALCO moved for the dismissal of the petition on the following
grounds:
This petition seeks to review the Resolution of respondent Secretary of Labor
and Employment Franklin M. Drilon dated November 3, 1989 which I
affirmed an Order of Med-Arbiter Renato P. Parungo (Case No. NCR-O-D- The employees sought to be represented by petitioner are either 1)
M-1-70), directing the holding of a certification election among certain managerial who are prohibited by law from forming or joining supervisory
employees of petitioner Manila Electric Company (hereafter "MERALCO") union; 2) security services personnel who are prohibited from joining or
as well as the Order dated January 16, 1990 which denied the Motion for assisting the rank-and-file union; 3) secretaries who do not consent to the
Reconsideration of MERALCO. petitioner's representation and whom petitioner can not represent; and 4)
rank-and-file employees represented by the certified or duly recognized
The facts are as follows: bargaining representative of the only rank-and-file bargaining unit in the
company, the Meralco Employees Workers Association (MEWA), in
On November 22, 1988, the Staff and Technical Employees Association of accordance with the existing Collective Bargaining Agreement with the
MERALCO (hereafter "STEAM-PCWF") a labor organization of staff and latter.
technical employees of MERALCO, filed a petition for certification election,
seeking to represent regular employees of MERALCO who are: (a) non- II
managerial employees with Pay Grades VII and above; (b) non-managerial The petition for certification election will disturb the administration of the
employees in the Patrol Division, Treasury Security Services Section, existing Collective Bargaining Agreement in violation of Art. 232 of the
Secretaries who are automatically removed from the bargaining unit; and (c) Labor Code.
employees within the rank and file unit who are automatically disqualified
from becoming union members of any organization within the same III
bargaining unit. The petition itself shows that it is not supported by the written consent of at
least twenty percent (20%) of the alleged 2,500 employees sought to be
Among others, the petition alleged that "while there exists a duly-organized represented. (Resolution, Sec. of Labor, pp. 223-224, Rollo)
union for rank and file employees in Pay Grade I-VI, which is the
MERALCO Employees and Worker's Association (MEWA) which holds a Before Med-Arbiter R. Parungo, MERALCO contended that employees from
valid CBA for the rank and file employees 1 , there is no other labor Pay Grades VII and above are classified as managerial employees who, under
organization except STEAM-PCWF claiming to represent the MERALCO the law, are prohibited from forming, joining or assisting a labor organization
employees. of the rank and file. As regards those in the Patrol Division and Treasury
Security Service Section, MERALCO maintains that since these employees
The petition was premised on the exclusion/disqualification of certain are tasked with providing security to the company, they are not eligible to
MERALCO employees pursuant to Art. I, Secs. 2 and 3 of the existing join the rank and file bargaining unit, pursuant to Sec. 2(c), Rule V, Book V
MEWA CBA as follows: of the then Implementing Rules and Regulations of the Labor Code (1988)
which reads as follows:
"ARTICLE I
"Sec. 2. Who may file petition. The employer or any legitimate labor
SCOPE organization may file the petition.

xxx xxx xxx "The petition, when filed by a legitimate labor organization, shall contain,
among others:
SECTION 2. Excluded from the appropriate bargaining unit and therefore
outside the scope of this Agreement are: "xxx xxx xxx;

(a) Employees in Patrol Division; "(c) description of the bargaining unit which shall be the employer unit unless
circumstances otherwise require, and provided, further: that the appropriate
(b) Employees in Treasury Security Services Section; bargaining unit of the rank and file employees shall not include security
guards
(c) Managerial Employees; and (As amended by Sec. 6, Implementing Rules of EO 111).

(d) Secretaries. ". . ." (p. 111, Labor Code, 1988 Ed.)

Any member of the Union who may now or hereafter be assigned or As regards those rank and file employees enumerated in Sec. 3, Art. I,
transferred to Patrol Division or Treasury Security Services Section, or MERALCO contends that since they are already beneficiaries of the MEWA-
becomes Managerial Employee or a Secretary, shall be considered CBA, they may not be treated as a separate and distinct appropriate
automatically removed from the bargaining unit and excluded from the bargaining unit.
coverage of this agreement. He shall thereby likewise be deemed
automatically to have ceased to be member of the union, and shall desist from MERALCO raised the same argument with respect to employees sought to
further engaging in union activity of any kind. be represented by STEAM-PCWF, claiming that these were already covered
by the MEWA-CBA.
SECTION 3. Regular rank-and-file employees in the organization elements
herein below listed shall be covered within the bargaining unit, but shall be On March 15, 1989, the Med-Arbiter ruled that having been excluded from
automatically disqualified from becoming union members: the existing Collective Bargaining Agreement for rank and file employees,
these employees have the right to form a union of their own, except those
1. Office of the Corporate Secretary employees performing managerial functions. With respect to those
2. Corporate Staff Services Department employees who had resented their alleged involuntary membership in the
3. Managerial Payroll Office existing CBA, the Med-Arbiter stated that the holding of a certification
4. Legal Service Department election would allow them to fully translate their sentiment on the matter,
5. Labor Relations Division and thus directed the holding of a certification election. The dispositive
6. Personnel Administration Division portion of the Resolution provides as follows:
7. Manpower Planning & Research Division
8. Computer Services Department "WHEREFORE, premises considered, a certification election is hereby
9. Financial Planning & Control Department ordered conducted among the regular rank-and-file employees of
10. Treasury Department, except Cash Section 11. General Accounting MERALCO to wit:
Section.
"1. Non-managerial employees with Pay Grades VII and above; "II. THE EMPLOYEES FROM PAY GRADES VII AND ABOVE ARE
RANK-AND-FILE EMPLOYEES.
"2. Non-managerial employees of Patrol Division, Treasury Security
Services Section and Secretaries; and "III. THE SECURITY GUARDS OR PERSONNEL MAY BE LUMPED
TOGETHER WITH THE RANK-AND-FILE UNION AND/OR THE
"3. Employees prohibited from actively participating as members of the SUPERVISORY UNION." (p. 8, Rollo)
union.
On February 26, 1990, We issued a temporary restraining order (TRO)
within 20 days from receipt hereof, subject to the usual pre-election against the implementation of the disputed resolution.
conference with the following choices:
In its petition, MERALCO has relented and recognized respondents
"1. Staff and Technical Employees Association of MERALCO (STEAM- STEAM-PCWF and FLAMES' desired representation of supervisory
PCWF); employees from Grades VII up. However, it believes that all that the
Secretary of Labor has to do is to establish a demarcation line between
"2. No Union. supervisory and managerial rank, and not to classify outright the group of
employees represented by STEAM-PCWF and FLAMES as rank and file
"SO ORDERED." (p. 222, Rollo) employees.

On April 4, 1989, MERALCO appealed, contending that "until such time that In questioning the Secretary of Labor's directive allowing security guards
a judicial finding is made to the effect that they are not managerial employee, (Treasury/Patrol Services Section) to be represented by respondents,
STEAM-PCWF cannot represent employees from Pay Grades VII and above, MERALCO contends that this contravenes the provisions of the recently
additionally reiterating the same reasons they had advanced for disqualifying passed RA 6715 and its implementing rules (specifically par. 2, Sec. 1, Rule
respondent STEAM-PCWF. II, Book V) which disqualifies supervisory employees and security guards
from membership in a labor organization of the rank and file (p. 11, Rollo).
On April 7, 1989, MEWA filed an appeal-in-intervention, submitting as
follows: The Secretary of Labor's Resolution was obviously premised on the
provisions of Art. 212, then par. (k), of the 1988 Labor Code defining
A. The Order of the Med-Arbiter is null and void for being in violation of "managerial" and "rank and file" employees, the law then in force when the
Article 245 of the Labor Code; complaint was filed.

B. The Order of the Med-Arbiter violates Article 232 of the Labor Code; and At the time, only two groups of employees were recognized, the managerial
and rank and file. This explains the absence of evidence on job descriptions
C. The Order is invalid because the bargaining unit it delineated is not an on who would be classified managerial employees. It is perhaps also for this
appropriated (sic) bargaining unit. reason why the Secretary of Labor limited his classification of the Meralco
employees belonging to Pay Grades VII and up, to only two groups, the
On May 4, 1989, STEAM-PCWF opposed the appeal-in-intervention. managerial and rank and file.

With the enactment of RA 6715 and the rules and regulations implementing However, pursuant to the Department of Labor's goal of strenghthening the
the same, STEAM-PCWF renounced its representation of the employees in constitutional right of workers to self-organization, RA 6715 was
Patrol Division, Treasury Security Services Section and rank-and-file subsequently passed which reorganized the employee-ranks by including a
employees in Pay Grades. third group, or the supervisory employees, and laying down the distinction
between supervisory employees and those of managerial ranks in Art. 212,
On September 13, 1989, the First Line Association of Meralco Supervisory renumbered par. [m], depending on whether the employee concerned has the
Employees. (hereafter FLAMES) filed a similar petition (NCR-OD-M-9- power to lay down and execute management policies, in the case of
731-89) seeking to represent those employees with Pay Grades VII to XIV, managerial employees, or merely to recommend them, in case of supervisory
since "there is no other supervisory union at MERALCO." (p. 266, Rollo). employees.
The petition was consolidated with that of STEAM-PCWF. In this petition, MERALCO has admitted that the employees belonging to
Pay Grades VII and up are supervisory (p. 10, Rollo). The records also show
On November 3, 1989, the Secretary of Labor affirmed with modification, that STEAM-PCWF had "renounced its representation of the employees in
the assailed order of the Med-Arbiter, disposing as follows: Patrol Division, Treasury Security Service Section and rank and file
employees in Pay Grades I-VI" (p. 6, Rollo); while FLAMES, on the other
"WHEREFORE, premises considered, the Order appealed from is hereby hand, had limited its representation to employees belonging to Pay Grades
affirmed but modified as far as the employees covered by Section 3, Article VII-XIV, generally accepted as supervisory employees, as follows:
I of the existing CBA in the Company are concerned. Said employees shall
remain in the unit of the rank-and file already existing and may exercise their "It must be emphasized that private respondent First Line Association of
right to self organization as above enunciated. Meralco Supervisory Employees seeks to represent only the Supervisory
Employees with Pay Grades VII to XIV.
"Further, the First Line Association of Meralco Supervisory Employees
(FLAMES) is included as among the choices in the certification election. "Supervisory Employees with Pay Grades VII to XIV are not managerial
"Let, therefore, the pertinent records of the case be immediately forwarded employees. In fact the petition itself of petitioner Manila Electric Company
to the Office of origin for the conduct of the certification election. on page 9, paragraph 3 of the petition stated as follows, to wit:

"SO ORDERED." (p. 7, Rollo) 'There was no need for petitioner to prove that these employees are not rank-
and-file. As adverted to above, the private respondents admit that these are
MERALCO's motion for reconsideration was denied on January 16, 1990. not the rank-and-file but the supervisory employees, whom they seek to
represent. What needs to be established is the rank where supervisory ends
On February 9, 1990, MERALCO filed this petition, premised on the and managerial begins.'
following ground:
and First Line Association of Meralco Supervisory Employees herein states
"RESPONDENT SECRETARY ACTED WITH GRAVE ABUSE OF that Pay Grades VII to XIV are not managerial employees. In fact, although
DISCRETION AND/OR IN EXCESS OF JURISDICTION AMOUNTING employees with Pay Grade XV carry the Rank of Department Managers,
TO LACK OF JURISDICTION IN RULING THAT: these employees only enjoys (sic) the Rank Manager but their
recommendatory powers are subject to evaluation, review and final action by
"I. ANOTHER RANK-AND-FILE BARGAINING UNIT CAN BE the department heads and other higher executives of the company."
ESTABLISHED INDEPENDENT, DISTINCT AND SEPARATE FROM (FLAMES' Memorandum, p. 305, Rollo).
THE EXISTING RANK-AND-FILE BARGAINING UNIT.
Based on the foregoing, it is clear that the employees from Pay Grades VII
and up have been recognized and accepted as supervisory. On the other hand, 'Sec. 1. . . .
those employees who have been automatically disqualified have been
directed by the Secretary of Labor to remain in the existing labor organization "Sec. 2. Who may file. Any legitimate labor organization or the employer,
for the rank and file, (the condition in the CBA deemed as not having been when requested to bargain collectively, may file the petition.
written into the contract, as unduly restrictive of an employee's exercise of
the right to self-organization). We shall discuss the rights of the excluded "The petition, when filed by a legitimate labor-organization shall contain,
employees (or those covered by Sec. 2, Art. I, MEWA-CBA later. among others:

Anent the instant petition therefore, STEAM-PCWF, and FLAMES would "(a) . . .
therefore represent supervisory employees only. In this regard, the authority
given by the Secretary of Labor for the establishment of two labor "(b) . . .
organizations for the rank and file will have to be disregarded since We
hereby uphold certification elections only for supervisory employees from "(c) description of the bargaining unit which shall be the employer unit unless
Pay Grade VII and up, with STEAM-PCWF and FLAMES as choices. circumstances otherwise require; and provided further, that the appropriate
bargaining unit of the rank-and-file employees shall not include supervisory
As to the alleged failure of the Secretary of Labor to establish a demarcation employees and/or security guards;
line for purposes of segregating the supervisory from the managerial
employees, the required parameter is really not necessary since the law itself, ". . ."
Art. 212-m, (as amended by Sec. 4 of RA 6715) has already laid down the
corresponding guidelines: Both rules, barring security guards from joining a rank and file organization,
appear to have been carried over from the old rules which implemented then
"Art. 212. Definitions . . . Art. 245 of the Labor Code, and which provided thus:

"(m) 'Managerial employee' is one who is vested with powers or prerogatives "Art. 245. Ineligibility of security personnel to join any labor organization.
to lay down and execute management policies and/or to hire, transfer, Security guards and other personnel employed for the protection and security
suspend, lay-off, recall, discharge, assign or discipline employees. of the person, properties and premises of the employer shall not be eligible
Supervisory employees are those who, in the interest of the employer, for membership in any labor organization."
effectively recommend such managerial actions if the exercise of such
authority is not merely routinary or clerical in nature but requires the use of On December 24, 1986, Pres. Corazon C. Aquino issued E.O No. 111 which
independent judgment. All employees not falling within any of the above eliminated the above-cited provision on the disqualification of security
definitions are considered rank-and-file employees for purposes of this guards.
Book."
What was retained was the disqualification of managerial employees,
In his resolution, the Secretary of Labor further elaborated: renumbered as Art. 245 (previously Art. 246), as follows:

". . . Thus, the determinative factor in classifying an employee as managerial, "ART. 245. Ineligibility of managerial employees to joint any labor
supervisory or rank-and-file is the nature of the work of the employee organization. Managerial employees are not eligible to join, assist or form
concerned. any labor organization."

"In National Waterworks and Sewerage Authority vs. National Waterworks With the elimination, security guards were thus free to join a rank and file
and Sewerage Authority Consolidated Unions (11 SCRA 766) the Supreme organization.
Court had the occasion to come out with an enlightening dissertation of the
nature of the work of a managerial employees as follows: On March 2, 1989, the present Congress passed RA 6715. 2 Section 18
thereof amended Art. 245, to read as follows:
'. . . that the employee's primary duty consists of the management of the
establishment or of a customarily recognized department or subdivision "Art. 245. Ineligibility of managerial employees to join any labor
thereof, that he customarily and regularly directs the work of other employees organization; right of supervisory employees. Managerial employees are not
therein, that he has the authority to hire or discharge other employees or that eligible to join, assist or form any labor organization. Supervisory employees
his suggestions and recommendations as to the hiring and discharging and or shall not be eligible for membership in a labor organization of the rank-and-
to the advancement and promotion or any other change of status of other file employees but may join, assist, or form separate labor organizations of
employees are given particular weight, that he customarily and regularly their own."
exercises discretionary powers . . . (56 CJS, pp. 666-668.)'" (p. 226, Rollo)
As will be noted, the second sentence of Art. 245 embodies an amendment
We shall now discuss the rights of the security guards to self-organize. disqualifying supervisory employees from membership in a labor
organization of the rank-and-file employees. It does not include security
MERALCO has questioned the legality of allowing them to join either the guards in the disqualification.
rank and file or the supervisory union, claiming that this is a violation of par.
2, Sec. 1, Rule II, Book V of the Implementing Rules of RA 6715, which The implementing rules of RA 6715, therefore, insofar as they disqualify
states as follows: security guards from joining a rank and file organization are null and void,
for being not germane to the object and purposes of EO 111 and RA 6715
"Sec. 1. Who may join unions . . . . upon which such rules purportedly derive statutory moorings. In Shell
Philippines, Inc. vs. Central Bank, G.R. No. 51353, June 27, 1988, 162
"xxx xxx xxx SCRA 628, We stated:

"Supervisory employees and security guards shall not be eligible for The rule-making power must be confined to details for regulating the mode
membership in a labor organization of the rank-and-file employees but may or proceeding to carry into effect the law as it has been enacted. The power
join, assist or form separate labor organizations of their own; . . . cannot be extended to amending or expanding the statutory requirements or
to embrace matters not covered by the statute. Rules that subvert the statute
". . ." cannot be sanctioned." (citing University of Sto. Tomas vs. Board of Tax
Appeals, 93 Phil. 376).
Paragraph 2, Sec. 1, Rule II, Book V, is similar to Sec. 2 (c) Rule V, also of
Book V of the implementing rules of RA 6715: While therefore under the old rules, security guards were barred from joining
a labor organization of the rank and file, under RA 6715, they may now freely
"Rule V. join a labor organization of the rank and file or that of the supervisory union,
depending on their rank. By accommodating supervisory employees, the
REPRESENTATION CASES AND INTERNAL-UNION CONFLICTS Secretary of Labor must likewise apply the provisions of RA 6715 to security
guards by favorably allowing them free access to a labor organization,
whether rank and file or supervisory, in recognition of their constitutional
right to self-organization.

We are aware however of possible consequences in the implementation of


the law in allowing security personnel to join labor unions within the
company they serve. The law is apt to produce divided loyalties in the faithful
performance of their duties. Economic reasons would present the employees
concerned with the temptation to subordinate their duties to the allegiance
they owe the union of which they are members, aware as they are that it is
usually union action that obtains for them increased pecuniary benefits.

Thus, in the event of a strike declared by their union, security personnel may
neglect or outrightly abandon their duties, such as protection of property of
their employer and the persons of its officials and employees, the control of
access to the employer's premises, and the maintenance of order in the event
of emergencies and untoward incidents.

It is hoped that the corresponding amendatory and/or suppletory laws be


passed by Congress to avoid possible conflict of interest in security
personnel.

ACCORDINGLY, the petition is hereby DISMISSED. We AFFIRM with


modification the Resolution of the Secretary of Labor dated November 3,
1989 upholding an employee's right to self-organization. A certification
election is hereby ordered conducted among supervisory employees of
MERALCO, belonging to Pay Grades VII and above, using as guideliness an
employee's power to either recommend or execute management policies,
pursuant to Art. 212 (m), of the Labor Code, as amended by Sec. 4 of RA
6715, with respondents STEAM-PCWF and FLAMES as choices.

Employees of the Patrol Division, Treasury Security Services Section and


Secretaries may freely join either the labor organization of the rank and file
or that of the supervisory union depending on their employee rank.
Disqualified employees covered by Sec. 3, Art. I of the MEWA-CBA, shall
remain with the existing labor organization of the rank and file, pursuant to
the Secretary of Labor's directive:

"By the parties' own agreement, they find the bargaining unit, which includes
the positions enumerated in Section 3, Article I of their CBA, appropriate for
purposes of collective bargaining. The composition of the bargaining unit
should be left to the agreement of the parties, and unless there are legal
infirmities in such agreement, this Office will not substitute its judgment for
that of the parties. Consistent with the history of collective bargaining in the
company, the membership of said group of employees in the existing rank-
and-file unit should continue, for it will enhance stability in that unit already
well established. However, we cannot approve of the condition set in Section
3, Article I of the CBA that the employees covered are automatically
disqualified from becoming union members. The condition unduly restricts
the exercise of the right to self organization by the employees in question. It
is contrary to law and public policy and, therefore, should be considered to
have not been written into the contract. Accordingly, the option to join or not
to join the union should be left entirely to the employees themselves." (p.
229, Rollo)

The Temporary Restraining Order (TRO) issued on February 26, 1990 is


hereby LIFTED. Costs against petitioner.

SO ORDERED.
UNITED PEPSI-COLA SUPERVISORY UNION (UPSU) vs. interactions with its environment. Typical titles of top managers are "chief
LAGUESMA executive officer," "president," and "senior vice-president." Actual titles vary
G.R. No. 122226 | 1998-03-25 from one organization to another and are not always a reliable guide to
MENDOZA, J: membership in the highest management classification. 2

Petitioner is a union of supervisory employees. It appears that on March 20, As can be seen from this description, a distinction exists between those who
1995 the union filed a petition for certification election on behalf of the route have the authority to devise, implement and control strategic and operational
managers at Pepsi-Cola Products Philippines, Inc. However, its petition was policies (top and middle managers) and those whose task is simply to ensure
denied by the med-arbiter and, on appeal, by the Secretary of Labor and that such policies are carried out by the rank-and-file employees of an
Employment, on the ground that the route managers are managerial organization (first-level managers/supervisors). What distinguishes them
employees and, therefore, ineligible for union membership under the first from the rank-and-file employees is that they act in the interest of the
sentence of Art. 245 of the Labor Code, which provides: employer in supervising such rank-and-file employees.

Ineligibility of managerial employees to join any labor organization; right of "Managerial employees" may therefore be said to fall into two distinct
supervisory employees. - Managerial employees are not eligible to join, assist categories: the "managers" per se, who compose the former group described
or form any labor organization. Supervisory employees shall not be eligible above, and the "supervisors" who form the latter group. Whether they belong
for membership in a labor organization of the rank-and-file employees but to the first or the second category, managers, vis-a-vis employers, are,
may join, assist or form separate labor organizations of their own. likewise, employees. 3

Petitioner brought this suit challenging the validity of the order dated August The first question is whether route managers are managerial employees or
31, 1995, as reiterated in the order dated September 22, 1995, of the Secretary supervisors.
of Labor and Employment. Its petition was dismissed by the Third Division
for lack of showing that respondent committed grave abuse of discretion. But Previous Administrative Determinations of
petitioner filed a motion for reconsideration, pressing for resolution its the Question Whether Route Managers
contention that the first sentence of Art. 245 of the Labor Code, so far as it are Managerial Employees
declares managerial employees to be ineligible to form, assist or join unions,
contravenes Art. III, Sec 8 of the Constitution which provides: It appears that this question was the subject of two previous determinations
by the Secretary of Labor and Employment, in accordance with which this
The right of the people, including those employed in the public and private case was decided by the med-arbiter.
sectors, to form unions, associations, or societies for purposes not contrary
to law shall not be abridged. In Case No. OS-MA-10-318-91, entitled Worker's Alliance Trade Union
(WATU) v. Pepsi-Cola Products Philippines, Inc., decided on November 13,
For this reason, the petition was referred to the Court en banc. 1991, the Secretary of Labor found:

The Issues in this Case We examined carefully the pertinent job descriptions of the subject
employees and other documentary evidence on record vis-a-vis paragraph
Two questions are presented by the petition: (1) whether the route managers (m), Article 212 of the Labor Code, as amended, and we find that only those
at Pepsi-Cola Products Philippines, Inc. are managerial employees and (2) employees occupying the position of route manager and accounting manager
whether Art. 245, insofar as it prohibits managerial employees from forming, are managerial employees. The rest i.e. quality control manager,
joining or assisting labor unions, violates Art. III, Section 8 of the yard/transport manager and warehouse operations manager are supervisory
Constitution. employees.

In resolving these issues it would be useful to begin by defining who are To qualify as managerial employee, there must be a clear showing of the
"managerial employees" and considering the types of "managerial exercise of managerial attributes under paragraph (m), Article 212 of the
employees." Labor Code as amended. Designations or titles of positions are not
controlling. In the instant case, nothing on record will support the claim that
Types of Managerial Employees the quality control manager, yard/transport manager and warehouse
operations manager are vested with said attributes. The warehouse operations
The term "manager" generally refers to "anyone who is responsible for manager, for example, merely assists the plant finance manager in planning,
subordinates and other organizational resources." 1 As a class, managers organizing, directing and controlling all activities relative to development
constitute three levels of a pyramid: and implementation of an effective management control information system
at the sale offices. The exercise of authority of the quality control manager,
Top management on the other hand, needs the concurrence of the manufacturing manager.
Middle Management
First-Line Management (also called Supervisor) As to the route managers and accounting manager, we are convinced that
==================== they are managerial employees. Their job descriptions clearly reveal so.
Operatives or Operating Employees
On July 6, 1992, this finding was reiterated in Case No. OS-A-3-71-92,
FIRST-LINE MANAGERS - The lowest level in an organization at which entitled In Re: Petition for Direct Certification and/or Certification Election-
individuals are responsible for the work of others is called first-line or first- Route Managers/Supervisory Employees of Pepsi-Cola Products Phils. Inc.,
level management. First-line managers direct operating employees only; they as follows:
do not supervise other managers. Examples of first-line managers are the
"foreman" or production supervisor in a manufacturing plant, the technical The issue brought before us is not of first impression. At one time, we had
supervisor in a research department, and the clerical supervisor in a large the occasion to rule upon the status of route manager in the same company
office. First-level managers are often called supervisors. vis a vis the issue as to whether or not it is supervisory employee or a
managerial employee. In the case of Workers Alliance Trade Unions NATU)
MIDDLE MANAGERS - The term middle management can refer to more vs. Pepsi Cola Products, Phils., Inc. (OS-MA-A-10-318-91), 15 November
than one level in an organization. Middle managers direct the activities of 1991, we ruled that a route manager is a managerial employee within the
other managers and sometimes also those of operating employees. Middle context of the definition of the law, and hence, ineligible to join, form or
managers' principal responsibilities are to direct the activities that implement assist a union. We have once more passed upon the logic of our Decision
their organizations' policies and to balance the demands of their superiors aforecited in the light of the issues raised in the instant appeal, as well as the
with the capacities of their subordinates. A plant manager in an electronics available documentary evidence on hand, and have come to the view that
firm is an example of a middle manager. there is no cogent reason to depart from our earlier holding. Route Managers
are, by the very nature of their functions and the authority they wield over
TOP MANAGERS - Composed of a comparatively small group of their subordinates, managerial employees. The prescription found in Art. 245
executives, top management is responsible for the overall management of the of the Labor Code, as amended therefore, clearly applies to them. 4
organization. It establishes operating policies and guides the organization's
Citing our ruling in Nasipit Lumber Co. v. National Labor Relations
Commission, 5 however, petitioner argues that these previous administrative These then are your functions as Pepsi-Cola Route Manager. Within these
determinations do not have the effect of res judicata in this case, because functions - managing your job and managing your people - you are
"labor relations proceedings" are "non-litigious and summary in nature accountable to your District Manager for the execution and completion of
without regard to legal technicalities." 6 Nasipit Lumber Co. involved a various tasks and activities which will make it possible for you to achieve
clearance to dismiss an employee issued by the Department of Labor. The your sales objectives.
question was whether in a subsequent proceeding for illegal dismissal, the
clearance was res judicata. In holding it was not, this Court made it clear that B. PRINCIPAL ACCOUNTABILITIES
it was referring to labor relations proceedings of a non-adversary character,
thus: 1.0 MANAGING YOUR JOB

The requirement of a clearance to terminate employment was a creation of The Route Manager is accountable for the following:
the Department of labor to carry out the Labor Code provisions on security
of tenure and termination of employment. The proceeding subsequent to the 1.1 SALES DEVELOPMENT
filing of an application for clearance to terminate employment was outlined
in Book V, Rule XIV of the Rules and Regulations Implementing the Labor 1.1.1 Achieve the sales plan.
Code. The fact that said rule allowed a procedure for the approval of the 1.1.2 Achieve all distribution and new account objectives.
clearance with or without the opposition of the employee concerned (Secs. 7 1.1.3 Develop new business opportunities thru personal contacts with
& 8), demonstrates the non-litigious and summary nature of the proceeding. dealers.
The clearance requirement was therefore necessary only as an expeditious 1.1.4 Inspect and ensure that all merchandizing [sic] objectives are achieved
shield against arbitrary dismissal without the knowledge and supervision of in all outlets.
the Department of Labor. Hence, a duly approved clearance implied that the 1.1.5 maintain and improve productivity of all cooling equipment and kiosks.
dismissal was legal or for cause (Sec. 2). 7 1.1.6 Execute and control all authorized promotions.
1.1.7 Develop and maintain dealer goodwill.
But the doctrine of res judicata certainly applies to adversary administrative 1.1.8 Ensure all accounts comply with company suggested retail pricing.
proceedings. As early as 1956, in Brillantes v. Castro, 8 we sustained the 1.1.9 Study from time to time individual route coverage and productivity for
dismissal of an action by a trial court on the basis of a prior administrative possible adjustments to maximize utilization of resources.
determination of the same case by the Wage Administration Service,
applying the principle of res judicata. Recently, in Abad v. NLRC 9 we 1.2 Administration
applied the related doctrine of stare decisis in holding that the prior
determination that certain jobs at the Atlantic Gulf and Pacific Co. were 1.2.1 Ensure the proper loading of route trucks before check-out and the
project employments was binding in another case involving another group of proper sorting of bottles before check-in.
employees of the same company. Indeed, in Nasipit Lumber Co., this Court 1.2.2 Ensure the upkeep of all route sales reports and all other related reports
clarified toward the end of its opinion that "the doctrine of res judicata applies and forms required on an accurate and timely basis.
. . . to judicial or quasi judicial proceedings and not to the exercise of 1.2.3 Ensure proper implementation of the various company policies and
administrative powers." 10 Now proceedings for certification election, such procedures incl. but not limited to shakedown; route shortage; progressive
as those involved in Case No. OS-M-A-10-318-91 and Case No. OS-A-3-71- discipline; sorting; spoilages; credit/collection; accident; attendance.
92, are quasi judicial in nature and, therefore, decisions rendered in such 1.2.4 Ensure collection of receivables and delinquent accounts.
proceedings can attain finality. 11
2.0 MANAGING YOUR PEOPLE
Thus, we have in this case an expert's view that the employees concerned are
managerial employees within the purview of Art. 212 which provides: The Route Manager is accountable for the following:

(m) "managerial employee" is one who is vested with powers or prerogatives 2.1 Route Sales Team Development
to lay down and execute management policies and/or to hire, transfer,
suspend, lay off, recall, discharge, assign or discipline employees. 2.1.1 Conduct route rides to train, evaluate and develop all assigned route
Supervisory employees are those who, in the interest of the employer, salesmen and helpers at least 3 days a week, to be supported by required route
effectively recommend such managerial actions if the exercise of such ride documents/reports & back check/spot check at least 2 days a week to be
authority is not merely routinary or clerical in nature but requires the use of supported by required documents/reports.
independent judgment. All employees not falling within any of the above 2 1.2 Conduct sales meetings and morning huddles. Training should focus on
definitions are considered rank-and-file employees for purposes of this Book. the enhancement of effective sales and merchandizing [sic] techniques of the
salesmen and helpers. Conduct group training at least 1 hour each week on a
At the very least, the principle of finality of administrative determination designated day and of specific topic.
compels respect for the finding of the Secretary of Labor that route managers
are managerial employees as defined by law in the absence of anything to 2.2 Code of Conduct
show that such determination is without substantial evidence to support it.
Nonetheless, the Court, concerned that employees who are otherwise 2.2.1 Maintain the company's reputation through strict adherence to PCPPI's
supervisors may wittingly or unwittingly be classified as managerial code of conduct and the universal standards of unquestioned business ethics.
personnel and thus denied the right of self- organization, has decided to 12
review the record of this case.
Earlier in this opinion, reference was made to the distinction between
DOLE's Finding that Route Managers are managers per se (top managers and middle managers) and supervisors (first-
Managerial Employees Supported by line managers). That distinction is evident in the work of the route managers
Substantial Evidence in the Record which sets them apart from supervisors in general. Unlike supervisors who
basically merely direct operating employees in line with set tasks assigned to
The Court now finds that the job evaluation made by the Secretary of Labor them, route managers are responsible for the success of the company's main
is indeed supported by substantial evidence. The nature of the job of route line of business through management of their respective sales teams. Such
managers is given in a four-page pamphlet, prepared by the company, called management necessarily involves the planning, direction, operation and
"Route Manager Position Description," the pertinent parts of which read: evaluation of their individual teams and areas which the work of supervisors
does not entail.
A. BASIC PURPOSE
The route managers cannot thus possibly be classified as mere supervisors
A Manager achieves objectives through others. because their work does not only involve, but goes far beyond, the simple
direction or supervision of operating employees to accomplish objectives set
As a Route Manager, your purpose is to meet the sales plan; and you achieve by those above them. They are not mere functionaries with simple oversight
this objective through the skillful MANAGEMENT OF YOUR JOB AND functions but business administrators in their own right. An idea of the role
THE MANAGEMENT OF YOUR PEOPLE. of route managers as managers per se can be gotten from a memo sent by the
director of metro sales operations of respondent company to one of the route This brings us to the second question, whether the first sentence of Art. 245
managers. It reads: 13 of the Labor Code, prohibiting managerial employees from forming,
assisting or joining any labor organization, is constitutional in light of Art.
03 April 1995 III, §8 of the Constitution which provides:

To : CESAR T . REOLADA The right of the people, including those employed in the public and private
sectors, to form unions, associations, or societies for purposes not contrary
From : REGGIE M. SANTOS to law shall not be abridged.

Subj : SALARY INCREASE As already stated, whether they belong to the first category (managers per se)
or the second category (supervisors), managers are employees. Nonetheless,
Effective 01 April 1995, your basic monthly salary of P11,710 will be in the United States, as Justice Puno's separate opinion notes, supervisors
increased to P12,881 or an increase of 10%. This represents the added have no right to form unions. They are excluded from the definition of the
managerial responsibilities you will assume due to the recent restructuring term "employee" in §2(3) of the Labor-Management Relations Act of
and streamlining of Metro Sales Operations brought about by the continuous 1947. 15 In the Philippines, the question whether managerial employees have
losses for the last nine (9) months. a right of self-organization has arisen with respect to first-level managers or
supervisors, as shown by a review of the course of labor legislation in this
Let me remind you that for our operations to be profitable, we have to sustain country.
the intensity and momentum that your group and yourself have shown last
March. You just have to deliver the desired volume targets, better negotiated Right of Self-Organization of Managerial
concessions, rationalized sustaining deals, eliminate or reduced overdues, Employees under Pre-Labor Code Laws
improved collections, more cash accounts, controlled operating expenses,
etc. Also, based on the agreed set targets, your monthly performance will be Before the promulgation of the Labor Code in 1974, the field of labor
closely monitored. relations was governed by the Industrial Peace Act (R.A. No. 875).

You have proven in the past that your capable of achieving your targets thru In accordance with the general definition above, this law defined
better planning, managing your group as a fighting team, and thru aggressive "supervisor" as follows:
selling. I am looking forward to your success and I expect that you just have
to exert your doubly best in turning around our operations from a losing to a SEC. 2 . . .
profitable one!
(k) "Supervisor" means any person having authority in the interest of an
Happy Selling!! employer, to hire, transfer, suspend, lay-off, recall, discharge, assign,
recommend, or discipline other employees, or responsibly to direct them, and
(Sgd.) R.M. SANTOS to adjust their grievances, or effectively to recommend such acts, if, in
connection with the foregoing, the exercise of such authority is not of a
The plasticized card given to route managers, quoted in the separate opinion merely routinary or clerical nature but requires the use of independent
of Justice Vitug, although entitled "RM's Job Description," is only a judgment. 16
summary of performance standards. It does not show whether route managers
are managers per se or supervisors. Obviously, these performance standards The right of supervisors to form their own organizations was affirmed:
have to be related to the specific tasks given to route managers in the four
page "Route Manager Position Description," and, when this is done, the SEC. 3. Employees' Right to Self-Organization. - Employees shall have the
managerial nature of their jobs is fully revealed. Indeed, if any, the card right to self-organization and to form, join or assist labor organizations of
indicates the great latitude and discretion given to route managers - from their own choosing for the purpose of collective bargaining through
servicing and enhancing company goodwill to supervising and auditing representatives of their own choosing and to engage in concerted activities
accounts, from trade (new business) development to the discipline, training for the purpose of collective bargaining and other mutual aid and protection.
and monitoring of performance of their respective sales teams, and so forth, Individuals employed as supervisors shall not be eligible for membership in
- if they are to fulfill the company's expectations in the "key result areas." a labor organization of employees under their supervision but may form
separate organizations of their own. 17
Article 212(m) says that "supervisory employees are those who, in the
interest of the employer, effectively recommend such managerial actions if For its part, the Supreme Court upheld in several of its decisions the right of
the exercise of such authority is not merely routinary or clerical in nature but supervisors to organize for purposes of labor relations. 18
requires the use of independent judgment." Thus, their only power is to
recommend. Certainly, the route managers in this case more than merely Although it had a definition of the term "supervisor," the Industrial Peace Act
recommend effective management action. They perform operational, human did not define the term "manager." But, using the commonly-understood
resource, financial and marketing functions for the company, all of which concept of "manager," as above stated, it is apparent that the law used the
involve the laying down of operating policies for themselves and their teams. term "supervisors" to refer to the sub-group of "managerial employees"
For example, with respect to marketing, route managers, in accordance with known as front-line managers. The other sub-group of "managerial
B.1.1.1 to B.1.1.9 of the Route Managers Job Description, are charged, employees," known as managers per se, was not covered.
among other things, with expanding the dealership base of their respective
sales areas, maintaining the goodwill of current dealers, and distributing the However, in Caltex Filipino Managers and Supervisors Association v. Court
company's various promotional items as they see fit. It is difficult to see how of Industrial Relations, 19 the right of all managerial employees to self-
supervisors can be given such responsibility when this involves not just the organization was upheld as a general proposition, thus:
routine supervision of operating employees but the protection and expansion
of the company's business vis-a-vis its competitors. It would be going too far to dismiss summarily the point raised by respondent
Company - that of the alleged identity of interest between the managerial
While route managers do not appear to have the power to hire and fire people staff and the employing firm. That should ordinarily be the case, especially
(the evidence shows that they only "recommended" or "endorsed" the taking so where the dispute is between management and the rank and file. It does
of disciplinary action against certain employees), this is because this is a not necessarily follow though that what binds the managerial staff to the
function of the Human Resources or Personnel Department of the company. corporation forecloses the possibility of conflict between them. There could
14 And neither should it be presumed that just because they are given set be a real difference between what the welfare of such group requires and the
benchmarks to observe, they are ipso facto supervisors. Adequate control concessions the firm is willing to grant. Their needs might not be attended to
methods (as embodied in such concepts as "Management by Objectives then in the absence of any organization of their own. Nor is this to indulge in
[MBO]" and "performance appraisals") which require a delineation of the empty theorizing. The record of respondent Company, even the very case
functions and responsibilities of managers by means of ready reference cards cited by it, is proof enough of their uneasy and troubled relationship.
as here, have long been recognized in management as effective tools for Certainly the impression is difficult to erase that an alien firm failed to
keeping businesses competitive. manifest sympathy for the claims of its Filipino executives. To predicate
under such circumstances that agreement inevitably marks their relationship, Credit Supervisor A
ignoring that discord would not be unusual, is to fly in the face of reality.
Asst. Stores Supvr. A
. . . The basic question is whether the managerial personnel can organize.
What respondent Company failed to take into account is that the right to self- Ref. Supervisory Draftsman
organization is not merely a statutory creation. It is fortified by our
Constitution. All are free to exercise such right unless their purpose is Refinery Shift Supvr. B
contrary to law. Certainly it would be to attach unorthodoxy to, not to say an
emasculation of, the concept of law if managers as such were precluded from Asst. Supvr. A - Operations (Refinery)
organizing. Having done so and having been duly registered, as did occur in
this case, their union is entitled to all the rights under Republic Act No. 875. Refinery Shift Supvr. B
Considering what is denominated as unfair labor practice under Section 4 of
such Act and the facts set forth in our decision, there can be only one answer Asst. Lab. Supvr. A (Refinery)
to the objection raised that no unfair labor practice could be committed by
respondent Company insofar as managerial personnel is concerned. It is, as St. Process Engineer B (Refinery)
is quite obvious, in the negative. 20
Asst. Supvr. A - Maintenance (Refinery)
Actually, the case involved front-line managers or supervisors only, as the
plantilla of employees, quoted in the main opinion, 21 clearly indicates: Asst. Supvr. B - Maintenance (Refinery)

CAFIMSA members holding the following Supervisory Payroll Position Supervisory Accountant (Refinery)
Title are Recognized by the Company
Communications Supervisor (Refinery)
Payroll Position Title
Finally, also deemed included are all other employees excluded from the rank
Assistant to Mgr. - National Acct. Sales and file unions but not classified as managerial or otherwise excludable by
law or applicable judicial precedents.
Jr. Sales Engineer
Right of Self-Organization of Managerial
Retail Development Asst. Employees under the Labor Code

Staff Asst. - 0 Marketing Thus, the dictum in the Caltex case which allowed at least for the theoretical
unionization of top and middle managers by assimilating them with the
Sales Supervisor supervisory group under the broad phrase "managerial personnel," provided
the lynchpin for later laws denying the right of self-organization not only to
Supervisory Assistant top and middle management employees but to front line managers or
supervisors as well. Following the Caltex case, the Labor Code, promulgated
Jr. Supervisory Assistant in 1974 under martial law, dropped the distinction between the first and
second sub-groups of managerial employees. Instead of treating the terms
Credit Assistant "supervisor" and "'manager" separately, the law lumped them together and
called them "managerial employees," as follows:
Lab. Supvr. - Pandacan
ART. 212. Definitions . . .
Jr. Sales Engineer B
(k) "Managerial Employee" is one who is vested with powers or prerogatives
Operations Assistant B to lay down and execute management policies and/or to hire, transfer,
suspend, lay off, recall, discharge, assign or discipline employees, or to
Field Engineer effectively recommend such managerial actions. All employees not falling
within this definition are considered rank and file employees for purposes of
Sr. Opers. Supvr. - MIA A/S this Book. 22

Purchasing Assistant The definition shows that it is actually a combination of the commonly
understood definitions of both groups of managerial employees,
Jr. Construction Engineer grammatically joined by the phrase "and/or."

St. Sales Supervisor This general definition was perhaps legally necessary at that time for two
reasons. First, the 1974 Code denied supervisors their right to self-organize
Deport Supervisor A as theretofore guaranteed to them by the Industrial Peace Act. Second, it
stood the dictum in the Caltex case on its head by prohibiting all types of
Terminal Accountant B managers from forming unions. The explicit general prohibition was
contained in the then Art. 246 of the Labor Code.
Merchandiser
The practical effect of this synthesis of legal concepts was made apparent in
Dist. Sales Prom. Supvr. the Omnibus Rules Implementing the Labor Code which the Department of
Labor promulgated on January 19, 1975. Book V, Rule II, §11 of the Rules
Instr. - Merchandising provided:

Asst. Dist. Accountant B Supervisory unions and unions of security guards to cease operation. - All
existing supervisory unions and unions of security guards shall, upon the
Sr. Opers. Supervisor effectivity of the Code, cease to operate as such and their registration
certificates shall be deemed automatically cancelled. However, existing
Jr. Sales Engineer A collective agreements with such unions, the life of which extends beyond the
date of effectivity of the Code, shall be respected until their expiry date
Asst. Bulk Ter. Supt. insofar as the economic benefits granted therein are concerned.

Sr. Opers. Supvr. Members of supervisory unions who do not fall within the definition of
managerial employees shall become eligible to join or assist the rank and file
labor organization, and if none exists, to form or assist in the forming of such MR. LERUM. Yes, as long as it will carry the idea that the right of the
rank and file organization. The determination of who are managerial employees in the private sector is recognized. 24
employees and who are not shall be the subject of negotiation between
representatives of the supervisory union and the employer. If no agreement Lerum thus anchored his proposal on the fact that (1) government employees,
is reached between the parties, either or both of them may bring the issue to supervisory employees, and security guards, who had the right to organize
the nearest Regional Office for determination. under the Industrial Peace Act, had been denied this right by the Labor Code,
and (2) there was a need to reinstate the right of these employees. In
The Department of Labor continued to use the term "supervisory unions" consonance with his objective to reinstate the right of government, security,
despite the demise of the legal definition of "supervisor" apparently because and supervisory employees to organize, Lerum then made his proposal:
these were the unions of front line managers which were then allowed as a
result of the statutory grant of the right of self-organization under the MR. LERUM. Mr. Presiding Officer, after a consultation with several
Industrial Peace Act. Had the Department of Labor seen fit to similarly ban Members of this Commission, my amendment will now read as follows: "The
unions of top and middle managers which may have been formed following right of the people INCLUDING THOSE EMPLOYED IN THE PUBLIC
the dictum in Caltex, it obviously would have done so. Yet it did not, AND PRIVATE SECTORS to form associations, unions, or societies for
apparently because no such unions of top and middle managers really then purposes not contrary to law shall not be abridged. In proposing that
existed. amendment I ask to make of record that I want the following provisions of
the Labor Code to be automatically abolished, which read:
Real Intent of the 1986 Constitutional Commission
ART. 245. Security guards and other personnel employed for the protection
This was the law as it stood at the time the Constitutional Commission and security of the person, properties and premises of the employers shall not
considered the draft of Art. III, §8. Commissioner Lerum sought to amend be eligible for membership in a labor organization.
the draft of what was later to become Art. III, §8 of the present
Constitution: ART. 246. Managerial employees are not eligible to join, assist, and form
any labor organization.
MR. LERUM. My amendment is on Section 7, page 2, line 19, which is to
insert between the words "people" and "to" the following: WHETHER THE PRESIDING OFFICER (Mr. Bengzon). What does the Committee say?
EMPLOYED BY THE STATE OR PRIVATE ESTABLISHMENTS. In
other words, the section will now read as follows: "The right of the people FR. BERNAS. The Committee accepts.
WHETHER EMPLOYED BY THE STATE OR PRIVATE
ESTABLISHMENTS to form associations, unions, or societies for purposes THE PRESIDING OFFICER. (Mr. Bengzon) The Committee has accepted
not contrary to law shall not be abridged." 23 the amendment, as amended.

Explaining his proposed amendment, he stated: Is there any objection? (Silence) The Chair hears none; the amendment, as
amended, is approved. 25
MR. LERUM. Under the 1935 Bill of Rights, the right to form associations
is granted to all persons whether or not they are employed in the government. The question is what Commissioner Lerum meant in seeking to
Under that provision, we allow unions in the government, in government- "automatically abolish" the then Art. 246 of the Labor Code. Did he simply
owned and controlled corporations, and in other industries in the private want "any kind of wording as long as it will amount to absolute recognition
sector, such as the Philippine Government Employees' Association, unions of private sector employees, without exception, to organize"? 26 Or, did he
in the GSIS, the SSS, the DBP and other government-owned and controlled instead intend to have his words taken in the context of the cause which
corporations. Also, we have unions of supervisory employees and of security moved him to propose the amendment in the first place, namely, the denial
guards. But what is tragic about this is that after the 1973 Constitution was of the right of supervisory employees to organize, because he said, "We are
approved and in spite of an express recognition of the right to organize in afraid that without any corresponding provision covering the private sector,
P.D. No. 442, known as the Labor Code, the right of government workers, security guards, supervisory employees or majority [of] employees will still
supervisory employees and security guards to form unions was abolished. be excluded, and that is the purpose of this amendment"? 27

And we have been fighting against this abolition. In every tripartite It would seem that Commissioner Lerum simply meant to restore the right of
conference attended by the government, management and workers, we have supervisory employees to organize. For even though he spoke of the need to
always been insisting on the return of these rights. However, both the "abolish" Art. 246 of the Labor Code which, as already stated, prohibited
government and employers opposed our proposal, so nothing came out of "managerial employees" in general from forming unions, the fact was that in
this until this week when we approved a provision which states: explaining his proposal, he repeatedly referred to "supervisory employees"
whose right under the Industrial Peace Act to organize had been taken away
Notwithstanding any provision of this article, the right to self-organization by Art. 246. It is noteworthy that Commissioner Lerum never referred to the
shall not be denied to government employees. then definition of "managerial employees" in Art. 212(m) of the Labor Code
which put together, under the broad phrase "managerial employees," top and
We are afraid that without any corresponding provision covering the private middle managers and supervisors. Instead, his repeated use of the term
sector, the security guards, the supervisory employees or majority employees "supervisory employees," when such term then was no longer in the statute
[sic] will still be excluded, and that is the purpose of this amendment. books, suggests a frame of mind that remained grounded in the language of
the Industrial Peace Act.
I will be very glad to accept any kind of wording as long as it will amount to
absolute recognition of private sector employees, without exception, to Nor did Lerum ever refer to the dictum in Caltex recognizing the right of all
organize. managerial employees to organize, despite the fact that the Industrial Peace
Act did not expressly provide for the right of top and middle managers to
THE PRESIDENT. What does the Committee say? organize. If Lerum was aware of the Caltex dictum, then his insistence on the
use of the term "supervisory employees" could only mean that he was
FR. BERNAS. Certainly, the sense is very acceptable, but the point raised by excluding other managerial employees from his proposal. If, on the other
Commissioner Rodrigo is well-taken. Perhaps, we can lengthen this a little hand, he was not aware of the Caltex statement sustaining the right to
bit more to read: "The right of the people WHETHER UNEMPLOYED OR organize to top and middle managers, then the more should his repeated use
EMPLOYED BY STATE OR PRIVATE ESTABLISHMENTS." of the term "supervisory employees" be taken at face value, as it had been
defined in the then Industrial Peace Act.
I want to avoid also the possibility of having this interpreted as applicable
only to the employed. At all events, that the rest of the Commissioners understood his proposal to
refer solely to supervisors and not to other managerial employees is clear
MR. DE LOS REYES. Will the proponent accept an amendment to the from the following account of Commissioner Joaquin G. Bernas, who writes:
amendment, Madam President?
In presenting the modification on the 1935 and 1973 texts, Commissioner
Eulogio R. Lerum explained that the modification included three categories
of workers: (1) government employees, (2) supervisory employees, and (3) classification as such is not seriously disputed by PEO-FFW; the five (5)
security guards. Lerum made of record the explicit intent to repeal provisions previous CBAs between PIDI and PEO-FFW explicitly considered them as
of P.D. 442, the Labor Code. The provisions referred to were: confidential employees. By the very nature of their functions, they assist and
act in a confidential capacity to, or have access to confidential matters of,
ART. 245. Security guards and other personnel employed for the protection persons who exercise managerial functions in the field of labor relations. As
and security of the person, properties and premises of the employers shall not such, the rationale behind the ineligibility of managerial employees to form,
be eligible for membership in a labor organization. assist or joint a labor union equally applies to them.

ART. 246. Managerial employees are not eligible to join, assist, and form In Bulletin Publishing Co., Inc. v. Hon. Augusto Sanchez, this Court
any labor organization. 28 elaborated on this rationale, thus:

Implications of the Lerum Proposal ". . . The rationale for this inhibition has been stated to be, because if these
managerial employees would belong to or be affiliated with a Union, the
In sum, Lerum's proposal to amend Art. III, §8 of the draft Constitution by latter might not be assured of their loyalty to the Union in view of evident
including labor unions in the guarantee of organizational right should be conflict of interests. The Union can also become company-dominated with
taken in the context of statements that his aim was the removal of the the presence of managerial employees in Union membership." 32
statutory ban against security guards and supervisory employees joining
labor organizations. The approval by the Constitutional Commission of his To be sure, the Court in Philips Industrial was dealing with the right of
proposal can only mean, therefore, that the Commission intended the confidential employees to organize. But the same reason for denying them
absolute right to organize of government workers, supervisory employees, the right to organize justifies even more the ban on managerial employees
and security guards to be constitutionally guaranteed. By implication, no from forming unions. After all, those who qualify as top or middle managers
similar absolute constitutional right to organize for labor purposes should be are executives who receive from their employers information that not only is
deemed to have been granted to top-level and middle managers. As to them confidential but also is not generally available to the public, or to their
the right of self-organization may be regulated and even abridged competitors, or to other employees. It is hardly necessary to point out that to
conformably to Art. III, §8. say that the first sentence of Art. 245 is unconstitutional would be to
contradict the decision in that case.
Constitutionality of Art. 245
WHEREFORE, the petition is DISMISSED.
Finally, the question is whether the present ban against managerial
employees, as embodied in Art. 245 (which superseded Art. 246) of the Labor SO ORDERED.
Code, is valid. This provision reads:

ART. 245. Ineligibility of managerial employees to join any labor


organization; right of supervisory employees. - Managerial employees are
not eligible to join, assist or form any labor organization. Supervisory
employees shall not be eligible for membership in a labor organization of the
rank-and-file employees but may join, assist or form separate labor
organizations of their own. 29

This provision is the result of the amendment of the Labor Code in 1989 by
R.A. No. 6715, otherwise known as the Herrera-Veloso Law. Unlike the
Industrial Peace Act or the provisions of the Labor Code which it superseded,
R.A. No. 6715 provides separate definitions of the terms "managerial" and
"supervisory employees," as follows:

ART. 212. Definitions. . . .

(m) "managerial employee" is one who is vested with powers or prerogatives


to lay down and execute management policies and/or to hire transfer,
suspend, lay off, recall, discharge, assign or discipline employees.
Supervisory employees are those who, in the interest of the employer,
effectively recommend such managerial actions if the exercise of such
authority is not merely routinary or clerical in nature but requires the use of
independent judgment. All employees not falling within any of the above
definitions are considered rank-and-file employees for purposes of this Book.

Although the definition of "supervisory employees" seems to have been


unduly restricted to the last phrase of the definition in the Industrial Peace
Act, the legal significance given to the phrase "effectively recommends"
remains the same. In fact, the distinction between top and middle managers,
who set management policy, and front-line supervisors, who are merely
responsible for ensuring that such policies are carried out by the rank and
file, is articulated in the present definition. 30 When read in relation to this
definition in Art. 212(m), it will be seen that Art. 245 faithfully carries out
the intent of the Constitutional Commission in framing Art. III, §8 of the
fundamental law.

Nor is the guarantee of organizational right in Art. III, §8 infringed by a


ban against managerial employees forming a union. The right guaranteed in
Art. III, §8 is subject to the condition that its exercise should be for
purposes "not contrary to law." In the case of Art. 245, there is a rational
basis for prohibiting managerial employees from forming or joining labor
organizations. As Justice Davide, Jr., himself a constitutional commissioner,
said in his ponencia in Philips Industrial Development, Inc. v. NLRC: 31

In the first place, all these employees, with the exception of the service
engineers and the sales force personnel, are confidential employees. Their
A. D. GOTHONG MANUFACTURING CORPORATION qualify as a manager of the Davao Branch the opening of which branch never
EMPLOYEES UNION-ALU vs. CONFESOR materialized.
G.R. No. 113638 | 1999-11-16
GONZAGA-REYES, J.: Respondent Secretary of Labor affirmed the finding of the Med-Arbiter.
Motion for Reconsideration of the above resolution having been denied,
Petitioner A. D. Gothong Manufacturing Corporation Employees Union- petitioner Union appeals to this Court by petition for review on certiorari
ALU seeks to reverse and set aside the decision of the Secretary of Labor alleging the following grounds:
promulgated on September 30, 1993 affirming in toto the Resolution of
Mediator-Arbiter, Achilles V. Manit declaring Romulo Plaza and Paul I. THAT THE SECRETARY OF LABOR AND EMPLOYMENT
Michael Yap as rank- and-file employees of A. D. Gothong Manufacturing CLEARLY COMMITTED MISAPPREHENSION OF FACTS/EVIDENCE
Corporation. AND IF IT WERE NOT FOR SUCH MISAPPREHENSION IT WOULD
HAVE ARRIVED AT DIFFERENT CONCLUSION FAVORABLE TO
On May 12, 1993, petitioner A. D. Gothong Manufacturing Corporation PETITIONER.
Employees Union-ALU ("Union") filed a petition for certification election in II. THAT THE SECRETARY OF LABOR AND EMPLOYMENT ACTED
its bid to represent the unorganized regular rank-and-file employees of WITH GRAVE ABUSE OF DISCRETION AND CONTRARY TO LAW
respondent A. D. Gothong Manufacturing Corporation ("Company") IN AFFIRMING IN TOTO THE DECISION OF HONORABLE
excluding its office staff and personnel. Respondent Company opposed the ACHILLES V. MANIT, DEPARTMENT OF LABOR AND
petition as it excluded office personnel who are rank and file employees. In EMPLOYMENT, REGIONAL OFFICE No. 7, CEBU CITY IN DENYING
the inclusion-exclusion proceedings, the parties agreed to the inclusion of PETITIONER'S MOTION FOR RECONSIDERATION.2 [Rollo, pp. 8-9.]
Romulo Plaza and Paul Michael Yap in the list of eligible voters on condition
that their votes are considered challenged on the ground that they were We find no merit in the instant petition.
supervisory employees.
The Labor Code recognizes two (2) principal groups of employees, namely,
The certification election was conducted as scheduled and yielded the the managerial and the rank and file groups. Article 212 (m) of the Code
following results: provides:

YES - - - - - - - - - - - - - - - 20 "(m) 'Managerial employee' is one who is vested with powers or prerogatives
NO - - - - - - - - - - - - - - - -19 to lay down and execute management policies and/or to hire, transfer,
Spoiled - - - - - - - - - - - - - - 0 suspend, lay-off, recall, discharge, assign or discipline employees.
Challenged - - -- - - - - - - - - _2 Supervisory employees are those who, in the interest of the employer,
Total votes cast - - - - - - - - - 41 effectively recommend such managerial actions if the exercise of such
authority is not merely routinary or clerical in nature but requires the use of
Both Plaza and Yap argued that they are rank-and-file employees. Plaza independent judgment. All employees not falling within any of the above
claimed that he was a mere salesman based in Cebu, and Yap argued that he definitions are considered rank-and-file employees for purposes of this
is a mere expediter whose job includes the facilitation of the processing of Book."
the bills of lading of all intended company shipments.
Under Rule I, Section 2 (c), Book III of the Implementing Rules of the Labor
Petitioner Union maintains that both Plaza and Yap are supervisors who are Code, to be a member of managerial staff, the following elements must
disqualified to join the proposed bargaining unit for rank-and-file employees. concur or co-exist, to wit: (1) that his primary duty consists of the
In support of its position paper, the petitioner Union submitted the following: performance of work directly related to management policies; (2) that he
customarily and regularly exercises discretion and independent judgment in
1. Joint affidavit of Ricardo Ca¤ete, et al. which alleges that Michael the performance of his functions; (3) that he regularly and directly assists in
Yap is a supervisory employee of A. D. Gothong Manufacturing Corporation the management of the establishment; and (4) that he does not devote more
and can effectively recommend for their suspension/dismissal. than twenty percent of his time to work other than those described above.

2. Affidavit of Pedro Diez which alleges that the affiant is a supervisor in the In the case of Franklin Baker Company of the Philippines vs. Trajano3 [157
production department of A. D. Gothong Manufacturing Corporation; that SCRA 416.], this Court stated:
the affiant knows the challenged voters because they are also supervisory
employees of the same corporation; that the challenged voters used to attend "The test of 'supervisory' or 'managerial status' depends on whether a person
the quarterly meeting of the staff employees of A. D. Gothong Manufacturing possess authority to act in the interest of his employer in the matter specified
Corporation; in Article 212 (k) of the Labor Code and Section 1 (m) of its Implementing
Rules and whether such authority is not merely routinary or clerical in nature,
3. Photocopy of the memorandum dated January 4, 1991 regarding the but requires the use of independent judgment. Thus, where such
compulsory attendance of department heads/supervisors to the regular recommendatory powers as in the case at bar, are subject to evaluation,
quarterly meeting of all regular workers of A. D. Gothong Manufacturing review and final action by the department heads and other higher executives
Corporation on January 13, 1991. Appearing therein are the names of the company, the same, although present, are not exercise of independent
ROMULO PLAZA and MICHAEL YAP; judgment as required by law.4 [See also Engineering Equipment, Inc. vs.
NLRC, 133 SCRA 752; National Warehousing Corp. vs. CIR, 7 SCRA 602
4. A not-so-legible photocopy of a memorandum dated March 1, 1989 [1963]; National Waterworks and Sewerage Authority vs. NWSA
wherein the name "ROMY PLAZA" is mentioned as the acting OIC of GT Consolidated Unions, 11 SCRA 766.]
Marketing in Davao; and
It has also been established that in the determination of whether or not certain
5. Photocopy of the minutes of the regular quarterly staff meeting on August employees are managerial employees, this Court accords due respect and
13, 1989 at Mandaue City wherein Michael Yap is mentioned as a shipping therefore sustains the findings of fact made by quasi-judicial agencies which
assistant and a newly hired member of the staff.1 [Rollo, p. 35.] are supported by substantial evidence considering their expertise in their
respective fields.5 [Philippine Airline Employees Ass. (PALEA) vs. Ferrer-
The Med-Arbiter declared that the challenged voters Yap and Plaza are rank- Calleja, 162 SCRA 426; Lacorte vs. Inciong, G. R. No. 52034, September
and-file employees. 27, 1988; Arica vs. NLRC, G. R. No. 78210, February 28, 1989; A. M. Oreta
& Co. Inc. vs. NLRC, G. R. No. 74004, August 10, 1989.]
Petitioner Union appealed to the Secretary of Labor insisting that Yap and
Plaza are supervisor and manager respectively of the corporation and are The petition has failed to show reversible error in the findings of the Med-
prohibited from joining the proposed bargaining unit of rank-and-file Arbiter and the Secretary of the Department of Labor.
employees. In an attempt to controvert the arguments of petitioner,
respondent Company stressed that Pacita Gothong is the company's corporate In ruling against petitioner Union, the Med-Arbiter ruled that the petitioner
secretary and not Baby L. Siador, who signed the minutes of the meeting Union failed to present concrete and substantial evidence to establish the fact
submitted in evidence. Respondent also argued that Romulo Plaza could not that challenged voters are either managerial or supervising employees; the
Med-Arbiter evaluated the evidence as follows:
that the plan to open a branch in Davao City did not materialize.8 [Annexes
"The said joint affidavit of Ricardo Ca¤ete, et al. and that of Pedro Diez D & E, Comments to petitioner's Petition for Review on Certiorari.]
merely tagged the challenged voters as supervisors, but nothing is mentioned
about their respective duties, powers and prerogatives as employees which The reliance of petitioner on the affidavit of Jose Loseo, Personnel Manager,
would have indicated that they are indeed supervisory employees. There is that Plaza and Yap were hired by him as department head and supervisor of
no statement about an instance where the challenged voters effectively the respondent Company cannot be sustained in light of the affidavit of said
recommended such managerial action which required the use of independent Loseo dated September 28, 1993, attesting that he was "forced to sign" the
judgment. earlier memorandum on the job assignment of Yap and Plaza. This affidavit
is sought to be discarded by respondent Company for being perjurious and
The aforementioned documents have not been properly identified which ill-motivated.9 [Rejoinder to petitioner Appellant's Reply; p. 247, Rollo.]
renders them inadmissible in evidence. But, granting that they are the exact Petitioner Union however reiterates that Loseo's affidavit is corroborated by
replica of a genuine and authentic original copy, there is nothing in them the other public documents indicating that Plaza and Yap are not rank-and-
which specifically and precisely tells that the challenged voters can exercise file employees.10 [Memorandum for petitioner; p. 371, Rollo.]
the powers and prerogatives to effectively recommended such managerial
actions which require the use of independent judgment.6 [Rollo, p. 35.] The issue raised herein is basically one of fact: whether in the light of the
evidence submitted by both parties, Plaza and Yap are managerial employees
In upholding the above findings, the respondent Secretary of Labor or rank-and-file employees.
rationalized:
This Court is not a trier of facts. As earlier stated, it is not the function of this
"Based on the foregoing, Romulo Plaza and Paul Michael Yap can not Court to examine and evaluate the probative value of all evidence presented
qualify as managerial and supervisory employees, respectively, because there to the concerned tribunal which formed the basis of its impugned decision or
is nothing in the documentary evidence offered by herein petitioner-appellant resolution. Following established precedents, it is inappropriate to review
showing that they are actually conferred or actually exercising the said that factual findings of the Med-Arbiter regarding the issue whether Romulo
managerial/supervisory attributes. Plaza and Paul Michael Yap are or are not rank-and-file employees
considering that these are matters within their technical expertise.11 [Trade
In the case of Romulo Plaza, we note that indeed there is nothing in the Unions of the Philippines vs. Laguesma, 236 SCRA 586.] They are binding
minutes of the staff meeting held on 05 March 1993, particularly on the report on this Court as we are satisfied that they are supported by substantial
of the Sales Department, indicating that said appellee had been exercising evidence, and we find no capricious exercise of judgment warranting reversal
managerial prerogatives by hiring workers and issuing a check for the by certiorari.
payment of rentals of a warehouse, relative to the company branch in Davao
City. The imputation on the exercise of the said prerogative is misleading if WHEREFORE, the petition is denied for lack of merit.
not malicious because a plain reading of that portion of the report shows in
clear and simple language that one who made the said hiring and payment No pronouncement as to costs.
was no other than Mr. John Chua, the Sales Manager. The only instance when
the name of Romy Plaza was mentioned in the said report was in reference SO ORDERED.
to his designation as an OIC of the Davao City Branch while all the aspect of
the creation of the said branch is awaiting final approval by the Company
president and general manager (p. 197, last paragraph, records). The setting
up of said branch however, did not materialize, as evidenced by the
certification issued by the Revenue District Office and Office of the Mayor
in Davao City (pp. 198-199, records).

Likewise, evidence pinpointing that Paul Michael Yap is a supervisory


employee is altogether lacking. The fact that he was designated as shipping
assistant/expediter is of no moment, because titles or nomenclatures attached
to the position is not controlling.

Finally, the job descriptions extant on records vividly exhibit no trace of the
performance of managerial or supervisory functions (pp. 124-126,
records)".7 [Rollo, pp. 29-31.]

In this petition, petitioner Union claims that the documentary evidence was
"misapprehended" by public respondent. Petitioner Union reiterates that: (1)
in minutes of the staff meeting of respondent Company on August 13, 1989,
duly signed by the President Albino Gothong and attested by Jose F. Loseo
presiding officer/VP and Gertrudo Lao, Assistant General Manager, Paul
Michael Yap was listed as one of the staff; (2) in the regular quarterly
meeting on January 4, 1991, the names of Yap and Plaza are listed under the
heading Department Heads/Supervisors duly signed by President/General
Manager Albino Gothong and Asst. General Manager Gertrudo Lao; and (3)
in the staff meeting of March 5, 1993, Plaza was assigned as officer-in-charge
of the company's branch in Davao.

We find no cogent reason to disturb the finding of the Med-Arbiter and the
Secretary of Labor that the copies of the minutes presented in evidence do
not prove that Yap and Plaza were managerial or supervisory employees. We
have examined the documentary evidence, and nowhere is there a statement
therein about any instance where the challenged voters effectively
recommended any managerial action which would require the use of
independent judgment. The last piece of evidence was not discussed by the
Med-Arbiter; however a perusal thereof would show that while one J. Chua
of the Sales Department reported that "Romy Plaza was in Davao right now
acting as OIC," the same document states that the Davao operations still had
to be finalized. On the other hand, the claim of respondent Company that
Plaza is the head of the Davao branch is belied by the certification of the City
Treasurer of Davao and of the Bureau of Internal Revenue of Mandaue City
SAN MIGUEL CORPORATION (MANDAUE PACKAGING In turn, petitioner filed a Comment, wherein it reiterated that respondent was
PRODUCTS PLANTS) vs. MANDAUE PACKING PRODUCTS not a legitimate labor organization at the time of the filing of the petition.
PLANTS-SAN PACKAGING PRODUCTS –SAN MIGUEL Petitioner also propounded that contrary to respondent’s objectives of
CORPORATION MONTHLIES RANK-AND-FILE UNION – FFW establishing an organization representing rank-and-file employees, two of
(MPPP-SMPP-SMAMRFU-FFW) respondent’s officers, namely Vice-President Emannuel L. Rosell and
G.R. No. 152356. August 16, 2005 Secretary Bathan, were actually supervisory employees. In support of this
TINGA, J.: allegation, petitioner attached various documents evidencing the designation
of these two officers in supervisory roles, as well as their exercise of various
The central question in this Petition for Review is on what date did supervisory functions.9 Petitioner cited Article 245 of the Labor Code, which
respondent Mandaue Packing Products Plants-San Miguel Packaging provides that supervisory employees shall not be eligible for membership in
Products–San Miguel Corporation Monthlies Rank-And-File Union–FFW a labor organization of the rank-and-file employees.10
acquire legal personality in accordance with the Implementing Rules of the
Labor Code. The matter is crucial since respondent filed a petition for On 20 August 1998, petitioner filed a petition to cancel the union registration
certification election at a date when, it is argued, it had yet to acquire the of respondent. However, this petition was denied, and such denial was
requisite legal personality. The Department of Labor and Employment subsequently affirmed by the Court of Appeals in a decision that has since
(DOLE) and the Court of Appeals both ruled that respondent had acquired become final.11
legal personality on the same day it filed the petition for certification election.
The procedure employed by the respondent did not strictly conform with the In the meantime, on 15 September 1998, Med-Arbiter Manit issued an Order
relevant provisions of law. But rather than insist on an overly literal reading dismissing respondent’s petition for certification election. The sole ground
of the law that senselessly suffocates the constitutionally guaranteed right to relied upon for the dismissal was the Med-Arbiter’s Opinion that as of the
self-organization, we uphold the assailed decisions and the liberal spirit that date of filing of the petition on 15 June 1998, respondent did not have the
animates them. legal personality to file the said petition for certification election.12 No
discussion was adduced on petitioner’s claims that some of respondent’s
Antecedent Facts officers were actually supervisory employees.

The present petition assailed the Decision dated 7 June 2001 rendered by the Respondent promptly appealed the 15 September 1998 Order to the DOLE.
Court of Appeals Eighth Division1 which in turn affirmed a Decision dated On 22 February 1999, DOLE Undersecretary Rosalinda Dimapilis-Baldoz
22 Feburary 1999 by the DOLE Undersecretary for Labor Relations, rendered a Decision reversing the Order. Undersecretary Baldoz concluded
Rosalinda Dimapilis-Baldoz, ordering the immediate conduct of a that respondent acquired legal personality as early as 15 June 1998, the date
certification election among the petitioner’s rank-and-file employees, as it submitted the required documents, citing Section 3, Rule VI of the New
prayed for by respondent. The following facts are culled from the records. Rules Implementing the Labor Code (Implementing Rules) which deems that
a local/chapter acquires legal personality from the date of filing of the
On 15 June 1998, respondent, identifying itself as an affiliate of Federation complete documentary requirements as mandated in the Implementing Rules.
of Free Workers (FFW), filed a petition for certification election with the The DOLE also ruled that the contention that two of respondent’s officers
DOLE Regional Office No. VII. In the petition, respondent stated that it were actually supervisors can be threshed out in the pre-election conferences
sought to be certified and to represent the permanent rank-and-file monthly where the list of qualified voters is to be determined. The dispositive portion
paid employees of the petitioner.2 The following documents were attached of the DOLE Decision stated:
to the petition: (1) a Charter Certificate issued by FFW on 5 June 1998
certifying that respondent as of that date was duly certified as a local or WHEREFORE, the appeal is GRANTED. The order dated 15 September
chapter of FFW; (2) a copy of the constitution of respondent prepared by its 1999 of the Med-Arbiter is REVERSED and SET ASIDE. Accordingly, let
Secretary, Noel T. Bathan and attested by its President, Wilfred V. Sagun; the records of the case be remanded to the office of origin for the immediate
(3) a list of respondent’s officers and their respective addresses, again conduct of certification election, subject to the usual pre-election conference,
prepared by Bathan and attested by Sagun; (4) a certification signifying that among the monthly-paid rank-and-file employees of the Mandaue Packaging
respondent had just been organized and no amount had yet been collected Products Plant San Miguel Corporation, with the following choices:
from its members, signed by respondent’s treasurer Chita D. Rodriguez and
attested by Sagun; and (5) a list of all the rank-and-file monthly paid 1. MANDAUE PACKAGING PRODUCT PLANT SAN MIGUEL
employees of the Mandaue Packaging Products Plants and Mandaue Glass PACKAGING PRODUCTS SAN MIGUEL CORPORATION
Plant prepared by Bathan and attested by Sagun.3 MONTHLIES RANK AND FILE UNION–FFW (MPPP-SMPP-
SMCMRFUFFW),
The petition was assigned to Mediator-Arbiter Achilles V. Manit of the
DOLE Regional Office No. VII, and docketed as Case No. R0700-9806-RU- 2. NO UNION.
013.4
Pursuant to Rule XI, Section 11.1 of the New Implementing Rules, the
On 27 July 1998, petitioner filed a motion to dismiss the petition for company is hereby directed to submit to the office of origin the certified list
certification election on the sole ground that herein respondent is not listed of current employees in the bargaining unit, along with the payrolls covering
or included in the roster of legitimate labor organizations based on the the members of the bargaining unit for the last three months prior to the
certification issued by the Officer-In-Charge, Regional Director of the DOLE issuance of this decision.
Regional Office No. VII, Atty. Jesus B. Gabor, on 24 July 1998.
SO DECIDED.13
On 29 July 1998, respondent submitted to the Bureau of Labor Relations the
same documents earlier attached to its petition for certification. The These two conclusions of the DOLE were affirmed in the assailed Decision
accompanying letter, signed by respondent’s president Sagun, stated that of the Court of Appeals. It is now our task to review whether these
such documents were submitted in compliance with the requirements for the conclusions are warranted under law and jurisprudence. First, we shall
creation of a local/chapter pursuant to the Labor Code and its Implementing discuss the aspect of respondent’s legal personality in filing the petition for
Rules; and it was hoped that the submissions would facilitate the listing of certification election.
respondent under the roster of legitimate labor organizations.5 On 3 August
1998, the Chief of Labor Relations Division of DOLE Regional Office No. First Issue: On the Acquisition of
VII issued a Certificate of Creation of Local/Chapter No. ITD. I-ARFBT-
058/98, certifying that from 30 July 1998, respondent has acquired legal Legal Personality by Respondent
personality as a labor organization/worker’s association, it having submitted
all the required documents.6 Statutory Provisions for Registration Of

Opting not to file a comment on the Motion to Dismiss,7 respondent instead Local/Chapter of Federation or National Union
filed a Position Paper wherein it asserted that it had complied with all the
necessary requirements for the conduct of a certification election, and that Before we proceed to evaluate the particular facts of this case, it would be
the ground relied upon in the Motion to Dismiss was a mere technicality.8 useful to review the statutory paradigm that governs the establishment and
acquisition of legal personality by a local/chapter of a labor organization. The
applicable rules have undergone significant amendments in the last decade, a) A charter certificate issued by the federation or national union indicating
thus a recapitulation of the framework is in order. the creation or establishment of the local/chapter;

The Labor Code defines a labor organization as any union or association of (b) The names of the local/chapter's officers, their addresses, and the
employees which exists in whole or in part for the purpose of collective principal office of the local/chapter;
bargaining or of dealing with employers concerning terms and conditions of
employment,14 and a "legitimate labor organization" as any labor (c) The local/chapter's constitution and by-laws; provided that where the
organization duly registered with the DOLE, including any branch or local local/chapter's constitution and by-laws is the same as that of the federation
thereof.15 Only legitimate labor organizations may file a petition for or national union, this fact shall be indicated accordingly.
certification election.16
All the foregoing supporting requirements shall be certified under oath by
Article 234 of the Labor Code enumerates the requirements for registration the Secretary or Treasurer of the local/chapter and attested by its President.
of an applicant labor organization, association, or group of unions or workers
in order that such entity could acquire legal personality and entitlement to the In contrast, an independent union seeking registration is further required
rights and privileges granted by law to legitimate labor organizations. These under Dept. Order No. 90 to submit the number and names of the members,
include a registration fee of fifty pesos (₱50.00); a list of the names of the and annual financial reports.27
members and officers, and copies of the constitution and by-laws of the
applicant union.17 Section 3, Rule VI of Department Order No. 9 provides when the
local/chapter acquires legal personality.
However, the Labor Code itself does not lay down the procedure for the
registration of a local or chapter of a labor organization. Such has been Section 3. Acquisition of legal personality by local chapter. – A local/chapter
traditionally provided instead in the Implementing Rules, particularly in constituted in accordance with Section 1 of this Rule shall acquire legal
Book V thereof. However, in the last decade or so, significant amendments personality from the date of filing of the complete documents enumerated
have been introduced to Book V, first by Department Order No. 9 which took therein. Upon compliance with all the documentary requirements, the
effect on 21 June 1997, and again by Department Order No. 40 dated 17 Regional Office or Bureau shall issue in favor of the local/chapter a
February 2003. The differences in the procedures laid down in these various certificate indicating that it is included in the roster of legitimate labor
versions are significant. However, since the instant petition for certification organizations.
was filed in 1998, the Implementing Rules, as amended by Department Order
No. 9, should govern the resolution of this petition.18 It is evident based on this rule that the local/chapter acquires legal personality
from the date of the filing of the complete documentary requirements, and
Preliminarily, we should note that a less stringent procedure obtains in the not from the issuance of a certification to such effect by the Regional Office
registration of a local or chapter than that of a labor organization. or Bureau. On the other hand, a labor organization is deemed to have acquired
Undoubtedly, the intent of the law in imposing lesser requirements in the case legal personality only on the date of issuance of its certificate of
of a branch or local of a registered federation or national union is to registration,28 which takes place only after the Bureau of Labor Relations or
encourage the affiliation of a local union with a federation or national union its Regional Offices has undertaken an evaluation process lasting up until
in order to increase the local union's bargaining powers respecting terms and thirty (30) days, within which period it approves or denies the application.29
conditions of labor.19 This policy has remained consistent despite the In contrast, no such period of evaluation is provided in Department Order
succeeding amendments to Book V of the Omnibus Implementing Rules, as No. 9 for the application of a local/chapter, and more importantly, under it
contained in Department Orders Nos. 9 and 40. such local/chapter is deemed to acquire legal personality "from the date of
filing" of the documents enumerated under Section 1, Rule VI, Book V.
The case of Progressive Development Corp. v. Secretary of Labor,20
applying Section 3, Rule II, Book V of the Implementing Rules, in force Apart from promoting a policy of affiliation of local unions with national
before 1997, ruled that "a local or chapter therefore becomes a legitimate unions,30 there is a practical reason for sanctioning a less onerous procedure
labor organization only upon submission of the following to the BLR: (1) a for the registration of a local/chapter, as compared to the national union. The
charter certificate, within thirty (30) days from its issuance by the labor local/chapter relies in part on the legal personality of the federation or
federation or national union; and (2) The constitution and by-laws, a national union, which in turn, had already undergone evaluation and approval
statement of the set of officers, and the books of accounts all of which are from the Bureau of Legal Relations or Regional Office. In fact, a federation
certified under oath by the secretary or treasurer, as the case may be, of such or national union is required, upon registration, to establish proof of
local or chapter, and attested to by its president."21 The submission by the affiliation of at least ten (10) locals or chapters which are duly recognized as
local/chapter of duly certified books of accounts as a prerequisite for the collective bargaining agent in the establishment or industry in which they
registration of the local/chapter was dropped in Department Order No. 9,22 operate; and the names and addresses of the companies where the locals or
a development noted by the Court in Pagpalain Haulers v. Hon. Trajano,23 chapters operate and the list of all the members in each of the companies.31
wherein it was held that the previous doctrines requiring the submission of Once the national union or federation acquires legal personality upon the
books of accounts as a prerequisite for the registration of a local/chapter "are issuance of its certificate or registration,32 its legal personality cannot be
already passé and therefore, no longer applicable."24 subject to collateral attack.33

Department Order No. 40, now in effect, has eased the requirements by which The fact that the local/chapter acquires legal personality from the moment
a local/chapter may acquire legal personality. Interestingly, Department the complete documentary requirements are submitted seems to imply that
Order No. 40 no longer uses the term "local/chapter," utilizing instead the duty of the Bureau or Regional Office to register the local/chapter is
"chartered local," which is defined as a "labor organization in the private merely ministerial. However, in Progressive Development Corporation v.
sector operating at the enterprise level that acquired legal personality through Laguesma,34 the Court, in ruling against a petition for certification filed by
the issuance of a charter certificate by a duly registered federation or national a chapter, held that the mere submission of the documentary requirements
union, and reported to the Regional Office."25 Clearly under the present does not render ministerial the function of the Bureau of Labor Relations in
rules, the first step to be undertaken in the creation of a chartered local is the according due recognition to the labor organization.35 Still, that case was
issuance of a charter certificate by the duly registered federation or national decided before the enactment of Department Order No. 9, including the
union. Said federation or national union is then obligated to report to the aforestated Section 3. Should we consider the said 1997 amendments as
Regional Office the creation of such chartered local, attaching thereto the having obviated our characterization in Progressive of the Bureau’s duty as
charter certificate it had earlier issued.26 non-ministerial?

But as stated earlier, it is Department Order No. 9 that governs in this case. Notwithstanding the amendments, it still is good policy to maintain that per
Section 1, Rule VI thereof prescribes the documentary requirements for the Department Order No. 9, the duty of the Bureau of Labor Relations to
creation of a local/chapter. It states: recognize the local/chapter upon the submission of the documentary
requirements is not ministerial, insofar as the Bureau is obliged to adjudge
Section 1. Chartering and creation of a local chapter — A duly registered the authenticity of the documents required to be submitted. For example, the
federation or national union may directly create a local/chapter by submitting Bureau is not mandated to accept just any purported charter certificate matter
to the Regional Office or to the Bureau two (2) copies of the following: how spurious it is in appearance. It is empowered to ascertain whether the
submitted charter certificate is genuine, and if finding that said certificate is In regular order, it is the federation or national union, already in possession
fake, deny recognition to the local/chapter. of legal personality, which initiates the creation of the local/chapter. It issues
a charter certificate indicating the creation or establishment of the
However, in ascertaining whether or not to recognize and register the local/chapter. It then submits this charter certificate, along with the names of
local/chapter, the Bureau or Regional Office should not look beyond the the local/chapter’s officers, constitution and by-laws to the Regional Office
authenticity and due execution of the documentary requirements for the or Bureau. It is the submission of these documents, certified under oath by
creation of the local/chapter as enumerated under Section 1, Rule VI, Book the Secretary or Treasurer of the local/chapter and attested by the President,
V of Department Order No. 9. Since the proper submission of these which vests legal personality in the local/chapter, which is then free to file
documentary requirements is all that is necessary to recognize a on its own a petition for certification election.
local/chapter, it is beyond the province of the Bureau or Regional Offices to
resort to other grounds as basis for denying legal recognition of the In this case, the federation in question, the FFW, did not submit any of these
local/chapter. For example, Department Order No. 9 does not require the documentary requirements to the Regional Office or Bureau. It did however
local/chapter to submit the names of its members as a condition precedent to issue a charter certificate to the putative local/chapter (herein respondent).
its registration.36 It therefore would be improper to deny legal recognition to Respondent then submitted the charter certificate along with the other
a local/chapter owing to questions pertaining to its individual members since documentary requirements to the Regional Office, but not for the specific
the local/chapter is not even obliged to submit the names of its individual purpose of creating the local/chapter, but for filing the petition for
members prior to registration. certification election.

Certainly, when a local/chapter applies for registration, matters raised against It could be properly said that at the exact moment respondent was filing the
the personality of the federation or national union itself should not be acted petition for certification, it did not yet possess any legal personality, since the
upon by the Bureau or Regional Office, owing to the preclusion of collateral requisites for acquisition of legal personality under Section 3, Rule VI of
attack. Instead, the proper matter for evaluation by the Bureau or Regional Department Order No. 9 had not yet been complied with. It could also be
Office should be limited to whether the local/chapter is indeed a duly created discerned that the intention of the Labor Code and its Implementing Rules
affiliate of the national union or federation. that only those labor organizations that have acquired legal personality are
capacitated to file petitions for certification elections. Such is the general
Parenthetically, under the present Implementing Rules as amended by rule.
Department Order No. 40, it appears that the local/chapter (or now,
"chartered local") acquires legal personality upon the issuance of the charter Yet there are peculiar circumstances in this case that allow the Court to rule
certificate by the duly registered federation or national union.37 This might that respondent acquired the requisite legal personality at the same time it
signify that the creation of the chartered local is within the sole discretion of filed the petition for certification election. In doing so, the Court
the federation or national union and thus beyond the review or interference acknowledges that the strict letter of the procedural rule was not complied
of the Bureau of Labor Relations or its Regional Offices. However, with. However, labor laws are generally construed liberally in favor of labor,
Department Order No. 40 also requires that the federation or national union especially if doing so affirms the constitutionally guaranteed right to self-
report the creation of the chartered local to the Regional Office. organization.

Acquisition by Respondent of Legal Personality True enough, there was no attempt made by the national federation, or the
local/chapter for that matter, to submit the enumerated documentary
We now proceed to determine if and when the respondent acquired legal requirements to the Regional Office or Bureau for the specific purpose of
personality under the procedure laid down by the rules then in effect, creating the local/chapter. However, these same documents were submitted
Department Order No. 9, that is. by the local/chapter to the Regional Office as attachments to its petition for
certification election. Under Section 3, Rule VI of Department Order No. 9,
At the onset, the arguments raised by petitioner on this point are plainly it is the submission of these same documents to the Regional Office or
erroneous. Petitioner cites the case of Toyota Motor Philippines v. Toyota Bureau that operates to vest legal personality on the local/chapter.
Motor Philippines Corporation Labor Union,38 and the purported holding
therein that "[if] it is true that at the time of the filing of the petition, the said Thus, in order to ascertain when respondent acquired legal personality, we
registration certificate has not been approved yet, then, petitioner lacks the only need to determine on what date the Regional Office or Bureau received
legal personality to file the petition."39 However, an examination of the case the complete documentary requirements enumerated under Section 1, Rule
actually reveals that the cited portion was lifted from one of the antecedent VI of Department Order No. 9. There is no doubt that on 15 June 1998, or
rulings of the Med-Arbiter in that case which had not even been affirmed or the date respondent filed its petition for certification election, attached
reinstated by the Court on review.40 Moreover, such pronouncement made thereto were respondent’s constitution, the names and addresses of its
prior to the enactment of Department Order No. 9 squarely contradicts officers, and the charter certificate issued by the national union FFW. The
Section 3, Rule VI thereof, which provides that legal personality of the first two of these documents were duly certified under oath by respondent’s
local/chapter is vested upon the submission of the complete documentary secretary Bathan and attested to by president Sagun.41
requirements.
It may be noted though that respondent never submitted a separate by-laws,
It is also worth noting that petitioner union in Toyota was an independent nor does it appear that respondent ever intended to prepare a set thereof.
labor union, and not a local/chapter, and under Department Order No. 9, Section 1(c), Rule VI, Book V of Department Order No. 9 provides that the
independent labor unions, unlike local/chapters, acquire legal personality submission of both a constitution and a set of by-laws is required, or at least
only upon issuance of the certificate of registration by the Bureau or Regional an indication that the local/chapter is adopting the constitution and by-laws
Office. Still, petitioner cites in its favor Section 5, Rule V of Dept. Order No. of the federation or national union. A literal reading of the provision might
9, which states that "the labor organization or workers’ association shall be indicate that the failure to submit a specific set of by-laws is fatal to the
deemed registered and vested with legal personality on the date of issuance recognition of the local/chapter. A more critical analysis of this requirement
of its certificate of registration." Again, the citation is obviously misplaced, though is in order, especially as it should apply to this petition.
as respondent herein is a local/chapter, the acquisition of its legal personality
being governed instead by Section 3, Rule VI. By-laws has traditionally been defined as regulations, ordinances, rules or
laws adopted by an association or corporation or the like for its internal
It is thus very clear that the issuance of the certificate of registration by the governance, including rules for routine matters such as calling meetings and
Bureau or Regional Office is not the operative act that vests legal personality the like.42 The importance of by-laws to a labor organization cannot be
upon a local/chapter under Department Order No. 9. Such legal personality gainsaid. Without such provisions governing the internal governance of the
is acquired from the filing of the complete documentary requirements organization, such as rules on meetings and quorum requirements, there
enumerated in Section 1, Rule VI. Admittedly, the manner by which would be no apparent basis on how the union could operate. Without a set of
respondent was deemed to have acquired legal personality by the DOLE and by-laws which provides how the local/chapter arrives at its decisions or
the Court of Appeals was not in strict conformity with the provisions of otherwise wields its attributes of legal personality, then every action of the
Department Order No. 9. Nonetheless, are the deviations significant enough local/chapter may be put into legal controversy.
for the Court to achieve a different conclusion from that made by the DOLE
and the Court of Appeals? However, if those key by-law provisions on matters such as quorum
requirements, meetings, or on the internal governance of the local/chapter are
themselves already provided for in the constitution, then it would be feasible
to overlook the requirement for by-laws. Indeed in such an event, to insist on There is no need to apply any of the above cases at present because the
the submission of a separate document denominated as "By-Laws" would be question raised by petitioner on this point is already settled law, as a result of
an undue technicality, as well as a redundancy. the denial of the independent petition for cancellation filed by petitioner
against respondent on 20 August 1998. The ground relied upon therein was
An examination of respondent’s constitution reveals it sufficiently the alleged fraud, misrepresentation and false statement in describing itself
comprehensive in establishing the necessary rules for its operation. Article as a union of rank and file employees when in fact, two of its officers,
IV establishes the requisites for membership in the local/chapter. Articles V Emmanuel Rosell and Noel Bathan, were occupying supervisory
and VI name the various officers and what their respective functions are. The positions.48 Said petition was denied by the Regional Director, this action
procedure for election of these officers, including the necessary vote was affirmed by the DOLE, the Court of Appeals, and the Supreme Court.49
requirements, is provided for in Article IX, while Article XV delineates the The denial made by the Court of Appeals and the Supreme Court may have
procedure for the impeachment of these officers. Article VII establishes the been based on procedural grounds,50 but the prior decisions of the Regional
standing committees of the local/chapter and how their members are Director and the DOLE ruled squarely on the same issue now raised by the
appointed. Article VIII lays down the rules for meetings of the union, petitioner. We quote from the Resolution of the DOLE dated 29 December
including the notice and quorum requirements thereof. Article X enumerates 1998:
with particularity the rules for union dues, special assessments, fines, and
other payments. Article XII provides the general rule for quorum in meetings . . . . [The] substantive issue that is now before us is whether or not the
of the Board of Directors and of the members of the local/chapter, and cites inclusion of the two alleged supervisory employees in appellee union’s
the applicability of the Robert’s Rules of Order43 in its meetings. And membership amounts to fraud, misrepresentation, or false statement within
finally, Article XVI governs and institutes the requisites for the amendment the meaning of Article 239(a) and (c) of the Labor Code.
of the constitution.
We rule in the negative.
Indeed, it is difficult to see in this case what a set of by-laws separate from
the constitution for respondent could provide that is not already provided for Under the law, a managerial employee is "one who is vested with powers or
by the Constitution. These premises considered, there is clearly no need for prerogatives to lay down and execute management policies and/or to hire,
a separate set of by-laws to be submitted by respondent. transfer, suspend, layoff, recall, discharge, assign or discipline employees."
A supervisory employee is "one who, in the interest of the employer,
The Court likewise sees no impediment in deeming respondent as having effectively recommends managerial actions if the exercise of such
acquired legal personality as of 15 June 1998, the fact that it was the recommendatory authority is not merely routinary or clerical in nature but
local/chapter itself, and not the FFW, which submitted the documents requires the use of independent judgment.’" Finally, "all employees not
required under Section 1, Rule VI of Department Order No. 9. The evident falling within the definition of managerial or supervisory employee are
rationale why the rule states that it is the federation or national union that considered rank-and-file employees". It is also well-settled that the actual
submits said documents to the Bureau or Regional Office is that the creation functions of an employee, not merely his job title, are determinative in
of the local/chapter is the sole prerogative of the federation or national union, classifying such employee as managerial, supervisory or rank and file.
and not of any other entity. Certainly, a putative local/chapter cannot, without
the imprimatur of the federation or national union, claim affiliation with the In the case of Emmanuel Rossell, appellant’s evidence shows that he
larger unit or source its legal personality therefrom. undertakes the filling out of evaluation reports on the performance of
mechanics, which in turn are used as basis for reclassification. Given a ready
In the ordinary course, it should have been FFW, and not respondent, which and standard form to accomplish, coupled with the nature of the evaluation,
should have submitted the subject documents to the Regional Office. it would appear that his functions are more routinary than recommendatory
Nonetheless, there is no good reason to deny legal personality or defer its and hardly leave room for independent judgment. In the case of Noel Bathan,
conferral to the local/chapter if it is evident at the onset that the federation or appellant’s evidence does not show his job title although it shows that his
national union itself has already through its own means established the recommendations on disciplinary actions appear to have carried some weight
local/chapter. In this case, such is evidenced by the Charter Certificate dated on higher management. On this limited point, he may qualify as a supervisory
9 June 1998, issued by FFW, and attached to the petition for certification employee within the meaning of the law. This may, however, be outweighed
election. The Charter Certificate expressly states that respondent has been by his other functions which are not specified in the evidence.
issued the said certificate "to operate as a local or chapter of the [FFW]". The
Charter Certificate expressly acknowledges FFW’s intent to establish Assuming that Bathan is a supervisory employee, this does not prove the
respondent as of 9 June 1998.44 This being the case, we consider it existence of fraud, false statement or misrepresentation. Because good faith
permissible for respondent to have submitted the required documents itself is presumed in all representations, an essential element of fraud, false
to the Regional Office, and proper that respondent’s legal personality be statement and misrepresentation in order for these to be actionable is intent
deemed existent as of 15 June 1998, the date the complete documents were to mislead by the party making the representation. In this case, there is no
submitted. proof to show that Bathan, or appellee union for that matter, intended to
mislead anyone. If this was appellee union’s intention, it would have
Second Issue: On the Alleged Presence refrained from using a more precise description of the organization instead
of declaring that the organization is composed of ‘rank and file monthlies’.
Of Supervisory Employees as Hence, the charge of fraud, false statement or misrepresentation cannot be
sustained.
Officers of the Respondent
Appellant’s reliance on the Toyota case must be tempered by the peculiar
The second issue hinges on a point of some controversy and frequent circumstances of the case. Even assuming that Bathan, or Rossel for that
discussion in recent years. Petitioner claims error in the common matter, are supervisory employees, the Toyota case cannot certainly be given
pronouncement in the assailed decisions that the matter concerning the two an interpretation that emasculates the right to self-organization and the
officers who are allegedly supervisory employees may be threshed out during promotion of free trade unionism. We take administrative notice of the
pre-election conferences. Petitioner cites the cases of Toyota Motors and realities in union organizing, during which the organizers must take their
Progressive Development Corporation-Pizza Hut v. Ledesma45 wherein the chances, oftentimes unaware of the fine distinctions between managerial,
Court ruled that the question of prohibited membership of both supervisory supervisory and rank and file employees. The grounds for cancellation of
and rank-and-file employees in the same union must be inquired into anterior union registration are not meant to be applied automatically, but indeed with
to the granting of an order allowing a certification election; and that a union utmost discretion. Where a remedy short of cancellation is available, that
composed of both of these kinds of employees does not possess the requisite remedy should be preferred. In this case, no party will be prejudiced if Bathan
personality to file for recognition as a legitimate labor organization. It should were to be excluded from membership in the union. The vacancy he will thus
be noted though that in the more recent case of Tagaytay Highlands create can then be easily filled up through the succession provision of
International Golf Club v. Tagaytay Highlands Employees Union,46 the appellee union’s constitution and by-laws. What is important is that there is
Court, notwithstanding Toyota and Progressive, ruled that after a certificate an unmistakeable intent of the members of appellee union to exercise their
of registration is issued to a union, its legal personality cannot be subject to right to organize. We cannot impose rigorous restraints on such right if we
collateral attack, but questioned only in an independent petition for are to give meaning to the protection to labor and social justice clauses of the
cancellation.47 Constitution.51
The above-cited pronouncement by Bureau of Labor Relations Director
Benedicto Ernesto R. Bitonio, Jr. in BLR-A-C-41-11-11-98 was affirmed by
the Court of Appeals and the Supreme Court. Hence, its pronouncement
affirming, notwithstanding the questions on the employment status of Rossell
and Bathan, the legitimacy of the respondent, stands as a final ruling beyond
the ambit of review, thus warranting the Court’s respect. There may be a
difference between this case, which involves a petition for certification
election, and the other case, which concerns a petition for cancellation.
However, petitioner opposes the petition for certification election on the
ground of the illegitimacy of respondent, owing to the alleged supervisory
nature of the duties of Rossell and Bathan. That matter has already been
settled in the final disposition of the petition for cancellation, and thus cannot
be unsettled by reason of this present petition.

Effect of Respondent’s Manifestation

Of Subsequent Developments

A final note. In its Memorandum, petitioner alleges that the bargaining unit
that respondent sought to represent is no longer the same because of the
dynamic nature of petitioner’s business, a lot of changes having occurred in
the work environment, and that four of respondent’s officers are no longer
connected with petitioner.52 Assuming that these manifestations are true,
they have no effect on the Court’s ruling that a certification election should
be immediately conducted with respondent as one of the available choices.
Petitioner’s bare manifestations adduce no reason why the certification
election should not be conducted forthwith. If there are matters that have
arisen since the filing of the petition that serve to delay or cancel the election,
these can be threshed out during the pre-election conferences. Neither is the
fact that some of respondent’s officers have since resigned from petitioner of
any moment. The local/chapter retains a separate legal personality from that
of its officers or members that remains viable notwithstanding any turnover
in its officers or members.

WHEREFORE, the Petition is DENIED. Costs against petitioner.

SO ORDERED.
SAN MIGUEL FOODS, INC. vs. LAGUESMA
G.R. No. 116172 October 10, 1996 SO ORDERED.4
HERMOSISIMA, JR., J.:p
Petitioner SMFI appealed the foregoing Order to the Secretary of Labor and
This is a petition for certiorari under Rule 65 to review and set aside two Employment alleging that the Med-Arbiter erred in directing the conduct of
Resolutions of Mediator-Arbiter Achilles V. Manit, dated January 5, 1994 certification election considering that the local or chapter of IBM at SMFI is
and April 6, 1994, and the affirmation Order on appeal of the public still not a legitimate labor organization with a right to be certified as the
respondent, Undersecretary Bienvenido E. Laguesma of the Department of exclusive bargaining agent in petitioner's establishment based on two
Labor and Employment. The petition below was entitled: "In Re: Petition for grounds: (1) the authenticity and due execution of the Charter Certificate
Direct Certification as the Sole and Exclusive Bargaining Agent of All submitted by IBM in favor of its local at SMFI cannot yet be ascertained as
Monthly Paid Employees of SMFI-Cebu B-Meg Feeds Plant," docketed as it is still not known who is the legitimate and authorized representative of the
OS-MA-A-3-51-94 (R0700-9309-RU-036). IBM Federation who may validly issue said Charter Certificate; and (2) a
group of workers or a local union shall acquire legal personality only upon
The essential facts are not disputed. the issuance of a Certificate of Registration by the Bureau of Labor Relations
under Article 234 of the Labor Code, which IBM at SMFI did not possess.
On September 24, 1993, a petition for certification election among the
monthly-paid employees of the San Miguel Food, Inc.-Cebu B-Meg Feeds In a resolution dated April 6, 1994, public respondent Undersecretary
Plant was filed by private respondent labor federation Ilaw at Buklod ng Bienvenido Laguesma, by authority of the Secretary of Labor and
Mangagawa (IBM, for brevity) before Med-Arbiter Achilles V. Manit, Employment, denied petitioner's appeal, viz.:
alleging, inter alia, that it is a legitimate labor organization duly registered
with the Department of Labor and Employment (DOLE) under Registration WHEREFORE, the appeal is hereby denied for lack of merit and the Order
Certificate No. 5369-IP. SMFI-Cebu B-Meg Feeds Plant (SMFI, for brevity), of the Med-Arbiter is hereby affirmed.
herein petitioner, is a business entity duly organized and existing under the
laws of the Philippines which employs roughly seventy-five (75) monthly Let the records of this case be forwarded to the Regional Office of origin for
paid employees, almost all of whom support the present petition. It was the immediate conduct of certification election subject to the usual pre-
submitted in said petition that there has been no certification election election conference.
conducted in SMFI to determine the sole and exclusive bargaining agent
thereat for the past two years and that the proposed bargaining unit, which is SO RESOLVED.5
SMFI's monthly paid employees, is an unorganized one. It was also stated
therein that petitioner IBM (herein private respondent) has already complied Thereafter, a Motion for Reconsideration was filed which was also denied by
with the mandatory requirements for the creation of its local or affiliate in the public respondent in his Order dated May 24, 1994.6
SMFI's establishment.
Hence, the instant petition interposing the following justifications:
On October 25, 1993, herein petitioner SMFI filed a Motion to Dismiss the
aforementioned petition dated September 24, 1993 on the ground that a 1) THE HONORABLE UNDERSECRETARY BIENVENIDO E.
similar petition remains pending between the same parties for the same cause LAGUESMA GRAVELY ABUSED HIS DISCRETION WHEN HE
of action before Med-Arbiter Achilles V. Manit. ARBITRARILY RULED THAT "A LOCAL OR CHAPTER OF A LABOR
FEDERATION, LIKE RESPONDENT IBM, NEED NOT OBTAIN A
SMFI was referring to an evidently earlier petition, docketed as CE CASE CERTIFICATE OF REGISTRATION FROM THE BUREAU OF LABOR
NO R0700-9304-RU-016, filed on April 28, 1993 before the office of Med- RELATIONS TO ACQUIRE LEGAL PERSONALITY," WHEN ARTICLE
Arbiter Manit. Indeed, both petitions involved the same parties, cause of 234 OF THE LABOR CODE OF THE PHILIPPINES AND SECTION 3 OF
action and relief being prayed for, which is the issuance of an order by the RULE II OF BOOK V OF THE RULES IMPLEMENTING THE LABOR
Med-Arbiter allowing the conduct for a certification election in SMFI's CODE, AS AMENDED, CLEARLY PROVIDES THAT A GROUP OF
establishment. The contention is that the judgment that may be rendered in WORKERS OR A LOCAL UNION SHALL ACQUIRE LEGAL
the first petition would be determinative of the outcome of the second PERSONALITY ONLY UPON THE ISSUANCE OF THE CERTIFICATE
petition, date September 24, 1993. OF REGISTRATION BY THE BUREAU OF LABOR RELATIONS. AND,

On December 2, 1993, private respondent IBM filed its Opposition to SMFI's 2) THE HONORABLE UNDERSECRETARY BIENVENIDO E.
Motion to Dismiss contending, among others, that the case referred to by LAGUESMA GRAVELY ABUSED HIS DISCRETION WHEN HE
SMFI had already been resolved by Med-Arbiter Manit in his Resolution and PREMATURELY AND ARBITRARILY RULED THAT RESPONDENT
Order dated July 26, 19931 and September 2, 1993,2 respectively, wherein IBM IS A LEGITIMATE LABOR ORGANIZATION WHEN THE
IBM's first petition for certification election was denied mainly due to IBM's AUTHENTICITY AND DUE EXECUTION OF THE CHARTER
failure to comply with certain mandatory requirements of the law. This denial CERTIFICATE SUBMITTED BY RESPONDENT IBM CANNOT YET
was affirmed by the Med-Arbiter in another Order dated November 12, BE ASCERTAINED BECAUSE IT IS STILL NOT KNOWN WHO ARE
19933 wherein the Resolutions dated July 26, 1993 and September 2, 1993 THE LEGITIMATE OFFICERS OF THE IBM FEDERATION WHO MAY
were made to stand. Thus, IBM argues that there having been no similar VALIDLY ISSUE SAID CHARTER CERTIFICATE AS THE CASE
petition pending before Med-Arbiter Manit, another petition for certification FILED TO RESOLVE THE ISSUE ON WHO ARE THE LEGITIMATE
election may be refiled as soon as the said requirements are met. These OFFICERS OF THE IBM FEDERATION IS STILL PENDING
requirements were finally satisfied before the second petition for certification RESOLUTION BEFORE THIS HONORABLE SUPREME COURT.7
election was brought on September 24, 1993.
The petition has no merit.
On January 5, 1994, Med-Arbiter Manit, this time, granted the second
petition for certification election of private respondent IBM in this wise: Petitioner asserts that IBM at SMFI is not a legitimate labor organization
notwithstanding the fact that it is a local or chapter of the IBM Federation.
Let, therefore, a certification election be conducted among the monthly paid This is so because under Article 234 of the Labor Code, any labor
rank and file employees of SMFI-CEBU B-MEG FEEDS PLANT at Lo-oc, organization shall acquire legal personality only upon the issuance of the
Mandaue City. The choices shall be: YES-for IBM AT SMFI-CEBU B- Certificate of Registration by the Bureau of Labor Relations.
MEG; and NO — for No Union.
We do not agree.
The parties are hereby notified of the pre-election conference which will take
place on January 17, 1994 at 3:00 o'clock in the afternoon to set the date and I
time of the election and to thresh out the mechanics thereof. On said date and
time the respondent is directed to submit the payroll of its monthly paid rank Article 212(h) of the Labor Code defines a legitimate labor organization as
and file employees for the month of June 1993 which shall be the basis for "any labor organization duly registered with the Department of Labor and
the list of the eligible voters. The petitioner is directed to be ready to submit Employment, and includes any branch or local thereof ."
a list of the monthly paid rank and file employees of SMFI-CEBU B-MEG
FEEDS PLANT when the respondent fails to submit the required payroll.
It is important to determine whether or not a particular labor organization is Implementing Rules of the Labor Code governs the procedure for union
legitimate since legitimate labor organizations have exclusive rights under affiliation, the relevant portions of which provide:
the law which cannot be exercised by non-legitimate unions, one of which is
the right to be certified as the exclusive representative of all the employees Sec. 3. Union Affiliation: Direct Membership with National Union. — An
in an appropriate collective bargaining unit for purposes of collective affiliate of a labor federation or national union may be a local or chapter
bargaining. These rights are found under Article 242 of the Labor Code, to thereof or an independently registered union.
wit:
(a) The labor federation or national union concerned shall issue a charter
Art. 242. Rights of legitimate labor organizations. — A legitimate labor certificate indicating the creation or establishment of a local or chapter, copy
organization shall have the right: of which shall be submitted to the Bureau of Labor Relations within thirty
(30) days from issuance of such charter certificate.
(a) To act as the representative of its members for the purpose of collective
bargaining; (b) An independently registered union shall be considered an affiliate of a
labor federation or national union after submission to the Bureau of the
(b) To be certified as the exclusive representative of all the employees in an contract or agreement of affiliation within thirty (30) days after its execution.
appropriate collective bargaining unit for purpose of collective bargaining;
xxx xxx xxx
(c) To be furnished by the employer, upon written request, with his annual
audited financial statement, including the balance sheet and the profit and (e) The local or chapter of a labor federation or national union shall have and
loss statement, within thirty (30) calendar days from the date of receipt of the maintain a constitution and by-laws, set of officers and books of accounts.
request, after the union has been duly recognized by the employer or certified For reporting purposes, the procedure governing the reporting of
as the sole and exclusive bargaining representative of the employees in the independently registered unions, federations or national unions shall be
bargaining unit, or within sixty (60) calendar days before the expiration of observed.
the existing collective bargaining agreement, or during the collective
bargaining negotiation; Paragraph (a) refers to a local or chapter of a federation which did not
undergo the rudiments of registration while paragraph (b) refers to an
(d) To own property, real or personal, for the use and benefit of the labor independently registered union which affiliated with a federation. Implicit in
organization and its members; the foregoing differentiation is the fact that a local or chapter need not be
independently registered. By force of law (in this case, Article 212 [h]), such
(e) To sue and be sued in its registered name; and local or chapter becomes a legitimate labor organization upon compliance
with the aforementioned provisions of Section 39 (a) and (e), without having
(f) To undertake all other activities designed to benefit the organization and to be issued a Certificate of Registration in its favor by the BLR.
its members, including cooperative, housing welfare and other projects not
contrary to law. The cases of Lopez Sugar Corporation v. Secretary of Labor and
Employment,10 Phoenix Iron and Steel Corporation v. Secretary of Labor
xxx xxx xxx and Employment 11 and Protection Technology, Inc. v. Secretary,
Department of Labor and Employment,12 all going back to our landmark
The pertinent question, therefore, must be asked: When does a labor holding in Progressive Development Corporation v. Secretary, Department
organization acquire legitimacy? of Labor and
Employment,13 unequivocably laid down the rule, thus:
Ordinarily, a labor organizations attains the status of legitimacy only upon
the issuance in its name of a Certificate of Registration by the Bureau of A local or chapter therefore becomes a legitimate labor organization only
Labor Relations pursuant to Articles 234 and 235 of the Labor Code, viz.: upon submission of the following to the BLR:

Art. 234. Requirements of registration. — Any applicant labor organization, 1) A charter certificate, within 30 days from its issuance by the labor
association or group of unions or workers shall acquire legal personality and federation or national union, and
shall be entitled to the rights and privileges granted by law to legitimate labor
organizations upon issuance of the certificate of registration based on the 2) The constitution and by-laws, a statement on the set of officers, and the
following requirements: books of accounts all of which are certified under oath by the secretary or
treasurer, as the case may be, of such local or chapter, and attested to by its
(a) Fifty pesos (P50.00) registration fee; president.

(b) The names of its officers, their addresses, the principal address of the Absent compliance with these mandatory requirements, the local or chapter
labor organization, the minutes of the organizational meetings and the list of does not become a legitimate labor organization.
the workers who participated in such meetings;
Corollarily, the satisfaction of all these requirements by the local or chapter
(c) The names of all its members comprising at least twenty percent (20%) shall vest upon it the status of legitimacy with all its concomitant statutory
of all the employees in the bargaining unit where it seeks to operate; privileges, one of which is the right to be certified as the exclusive
representative of all the employees in an appropriate bargaining unit.
(d) If the applicant union has been in existence for one or more years, copies
of its annual financial reports; and In the case at bench, public respondent Bienvenido E. Laguesma, in affirming
the finding of the Med-Arbiter that IBM at SMFI is a legitimate labor
(e) Four (4) copies of the constitution and by-laws of the applicant union, organization,14 made the following material pronouncement amply
minutes of its adoption or ratification, and the list of the members who supported by the records;
participated in it.
[t]he resolution of the issue raised by the respondent on whether or not
Art. 235. Action on application. — The Bureau shall act on all applications petitioner is a legitimate labor organization will depend on the documents
for registration within thirty (30) days from filing. submitted by the petitioner in the second petition.

All requisite documents and papers shall be certified under oath by the A close scrutiny of the records shows that at the time of the filing of the
secretary or the treasurer of the organization, as the case may be, and attested subject petition on 24 September 1993 by the petitioner Ilaw at Buklod ng
to by its president. Manggagawa, for and in behalf of its local affiliate IBM at SMFI-CEBU B-
MEG, the latter has been clothed with the status and/or character of a
The foregoing procedure is not the only way by which a labor union may legitimate labor organization. This is so, because on 19 July 1993, petitioner
become legitimate, however. When an unregistered union becomes a branch, submitted to the Bureau of Labor Relations (BLR), this Department, the
local or chapter of a federation, some of the aforementioned requirements for following documents: charter certificate, constitution and by-laws, names
registration are no longer required.8 Section 3, Rule II, Book V of the and addresses of the union officers and certification of the union's secretary
on the non-availability of the union's Books of Accounts. Said documents disputes on representation may be laid to rest, by the unequivocal vote of the
(expect the charter certificate) are certified under oath and attested to by the employees themselves. 29 Indeed, it is the keystone of industrial
local union's secretary and President, respectively.15 democracy.30

Petitioner SMFI does not dispute the fact that IBM at SMFI has complied Petitioner next asseverates that the Charter Certificate submitted by the
with the second set or requirements, i.e., constitution, by-laws, et. al. What private respondent was defective in that it was not certified under oath and
is controverted is the non-compliance with the requirement as to the charter attested to by the organization's secretary and President.
certificate which must be submitted to the BLR within thirty (30) days from
its issuance by the labor federation. While the presence of a charter certificate Petitioner is grasping at straws. Under our ruling in the Progressive
is conceded, petitioner maintains that the validity and authenticity of the Development Corporation31 case, what is required to be certified under oath
same cannot yet be ascertained as its is still not known who is the legitimate by the secretary or treasurer and attested to by the local's president are the
and authorized representative of the IBM Federation who may validly issue "constitution and by-laws, a statement on the set of officers, and the books of
said charter certificate in favor of its local, IBM at SMFI. According to accounts" of the organization. The charter certificate issued by the mother
petitioner, there are two (2) contending sets of officers of the IBM Federation union need not be certified under oath by the secretary or treasurer and
at the time the charter certificate was issued in favor of IBM at SMFI, the attested to by the local's president.
faction of Mr. Severino O. Meron and that of Mr. Edilberto B. Galvez.
IV
On this point, public respondent, in upholding the legitimate status of IBM
at SMFI, backed up by the Solicitor General, had this to say: Petitioner, in its Reply to public respondent's Comment, nevertheless calls
the attention of this court to the fact that, contrary to the assertion of private
The contention of the respondent that unless and until the issue on who is the respondent IBM that it is a legitimate labor federation and therefore has the
legitimate national president, of the Ilaw at Buklod ng Mangagawa is capacity and authority to create a local or chapter at SMFI, the Chief of the
resolved, the petitioner cannot claim that is has a valid charter certificate Labor Organizations Division of the Bureau of Labor Relations — Manila
necessary for it to acquire legal personality is untenable. We wish to stress had allegedly issued a certification last January 17, 1995 to the effect that
that the resolution of the said issue will not in any way affect the validity of private respondent is not a legitimate labor federation.32
the charter certificate issued by the IBM in favor of the local union. It must
be borne in mind that the said charter certificate was issued by the IBM in its This is a factual issue which petitioner should have raised before the Med-
capacity as a labor organization, a juridical entity which has a separate and Arbiter so as to allow the private respondent ample opportunity to present
distinct legal personality from its members. When as in this case, there is no evidence to the contrary. This Court is definitely not the proper venue to
showing that the Federation acting as a separate entity is questioning the consider this matter for it is not a trier of facts. It is noteworthy that petitioner
legality of the issuance of the said charter certificate, the legality of the did not challenge the legal personality of the federation in the proceedings
issuance of the same in favor of the local union is presumed. This, before the Med-Arbiter. Nor was this issue raised in petitioner's appeal to the
notwithstanding the alleged controversy on the leadership of the Office of the Secretary of Labor and Employment. This matter is being raised
federation.16 for the first time in this petition. An issue which was neither alleged in the
pleadings nor raised during the proceedings below cannot be ventilated for
We agree with this position of the public respondent and the Solicitor the first time before this Court. It would be offensive to the basic rule of fair
General. In addition, private respondent's Comment to this petition indicates play, justice and due process.33 Certiorari is a remedy narrow in its scope
that in the election of officers held to determine the representatives of IBM, and inflexible in character. It is not a general utility tool in the legal
the faction of Mr. Meron lost to the group of Mr. Edilberto Galvez, and the workshop.34 Factual issues are not a proper subject for certiorari, as the
latter was acknowledged as the duly elected IBM National President.17 Thus, power of the Supreme Court to review labor cases is limited to the issue of
the authority of Mr. Galvez to sign the charter certificate of IBM at SMFI, as jurisdiction and grave abuse of discretion.35 It is simply unthinkable for the
President of the IBM Federation,18 can no longer be successfully questioned. public respondent Undersecretary of Labor to have committed grave abuse
A punctilious examination of the records presents no evidence to the contrary of discretion in this regard when the issue as to the legal personality of the
and petitioner, instead of squarely refuting this point, skirted the issue by private respondent IBM Federation was never interposed in the appeal before
insisting that the mere presence of two contending factions in the IBM said forum.
prevents the issuance of a valid and authentic charter certificate in favor of
IBM at SMFI. This averment of petitioner simply does not deserve any merit. V

II Finally, the certification election sought to be stopped by petitioner is, as of


now, fait accompli. The monthly paid rank-and-file employees of SMFI have
In any case, this Court notes that it is petitioner, the employer, which has already articulated their choice as to who their collective bargaining agent
offered the most tenacious resistance to the holding of a certification election should be. In the certification election held on August 20, 1994,36 the SMFI
among its monthly-paid rank-and-file employees. This must not be so, for workers chose IBM at SMFI to be their sole and exclusive bargaining agent.
the choice of a collective bargaining agent is the sole concern of the This democratic decision deserve utmost respect. Again, it bears stressing
employees.19 The only exception to this rule is where the employer has to that labor legislation seeks in the main to protect the interest of the members
file the petition for certification election pursuant to Article 25820 of the of the working class. It should never be used to subvert their will.3 7
Labor Code because it was requested to bargain collectively,21 which
exception finds no application in the case before us. Its role in a certification WHEREFORE, the petition is DENIED. Costs again petitioner.
election has aptly been described in Trade Unions of the Philippines and
Allied Services (TUPAS) v. Trajano,22 as that of a mere by-stander. It has SO ORDERED.
no legal standing in a certification election as it cannot oppose the petition or
appeal the Med-Arbiter's orders related thereto. An employer that involves
itself in a certification election lends suspicion to the fact it wants to create a
company union.23 This Court should be the last agency to lend support to
such an attempt at interference with a purely internal affair of labor.24

While employers may rightfully be notified or informed of petitions of such


nature, they should not, however, be considered parties thereto with the
concomitant right to oppose it. Sound policy dictates that they should
maintain a strictly hands-off policy.25

It bears stressing that no obstacle must be placed to the holding of


certification elections,26 for it is a statutory policy that should not be
circumvented.27 The certification election is the most democratic and
expeditious method by which the laborers can freely determine the union that
shall act as their representative in their dealings with the establishment where
they are working.28 It is the appropriate means whereby controversies and
ACEDERA vs. INTERNATIONAL CONTAINER TERMINAL On December 8, 1997, petitioners-appellants filed with the Labor Arbiter a
SERVICES, INC. (ICTSI) Complaint-in-Intervention with Motion to Intervene.16 In the petition at bar,
G.R. No. 146073 January 13, 2003 they justified their move to intervene in this wise:
CARPIO-MORALES, J.:
[S]hould the union succeed in prosecuting the case and in getting a favorable
For consideration is the petition for review on certiorari assailing the decision reward it is actually they that would benefit from the decision. On the other
of the Court of Appeals affirming that of the National Labor Relations hand, should the union fail to prove its case, or to prosecute the case
Commission (NLRC) which affirmed the decision of the Labor Arbiter diligently, the individual workers or members of the union would suffer great
denying herein petitioners-appellants’ Complaint-in-Intervention with and immeasurable loss. … [t]hey wanted to insure by their intervention that
Motion for Intervention. the case would thereafter be prosecuted with all due diligence and would not
again be dismissed for lack of interest to prosecute on the part of the union.17
The antecedent facts are as follows:
The Labor Arbiter rendered a decision, the dispositive portion of which
Petitioners-appellants Jerry Acedera, et al. are employees of herein private reads:
respondent International Container Terminal Services, Inc. (ICTSI) and are
officers/members of Associated Port Checkers & Workers Union- WHEREFORE, decision is hereby rendered declaring that the correct divisor
International Container Terminal Services, Inc. Local Chapter (APCWU- in computing the daily wage and other labor standard benefits of the
ICTSI), a labor organization duly registered as a local affiliate of the employees of respondent ICTSI who are members of complainant Union as
Associated Port Checkers & Workers Union (APCWU). well as the other employees similarly situated is two hundred fifty (250) days
such that said respondent is hereby ordered to pay the employees concerned
When ICTSI started its operations in 1988, it determined the rate of pay of the differentials representing the underpayment of said salaries and other
its employees by using 304 days, the number of days of work of the benefits reckoned three (3) years back from February 26, 1997, the date of
employees in a year, as divisor.2 filing of this complaint or computed from February 27 1994 until paid, but
for purposes of appeal, the salary differentials are temporarily computed for
On September 28, 1990, ICTSI entered into its first Collective Bargaining one year in the amount of Four Hundred Sixty Eight Thousand Forty Pesos
Agreement (CBA) with APCWU with a term of five years effective until (P468,040.00).18
September 28, 1995.3 The CBA was renegotiated and thereafter renewed
through a second CBA that took effect on September 29, 1995, effective for In the same decision, the Labor Arbiter denied petitioners-appellants’
another five years.4 Both CBAs contained an identically-worded provision Complaint-in-Intervention with Motion for Intervention upon a finding that
on hours and days of work reading: they are already well represented by APCWU.19

Article IX On appeal, the NLRC reversed the decision of the Labor Arbiter and
dismissed APCWU’s complaint for lack of merit.20 The denial of
Regular Hours of Work and Days of Labor petitioners-appellants’ intervention was, however, affirmed.21

Section 1. The regular working days in a week shall be five (5) days on any Unsatisfied with the decision of the NLRC, APCWU filed a petition for
day from Monday to Sunday, as may be scheduled by the COMPANY, upon certiorari with the Court of Appeals while petitioners-appellants filed theirs
seven (7) days prior notice unless any of this day is declared a special with this Court which referred the petition22 to the Court of Appeals.
holiday.5 (Italics omitted)
The Court of Appeals dismissed APCWU’s petition on the following
In accordance with the above-quoted provision of the CBA, the employees’ grounds: failure to allege when its motion for reconsideration of the NLRC
work week was reduced to five days or a total of 250 days a year. ICTSI, decision was filed, failure to attach the necessary appendices to the petition,
however, continued using the 304-day divisor in computing the wages of the and failure to file its motion for extension to file its petition within the
employees.6 reglementary period.23

On November 10, 1990, the Regional Tripartite Wage and Productivity As for petitioners-appellants’ petition for certiorari, it was dismissed by the
Board (RTWPB) in the National Capital Region decreed a P17.00 daily wage Court of Appeals in this wise:
increase for all workers and employees receiving P125.00 per day or lower
in the National Capital Region.7 The then president of APCWU, together It is clear from the records that herein petitioners, claiming to be employees
with some union members, thus requested the ICTSI’s Human Resource of respondent ICTSI, are already well represented by its employees union,
Department/Personnel Manager to compute the actual monthly increase in APCWU, in the petition before this Court (CA-G.R. SP. No. 53266) although
the employees’ wages by multiplying the RTWPB mandated increase by 365 the same has been dismissed. The present petition is, therefore a superfluity
days and dividing the product by 12 months.8 that deserves to be dismissed. Furthermore, only Acedera signed the
Certificate of non-forum shopping. On this score alone, this petition should
Heeding the proposal and following the implementation of the new wage likewise be dismissed. We find that the same has no merit considering that
order, ICTSI stopped using 304 days as divisor and started using 365 days in herein petitioners have not presented any meritorious argument that would
determining the daily wage of its employees and other consequential justify the reversal of the Decision of the NLRC.
compensation, even if the employees’ work week consisted of only five days
as agreed upon in the CBA.9 Article IX of the CBA provides:

In early 1997, ICTSI went on a retrenchment program and laid off its on-call Regular Hours of Work and Days of Labor
employees.10 This prompted the APCWU-ICTSI to file a notice of strike
which included as cause of action not only the retrenchment of the employees "Section 1. The regular working days in a week shall be five (5) days on any
but also ICTSI’s use of 365 days as divisor in the computation of wages.11 day from Monday to Sunday, as may be scheduled by the COMPANY, upon
The dispute respecting the retrenchment was resolved by a compromise seven (7) days prior notice unless any of this day is declared a special
settlement12 while that respecting the computation of wages was referred to holiday."
the Labor Arbiter.13
This provision categorically states the required number of working days an
On February 26, 1997, APCWU, on behalf of its members and other employee is expected to work for a week. It does not, however, indicate the
employees similarly situated, filed with the Labor Arbiter a complaint against manner in which an employee’s salary is to be computed. In fact, nothing in
ICTSI which was dismissed for APCWU’s failure to file its position paper.14 the CBA makes any referral to any divisor which should be the basis for
Upon the demand of herein petitioners-appellants, APCWU filed a motion to determining the salary. The NLRC, therefore, correctly ruled that" xxx the
revive the case which was granted. APCWU thereupon filed its position absence of any express or specific provision in the CBA that 250 days should
paper on August 22, 1997.15 be used as divisor altogether makes the position of the Union untenable."

xxx
Considering that herein petitioners themselves requested that 365 days be similarly situated," the title of the case filed by it at the Labor Arbiter’s Office
used as the divisor in computing their wage increase and later did not raise so expressly states.
or object to the same during the negotiations of the new CBA, they are clearly
estopped to now complain of such computation only because they no longer While a party acting in a representative capacity, such as a union, may be
benefit from it. Indeed, the 365 divisor for the past seven (7) years has already permitted to intervene in a case, ordinarily, a person whose interests are
become practice and law between the company and its employees.24 already represented will not be permitted to do the same28 except when there
(Emphasis supplied) is a suggestion of fraud or collusion or that the representative will not act in
good faith for the protection of all interests represented by him.29
xxx
Petitioners-appellants cite the dismissal of the case filed by ICTSI, first by
Hence, the present petition of petitioners-appellants who fault the Court of the Labor Arbiter, and later by the Court of Appeals.30 The dismissal of the
Appeals as follows: case does not, however, by itself show the existence of fraud or collusion or
a lack of good faith on the part of APCWU. There must be clear and
I convincing evidence of fraud or collusion or lack of good faith independently
of the dismissal. This, petitioners-appellants failed to proffer.
. . . in rejecting the CBA of the parties as the source of the divisor to determine
the workers’ daily rate totally disregarded the applicable landmark decisions Petitioners-appellants likewise express their fear that APCWU would not
of the Honorable Supreme Court on the matter. prosecute the case diligently because of its "sweetheart relationship" with
ICTSI.31 There is nothing on record, however, to support this alleged
II relationship which allegation surfaces as a mere afterthought because it was
never raised early on. It was raised only in petitioners-appellants’ reply to
. . . [IN] disregard[ING] applicable decisions of this Honorable Court when ICTSI’s comment in the petition at bar, the last pleading submitted to this
it ruled that the petitioners-appellants are already in estoppel. Court, which was filed on June 20, 2001 or more than 42 months after
petitioners-appellants filed their Complaint-in-Intervention with Motion to
III Intervene with the Labor Arbiter.

. . . in ruling that the petitioners-appellants have no legal right to intervene in To reiterate, for a member of a class to be permitted to intervene in a
and pursue this case and that their intervention is a superfluity. representative action, fraud or collusion or lack of good faith on the part of
the representative must be proven. It must be based on facts borne on record.
IV Mere assertions, as what petitioners-appellants proffer, do not suffice.

. . . in holding, although merely as an obiter dictum, that only petitioner Jerry The foregoing discussion leaves it unnecessary to discuss the other assigned
Acedera signed the certificate of non-forum shopping.25 errors.

The third assigned error respecting petitioners-appellants’ right to intervene WHEREFORE, the present petition is hereby denied.
shall first be passed upon, it being determinative of their right to raise the
other assigned errors. SO ORDERED.

Petitioners-appellants anchor their right to intervene on Rule 19 of the 1997


Rules of Civil Procedure, Section 1 of which reads:

Section 1. Who may intervene.- A person who has legal interest in the matter
in litigation, or in the success of either of the parties, or an interest against
both, or is so situated to be adversely affected by a distribution or other
disposition of property in the custody of the court or of an officer thereof
may, with leave of court, be allowed to intervene in the action. The court
shall consider whether or not the intervention will unduly delay or prejudice
the adjudication of the rights of the original parties, and whether or not the
intervenor’s right may be fully protected in a separate proceeding.

They stress that they have complied with the requisites for intervention
because (1) they are the ones who stand to gain or lose by the direct legal
operation and effect of any judgment that may be rendered in this case, (2)
no undue delay or prejudice would result from their intervention since their
Complaint-in-Intervention with Motion for Intervention was filed while the
Labor Arbiter was still hearing the case and before any decision thereon was
rendered, and (3) it was not possible for them to file a separate case as they
would be guilty of forum shopping because the only forum available for them
was the Labor Arbiter.26

Petitioners-appellants, however, failed to consider, in addition to the rule on


intervention, the rule on representation, thusly:

Sec. 3. Representatives as parties.- Where the action is allowed to be


prosecuted or defended by a representative or someone acting in a fiduciary
capacity, the beneficiary shall be included in the title of the case and shall be
deemed to be the real party in interest. A representative may be a trustee of
an express trust, a guardian, an executor or administrator, or a party
authorized by law or these Rules. . . 27 (Emphasis supplied)

A labor union is one such party authorized to represent its members under
Article 242(a) of the Labor Code which provides that a union may act as the
representative of its members for the purpose of collective bargaining. This
authority includes the power to represent its members for the purpose of
enforcing the provisions of the CBA. That APCWU acted in a representative
capacity "for and in behalf of its Union members and other employees
BAPTISTA vs. VILLANUEVA Article 249 (a) and (b) of the Labor Code. The LA clarified that only the
G.R. No. 194709 July 31, 2013 union officers of RPNEU could be held responsible for ULP, so they
MENDOZA, J.: exonerated six (6) of the original defendants who were mere union members.
The LA also ordered the reinstatement of petitioners as bonafide members of
This Petition for Review on Certiorari1 under Rule 45 of the 1997 Rules of RPNEU. The decretal portion reads:
Civil Procedure filed by Minette Baptista, Bannie Edsel San Miguel and Ma.
Fe Dayon (petitioners) assails the March 9, 2010 Decision2 and the WHEREFORE, premises above considered, a decision is being issued
December 1, 2010 Resolution3 of the Court of Appeals (CA) in CA-G.R. SP declaring union officers Ruth Bayquen, Ruby Castañeda, Alfred Landas,
No. 105027, which affirmed the March 31, 2008 Decision4 of the National Roce Garces, Board of Directors Federico Muñoz, Janette Roldan, Rosario
Labor Relations Commission (NLRC) dismissing the complaint for Unfair Villanueva, Menandro Samson, Salvador Diwa and Eugene Cruz guilty of
Labor Practice (ULP) filed against the named respondents. unfair labor practice for violating Article 249, paragraph A and B of the
Labor Code. Respondents are also ordered to cease and desist from further
The Facts committing unfair labor practice and order the reinstatement of the
complainants as bonafide members of the union.
Petitioners were former union members of Radio Philippines Network
Employees Union (RPNEU), a legitimate labor organization and the sole and The other claims are hereby denied for lack of factual and legal basis.
exclusive bargaining agent of the rank and file employees of Radio
Philippines Network (RPN), a government-sequestered corporation involved SO ORDERED.20
in commercial radio and television broadcasting affairs, while the
respondents were the union’s elected officers and members. Undaunted, the respondents appealed the LA decision to the NLRC.

On April 26, 2005, on suspicion of union mismanagement, petitioners, In its Decision,21 dated March 31, 2008, the NLRC vacated and set aside the
together with some other union members, filed a complaint for impeachment LA decision and dismissed the complaint for ULP for lack of merit. The
of their union president, Reynato Siozon, before the executive board of RPN, NLRC found that petitioners filed a suit calling for the impeachment of the
which was eventually abandoned. They later re-lodged the impeachment officers and members of the Executive Board of RPNEU without first
complaint, this time, against all the union officers and members of RPNEU resorting to internal remedies available under its own Constitution and By-
before the Department of Labor and Employment (DOLE). They likewise Laws. The NLRC likewise decreed that the LA’s order of reinstatement was
filed various petitions for audit covering the period from 2000 to 2004.5 improper because the legality of the membership expulsion was not raised in
the proceedings and, hence, beyond the jurisdiction of the LA.22 The fallo
Thereafter, two (2) written complaints, dated May 26, 2005 and May 27, of the NLRC decision reads:
2005, were filed against petitioners and several others for alleged violation
of the union’s Constitution and By-Laws.6 Months later, on September 19, WHEREFORE, the partial appeal filed by the respondents is GRANTED.
2005, a different group of union members filed a third complaint against The decision, dated 30 April 2007 is VACATED and SET ASIDE. The
petitioners and 12 others,7 before the Chairman of RPNEU’s Committee on complaint is dismissed for lack of merit.
Grievance and Investigation (the Committee) citing as grounds the
"commission of an act which violates RPNEU Constitution and By-Laws, SO ORDERED.23
specifically, Article IX, Section 2.2 for joining or forming a union outside
the sixty (60) days period and Article IX, Section 2.5 for urging or advocating Petitioners filed for a motion for reconsideration, but the NLRC denied it in
that a member start an action in any court of justice or external investigative its Resolution,24 dated May 30, 2008.
body against the Union or its officer without first exhausting all internal
remedies open to him or available in accordance with the CBL."8 These The CA, in its March 9, 2010 Decision, sustained the NLRC decision. The
complaints were, later on, consolidated.9 CA stated that the termination of employment by virtue of a union security
clause was recognized in our jurisdiction. It explained that the said practice
Thereafter, petitioners received a memorandum notice from Jeric Salinas, fortified the union and averted disunity in the bargaining unit within the
Chairman of the Committee, requesting them to answer the complaint and duration of the CBA. The CA declared that petitioners were accorded due
attend a hearing scheduled on October 3, 2005.10 Petitioners and their group, process before they were removed from office. In fact, petitioners were given
through an exchange of communications with the Committee, denied the the opportunity to explain their case and they actually availed of said
charges imputed against them and contested the procedure adopted by the opportunity by submitting letters containing their arguments.25
Committee in its investigation. On November 9, 2005, the Committee
submitted their recommendation of expulsion from the union to RPNEU’s Petitioners moved for reconsideration, but the CA likewise denied the same
Board of Directors.11 On December 21, 2005, the RPNEU’s Board of in its December 1, 2010 Resolution,26 The CA expounded:
Directors affirmed the recommendation of expulsion of petitioners and the
12 others from union membership in a Board Resolution No. 018-2005.12 Anent petitioners’ charge of ULP against respondents, the records are barren
Through a Memorandum,13 dated December 27, 2005, petitioners were of proof to sustain such charge. What remains apparent is that petitioners
served an expulsion notice from the union, which was set to take effect on were expelled from the union due to their violation of Section 2.5 of Article
December 29, 2005. On January 2, 2006, petitioners with the 12 others wrote IX of the CBL which punishes the act of "urging or advocating that a member
to RPNEU’s President and Board of Directors that their expulsion from the start an action in any court of justice or external investigative body against
union was an ultra vires act because the Committee failed to observe the basic the Union or any of its officer, without first exhausting all internal remedies
elements of due process because they were not given the chance to physically open to him or available in accordance with the Constitution and By-Laws
confront and examine their complainants.14 of Union." As petitioners’ expulsion was pursuant to the union’s CBL, We
absolve respondents of the charges of ULP absent any substantial evidence
In a letter, dated January 24, 2006, RPNEU’s officers informed their to sustain it.
company of the expulsion of petitioners and the 12 others from the union and
requested the management to serve them notices of termination from The importance of a union’s constitution and bylaws cannot be
employment in compliance with their CBA’s union security clause.15 On overemphasized. They embody a covenant between a union and its members
February 17, 2006, RPN HRD Manager, Lourdes Angeles, informed and constitute the fundamental law governing the member’s rights and
petitioners and the 12 others of the termination of their employment effective obligations. As such, the union’s constitution and bylaws should be upheld,
March 20, 2006, enforcing Article II, Section 216 also known as the union as long as they are not contrary to law, good morals or public policy. In
security clause of their current CBA.17 Diamonon v. Department of Labor and Employment, the High Court
affirmed the validity and importance of the provision in the CBL of
Aggrieved, petitioners filed three (3) separate complaints for ULP against the exhaustion of administrative remedies, viz:
respondents, which were later consolidated,18 questioning legality of their
expulsion from the union and their subsequent termination from When the Constitution and by-laws of both unions dictated the remedy for
employment. intra-union dispute, such as petitioner’s complaint against private
respondents for unauthorized or illegal disbursement of union funds, this
In a decision,19 dated April 30, 2007, the Labor Arbiter (LA) ruled in favor should be resorted to before recourse can be made to the appropriate
of the petitioners and adjudged the respondents guilty of ULP pursuant to administrative or judicial body, not only to give the grievance machinery or
appeals’ body of the union the opportunity to decide the matter by itself, but proper investigation because, if proven, would constitute grounds for their
also to prevent unnecessary and premature resort to administrative or judicial expulsion from the union. As such, Article X, Investigation Procedures and
bodies. Thus, a party with an administrative remedy must not merely initiate Appeal Process of RPNEU’s Constitution and By-Laws, which reads –
the prescribed administrative procedure to obtain relief, but also pursue it to
its appropriate conclusion before seeking judicial intervention.27 SECTION 1. Charge against any member or officer of the Union shall be
submitted to the Board of Directors (BOD) in writing, which shall refer the
Thus, petitioners advance the following same, if necessary, to the committee on Grievance and Investigation. The
Committee shall hear any charge and subsequently, forward its finding and
GROUNDS/ARGUMENTS IN SUPPORT OF THE PETITION recommendation to the BOD. The BOD has the power to approve or nullify
the recommendation of the Committee on Grievance and Investigation based
1. WITH DUE RESPECT, THE HONORABLE COURT OF APPEALS on the merit of the appeal.
MISERABLY FAILED TO APPRECIATE THE REAL ISSUE IN THIS
CASE. was correctly applied under the circumstances.

2. WITH DUE RESPECT, THE DECISION AND RESOLUTION Besides, any supposed procedural flaw in the proceedings before the
ARRIVED AT BY THE HONORABLE COURT OF APPEALS ARE NOT Committee was deemed cured when petitioners were given the opportunity
IN ACCORD WITH LAW AND APPLICABLE JURISPRUDENCE, to be heard. Due process, as a constitutional precept, is satisfied when a
THEREBY GRAVELY ABUSING ITS DISCRETION AMOUNTING TO person was notified of the charge against him and was given an opportunity
LACK OR IN EXCESS OF JURISDICTION.28 to explain or defend himself. In administrative proceedings, the filing of
charges and giving reasonable opportunity for the person so charged to
Petitioners submit that the respondents committed ULP under Article 289 (a) answer the accusations against him constitute the minimum requirements of
and (b) of the Labor Code.29 They insist that they were denied substantive due process.34 The essence of due process is simply to be heard, or as applied
and procedural due process of law when they were expelled from the to administrative proceedings, an opportunity to explain one’s side, or an
RPNEU. opportunity to seek a reconsideration of the action or ruling complained of.35
It cannot be denied that petitioners were properly notified of the charges filed
The petition is bereft of merit. against them and were equally afforded the opportunity to present their side.

The primary concept of ULP is embodied in Article 247 of the Labor Code, Next, petitioners point out that they were not given the opportunity to
which provides: personally face and confront their accusers, which were violative of their
right to examine the complainants and the supposed charges against them.36
Article 247. Concept of unfair labor practice and procedure for prosecution
thereof.––Unfair labor practices violate the constitutional right of workers Petitioners’ contention is without merit. Mere absence of a one-onone
and employees to self-organization, are inimical to the legitimate interests of confrontation between the petitioners and their complainants does not
both labor and management, including their right to bargain collectively and automatically affect the validity of the proceedings before the Committee.
otherwise deal with each other in an atmosphere of freedom and mutual Not all cases necessitate a trial-type hearing.37 As in this case, what is
respect, disrupt industrial peace and hinder the promotion of healthy and indispensable is that a party be given the right to explain one’s side, which
stable labor-management relations. was adequately afforded to the petitioners.

In essence, ULP relates to the commission of acts that transgress the workers’ It is well-settled that workers’ and employers’ organizations shall have the
right to organize. As specified in Articles 248 and 249 of the Labor Code, the right to draw up their constitutions and rules to elect their representatives in
prohibited acts must necessarily relate to the workers' right to self- full freedom, to organize their administration and activities and to formulate
organization and to the observance of a CBA.30 Absent the said vital their programs.38 In this case, RPNEU’s Constitution and By-Laws
elements, the acts complained, although seemingly unjust, would not expressly mandate that before a party is allowed to seek the intervention of
constitute ULP.31 the court, it is a pre-condition that he should have availed of all the internal
remedies within the organization. Petitioners were found to have violated the
In the case at bench, petitioners claim that the respondents, as union officers, provisions of the union’s Constitution and By-Laws when they filed petitions
are guilty of ULP for violating paragraphs (a) and (b) of Article 249 of the for impeachment against their union officers and for audit before the DOLE
Labor Code, to wit: without first exhausting all internal remedies available within their
organization. This act is a ground for expulsion from union membership.
ART. 249. UNFAIR LABOR PRACTICES OF LABOR Thus, petitioners’ expulsion from the union was not a deliberate attempt to
ORGANIZATIONS.- It shall be unfair labor practice for a labor curtail or restrict their right to organize, but was triggered by the commission
organization, its officers, agents or representatives: of an act, expressly sanctioned by Section 2.5 of Article IX of the union’s
Constitution and By-Laws.1âwphi1
(a) To restrain or coerce employees in the exercise of their rights to self-
organization. However, a labor organization shall have the right to prescribe For a charge of ULP against a labor organization to prosper, the onus
its own rules with respect to the acquisition or retention of membership: probandi rests upon the party alleging it to prove or substantiate such claims
by the requisite quantum of evidence.39 In labor cases, as in other
(b) To cause or attempt to cause an employer to discriminate against an administrative proceedings, substantial evidence or such relevant evidence
employee, including discrimination against an employee with respect to as a reasonable mind might accept as sufficient to support a conclusion is
whom membership in such organization has been denied or to terminate an required.40 Moreover, it is indubitable that all the prohibited acts constituting
employee on any ground other than the usual terms and conditions under unfair labor practice should materially relate to the workers' right to self-
which membership or continuation of membership is made available to other organization.41
members;
Unfortunately, petitioners failed to discharge the burden required to prove
Petitioners posit that the procedure that should have been followed by the the charge of ULP against the respondents. Aside from their self-serving
respondents in resolving the charges against them was Article XVII, allegations, petitioners were not able to establish how they were restrained
Settlement of Internal Disputes of their Constitution and By-Laws, or coerced by their union in a way that curtailed their right to self-
specifically, Section 232 thereof, requiring members to put their grievance in organization. The records likewise failed to sufficiently show that the
writing to be submitted to their union president, who shall strive to have the respondents unduly persuaded management into discriminating against
parties settle their differences amicably. Petitioners maintain that any form petitioners. other than to bring to its attention their expulsion from the union,
of grievance would be referred only to the committee upon failure of the which in turn, resulted in the implementation of their CBA' s union security
parties to settle amicably.33 clause. As earlier stated, petitioners had the burden of adducing substantial
evidence to support its allegations of ULP,42 which burden they failed to
The Court is not persuaded. discharge. In fact, both the NLRC and the CA found that petitioners were
unable to prove their charge of ULP against the respondents.
Based on RPNEU’s Constitution and By-Laws, the charges against
petitioners were not mere internal squabbles, but violations that demand
It is axiomatic that absent any clear showing of abuse, arbitrariness or
capriciousness, the findings of fact by the NLRC, especially when affirmed
by the CA, as in this case, are binding and conclusive upon the Court.43
Having found none, the Court finds no cogent reason to deviate from the
challenged decision.

WHEREFORE, the petition is DENIED. The March 9, 2010 Decision and


the December 1, 2010 Resolution of the Court of Appeals in CA-G.R. SP No.
105027 are AFFIRMED.

SO ORDERED.
THE HERITAGE HOTEL MANILA vs. PINAG-ISANG GALING AT and the list as well as in the number of signatories to the attendance and
LAKAS NG MGA MANGGAGAWA SA HERITAGE MANILA signature sheets. The minutes reported that only 90 employees attended the
(PIGLAS-HERITAGE) meeting. The company further alleged that 33 members of respondent
G.R. No. 177024 October 30, 2009 PIGLAS union were members of the defunct HHE union. This, according to
ABAD, J.: the company, violated the policy against dual unionism and showed that the
new union was merely an alter ego of the old.
This case is about a company’s objections to the registration of its rank and
file union for non-compliance with the requirements of its registration. On February 22, 2005 the DOLE-NCR denied the company’s petition to
cancel respondent PIGLAS union’s registration for the reason that the
The Facts and the Case discrepancies in the number of members stated in the application’s
supporting documents were not material and did not constitute
Sometime in 2000, certain rank and file employees of petitioner Heritage misrepresentation. As for the charge of dual unionism, the same is not a
Hotel Manila (petitioner company) formed the "Heritage Hotel Employees ground for canceling registration. It merely exposed a union member to a
Union" (the HHE union). The Department of Labor and Employment- possible charge of disloyalty, an internal matter. Here, the members of the
National Capital Region (DOLE-NCR) later issued a certificate of former union simply exercised their right to self-organization and to the
registration1 to this union. freedom of association when they subsequently joined the PIGLAS union.19

Subsequently, the HHE union filed a petition for certification election2 that On appeal, the Bureau of Labor Relation (BLR) affirmed the ruling of the
petitioner company opposed. The company alleged that the HHE union DOLE-NCR. It reasoned that respondent PIGLAS union’s organization
misrepresented itself to be an independent union, when it was, in truth, a local meeting lasted for 12 hours. It was possible for the number of attendees to
chapter of the National Union of Workers in Hotel and Restaurant and Allied have increased from 90 to 128 as the meeting progressed. Besides, with a
Industries (NUWHRAIN). The company claimed that the HHE union total of 250 employees in the bargaining unit, the union needed only 50
intentionally omitted disclosure of its affiliation with NUWHRAIN because members to comply with the 20 percent membership requirement. Thus, the
the company’s supervisors union was already affiliated with it.3 Thus, the union could not be accused of misrepresentation since it did not pad its
company also filed a petition for the cancellation of the HHE union’s membership to secure registration.
registration certificate.4
As for the issue of dual unionism, it has become moot and academic, said the
Meanwhile, the Med-Arbiter granted the HHE union’s petition for BLR, because of the dissolution of the old union and the cancellation of its
certification election.5 Petitioner company appealed the decision to the certificate of registration.20
Secretary of Labor but the latter denied the appeal.6 The Secretary also
denied petitioner’s motion for reconsideration, prompting the company to file Petitioner company filed a petition for certiorari with the Court of Appeals,21
a petition for certiorari7 with the Court of Appeals. assailing the order of the BLR. But the latter court dismissed the petition, not
being accompanied by material documents and portions of the record.22 The
On October 12, 2001 the Court of Appeals issued a writ of injunction against company filed a motion for reconsideration, attaching parts of the record that
the holding of the HHE union’s certification election, effective until the were deemed indispensable but the court denied it for lack of merit.23 Hence,
petition for cancellation of that union’s registration shall have been resolved the company filed this petition for review under Rule 45.
with finality.8 The decision of the Court of Appeals became final when the
HHE union withdrew the petition for review that it filed with this Court.9 Issues Presented

On December 10, 2003 certain rank and file employees of petitioner The petition presents the following issues:
company held a meeting and formed another union, the respondent Pinag-
Isang Galing at Lakas ng mga Manggagawa sa Heritage Manila (the PIGLAS 1. Whether or not the Court of Appeals erred in dismissing the petition for
union). This union applied for registration with the DOLE-NCR10 and got certiorari before it for failure of petitioner company to attach certain material
its registration certificate on February 9, 2004. Two months later, the portions of the record;
members of the first union, the HHE union, adopted a resolution for its
dissolution. The HHE union then filed a petition for cancellation of its union 2. Whether or not the union made fatal misrepresentation in its application
registration.11 for union registration; and

On September 4, 2004 respondent PIGLAS union filed a petition for 3. Whether or not "dual unionism" is a ground for canceling a union’s
certification election12 that petitioner company also opposed, alleging that registration.
the new union’s officers and members were also those who comprised the
old union. According to the company, the employees involved formed the The Rulings of the Court
PIGLAS union to circumvent the Court of Appeals’ injunction against the
holding of the certification election sought by the former union. Despite the First. While the Court of Appeals correctly dismissed the company’s petition
company’s opposition, however, the Med-Arbiter granted the petition for initially for failure to attach material portions of the record, the court should
certification election.13 have bended back a little when petitioner company subsequently attached
those missing materials to its motion for reconsideration. As a general rule,
On December 6, 2004 petitioner company filed a petition to cancel the union petitions for certiorari that lack copies of essential pleadings and portions of
registration of respondent PIGLAS union.14 The company claimed that the the record may be dismissed but this rule has not been regarded as absolute.
documents submitted with the union’s application for registration bore the The omission may be cured.24
following false information:
The Court of Appeals has three courses of action when the annexes to the
(a) The List of Members showed that the PIGLAS union had 100 union petition are insufficient. It may dismiss the petition,25 require the submission
members;15 of the relevant documents, or order the filing of an amended petition with the
required pleadings or documents. A petition lacking in essential pleadings or
(b) The Organizational Minutes said that 90 employees attended the meeting portions of the record may still be given due course, or reinstated if earlier
on December 10, 2003;16 dismissed, upon subsequent submission of the necessary documents or to
serve the higher interest of justice.26
(c) The Attendance Sheet of the meeting of December 10, 2003 bore the
signature of 127 members who ratified the union’s Constitution and By- Second. Since a remand of the case to the Court of Appeals for a
Laws;17 and determination of the substantive issues will only result in more delays and
since these issues have been amply argued by the opposing sides in the
(d) The Signature Sheet bore 128 signatures of those who attended that various pleadings and documents they submitted to this Court, the case may
meeting.18 now be resolved on the merits.

Petitioner company alleged that the misrepresentation was evidenced by the Did respondent PIGLAS union commit fraud and misrepresentation in its
discrepancy in the number of union members appearing in the application application for union registration? We agree with the DOLE-NCR and the
BLR that it did not. Except for the evident discrepancies as to the number of
union members involved as these appeared on the documents that supported
the union’s application for registration, petitioner company has no other
evidence of the alleged misrepresentation. But those discrepancies alone
cannot be taken as an indication that respondent misrepresented the
information contained in these documents.

The charge that a labor organization committed fraud and misrepresentation


in securing its registration is a serious charge and deserves close scrutiny. It
is serious because once such charge is proved, the labor union acquires none
of the rights accorded to registered organizations. Consequently, charges of
this nature should be clearly established by evidence and the surrounding
circumstances.27

Here, the discrepancies in the number of union members or employees stated


in the various supporting documents that respondent PIGLAS union
submitted to labor authorities can be explained. While it appears in the
minutes of the December 10, 2003 organizational meeting that only 90
employees responded to the roll call at the beginning, it cannot be assumed
that such number could not grow to 128 as reflected on the signature sheet
for attendance. The meeting lasted 12 hours from 11:00 a.m. to 11:00 p.m.
There is no evidence that the meeting hall was locked up to exclude late
attendees.1 a vv p h i 1

There is also nothing essentially mysterious or irregular about the fact that
only 127 members ratified the union’s constitution and by-laws when 128
signed the attendance sheet. It cannot be assumed that all those who attended
approved of the constitution and by-laws. Any member had the right to hold
out and refrain from ratifying those documents or to simply ignore the
process.

At any rate, the Labor Code28 and its implementing rules29 do not require
that the number of members appearing on the documents in question should
completely dovetail. For as long as the documents and signatures are shown
to be genuine and regular and the constitution and by-laws democratically
ratified, the union is deemed to have complied with registration requirements.

Petitioner company claims that respondent PIGLAS union was required to


submit the names of all its members comprising at least 20 percent of the
employees in the bargaining unit. Yet the list it submitted named only 100
members notwithstanding that the signature and attendance sheets reflected
a membership of 127 or 128 employees. This omission, said the company,
amounted to material misrepresentation that warranted the cancellation of the
union’s registration.

But, as the labor authorities held, this discrepancy is immaterial. A


comparison of the documents shows that, except for six members, the names
found in the subject list are also in the attendance and signature sheets.
Notably, the bargaining unit that respondent PIGLAS union sought to
represent consisted of 250 employees. Only 20 percent of this number or 50
employees were required to unionize. Here, the union more than complied
with such requirement.

Labor laws are liberally construed in favor of labor especially if doing so


would affirm its constitutionally guaranteed right to self-organization.30
Here, the PIGLAS union’s supporting documents reveal the unmistakable
yearning of petitioner company’s rank and file employees to organize. This
yearning should not be frustrated by inconsequential technicalities.

Third. The fact that some of respondent PIGLAS union’s members were also
members of the old rank and file union, the HHE union, is not a ground for
canceling the new union’s registration. The right of any person to join an
organization also includes the right to leave that organization and join
another one. Besides, HHE union is dead. It had ceased to exist and its
certificate of registration had already been cancelled. Thus, petitioner’s
arguments on this point may also be now regarded as moot and academic.

WHEREFORE, the Court DENIES the petition and AFFIRMS the decision
of the Bureau of Labor Relations in BLR-A-26-3-05 dated May 26, 2006.

SO ORDERED.
THE HERITAGE HOTEL MANILA vs. NATIONAL UNION OF are the ones who would be disadvantaged by the non-submission of financial
WORKERS IN THE HOTEL, RESTAURANT AND ALLIED reports; (c) it has already complied with the reportorial requirements, having
INDUSTRIES-HERITAGE HOTEL MANILA SUPERVISORS submitted its financial statements for 1996, 1997, 1998, and 1999, its updated
CHAPTER (NUWHRAIN-HHMSC) list of officers, and its list of members for the years 1995, 1996, 1997, 1998,
G.R. No. 178296 January 12, 2011 and 1999; (d) the petition is already moot and academic, considering that the
NACHURA, J.: certification election had already been held, and the members had manifested
their will to be represented by respondent.
Before the Court is a petition for review on certiorari of the Decision1 of the
Court of Appeals (CA) dated May 30, 2005 and Resolution dated June 4, Citing National Union of Bank Employees v. Minister of Labor, et al.9 and
2007. The assailed Decision affirmed the dismissal of a petition for Samahan ng Manggagawa sa Pacific Plastic v. Hon. Laguesma,10 the Med-
cancellation of union registration filed by petitioner, Grand Plaza Hotel Arbiter held that the pendency of a petition for cancellation of registration is
Corporation, owner of Heritage Hotel Manila, against respondent, National not a bar to the holding of a certification election. Thus, in an Order11 dated
Union of Workers in the Hotel, Restaurant and Allied Industries-Heritage January 26, 2001, the Med-Arbiter dismissed petitioner’s protest, and
Hotel Manila Supervisors Chapter (NUWHRAIN-HHMSC), a labor certified respondent as the sole and exclusive bargaining agent of all
organization of the supervisory employees of Heritage Hotel Manila. supervisory employees.

The case stemmed from the following antecedents: Petitioner subsequently appealed the said Order to the DOLE Secretary.12
The appeal was later dismissed by DOLE Secretary Patricia A. Sto. Tomas
On October 11, 1995, respondent filed with the Department of Labor and (DOLE Secretary Sto. Tomas) in the Resolution of August 21, 2002.13
Employment-National Capital Region (DOLE-NCR) a petition for Petitioner moved for reconsideration, but the motion was also denied.14
certification election.2 The Med-Arbiter granted the petition on February 14,
1996 and ordered the holding of a certification election.3 On appeal, the In the meantime, Regional Director Alex E. Maraan (Regional Director
DOLE Secretary, in a Resolution dated August 15, 1996, affirmed the Med- Maraan) of DOLE-NCR finally resolved the petition for cancellation of
Arbiter’s order and remanded the case to the Med-Arbiter for the holding of registration. While finding that respondent had indeed failed to file financial
a preelection conference on February 26, 1997. Petitioner filed a motion for reports and the list of its members for several years, he, nonetheless, denied
reconsideration, but it was denied on September 23, 1996. the petition, ratiocinating that freedom of association and the employees’
right to self-organization are more substantive considerations. He took into
The preelection conference was not held as initially scheduled; it was held a account the fact that respondent won the certification election and that it had
year later, or on February 20, 1998. Petitioner moved to archive or to dismiss already been certified as the exclusive bargaining agent of the supervisory
the petition due to alleged repeated non-appearance of respondent. The latter employees. In view of the foregoing, Regional Director Maraan—while
agreed to suspend proceedings until further notice. The preelection emphasizing that the non-compliance with the law is not viewed with favor—
conference resumed on January 29, 2000. considered the belated submission of the annual financial reports and the list
of members as sufficient compliance thereof and considered them as having
Subsequently, petitioner discovered that respondent had failed to submit to been submitted on time. The dispositive portion of the decision15 dated
the Bureau of Labor Relations (BLR) its annual financial report for several December 29, 2001 reads:
years and the list of its members since it filed its registration papers in 1995.
Consequently, on May 19, 2000, petitioner filed a Petition for Cancellation WHEREFORE, premises considered, the instant petition to delist the
of Registration of respondent, on the ground of the non-submission of the National Union of Workers in the Hotel, Restaurant and Allied Industries-
said documents. Petitioner prayed that respondent’s Certificate of Creation Heritage Hotel Manila Supervisors Chapter from the roll of legitimate labor
of Local/Chapter be cancelled and its name be deleted from the list of organizations is hereby DENIED.
legitimate labor organizations. It further requested the suspension of the
certification election proceedings.4 SO ORDERED.16

On June 1, 2000, petitioner reiterated its request by filing a Motion to Dismiss Aggrieved, petitioner appealed the decision to the BLR.17 BLR Director
or Suspend the [Certification Election] Proceedings,5 arguing that the Hans Leo Cacdac inhibited himself from the case because he had been a
dismissal or suspension of the proceedings is warranted, considering that the former counsel of respondent.
legitimacy of respondent is seriously being challenged in the petition for
cancellation of registration. Petitioner maintained that the resolution of the In view of Director Cacdac’s inhibition, DOLE Secretary Sto. Tomas took
issue of whether respondent is a legitimate labor organization is crucial to the cognizance of the appeal. In a resolution18 dated February 21, 2003, she
issue of whether it may exercise rights of a legitimate labor organization, dismissed the appeal, holding that the constitutionally guaranteed freedom of
which include the right to be certified as the bargaining agent of the covered association and right of workers to self-organization outweighed
employees. respondent’s noncompliance with the statutory requirements to maintain its
status as a legitimate labor organization.
Nevertheless, the certification election pushed through on June 23, 2000.
Respondent emerged as the winner.6 Petitioner filed a motion for reconsideration,19 but the motion was likewise
denied in a resolution20 dated May 30, 2003. DOLE Secretary Sto. Tomas
On June 28, 2000, petitioner filed a Protest with Motion to Defer admitted that it was the BLR which had jurisdiction over the appeal, but she
Certification of Election Results and Winner,7 stating that the certification pointed out that the BLR Director had voluntarily inhibited himself from the
election held on June 23, 2000 was an exercise in futility because, once case because he used to appear as counsel for respondent. In order to maintain
respondent’s registration is cancelled, it would no longer be entitled to be the integrity of the decision and of the BLR, she therefore accepted the
certified as the exclusive bargaining agent of the supervisory employees. motion to inhibit and took cognizance of the appeal.
Petitioner also claimed that some of respondent’s members were not
qualified to join the union because they were either confidential employees Petitioner filed a petition for certiorari with the CA, raising the issue of
or managerial employees. It then prayed that the certification of the election whether the DOLE Secretary acted with grave abuse of discretion in taking
results and winner be deferred until the petition for cancellation shall have cognizance of the appeal and affirming the dismissal of its petition for
been resolved, and that respondent’s members who held confidential or cancellation of respondent’s registration.
managerial positions be excluded from the supervisors’ bargaining unit.
In a Decision dated May 30, 2005, the CA denied the petition. The CA opined
Meanwhile, respondent filed its Answer8 to the petition for the cancellation that the DOLE Secretary may legally assume jurisdiction over an appeal from
of its registration. It averred that the petition was filed primarily to delay the the decision of the Regional Director in the event that the Director of the
conduct of the certification election, the respondent’s certification as the BLR inhibits himself from the case. According to the CA, in the absence of
exclusive bargaining representative of the supervisory employees, and the the BLR Director, there is no person more competent to resolve the appeal
commencement of bargaining negotiations. Respondent prayed for the than the DOLE Secretary. The CA brushed aside the allegation of bias and
dismissal of the petition for the following reasons: (a) petitioner is estopped partiality on the part of the DOLE Secretary, considering that such allegation
from questioning respondent’s status as a legitimate labor organization as it was not supported by any evidence.
had already recognized respondent as such during the preelection
conferences; (b) petitioner is not the party-in-interest, as the union members
The CA also found that the DOLE Secretary did not commit grave abuse of foregoing to the present case, it is clear that the DOLE Secretary, as the
discretion when she affirmed the dismissal of the petition for cancellation of person exercising the power of supervision and control over the BLR, has the
respondent’s registration as a labor organization. Echoing the DOLE authority to directly exercise the quasi-judicial function entrusted by law to
Secretary, the CA held that the requirements of registration of labor the BLR Director.
organizations are an exercise of the overriding police power of the State,
designed for the protection of workers against potential abuse by the union It is true that the power of control and supervision does not give the
that recruits them. These requirements, the CA opined, should not be Department Secretary unbridled authority to take over the functions of his or
exploited to work against the workers’ constitutionally protected right to self- her subordinate. Such authority is subject to certain guidelines which are
organization. stated in Book IV, Chapter 8, Section 39(1)(a) of the Administrative Code of
1987.29 However, in the present case, the DOLE Secretary’s act of taking
Petitioner filed a motion for reconsideration, invoking this Court’s ruling in over the function of the BLR Director was warranted and necessitated by the
Abbott Labs. Phils., Inc. v. Abbott Labs. Employees Union,21 which latter’s inhibition from the case and the objective to "maintain the integrity
categorically declared that the DOLE Secretary has no authority to review of the decision, as well as the Bureau itself."30
the decision of the Regional Director in a petition for cancellation of union
registration, and Section 4,22 Rule VIII, Book V of the Omnibus Rules Petitioner insists that the BLR Director’s subordinates should have resolved
Implementing the Labor Code. the appeal, citing the provision under the Administrative Code of 1987 which
states, "in case of the absence or disability of the head of a bureau or office,
In its Resolution23 dated June 4, 2007, the CA denied petitioner’s motion, his duties shall be performed by the assistant head."31 The provision clearly
stating that the BLR Director’s inhibition from the case was a peculiarity not does not apply considering that the BLR Director was neither absent nor
present in the Abbott case, and that such inhibition justified the assumption suffering from any disability; he remained as head of the BLR. Thus, to dispel
of jurisdiction by the DOLE Secretary. any suspicion of bias, the DOLE Secretary opted to resolve the appeal
herself.
In this petition, petitioner argues that:
Petitioner was not denied the right to due process when it was not notified in
I. advance of the BLR Director’s inhibition and the DOLE Secretary’s
assumption of the case. Well-settled is the rule that the essence of due process
The Court of Appeals seriously erred in ruling that the Labor Secretary is simply an opportunity to be heard, or, as applied to administrative
properly assumed jurisdiction over Petitioner’s appeal of the Regional proceedings, an opportunity to explain one’s side or an opportunity to seek a
Director’s Decision in the Cancellation Petition x x x. reconsideration of the action or ruling complained of.32 Petitioner had the
opportunity to question the BLR Director’s inhibition and the DOLE
A. Jurisdiction is conferred only by law. The Labor Secretary had no Secretary’s taking cognizance of the case when it filed a motion for
jurisdiction to review the decision of the Regional Director in a petition for reconsideration of the latter’s decision. It would be well to state that a critical
cancellation. Such jurisdiction is conferred by law to the BLR. component of due process is a hearing before an impartial and disinterested
tribunal, for all the elements of due process, like notice and hearing, would
B. The unilateral inhibition by the BLR Director cannot justify the Labor be meaningless if the ultimate decision would come from a partial and biased
Secretary’s exercise of jurisdiction over the Appeal. judge.33 It was precisely to ensure a fair trial that moved the BLR Director
to inhibit himself from the case and the DOLE Secretary to take over his
C. The Labor Secretary’s assumption of jurisdiction over the Appeal without function.
notice violated Petitioner’s right to due process.
Petitioner also insists that respondent’s registration as a legitimate labor
II. union should be cancelled. Petitioner posits that once it is determined that a
ground enumerated in Article 239 of the Labor Code is present, cancellation
The Court of Appeals gravely erred in affirming the dismissal of the of registration should follow; it becomes the ministerial duty of the Regional
Cancellation Petition despite the mandatory and unequivocal provisions of Director to cancel the registration of the labor organization, hence, the use of
the Labor Code and its Implementing Rules.24 the word "shall." Petitioner points out that the Regional Director has admitted
in its decision that respondent failed to submit the required documents for a
The petition has no merit. number of years; therefore, cancellation of its registration should have
followed as a matter of course.
Jurisdiction to review the decision of the Regional Director lies with the
BLR. This is clearly provided in the Implementing Rules of the Labor Code We are not persuaded.
and enunciated by the Court in Abbott. But as pointed out by the CA, the
present case involves a peculiar circumstance that was not present or covered Articles 238 and 239 of the Labor Code read:
by the ruling in Abbott. In this case, the BLR Director inhibited himself from
the case because he was a former counsel of respondent. Who, then, shall ART. 238. CANCELLATION OF REGISTRATION; APPEAL
resolve the case in his place?
The certificate of registration of any legitimate labor organization, whether
In Abbott, the appeal from the Regional Director’s decision was directly filed national or local, shall be canceled by the Bureau if it has reason to believe,
with the Office of the DOLE Secretary, and we ruled that the latter has no after due hearing, that the said labor organization no longer meets one or
appellate jurisdiction. In the instant case, the appeal was filed by petitioner more of the requirements herein prescribed.34
with the BLR, which, undisputedly, acquired jurisdiction over the case. Once
jurisdiction is acquired by the court, it remains with it until the full ART. 239. GROUNDS FOR CANCELLATION OF UNION
termination of the case.25 REGISTRATION.

Thus, jurisdiction remained with the BLR despite the BLR Director’s The following shall constitute grounds for cancellation of union registration:
inhibition. When the DOLE Secretary resolved the appeal, she merely
stepped into the shoes of the BLR Director and performed a function that the xxxx
latter could not himself perform. She did so pursuant to her power of
supervision and control over the BLR.26 (d) Failure to submit the annual financial report to the Bureau within thirty
(30) days after the closing of every fiscal year and misrepresentation, false
Expounding on the extent of the power of control, the Court, in Araneta, et entries or fraud in the preparation of the financial report itself;
al. v. Hon. M. Gatmaitan, et al.,27 pronounced that, if a certain power or
authority is vested by law upon the Department Secretary, then such power xxxx
or authority may be exercised directly by the President, who exercises
supervision and control over the departments. This principle was (i) Failure to submit list of individual members to the Bureau once a year or
incorporated in the Administrative Code of 1987, which defines "supervision whenever required by the Bureau.35
and control" as including the authority to act directly whenever a specific
function is entrusted by law or regulation to a subordinate.28 Applying the
These provisions give the Regional Director ample discretion in dealing with (d) Its list of members at least once a year or whenever required by the
a petition for cancellation of a union’s registration, particularly, determining Bureau.
whether the union still meets the requirements prescribed by law. It is
sufficient to give the Regional Director license to treat the late filing of Failure to comply with the above requirements shall not be a ground for
required documents as sufficient compliance with the requirements of the cancellation of union registration but shall subject the erring officers or
law. After all, the law requires the labor organization to submit the annual members to suspension, expulsion from membership, or any appropriate
financial report and list of members in order to verify if it is still viable and penalty.
financially sustainable as an organization so as to protect the employer and
employees from fraudulent or fly-by-night unions. With the submission of ILO Convention No. 87, which we have ratified in 1953, provides that
the required documents by respondent, the purpose of the law has been "workers’ and employers’ organizations shall not be liable to be dissolved or
achieved, though belatedly. suspended by administrative authority." The ILO has expressed the opinion
that the cancellation of union registration by the registrar of labor unions,
We cannot ascribe abuse of discretion to the Regional Director and the DOLE which in our case is the BLR, is tantamount to dissolution of the organization
Secretary in denying the petition for cancellation of respondent’s registration. by administrative authority when such measure would give rise to the loss of
The union members and, in fact, all the employees belonging to the legal personality of the union or loss of advantages necessary for it to carry
appropriate bargaining unit should not be deprived of a bargaining agent, out its activities, which is true in our jurisdiction. Although the ILO has
merely because of the negligence of the union officers who were responsible allowed such measure to be taken, provided that judicial safeguards are in
for the submission of the documents to the BLR. place, i.e., the right to appeal to a judicial body, it has nonetheless reminded
its members that dissolution of a union, and cancellation of registration for
Labor authorities should, indeed, act with circumspection in treating petitions that matter, involve serious consequences for occupational representation. It
for cancellation of union registration, lest they be accused of interfering with has, therefore, deemed it preferable if such actions were to be taken only as
union activities. In resolving the petition, consideration must be taken of the a last resort and after exhausting other possibilities with less serious effects
fundamental rights guaranteed by Article XIII, Section 3 of the Constitution, on the organization.40
i.e., the rights of all workers to self-organization, collective bargaining and
negotiations, and peaceful concerted activities. Labor authorities should bear The aforesaid amendments and the ILO’s opinion on this matter serve to
in mind that registration confers upon a union the status of legitimacy and fortify our ruling in this case. We therefore quote with approval the DOLE
the concomitant right and privileges granted by law to a legitimate labor Secretary’s rationale for denying the petition, thus:
organization, particularly the right to participate in or ask for certification
election in a bargaining unit.36 Thus, the cancellation of a certificate of It is undisputed that appellee failed to submit its annual financial reports and
registration is the equivalent of snuffing out the life of a labor organization. list of individual members in accordance with Article 239 of the Labor Code.
For without such registration, it loses - as a rule - its rights under the Labor However, the existence of this ground should not necessarily lead to the
Code.37 cancellation of union registration. Article 239 recognizes the regulatory
authority of the State to exact compliance with reporting requirements. Yet
It is worth mentioning that the Labor Code’s provisions on cancellation of there is more at stake in this case than merely monitoring union activities and
union registration and on reportorial requirements have been recently requiring periodic documentation thereof.
amended by Republic Act (R.A.) No. 9481, An Act Strengthening the
Workers’ Constitutional Right to Self-Organization, Amending for the The more substantive considerations involve the constitutionally guaranteed
Purpose Presidential Decree No. 442, As Amended, Otherwise Known as the freedom of association and right of workers to self-organization. Also
Labor Code of the Philippines, which lapsed into law on May 25, 2007 and involved is the public policy to promote free trade unionism and collective
became effective on June 14, 2007. The amendment sought to strengthen the bargaining as instruments of industrial peace and democracy.1avvphi1 An
workers’ right to self-organization and enhance the Philippines’ compliance overly stringent interpretation of the statute governing cancellation of union
with its international obligations as embodied in the International Labour registration without regard to surrounding circumstances cannot be allowed.
Organization (ILO) Convention No. 87,38 pertaining to the non-dissolution Otherwise, it would lead to an unconstitutional application of the statute and
of workers’ organizations by administrative authority.39 Thus, R.A. No. emasculation of public policy objectives. Worse, it can render nugatory the
9481 amended Article 239 to read: protection to labor and social justice clauses that pervades the Constitution
and the Labor Code.
ART. 239. Grounds for Cancellation of Union Registration.—The following
may constitute grounds for cancellation of union registration: Moreover, submission of the required documents is the duty of the officers
of the union. It would be unreasonable for this Office to order the cancellation
(a) Misrepresentation, false statement or fraud in connection with the of the union and penalize the entire union membership on the basis of the
adoption or ratification of the constitution and by-laws or amendments negligence of its officers. In National Union of Bank Employees vs. Minister
thereto, the minutes of ratification, and the list of members who took part in of Labor, L-53406, 14 December 1981, 110 SCRA 296, the Supreme Court
the ratification; ruled:

(b) Misrepresentation, false statements or fraud in connection with the As aptly ruled by respondent Bureau of Labor Relations Director Noriel:
election of officers, minutes of the election of officers, and the list of voters; "The rights of workers to self-organization finds general and specific
constitutional guarantees. x x x Such constitutional guarantees should not be
(c) Voluntary dissolution by the members. lightly taken much less nullified. A healthy respect for the freedom of
association demands that acts imputable to officers or members be not easily
R.A. No. 9481 also inserted in the Labor Code Article 242-A, which visited with capital punishments against the association itself."
provides:
At any rate, we note that on 19 May 2000, appellee had submitted its financial
ART. 242-A. Reportorial Requirements.—The following are documents statement for the years 1996-1999. With this submission, appellee has
required to be submitted to the Bureau by the legitimate labor organization substantially complied with its duty to submit its financial report for the said
concerned: period. To rule differently would be to preclude the union, after having failed
to meet its periodic obligations promptly, from taking appropriate measures
(a) Its constitution and by-laws, or amendments thereto, the minutes of to correct its omissions. For the record, we do not view with favor appellee’s
ratification, and the list of members who took part in the ratification of the late submission. Punctuality on the part of the union and its officers could
constitution and by-laws within thirty (30) days from adoption or ratification have prevented this petition.41
of the constitution and by-laws or amendments thereto;
WHEREFORE, premises considered, the Court of Appeals Decision dated
(b) Its list of officers, minutes of the election of officers, and list of voters May 30, 2005 and Resolution dated June 4, 2007 are AFFIRMED.
within thirty (30) days from election;
SO ORDERED.
(c) Its annual financial report within thirty (30) days after the close of every
fiscal year; and
TAKATA (PHILIPPINES) CORPORATION vs. BUREAU OF LABOR ground of forum shopping as respondent filed two separate appeals in two
RELATIONS separate venues; and for failing to avail of the correct remedy within the
G.R. No. 196276 June 4, 2014 period; and that the certificate of registration was tainted with fraud,
PERALTA, J.: misrepresentation and falsification.

Before us is a petition for review on certiorari filed by petitioner TAKATA In its Answer,13 respondent claimed that there was no forum shopping as
Philippines Corporation assailing the Decision1 dated December 22, 2010 BMP's Paralegal Officer was no longer authorized to file an appeal on behalf
and the Resolution2 dated March 28, 2011 of the Court of Appeals in CA- of respondent as the latter's link with BMP was already terminated and only
G.R. SP No. 112406. the Union President was authorized to file the appeal; and that it complied
with Department Order No. 40-03.
On July 7, 2009, petitioner filed with the Department of Labor and
Employment (DOLE) Regional Office a Petition3 for Cancellation of the On December 9, 2009, after considering respondent's Appeal Memorandum
Certificate of Union Registration of Respondent Samahang Lakas with Formal Entry of Appearance and petitioner's Answer, the BLR rendered
Manggagawa ng Takata (SALAMA1) on the ground that the latter is guilty its Decision14 reversing the Order of the Regional Director, the decretal
of misrepresentation, false statement and fraud with respect to the number of portion of which reads:
those who participated in the organizational meeting, the adoption and
ratification of its Constitution and By-Laws, and in the election of its officers. WHEREFORE, the appeal is hereby GRANTED. The Decision of Regional
It contended that in the May 1, 2009 organizational meeting of respondent, Director Ricardo S. Martinez, Sr., dated 27 August 2009, is hereby
only 68 attendees signed the attendance sheet, and which number comprised REVERSEDand SET ASIDE.
only 17% of the total number of the 396 regular rank- and-file employees
which respondent sought to represent, and hence, respondent failed to Accordingly, Samahang Lakas Manggagawa ng TAKATA (SALAMAT)
comply with the 20% minimum membership requirement. Petitioner insisted shall remain in the roster of labor organizations.15
that the document "Pangalan ng mga Kasapi ng Unyon" bore no signatures
of the alleged 119 union members; and that employees were not given In reversing, the BLR found that petitioner failed to prove that respondent
sufficient information on the documents they signed; that the document deliberately and maliciously misrepresented the number of rank-and-file
"Sama-Samang Pahayag ng Pagsapi" was not submitted at the time of the employees. It pointed out petitioner's basis for the alleged noncompliance
filing of respondent's application for union registration; that the 119 union with the minimum membership requirement for registration was the
members were actually only 117; and, that the total number of petitioner's attendance of 68 members to the May 1, 2009 organizational meeting
employees as of May 1, 2009 was 470, and not 396 as respondent claimed.4 supposedly comprising only 17% of the total 396 regular rank-and-file
employees. However, the BLR found that the list of employees who
Respondent denied the charge and claimed that the 119 union members were participated in the organizational meeting was a separate and distinct
more than the 20% requirement for union registration. The document "Sama- requirement from the list of the names of members comprising at least 20%
Samang Pahayag ng Pagsapi sa Unyon" which it presented in its petition for of the employees in the bargaining unit; and that there was no requirement
certification election5 supported their claim of 119 members. Respondent for signatures opposite the names of the union members; and there was no
also contended that petitioner was estopped from assailing its legal evidence showing that the employees assailed their inclusion in the list of
personality as it agreed to a certification election and actively participated in union members.
the pre-election conference of the certification election proceedings.6
Respondent argued that the union members were informed of the contents of Petitioner filed a motion for reconsideration, which was denied by the BLR
the documents they signed and that the 68 attendees to the organizational in a Resolution16 dated January 8, 2010.
meeting constituted more than 50% of the total union membership, hence, a
quo rumexisted for the conduct of the said meeting.7 Undaunted, petitioner went to the CA via a petition for certiorari under Rule
65.
On August 27, 2009, DOLE Regional Director, Atty. Ricardo S. Martinez,
Sr., issued a Decision8 granting the petition for cancellation of respondent's After the submission of the parties' respective pleadings, the case was
certificate of registration, the dispositive portion of which reads: submitted for decision.

WHEREFORE, from the foregoing considerations, the petition is hereby On December 22, 2010, the CA rendered its assailed decision which denied
GRANTED. Accordingly, the respondent Union Certificate of Registration the petition and affirmed the decision of the BLR. Petitioner's motion for
No. RO400A-2009-05-01-UR-LAG, dated May 19, 2009 is hereby reconsideration was denied in a Resolution dated March 29, 2011.
REVOCKED (sic) and /or CANCELLED pursuant to paragraph (a) & (b),
Section 3, Rule XIV of Department Order No. 40-03 and the Samahang Hence this petition for review filed by petitioner raising the following issues,
Lakas ng Manggagawa ng TAKATA (SALAMAT) is hereby delisted from to wit:
the roll of legitimate labor organization of this office.9
THE HONORABLE COURT OF APPEALS COMMITTED GRAVE AND
In revoking respondent's certificate of registration, the Regional Director SERIOUS ERROR IN AFFIRMING THE DECISION OF PUBLIC
found that the 68 employees who attended the organizational meeting was RESPONDENT BLR AND NOT FINDING ANY VIOLATION BY
obviously less than 20% of the total number of 396 regular rank-and-file SAMAHANG LAKAS MANGGAGAWA SA TAKATA (SALAMAT) OF
employees which respondent sought to represent, hence, short of the union THE RULE ON FORUM SHOPPING IN THE FILING OF TWO
registration requirement; that the attendance sheet which contained the VERIFIED APPEALS FOR AND ITS BEHALF. BOTH OF THE
signatures and names of the union members totalling to 68 contradicted the APPEALS SHOULD HAVE BEEN DISMISSED OUTRIGHT BY PUBLIC
list of names stated in the document denominated as "Pangalan ng mga RESPONDENT BLR, ON GROUND OF FORUM SHOPPING.
Kasaping Unyon." The document "Sama-Samang Pahayag ng Pagsapi" was
not attached to the application for registration as it was only submitted in the THE HONORABLE COURT OF APPEALS SERIOUSLY ERRED IN
petition for certification election filed by respondent at a later date. The FINDING THAT THE APPLICATION FOR REGISTRATION OF
Regional Director also found that the proceedings in the cancellation of SAMAHANG LAKAS MANGGAGAWA SA TAKATA (SALAMAT)
registration and certification elections are two different and entirely separate WAS COMPLIANT WITH THE LAW. CONSIDERING THE
and independent proceedings which were not dependent on each other. CIRCUMSTANCES OBTAINING IN THE REGISTRATION OF
SALAMAT, IT IS CLEAR THAT THE SAME IS TAINTED WITH
Dissatisfied, respondent, through Bukluran ng Manggagawang Pilipino FRAUD, MISREPRESENTATION AND FALSIFICATION. SALAMAT
(BMP) Paralegal Officer, Domingo P. Mole, filed a Notice and DID NOT POSSESS THE REQUIREDNUMBER OF MEMBERS AT THE
Memorandum of Appeal10 with the Bureau of Labor Relations (BLR). TIME OF FILING OF ITS APPLICATION FOR REGISTRATION,
However, on September 28,2009, respondent, through its counsels, Attys. HENCE, IT SHOULD BE HELD GUILTY OF MISREPRESENTATION,
AND FALSE STATEMENTS AND FRAUD IN CONNECTION
Napoleon C. Banzuela, Jr. and Jehn Louie W. Velandrez, filed an Appeal THEREWITH.17
Memorandum with Formal Entry of Appearance11 to the Office of the
DOLE Secretary, which the latter eventually referred to the BLR. Petitioner Anent the first issue, petitioner contends that respondent had filed two
filed an Opposition to the Appeals12 praying for their dismissal on the separate appeals with two different representations at two different venues,
in violation of the rule on multiplicity of suits and forum shopping, and legitimate labor organizations upon issuance of the certificate of registration
instead of dismissing both appeals, the appeal erroneously filed before the based on the following requirements:
Labor Secretary was the one held validly filed, entertained and even granted;
that it is not within the discretion of BLR to choose which between the two (a) Fifty pesos (₱50.00)registration fee;
appeals should be entertained, as it is the fact of the filing of the two appeals
that is being prohibited and not who among the representatives therein (b) The names of its officers, their addresses, the principal address of the
possessed the authority. labor organization, the minutes of the organizational meetings and the list of
the workers who participated in such meetings;
We are not persuaded.
(c) In case the applicant is an independent union, the names of all its members
We find no error committed by the CA in finding that respondent committed comprising at least twenty percent (20%) of all the employees in the
no forum shopping. As the CA correctly concluded, to wit: bargaining unit where it seeks to operate;

It is undisputed that BMP Paralegal Officer Domingo P. Mole was no longer (d) If the applicant union has been in existence for one or more years, copies
authorized to file an appeal on behalf of union SALAMAT and that BMP of its annual financial reports; and
was duly informed that its services was already terminated. SALAMAT even
submitted before the BLR its "Resolusyon Blg. 01-2009" terminating the (e) Four copies of the constitution and by-laws of the applicant union,
services of BMP and revoking the representation of Mr. Domingo Mole in minutes of its adoption or ratification, and the list of the members who
any of the pending cases being handled by him on behalf of the union. So, participated in it."
considering that BMP Paralegal Officer Domingo P. Mole was no longer
authorized to file an appeal when it filed the Notice and Memorandum of And after the issuance of the certificate of registration, the labor
Appeal to DOLE Regional Office No. IV-A, the same can no longer be organization's registration could be assailed directly through cancellation of
treated as an appeal filed by union SALAMAT. Hence, there is no forum registration proceedings in accordance with Articles 238 and 239 of the
shopping to speak of in this case as only the Appeal Memorandum with Labor Code. And the cancellation of union certificate of registration and the
Formal Entry of Appearance filed by Atty. Napoleon C. Banzuela, Jr. and grounds thereof are as follows:
Atty. Jehn Louie W. Velandrez is sanctioned by SALAMAT.18
ART. 238. Cancellation of Registration. - The certificate of registration of
Since Mole's appeal filed with the BLR was not specifically authorized by any legitimate labor organization, whether national or local, may be
respondent, such appeal is considered to have not been filed at all. It has been cancelled by the Bureau, after due hearing, only on the grounds specified in
held that "if a complaint is filed for and in behalf of the plaintiff who is not Article 239 hereof.
authorized to do so, the complaint is not deemed filed.
ART. 239. Grounds for Cancellation of Union Registration. - The following
An unauthorized complaint does not produce any legal effect."19 may constitute grounds for cancellation of union registration:

Respondent through its authorized representative filed its Appeal (a) Misrepresentation, false statement or fraud in connection with the
Memorandum with Formal Entry of Appearance before the Labor Secretary, adoption or ratification of the constitution and by-laws or amendments
and not with the BLR. As the appeal emanated from the petition for thereto, the minutes of ratification, and the list of members who took part in
cancellation of certificate of registration filed with the Regional Office, the the ratification;
decision canceling the registration is appealable to the BLR, and not with the
Labor Secretary. However, since the Labor Secretary motu propio referred (b) Misrepresentation, false statements or fraud in connection with the
the appeal with the BLR, the latter can now act on it. Considering that Mole's election of officers, minutes of the election of officers, and the list of voters;
appeal with the BLR was not deemed filed, respondent’s appeal, through
Banzuela and Associates, which the Labor Secretary referred to the BLR was (c) Voluntary dissolution by the members.
the only existing appeal with the BLR for resolution. There is, therefore, no
merit to petitioner's claim that BLR chose the appeal of Banzuela and Petitioner's charge that respondent committed misrepresentation and fraud in
Associates over Mole's appeal. securing its certificate of registration is a serious charge and must be carefully
evaluated. Allegations thereof should be compounded with supporting
The case of Abbott Laboratories Philippines, Inc. v. Abbott Laboratories circumstances and evidence.21 We find no evidence on record to support
Employees Union20 cited by petitioner is not at all applicable in this case as petitioner's accusation.
the issue therein is the authority of the Labor Secretary to review the decision
of the Bureau of Labor Relations rendered in the exercise of its appellate Petitioner's allegation of misrepresentation and fraud is based on its claim
jurisdiction over decision of the Regional Director in cases involving that during the organizational meeting on May 1, 2009, only 68 employees
cancellations of certificate of registration of labor unions. We found no grave attended, while respondent claimed that it has 119 members as shown in the
abuse of discretion committed by the Secretary of Labor in not acting on document denominated as "Pangalan ng mga Kasapi ng Unyon;" hence,
therein petitioner's appeal. The decision of the Bureau of Labor Relations on respondent misrepresented on the 20% requirement of the law as to its
cases brought before it on appeal from the Regional Director are final and membership.
executory. Hence, the remedy of the aggrieved party is to seasonably avail
of the special civil action of certiorari under Rule 65 and the Rules of Court. We do not agree.
In this case, after the Labor Secretary motu propio referred respondent's
appeal filed with it to the BLR which rendered its decision reversing the It does not appear in Article 234 (b) of the Labor Code that the attendees in
Regional Director, petitioner went directly to the CA via a petition for the organizational meeting must comprise 20% of the employees in the
certiorari under Rule 65. bargaining unit. In fact, even the Implementing Rules and Regulations of the
Labor Code does not so provide. It is only under Article 234 (c) that requires
As to the second issue, petitioner seeks the cancellation of respondent's the names of all its members comprising at least twenty percent (20%) of all
registration on grounds offraud and misrepresentation bearing on the the employees in the bargaining unit where it seeks to operate. Clearly, the
minimum requirement of the law as to its membership, considering the big 20% minimum requirement pertains to the employees’ membership in the
disparity in numbers, between the organizational meeting and the list of union and not to the list of workers who participated in the organizational
members, and so misleading the BLR that it obtained the minimum required meeting. Indeed, Article 234 (b) and (c) provide for separate requirements,
number of employees for purposes of organization and registration. which must be submitted for the union's registration, and which respondent
did submit. Here, the total number of employees in the bargaining unit was
We find no merit in the arguments. 396, and 20% of which was about 79. Respondent submitted a document
entitled "Pangalan ng Mga Kasapi ng Unyon" showing the names of 119
Art. 234 of the Labor Code provides: employees as union members, thus respondent sufficiently complied even
beyond the 20% minimum membership requirement. Respondent also
ART. 234. Requirements of Registration. - A federation, national union or submitted the attendance sheet of the organizational meeting which
industry or trade union center or an independent union shall acquire legal contained the names and signatures of the 68 union members who attended
personality and shall be entitled to the rights and privileges granted by law to the meeting. Considering that there are 119 union members which are more
than 20% of all the employees of the bargaining unit, and since the law does
not provide for the required number of members to attend the organizational
meeting, the 68 attendees which comprised at least the majority of the 119
union members would already constitute a quorum for the meeting to proceed
and to validly ratify the Constitution and By-laws of the union. There is,
therefore, no basis for petitioner to contend that grounds exist for the
cancellation of respondent's union registration. For fraud and
misrepresentation to be grounds for cancellation of union registration under
Article 239 of the Labor Code, the nature of the fraud and misrepresentation
must be grave and compelling enough to vitiate the consent of a majority of
union members.22

Petitioner's claim that the alleged union members signed documents without
adequate information is not persuasive. The one who alleges a fact has the
burden of proving it and a mere allegation is not evidence.23 In fact, we note
that not one of those listed in the document denominated as "Pangalan ng
Mga Kasaping Unyon" had come forward to deny their membership with
respondent. Notably, it had not been rebutted that the same union members
had signed the document entitled "Sama-Samang Pahayag ng Pagsapi," thus,
strengtheningtheir desire to be members of the respondent union.

Petitioner claims that in the list of members, there was an employee whose
name appeared twice and another employee who was merely a project
employee. Such could not be considered a misrepresentation in the absence
of showing that respondent deliberately did so for the purpose of increasing
their union membership. In fact, even if those two names were not included
in the list of union members, there would still be 117 members which was
still more than 20% of the 396 rank-and-file employees.

As to petitioner's argument that the total number of its employees as of May


1, 2009 was 470, and not396 as respondent claimed, still the 117 union
members comprised more than the 20% membership requirement for
respondent's registration.

In Mariwasa Siam Ceramics v. Secretary of the Department of Labor and


Employment,24 we said:

For the purpose of de-certifying a union such as respondent, it must be shown


that there was misrepresentation, false statement or fraud in connection with
the adoption or ratification of the constitution and by-laws or amendments
thereto, the minutes of ratification; or, in connection with the election of
officers, the minutes of the election of officers, the list of voters, or failure to
submit these documents together with the list of the newly elected-appointed
officers and their postal addresses to the BLR.

The bare fact that two signatures appeared twice on the list of those who
participated in the organizational meeting would not, to our mind, provide a
valid reason to cancel respondent’s certificate of registration. The
cancellation of a union’s registration doubtless has an impairing dimension
on the right of labor to self-organization. For fraud and misrepresentation to
be grounds for cancellation of union registration under the Labor Code, the
nature of the fraud and misrepresentation must be grave and compelling
enough to vitiate the consent of a majority of union members.1âwphi1

In this case, we agree with the BLR and the CA that respondent could not
have possibly committed misrepresentation, fraud, or false statements. The
alleged failure of respondent to indicate with mathematical precision the total
number of employees in the bargaining unit is of no moment, especially as it
was able to comply with the 20% minimum membership requirement. Even
if the total number of rank-and-file employees of petitioner is 528, while
respondent declared that it should only be 455, it still cannot be denied that
the latter would have more than complied with the registration
requirement.25

WHEREFORE, premises considered, the petition for review is DENIED.


The Decision dated December 22, 2010 and the Resolution dated March 28,
2011 of the Court of Appeals, in CA-G.R. SP No. 112406, are AFFIRMED.

SO ORDERED.
DE OCAMPO MEMORIAL SCHOOLS, INC. vs. BIGKIS inappropriate bargaining unit would not necessarily result in the cancellation
MANGGAGAWA SA DE OCAMPO MEMORIAL SCHOOL, INC. of union registration, and the inclusion of a disqualified employee in a union
G.R. No. 192648 March 15, 2017 is not a ground for cancellation.22 Even if BMDOMSI shared the same set
JARDELEZA, J.: of officers and members of BMDOMMC, the latter had already been delisted
on March 3, 2004 and there is no prohibition against organizing another
This is a Petition for Review on Certiorari1 assailing the Court of Appeals union.23
(CA) Decision2 dated July 15, 2009 and the Resolution3 dated June 21, 2010
(assailed Decision). The assailed Decision affirmed the Decision4 dated De Ocampo filed a Petition for Certiorari24 with the CA seeking to annul
December 29, 2004 of the Bureau of Labor Relations (BLR), Department of and set aside the BLR Decision as well as the Resolution25 dated January
Labor and Employment (DOLE) in Case No. BLR-A-C-75-8- 24-04, In Re: 24, 2005 denying its motion for reconsideration.
Petition for Cancellation of Union Registration of Bigkis Manggagawa sa De
Ocampo Memorial School, Inc., - Lakas Union Registration Number (NCR- The CA affirmed the Decision of the BLR. It ruled that there was no
12-CC-002-2003). misrepresentation, false statement or fraud in the application for registration.

I The record shows that, as BMDOMSI had indicated, the bargaining unit as
described is composed of rank-and-file employees with occupational
De Ocampo Memorial Schools, Inc. (De Ocampo) is a domestic corporation classifications under technical and faculty.26 The CA found that there could
duly-organized and existing under the laws of the Philippines. It has two main be no misrepresentation as the members appearing in the minutes of the
divisions, namely: De Ocampo Memorial Medical Center (DOMMC), its general membership meeting, and the list of members who attended the
hospital entity, and the De Ocampo Memorial Colleges (DOMC), its school meeting and ratified the union constitution and by-laws, are in truth
entity.5 employees of the school, though some service the hospital.27 The CA also
ruled that, other than De Ocampo's bare allegations, there was no proof of
On September 26, 2003, Union Registration No. NCR-UR-9-3858- 2002 was intent to defraud or mislead on the part of BMDOMSI. Hence, the charge of
issued in favor of Bigkis Manggagawa sa De Ocampo Memorial Medical fraud, false statement or misrepresentation cannot be sustained.28
Center - LAKAS (BMDOMMC).6
However, the CA observed that the members of the union, who are from
Later, on December 5, 2003, Bigkis Manggagawa sa De Ocampo Memorial academic, non-academic, and general services, do not perform work of the
School, Inc. (BMDOMSI) was issued a Union Registration/Certificate of same nature, receive the same wages and compensation, nor share a common
Creation of Local Chapter No. NCR-l 2-CC-002- 2003 and declared a stake in concerted activities.29 While these factors dictate the separation of
legitimate labor organization.7 the categories of employees for purposes of collective bargaining,30 the CA
reasoned that such lack of mutuality and commonality of interest of the union
On March 4, 2004, De Ocampo filed a Petition for Cancellation of Ce1iificate members is not among the grounds for cancellation of union registration
of Registration8 with the Department of Labor and Employment - National under Article 239 of the Labor Code.31
Capital Region (DOLE-NCR). It sought to cancel the Certificate of
Registration of BMDOMSI on the following grounds: 1) misrepresentation, De Ocampo filed a motion for reconsideration which was denied in the
false statement and fraud in connection with its creation and registration as a assailed Resolution dated June 21, 2010. Hence, this petition.
labor union as it shared the same set of officers and members with
BMDOMMC; 2) mixed membership of rank-and-file and De Ocampo maintains that BMDOMSI committed misrepresentation and
managerial/supervisory employees; and 3) inappropriate bargaining unit.9 fraud in connection with its application, creation and registration. It
intentionally suppressed the fact that at the time of its application, there was
On April 13, 2004, De Ocampo filed a Supplemental Petition,10 informing another union known as BMDOMMC, with whom they shared the same set
the DOLE-NCR of the cancellation of the Certificate of Registration of of officers and members.32 It was also made to appear that BMDOMMC is
BMDOMMC in Case No. NCR-OD-0307-009-LRD. It attached a copy of a labor union representing a separate bargaining unit whose personality,
the Decision11 of the DOLE-NCR dated March 3, 2004, which cancelled and affairs and composition are unknown to BMDOMSI.33 Lastly, BMDOMSI
struck off Union Registration No. NCR-UR-9-3858- 2002 from the registry suppressed the fact that its members have no mutuality or commonality of
of legitimate labor organizations for being an inappropriate bargaining interest as they belong to different work classifications, nature and
unit.12 designations.34

On May 18, 2004, BMDOMSI filed its Comment-Opposition to Petition for II


Cancellation of Certificate of Registration and Supplemental Petition,13
denying De Ocampo's allegations and claiming that the latter only wants to We deny the petition.
impede the formation of the union.
Article 247, previously Aiiicle 239 of the Labor Code35 provides:
In a Decision14 dated July 26, 2004, Acting Regional Director Ciriaco A.
Lagunzad III of the DOLE-NCR ruled that BMDOMSI committed Art. 247. Grounds for Cancellation of Union Registration. - The following
misrepresentation by making it appear that the bargaining unit is composed may constitute grounds for cancellation of union registration:
of faculty and technical employees. In fact, all the union officers and most of
the members are from the General Services Division.15 Furthermore, the (a) Misrepresentation, false statement or fraud in connection with the
members of the union do not share commonality of interest, as it is composed adoption or ratification of the constitution and by-laws or amendments
of academic and non-academic personnel.16 The nature of work of the thereto, the minutes of ratification, and the list of members who took part in
employees of the General Services Division, while falling within the the ratification;
category of non-academic personnel, differs from that of the other
nonacademic employees composed of clerks, messengers, etc., since they (b) Misrepresentation, false statements or fraud in connection with the
also serve the hospital component of De Ocampo.17 election of officers, minutes of the election of officers, and the list of voters;

BMDOMSI then filed an appeal to the BLR alleging that the union members (c) Voluntary dissolution by the members.
are all employees of De Ocampo and that the bargaining unit it sees to
represent is appropriate.18 For fraud and misrepresentation to constitute grounds for cancellation of
union registration under the Labor Code, the nature of the fraud and
In a Decision19 dated December 29, 2004, the BLR reversed the Regional misrepresentation must be grave and compelling enough to vitiate the
Director's finding of misrepresentation, false statement or fraud in consent of a majority of union members.36
BMDOMSI's application for registration. According to the BLR, De Ocampo
failed to adduce proof to support its allegation of mixed membership within De Ocampo insists that "by conveniently disregarding" BMDOMMC's
respondent union.20 Further, and contrary to De Ocampo's claim, records existence during the filing of its application, despite having the same set of
show that BMDOMSI stated in its application that its members are composed officers and members,37 BMDOMSI "had misrepresented facts, made false
of rank-and-file employees falling under either faculty or technical statements and committed fraud in its application for union registration for
occupational classifications.21 The BLR also held that the existence of an
alleging facts therein which they [know] or ought to have known to be SO ORDERED.
false."38

We agree with the BLR and the CA that BMDOMSI did not commit fraud
or misrepresentation in its application for registration.1âwphi1 In the form
"Report of Creation of Local Chapter"39 filed by BMDOMSI, the applicant
indicated in the portion "Description of the Bargaining Unit" that it is
composed of "Rank and File" and under the "Occupational Classification," it
marked "Technical" and "Faculty."

Further, the members appearing in the Minutes of the General Membership


and the List of Workers or Members who attended the organizational meeting
and adopted/ratified the Constitution and By-Laws are, as represented,
employees of the school and the General Services Division, though some of
the latter employees service the hospital.40

Moreover, there is nothing in the form "Report of Creation of Local Chapter"


that requires the applicant to disclose the existence of another union, much
less the names of the officers of such other union. Thus, we cannot see how
BMDOMSI made the alleged misrepresentation or false statements in its
application.

De Ocampo likewise claims that BMDOMSI committed fraud and


misrepresentation when it suppressed the fact that there exists "no mutuality
and/or communality of interest"41 of its members. This, De Ocampo asserts,
is a ground for the cancellation of its registration.

We disagree.

While the CA may have ruled that there is no mutuality or commonality of


interests among the members of BMDOMSI, this is not enough reason to
cancel its registration. The only grounds on which the cancellation of a
union's registration may be sought are those found in Article 247 of the Labor
Code. In Tagaytay Highlands International Golf Club Incorporated v.
Tagaytay Highlands Employees Union-PTGW0,42 we ruled that "[t]he
inclusion in a union of disqualified employees is not among the grounds for
cancellation, unless such inclusion is due to misrepresentation, false
statement or fraud under the circumstances enumerated in Sections (a) and
(c) of Article [247] x x x of the Labor Code."43 Thus, for purposes of de-
certifying a union, it is not enough to establish that the rank-and-file union
includes ineligible employees in its membership. Pursuant to paragraphs (a)
and (b) of Article 247 of the Labor Code, it must be shown that there was
misrepresentation, false statement or fraud in connection with: (1) the
adoption or ratification of the constitution and by-laws or amendments
thereto; (2) the minutes of ratification; (3) the election of officers; (4) the
minutes of the election of officers; and (5) the list of voters.44 Failure to
submit these documents together with the list of the newly elected-appointed
officers and their postal addresses to the BLR may also constitute grounds
for cancellation, lack of mutuality of interests, however, is not among said
grounds.45

The BLR and the CA's finding that the members of BMDOMSI are rank-
and-file employees is supported by substantial evidence and is binding on
this Court.46 On the other hand, other than the allegation that BMDOMSI
has the same set of officers with BMDOMMC and the allegation of mixed
membership of rank-and-file and managerial or supervisory employees, De
Ocampo has cited no other evidence of the alleged fraud and
misrepresentation.

A final word. A party seeking the cancellation of a union's certificate of


registration must bear in mind that:

x x x [A] direct challenge to the legitimacy of a labor organization based on


fraud and misrepresentation in securing its certificate of registration is a
serious allegation which deserves careful scrutiny. Allegations thereof
should be compounded with supporting circumstances and evidence. The
records of the case are devoid of such evidence. Furthermore, this Court is
not a trier of facts, and this doctrine applies with greater force in labor cases.
Findings of fact of administrative agencies and quasi-judicial bodies, such as
the BLR, which have acquired expertise because their jurisdiction is confined
to specific matters, are generally accorded not only great respect but even
finality.47

WHEREFORE, the petition is hereby DENIED for lack of merit. The


Decision of the Court of Appeals in CA-G.R. SP No. 89162 dated July 15,
2009 is AFFIRMED.
ASIAN INSTITUTE OF MANAGEMENT vs. ASIAN INSTITUTE OF On October 22, 2010, the CA rendered its Decision19 containing the
MANAGEMENT FACULTY ASSOCIATION following pronouncement:
G.R. No. 207971 January 23, 2017
DEL CASTILLO, J.: AIM insists that the members of its tenure-track faculty are managerial
employees, and therefore, ineligible to join, assist or form a labor
This Petition for Review on Certiorari1assails the January 8, 2013 Decision2 organization. It ascribes grave abuse of discretion on SOLE20 for its rash
of the Court of Appeals (CA) which dismissed the Petition for Certiorari3in conclusion that the members of said tenure-track faculty are not managerial
CA-G.R. SP No. 114122, and its subsequent June 27, 2013 Resolution 4 employees solely because the faculty's actions are still subject to evaluation,
denying herein petitioner's Motion for Reconsideration. 5 review or final approval by the board of trustees ("BOT'). AIM argues that
the BOT does not manage the day-to-day affairs, nor the making and
Factual Antecedents implementing of policies of the Institute, as such functions are vested with
the tenure-track faculty.
Petitioner Asian Institute of Management (AIM) is a duly registered non-
stock, non-profit educational institution. Respondent Asian Institute of We agree.
Management Faculty Association (AFA) is a labor organization composed
of members of the AIM faculty, duly registered under Certificate of Article 212(m) of the Labor Code defines managerial employees as:
Registration No. NCR-UR-12-4076-2004.
'ART. 212. Definitions. – x x x
On May 16, 2007, respondent filed a petition for certification election6
seeking to represent a bargaining unit in AIM consisting of forty (40) faculty (m) 'Managerial employee' is one who is vested with powers or prerogatives
members. The case was docketed as DOLE Case No. NCR-OD-M-0705-007. to lay down and execute management policies and/or to hire, transfer,
Petitioner opposed the petition, claiming that respondent's members are suspend, lay-off, recall, discharge, assign or discipline employees.
neither rank-and-file nor supervisory, but rather, managerial employees.7 Supervisory employees are those who, in the interest of the employer,
effectively recommend such managerial actions if the exercise of such
On July 11, 2007, petitioner filed a petition for cancellation of respondent's authority is not merely routinary or clerical in nature but requires the use of
certificate of registration8 - docketed as DOLE Case No. NCROD-0707-001- independent judgment. All employees not falling within any of the above
LRD - on the grounds of misrepresentation in registration and that respondent definitions are considered rank-and-file employees for purposes of this
is composed of managerial employees who are prohibited from organizing as Book.'
a union.
There are, therefore, two (2) kinds of managerial employees under Art.
On August 30, 2007, the Med-Arbiter in DOLE Case No. NCR-OD-M-0705- 212(m) of the Labor Code. Those who 'lay down x x x management policies',
007 issued an Order9 denying the petition for certification election on the such as the Board of Trustees, and those who 'execute management policies
ground that AIM' s faculty members are managerial employees. This Order and/or hire, transfer, suspend, lay-off, recall, discharge, assign or discipline
was appealed by respondent before the Secretary of the Department of Labor employees'.
and Employment (DOLE), 10 who reversed foe same via a February 20, 2009
Decision 11 and May 4, 2009 Resolution, 12 decreeing thus: xxxx

WHEREFORE, the appeal filed by the Asian Institute of Management On its face, the SOLE's opinion is already erroneous because in claiming that
Faculty Association (AIMFA) is GRANTED. The Order dated 30 August the 'test of 'supervisory' or 'managerial status' depends on whether a person
2007 of DOLE-NCR Mediator-Arbiter Michael T. Parado is hereby possesses authority to act in the interest of his employer in the matter
REVERSED and SET ASIDE. specified in Article 212(m) of the Labor Code and Section l(m) of its
Implementing Rules', he obviously was referring to the old definition of a
Accordingly, let the entire records of the case be remanded to DOLEN CR managerial employee. Such is evident in his use of 'supervisory or managerial
for the conduct of a certification election among the faculty members of the status', and reference to 'Section l(m) of its Implementing Rules'. For
Asian Institute of Management (AIM), with the following choices: presently, as aforequoted in Article 212(m) of the Labor Code and as
amended by Republic Act 6715 which took effect on March 21, 1989, a
1. ASIAN INSTITUTE OF MANAGEMENT FACULTY ASSOCIATION managerial employee is already different from a supervisory employee. x x
(AIMFA); and x

2. No Union. xxxx

SO ORDERED.13 In further opining that a managerial employee is one whose 'authority is not
merely routinary or clerical in nature but requires the use of independent
Meanwhile, in DOLE Case No. NCR-OD-0707-001-LRD, an Order14 dated judgment', a description which fits now a supervisory employee under
February 16, 2009 was issued by DOLE-NCR Regional Director Raymundo Section l(t), Rule I, Book V of the Omnibus Rules Implementing the Labor
G. Agravante granting AIM's petition for cancellation of respondent's Code, it then follows that the SOLE was not aware of the change in the law
certificate of registration and ordering its delisting from the roster of and thus gravely abused its discretion amounting to lack of jurisdiction in
legitimate labor organizations. 1bis Order was appealed by respondent before concluding that AIM's 'tenure-track' faculty are not managerial employees.
the Bureau labor Relations15 (BLR), which, in a December 29, 2009
Decision,16 reversed the same and ordered respondent's retention in the SOLE further committed grave abuse of discretion when it concluded that
roster of legitimate labor organizations. The BLR held that the grounds relied said tenure-track faculty members are not managerial employees on the basis
upon in the petition for cancellation are not among the grounds authorized of a 'footnote' in AIM's Policy Manual, which provides that 'the policy[-]
under Article 239 of the Labor Code, 17 and that respondent's members are making authority of the faculty members is merely recommendatory in nature
not managerial employees. Petitioner moved to reconsider, but was rebuffed considering that the faculty standards they formulate are still subject to
in a March 18, 2010 Resolution. 18 evaluation, review or final approval by the [AIM]'s Board of Trustees'. x x x

CA-G.R. SP No.109487 and G.R. No.197089 xxxx

Petitioner filed a Petition for Certiorari before the CA, questioning the DOLE Clearly, AIM's tenure-track faculty do not merely recommend faculty
Secretary's February 20, 2009 Decision and May 4, 2009 Resolution relative standards.1âwphi1 They 'determine all faculty standards', and are thus
to DOLE Case No. NCR-OD-M-0705-007, or respondent's petition for managerial employees. The standards' being subjected to the approval of the
certification election. Docketed as CA-G.R. SP No. 109487, the petition is Board of Trustees would not make AIM's tenure-track faculty non-
based on the arguments that 1) the bargaining unit within AIM sought to be managerial because as earlier mentioned, managerial employees are now of
represented is composed of managerial employees who are not eligible to two categories: (1) those who 'lay down policies', such as the members of the
join, assist, or form any labor organization, and 2) respondent is not a Board of Trustees, and (2) those who 'execute management policies (etc.)’,
legitimate labor organization that may conduct a certification election. such as AIM's tenure-track faculty.
xxxx For the purpose of de-certifying a union such as respondent, it must be shown
that there was misrepresentation, false statement or fraud in connection with
It was also grave abuse of discretion on the part of the SOLE when he opined the adoption or ratification of the constitution and by-laws or amendments
that AIM' s tenure-track faculty members are not managerial employees, thereto; the minutes of ratification; or, in connection with the election of
relying on an impression that they were subjected to rigid observance of officers, the minutes of the election of officers, the list of voters, or failure to
regular hours of work as professors. x x x submit these documents together with the list of the newly elected-appointed
officers and their postal addresses to the BLR.
xxxx
The bare fact that two signatures appeared twice on the list of those who
More importantly, it behooves the SOLE to deny AFA's appeal in light of the participated in the organizational meeting would not, to our mind, provide a
February 16, 2009 Order of Regional Director Agravante delisting AFA from valid reason to cancel respondent's certificate of registration. The
the roster of legitimate labor organizations. For, only legitimate labor cancellation of a union's registration doubtless has an impairing dimension
organizations are given the right to be certified as sole and exclusive on the right of labor to self-organization. For fraud and misrepresentation to
bargaining agent in an establishment. be grounds for cancellation of union registration under the Labor Code, the
nature of the fraud andmisrepresentation must be grave and compelling
xxxx enough to

Here, the SOLE committed grave abuse of discretion by giving due course to vitiate the consent of a majority of union members.23
AFA's petition for certification election, despite the fact that: (1) AFA's
members are managerial employees; and (2) AFA is not a legitimate labor In this regard, it has also been held that:
organization. 'These facts rendered AFA ineligible, and without any right to
file a petition for certification election, the object of which is to determine Another factor which militates against the veracity of the allegations in the
the sole and exclusive bargaining representative of qualified AIM employees. Sinumpaang Petisyon is the lack of particularities on how, when and where
respondent union perpetrated the alleged fraud on each member. Such details
WHEREFORE, the instant petition is GRANTED. The assailed Decision are crucial for, in the proceedings for cancellation of union registration on
dated February 20, 2009 and Resolution dated May 4, 2009 are hereby the ground of fraud or misrepresentation, what needs to be established is that
REVERSED and SET ASIDE. The Order dated August 30, 2007 of the specific act or omission of the union deprived the complaining
Mediator-Arbiter Parado is hereby REINSTATED. employees-members of their right to choose.24

SO ORDERED.21 (Emphasis in the original) A cursory reading of the Petition shows that AIM did NOT allege any
specific act of fraud or misrepresentation committed by AFA. What is clear
Respondent sought reconsideration, but was denied. It thus instituted a is that the Institute seeks the cancellation of the registration of AFA based on
Petition for Review on Certiorari before this Court on July 4, 2011. The Article 245 of the Labor Code on the ineligibility of managerial employees
Petition, docketed as G.R. No. 197089, remains pending to date. to form or join labor unions. Unfortunately for the petitioner, even assuming
that there is a violation of Article 245, such violation will not result in the
The Assailed Ruling of the Court of Appeals cancellation of the certificate of registration of a labor organization.

Meanwhile, relative to DOLE Case No. NCR-OD-0707-001-LRD or It should be stressed that a Decision had already been issued by the DOLE in
petitioner AIM's petition for cancellation of respondent's certificate of the Certification Election case; and the Decision ordered the conduct of a
registration, petitioner filed on May 24, 20 l 0 a Petition for certification election among the faculty members of the Institute, basing its
Certiorari22before the CA, questioning the BLR's December 29, 2009 directive on the finding that the members of AFA were not managerial
decision and March 18, 2010 resolution. The petition, docketed as CA-G.R. employees and are therefore eligible to form, assist and join a labor union.
SP No. 114122, alleged that the BLR committed grave abuse of discretion in As a matter of fact, the certification election had already been held on
granting respondent's appeal and affirming its certificate of registration October 16, 2009, albeit the results have not yet been resolved as
notwithstanding that its members are managerial employees who may not inclusion/exclusion proceedings are still pending before the DOLE. The
join, assist, or form a labor union or organization. remedy available to the Institute is not the instant Petition, but to question the
status of the individual union members of the AFA in the inclusion/exclusion
On January 8, 2013, the CA rendered the assailed Decision, stating as proceedings pursuant to Article 245-A of the Labor Code, which reads:
follows:
Article 245-A. Effect of inclusion as members of employees outside the
The petition lacks merit bargaining unit. - The inclusion as union members of employees outside the
bargaining unit shall not be a ground for the cancellation of the registration
xxxx of the union. Said employees are automatically deemed removed from the
list of membership of said union.
It is therefore incumbent upon the Institute to prove that the BLR committed
grave abuse of discretion in issuing the questioned Decision.1âwphi1 Petitioner insists that Article 245-A is not applicable to this case as all AF A
Towards this end, AIM must lay the basis by showing that any of the grounds members are managerial employees. We are not persuaded.
provided under Article 239 of the Labor Code, exists, to wit:
The determination of whether any or all of the members of AFA should be
Article 239. Grounds for cancellation of union registration. - The following considered as managerial employees is better left to the DOLE because,
may constitute grounds for cancellation of union registration:
It has also been established that in the determination of whether or not certain
(a) Misrepresentation, false statement or fraud in connection with the employees are managerial employees, this Court accords due respect and
adoption or ratification of the constitution and by-laws or amendments therefore sustains the findings of fact made by quasi-judicial agencies which
thereto, the minutes of ratification, and the list of members who took part in are supported by substantial evidence considering their expertise in their
the ratification; respective fields.25

(b) Misrepresentation, false statements or fraud in connection with the From the discussion, it is manifestly clear that the petitioner foiled to prove
election of officers, minutes of the election of officers, and the list of voters; that the BLR committed grave abuse of discretion; consequently, the Petition
must fail.
(c) Voluntary dissolution by the members.
WHEREFORE, the Petition is hereby DENIED. The Decision and
Article 238 of the Labor Code provides that the enumeration of the grounds Resolution of public respondent Bureau of Labor Relations in BLR-A-C-19-
for cancellation of union registration, is exclusive; in other words, no other 3-6-09 (NCR-OD-0707-001) are hereby AFFIRMED.
grow1ds for cancellation is acceptable, except for the three (3) grounds stated
in Article 239. The scope of the grounds for cancellation has been explained- SO ORDERED.26 (Emphasis in the original)
Petitioner filed its Motion for Reconsideration, which was denied by the CA final and conclusive in the second if that same point or question was in issue
via its June 27, 2013 Resolution. Hence, the instant Petition. and adjudicated in the first suit. x x x Identity of cause of action is not
required, but merely identity of issues.31 (Citation omitted)
In a November 10, 2014 Resolution,27 the Court resolved to give due course
to the Petition. WHEREFORE, considering that the outcome of this case depends on the
resolution of the issue relative to the nature of respondent's membership
Issue pending in G.R. No. 197089, this case is ordered CONSOLIDATED with
G.R. No. 197089.
Petitioner claims that the CA seriously erred in affirming the dispositions of
the BLR and thus validating the respondent's certificate of registration SO ORDERED.
notwithstanding the fact that its members are all managerial employees who
are disqualified from joining, assisting, or forming a labor organization.

Petitioner's Arguments

Praying that the assailed CA dispositions be set aside and that the DOLE-
NCR Regional Director's February 16, 2009 Order granting AIM's petition
for cancellation of respondent's certificate of registration and ordering its
delisting from the roster of legitimate labor organizations be reinstated
instead, petitioner maintains in its Petition and Reply28 that respondent's
members are all managerial employees; that the CA erred in declaring that
even if respondent's members are all managerial employees, this alone is not
a ground for cancellation of its certificate of registration; that precisely, the
finding in DOLE Case No. NCR-ODM- 0705-007, which the CA affirmed
in CA-G.R. SP No. 109487, is that respondent's members are managerial
employees; that respondent's declaration that its members are eligible to join,
assist, or form a labor organization is an act of misrepresentation, given the
finding in CA-G.R. SP No. 109487 that they are managerial employees; and
that the grounds for cancellation of union registration enumerated in Article
239 of the Labor Code are not exclusive.

Respondent's Arguments

In its Comment,29 respondent maintains that the CA was right to treat


petitioner’s case for cancellation of its union registration with
circumspection; that petitioner's ground for filing the petition for cancellation
is not recognized under Article 239; that petitioner's accusation of
misrepresentation is unsubstantiated, and is being raised for the first time at
this stage; that its members are not managerial employees; and that
petitioner's opposition to respondent's attempts at self-organization
constitutes harassment, oppression, and violates the latter's rights under the
Labor Code and the Constitution.

Our Ruling

In Holy Child Catholic School v. Hon. Sto. Tomas,30this Court declared that
"[i]n case of alleged inclusion of disqualified employees in a union, the
proper procedure for an employer like petitioner is to directly file a petition
for cancellation of the union's certificate of registration due to
misrepresentation, false statement or fraud under the circumstances
enumerated in Article 239 of the Labor Code, as amended."

On the basis of the ruling in the above-cited case, it can be said that petitioner
was correct in filing a petition for cancellation of respondent's certificate of
registration. Petitioner's sole ground for seeking cancellation of respondent's
certificate of registration - that its members are managerial employees and
for this reason, its registration is thus a patent nullity for being an absolute
violation of Article 245 of the Labor Code which declares that managerial
employees are ineligible to join any labor organization --- is, in a sense, an
accusation that respondent is guilty of misrepresentation for registering under
the claim that its members are not managerial employees.

However, the issue of whether respondent's members are managerial


employees is still pending resolution by way of petition for review on
certiorari in G.R. No. 197089, which is the culmination of all proceedings in
DOLE Case No. NCR-OD-M-0705-007 -- where the issue relative to the
nature of respondent's membership was first raised by petitioner itself and is
there fiercely contested. The resolution of this issue cannot be pre-empted;
until it is determined with finality in G.R. No. l 97089, the petition for
cancellation of respondent's certificate of registration on the grounds alleged
by petitioner cannot be resolved. As a matter of courtesy and in order to avoid
conflicting decisions, We must await the resolution of the petition in G.R.
No. 197089.

x x x If a particular point or question is in issue in the second action, and the


judgment will depend on the determination of that particular point or
question, a former judgment between the same parties or their privies will be
UST FACULTY UNION (USTFU) vs. BITONIO JR.
G.R. No. 131235 November 16, 1999 On 04 October 1996, the med-arbiter in Case No. NCR-OD-M-9610-001
PANGANIBAN, J.: issued a temporary restraining order against herein appellees enjoining them
from conducting the election scheduled on 05 October 1996.
There is a right way to do the right thing at the right time for the right reasons,
1 and in the present case, in the right forum by the right parties. While Also on 04 October 1996, and as earlier announced by the UST secretary
grievances against union leaders constitute legitimate complaints deserving general, the general faculty assembly was held as scheduled. The general
appropriate redress, action thereon should be made in the proper forum at the assembly was attended by members of the USTFU and, as admitted by the
proper time and after observance of proper procedures. Similarly, the election appellants, also by "non-USTFU members [who] are members in good
of union officers should be conducted in accordance with the provisions of standing of the UST Academic Community Collective Bargaining Unit" (See
the union's constitution and bylaws, as well as the Philippine Constitution paragraph XI, Respondents' Comment and Motion to Dismiss). On this
and the Labor Code. Specifically, while all legitimate faculty members of the occasion, appellants were elected as USTFU's new set of officers by
University of Santo Tomas (UST) belonging to a collective bargaining unit acclamation and clapping of hands (See paragraphs 40 to 50, Annex "12",
may take part in a duly convened certification election, only bona fide Appeal).
members of the UST Faculty Union (USTFU) may participate and vote in a
legally called election for union officers. Mob hysteria, however well- The election of the appellants came about upon a motion of one Atty. Lopez,
intentioned, is not a substitute for the rule of law. admittedly not a member of USTFU, that the USTFU CBL and "the rules of
the election be suspended and that the election be held [on] that day" (See —
The Case paragraph 39, Idem.)

The Petition for Certiorari before us assails the August 15, 1997 Resolution On 11 October 1996, appellees filed the instant petition seeking injunctive
2 of Director Benedicto Ernesto R. Bitonio Jr. of the Bureau of Labor reliefs and the nullification of the results of the 04 October 1996 election.
Relations (BLR) in BLR Case No. A-8-49-97, which affirmed the February Appellees alleged that the holding of the same violated the temporary
11, 1997 Decision of Med-Arbiter Tomas F. Falconitin. The med-arbiters restraining order issued in Case No. NCR-OD-M-9610-001. Accusing
Decision disposed as follows: appellants of usurpation, appellees characterized the election as spurious for
being violative of USTFU's CBL, specifically because the general assembly
WHEREFORE, premises considered, judgment is hereby rendered declaring resulting in the election of appellants was not called by the Board of Officers
the election of USTFU officers conducted on October 4, 1996 and its election of the USTFU; there was no compliance with the ten-day notice rule required
results as null and void ab initio. by Section 1, Article VIII of the CBL; the supposed elections were conducted
without a COMELEC being constituted by the Board of Officers in
Accordingly, respondents Gil Gamilla, et al are hereby ordered to cease and accordance with Section 1, Article IX of the CBL; the elections were not by
desist from acting and performing the duties and functions of the legitimate secret balloting as required by Section 1, Article V and Section 6, Article IX
officers of [the] University of Santo Tomas Faculty Union (USTFU) pursuant of the CBL, and, the general assembly was convened by faculty members
to [the] union's constitution and by-laws (CBL). some of whom were not members of USTFU, so much so that non-USTFU
members were allowed to vote in violation of Section 1, Article V of the
The Temporary Restraining Order (TRO) issued by this Office on December CBL.
11, 1996 in connection with the instant petition, is hereby made and declared
permanent. 3 On 24 October 1996, appellees filed another urgent ex-parte motion for a
temporary restraining order, this time alleging that appellants had served the
Likewise challenged is the October 30, 1997 Resolution 4 of Director former a notice to vacate the union office. For their part, appellants moved to
Bitonio, which denied petitioners' Motion for Reconsideration. dismiss the original petition and the subsequent motion on jurisdictional
grounds. Both the petition and the motion were captioned to be for
The Facts "Prohibition, Injunction with Prayer for Preliminary Injunction and
Temporary Restraining Order." According to the appellants, the med-arbiter
The factual antecedents of the case are summarized in the assailed Resolution has no jurisdiction over petitions for prohibition, "including the ancillary
as follows: remedies of restraining order and/or preliminary injunction, which are merely
incidental to the main petition for PROHIBITION" (Paragraph XVIII3,
Petitioners-appellees [herein Private Respondents] Marino, et. al. (appellees) Respondents' Comment and Motion to Dismiss). Appellants also averred that
are duly elected officers of the UST Faculty Union (USTFU). The union has they now constituted the new set of union officers having been elected in
a subsisting five-year Collective Bargaining Agreement with its employer, accordance with law after the term of office of appellees had expired. They
the University of Santo Tomas (UST). The CBA was registered with the further maintained that appellees' scheduling of the 5 October 1996 elections
Industrial Relations Division, DOLE-NCR, on 20 February 1995. It is set to was illegal because no rules and regulations governing the elections were
expire on 31 May 1998. promulgated as required by USTFU's CBL and that one of the members of
the COMELEC was not a registered member of USTFU. Appellants likewise
On 21 September 1996, appellee Collantes, in her capacity as Secretary noted that the elections called by the appellees should have been postponed
General of USTFU, posted a notice addressed to all USTFU members to allow the promulgation of rules and regulations and to "insure a free, clean,
announcing a general assembly to be held on 05 October 1996. Among honest and orderly elections and to afford at the same time the greater
others, the general assembly was called to elect USTFU's next set of officers. majority of the general membership to participate" (See paragraph V, Idem).
Through the notice, the members were also informed of the constitution of a Finally, appellants contended that the holding of the general faculty assembly
Committee on Elections (COMELEC) to oversee the elections. (Annex "B", on 04 October 1996 was under the control of the Council of College/Faculty
petition) Club Presidents in cooperation with the USTFU Reformist Alliance and that
they received the Temporary Restraining Order issued in Case No. NCR-OD-
On 01 October 1996, some of herein appellants filed a separate petition with M-9610-001 only on 07 October 1996 and were not aware of the same on 04
the Med-Arbiter, DOLE-NCR, directed against herein appellees and the October 1996.
members of the COMELEC. Docketed as Case No. NCR-OD-M-9610-001,
the petition alleged that the COMELEC was not constituted in accordance On 03 December 1996, appellants and UST allegedly entered into another
with USTFU's constitution and by-laws (CBL) and that no rules had been CBA covering the period from 01 June 1996 to 31 May 2001 (Annex 11,
issued to govern the conduct of the 05 October 1996 election. appellants' Rejoinder to the Reply and Opposition).

On 02 October 1996, the secretary general of UST, upon the request of the Consequently, appellees again moved for the issuance of a temporary
various UST faculty club presidents (See paragraph VI, Respondents' restraining order to prevent appellants from making further representations
Comment and Motion to Dismiss), issued notices allowing all faculty that [they] had entered into a new agreement with UST. Appellees also
members to hold a convocation on 04 October 1996 (See Annex "C" Petition; reiterated their earlier stand that appellants were usurping the former's duties
Annexes "4" to "10", Appeal). Denominated as [a] general faculty assembly, and functions and should be stopped from continuing such acts.
the convocation was supposed to discuss the "state of the unratified UST-
USTFU CBA" and "status and election of USTFU officers" (Annex "11", On 11 December 1996, over appellants' insistence that the issue of
Appeal) jurisdiction should first be resolved, the med-arbiter issued a temporary
restraining order directing the respondents to cease and desist from
performing any and all acts pertaining to the duties and functions of the The petition is not meritorious. Petitioners fail to convince this Court that
officers and directors of USTFU. Director Bitonio gravely abused his discretion in affirming the med-arbiter
and in refusing to recognize the binding effect of the October 4, 1996 general
In the meantime, appellants claimed that the new CBA was purportedly assembly called by the UST administration.
ratified by an overwhelming majority of UST's academic community on 12
December 1996 (Annexes 1 to 10, Idem). For this reason, appellants moved First Issue:
for the dismissal of what it denominated as appellees' petition for prohibition
on the ground that this had become moot and academic. 5 Right to Self-Organization

Petitioners appealed the med-arbiter's Decision to the labor secretary, 6 who and Union Membership
transmitted the records of the case to the Bureau of Labor Relations which,
under Department Order No. 9, was authorized to resolve appeals of intra- At the outset, the Court stresses that National Federation of Labor (NFL) v.
union cases, consistent with the last paragraph of Article 241 of the Labor Laguesma 11 has held that challenges against rulings of the labor secretary
Code. 7 and those acting on his behalf, like the director of labor relations, shall be
acted upon by the Court of Appeals, which has concurrent jurisdiction with
The Assailed Ruling this Court over petitions for certiorari. However, inasmuch as the memoranda
in the instant case have been filed prior to the promulgation and finality of
Agreeing with the med-arbiter that the USTFU officers' purported election our Decision in NFL, we deem it proper to resolve the present controversy
held on October 4, 1994 was void for having been conducted in violation of directly, instead of remanding it to the Court of Appeals. Having disposed of
the union's Constitution and Bylaws (CBL), Public Respondent Bitonio the foregoing procedural matter, we now tackle the issues in the present case
rejected petitioners' contention that it was a legitimate exercise of their right seriatim.
to self-organization. He ruled that the CBL, which constituted the covenant
between the union and its members, could not be suspended during the Self-organization is a fundamental right guaranteed by the Philippine
October 4, 1996 general assembly of all faculty members, since that Constitution and the Labor Code. Employees have the right to form, join or
assembly had not been convened or authorized by the USTFU. assist labor organizations for the purpose of collective bargaining or for their
mutual aid and protection. 12 Whether employed for a definite period or not,
Director Bitonio likewise held that the October 4, 1996 election could not be any employee shall be considered as such, beginning on his first day of
legitimized by the recognition of the newly "elected" set of officers by UST service, for purposes of membership in a labor union. 13
or by the alleged ratification of the new CBA by the general membership of
the USTFU. Ruled Respondent Bitonio: Corollary to this right is the prerogative not to join, affiliate with or assist a
labor union. 14 Therefore, to become a union member, an employee must, as
This submission is flawed. The issue at hand is not collective bargaining a rule, not only signify the intent to become one, but also take some positive
representation but union leadership, a matter that should concern only the steps to realize that intent. The procedure for union membership is usually
members of USTFU. As pointed out by the appellees, the privilege of embodied in the union's constitution and bylaws. 15 An employee who
determining who the union officers will be belongs exclusively to the becomes a union member acquires the rights and the concomitant obligations
members of the union. Said privilege is exercised in an election proceeding that go with this new status and becomes bound by the union's rules and
in accordance with the union's CBL and applicable law. regulations.

To accept appellants' claim to legitimacy on the foregoing grounds is to When a man joins a labor union (or almost any other democratically
invest in appellants the position, duties, responsibilities, rights and privileges controlled group), necessarily a portion of his individual freedom is
of USTFU officers without the benefit of a lawful electoral exercise as surrendered for the benefit of all members. He accepts the will of the majority
defined in USTFU's CBL and Article 241(c) of the Labor Code. Not to of the members in order that he may derive the advantages to be gained from
mention the fact that labor laws prohibit the employer from interfering with the concerted action of all. Just as the enactments of the legislature bind all
the employees in the latter' exercise of their right to self-organization. To of us, to the constitution and by-laws of the union (unless contrary to good
allow appellants to become USTFU officers on the strength of management's morals or public policy, or otherwise illegal), which are duly enacted through
recognition of them is to concede to the employer the power of determining democratic processes, bind all of the members. If a member of a union
who should be USTFU's leaders. This is a clear case of interference in the dislikes the provisions of the by-laws, he may seek to have them amended or
exercise by USTFU members of their right to self-organization. 8 may withdraw from the union; otherwise, he must abide by them. It is not the
function of courts to decide the wisdom or propriety of legitimate by-laws of
Hence, this Petition. 9 a trade union.

The Issues On joining a labor union, the constitution and by-laws become a part of the
member's contract of membership under which he agrees to become bound
The main issue in this case is whether the public respondent committed grave by the constitution and governing rules of the union so far as it is not
abuse of discretion in refusing to recognize the officers "elected" during the inconsistent with controlling principles of law. The constitution and by-laws
October 4, 1996 general assembly. Specifically, petitioners in their of an unincorporated trade union express the terms of a contract, which define
Memorandum urge the Court to resolve the following questions: 10 the privileges and rights secured to, and duties assumed by, those who have
become members. The agreement of a member on joining a union to abide
(1) Whether the Collective Bargaining Unit of all the faculty members in that by its laws and comply with the will of the lawfully constituted majority does
General Faculty Assembly had the right in that General Faculty Assembly to not require a member to submit to the determination of the union any question
suspend the provisions of the Constitution and By-Laws of the USTFU involving his personal rights. 16
regarding the elections of officers of the union[.]
Petitioners claim that the numerous anomalies allegedly committed by the
(2) Whether the suspension of the provisions of the Constitution and By- private respondents during the latter's incumbency impelled the October 4,
Laws of the USTFU in that General Faculty Assembly is valid pursuant to 1996 election of the new set of USTFU officers. They assert that such
the constitutional right of the Collective Bargaining Unit to engage in exercise was pursuant to their right to self-organization.
"peaceful concerted activities" for the purpose of ousting the corrupt regime
of the private respondents[.] Petitioners' frustration over the performance of private respondents, as well
as their fears of a "fraudulent" election to be held under the latter's
(3) Whether the overwhelming ratification of the Collective Bargaining supervision, could not justify the method they chose to impose their will on
Agreement executed by the petitioners in behalf of the USTFU with the the union. Director Bitonio aptly elucidated: 17
University of Santo Tomas has rendered moot and academic the issue as to
the validity of the suspension of the Constitution and By-Laws and the The constitutional right to self-organization is better understood in the
elections of October 4, 1996 in the General Faculty Assembly[.] context of ILO Convention No. 87 (Freedom of Association and Protection
of Right to Organize), to which the Philippines is signatory. Article 3 of the
The Court’s Ruling Convention provides that workers' organizations shall have the right to draw
up their constitution and rules and to elect their representatives in full Unquestionably, the assembly was not a union meeting. It was in fact a
freedom, free from any interference from public authorities. The freedom gathering that was called and participated in by management and non-union
conferred by the provision is expansive; the responsibility imposed on union members. By no legal fiat was such assembly transformed into a union
members to respect the constitution and rules they themselves draw up activity by the participation of some union members.
equally so. The point to be stressed is that the union's CBL is the fundamental
law that governs the relationship between and among the members of the Second, there was no commission on elections to oversee the election, as
union. It is where the rights, duties and obligations, powers, functions and mandated by Sections 1 and 2 of Article IX of the USTFU's CBL, which
authority of the officers as well as the members are defined. It is the organic provide:
law that determines the validity of acts done by any officer or member of the
union. Without respect for the CBL, a union as a democratic institution ARTICLE IX - UNION ELECTION
degenerates into nothing more than a group of individuals governed by mob
rule. Sec. 1. There shall be a Committee on Election (COMELEC) to be created
by the Board of Officers at least thirty (30) days before any regular or special
Union Election vs. election. The functions of the COMELEC include the following:

Certification Election a) Adopt and promulgate rules and regulations that will ensure a free, clean,
honest and orderly election, whether regular or special;
A union election is held pursuant to the union's constitution and bylaws, and
the right to vote in it is enjoyed only by union members. A union election b) Pass upon qualifications of candidates;
should be distinguished from a certification election, which is the process of
determining, through secret ballot, the sole and exclusive bargaining agent of c) Rule on any question or protest regarding the conduct of the election
the employees in the appropriate bargaining unit, for purposes of collective subject to the procedure that may be promulgated by the Board of Officers;
bargaining. 18 Specifically, the purpose of a certification election is to and
ascertain whether or not a majority of the employees wish to be represented
by a labor organization and, in the affirmative case, by which particular labor d) Proclaim duly elected officers.
organization. 19
Sec. 2. The COMELEC shall be composed of a chairman and two members
In a certification election, all employees belonging to the appropriate all of whom shall be appointed by the Board of Officers.
bargaining unit can vote. 20 Therefore, a union member who likewise
belongs to the appropriate bargaining unit is entitled to vote in said election. xxx xxx xxx 24
However, the reverse is not always true; an employee belonging to the
appropriate bargaining unit but who is not a member of the union cannot vote Third, the purported election was not done by secret balloting, in violation of
in the union election, unless otherwise authorized by the constitution and Section 6, Article IX of the USTFU's CBL, as well as Article 241 (c) of the
bylaws of the union. Verily, union affairs and elections cannot be decided in Labor Code.
a non-union activity.
The foregoing infirmities considered, we cannot attribute grave abuse of
In both elections, there are procedures to be followed. Thus, the October 4, discretion to Director Bitonio's finding and conclusion. In Rodriguez v.
1996 election cannot properly be called a union election, because the Director, Bureau of Labor Relations, 25 we invalidated the local union
procedure laid down in the USTFU's CBL for the election of officers was not elections held at the wrong date without prior notice to members and
followed. It could not have been a certification election either, because conducted without regard for duly prescribed ground rules. We held that the
representation was not the issue, and the proper procedure for such election proceedings were rendered void by the lack of due process — undue haste,
was not followed. The participation of non-union members in the election lack of adequate safeguards to ensure integrity of the voting, and the absence
aggravated its irregularity. of the notice of the dates of balloting.

Second Issue: Third Issue:

USTFU's Constitution and Suspension of USTFU's CBL

By Laws Violated Petitioners contend that the October 4, 1996 assembly "suspended" the
union's CBL. They aver that the suspension and the election that followed
The importance of a union's constitution and bylaws cannot be were in accordance with their "constituent and residual powers as members
overemphasized. They embody a covenant between a union and its members of the collective bargaining unit to choose their representatives for purposes
and constitute the fundamental law governing the members' rights and of collective bargaining." Again they cite the numerous anomalies allegedly
obligations. 21 As such, the union's constitution and bylaws should be committed by the private respondents as USTFU officers. This argument
upheld, as long as they are not contrary to law, good morals or public policy. does not persuade.

We agree with the finding of Director Bitonio and Med-Arbiter Falconitin First, as has been discussed, the general faculty assembly was not the proper
that the October 4, 1996 election was tainted with irregularities because of forum to conduct the election of USTFU officers. Not all who attended the
the following reasons. assembly were members of the union; some, apparently, were even
disqualified from becoming union members, since they represented
First, the October 4, 1996 assembly was not called by the USTFU. It was management. Thus, Director Bitonio correctly observed:
merely a convocation of faculty clubs, as indicated in the memorandum sent
to all faculty members by Fr. Rodel Aligan, OP, the secretary general of the Further, appellants cannot be heard to say that the CBL was effectively
University of Santo Tomas. 22 It was not convened in accordance with the suspended during the 04 October 1996 general assembly. A union CBL is a
provision on general membership meetings as found in the USTFU's CBL, covenant between the union and its members and among members (Johnson
which reads: and Johnson Labor Union-FFW, et al. v. Director of Labor Relations, 170
SCRA 469). Where ILO Convention No. 87 speaks of a union's full freedom
ARTICLE VIII-MEETINGS OF THE UNION to draw up its constitution and rules, it includes freedom from interference
by persons who are not members of the union. The democratic principle that
Sec. 1. The Union shall hold regular general membership meetings at least governance is a matter for the governed to decide upon applies to the labor
once every three (3) months. Notices of the meeting shall be sent out by the movement which, by law and constitutional mandate, must be assiduously
Secretary-General at least ten (10) days prior to such meetings by posting in insulated against intrusions coming from both the employer and complete
conspicuous places, preferably inside Company premises, said notices. The strangers if the "protection to labor clause" of the constitution is to be
date, time and place for the meetings shall be determined by the Board of guaranteed. By appellant's own evidence, the general faculty assembly of 04
Officers. 23 October 1996 was not a meeting of USTFU. It was attended by members and
non-members alike, and therefore was not a forum appropriate for transacting
union matters. The person who moved for the suspension of USTFU's CBL
was not a member of USTFU. Allowing a non-union member to initiate the
suspension of a union's CBL, and non-union members to participate in a
union election on the premise that the union's CBL had been suspended in
the meantime, is incompatible with the freedom of association and protection
of the right to organize.

If there are members of the so-called "academic community collective


bargaining unit" who are not USTFU members but who would nevertheless
want to have a hand in USTFU's affairs, the appropriate procedure would
have been for them to become members of USTFU first. The procedure for
membership is very clearly spelled out in Article IV of USTFU's CBL.
Having become members, they could then draw guidance from Ang
Malayang Manggagawa Ng Ang Tibay v. Ang Tibay, 103 Phil. 669. Therein
the Supreme Court held that "if a member of the union dislikes the provisions
of the by-laws he may seek to have them amended or may withdraw from the
union; otherwise he must abide by them." Under Article XVII of USTFU's
CBL, there is also a specific provision for constitutional amendments. What
is clear therefore is that USTFU's CBL provides for orderly procedures and
remedies which appellants could have easily availed [themselves] of instead
of resorting to an exercise of their so-called "residual power". 26

Second, the grievances of the petitioners could have been brought up and
resolved in accordance with the procedure laid down by the union's CBL 27
and by the Labor Code. 28 They contend that their sense of desperation and
helplessness led to the October 4, 1996 election. However, we cannot agree
with the method they used to rectify years of inaction on their part and
thereby ease bottled-up frustrations, as such method was in total disregard of
the USTFU's CBL and of due process. The end never justifies the means.

We agree with the solicitor general's observation that "the act of suspending
the constitution when the questioned election was held is an implied
admission that the election held on that date [October 4, 1996] could not be
considered valid under the existing USTFU constitution . . .." 29

The ratification of the new CBA executed between the petitioners and the
University of Santo Tomas management did not validate the void October 4,
1996 election. Ratified were the terms of the new CBA, not the issue of union
leadership — a matter that should be decided only by union members in the
proper forum at the proper time and after observance of proper procedures.

Epilogue

In dismissing this Petition, we are not passing upon the merits of the
mismanagement allegations imputed by the petitioners to the private
respondents; these are not at issue in the present case. Petitioners can bring
their grievances and resolve their differences with private respondents in
timely and appropriate proceedings. Courts will not tolerate the unfair
treatment of union members by their own leaders. When the latter abuse and
violate the rights of the former, they shall be dealt with accordingly in the
proper forum after the observance of due process.

WHEREFORE, the Petition is hereby DISMISSED and the assailed


Resolutions AFFIRMED. Costs against petitioners.

SO ORDERED.
UNIVERSITY OF SANTO TOMAS vs. SAMAHANG On the other hand, respondent reduced its demands for the first year from
MANGGAGAWA NG UST (SM-UST) P8,000.00 monthly salary increase per employee to P7,000.00, and from
G.R. No. 169940 September 14, 2009 P75,000.00 signing bonus to P60,000.00 for each employee, but petitioner
YNARES-SANTIAGO, J.: insisted on its final offer. As a result, respondent declared a deadlock and
filed a notice of strike with the National Conciliation and Mediation Board -
Assailed in this petition for review on certiorari is the January 31, 2005 National Capital Region (NCMB-NCR).
Decision1 of the Court of Appeals in CA-G.R. SP No. 72965, which affirmed
the May 31, 2002 Order of the Secretary of the Department of Labor and Conciliation and mediation proved to be futile, such that in January 2002,
Employment (DOLE) directing the parties to execute a Collective Bargaining majority of respondent’s members voted to stage a strike. However, the
Agreement incorporating the terms in said Order with modification that the DOLE Secretary timely assumed jurisdiction over the dispute, and the parties
signing bonus is increased to P18,000.00. Also assailed is the September 23, were summoned and heard on their respective claims, and were required to
2005 Resolution2 denying the motion for reconsideration. submit their respective position papers.

Respondent Samahang Manggagawa ng U.S.T. (SM-UST) was the On May 31, 2002, the DOLE Secretary issued an Order,3 the pertinent
authorized bargaining agent of the non-academic/non-teaching rank-and-file portions of which read, as follows:
daily- and monthly-paid employees (numbering about 619) of petitioner, the
Pontifical and Royal University of Santo Tomas, The Catholic University of x x x In arguing on the reasonableness of its demands, it cites the income of
the Philippines (or UST), a private university in the City of Manila run by the school from tuition fee increases and the allocation of this amount to the
the Order of Preachers. In October 2001, during formal negotiations for a faculty and non-teaching employees of the School x x x. According to the
new collective bargaining agreement (CBA) for the academic year 2001 Union, the School’s estimate of the tuition fee increase for the school year
through 2006, petitioner submitted its "2001-2006 CBA Proposals" which, 2003-2004 at P76,410,000.00 is erroneous. The Union argues that the total
among others, contained the following economic provisions: income of the School from tuition fee increases for school year 2003-2004 is
P101,000,000.00 more or less, or a net of P98,252,187.36, after deducting
A. ACADEMIC YEAR 2001-2002 adjustments for additional charges, allowances and discounts. This is based
on the computation of the School’s Assistant Chief Accountant x x x.
1. Salary increase of P800.00 per month
xxxx
2. Signing bonus of P10,000.00
The Union feels that the members of the bargaining unit are the least favored.
3. Additional Christmas bonus of P2,000.00 On the wage increases alone, the Union points out that a comparison of the
average monthly salary of the non-academic personnel from school year
B. ACADEMIC YEAR 2002-2003 1995-1996 up to school year 1999-2000 shows a declining relative
percentage. For this period, the bargaining unit enjoyed an average monthly
1. Salary increase of P1,500.00 per month salary increase of 14.234%, the lowest being 8.9% in school year 1998-1999
and the highest being 15.38% in school year 1995-1996. The School’s offer
2. Additional Christmas bonus of P2,000.00 for this CBA cycle translates to an increase of only 8.23%, specified as
follows: (1) 5.69% increase in school year 2000-2001 (P1,000.00); (2) 9.15%
3. P6,000,000.00 for salary restructuring increase in school year 2001-2002 (P1,700.00); and (3) 9.86% increase in
2002-2003 (P2,000.00).
C. ACADEMIC YEAR 2003-2004
The Union also submits a comparative chart of the allocation to non-
1. Salary increase of P1,700.00 per month academic personnel of the 70% increase in tuition fees from school year
1996-1997 to 1999-2000 x x x. The average percentage allocation to non-
2. Additional Christmas bonus of P2,000.00 academic personnel during this period is 32.8% of the total 70% of total
tuition fee increases, the lowest being 20.83% for the school year 1999-2000
In November 2001, the parties agreed in principle on all non-economic and the highest being 43.11% of the total allocation in 1997-1998. Using
provisions of the proposed CBA, except those pertaining to Agency Contract P101,036,330.37 as the estimated increase in tuition fee, 70% of this amount,
or contractualization (Art. III, Sec. 3 of the proposed CBA), Union Leave of net of adjustment, is P68,775,831.15 x x x. The Union argues that it is entitled
the SM-UST President (No. 4 of the Addendum to the proposed CBA), and to at least the average percentage of allocation to it for the past four (4) school
hiring preference. years which is at 32.85%, or P22,592,860.53 of the total allocation of
P68,775,831.15.
In December 2001, petitioner submitted its final offer on the economic
provisions, thus: It maintains, however, that it is entitled to more than the average percentage
of its allocation of the total 70% because it is School practice to allocate more
A. ACADEMIC YEAR 2001-2002 than 70% of the total tuition fee increases for the salaries and benefits of
School employees. Comparing the employees’ share in the tuition fee
1. Salary increase of P1,000.00 per month increases from school year 1996-1997 to 1999-2000, the School allocated an
average percentage of 76.75% for the benefits and salaries of its personnel,
2. Signing bonus of P10,000.00 or from a low of 72% in 1998-1999 to a high of 84.4% in 1996-1997 x x x.
If the average is applied this year, the Union argues that the available amount
3. Additional Christmas bonus of P2,000.00 is P75,407,786.29. Because of this practice, the Union maintains that the
School is already estopped from arguing that the allocation for employee
B. ACADEMIC YEAR 2002-2003 wages and benefits should not exceed 70% of tuition fee increases.

1. Salary increase of P1,700.00 per month Aside from this amount, the Union maintains that it is entitled to an additional
P15,475,000.00, sourced from other income, for the signing bonus or one-
2. Additional Christmas bonus of P2,000.00 time grant of P25,000.00 per member x x x. The Union alleges that it is
school practice to appropriate other funds for the wages and benefits of its
3. P6,190,000.00 to be distributed in the form of salary restructuring employees. For the school year 1996-1997, the School used funds from other
sources to fund the P2,000,000.00 hospitalization fund and 50% of the
C. ACADEMIC YEAR 2003-2004 signing bonus for the academic personnel; in 1997-1998 and 1998-1999, it
used additional funds for the P1,000,000.00 hospitalization fund of the
1. Salary increase of P2,000.00 per month academic personnel; and in 1999-2000, it used other funds to finance the one-
time grant of P10,000.00 each to the non-academic personnel and additional
2. Additional Christmas bonus of P2,000.00 P4,000,000.00 for the hospitalization fund of the academic employees or a
total of P17,592,500.00 for the past four (4) academic years x x x.
The School cannot claim that the funds are insufficient to cover the expenses
for the CBA because for the fiscal year 2000-2001 alone, the accumulated Christmas Bonus
excess of revenues over expenses at the end of the year totaled
P148,881,678.00 x x x. The Statement of Revenues and Expenses from We note that the members of the bargaining unit receive a P6,500.00
School Operations collated from the audited Financial Statements of the Christmas bonus. Considering this current level, we believe that the School’s
School for the school years 1996-1997 up to 2000-2001 shows that except offer of P2,000.00 for each of the next three (3) years of the CBA is already
for school years 1996-1997 and 2000-2001, the School posted a net income reasonable. Under this grant, the workers’ Christmas bonus will stand at a
from school operations. Its average annual net income from school total of P12,500 at the end of the third year.
operations alone is P7,956,187.00 and the net loss in 2000-2001 was a result
of the revaluation of the Main Building as part of the assets from its fully Hospitalization Benefit
depreciated value so that a new depreciation cost was reported and charged
to general expenses. We believe that the current practice is already reasonable and should be
maintained.
From the foregoing arguments, the Union demands that an amount should be
allocated to it annually to finance its demands as follows: Meal Allowance

1st Year – P38,067,860.00 distributed as follows: P22,592,860.53 (share The Union failed to show any justification for its demand on this item, hence
from tuition fee increases) for the economic benefits with sliding effect on its demand on the increase of meal allowance is denied.
the succeeding years; plus P15,475,000.00 for the one-time signing bonus of
P25,000.00 for each employee sourced from other funds. Rice Allowance

2nd Year – P33,568,970.00 to apply to its demand for salary increase, We believe an additional 2 sacks of rice on top of the existing 6 sacks of rice
Christmas bonus, rice subsidy and clothing/uniform allowance. is reasonable and is hereby granted, effective on the second year.

3rd Year – P46,653,295.37 to apply to its demand for salary increase, Medical Allowance
Christmas bonus, medicine allowance, mid-year bonus allowance and meal
allowance. In the absence of any clear justification for an improvement of this benefit,
we find the existing practice to be already reasonable and should be
Based on the Union’s computation, its demands will cost the School a total maintained.
of P133,765,125.37 for the entire three (3) year period.
Uniform/Clothing
xxxx
The Union has not established why the School should grant the benefit; hence
Given all the foregoing, we cannot follow the Union’s formula and in effect this demand is denied.
disregard the School’s two other bargaining units; to do so is a distortion of
economic reality that will not bring about long term industrial peace. We Mid-year Bonus
cannot simply adopt the School’s proposal in light of the parties’ bargaining
history, particularly the pattern of increases in the last cycle. Considering all The P3,000.00 bonus is already fair and should be maintained.
these, we believe the following to be a fair and reasonable resolution of the
wage issue. Hazard Pay

1st Year – P1,000.00/month There is no basis to increase this benefit, the current level being fair and
reasonable.
2nd Year – P2,000.00/month
Educational Benefit
3rd Year – P2,200.00/month
The existing provision is already generous and should be maintained.
These increases, at a three-year total of P68,337,600, are less than the three
(3)-year increases in the last CBA cycle to accommodate the School’s proven Retirement Plan
lack of capacity to afford a higher increase, but are still substantial enough to
accommodate the workers’ needs while taking into account the symmetry We are convinced that the 100% of basic salary per year of service is already
that must be maintained with the wages of the other bargaining units. On a reasonable and should be maintained.
straight line aggregate of P5,200.00, the non-academic personnel will receive
P498.48 less than an Instructor I (member of the faculty union) who received Hiring Preference
an aggregate of P5,698.48, thus maintaining the gap between the teaching
and non-teaching personnel. The salary difference will as well be maintained Based on the Minutes of Meeting on 18 October 2001 and 8 November 2001,
over the three (3)-year period of the CBA. An RFI employee (member of the the parties agreed to retain the existing provision; hence, our ruling on this
union’s bargaining unit) will receive a monthly salary of P21,695.95 while matter is no longer called for.
an Instructor I (faculty union member) will have a salary of P22,948.00;
while an RF5-5/A (member of the union’s bargaining unit) will receive a Contractualization
salary of P23,462.97 compared to an Asst. Prof. 1 (faculty) who will receive
P29,250.96. From a total cost of salary increases for the first year at The Union’s proposed amendments are legal prohibitions which need not be
P7,428,000, these costs will escalate to P22,284,000 in the second year, and incorporated in the CBA. The Union has alternative remedies if it desires to
to P38,625,000 at the third year. Given these figures, the amounts available assail the School’s contracts with agencies.
for distribution and the member of groups sharing these amounts, these
increases are by no means minimal. Full-time Union Leave of Union President

Signing Bonus The Union failed to provide convincing reasons why this demand should be
favorably granted; hence, the same is denied.
A review of the past bargaining history of the parties shows that the School
as a matter of course grants a signing bonus. This ranged from P8,000.00 Other Demands
during the first three (3) years of the last CBA to P10,000.00 during the
remaining two (2) years of the re-negotiated term. In this instance, the All other demands not included in the defined deadlock issues are deemed
School’s offer of P10,000.00 signing bonus is already reasonable considering abandoned, except for existing benefits which the School shall continue to
that the School could have taken the position that no signing bonus is due on grant at their current levels consistent with the principle of non-diminution
compulsory arbitration in line with the ruling in Meralco v. Quisumbing et of benefits.
al., G.R. No. 127598, 27 January 1999.
WHEREFORE, premises considered, the parties are hereby directed to Academic 1,452 employees awarded P45 million
execute within ten (10) days from receipt of this Order a Collective Non-academic 619 employees awarded P15.475 million
Bargaining Agreement incorporating the terms and conditions of this Order Academic &
as well as other agreements made in the course of negotiations and on Administrative 219 employees awarded P8 million
conciliation.4
Total awarded P68,475,000.00
Respondent filed a motion for reconsideration but it was denied by the The difference between P68,775,831 (70% of incremental tuition fee
Secretary of Labor. Thus, respondent filed an original petition for certiorari proceeds) and P68,475,000 (total actual allocation or award to the two
with the Court of Appeals, claiming that the awards made by the DOLE bargaining units and the school officials) is P300,831.00, which is only
Secretary are not supported by the evidence on record and are contrary to law .437% of the 70% mandatory allocation (P68,775,831.00).
and jurisprudence.
The Supreme Court in the case of Cebu Institute of Medicine v. Cebu
On January 31, 2005, the appellate court rendered the assailed Decision, the Institute of Medicine Employees’ Union National Federation of Labor held
dispositive portion of which reads, as follows: that SSS, Medicare and Pag-Ibig employer’s share may be charged against
the "seventy percent (70%) incremental tuition fee increase (sic)" as they are,
WHEREFORE, premises considered, the petition is partially GRANTED. after all, for the benefit of the University’s teaching and non-teaching
The assailed Order of May 31, 2002 of Secretary Patricia Sto. Tomas is personnel. The High Court further ruled that "the private educational
hereby AFFIRMED with the modification that the P10,000.00 signing bonus institution concerned has the discretion on the disposition of the seventy
awarded is increased to P18,000.00. percent (70%) incremental tuition fee increase (sic). It enjoys the privilege of
determining how much increase in salaries to grant and the kind and amount
SO ORDERED.5 of allowances and other benefits to give. The only precondition is that
seventy percent (70%) of the incremental tuition fee increase (sic) goes to the
In arriving at the foregoing disposition, the appellate court noted that: payment of salaries, wages, allowances and other benefits of teaching and
non-teaching personnel."a1f
Based on UST Chief Accountant Antonio J. Dayag’s Certification, the tuition
fee increment for the SY 2001-2002 amounted to P101,036,330.37. From In the (sic) light of the foregoing jurisprudence, the University, in order to
this amount, the tuition fee adjustment amounting to P2,785,143.00 was comply with R.A. 6728, must fully allocate the 70% of the tuition fee
deducted leaving a net tuition fee increment of P98,251,189.36. increases to salaries, wages, allowances and other benefits of the teaching
and non-teaching personnel. The amount of P300,831.00 must therefore be
Pursuant to Section 5 (2) RA 6728, seventy percent (70%) of P98,251,187.36 allocated either as salary increment or fringe benefits of the non-teaching
or P68,775,831.15 is the amount UST has to allocate for salaries, wages, personnel.
allowances and other benefits of its 2,290 employees, categorized as follows:
619 non-teaching personnel represented by herein petitioner SM-UST; 1,452 We noted that UST’s non-teaching employees enjoy several fringe benefits.
faculty members represented by UST-Faculty Union (UST-FU) and 219
academic/administrative officials. The last group of employees is excluded We listed them down and estimated their costs for AY 2001-2002:
from the coverage of the two bargaining units.
1. P3,000.00 mid-year bonus P1,857,000.00
Public respondent, taking into consideration the bargaining history of the 2. 6 sacks of rice/employee
parties, the needs of the members of Union in relation to the capability of its @ P1,000.00/sack 3,714,000.00
employer, UST, to grant its demands, the impact of the award on the UST- 3. Hospitalization benefit 2,476,000.00
Faculty Union members (UST-FU), and how the present salary and benefits 4. Meal allowance
of the non-academic personnel compare with the compensation of the (P600/month/employee) 4,456,800.00
employees of other learning institutions, arrived at the following "fair and 5. Hazard pay (P200/month for
reasonable" resolution to the wage issue: 198 entitled employees)
8,430,780.00
1st year – P1,000.00/month 6. Medicine Allowance
(P1,000/month/employee) 7,428,000.00 20,407,000.00
2nd year – P2,000.00/month 7. SSS (P910.00 employer’s
share per employee) 6,759,480.00
3rd year – P2,000.00/month 8. Pag-Ibig (2% of the basic pay) 742,800.00
9. Phil Health (P125.00/employee) 928,500.00
Based on public respondent’s arbitral award for the first year (AY 2001- Total P28,837,780.00
2002), We determine the allocation that SM-UST would get from the 70% of =============
the tuition fee increment for AY 2001-2002 by approximating UST’s The allocation for salary increases, 13th month pay, signing bonus and
expense on the increment of salaries/wages, allowances and benefits of the Christmas bonus for UST’s teaching and non-teaching employees, as well as
non-teaching personnel: the school officials, amount to P68.475 million. This represents almost 70%
of the UST incremental tuition fee proceeds for AY 2001-2002. Considering
1. Increment on Salaries/Wages the fringe benefits being extended to UST employees, it is safe to assume that
+ 13th month pay the funds for such benefits need to be sourced from the University’s other
(P1,000 x 13 months x 619 employees) P 8,047,000.00 revenues. We looked into UST’s financial statements to determine its
2. Signing Bonus financial standing. The financial statements duly audited by independent and
(P10,000/employee) 6,190,000.00 credible external auditors constitute the normal method of proof of profit and
3. P2,000 Christmas Bonus 1,238,000.00 loss performance of a company. We examined UST audited financial
Total statements from 1997 to 2001 and found that the University’s "other
P15,475,000.00 incomes" come from parking fees, rent income and interest income. It,
============= likewise, derives income from school operations:
The amount of P15,475,000.00 represents 22.50% of the allocated
P68,775,831.00 (70% of the tuition fee increment for AY 2001-2002). UST
has allocated P45 million or 65.43% of the P68,775,831 to UST-Faculty 1999 2000 2001
Union. Income from
Operations P19,874,937.00 (24,222,602) (40,905,598)
Is the distribution equitable? If the share from the allocated P68,775,831.00 Other Income 85,995,039.00 77,335,032.00
for each bargaining unit would be based on the union’s membership, then the 78,358,303
distribution appears fair and reasonable: Excess of Revenues Over
Expenses Before
xxxx Income Tax 96,869,976.00 53,112,480.00
(29,726,651)
Provision for EMPLOYEES OF PETITIONER WERE SOURCED OUT FROM ITS
Income Tax 2,122,518.00 2,602,305.00 OTHER INCOME.
Excess of Revenues
Over Expenses 94,747,458.00 50,510,175.00 V.
(32,115,272)
ACCUMULATED THE HONORABLE COURT OF APPEALS COMMITTED PALPABLE
EXCESS OF ERROR OF SUBSTANCE AMOUNTING TO GRAVE ABUSE OF
REVENUES OVER DISCRETION WHEN IT IGNORED THE TIME HONORED
EXPENSES AT PRINCIPLES GOVERNING PETITION FOR CERTIORARI
END OF YEAR P180,996,950.00 P130,486,775.00 INVOLVING LABOR CASES.7
P148,881,678
Thus, if We charge the employees’ other benefits from the accumulated Petitioner alleges that, as of December 11, 2002, 526 regular non-academic
excess of revenues, We will come up with the following: employees – out of a total of 619 respondent’s members – have decided to
unconditionally abide by the May 31, 2002 Order of the DOLE Secretary.8
Accumulated Excess of Revenues A letter signed by the 526 non-academic employees allegedly reads:
Over Expenses (2001) P148,881,678.00
Less: December 3, 2002
Other Benefits of Non-Teaching Personnel 28,837,780.00
Balance TO: REV. FR. TAMERLANE R. LANA, O.P.
P120,043,898.00 Rector
Even if the other benefits of the faculty members were to be charged from
the remaining balance of the Accumulated Excess of Revenues Over REV. FR. JUAN V. PONCE, O.P.
Expenses, there would still be sufficient amount to fund the other benefits of Vice-Rector
the non-teaching personnel.
KAMI NA NAKALAGDA SA IBABA AY NAGPAPAABOT NG AMING
xxxx TAHASANG PAGTANGGAP SA AWARD NG SECRETARY OF
LABOR SA AMING (CBA) DEADLOCK CASE.
However, while We subscribe to UST’s position on "salary distortion", Our
earlier findings support the petitioner’s contention that the UST has SANA PO AY MA-RELEASE ANG AMING MGA WAGE
substantial accumulated income and thus, We deem it proper to award an ADJUSTMENTS AT IBA PANG BENEPISYO BAGO MAG DECEMBER
increase, not in salary, to prevent any salary distortion, but in signing bonus. 15, 2002.
The arbitral award of P10,000 signing bonus per employee awarded by public
respondent is hereby increased to P18,000.00. x x x x9

We are well aware of the need for the University to maintain a sound and Petitioner claims that it began paying the wage adjustment and other benefits
viable financial condition in the light of the decreasing number of its pursuant to the May 31, 2002 Order of the DOLE Secretary; and that to date,
enrollees and the increasing costs of construction of buildings and 572 out of the 619 members of respondent have been paid. It now argues that
modernization of equipment, libraries, laboratories and other similar by their acceptance of the award and the resulting payments made to them,
facilities. To balance this concern of the University with the need of its non- the said union members have ratified its offer and thus rendered moot the
academic employees, the additional award, which We deem reasonable, and case before the Court of Appeals (CA-G.R. SP No. 72965).
to be funded from the University’s accumulated income, is thus limited to
the increase in signing bonus.6 Petitioner also argues that the Court of Appeals erred in ordering it to source
part of its judgment award from the school’s other income, claiming that
Petitioner filed a motion for reconsideration, which the appellate court denied Republic Act 672810 does not compel or require schools to allocate more
in its September 23, 2005 Resolution. Hence, the instant petition which raises than 70% of the incremental tuition fee increase for the salaries and benefits
the following issues: of its employees. Citing an authority in education law, it stresses that –

I. Clearly, only 70% may be used for the "payment of salaries, wages,
allowances and other benefits of teaching and non-teaching personnel," since
THE HONORABLE COURT OF APPEALS COMMITTED PALPABLE 20% "shall go to the improvement or modernization of buildings, equipment,
ERROR OF SUBSTANCE WHEN IT RULED THAT THE MEMBERS OF libraries, laboratories, gymnasia and similar facilities and the payment of
PRIVATE RESPONDENT DID NOT VOLUNTARILY AND other costs of operation."
KNOWINGLY ACCEPT THE ARBITRAL AWARD OF THE
SECRETARY OF DOLE. A school does not exist solely for the benefit of its teachers and non-teaching
personnel. A school is principally established to deliver quality education at
II. all levels, as the Constitution requires. Therefore, any tuition fee increase
authorized by either the DepEd Secretary, the CHED or the Director General
THE HONORABLE COURT OF APPEALS COMMITTED PALPABLE of the TESDA for private schools should not solely benefit the teaching and
ERROR OF SUBSTANCE AMOUNTING TO GRAVE ABUSE OF non-teaching personnel but should rather be used for the welfare of the entire
DISCRETION WHEN IT INCREASED THE SIGNING BONUS school community, particularly the students. The students are entitled as a
AWARDED BY THE SECRETARY OF DOLE TO EACH OF THE matter of right to the improvement and modernization of the school
MEMBERS OF PRIVATE RESPONDENT FROM P10,000.00 TO "buildings, equipment," as this is fundamental to the maintenance or
P18,000.00. improvement of the quality of education they receive.

III. Thus, if schools use any part of the 20% reserved for the upgrading of school
facilities to supplement the salaries of their academic and non-academic
THE HONORABLE COURT OF APPEALS HAS COMPLETELY personnel, they would not only be violating the students’ constitutional right
IGNORED THE CLEAR MANDATE AND INTENTION OF R.A. 6728 to quality education through "improvement and modernization" but also
OTHERWISE KNOWN AS THE GOVERNMENT ASSISTANCE TO committing a serious infraction of the mandatory provisions of RA 6728.
STUDENTS AND TEACHERS IN PRIVATE EDUCATION ACT.
The law is silent, however, on the remaining ten percent of the tuition fee
IV. increase. The DepEd has referred to it as the "return of investment" for
proprietary schools and the "free portion" for non-stock, non-profit
THE HONORABLE COURT OF APPEALS COMMITTED PALPABLE educational institutions. This ten percent (10%) is the only portion of the
ERROR OF SUBSTANCE AMOUNTING TO GRAVE ABUSE OF tuition fee increase which schools may use as they wish.11
DISCRETION WHEN IT RULED THAT THE FRINGE BENEFITS
BEING ENJOYED BY THE ACADEMIC AND NON-ACADEMIC
Petitioner thus concedes liability only up to P300,831.00, which is the To put matters in their proper context, we must first simplify the facts.
remaining balance of the undistributed amount of P68,775,831.00, which
represents 70% of the incremental tuition fee proceeds for the period in Although the parties were negotiating on the CBA for academic years 2001
question. through 2006 (2001-2006 CBA Proposals), we are here concerned only with
the economic provisions for the academic year (AY) 2001-2002, specifically
Petitioner contends further that the appellate court’s award of additional the appellate court’s increased award of signing bonus, from P10,000.00 as
signing bonus (from P10,000.00 to P18,000.00) is contrary to the nature and originally granted by the DOLE Secretary, to P18,000.00; the parties do not
principle behind the grant of such benefit, which is one given as a matter of appear to question any other disposition made by the DOLE Secretary.
discretion and cannot be demanded by right,12 a consideration paid for the
goodwill that existed in the negotiations, which culminate in the signing of a Thus, it has been determined that from the tuition fees for the academic year
CBA.13 Petitioner claims that since this condition is absent in the parties’ in question, petitioner earned an increment of P101,036,330.37. Under R.A.
case, it was erroneous to have rewarded respondent with an increased signing 6728, 70% of that amount – or the net18 amount of P68,775,831.15 – should
bonus. be allotted for payment of salaries, wages, allowances and other benefits of
teaching and non-teaching personnel except administrators who are principal
Finally, petitioner endorses the original award of the DOLE Secretary, stockholders of the school.
calling her disposition of the case "fair and equitable"14 and deserving of our
attention, in light of the principle that – Of this amount (P68,775,831.15), an aggregate of P15,475,000.00 (or 22.5
%) was allocated to the university’s non-teaching or non-academic
The conclusions reached by public respondent (Secretary of Labor) in the personnel, by way of the following:
discharge of her statutory duty as compulsory arbitrator, demand the high
respect of this Court. The study and settlement of these disputes fall within Increment on Salaries/Wages
public respondent's distinct administrative expertise. She is especially trained plus 13th month pay
for this delicate task, and she has within her cognizance such data and (P1,000 x 13 months x 619
information as will assist her in striking the equitable balance between the non-academic personnel) P 8,047,000.00
needs of management, labor, and the public. Unless there is clear showing of Signing Bonus
grave abuse of discretion, this Court cannot and will not interfere with the (P10,000 per employee) 6,190,000.00
labor expertise of public respondent x x x.15 P2,000 Christmas Bonus 1,238,000.00
TOTAL
On the other hand, respondent seeks to sustain the appellate court’s 15,475,000.00
disposition, echoing its ruling that even though majority of the non-teaching On the other hand, the amount of P45 million (or 65.43% of P68,775,831.15)
employees agreed to petitioner’s offer and accepted payment thereupon, they was allocated to the teaching personnel.
are not precluded from receiving additional benefits that the courts may
award later on, bearing in mind that – After distribution of the respective shares of the teaching and non-teaching
personnel, there remained a balance of P300,831.00 from the
the employer and the employee do not stand on the same footing. Considering P68,775,831.15.
the country’s prevailing economic conditions, the employee oftentimes finds
himself in no position to resist money proffered, thus, his case becomes one In addition to the salary increase, signing and Christmas bonuses, the Court
of adherence and not of choice. This being the case, they are deemed not to of Appeals extended to respondent’s members the following fringe benefits
have waived any of their rights.16 for AY 2001-2002, which benefits petitioner has been giving its non-teaching
employees in the past, and which are included in the DOLE Secretary’s
As regards petitioner’s assertion that the funds to cover for the cost of the award – an award which petitioner prays for this Court to affirm in toto:
other benefits awarded by the DOLE Secretary may not be sourced from its
other income pursuant to R.A. 6728 as these benefits should only be paid out 1. P3,000.00 mid-year bonus P1,857,000.00
from the 70% tuition fee increment, respondent argues that R.A. 6728 – 2. 6 sacks of rice/employee
@ P1,000/sack 3,714,000.00
does not provide that the increase or improvement of the salaries and fringe 3. Hospitalization benefit 2,476,000.00
benefits of the employees should be exclusively funded from the income of 4. Meal allowance
the University which is derived from the increase in tuition fees. In fact, the (P600/month/employee) 4,456,800.00
statute has no application with respect to the manner of disposition of the 5. Hazard pay (P200/month for
other incomes (as distinguished from income derived from tuition fee 198 entitled employees) 8,430,780.00
increases) of the University, nor does it preclude or exempt the latter from 6. Medicine Allowance
using its other income or part thereof to fund the cost of increases or (P1,000/month/employee) 7,428,000.00 20,407,000.00
improvements in the salaries and benefits of its employees. x x x 7. SSS (P910.00 employer’s
share per employee) 6,759,480.00
15. Contrary to the assertion of Petitioner, it is very clear that the funds used 8. Pag-Ibig (2% of the basic pay) 742,800.00
by the University to cover the cost of other fringe benefits (under the existing 9. Philhealth (P125.00/employee) 928,500.00
CBA) granted to the non-academic employees for AY 2001-2002 in the
amount of P28,837,780.00 as observed by the Court of Appeals, came from
the other income of the University and not from the share of the said Total P28,837,780.00
employees in the income derived from the tuition fee increases during the Clearly, these fringe benefits would have to be obtained from sources other
same period. Logically, the grant of the said fringe benefits could not have than the incremental tuition fee proceeds (P68,775,831.15), since only
come from the amount of P15,475,000.00 which was already allocated by the P15,475,000.00 thereof was set aside for the non-teaching personnel; the rest
University to cover the total cost of the increases in the salaries, grant of was allocated to the teaching personnel.
signing bonus, and increase in the Christmas bonus to the non-academic
employees for AY 2001-2002.17 The appellate court, moreover, granted an increase in the signing bonus, that
is, from the DOLE Secretary’s award of P10,000.00, to P18,000.00. This,
On the appellate court’s award of additional signing bonus, respondent exactly, is the parties’ point of contention.
argues that since no strike or any untoward incident occurred, goodwill
between the parties remained, which entitles respondent’s members to Going now to the question of whether respondent’s members’ individual
receive their signing bonus. Besides, respondent asserts that since petitioner acceptance of the award and the resulting payments made by petitioner
did not appeal the DOLE Secretary’s award, it may not now argue against its operate as a ratification of the DOLE Secretary’s award which renders CA-
grant, the issue remaining being the propriety of the awarded amount; that is, G.R. SP No. 72965 moot, we find that such do not operate as a ratification of
whether or not it was proper for the appellate court to have raised it from the DOLE Secretary’s award; nor a waiver of their right to receive further
P10,000.00 to P18,000.00. benefits, or what they may be entitled to under the law. The appellate court
correctly ruled that the respondent’s members were merely constrained to
We resolve to PARTIALLY GRANT the petition. accept payment at the time. Christmas was then just around the corner, and
the union members were in no position to resist the temptation to accept
much-needed cash for use during the most auspicious occasion of the year.
Time and again, we have held that necessitous men are not, truly speaking,
free men; but to answer a present emergency, will submit to any terms that
the crafty may impose upon them.19

Besides, as individual components of a union possessed of a distinct and


separate corporate personality, respondent’s members should realize that in
joining the organization, they have surrendered a portion of their individual
freedom for the benefit of all the other members; they submit to the will of
the majority of the members in order that they may derive the advantages to
be gained from the concerted action of all.20 Since the will of the members
is personified by its board of directors or trustees, the decisions it makes
should accordingly bind them. Precisely, a labor union exists in whole or in
part for the purpose of collective bargaining or of dealing with employers
concerning terms and conditions of employment.21 What the individual
employee may not do alone, as for example obtain more favorable terms and
conditions of work, the labor organization, through persuasive and coercive
power gained as a group, can accomplish better.1avvphi1

Regarding petitioner’s assertion that it was unlawful for the Court of Appeals
to have required it to source the award of fringe benefits (in the amount of
P28,837,780.00) from the school’s other income, since R.A. 6728 does not
compel or require schools to allocate more than 70% of the incremental
tuition fee increase for the salaries and benefits of its employees, we find it
unnecessary to rule on this matter. These fringe benefits are included in the
DOLE Secretary’s award – an award which petitioner seeks to affirm in toto;
this being so, it cannot now argue otherwise. Since it abides by the DOLE
Secretary’s award, which it finds "fair and equitable," it must raise the said
amount through sources other than incremental tuition fee proceeds.

Finally, we come to the appellate court’s award of additional signing bonus,


which we find to be unwarranted under the circumstances. A signing bonus
is a grant motivated by the goodwill generated when a CBA is successfully
negotiated and signed between the employer and the union.22 In the instant
case, no CBA was successfully negotiated by the parties. It is only because
petitioner prays for this Court to affirm in toto the DOLE Secretary’s May
31, 2002 Order that we shall allow an award of signing bonus. There would
have been no other basis to grant it if petitioner had not so prayed. We shall
take it as a manifestation of petitioner’s liberality, which we cannot now
allow it to withdraw. A bonus is a gratuity or act of liberality of the giver;23
when petitioner filed the instant petition seeking the affirmance of the DOLE
Secretary’s Order in its entirety, assailing only the increased amount of the
signing bonus awarded, it is considered to have unqualifiedly agreed to grant
the original award to the respondent union’s members.

WHEREFORE, the petition is PARTIALLY GRANTED. The signing bonus


of EIGHTEEN THOUSAND PESOS (P18,000.00) per member of
respondent Samahang Manggagawa ng U.S.T. as awarded by the Court of
Appeals is REDUCED to TEN THOUSAND PESOS (P10,000.00). All other
findings and dispositions made by the Court of Appeals in its January 31,
2005 Decision and September 23, 2005 Resolution in CA-G.R. SP No. 72965
are AFFIRMED.

SO ORDERED.
HEIRS OF TEODOLO M. CRUZ vs. COURT OF INDUSTRIAL earned during the period of the back wages by each claimant and before
RELATIONS deduction of amounts corresponding to the back wages of claimants who died
G.R. No. L-23331-32 December 27, 1969 before April 30, 1962" at P6,380.00 for each of the 32 workers and
TEEHANKEE, J.: P28,000.00 for each of the 3 drivers

These cases are separate appeals filed by respective petitioners from P288,160.00
respondent Court's Orders of November 8, 1963 and March 9, 1964
approving by a split 3 to 1 vote the settlement for P100,000.00 of the (b) For overtime and premium pay from January 1, 1948 to December 31,
estimated P423,756.74 — judgment liability of respondent firm in favor of 1952 of some 104 workers, in varying amounts.5
the claimants-members of the Santiago labor Union, executed on November
8, 1963 between respondent firm and the labor union as represented by a 125,216.74
majority of its board of directors. The appeals are jointly resolved in this
decision. (c) For minimum wage differentials of P2.00 daily from September 10, 1951
to December 31, 1951 of 60 women workers
Petitioners in Cases L-23331-32 are the retained lawyers of the Santiago
Labor Union who question respondent Court's approval of respondent firm's 10,380.00
settlement of the union members' judgment claims with the union board of
directors, without their knowledge and consent, notwithstanding their duly TOTAL
recorded attorneys' lien, and over the objection of a board member that the
union board had no authority to compromise or quit-claim the judgment P423,756.74
rights of the union members.1
Petitioners claim, furthermore, that "in this computation, however, the filed
Petitioners in Cases L-23361-62 are forty-nine (49) claimants-members of examiners did not include the claims of seventy (70) other laborers whose
the Santiago Labor Union who assail respondent Court's approval of the total claims (for back wages), at the rate of P6,300.00 each, would be
questioned settlement, without their authority as the real parties in interest, P441,000.00. Therefore, the correct grand total amount due the laborers
and who denounce the settlement as unconscionable and having been entered would be P864,756.74."6
into by the majority of the union board "under circumstances of fraud, deceit,
mispresentation and/or concealment, especially where a member of the Court The Chief Examiner's Report showed respondent firm's total assets as at
has actively used his official and personal influence to effect the settlement October 31, 1962 to be P191.151.08 (cash account of P148,411.20, fixed
which is manifestly unjust to laborers who by reason of their financial assets of buildings, machinery & equipment, corn mill, etc. with a book value
disadvantages in a conflict with their employers need all the aid of the Court of P40,073.75 and deferred charges of P2,666.14), and its net worth to be in
for their protection, consonant with law, justice and equity."2 the same amount of P191,151.08, (capital stock paid up of P232,000.00 less
deficit of P40,848.92). the Report further stated that in January, 1962 and on
The factual background goes as far back as June 21, 1952, when the Santiago August 9, 1962, respondent firm sold its trucks, jeep and one car, with a net
Labor Union, composed of workers of the Santiago Rice Mill, a business book value of P2,628.71 for P27,000.00 or a net gain of P24,371.29.
enterprise engaged in the buying and milling of palay at Santiago, Isabela, Petitioners claim that the book value of respondent firm's fixed assets is only
and owned operated by King Hong Co., filed before the respondent Court of one-sixth of their actual market value of P240.442.50, and that its total
Industrial Relations Cases Nos. 709-V and V-1 hereof, a petition for overtime leviable assets therefore amounted to close to P390,000.00, without taking
pay, premium pay for night, Sunday and holiday work, and for reinstatement into account the huge income potential of its rice mill operations. Respondent
of workers illegally laid off. As of then, the total sum claimed by the workers, firm disputes such a figure as "completely gratuitous and without basis in
as itemized in their amended petition of September 2, 1952 — P100,816.36 fact."7
for overtime pay, P19,350.00 for premium pay and P3,360.00 for differential
pay under the Minimum Wage Law — amounted to P123,526.36.3 A general opposition to the Chief Examiner's Report was filed by respondent
firm. Judge Emiliano G. Tabigne, as the trial judge, supra, ordered a hearing
As recorded in this Court's decision of August 31, 1962 in Santiago Rice thereon on December 22, 1962, as a condition precedent to execution of the
Mill, et al. vs. Santiago Labor Union,4 which affirmed the Court of Industrial judgment. Such Report was submitted for resolution and approval at the
Relations judgment in favor of the workers, "on September 19, 1958, after a hearing of December 22, 1962, but the records before us fail to show that the
protracted hearing during which scores of witnesses and voluminous exhibits trial judge ever acted on or approved the Report.
were presented, the court, thru Judge Emiliano G. Tabigne, rendered decision
dismissing the petition of the union for lack of merit and want of jurisdiction; Before and after the submittal of the Chief Examiner's Report of December
but, upon a motion for reconsideration, the Court of Industrial en banc, by a 14, 1962, the union pressed for execution of the final judgment in favor of its
split decision of 3-2 vote, issued a resolution reversing the decision of the claimants-members. It filed, furthermore, on December 20, 1962, an Urgent
trial judge. The dispositive part of said resolution reads: Motion for Preliminary Attachment, in view of the disposition by respondent
firm of its trucks and automotive equipment and by virtue of the fact admitted
"WHEREFORE, the respondents are hereby ordered to pay the overtime by respondent firm that it had stopped operations preparatory to liquidation,
claim of both male and female claimants herein computed at their basic pay by reason of the alien nationality of most of its stockholders, under the
for each period in question; the legal premium for night, Sunday and holiday provisions of Republic Act No. 3018 nationalizing the rice and corn industry.
work or services rendered by the male claimants herein computed also on the In another motion of December 4, 1962, the union had asked that the Court
proven basic wage or salary at the time in question; to pay the overtime claim at least order respondent firm to put up a bond of P500,000.00 to answer for
of their drivers computed on their respective monthly salaries; to pay the the payment of the judgment or to deposit said amount in Court.
differentials due each of the women claimants on their wages from August
4, 1951 at the rate of P2.00 daily and P3.000 daily from August 4, 1952; and Petitioners assert that these motions were left hanging until the union filed a
to reinstate the claimants both male and female, who have testified and mandamus petition with this Court,8 after which the trial judge issued and
proved their having been illegally laid-off, with the right of respondents to released on April 15, 1963 his Order dated March 30, 1963. In this Order, the
deduct from the back wages due each claimant any amount earned during the trial Judge, recognizing that "petitioner (union) and its members concerned
period of the illegal dismissal." should be extended the necessary protection of their rights" ordered
respondent firm, within 10 days from its finality, to deposit in Court the sum
The worker's decade of travail was not yet to be at an end, however, despite of one hundred thousand (P100,000.00) pesos and to file a surety bond of
this Court's affirmance of the judgment for the workers. After the remand of equal amount, "to guarantee the payment of whatever amount (a) due
the records for enforcement by respondent Court, and the corresponding petitioner (union) and its members concerned after this Court shall have
examination of books, said Court's Chief Examiner filed his Partial Report finally decided the obligation of herein respondents under the judgment."
of December 14, 1962, wherein the judgment award in favor of the workers This Order was affirmed by respondent court en banc, in its Resolution
was determined and computed, as follows: denying respondent firm's motion for reconsideration thereof.

(a) For back wages from January 1, 1953 to April 30, 1962 of all the 35 Respondent sought a review by this Court of the said Order and Resolution
employees and laborers (26 workers, 6 laborers and 3 drivers) who testified requiring it to deposit P100,000.00 and to file a surety bond of equal amount
in court, per dispositive part of the judgment, "before deducting the amounts to guarantee payment of its judgment obligation in Santiago Rice Mill et al.
vs. Santiago Labor Union, etc., docketed as Cases G.R. Nos. L-21758-59 of P110,000.00 in full settlement of the claims of petitioners is with the full
this Court. this Court, in its Resolution of September 20, 1963, dismissed for agreement of the said respondent. We are disposed to deposit the amount of
lack of merit respondent's petition for review, and the dismissal became final P110,000.00 on or about Friday, November 8, 1963, and said deposit to be
on October 24, 1963. made with the Disbursing Officer of this Court and said deposit to be in
certified checks of a local bank and which is actually equivalent to cash. In
Earlier, June 25, 1963, pursuant to the request of the parties, who had advised line further with the suggestion of the Honorable Judge, we ware willing to
the trial judge that they would meet at the premises of respondent firm at assume the payment of the deposit fee upon our depositing the said amount
Santiago, Isabela, to take up direct negotiations for the possible settlement of of P110,000.00. There is a previous understanding which was not made of
the judgment, a team of employees of the Court had been sent to help in the record as to the fact that to enable the members of the board of directors of
negotiations. the transcript of the negotiations records that respondent had the petitioner union to come back to Manila next week to enable them to sign
then offered the Union the maximum amount of P110,000.00 in full the settlement papers, we have agreed to advance the sum of TWO
settlement of its obligations to the members-claimants of the Union under the HUNDRED PESOS (P200.00) to the petitioner for the account of said
judgment, but that the union rejected the offer and counter-offered the settlement and which will be used by the said petitioners in their travelling
minimum amount of P200,000.00. expenses between Manila and Santiago, going and coming.

The Union meanwhile filed to no avail a series of urgent motions on May 8, COURT:
July 1, August 29 and September 6, 1963 for approval of the Chief
Examiner's Partial Report of December 14, 1962 and for enforcement, Noted.
through a writ of execution or contempt proceedings, of the Order of March
30, 1963 requiring firm to deposit a total of P200,000 in cash and bond to MR. MAYLEM:
guarantee payment of the judgment. Upon the finality of this Court's
Resolution dismissing respondent's petition for a review of said Order of We request the Court that Mrs. Mary Concepcion should be presented during
March 30, 1963, the union again filed on October 29, 1963 still another the signing of the agreement on or about November 8, 1963, at 2:30 P.M.
Urgent Motion, advising the trial judge of this Court's action rejecting
respondent's appeal and invoking the Court's ministerial duty of enforcing its COURT:
said Order — in vain again, as shall presently be seen.
NOTED.9
The trial judge took no action on this latest Urgent Motion of the Union,
wherein it emphasized that respondent, with this Court's action rejecting its As against the official transcript of the proceedings of the conference above
appeal, no longer had any excuse for refusing its appeal, no longer had any reproduced, petitioner Natividad Magalpo, a director of the union, together
excuse for refusing to comply with the deposit Order. Instead, an with petitioners Lydia Bulos and Paciencia Batoon, both union members-
unscheduled conference was called and held on October 31, 1963 in the claimants, filed on November 5, 1963, through their present counsel, who
chambers of the trial judge, and attended by representatives of respondent duly entered their appearance, their verified "Manifestation and Objection
firm, including their counsels of record, on one hand and Segundino S. with Ex-Parte Urgent Motion", relating what transpired at the conference,
Maylem, president of the union and eight directors of the union, on the other. charging the union president, Maylem, with bad faith in that he never
Four of these nine union representatives, including the union president previously advised the union representatives that the conference of October
himself, had no claims of rewards whatever under the judgment. Said union 31, 1963 was to discuss a compromise settlement nor that this court's
officials were not assisted by counsel, as petitioner Mary Concepcion, resolution dismissing respondent's appeal from the trial judge's Order dated
counsel of record of the union, was not present, not having been notified of March 30, 1963 requiring respondent to deposit P200,000.00 in cash and
the conference. surety bond had already become final, and asking the trial judge to shelve the
proposed settlement until respondent firm shall have complied with the said
At this conference of October 31, 1963, respondent firm made again the same deposit order. The pertinent portions of said Objection and Urgent Motion
offer to settle and quitclaim the judgment in favor of the union members for read:
the same amount of P110,000.00, which offer had already been rejected by
the union at the earlier conference held on June 25, 1963 at Santiago, Isabela, 3. That during the conference, the matter of amicably settling the case was
supra. But this time, as appears from the transcript of the conference, discussed; petitioners representatives pressed for at least P150,000.00 as a
respondent and the directors of the union decided to settle the case amicably fair amount and the representatives of the respondents were insisting on their
with the payment by the firm of the same amount of P110,000.00 which was offer of a definite sum of P110,000.00;
deposited with the Court's disbursing officer "immediately upon the signing 4. That in the course of the conference, no mention at all was made of the
of the settlement which will be prepared by the respondent firm through its entry of judgment in G.R. Nos. L-21758-59, Supreme Court of the
counsel." The complete transcript of the conference, as reproduced by Philippines, entitled "Santiago Rice Mill, et al. vs. Santiago Labor Union,
respondent in its brief, follows: etc." on October 24, 1963, thereby becoming final and executory; that the
aforesaid entry of judgment reads as follows:
COURT:
"After a consideration of the allegation of the petition filed in cases
The parties have solicited the intervention of the court for the settlement of L-21758 and L-21759 (Santiago rice Mill, etc. vs. Santiago Labor Union, et
this case. They have decided to settle it amicably with the condition that the al.) for review of the order and resolution of the Court of Industrial Relations
management will pay ONE HUNDRED TEN THOUSAND PESOS referred to therein, the COURT RESOLVED to dismiss the petition for lack
(P110,000.00) cash, and that the said amount will be deposited with the of merit."
Disbursing Officer of the Court immediately upon the signing of the
settlement which will be prepared by the respondent firm through its counsel. 5. That by the terms of the afore-cited entry of judgment, the Respondent's,
Now, Mr. Maylem, make your manifestation on record. in effect, are ordered to deposit the sum of P100,000 in cash, Philippine
Currency and similar amount P100,000 in surety bond, pursuant to the order
MR. MAYLEM: of this Honorable court of March 30, 1963, which was affirmed in the
abovecited Supreme Court resolutions;
As per unanimous decision of the present members of the board composing
of nine, the three are not members of the board, present before this Honorable 6. That as a consequence of the ignorance of the Board of Directors of
Court to date, (sic) they have agreed to accept the proffer of ONE Petitioner of this entry, then present, they tentatively agreed to the offer of
HUNDRED TEN THOUSAND PESOS (P110,000.00) as full settlement of P110,000.00 of Respondents, until November 8, 1963 when the final
their claims in Cases Nos. 709-V and 709-V (1). conference before this Honorable Court will be held;

ATTY. GARCIA: 7. That movants consented to come to Manila on the understanding that the
conference was to be held with the Attorney-in-fact of the petitioner, the
In behalf of the respondent and the management of the said respondent and "CREAM, INC.," formerly, Credit Research and Intelligence, its exclusive
also in behalf of Mr. Pino, who is the attorney-in-fact of the respondent authorized representative for the evaluation, adjustment and liquidation of its
corporation, with full power to enter into this settlement, we wish to manifest claim against Respondent, that they were very much taken back in having
and uniform this Honorable Court that the acceptance of the proffer of been taken to the Court of Industrial Relations on October 31, 1963 by the
President of the Petitioner, Mr. Segundino S. Maylem; that even while they trial judge, was likewise ignored. Notwithstanding that notice of the
were already inside the building, they were informed that the purpose was to conference set for November 8, 1963 at 2:30 p.m. was served on November
talk about a compromise settlements with respondent's representatives; as a 5, 1963 on the union counsel, petitioner Mary Concepcion, the scheduled
result of these circumstances, your movants although present, were not able conference was never held.
to register their objections to the proceedings; that immediately after the
aforesaid conference, the herein movants came to know of the entry of Unexplained, Maylem, the union president and nine other members of the
judgment in the Supreme Court, infra; (sic) union's board of directors (out of 13 board members) even before the
scheduled hour of the conference of November 8, 1963 at 2:30 p.m. had
8. That the herein Movant's register and manifest their objections to the earlier executed a "Settlement" on said date, without the knowledge, advice,
proceeding held and to the tentative agreement manifested by the Board of and conformity of the union counsel, with respondent firm's attorney-in- fact,
Directors of the Santiago Labor Union then present, on the following who was duly assisted by respondent's two counsels, who likewise executed
grounds: the "Settlement." In this "Settlement", the said union officials claiming to act
"with the authorization of the Board of Directors and its members, "in
a) That the Board of Directors did not have any express authority of the consideration of the sum of P110,000.00, or one-fourth of the estimated
members of the Santiago Labor Union to enter into any compromise for the P423,756.74-judgment liability of respondent firm, as computed in the
sum of P110,000.00; on the contrary, the latest authority granted its Attorney- respondent Court's Chief Examiner's Partial Report of December 14, 1962,
in-fact, the "CREAM, INC." was for the sum of P150,000.00 which authority "waived and quitclaimed . . . any and all claims it (the union) may have
was given only, very recently: against the respondent as well as the claim of each and every one of the
members of the said petitioner union against the respondent firm." The union
b) That the proceedings on October 31, 1963 was tainted by apparent bad further "warranted" in said "Settlement" "that aside from the petitioner
faith on the part of the President of the Petitioner, Mr. Segundino s. Maylem, (union) itself and the members thereof, there are no other persons who have
in that there never was a time before the conference when he intimated or any interest over the judgment debt and that if it should happen that other
otherwise made known to the movants, that a conference would be held persons shall make a claim against the respondent and/or said judgment debt,
before Judge Emiliano Tabigne. The only reason for the trip to Manila was that the respondent, nevertheless, shall no longer be liable therefor."11
the conference with "CREAM, Inc." officials;
The "Settlement" was immediately submitted to the trial judge who forthwith
c) That the effect of the entry of judgment in G.R. Nos. on the same day, November 8, 1963, issued his Order, approving the same,
L-21758-59, infra, was not explained to the members of the Board of and entered into respondent Court's records at 1:45 p.m. of the same day, as
Petitioner at any time, much less made known, although it was later follows:
ascertained that President Segundino s. Maylem all the time, BEFORE THE
CONFERENCE, knew of the existence of the order; what was emphasized Considering that the bases of the above quoted settlement is well founded
was the claim of the Respondents that they are unable to pay more than and justified and not contrary to law, morals and/or public policy, approval
P110,000.00; (emphasis supplied.) of the same is, therefore, in order.

d) That the amount of P110,000.00 is unconscionable, considering that the WHEREFORE, the Court hereby approves the settlement of the parties in
total claims of the members of the Petitioner, is more than P400,000, not to these cases; and shall as between the parties to the same be deemed to be a
mention that all the time the negotiations were being made the Supreme decision and/or award in these matters therein treated in the aforesaid
Court's final order makes mandatory Respondent's deposit of P100,000, cas settlement; and upon acknowledgment of the sum of money in the said
in Philippine Currency and P100,000 in surety bond. settlement, these cases shall be deemed closed and terminated.

9. That Movant's vehemently disagree to any settlement as tentatively agreed Petitioners-lawyers Mary Concepcion, et al. upon learning of the
upon, for, in effect, they will only get fourteen percent, (14%) approximately, "Settlement" and respondent's deposit with the Court of the sum of
or one-seventh of the amounts as computed by the Chief Examiner of this P110,000.00 in pursuance thereof filed in the afternoon of November 8, 1963
Honorable Court; a motion for withdrawal of the sum of P33,000.00 equivalent to their 30%
contingent fee, without prejudice to such action as they may take for
xxx xxx xxx enforcing their lien to its full extent. The trial judge granted such motion in
its Order of November 9, 1963. In due course, said petitioners moved for
WHEREFORE, it is respectfully prayed that: reconsideration and setting aside of the trial judge's Order of November 8,
1963 approving the "Settlement" and prayed respondent Court en banc to
a) Respondent be required to deposit the sum of P100,000.00 in cash, reinstate the judgment against respondent and to enforce the deposit order
Philippine Currency, and P100,000.00 in surety bond, pursuant to the entry dated March 30, 1963.
of judgment in G.R. Nos. L-21758-59;
Petitioners Magalpo, Bulos and Batoon, likewise moved respondent Court en
b) That these movants be afforded opportunity by this Honorable Court to be banc to reconsider and set aside the trial judge's approval of the "Settlement",
heard regarding the surety bond to be submitted by the Respondent, before in disregard of their objection and pending motions of November 5, 1963 to
approval thereof; shelve the proposed settlement and to enforce the deposit Order. On
December 26, 1963, they were joined in their plea for reconsideration by
c) The tentative settlement be shelved; forty-seven other union members-claimants, Co-petitioners at bar.

d) The further action on any settlement or compromise be held in abeyance Respondent, on the other hand, filed its opposition to the motions for
to await compliance by the Respondent of the entry of judgment in G.R. Nos. reconsideration, questioning the personality and interest of petitioners-
L-21758-59; movants Magalpo and her 2 other co-movants and asserting that they were
bound by the "Settlement" entered into by their union's board of directors. It
e) Hearings on the Report of the Chief Examiner be resumed immediately alleged that it had deposited with respondent Court the sum of P110,000.00
and without interruption in view of the provisions of Republic Act 3108, until stipulated in the "Settlement" on the same day of its approval by the trial
final termination as soon as possible long before December 31, 1963,10 judge. It filed with respondent Court on November 21, 1963 a letter of
ratification dated November 10, 1963 addressed to the trial judge and
There petitioners further filed on the same date, November 4, 1963 an urgent purportedly signed by some 79 union members-claimants confirming and
Ex parte Motion for the issuance of a writ of execution for the enforcement accepting the settlement executed by the union board. Petitioners in their
of the deposit order against respondent firm, and asked the trial judge to act brief list 21 of these signatures as questionable, asserting that they are at
on their two urgent motions upon receipt thereof. variance with other corresponding signatures in the Payroll dated November
8, 1963 submitted to respondent Court on November 21, 1963, such that
Both urgent motions were totally ignored by both the trial judge as well as "either one or the other signature is a forgery." Respondent counters that there
by the respondent firm, despite due notice on the latter. The request of the is "absolutely no truth to the claim" and that the signers of the ratification
union president, Maylem, at the October 31, 1963 conference that the trial letter "have all received their individual shares of the P110,000.00 settlement
judge have the union counsel present during the proposed signing of the paid by respondent company and this in itself is a ratification on their part of
settlement agreement set for November 8, 1963, as expressly noted by the said settlement." Nothing appears in the record, however, as to whether and
in what manner the respondent Court determined the authenticity of the the timely opposition formally filed by three members (later joined by forty-
signatures. Respondent further filed on December 18, 1963 a motion for seven other members) expressly calling attention to the union board's bad
reconsideration of the trial judge's Order approving payment of P33,000.00 faith in the premises and lack of any express authority to enter into the
to the petitioners-attorneys by way of attorneys' fees. settlement, and without giving the union the opportunity of being heard and
assisted by counsel and notwithstanding the fact that respondent firm, which
On August 1, 1964, and August 4, 1964, after petitioners had filed on had sufficient cash and fixed assets, was under legal compulsion by virtue of
November 29, December 2 and 17, 1963 and January 16, 1964 various urgent respondent court's own final order to deposit P100,000.00 in cash and another
motions to set for hearing and for resolution, they were served with copies of P100,000.00 in surety bond to guarantee payment of the union members'
respondent Court's en banc Resolution dated March 9, 1964, penned by the judgment claims?
trial judge, "finding no sufficient justifications to set aside, disturb or modify
the Order issued in these cases on November 8 and 9, 1963" and denying all The question answers itself. The precipitate approval of the purported
three motions for reconsideration. Judges Amando C. Bugayong and settlement under the circumstances goes against the grain of fundamental
Ansberto F. Paredes concurred under date of July 29, 1964 with the considerations of justice, equity and due process.
Resolution, while Judge Arsenio Martinez took no part. No statement of the
material allegations of, and issues raised in, the pertinent pleadings set out in 1. To begin with, petitioners were not accorded due process of law, when, for
detail hereinabove nor reasons for the conclusion of insufficient justification reasons unexplained in the record, the conference set for November 8, 1963
reached by the majority resolution are given therein. at 2:30 p.m. to take up formally the proposed settlement was cancelled and
never held. (supra, pp. 8-9) Notice thereof had been served on the union
Then Presiding Judge Jose S. Bautista dissented. "Taking into account the counsel, in accordance with the express request of the union president, as
precipitate approval of settlement over the objection of some union members expressly noted by the trial judge. Yet, such notice was deliberately
concerned and without hearing them, on the strength simply of the disregarded and the union was deprived of the assistance of its counsel.13
manifestation of the petitioner's Board of Directors that it had authority to Instead, the settlement as unilaterally drafted by respondent's counsel (supra,
compromise when previously said union members concerned had already p. 7) was executed ahead of the scheduled hour of the conference that turned
manifested in Annex "E" (Exhibit "G", at bar) that there was no such out to be a non-conference, by the union president with nine other members
authority," he voted "that the case be restored to the status quo as of October of the union's board of directors, without the knowledge, advice and
30, 1963, but the payment already made to the union members be considered conformity of the union counsel, while respondent was duly assisted by its
as partial payments on account, subject to final liquidation and adjustment; two counsels. By 1:45 p.m. of the same day, the settlement had been
that an order of execution of the judgment in cases Nos. G.R. L-21758 and approved by the trial judge as "not contrary to law, morals and public policy."
L-21759 of the Supreme Court be issued (upholding the Order of March 30, Similarly, petitioners Magalpo, a board member herself and her co-
1963 for deposit of P200,000.00 in cash and surety bond) be issued and that petitioners Bulos and Batoon were not accorded an opportunity for a fair
the Hearing Officer shall resume the hearing of the Examiner's Report. hearing on their grave charges against the union leadership and their urgent
motions to shelve the proposed settlement and to enforce the final order of
Hence, the appeals of petitioners. respondent court requiring respondent firm to deposit P200,000.00 in cash
and surety bond for satisfaction of the union members' judgment, as said
The Santiago Labor Union, impleaded as party respondent in Cases L-23361- motions were totally ignored by the trial judge and not touched upon at all in
62, filed its Answer on September 24, 1964, "putting its weight behind the his Order rashly approving the settlement.
prayers of the petitioners." The Answer reveals that the union members,
feeling betrayed, had disauthorized and removed from office Maylem, the 2. The lack of due deliberation and caution in the trial judge's instant approval
union president and his board of directors who had executed the "Settlement" of the settlement is seen from the stipulations therein that the union thereby
with respondent firm and disclaimed the documents of ratification that they waived and quitclaimed any and all claims which it may have against the
had signed at the behest of Maylem. The union averred in its Answer that: respondent, as well as the claim of each and every one of the members of the
union against respondent, when precisely the authority of the union board
a) The real parties in interest in Cases 709-V and 709 V(1), CIR, are the members to enter into any such compromise or settlement was under express
members of respondent Labor Union; challenge by petitioner Magalpo, a board member herself in her Objection
and Urgent Motion to shelve the settlement filed on November 5, 1963,
b) The records of the respondent labor union do not show any grant by the which the trial judge completely disregarded. Petitioner Magalpo further
members to the former incumbency of any previous authority to negotiate made serious charges that Maylem, the union president, had misled the board
the claim or subsequent ratification of the settlement for P110,000.00 for it members into attending the unscheduled conference held on October 31,
is unthinkable and ridiculous for the real parties in interest to give away 1963 before the trial judge, and had deliberately concealed from them the
gratuitously what had been awarded to them in a final judgment, for a much fact of entry on October 24, 1963 of the Order of this Court in G.R. Nos. L-
lesser amount than that of the award; 21758-59 upholding the P200,000.00 deposit Order of respondent court and
the effect thereof of making mandatory upon the trial judge, in accordance
c) The members are unanimous in the assertion that the documents they with the terms of his own order, the issuance of a writ for execution or
signed at the behest of former President Segundino S. Maylem were enforcement to compel respondent to so deposit P100,000.00 in cash and an
represented and understood to be but an authority to collect a part of the court equal amount in surety bond to guarantee satisfaction of the union members'
award to the members; judgment against respondent. In point of facts, the union's own Urgent
Motion of October 29, 1963, emphasizing that respondent no longer had any
d) That the records of the respondent labor union disclose that the members excuse for not complying with the deposit order, as well as petitioner
of the union have unanimously acted, in their individual capacities to proceed Magalpo, et al.'s Urgent ex parte motion of November 4, 1963 to the same
with the prosecution and collection of whatever sums they might yet be effect were pending before the trial judge, unresolved and unacted upon.
entitled to collect, in order to show unequivocally that the negotiation made Petitioners Magalpo, et al. had reason therefore, to assail the proposed
by former President Segundino S. Maylem and his board of directors was settlement for P110,000.00 as unconscionable, when at the very least the
unauthorized, and to spotlight the betrayal of the members of the Union by union members could be assured of P200,000.00 under the deposit order to
said Segundino S. Maylem and his board of directors of the former union satisfy their judgment credit, while the report of respondent court's examiner
incumbency; showed that respondent firm had sufficient assets, (supra, p. 5), and
considering that their partial judgment credit, as estimated by respondent
6. That fundamentally, there is no contentious issue between the petitioners court's examiner, amounted to more than P400,000.00.
and respondent labor union; if at all, the only distinction is between the
personality of the real parties in interest, the union members who have 3. The trial judge's rush approval of the settlement disregarded the grave
initiated and instituted this petition as against the limited and formal adverse consequences thereof to the union members. The settlement, as
personality of the respondent labor union to represent them when so prepared by respondent's counsel, provided for a union warranty that aside
authorized by their collective will."12 from the union itself and the members thereof, "there are no other persons
who have any interests over the judgment debt and that if it should happen
The core question is whether this Court can give its sanction to respondent that other persons shall make a claim against the respondent and/or said
Court's majority resolution upholding the trial judge's approval of the union judgment debt, that the respondent, nevertheless, shall no longer be liable
board's settlement for P110,000.00 of the estimated P423,766.74-judgment therefor." Such warranty was against the very facts of record, which showed
liability of respondent firm in favor of the individual union members, over that as early as June 21, 1963, petitioners-counsels in Cases L-23331-32 had
duly recorded their attorneys' lien of "30 % of whatever amount may finally prayer for redress, categorically asserting that the union records do not show
be awarded in favor of the petitioner." Thus, technically, since the award in any grant by the members to the former union board under Maylem to
favor of the union members amounted to more than P400,000.00, the "negotiate the claim or subsequent ratification of the settlement for
settlement for P110,000.00 would conceivably just about cover the 30% P110,000.00" which is "unthinkable and ridiculous." (supra, p. 15) Under
attorneys' fees payable to the petitioners-counsels under the contract, if they such circumstances, the letter of ratification of the settlement purportedly
were so minded to enforce it and bad faith on the union's part were shown, signed by some 79 members, many of whose signatures thereon are
with the union members left holding an empty bag.14 Such onerous terms of denounced as forgeries and which ratification was not authenticated in the
the settlement could not then properly be approved by the trial judge as "not proceedings below and has been expressly disowned by petitioners herein,
contrary to law, morals and public policy." cannot be given any legal significance or effect.

4. All these underscore the failure of due process when petitioners were 6. When it is further taken into consideration that the judgment award, as
deprived of the formal conference on the proposed settlement scheduled for affirmed by this Court's decision of August 31, 1962,17 was for the payment
November 8, 1963 and of their right to be assisted by the union counsel as of overtime, premium and differential pay to the individual union members
expressly requested, so that a fair hearing could be accorded petitioners and as claimants and for the reinstatement of the individual union members who
an opportunity afforded them to air their serious charges of bad faith and lack testified and proved their having been illegally laid-off, which represent a
of authority against the union leadership. Certainly, all these serious personal material interest directly in favor of the individual union members,
questions and charges made by petitioners could have been threshed out and as against the lack of material interest on the part of the union as such, the
verified, if the formal conference scheduled for November 8, 1963 had been union's lack of authority to execute the settlement, in the absence of express
held with the presence of union counsel, considering that the latter likewise or specific authorization by the union members, becomes patent. The
had a right to be heard, since they had duly made of record their attorneys' authority of the union as such, to execute a settlement of the judgment award
lien upon the judgment.15 Respondent, in its brief, asserts that it vividly in favor of the individual union members, cannot be presumed but must be
remembers that the trial judge repeatedly made mention of the P200,000.00 expressly granted.
deposit order during the unscheduled conference of October 31, 1963 and
"even explained the matter to the members of the board in their native 7. Recently, in the analogous case of La Campana Food Products, Inc. etc.
dialect." But the transcript of the conference reproduced above (supra, pp. 7- Employees Ass'n vs. Court of Industrial Relations, et al.,18 this Court ruled
9) does not bear out this assertion. The transcript is obviously deficient and upon the merits of the union's appeal, and set aside the Industrial Court's
does not reflect the actual discussions and proceedings. This is to be questioned orders which would reopen its previous judgment finding the
deplored, for in a matter of such great importance, especially where the union employer guilty of unfair labor practice and ordering the reinstatement of,
officials were unassisted by counsel in an unscheduled conference, care and payment of back wages from December 4, 1963 to, twenty-one (21)
should be taken by the trial judge that the proceedings are faithfully recorded. union members. In handing down its decision, this Court disregarded the
Thus, although the transcript again fails to make any mention of it, petitioner union's motion to dismiss the appeal, filed through new counsel
respondent, in its brief, in effect provides support for petitioners' plaint while the case was pending decision, alleging that the union's legislative
against the unscheduled conference and precipitate approval of the settlement council had adopted a resolution relieving the former union counsel of his
behind the back of union counsel, when it states that "the presiding judge services and authorizing the dismissal of the case, on the premise that such
tried to help the parties reach a settlement by stressing to the union that there dismissal "would serve the best interests of both parties who are now in the
was no sense in demanding more than P110,000.00 from the respondent if process of formulating a collective bargaining agreement in their earnest
that was all it could afford, and that any more delay in the execution of its desire to establish industrial peace and promote the economic well-being of
award to the union members might lead to their getting much less than the all the parties concerned." For this Court ruled that the union's loss of interest
P110,000.00 already being offered by respondents," and "while it is true that in the case was no ground for dismissing the case, since "the labor union as
the presiding judge took an active part in helping the parties reach such a body in reality has not so great a material interest in the controversy as
settlement, it was only in line with the policy of the law encouraging would prejudice it in the event of dismissal. It is the twenty-one (21)
settlement of cases even after final judgment."16 The obvious fallacy of this members for whose benefit the ULP case was prosecuted who stand to take
untenable posture assumed by the trial judge, of course, is that with this Court tremendous losses" and suffer injustice. Upholding the individual union
having upheld his P200,000.00 deposit order, it made every sense to enforce members in their stand of vindicating their rights acquired under the final
execution of said order, which it was practically his ministerial duty to do so, judgment as against the union's legislative council's resolution to dismiss the
to assure the union members of recovery of their judgment credit at the very case, this Court, speaking through Mr. Justice Sanchez, thus held: —
least to the extent of P200,000.00, as the trial judge had expressly recognized
therein that "petitioner (union) and its members concerned should be We now come to the motion to dismiss filed in this Court on March 10, 1969
extended the necessary protection of their rights." Any further delay in the by new counsel for petitioner. In that motion, we read the averment that the
execution of the judgment award in favor of the union members could readily petitioning union, "after careful and serious consideration of their Petition,
be obviated, if the trial judge would but expedite the hearings for approval of taken in the light of recent developments affecting their relationship with the
the Court examiner's Report which had been filed and left pending since respondent-company, have decided that they have lost interest in the further
December 14, 1962. As correctly contended by petitioners, he could have prosecution of their claims"; that the union's legislative council, on February
placed the union members, unassisted as they were by counsel, on an equal 5, 1969, adopted a resolution authorizing the new counsel to file a motion
footing in negotiating with respondent by a mere stroke of his pen by ordering dismissing this case; that the former counsel who directed this case before
the enforcement of his final P200,000.00 deposit order, as to which there no this Court, Atty. Eulogio R. Lerum, had been relieved of his services in a
longer existed any obstacle. We find the forcing through of the settlement, letter of the union dated January 13, 1969; and that "the dismissal of this
under such circumstances, arbitrary, unfair and unconscionable. instant case would serve the best interests of both parties who are now in the
process of formulating a collective bargaining agreement in their earnest
5. Another vital reason for striking down the settlement is the lack of any desire to establish industrial peace and promote the economic well-being of
express or specific authority of the president and majority of the union board all parties concerned." This drew a reply from Atty. Eulogio R. Lerum that
of directors to execute the same and scale down the estimated P423,756.74- "while he admits that he had received termination notice from the alleged
judgment liability of respondent firm in favor of the individual union officers of the abovenamed union, he had not been disauthorized by the
members to P110,000.00. On the contrary, petitioner board member Magalpo complainants who had retained him to appear in their behalf" and that "said
timely challenged the authority of the union board to execute any such complainants are against the dismissal of their case for the reason that they
settlement, expressly informing the trial judge that the union had specifically want to vindicate their rights and it is against public policy to settle an unfair
appointed an entity in Manila, the "CREAM, Inc.", formerly Credit Research labor practice by amicable settlement (Sec. 5 [a], Rep. Act 875)."
and Intelligence, as its attorney-in-fact and "exclusive authorized
representative for the evaluation, adjustment and liquidation of its claim While it may be true that the labor union itself has lost interest in the case,
against respondent." Forty-seven other union members-claimants joined we do not believe that such should give ground for the dismissal of this case.
petitioner Magalpo in their denunciation of the union board's unauthorized The labor union as a body in reality has not so great a material interest in the
action, and in their plea for reconsideration with respondent court. Forty-nine controversy as would prejudice it in the event of dismissal. It is the twenty-
union members-claimants entitled to the bulk of the judgment award have one (21) members for whose benefit the ULP case was prosecuted who stand
filed this appeal from the adverse rulings of the Court below. These union to take tremendous losses. Nor is the argument that union and employer are
members have repudiated the former union president, Maylem and his board now in the process of formulating a collective bargaining agreement of any
of directors, for having betrayed the union members, and the new union consequence. That would not be affected by the decision we now render as
leadership, in its Answer filed with the Court, has joined petitioners in their an aftermath of the ULP case. Unless of course such a dismissal is a quid pro
quo before the parties could sit around the bargaining table. Which surely settlement were "bound by the actions of the Union, that is to say, a majority
enough is not to the 'best interests' of the laborers. of the members of the union." There was no question there that the union had
acted with the authority of the union membership. No deceit or concealment
And, as we examine the record, we observe none of the members of the or misrepresentation tainted the settlement. Neither was the amount of the
legislative council who adopted the resolution relied upon in the motion to settlement denounced as unconscionable. The employer there, Premiere
dismiss is personally affected by the decision rendered by the CIR in Case Productions, Inc., agreed to pay the amount of P200,000.00 which appeared
3985-ULP. That decision, it will be recalled, directs private respondents to be a reasonable settlement as against the judgment credit of the union
herein not only to reinstate the twenty-one (21) union members without loss workers, and further agreed to lease to the union its equipment and facilities
of seniority and other benefits and privileges but also to pay their respective for the Union to produce two moving pictures, apparently to cover the other
backwages from December 4, 1963, date of filing of the charge, basis of the wage claims of the union workers which were still pending trial and
complaint, until actual reinstatement. It is easy enough to perceive the resolution. In Diomela, the labor-management disputes were settled amicably
injustice which may be visited upon these twenty-one (21) union members if with the unfair labor practice charge against the employer, Squibb and Sons,
the petition herein were to be dismissed. For then, a new trial will be had, (Phil.) being withdrawn, upon motion signed by the union president and the
with the consequent trouble, expense, anxiety and another long delay before three employees against whom the acts of unfair labor practice charged in the
they could enjoy the fruits of their victory which they have legally and complaint had been allegedly committed, to which motion the Court's
definitely won only after a long and protracted legal battle. At any rate, it is prosecutor gave his conformity, and with the employer, which had secured a
better on balance that we foreclose a flanking movement which could destroy permanent writ of injunction restraining the strikers who had apparently
rather than uphold the rights — to reinstatement and monetary award — of declared an illegal strike, against the commission of acts of violence, threats
individual laborers acquired under the final judgment. and intimidation, agreeing to pay three months separation pay to each striking
employee. There was no question, therefore, of the authority of the union
8. Just as this Court has stricken down unjust exploitation of laborers by president to withdraw the unfair labor practice charge, as the three employees
oppressive employers, so will it strike down their unfair treatment by their directly affected had co-signed the withdrawal motion with him. The
own unworthy leaders. The Constitution enjoins the State to afford protection subsequent move of Diomela and 23 co-petitioners to disauthorize the union
to labor.19 Fair dealing is equally demanded of unions as well as of and its counsel of record, was by their own pleading overruled by the
employers in their dealings with employees. The union has been evolved as majority of the union membership. The other acts of unfair labor practice
an organization of collective strength for the protection of labor against the sought to be filed by Diomela and his companions were there ruled out as
unjust exactions of capital, but equally important is the requirement of fair splitting a cause of action and harassing the employer with subsequent
dealing between the union and its members, which is fiduciary in nature, and charges, based upon acts committed during the same period of time and
arises out of two factors: "one is the degree of dependence of the individual which should have been included in the charges first preferred. What should
employee on the union organization; the other, a corollary of the first, is the be borne in mind is that the interests of the individual worker can be better
comprehensive power vested in the union with respect to the individual."20 protected on the whole by a strong union aware of its moral and legal
The union may be considered but the agent of its members for the purpose of obligations to represent the rank and file faithfully and secure for them the
securing for them fair and just wages and good working conditions and is best wages and working terms and conditions in the process of collective
subject to the obligation of giving the members as its principals all bargaining. As has been aptly pointed out, the will of the majority must
information relevant to union and labor matters entrusted to it. As already prevail over that of the minority in the process, for "under the philosophy of
discussed above, the union leadership in the case at bar was recreant in its collective responsibility, an employer who bargains in good faith should be
duty towards the union members in apparently having failed to disclose to entitled to rely upon the promises and agreements of the union
the union members the full situation of their judgment credit against representatives with whom he must deal under the compulsion of law and
respondent, to wit, that they were in the advantageous position of being able contract. The collective bargaining process should be carried on between
to require enforcement of the respondent court's P200,000.00-deposit order, parties who can mutually respect and rely upon the authority of each
and in presuming that it had authority to waive and quitclaim the estimated other."23 Where, however, collective bargaining process is not involved, and
P423,756.74-judgment credit of the union members for the unconscionable what is at stake are back wages already earned by the individual workers by
amount of P110,000.00, which had already been previously rejected by the way of overtime, premium and differential pay, and final judgment has been
workers. Respondent firm could not claim that it dealt in good faith with the rendered in their favor, the present case, the real parties in interest with direct
union officials, for it hastily executed the purported settlement material interest, as against the union which has only served as a vehicle for
notwithstanding the serious charges of bad faith against the union leadership, collective action to enforce their just claims, are the individual workers
and the non-holding of the scheduled conference where the union leaders, at themselves.24 Authority of the union to waive or quitclaim all or part of the
their express request, could be duly assisted by union counsel. It is judgment award in favor of the individual workers cannot be lightly
noteworthy that respondent never filed with the court below any denial or presumed but must be expressly granted, and the employer, as judgment
responsive pleading traversing the factual allegations in petitioner Magalpo's debtor, must deal in all good faith with the union as the agent of the individual
Manifestation and Objection charging that at the unscheduled conference of workers. The Court in turn should certainly verify and assure itself of the fact
October 31, 1963, the proposed settlement was in effect railroaded with the and extent of the authority of the union leadership to execute any compromise
fact of the finality of the P200,000.00-deposit order not having been or settlement of the judgment on behalf of the individual workers who are
disclosed to the union representatives. Such failure on the part of respondent the real judgment creditors.
constitutes an implied admission of the material averments. Respondent's
justification now that it did not file any responsive pleading or denial because We therefore sustain the minority opinion of then Presiding Judge Bautista
Magalpo and her co-petitioners had no personality to file their pleadings as of respondent Court that the settlement was precipitately approved without
they were not parties to the cases in the lower court is of no avail, for they verification of the union board's authority to execute the compromise
were actually the awardees and beneficiaries under the judgment against settlement, and find that there was no such authority. The said settlement is
respondent and the union was but their agent. Deplorable also is the failure therefore set aside and the cases below are restored to the status quo, as of
of the trial judge to defer precipitate action on approval of the settlement until October 30, 1963, with the payments already made to the union members to
the union could be afforded the opportunity of a hearing thereon duly assisted be considered as partial payments on account, subject to final liquidation and
by counsel, and failure later of the majority of respondent court in the adjustment. It is directed that an order for the enforcement of the
reconsideration proceedings, as well, to look seriously into the grave charges P200,000.00-deposit order dated March 30, 1963 issued in the cases below,
of bad faith and deception against the union officials and their lack of and upheld in Cases G.R. Nos. L-21758-59 of this Court dismissing the
authority to execute the settlement. All of these charges were just swept under respondent's petition for review, be forthwith issued, and that hearings on the
the rug, and summarily dismissed, without even being mentioned, in the Chief Examiner's Report of December 14, 1962 be resumed immediately and
unreasoned en banc Resolution, finding arbitrarily as against the facts herein without interruption so that the amounts due under the judgment to the
collated by this Court from the pertinent pleadings and annexes furnished it, individual union members may be finally determined without further delay.
"no sufficient justification to set aside, disturb or modify" the questioned It is unfortunate that pending these proceedings, no application for
approval of the settlement. . preliminary injunction restraining respondent firm from disposing of its
assets was made, since as stated above, (supra, p. 5) respondent had stopped
9. The cases of Jesalva, et al. vs. Bautista,21 and Diomela, et al. vs. Court of operations in 1962 preparatory to liquidation, by virtue of the provisions of
Industrial Relations,22 cited by respondent, clearly have no application in the Republic Act No. 3018 nationalizing the rice and corn industry. The
present case. In Jesalva, seventeen cases in different stages of hearing or respondent firm's stockholders are, however, charged with notice of the
execution before the Industrial Court were settled by a compromise firm's liability by virtue of the pendency of these appeals, and should any
agreement, and this Court held that the three petitioners who questioned the liquidating dividends have been distributed and paid to them in the meantime,
they shall stand liable for the satisfaction of the union workers' judgment
against respondent to the extent of such dividends respectively paid to and
received by them. Similarly, any outstanding unpaid subscriptions or
balances of subscriptions to the firm's capital stock, estimated at
P20,000.00,25 shall be subject to garnishment and execution in satisfaction
of the judgment. As to the contingent 30% attorneys' fees of petitioners-
lawyers, the Court deems it proper at this stage, to direct in the exercise of its
authority to control the amount of such fees, that petitioners-lawyers may
collect their stipulated contingent 30% attorneys' fees to the extent that
additional amounts may be realized on the union workers' judgment up to the
sum of P150,000.00, including the initial payment of P110,000.00, (on which
they have already collected their corresponding fee), such that any further
amounts collected beyond said sum of P150,000.00 shall no longer be subject
to said contingent fee.

WHEREFORE, the respondent Court's Orders of November 8, 1963 and


March 9, 1964 are hereby declared null and void and set aside. The
respondent court is directed to proceed immediately with the execution of the
judgment rendered by it against respondent firm in Cases Nos. 709-V and V-
1 as affirmed by this Court's decision of August 31,
1962,26 in accordance with the directives set forth in the next preceding
paragraph, which is incorporated by reference as an integral portion of the
dispositive part of this decision. With costs against private respondent in both
cases herein decided.
SALUNGA vs. COURT OF INDUSTRIAL RELATIONS the Union and that, on the expiration of the 15-day grace granted to him by
G.R. No. L-22456 September 27, 1967 the Company, the decision thereof to terminate his services would take effect.
CONCEPCION, C.J.:
Thereupon, or on October 11, 1961, petitioner notified the PAFLU that he
Appeal by petitioner Francisco Salunga from a resolution of the Court of was appealing to its supreme authority — the PAFLU National Convention
Industrial Relations, sitting en banc, dismissing unfair labor practice charges — and requested that action on his case be deferred until such time as the
against the National Brewery and Allied Industries Labor Union of the Convention shall have acted on his appeal. A letter of the same date and tenor
Philippines (PAFLU) — hereinafter referred to as the Union — John de was sent, also, by the petitioner to the Union. Furthermore, he asked the
Castillo, Cipriano Cid, San Miguel Brewery, Inc. — hereinafter referred to Company to maintain the status quo, in the meantime. This notwithstanding,
as the Company — and Miguel Noel. at the close of the business hours, on October 15, 1961, petitioner was
discharged from the employment of the Company, through its assistant-
Petitioner had, since 1948, been an employee of the Company, which, on secretary and vice-president, herein respondent Miguel Noel.
October 2, 1959, entered with the Union, of which respondent John de
Castillo is the president, into a collective bargaining agreement, effective up At petitioner's behest, on or about December 7, 1961, a prosecutor of the
to June 30, 1962. Section 3 thereof reads: Court of Industrial Relations commenced, therefore, the present proceedings,
for unfair labor practice, against the Union, its president, respondent John de
The company agrees to require as a condition of employment of those Castillo, respondent Cipriano Cid, as PAFLU president, the Company, and
workers covered by this agreement who either are members of the UNION its aforementioned Vice-President Miguel Noel. In due course, thereafter, the
on the date of the signing of this agreement, or may join the UNION during trial Judge rendered a decision the dispositive part of which reads:
the effectivity of this agreement, that they shall not voluntarily resign from
the UNION earlier than thirty (30) days before the expiry date of this IN VIEW OF ALL THE FOREGOING, the San Miguel Brewery, Inc. and
agreement as provided in Article XIII hereof, provided, however, that Miguel Noel and National Brewery & Allied Industrial Labor Union of the
nothing herein contained shall be construed to require the company to Philippines (PAFLU), John de Castillo, and Cipriano Cid, are hereby
enforce any sanction whatsoever against any employee or worker who fails declared guilty of unfair labor practices as charged, and ordered to cease and
to retain his membership in the UNION as hereinbefore stated, for any cause desist from further committing such unfair labor practice acts complained of;
other than voluntary resignation or non-payment of regular union dues on the and as affirmative reliefs:
part of said employee or worker. (Exh. 4-A-Union.) .
(a) The National Brewery & Allied Industries Labor Union of the Philippines
Petitioner was a member of the Union since 1953. For reasons later to be (PAFLU), John de Castillo and Cipriano Cid, their officers and agents, are
stated, on August 18, 1961, he tendered his resignation from the Union, hereby directed to readmit and to continue the membership of Francisco
which accepted it on August 26, 1961, and transmitted it to the Company on Salunga in the membership rolls of the union after paying all union dues,
August 29, 1961, with a request for the immediate implementation of said with all the rights and privileges being enjoyed by bonafide members;
section 3. The Company having informed him that his aforementioned
resignation would result in the termination of his employment, in view of (b) The San Miguel Brewery, Inc., and Miguel Noel, their officers and agents
said section, petitioner wrote to the Union, on August 31, 1961, a letter are hereby directed to immediately reinstate Francisco Salunga to his former
withdrawing or revoking his resignation and advising the Union to continue or substantially equivalent position with one-half back wages, without
deducting his monthly union dues. He, moreover, furnished a copy of this prejudice, however, to his seniority and/or other rights and privileges; and
communication to the Company. The latter, in turn, notified the Union of the
receipt of said copy and that "in view thereof, we shall not take any action on (c) Respondents Union and Company, their respective officers and agents,
this case and shall consider Mr. Francisco Salunga still a member of your are likewise directed to post two copies of this decision in conspicuous places
union and continue deducting his union dues." On September 8, 1961, the in their respective offices or plants for a period of one month, furnishing this
Union told the Company that petitioner's membership could not be reinstated Court with certificate of compliance after the expiration of said period.
and insisted on his separation from the service, conformably with the
stipulation above-quoted. The Company replied, on September 12, 1961, On motion for reconsideration of the respondents, this decision was reversed
stating: by the Court of Industrial Relations — sitting en banc with two (2) judges
concurring in the result and the trial judge dissenting — which dismissed the
. . . We asked Mr. Salunga if he realized that by resigning from the Union he case. Hence, this appeal by the petitioner.
would in effect be forfeiting his position in the company. When he answered
in the negative, we showed him a copy of our Collective Bargaining The appeal is well taken, for, although petitioner had resigned from the Union
Agreement and called his attention to Sec. 3, Art. II thereof. He then told us and the latter had accepted the resignation, the former had, soon later — upon
that he did not realize that he would be losing his job if he were to resign learning that his withdrawal from the Union would result in his separation
from the Union. We did not at any time ask or urge him to withdraw his from the Company, owing to the closed-shop provision above referred to —
resignation; neither are we now asking or insisting that you readmit him into revoked or withdrawn said resignation, and the Union refused to consent
your membership. We thought that informing him of the consequences of his thereto without any just cause therefor. The Union had not only acted
resignation from the Union, was the only humane thing to do under the arbitrarily in not allowing petitioner to continue his membership. The trial
circumstances. Judge found said refusal of the Union officers to be due to his critical attitude
towards certain measures taken or sanctioned by them. As set forth in the
Nevertheless, if notwithstanding our foregoing clarification you still consider decision of the trial Judge:
him as having actually resigned from your organization, and you insist that
we dismiss him from the service in accordance with Sec. 3, Article II of our . . . Prior to August, 1961, he had been criticizing and objecting to what he
agreement, we will have no alternative but to do so. (Exh. E) believed were illegal or irregular disbursements of union funds, i.e., allowing
Florencio Tirad, a union official, to receive six months advanced salaries
In a letter to the Company, dated September 20, 1961, the Union reiterated when Tirad went to the United States, which objection he openly manifested
its request for implementation of said section 3, for which reason, on in a meeting of the board of directors and stewards, but instead of receiving
September 22, 1961, the Company notified petitioner that, in view of said favorable response, he (Salunga) was twitted and felt insulted by the laughter
letter and the aforementioned section, "we regret we have to terminate your of those present that he would be the next man to be sent to America; second,
employment for cause. You are, therefore, hereby notified of your dismissal granting Ricardo Garcia, union secretary, two months advanced salaries
from the service effective as of the close of business hours, September 30, when preparing for the bar examinations, which objection he broached to
1961." union officer Efren Meneses; third, the union's additional monthly expense
for the salary of a counsel when the PAFLU, their mother union is well
Meanwhile, petitioner had sought the intervention of PAFLU's National staffed with a number of lawyers who could attend to and handle their cases
President, respondent Cipriano Cid, to which the Union was affiliated, for a and other legal matters, and to which mother union the NABAILUP has been
review of the latter's action. The PAFLU gave due course to petitioner's paying a monthly assessment of more than P1,000.00; and fourth, giving
request for review and asked the Company, on September 29, 1961, to defer salary to Charles Mitschek who was dismissed by the company but denying
his dismissal, for at least two (2) weeks, so that its (PAFLU's) Executive the same privilege to other similarly situated member-employees. Salunga
Board could act on his appeal. On October 6, 1961, respondent Cid advised was later removed by the union from his position as steward without his
petitioner that the PAFLU had found no ground to review the action taken by knowledge. It also appears that the power of attorney executed in his favor
by co-worker Alejandro Miranda for the collection of Miranda's indebtedness
of P60.00 to him (the latter has certain amount in possession of the Union) Under these circumstances, the Company was not "unfair" to the petitioner.
was not honored by the union.1awphîl.nèt On the contrary, it did not merely show a commendable understanding of and
sympathy for his plight. It even tried to help him, although to such extent
xxx xxx xxx only as was consistent with its obligation to refrain from interfering in purely
internal affairs of the Union. At the same time, the Company could not safely
The record is clear that feeling dejected by the inaction of the union officials inquire into the motives of the Union officers, in refusing to allow the
on his grievances and objections to what he believed were illegal petitioner to withdraw his resignation. Inasmuch as the true motives were not
disbursements of union funds, coupled with the fact that he was later removed manifest, without such inquiry, and petitioner had concededly tendered his
from his position as a union steward without his knowledge, as well as the resignation of his own free will, the arbitrary nature of the decision of said
fact that the union did not honor the power of attorney executed in his favor officers was not such as to be apparent and to justify the company in
by Alejandro Miranda, a co-worker, for the collection of Miranda's regarding said decision unreasonable. Upon the other hand, the Company can
indebtedness of P60.00 to him, he submitted his letter of resignation from the not be blamed for assuming the contrary, for petitioner had appealed to the
union on August 18, 1961. It must be stated here that no evidence was National Officers of the PAFLU and the latter had sustained the Union. The
adduced by the respondent union to overcome complainant's testimonies Company was justified in presuming that the PAFLU had inquired into all
about his objections to the disbursements of union funds but only tried to relevant circumstances, including the motives of the Union Officers.
elicit from him, on cross examination, that the funds of the union are only
disbursed upon authority of the Executive Board of the union. . . . In finding, this notwithstanding, that the Company is guilty of unfair labor
practice, the trial Judge seemed to have been unduly influenced by the fact
It should be noted that the Court of Industrial Relations en banc did not that the former had dismissed the petitioner despite his announced intention
reverse these findings of fact or even question the accuracy thereof. What is to appeal from the decision of the Union and that of the Officers of PAFLU
more, the officers of the Union have, in effect, confirmed the fact that their to its "Supreme authority", namely, the PAFLU's "National Convention". In
refusal to allow the withdrawal of petitioner's resignation had been due to his other words, said Judge felt that the Company should have waited for the
aforementioned criticisms. Indeed said officers tried to justify themselves by action of the national convention before issuing the notice of dismissal.
characterizing said criticisms as acts of disloyalty to the Union, which, of
course, is not true, not only because the criticism assailed, not the Union, but There is no evidence, however, that petitioner had really brought this matter
certain acts of its officers, and, indirectly, the officers themselves, but also to said "Convention". Much less is there any proof that the latter had
because the constitution and by-laws of the Union explicitly recognize the sustained him and reversed the PAFLU officers and the Union. Thus, the
right of its members to give their views on "all transactions made by the record does not show that petitioner was prejudiced by the Company's failure
Union." As a consequence, the resolution appealed from cannot be affirmed to maintain the status quo, after the Union had been sustained by said officers.
without, in effect, nullifying said right which, independently of the In fact, petitioner did not even try to establish that he had submitted to the
constitution and by-laws of the Union, is part and parcel of the freedom of Company — as he has not introduced in the lower court — satisfactory proof
speech guaranteed in the Constitution of our Republic, as a condition sine that an appeal had really been taken by him to the aforementioned
qua non to the sound growth and development of labor organizations and Convention. In short, it was error to hold the Company guilty of unfair labor
democratic institutions. practice.

Although, generally, a state may not compel ordinary voluntary associations Just the same, having been denied readmission into the Union and having
to admit thereto any given individual, because membership therein may be been dismissed from the service owing to an unfair labor practice on the part
accorded or withheld as a matter of privilege,1 the rule is qualified in respect of the Union, petitioner is entitled to reinstatement as member of the Union
of labor unions holding a monopoly in the supply of labor, either in a given and to his former or substantially equivalent position in the Company,
locality, or as regards a particular employer with which it has a closed-shop without prejudice to his seniority and/or rights and privileges, and with back
agreement.2 The reason is that pay, which back pay shall be borne exclusively by the Union. In the exercise
of its sound judgment and discretion, the lower court may, however, take
. . . The closed shop and the union shop cause the admission requirements of such measures as it may deem best, including the power to authorize the
trade union to become affected with the public interest. Likewise, a closed Company to make deductions, for petitioner's benefit, from the sums due to
shop, a union shop, or maintenance of membership clauses cause the the Union, by way of check off or otherwise, with a view to executing this
administration of discipline by unions to be affected with the public interest.3 decision, and, at the same time, effectuating the purposes of the Industrial
Peace Act.
Consequently, it is well settled that such unions are not entitled to arbitrarily
exclude qualified applicants for membership, and a closed-shop provision With this modification, the aforementioned decision of the trial Judge is
would not justify the employer in discharging, or a union in insisting upon hereby affirmed in all other respects, and the appealed resolution of the Court
the discharge of, an employee whom the union thus refuses to admit to of Industrial Relations en banc is reversed, with costs against respondents,
membership, without any reasonable ground therefor.4 Needless to say, if except the Company.
said unions may be compelled to admit new members, who have the requisite
qualifications, with more reason may the law and the courts exercise the
coercive power when the employee involved is a long standing union
member, who, owing to provocations of union officers, was impelled to
tender his resignation, which he forthwith withdrew or revoked. Surely, he
may, at least, invoke the rights of those who seek admission for the first time,
and can not arbitrarily he denied readmission.

We cannot agree, however, with the finding of the trial Judge to the effect
that the Company was guilty of unfair labor practice. The Company was
reluctant — if not unwilling — to discharge the petitioner. When the Union
first informed the Company of petitioner's resignation and urged
implementation of section 3 of the bargaining contract, the Company advised
petitioner of the provision thereof, thereby intimating that he had to withdraw
his resignation in order to keep his employment. Besides, the Company
notified the Union that it (the Company) would not take any action on the
case and would consider the petitioner, "still a member" of the Union. When
the latter, thereafter, insisted on petitioner's discharge, the Company still
demurred and explained it was not taking sides and that its stand was
prompted merely by "humane" considerations, springing from the belief that
petitioner had resigned from the Union without realizing its effect upon his
employment. And, as the Union reiterated its demand, the Company notified
petitioner that it had no other alternative but to terminate his employment,
and dismissed him from the service, although with "regret".
VILLAR vs. INCIONG barred by the new CBA concluded on February 15, 1977; (5) there was no
G.R. No. L-50283-84 April 20, 1983 valid disaffiliation from PAFLU; and (6) the supporting signatures were
GUERRERO, J.: procured through false pretenses.

Petition for review by certiorari to set aside the Order dated February 15, Finding that the petition involved the same parties and causes of action as the
1979 of respondent Deputy Minister Amado G. Inciong affirming the case previously indorsed to the TUCP, the Med-Arbiter dismiss the petition
Decision of the OIC of Regional Office No. 4 dated October 14, 1978 which filed by herein petitioner Villar, which dismissal is still pending appeal
jointly resolved RO4-Case No. T-IV-3549-T and RO4-Case No. RD 4-4088- before the Bureau of Labor Relations.
77-T.
In the meantime, on February 14, 1977, the Amigo Employees Union-
The facts are as follows: PAFLU called a special meeting of its general membership. A Resolution
was thereby unanimously approved which called for the investigation by the
Petitioners were members of the Amigo Employees Union-PAFLU, a duly PAFLU national president, pursuant to the constitution and by-laws of the
registered labor organization which, at the time of the present dispute, was Federation, of all of the petitioners and one Felipe Manlapao, for
the existing bargaining agent of the employees in private respondent Amigo "continuously maligning, libelling and slandering not only the incumbent
Manufacturing, Inc. (hereinafter referred to as Company). The Company and officers but even the union itself and the federation;" spreading 'false
the Amigo Employees Union-PAFLU had a collective bargaining agreement propaganda' that the union officers were 'merely appointees of the
governing their labor relations, which agreement was then about to expire on management', and for causing divisiveness in the union.
February 28, 1977. Within the last sixty (60) days of the CBA, events
transpired giving rise to the present dispute. Pursuant to the Resolution approved by the Amigo Employees Union-
PAFLU, the PAFLU, through its national President, formed a Trial
On January 5, 1977, upon written authority of at least 30% of the employees Committee to investigate the local union's charges against the petitioners for
in the company, including the petitioners, the Federation of Unions of Rizal acts of disloyalty inimical to the interest of the local union, as well as
(hereinafter referred to as FUR) filed a petition for certification election with directing the Trial Committee to subpoena the complainants (Amigo
the Med-Arbiter's Office, Regional Office No. 4 of the Ministry of Labor and Employees Union-PAFLU) and the respondents (herein petitioners) for
Employment. The petition was, however, opposed by the Philippine investigation, to conduct the said investigation and to submit its findings and
Association of Free Labor Unions (hereinafter referred to as PAFLU) with recommendations for appropriate action.
whom, as stated earlier, the Amigo Employees Union was at that time
affiliated. PAFLU's opposition cited the "Code of Ethics" governing inter- And on the same date of February 15, 1977, the Amigo Employees Union-
federation disputes among and between members of the Trade Unions PAFLU and the Company concluded a new CBA which, besides granting
Congress of the Philippines (hereinafter referred to as TUCP). Consequently, additional benefits to the workers, also reincorporated the same provisions of
the Med-Arbiter indorsed the case to TUCP for appropriate action but before the existing CBA, including the union security clause reading, to wit:
any such action could be taken thereon, the petitioners disauthorized FUR
from continuing the petition for certification election for which reason FUR ARTICLE III
withdrew the petition. UNION SECURITY WITH RESPECT TO PRESENT MEMBERS

On February 7, 1977, the same employees who had signed the petition filed All members of the UNION as of the signing of this Agreement shall remain
by FUR signed a joint resolution reading in toto as follows: members thereof in good standing. Therefore, any members who shall resign,
be expelled, or shall in any manner cease to be a member of the UNION,
Sama-Samang Kapasiyahan shall be dismissed from his employment upon written request of the UNION
to the Company. 2
1. TUMIWALAG bilang kasaping Unyon ng Philippine Association of Free
Labor Unions (PAFLU) at kaalinsabay nito, inaalisan namin ang PAFLU ng Subsequently, petitioners were summoned to appear before the PAFLU Trial
kapangyarihan na katawanin kami sa anumang pakikipagkasundo (CBA) sa Committee for the aforestated investigation of the charges filed against them
Pangasiwaan ng aming pinapasukan at kung sila man ay nagkasundo o by the Amigo Employees Union-PAFLU. Petitioners, however, did not
magkakasundo sa kabila ng pagtitiwalag na ito, ang nasabing kasunduan ay attend but requested for a "Bill of Particulars" of the charges, which charges
hindi namin pinagtitibay at tahasang aming itinatakwil/tinatanggihan; were stated by the Chairman of the committee as follows:

2. BINABAWI namin ang aming pahintulot sa Federation of Unions of Rizal 1. Disaffiliating from PAFLU and affiliating with the Federation of Unions
(FUR) na katawanin kami sa Petition for Certification Election (RO4-MED of Rizal (FUR).
Case No. 743-77) at/o sa sama-samang pakikipagkasundo sa aming patrons;
2. Filling petition for certification election with the Bureau of Labor
3. PANATILIHIN na nagsasarili (independent) ang aming samahan, AMIGO Relations and docketed as Case No. R04-MED-830-77 and authorizing a
EMPLOYEES' UNION, alinsunod sa Artikulo 240 ng Labor Code; certain Dolores Villar as your authorized representative without the official
sanction of the mother Federation- PAFLU.
4. MAGHAIN KAAGAD ang aming Unyong nagsasarili, sa pamumuno ng
aming pangsamantalang Opisyal na kinatawan, si Ginang DOLORES 3. Maligning, libelling and slandering the incumbent officers of the union as
VILLAR, ng Petition for Certification Election sa Department of Labor, para well as of the PAFLU Federation.
kilalanin ang aming Unyong nagsasarili bilang Tanging kinatawan ng mga
manggagawa sa sama-samang pakikipagkasundo (CBA); 4. By spreading false propaganda among members of the Amigo Employees
Union-PAFLU that the incumbent union officers are 'merely appointees' of
5. BIGYAN ng kopya nito ang bawa't kinauukulan at ang mga kapasiyahang the management.
ito ay magkakabisa sa oras na matanggap ng mga kinauukulan ang kani-
kanilang sipi nito.1 5. By sowing divisiveness instead of togetherness among members of the
Amigo Employees Union-PAFLU.
Immediately thereafter or on February 9, 1977, petitioner Dolores Villar,
representing herself to be the authorized representative of the Amigo 6. By conduct unbecoming as members of the Amigo Employees Union-
Employees Union, filed a petition for certification election in the Company PAFLU which is highly prejudicial to the union as well as to the PAFLU
before Regional Office No. 4, with the Amigo Employees Union as the Federation.
petitioner. The Amigo Employees Union-PAFLU intervened and moved for
the dismissal of the petition for certification election filed by Dolores Villar, All these charges were formalized in a resolution of the incumbent officers
citing as grounds therefor, viz: (a) the petition lacked the mandatory requisite of the Amigo Employees Union-PAFLU dated February 14, 1977. 3
of at least 30% of the employees in the bargaining unit; (2) Dolores Villar
had no legal personality to sign the petition since she was not an officer of Not recognizing PAFLU's jurisdiction over their case, petitioners again
the union nor is there factual or legal basis for her claim that she was the refused to participate in the investigation rescheduled and conducted on
authorized representative of the local union; (3) there was a pending case for March 9, 1979. Instead, petitioners merely appeared to file their Answer to
the same subject matter filed by the same individuals; (4) the petition was the charges and moved for a dismissal.
4, docketed as RO4-Case No. RD-4-4088-77-T, praying that after due notice
Petitioners contend in their Answer that neither the disaffiliation of the and hearing, "(1) A preliminary injunction be issued forthwith to restrain the
Amigo Employees Union from PAFLU nor the act of filing the petition for respondents from doing the act herein complained of, namely: the dismissal
certification election constitute disloyalty as these are in the exercise of their of the individual complainants from their employment; (2) After due hearing
constitutional right to self-organization. They further contended that PAFLU on the merits of the case, an Order be entered denying and/or setting aside
was without jurisdiction to investigate their case since the charges, being the Decision dated March 15, 1977 and the Resolution dated March 28, 1977,
intra-union problems within the Amigo Employees Union-PAFLU, should issued by respondent Onofre P. Guevara, National President of respondent
be conducted pursuant to the provisions of Article XI, Sections 2, 3, 4 and 5 PAFLU; (3) The Appeal of the individual complainants to the General
of the local union's constitution and by-laws. Membership of the complainant AMIGO EMPLOYEES UNION, dated
March 22, 1977, pursuant to Sections 2, 3, 4 & 5, Article XI in relation of
The complainants, all of whom were the then incumbent officers of the Section 1, Article XII of the Union Constitution and By-Laws, be given due
Amigo Employees Union-PAFLU, however, appeared and adduced their course; and (4) Thereafter, the said preliminary injunction be made
evidence supporting the charges against herein petitioners. permanent, with costs, and with such further orders/reliefs that are just and
equitable in the premises."9
Based on the findings and recommendations of the PAFLU trial committee,
the PAFLU President, on March 15, 1977, rendered a decision finding the In these two cases filed before the Regional Office No. 4, the parties adopted
petitioners guilty of the charges and disposing in the last paragraph thereof, their previous positions when they were still arguing before the PAFLU trial
to wit, committee.

Excepting Felipe Manlapao, the expulsion from the AMIGO EMPLOYEES On October 14, 1977, Vicente Leogardo, Jr., Officer-in-Charge of Regional
UNION of all the other nine (9) respondents, Dionisio Ramos, Recitation Office No. 4, rendered a decision jointly resolving said two cases, the
Bernus, Dolores Villar, Romeo Dequito, Rolando de Guzman, Anselma dispositive portion of which states, to wit,
Andan, Rita Llagas, Benigno Mamaradlo and Orlando Acosta is hereby
ordered, and as a consequence the Management of the employer, AMIGO IN VIEW OF THE FOREGOING, judgment is hereby rendered granting the
MANUFACTURING, INC. is hereby requested to terminate them from their application of the Amigo Manufacturing, Inc., for clearance to terminate the
employment in conformity with the security clause in the collective employment of Dolores D. Villar, Dionisio Ramos, Benigno Mamaraldo,
bargaining agreement. Further, the Trial Committee is directed to investigate Orlando Acosta, Recitacion Bernus, Anselma Andan, Rolando de Guzman,
Felipe Manlapao when he shall have reported back for duty. 4 and Rita Llagas. The application of oppositors, under RO4-Case No. RD-4-
4088-77, for a preliminary injunction to restrain the Amigo Manufacturing,
Petitioners appealed the Decision to the PAFLU, citing the same grounds as Inc. from terminating their employment and from placing them under
before, and in addition thereto, argued that the PAFLU decision cannot preventive suspension, is hereby DISMISSED. 10
legally invoke a CBA which was unratified, not certified, and entered into
without authority from the union general membership, in asking the Not satisfied with the decision, petitioners appealed to the Office of the
Company to terminate them from their employment. The appeal was, Secretary of Labor. By Order dated February 15, 1979, the respondent
likewise, denied by PAFLU in a Resolution dated March 28, 1977. Amado G. Inciong, Deputy Minister of Labor, dismissed their appeal for lack
of merit. 11
After denying petitioner's appeal, PAFLU on March 28, 1977 sent a letter to
the Company stating, to wit, Hence, the instant petition for review, raising the following issues:

We are furnishing you a copy of our Resolution on the Appeal of the A. Is it not error in both constitutional and statutory law by the respondent
respondent in Administrative Case No. 2, Series of 1977, Amigo Employees Minister when he affirmed the decision of the RO4-Officer-in-Charge
Union-PAFLU vs. Dionisio Ramos, et al. allowing the preventive suspension and subsequent dismissal of petitioners
by reason of the exercise of their right to freedom of association?
In view of the denial of their appeal and the Decision of March 15, 1977
having become final and executory we would appreciate full cooperation on B. Is it not error in law by the respondent Minister when he upheld the
your part by implementing the provision of our CBA on security clause by decision of the RO4 OIC which sustained the availment of the respondent
terminating the respondents concerned from their employment.5 PAFLU's constitution over that of the local union constitution in the
settlement of intra-union dispute?
This was followed by another letter from PAFLU to the Company dated April
25, 1977, reiterating the demand to terminate the employment of the C. Is it not error in law amounting to grave abuse of discretion by the Minister
petitioners pursuant to the security clause of the CBA, with a statement in affirming the conclusion made by the RO4 OIC, upholding the legal
absolving the Company from any liability or damage that may arise from applicability of the security clause of a CBA over alleged offenses committed
petitioner's termination. earlier than its conclusion, and within the 60-day freedom period of an old
CBA? 12
Acting on PAFLU's demand, the Company informed PAFLU that it will first
secure the necessary clearances to terminate petitioners. By letter dated April The main thrust of the petition is the alleged illegality of the dismiss of the
28, 1977, PAFLU requested the Company to put petitioners under preventive petitioners by private respondent Company upon demand of PAFLU which
suspension pending the application for said clearances to terminate the invoked the security clause of the collective bargaining agreement between
petitioners, upon a declaration that petitioners' continued stay within the the Company and the local union, Amigo Employees Union-PAFLU.
work premises will "result in the threat to the life and limb of the other Petitioners contend that the respondent Deputy Minister acted in grave abuse
employees of the company."6 of discretion when he affirmed the decision granting the clearance to
terminate the petitioners and dismissed petitioners' complaint, and in support
Hence, on April 29, 1977, the Company filed the request for clearance to thereof, allege that their constitutional right to self-organization had been
terminate the petitioners before the Department of Labor, Regional Office impaired. Petitioner's contention lacks merit.
No. 4. The application, docketed as RO4-Case No. 7-IV-3549-T, stated as
cause therefor, "Demand by the Union Pursuant to the Union Security It is true that disaffiliation from a labor union is not open to legal objection.
Clause," and further, as effectivity date, "Termination-upon issuance of It is implicit in the freedom of association ordained by the Constitution. 13
clearance; Suspension-upon receipt of notice of workers concerned." 7 But this Court has laid down the ruling that a closed shop is a valid form of
Petitioners were then informed by memorandum dated April 29, 1977 that union security, and such provision in a collective bargaining agreement is not
the Company has applied for clearance to terminate them upon demand of a restriction of the right of freedom of association guaranteed by the
PAFLU, and that each of them were placed under preventive suspension Constitution. 14
pending the resolution of the said applications. The security guard was,
likewise, notified to refuse petitioners entry into the work premises. 8 In the case at bar, it appears as an undisputed fact that on February 15, 1977,
the Company and the Amigo Employees Union-PAFLU entered into a
In an earlier development, on April 25, 1977, or five days before petitioners Collective Bargaining Agreement with a union security clause provided for
were placed under preventive suspension, they filed a complaint with in Article XII thereof which is a reiteration of the same clause in the old CBA.
application for preliminary injunction before the same Regional Office No. The quoted stipulation for closed-shop is clear and unequivocal and it leaves
no room for doubt that the employer is bound, under the collective bargaining the list of members of the Amigo Employees Union-PAFLU, and thereafter,
agreement, to dismiss the employees, herein petitioners, for non- union recommended to the Amigo Manufacturing, Inc.; the termination of the
membership. Petitioners became non-union members upon their expulsion employment of the oppositors. 15
from the general membership of the Amigo Employees Union-PAFLU on
March 15, 1977 pursuant to the Decision of the PAFLU national president. We see no reason to disturb the same.

We reject petitioners' theory that their expulsion was not valid upon the The contention of petitioners that the charges against them being intra-union
grounds adverted to earlier in this Decision. That PAFLU had the authority problems, should have been investigated in accordance with the constitution
to investigate petitioners on the charges filed by their co-employees in the and by-laws of the Amigo Employees Union-PAFLU and not of the PAFLU,
local union and after finding them guilty as charged, to expel them from the is not impressed with merit. It is true that under the Implementing Rules and
roll of membership of the Amigo Employees Union-PAFLU is clear under Regulations of the Labor Code, in case of intra-union disputes, redress must
the constitution of the PAFLU to which the local union was affiliated. And first be sought within the organization itself in accordance with its
pursuant to the security clause of the new CBA, reiterating the same clause constitution and by-laws. However, it has been held that this requirement is
in the old CBA, PAFLU was justified in applying said security clause. We not absolute but yields to exception under varying circumstances. Thus, in
find no abuse of discretion on the part of the OIC of Regional Office No. 4 Kapisanan ng mga Manggagawa sa MRR vs. Hernandez, 20 SCRA 109, We
in upholding the validity of the expulsion and on the part of the respondent held:
Deputy Minister of Labor in sustaining the same. We agree with the OIC's
decision, pertinent portion of which reads: In the case at bar, noteworthy is the fact that the complaint was filed against
the union and its incumbent officers, some of whom were members of the
Stripped of non-essentials, the basic and fundamental issue in this case tapers board of directors. The constitution and by-laws of the union provide that
down to the determination of WHETHER OR NOT PAFLU HAD THE charges for any violations thereof shall be filed before the said board. But as
AUTHORITY TO INVESTIGATE OPPOSITORS AND, THEREAFTER, explained by the lower court, if the complainants had done so the board of
EXPEL THEM FROM THE ROLL OF MEMBERSHIP OF THE AMIGO directors would in effect be acting as respondent investigator and judge at the
EMPLOYEES UNION-PAFLU. same time. To follow the procedure indicated would be a farce under the
circumstances, where exhaustion of remedies within the union itself would
Recognized and salutary is the principle that when a labor union affiliates practically amount to a denial of justice or would be illusory or vain, it will
with a mother union, it becomes bound by the laws and regulations of the not be insisted upon, particularly where property rights of the members are
parent organization. Thus, the Honorable Secretary of Labor, in the case of involved, as a condition to the right to invoke the aid of a court.
Amador Bolivar, et al. vs. PAFLU, et al., NLRC Case No. LR-133 & MC-
476, promulgated on December 3, 1973, declared- The facts of the instant petition stand on all fours with the aforecited case that
the principle therein enunciated applies here as well. In the case at bar, the
When a labor union affiliates with a parent organization or mother union, or petitioners were charged by the officers of the Amigo Employees Union-
accepts a charter from a superior body, it becomes subject to the laws of the PAFLU themselves who were also members of the Board of Directors of the
superior body under whose authority the local union functions. The Amigo Employees Union-PAFLU. Thus, were the petitioners to be charged
constitution, by-laws and rules of the parent body, together with the charter and investigated according to the local union's constitution, they would have
it issues pursuant thereto to the subordinate union, constitute an enforceable been tried by a trial committee of three (3) elected from among the members
contract between the parent body and the subordinate union, and between the of the Board who are themselves the accusers. (Section 2, Article 11,
members of the subordinate union inter se. (Citing Labor Unions, Dangel and Constitution of the Local Union). Petitioners would be in a far worse position
Shriber, pp. 279-280). had this procedure been followed. Nonetheless, petitioners admit in their
petition that two (2) of the six (6) charges, i.e. disaffiliation and filing a
It is undisputable that oppositors were members of the Amigo Employees petition for certification election, are not intra-union matters and, therefore,
Union at the time that said union affiliated with PAFLU; hence, under the are cognizable by PAFLU.
afore-quoted principle, oppositors are bound by the laws and regulations of
PAFLU. Petitioners insist that their disaffiliation from PAFLU and filing a petition for
certification election are not acts of disloyalty but an exercise of their right
Likewise, it is undeniable that in the investigation of the charges against to self-organization. They contend that these acts were done within the 60-
them, oppositors were accorded 'due process', because in this jurisdiction, the day freedom period when questions of representation may freely be raised.
doctrine is deeply entrenched that the term 'due process' simply means that Under the peculiar facts of the case, We find petitioners' insistence untenable.
the parties were given the opportunity to be heard. In the instant case, ample
and unmistakable evidence exists to show that the oppositors were afforded In the first place, had petitioners merely disaffiliated from the. Amigo
the opportunity to present their evidence, but they themselves disdained or Employees Union-PAFLU, there could be no legal objections thereto for it
spurned the said opportunity given to them. was their right to do so. But what petitioners did by the very clear terms of
their "Sama-Samang Kapasiyahan" was to disaffiliate the Amigo Employees
PAFLU, therefore, correctly and legally acted when, pursuant to its Union-PAFLU from PAFLU, an act which they could not have done with
Constitution and By-Laws, it conducted and proceeded with the investigation any effective consequence because they constituted the minority in the
of the charges against the oppositors and found them guilty of acts prejudicial Amigo Employees Union-PAFLU.
and inimical to the interests of the Amigo Employees Union- PAFLU, to wit:
that of falsely and maliciously slandering the officers of the union; spreading Extant from the records is the fact that petitioners numbering ten (10), were
false propaganda among the members of the Amigo Employees Union- among the ninety-six (96) who signed the "Sama-Samang Kapasiyahan"
PAFLU; calling the incumbent officers as mere appointees and robots of whereas there are two hundred thirty four (234) union members in the Amigo
management; calling the union company-dominated or assisted union; Employees Union-PAFLU. Hence, petitioners constituted a small minority
committing acts unbecoming of the members of the union and destructive of for which reason they could not have successfully disaffiliated the local
the union and its members. union from PAFLU. Since only 96 wanted disaffiliation, it can be inferred
that the majority wanted the union to remain an affiliate of PAFLU and this
Inherent in every labor union, or any organization for that matter, is the right is not denied or disputed by petitioners. The action of the majority must,
of self-preservation. When members of a labor union, therefore, sow the therefore, prevail over that of the minority members. 16
seeds of dissension and strife within the union; when they seek the
disintegration and destruction of the very union to which they belong, they Neither is there merit to petitioners' contention that they had the right to
thereby forfeit their rights to remain as members of the union which they seek present representation issues within the 60-day freedom period. It is true, as
to destroy. Prudence and equity, as well as the dictates of law and justice, contended by petitioners, that under Article 257 of the Labor Code and
therefore, compelling mandate the adoption by the labor union of such Section 3, Rule 2, Book 2 of its Implementing Rules, questions of exclusive
corrective and remedial measures in keeping with its laws and regulations, bargaining representation are entertainable within the sixty (60) days prior to
for its preservation and continued existence; lest by its folly and inaction, the the expiry date of an existing CBA, and that they did file a petition for
labor union crumble and fall. certification election within that period. But the petition was filed in the name
of the Amigo Employees Union which had not disaffiliated from PAFLU,
Correctly and legally, therefore, the PAFLU acted when, after proper the mother union. Petitioners being a mere minority of the local union may
investigation and finding of guilt, it decided to remove the oppositors from not bind the majority members of the local union.
does not constitute an unfair labor practice exclusively cognizable by the
Moreover, the Amigo Employees Union, as an independent union, is not duly Court of Industrial Relations. (Seno vs. Mendoza, 21 SCRA 1124).
registered as such with the Bureau of Labor Relations. The appealed decision
of OIC Leogardo of Regional Office No. 4 states as a fact that there is no Finally, We reject petitioners' contention that respondent Minister committed
record in the Bureau of Labor Relations that the Amigo Employees Union error in law amounting to grave abuse of discretion when he affirmed the
(Independent) is registered, and this is not disputed by petitioners, conclusion made by the RO4 OIC, upholding the legal applicability of the
notwithstanding their allegation that the Amigo Employees Union is a duly security clause of a CBA over alleged offenses committed earlier than its
registered labor organization bearing Ministry of Labor Registration conclusion and within the 60-day freedom period of an old CBA. In the first
Certification No. 5290-IP dated March 27, 1967. But the independent union place, as We stated earlier, the security clause of the new CBA is a
organized after the "Sama-Samang Kapasiyahan" executed February 7, 1977 reproduction or reiteration of the same clause in the old CBA. While
could not have been registered earlier, much less March 27, 1967 under petitioners were charged for alleged commission of acts of disloyalty
Registration Certificate No. 5290-IP. As such unregistered union, it acquires inimical to the interests of the Amigo Employees Union-PAFLU in the
no legal personality and is not entitled to the rights and privileges granted by Resolution of February 14, 1977 of the Amigo Employees Union- PAFLU
law to legitimate labor organizations upon issuance of the certificate of and on February 15, 1977 PAFLU and the Company entered into and
registration. Article 234 of the New Labor Code specifically provides: concluded a new collective bargaining agreement, petitioners may not escape
the effects of the security clause under either the old CBA or the new CBA
Art. 234. Requirements of Registration.—Any applicant labor organization, by claiming that the old CBA had expired and that the new CBA cannot be
association, or group of unions or workers shall acquire legal personality and given retroactive enforcement. To do so would be to create a gap during
shall be entitled to the rights and privileges granted by law to legitimate labor which no agreement would govern, from the time the old contract expired to
organizations upon issuance of the certificate of registration. .... the time a new agreement shall have been entered into with the union. As this
Court said in Seno vs. Mendoza, 21 SCRA 1124, "without any agreement to
In Phil. Association of Free Labor Unions vs. Sec. of Labor, 27 SCRA 40, govern the relations between labor and management in the interim, the
We had occasion to interpret Section 23 of R.A. No. 875 (Industrial Peace situation would well be productive of confusion and result in breaches of the
Act) requiring of labor unions registration by the Department of Labor in law by either party. "
order to qualify as "legitimate labor organization," and We said:
The case of Seno vs. Mendoza, 21 SCRA 1124 mentioned previously needs
The theory to the effect that Section 23 of Republic Act No. 875 unduly further citation of the facts and the opinion of the Court, speaking through
curtails the freedom of assembly and association guaranteed in the Bill of Justice Makalintal who later became Chief Justice, and We quote:
Rights is devoid of factual basis. The registration prescribed in paragraph (b)
of said section 17 is not a limitation to the right of assembly or association, It appears that petitioners other than Januario T. Seno who is their counsel,
which may be exercised with or without said registration. The latter is merely were members of the United Seamen's Union of the Philippines. Pursuant to
a condition sine qua non for the acquisition of legal personality by labor a letter-request of the Union stating that they 'had ceased to be members in
organizations, associations or unions and the possession of the 'rights and good standing' and citing a closed shop clause in its bargaining agreement
privileges granted by law to legitimate labor organizations.' The Constitution with respondent Carlos A. Go Thong & Co., the latter dismissed said
does not guarantee these rights and privileges, much less said personality, petitioners. Through counsel, petitioners requested that they be reinstated to
which are mere statutory creations, for the possession and exercise of which their former positions and paid their backwages, otherwise they would picket
registration is required to protect both labor and the public against abuses, respondents' offices and vessels. The request was denied on the ground that
fraud, or impostors who pose as organizers, although not truly accredited the dismissal was unavoidable under the terms of the collective bargaining
agents of the union they purport to represent. Such requirement is a valid agreement. ...
exercise of the police power, because the activities in which labor
organizations, associations and union or workers are engaged affect public We, therefore, hold and rule that petitioners, although entitled to disaffiliate
interest, which should be protected. from their union and form a new organization of their own, must, however,
suffer the consequences of their separation from the union under the security
Simply put, the Amigo Employees Union (Independent) Which petitioners clause of the CBA.
claim to represent, not being a legitimate labor organization, may not validly
present representation issues. Therefore, the act of petitioners cannot be WHEREFORE, IN VIEW OF ALL THE FOREGOING, the Order appealed
considered a legitimate exercise of their right to self-organization. Hence, from affirming the joint decision of the OIC of Regional Office No. 4 in RO4-
We affirm and reiterate the rationale explained in Phil Association of Free Case No. T-IV-3549-T and RO4 Case No. RD-4-4088-77-T granting
Labor Unions vs. Sec. of Labor case, supra, in order to protect legitimate clearance to terminate petitioners as well as dismissing their complaint with
labor and at the same time maintain discipline and responsibility within its application for preliminary injunction, is hereby AFFIRMED. No costs.
ranks.
SO ORDERED.
The contention of petitioners that the new CBA concluded between Amigo
Employees Union-PAFLU and the Company on February 15, 1977
containing the union security clause cannot be invoked as against the
petitioners for offenses committed earlier than its conclusion, deserves scant
consideration. We find it to be the fact that the union security clause provided
in the new CBA merely reproduced the union security clause provided in the
old CBA about to expire. And since petitioners were expelled from Amigo
Employees Union-PAFLU on March 28, 1982 upon denial of their Motion
for Reconsideration of the decision expelling them, the CBA of February 15,
1977 was already applicable to their case. The "closed-shop provision" in the
CBA provides:

All members of the UNION as of the signing of this Agreement shall remain
members thereof in good standing. Therefore, any members who shall resign,
be expelled, or shall in any manner cease to be a member of the UNION,
shall be dismissed from his employment upon written request of the UNION
to the Company. (Art. III)

A closed-shop is a valid form of union security, and a provision therefor in a


collective bargaining agreement is not a restriction of the right of freedom of
association guaranteed by the Constitution. (Manalang, et al. vs. Artex
Development Co., Inc., et al., L-20432, October 30, 1967, 21 SCRA 561).
Where in a closed-shop agreement it is stipulated that union members who
cease to be in good standing shall immediately be dismissed, such dismissal
UNITED POLYRESINS, INC. vs. PINUELA On January 7, 2008, respondent filed a complaint before the National
G.R. No. 209555 July 31, 2017 Conciliation and Mediation Board (NCMB), claiming that petitioners refused
DEL CASTILLO, J.: to bargain collectively. During the scheduled conferences before the NCMB,
petitioners raised the issue of non-payment of the ₱300,000.00 owing to UPI
This Petition for Review on Certiorari1 assails the December 11, 2012 and insisted on its payment; they also threatened to deduct the amount of
Decision2 and October 10, 2013 Resolution3 of the Court of Appeals (CA) ₱l,500.00 from the respective salaries of the union members.11
in CA-G.R: SP No. 115402 which set aside the June 11, 2011 Decision4 of
the National Labor Relations Commission (NLRC) in NLRC-LAC Case No. Because of the recurring threat of failed CBA negotiations and salary
06- 001577-09. deductions as means of recovering the ₱300,000.00 loaned to the union,
union members began to demand the holding of a special election of union
Factual Antecedents officers. They likewise accused respondent and the other union officers of
mismanagement, unduly hanging on to their positions, and lack of
Petitioner United Polyresins, Inc. (UPI) is a registered domestic corporation accountability.12
doing business in San Pedro, Laguna, while petitioners Ernesto Uy Soon, Jr.
and Julito Uy Soon are its corporate officers. Thus, in March 2008, special elections were held, and a new union President
and set of officers were elected.13
Respondent Marcelino Pinuela was employed by UPI in 1987. He became a
member of the labor union, Polyresins Rank and File Association (PORFA), On March 29, 2008, the union's new set of officers conducted an
and was elected President thereof in May, 2005 and slated to serve until the investigation into the fact that the union had little or no funds remaining in
end of 2007. its bank account. Respondent attended the investigation, and admitted that
the union had no more funds as they were "utilized in the prosecution of cases
The collective bargaining agreement (CBA) then existing between UPI and during his incumbency."14 He likewise failed to make a formal turnover of
PORFA provided that: documents to the new President. Respondent was required to surrender union
documents in his possession on the next scheduled meeting.15
Section 3. The Company shall grant to the Union the amount of Three
Hundred Thousand Pesos (₱300,000.00) free of interest as the union's capital On April 8, 2008, another inquiry was held where respondent was present.
for establishing a cooperative to meet the needs of its members. Said loan The investigation centered on respondent's continued failure to account for
shall fall due and become payable at the same date that this Bargaining the union's bank accounts, documents, and deposits made during his
Agreement expires, to wit - December 31, 2007. In the event of non-payment, incumbency, and his failure to formally turn over union's papers to the new
all officers and members will be personally accountable. In case of additional officers. After the meeting, respondent and the new officers proceeded to the
funds, they can make a written request [addressed] to the President of the bank, where they discovered that the PORFA account had already been
company.5 closed.16

The CBA likewise contained a union security clause which provided that On April 10, 2008, the new set of union officers issued a Resolution17
employees who cease to be PORFA members in good standing by reason of expelling respondent from PORF A for being guilty of the following
resignation or expulsion shall not be retained in the employ of UPI. violations:

Upon his assumption as union President, respondent wrote the former union 1. No annual financial statement.
President, Geoffrey Cielo (Cielo), to turn over the records, papers, documents
and financial statements of the union. Cielo surrendered the union's bank 2. No listings or ledger of union member's [sic] emergency loans.
account documents, among others, which indicated that the union had an
available ₱78,723.60 cash balance. Cielo likewise submitted a Financial 3. Unposted cheques on the Union's passbook collected from umon members
Report indicating that the union had ₱208,623.60 in cash and ₱l59,500.00 in [sic] monthly dues.
receivables.
4. Our union checking account at Security Bank were [sic] Zero
Finding that the bank documents and Cielo's report did not match, and Cielo balance/closed account.
unable to explain the discrepancies, the union's Executive Committee, which
was headed by respondent, resolved to hire a certified public accountant to 5. No receipts/cash disbursement presented for the union operational [sic]
conduct an audit of the union's finances. In a December 1, 2005 report, the expenses.
accountant concluded that the union's finances, income, and disbursements
for the years 2003 and 2004 were not properly documented, recorded, and 6. Unable to return the ₱300,000.00 lent by the management free of interest.
reported. He recommended that the union officers "take a seminar on basic (Art. XXVII, Section 3 of our CBA).
bookkeeping and accounting;"6 that the union adopt and/or install the
necessary accounting and internal control systems; that the union prepare the 7. Unable to explain and present documents to support where the agency fees
proper financial statements; and that the officers take corrective measures in and union dues collected from legitimate union members were used.18
financial management as an integral part of sound management.7
The officers held that these violations constituted an infringement of the
Meanwhile, during respondent's term as PORF A President, it appeared that union's Constitution, particularly Article XV, Section 1, paragraphs (e) and
UPI automatically deducted from the respective salaries of PORFA members (f) thereof, which specifically prohibit the misappropriation of union funds
amounts representing union membership dues and loan payments. These and property and give ground for the impeachment and recall of union
amounts, which totalled ₱2,402,533.43, were then regularly turned over by officers.19
UPI to PORFA in the form of fifty eight (58) crossed checks, made payable
to PORFA.8 These amounts were then deposited and credited to PORFA's In an April 11, 2008 letter20 to petitioners, PORF A communicated
account.9 respondent's expulsion from the union.

On December 8, 2007, or several days before the ₱300,000.00 loan by UPI On April 14, 2008, petitioners issued a letter of termination21 to respondent,
to PORFA became due, petitioners, respondent, and the other union officers to take effect immediately.
met to discuss the proposed new CBA. Thereat, petitioners told respondent
that until the ₱300,000.00 is returned, the former shall not discuss the Ruling of the Labor Arbiter
proposed CBA. Respondent explained that the union did not have the
finances and had only ₱78,723.60, which was the original amount turned Respondent filed a complaint against petitioners before the Labor Arbiter for
over by Cielo to respondent when the latter assumed office as union illegal dismissal, with monetary claims and damages, which was docketed as
President. Petitioners then told respondent and the other union officers that NLRC Case No. RAB-IV-08-27303-08-L. He claimed that his dismissal was
if the amount is not returned, the same will be deducted from the salaries of effected in bad faith and without due process and was thus illegal. Petitioners
the union members.10 countered that respondent's dismissal is valid under the union security clause
of the CBA; that his failure to return the ₱300,000.00 loan to the union due
to mismanagement/misappropriation constitutes just cause for his expulsion
from the union, as well as dismissal from employment; that he was accorded had custody of ₱78,723.60 as union funds as of June 3, 2005, still he could
substantive and procedural due process; that the herein individual petitioners not account the whereabouts of the said money. As a signatory to the said
may not be held liable for respondent's claims; and that accordingly, the case account, complainant cannot be considered as entirely faultless since he was
should be dismissed. grossly negligent in the custody of the funds. There is substantial basis in
complainant's dismissal thus, the award of backwages and 13th month pay
On April 20, 2009, the Labor Arbiter issued a Decision22 dismissing should be deleted. However, even if We find complainant's dismissal to be
respondent's complaint on the finding that respondent was not illegally valid, there is equally no evidence showing that he pocketed the missing
terminated, thus: funds of the union. In this regard since he had rendered a considerable
number of years in the service (21 years) complainant may be awarded
While complainant, as then Union President, denies any misappropriation of separation pay at the rate of 'ii month salary for every year of service (396 x
union funds, it is undisputed that he failed to account for the missing union 13 x 21 years) from the inception of his employment till his dismissal in the
funds and to return the ₱300,000.00 which the respondent company had lent interest of justice and compassion since his infraction did not involve serious
for the union's assistance upon the expiration of the CBA dated December misconduct.
31, 2007.
Further, We also hold that while complainant's dismissal was valid pursuant
More importantly, in the investigation conducted by the newly elected to the enforcement of the Union Security Clause, respondents however did
officers of the union, it was uncovered that union funds were in fact not comply with the requisite procedural due process. As held in the case of
personally used by the former officers of PORF A which includes Agabon vs. NLRC, x x x the Supreme Court held that where the dismissal is
complainant. for a cause recognized by the prevailing jurisprudence, the absence of the
statutory due process should not nullify the dismissal or render it illegal x x
Thus, the union passed a resolution expelling complainant from the PORFA x. Accordingly, for violating complainant's statutory rights, respondents
union and the corresponding letter was sent to the respondent company should indemnify him the amount of ₱30,000.00 as nominal damages in
informing the latter of complainant's expulsion coupled with a addition to his separation pay.
recommendation that complainant be terminated from employment pursuant
to the union security clause of the CBA. WHEREFORE, premises considered, respondents-appellees' Motion for
Reconsideration is GRANTED, a new Decision is rendered finding
Given the foregoing, we rule that complainant was validly dismissed since complainant's dismissal as valid. Respondents-appellees are however
the respondent company merely did its obligation under the CBA by ordered to pay complainant the amounts of ₱108,108.00 and ₱30,000.00 as
terminating the services of complainant who ceased to be a member in good separation pay and nominal damages.
standing of the PORF A union by reason of expulsion.
All other claims whether monetary or otherwise are hereby DISMISSED.
WHEREFORE, premises considered, judgment is hereby rendered
DISMISSING the instant complaint for lack of merit. SO ORDERED.26

SO ORDERED.23 Ruling of the Court of Appeals

Ruling of the National Labor Relations Commission In a Petition for Certiorari27 before the CA and docketed as CA-G.R. SP No.
115402, respondent sought to reverse the above NLRC Decision and
Respondent appealed before the NLRC, which initially overturned the Labor reinstate its December 8, 2009 Decision, arguing that the Commission
Arbiter in a December 8, 2009 Decision,24 which decreed as follows: gravely erred in concluding that he was personally accountable for the
missing funds, the closing of PORFA's bank account, and that he was grossly
WHEREFORE, the assailed Decision is hereby SET ASIDE and a NEW one negligent in the custody of the union funds. In their Comment,28 petitioners
is entered declaring the complainant-appellant's dismissal to be illegal. countered that respondent's dismissal was attended by due process; that he is
Respondents Union [sic] and respondent company are hereby declared guilty of the infractions for which he was dismissed; and that his guilt had
jointly and severally liable to pay complainant his full backwages from the been proved by substantial evidence.
date he was dismissed until date instant [sic] and to pay his separation pay
equivalent to one month salary per year of service computed as follows: On December 11, 2012, the CA issued the assailed Decision containing the
following pronouncement:
BACKWAGES
04/14/08 - 10/14/09 Petitioner insists that he is innocent of the charges against him made by the
₱396 x 26 days x 18 mos. PORF A (the union), particularly the embezzlement of the union funds. He
₱l0,296.00 x 18 days = ₱l85,328.00 vehemently denied misappropriation of the same and that the PORFA Union
SEPARATION PAY officers conspired with the Respondents in removing him as a member in
₱396.00 x 26 x 22yrs. good standing of the said union and his subsequent dismissal as employee
₱l0,296 x 22yrs. = ₱226,512.00 pursuant to the CBA's union security clause.
13th Month Pay
₱185,328.00 I 12 = ₱15,444.00 Respondents on the other hand, denied the Petitioner's allegation of
Grand Total ₱427,284.00 conspiracy and that in fact, there was a series of conferences conducted
SO ORDERED.25 jointly by the management and the union on the matter of lost union funds
and that the Petitioner was made aware of the charges against him before he
However, on motion for reconsideration, the NLRC issued its June 11, 2011 was terminated. They claim that the management participated in the
Decision, which held as follows: investigations and that it was shown that even if the Petitioner as president
of the union did not misappropriate the funds nevertheless he committed
What cannot escape from [sic] our attention and consideration are the omission/gross negligence for which reason he was expelled therefrom. The
following: (1) there was an obligation x x x to return the amount of Respondents also claim that Petitioner was accorded procedural due process
₱300,000.00 to the respondent upon termination of the CBA on December during the investigations.
31, 2007, (2) complainant, as the President of the Union at the time the loan
was due and demandable, failed to account for said funds, and under the same It is basic in labor jurisprudence that the burden of proof rests upon
provision, was to be held personally accountable, (3) Pinuela actually management to show that the dismissal of its worker was based on a just
participated x x x in the whole process of determining accountability over the cause. When an employer exercises its power to terminate an employee by
union funds, (4) denied knowledge over and receipt of the missing funds, enforcing the union security clause, it needs to determine and prove the
despite his being among those charged with its custody and safe-keep, as the following: (1) the union security clause is applicable; (2) the union is
Union President. requesting for the enforcement of the union security provision in the CBA;
and (3) there is sufficient evidence to support the decision of the union to
It is also to be noted that the complainant as union president, could not expel the employee from the union.
explain nor comment on the fact that their union's bank account is already a
closed account. Even if We assume and in fact complainant admitted that he
The dispute before Us does not raise any issue with respect to the first two III.
requisites; the issue being whether there was sufficient evidence to support
Petitioner's expulsion from PORFA. In arriving at any conclusion thereto, the THE APPELLATE COURT ERRED IN RULING THAT THE
Petitioner must first be accorded due process of law.x x x RESPONDENT IS ENTITLED TO SEPARATION PAY, BACKWAGES
FROM DISMISSAL TO THE FINALITY OF ITS DECISION, AND 13TH
xxxx MONTH PAY.33

On both questions of whether there exist[s] sufficient evidence to support Petitioners' Arguments
Petitioner's expulsion from the union (substantive due process), and whether
Petitioner was properly informed of the accusation against him and his Praying that the assailed CA dispositions be set aside and that respondent's
dismissal from employment (procedural due process), We answer in the case be dismissed instead, petitioners maintain in their Petition and Reply34
negative. that substantive and procedural due process were observed in respondent's
case; that respondent was apprised of the charges against him and given the
An examination of the submitted evidence before the Labor Arbiter show opportunity to refute them; that the evidence points to the conclusion that he
[sic] that the same are not enough to prove the alleged charges of misappropriated the union's funds and was unable to explain the dissipation
misappropriation against the Petitioner and neither was he properly informed thereof; that for what he has done, respondent violated Article XV, Section
thereof. 1, paragraphs (e) and (f) of the union's Constitution; that respondent's
dismissal on the basis of the union security clause in the CBA was thus valid,
xxxx based on substantial proof, and in accord with the pronouncement in Carino
v. National Labor Relations Commission,35 where the dismissal of an
On the other hand, the Petitioner have [sic] shown adequate explanation employee was upheld on the basis of the union security and expulsion clauses
about the funds of the union that came to his possession. The Memorandum contained in the CBA; and that since his dismissal is valid, then he is not
of Ramon M. Martinez, a Certified Public Accountant, show [sic] that he entitled to his monetary claims.
made an audit of the funds of the union during the previous administration
and that the actual funds the union had was merely ₱34,344.25 when Respondent's Arguments
Petitioner took over. This amount was not even shown to have been
misappropriated by the Petitioner. In his Comment,36 respondent maintains that the CA did not err in finding
that the evidence against him was insufficient; that the CA was correct in
Compounding this want of substantive evidence is the lack of procedural due ruling that his right to procedural due process was violated when he was not
process that Petitioner was entitled to. As [has] been previously discussed, properly informed of the charges against him; and that for these reasons, he
the Petitioner was not given the proper first notice. Thereafter, despite such was illegally dismissed and thus entitled to his monetary claims.
lack of first notice, on the mere letter of the union that he was expelled
therefrom because of alleged causes, the Petitioner was dismissed from Our Ruling
employment by the Respondents in the termination letter dated 14 April 2008
on the sole basis of union security clause. Such action cannot be The Court denies the Petition.
countenanced. In the same Inguillo case, the Supreme Court also ruled:
Respondent's expulsion from PORFA is grounded on Article XV, Section 1,
'Thus, as held in that case, 'the right of an employee to be informed of the paragraphs (e) and (f) of the union's Constitution, which provides:
charges against him and to reasonable opportunity to present his side in a
controversy with either the company or his own Union is not wiped away by ARTICLE-XV
a Union Security Clause or a Union Shop Clause in a collective bargaining IMPEACHMENT AND RECALL
agreement. An employee is entitled to be protected not only from a company
which disregards his rights but also from his own Union, the leadership of Section 1. Any of the following shall be ground for the impeachment or recall
which could yield to the temptation of swift and arbitrary expulsion from of the union officers.
membership and mere dismissal from his job.'
a. Committing or causing the commission directly or indirectly of acts against
In sum, the NLRC gravely abused its discretion in reconsidering its earlier the interest and welfare of the union;
Decision which is more in accord with the evidence on record.
b. Malicious attack against the union, its officers or against a fellow union
WHEREFORE, the petition is hereby GRANTED. The assailed Decision officer or member;
dated 11 June 201029 is hereby SET ASIDE. The Decision dated 8 December
2009 is REINSTATED with the MODIFICATION that the backwages shall c. Failure to comply with the obligation to tum over and return to union
be recomputed from the date of Petitioner's dismissal to the finality of this treasurer within three (3) days unexpanded [sic] sum of money received from
Decision. the money funds to answer for an authorized union purpose;

SO ORDERED.30 (Citations omitted) d. Gross misconduct unbecoming of a union officer;

Petitioners filed a Motion for Reconsideration,31 which was denied by the e. Misappropriation of union funds and property. This is without prejudice to
CA in its October 10, 2013 Resolution.1âwphi1 Hence, the instant Petition. the filing of an appropriate criminal or civil action against the responsible
officer/(s) by any interested party;
Issues
f. Willful violation of any provision of the constitution or rules, regulations,
In a June 22, 2015 Resolution,32 the Court resolved to give due course to the measures, resolution(s) and decision of the union.37 (Emphasis supplied)
Petition, which contains the following assignment of errors:
However, these provisions refer to impeachment and recall of union officers,
I. and not expulsion from union membership. This is made clear by Section
2(e) of the same Article XV, which provides that "(t)he union officers
THE APPELLATE COURT ERRED IN RULING THAT THE CHARGES impeached shall 'IPSO FACTO' to [sic] be considered resigned or ousted
OF MISAPPROPRIATION AGAINST THE RESPONDENT WERE from office and shall no longer be elected nor appointed to any position in
INSUFFICIENT (SUBSTANTIVE DUE PROCESS) the union." In short, any officer found guilty of violating these provisions
shall simply be removed, impeached or recalled, from office, but not expelled
II. or stripped of union membership.

THE APPELLATE COURT ERRED IN RULING THAT THE It was therefore error on the part of PORFA and petitioners to terminate
RESPONDENT WAS NOT PROPERLY INFORMED OF THE CHARGES respondent's employment based on Article XV, Section 1, paragraphs (e) and
AGAINST HIM (PROCEDURAL DUE PROCESS). (f) of the union's Constitution. Such a ground does not constitute just cause
for termination.
A review of the PORFA Constitution itself reveals that the only provision
authorizing removal from the union is found in Article X, Section 6, that is,
on the ground of failure to pay union dues, special assessments, fines, and
other mandatory charges.38 On the other hand, grounds for disqualification
from membership may be found in Article IV, which states that-

Section 3. The following are not eligible neither [sic] for membership nor to
election or appointment to any position in the union:

a. Subversive or persons who profess subversive ideas.

b. Persons who have been convicted of crime involving moral turpitude.

c. Persons who are not employees of the company.39

These provisions do not apply in respondent's case. Although he was


eventually charged with estafa,40 a crime involving moral turpitude,41 still,
he has not been convicted of the crime. For this reason, he may not be
disqualified as union member.

Thus, for what he is charged with, respondent may not be penalized with
expulsion from the union, since this is not authorized and provided for under
PORFA's Constitution.

Contrary to petitioners' claim, Carino v. National Labor Relations


Commission is not applicable here. In that case, the employee was terminated
on the basis of existing suspension and expulsion provisions contained in the
CBA and rules on discipline found in the union's Constitution. There are no
such provisions in PORFA's Constitution; neither has it been shown that
there are similar stipulations in the parties' CBA.

The matter of respondent's alleged failure to return petitioners' ₱300,000.00


which was lent to PORFA is immaterial as well. It may not be used as a
ground to terminate respondent's employment; under the Labor Code, such a
contribution by petitioners to PORFA is illegal and constitutes unfair labor
practice.

ART. 248. Unfair labor practices of employers. - It shall be unlawful for an


employer to commit any of the following unfair labor practice:

xxxx

(d) To initiate, dominate, assist or otherwise interfere with the formation or


administration of any labor organization, including the giving of financial or
other support to it or its organizers or supporters;42 (Emphasis supplied)

This could be an opportune time for the union to consider amending its
Constitution in order to provide for specific rules on the discipline of its
members, not just its officers. After all, it is given the right under the Labor
Code, "to prescribe its own rules with respect to the acquisition or retention
of membership."43 But it may not insist on expelling respondent from PORF
A and assist in his dismissal from UPI without just cause, since it is an unfair
labor practice for a labor organization to "cause or attempt to cause an
employer to discriminate against an employee, including discrimination
against an employee with respect to whom membership in such organization
has been denied or to terminate an employee on any ground other than the
usual terms and conditions under which membership or continuation of
membership is made available to other members."44

On account of the foregoing disquisition, the other issues raised by the parties
need not be discussed.

WHEREFORE, for the foregoing reasons, the Petition is hereby DENIED.


The December 11, 2012 Decision and October 10, 2013 Resolution of the
Court of Appeals in CA-G.R. SP No. 115402 are AFFIRMED.

SO ORDERED.
TANCINCO vs. FERRER-CALLEJA voters unless proven to be managerial employees. 10 Petitioners' motion for
G.R. No. 78131 January 20, 1988 reconsideration was likewise denied.
GANCAYCO, J.:
Dissatisfied with the turn of events narrated above petitioners elevated the
This special civil action for certiorari seeks to annul the Resolution of case to this Court by way of the instant petition for certiorari under Rule 65
February 12, 1987 and the Decision of December 10, 1986 of the Bureau of of the Rules of Court. Petitioners allege that public respondent director of
Labor Relations * in BLR Case No. A922186, setting aside the order of July Labor Relations committed grave abuse of discretion in ordering the Med-
25, 1986 which decreed the inclusion and counting of the 56 segregated votes Arbiter to disregard the 56 segregated votes and proclaim private respondents
for the determination of the results of the election of officers of Imperial as the duly elected officers of ITM-MEA whereas said respondent ruled that
Textile Mills Inc. Monthly Employees Association (ITM-MEA). the grounds relied upon by ANGLO for the exclusion of voters are arbitrary,
whimsical and without legal basis.
Private respondents are the prime organizers of ITM-MEA. While said
respondents were preparing to file a petition for direct certification of the The petition is impressed with merit. The record of the case shows that public
Union as the sole and exclusive bargaining agent of ITM's bargaining unit, respondent categorically declared as arbitrary, whimsical and without legal
the union's Vice-President, Carlos Dalmacio was promoted to the position of basis the grounds 11 relied upon by ANGLO in disenfranchising the 56
Department Head, thereby disqualifying him for union membership. Said voters in question. However, despite said finding public respondent ruled to
incident, among others led to a strike spearheaded by Lacanilao group, set aside the Resolution of July 25, 1986 of the Med-Arbiter based on its own
respondents herein. Another group however, led by herein petitioners staged findings 12 that 51 of the 56 disenfranchised voters were not yet union
a strike inside the company premises. After four (4) days the strike was members at the time of the election of union officers on May 26, 1986 on the
settled. On May 10, 1986 an agreement was entered into by the ground that their names do not appear in the records of the Union submitted
representatives of the management, Lacanilao group and the Tancinco group to the Labor Organization Division of the Bureau of Labor on April 24, 1986.
the relevant terms of which are as follows:
The finding does not have a leg to stand on. Submission of the employees
"1. That all monthly-paid employees shall be United under one union, the names with the BLR as qualified members of the union is not a condition
ITM Monthly Employees Association (ITM-MEA), to be affiliated with sine qua non to enable said members to vote in the election of union's officers.
ANGLO; It finds no support in fact and in law. Per public respondent's findings, the
April 24, 1986 list consists of 158 union members only 13 wherein 51 of the
2. That the management of ITM recognizes ANGLO as the sole and exclusive 56 challenged voters' names do not appear. Adopting however a rough
bargaining agent of all the monthly-paid employees; estimate of a total number of union members who cast their votes of some
333 14 and excluding therefrom the 56 challenged votes, if the list is to be
3. That an election of union officers shall be held on 26 May l986, from 8:00 the basis as to who the union members are then public respondent should
a.m. to 5:00 p.m.; have also disqualified some 175 of the 333 voters. It is true that under article
242(c) of the Labor Code, as amended, only members of the union can
4. That the last day of filing of candidacy shall be on l9 May l986 at 4:00 participate in the election of union officers. The question however of
p.m.; eligibility to vote may be determined through the use of the applicable payroll
period and employee's status during the applicable payroll period. The
5. That a final pre-election conference to finalize the list of qualified voters payroll of the month next preceding the labor dispute in case of regular
shall be held on 19 May 1986, at 5:00 p.m.;" 1 employees 15 and the payroll period at or near the peak of operations in case
of employees in seasonal industries. 16
On May 19, 1986, a pre-election conference was held, but the parties failed
to agree on the list of voters. During the May 21, 1986 pre-election In the case before Us, considering that none of the parties insisted on the use
conference attended by MOLE officers, ANGLO through its National of the payroll period-list as voting list and considering further that the 51
Secretary, a certain Mr. Cornelio A. Sy made a unilateral ruling excluding remaining employees were correctly ruled to be qualified for membership,
some 56 employees consisting of the Manila office employees, members of their act of joining the election by casting their votes on May 26, 1986 after
Iglesia ni Kristo, non-time card employees, drivers of Mrs. Salazar and the the May 10, 1986 agreement is a clear manifestation of their intention to join
cooperative employees of Mrs. Salazar. Prior to the holding of the election the union. They must therefore be considered ipso facto members thereof
of union officers petitioners, 2 through a letter addressed to the Election Said employees having exercised their right to unionism by joining ITM-
Supervisor, MOLE San Fernando Pampanga, protested said ruling but no MEA their decision is paramount. Their names could not have been included
action was taken. On May 26, 1986, the election of officers was conducted in the list of employee submitted on April 24, 1986 to the Bureau of Labor
under the supervision of MOLE wherein the 56 employees in question for the agreement to join the union was entered into only on May 10, 1986.
participated but whose votes were segregated without being counted. Indeed the election was supervised by the Department of Labor where said
Lacanilao's group won. Lacanilao garnered 119 votes with a margin of three 56 members were allowed to vote. Private respondents never challenged their
(3) votes over Tancinco prompting petitioners to make a protest. Thereafter, right to vote then.
petitioners filed a formal protest with the Ministry of Labor Regional Office
in San Fernando, Pampanga 3 claiming that the determination of the The Solicitor General in his manifestation agreed with petitioners that public
qualification of the 56 votes is beyond the competence of ANGLO. Private respondent committed a grave abuse of discretion in deciding the issue on
respondents maintain the contrary on the premise that definition of union's the basis of the records of membership of the union as of April 24, 1986 when
membership is solely within their jurisdiction. this issue was not put forward in the appeal.

On the basis of the position papers submitted by the parties MOLE's Med It is however the position of private respondents that since a collective
Arbiter 4 issued an order dated July 25, 1986 directing the opening and bargaining agreement (CBA) has been concluded between the local union
counting of the segregated votes. 5 From the said order private respondents and ITM management the determination of the legal question raised herein
appealed to the Bureau of Labor Relations (BLR) justifying the may not serve the purpose which the union envisions and may destroy the
disenfranchisement of the 56 votes. Private respondents categorized the cordial relations existing between the management and the union.
challenged voters into four groups namely, the Manila Employees, that they
are personal employees of Mr. Lee; the Iglesia ni Kristo, that allowing them We do not agree. Existence of a CBA and cordial relationship developed
to vote will be anomalous since it is their policy not to participate in any form between the union and the management should not be a justification to
of union activities; the non-time card employees, that they are managerial frustrate the decision of the union members as to who should properly
employees; and the employees of the cooperative as non-ITM employees. 6 represent them in the bargaining unit. Neither may the inclusion and counting
On December 10, 1986, BLR rendered a decision 7 holding the exclusion of of the 56 segregated votes serve to disturb the existing relationship with
the 56 employees as arbitrary, whimsical, and wanting in legal basis 8 but set management as feared by herein private respondents. Respondents
aside the challenged order of July 26, 1986 on the ground that 51 ** of 56 themselves pointed out that petitioners joined the negotiating panel in the
challenged voters were not yet union members at the time of the election per recently concluded CBA. This fact alone is conclusive against herein
April 24, 1986 list submitted before the Bureau. 9 The decision directed petitioners and hence will estop them later if ever, from questioning the CBA
among others the proclamation of Lacanilao's group as the duly elected which petitioners concurred with. Furthermore, the inclusion and counting of
officers and for ITM-MEA to absorb in the bargaining unit the challenged the 56 segregated votes would not necessarily mean success in favor of herein
petitioners as feared by private respondents herein. Otherwise, could this be
the very reason behind their fears why they made it a point to nullify said
votes?

WHEREFORE, premises considered, the petition for certiorari is


GRANTED. The temporary restraining order issued by this Court on May
13, 1987 is hereby made permanent. The questioned Resolution of February
12, 1987 and the Decision of December 10, 1986 are hereby set aside for
being null and void and the Order of July 25, 1986 of the Mediator Arbiter is
hereby declared immediately executory.

Cost against private respondents.

SO ORDERED.
KAPISANAN NG MANGGAGAWANG PINAGYAKAP (KMP) vs. be held accountable for the non-production of the books of accounts of the
TRAJANO Union for the years 1977, 1978 and 1979 because they were not the officers
G.R. No. L-62306 January 21, 1985 then and not one of the former officers of the Union had turned over to them
RELOVA, J.: the records in question. Further, they averred that the non-ratification of the
constitution and by-laws of the Union and the non-segregation of the Union
Petitioners seek to annul the resolution and order, dated August 13 and funds occurred before they became officers and that they have already been
October 19, 1982, respectively, of public respondent Director Cresenciano correcting the same.
B. Trajano of the Bureau of Labor Relations, Ministry of Labor and
Employment, in BLR Case No. A-0100-82 (RO4-A-LRD-M-9-35-81), On April 28, 1982, Med-Arbiter Antonio D. Cabibihan ordered the holding
entitled: "Catalino Silvestre, et al., vs. Kapisanan ng Manggagawang of a referendum, to be conducted under the supervision of the Bureau of
Pinagyakap (KMP) Labor Union and its Officers" affirming Med-Arbiter Labor Relations, to decide on the issue of whether to expel or suspend the
Antonio D. Cabibihan's order dated April 28, 1982, directing the said Union union officers from their respective positions.
to hold and conduct, pursuant to its constitution and by-laws and under the
supervision of the Bureau of Labor Relations, a general membership meeting, Petitioners appealed the said order of Med-Arbiter Cabibihan to herein public
to vote for or against the expulsion or suspension of the herein petitioner respondent Director Trajano of the Bureau of Labor Relations, Ministry of
union officers. Labor, Manila, claiming that the same is not in accordance with the facts
contained in the records and is contrary to law. They pointed out that the
Records show that on June 30, 1981 a written request for accounts disallowed expenditures of P1,278.00 were made in good faith and not used
examination of the financial status of the Kapisanan ng Manggagawang for the personal benefit of herein union officers but, instead, contributed to
Pinagyakap (KMP) Labor Union (Union for brevity), the existing labor union the benefit of the members. On the alleged failure to maintain and submitted
at Franklin Baker Company in San Pablo City, was filed by private the books of accounts for the years 1977, 1978 and 1979, they argued that
respondent Catalino Silvestre and thirteen (13) other employees, who are also they were elected in 1980 only and, therefore, they could not be made
members of the said Union. Acting on said request, Union Account Examiner responsible for the omissions of their predecessors who failed to turn over
Florencio R. Vicedo of the Ministry of Labor and Employment conducted union records for the questioned period. Anent their alleged failure to
the necessary investigation and, thereafter, submitted a report, with the maintain segregated disbursement receipts in accordance with the five (5)
following findings: segregated funds, petitioners maintained that the same did not result to any
loss of funds and such error in procedure had already been corrected. They
A. Disallowed expenditures — P1,278.00, as reflected in the following also demonstrated that there would be a general election on October 4, 1982,
breakdown: at which time, both the election and the desired referendum could be
undertaken to determine the membership at minimum expense. They prayed
1. January 9, 1980 — Excess claim for refund P1.00 that the resolution on the issue be held in abeyance.

2. March 13, 1980 — Payment for sound system P90.00 Private respondents, on the other hand, claimed that the Med-Arbiter erred in
calling a referendum to decide the issue. They reiterated that the appropriate
3. March 12, 1980 — Picture taking, entrance fee in Manila Zoo with Atty. action should be the expulsion of the herein union officers.
Delos Santos P75.00
On August 13, 1982, public respondent Director Trajano dismissed both
4. March 24, 1980 — Payment for sound System P90.00 appeals of petitioners and private respondents and affirmed in toto the order
of Med-Arbiter Cabibihan.
5. July 16, 1980 — Jeep hired P264.00
Petitioners filed a Motion for Reconsideration of the Resolution of August
6. August 30, 1980 — Partial payment of traveling expenses disallowed 13, 1982 of Public respondent Director Trajano, reiterating their arguments
P68.00 in their appeal and further clarifying that what the Union Account Officer
Florencio R. Vicedo found was that the amount of P1,278.00 was not
7. October 30, 1980 — Representation expenses P180.00 supported by official receipts and therefore should not be allowed as
disbursement from the union funds; and that he did not say that the amount
8. May 31, 1981 — Payment for long distance call P10.00 was converted by them for their own personal benefit. They, likewise,
informed public respondent Director Trajano that in the general election held
9. May 31, 1981— Payment for legal expenses P500.00 on October 4, 1982, all of them, except petitioners Ambrocio dela Cruz and
Eliseo Celerio, who ran for the positions of Vice-President and member of
TOTAL............................................................. P1,278.00 the Board of Directors, respectively, were elected by the overwhelming
majority of the members, while private respondents Catalino Silvestre and
B. Respondent union officers failed to keep, maintain and submit for Cesar Alfaro who also ran for the position of Auditor, lost. Thereafter, they
verification the records of union accounts for the years 1977, and 1978, 1979, moved for the dismissal of the appeal for having been rendered moot and
or purposely suppressed the same; academic by their re-election.

C. Respondent union officers failed to maintain segregated disbursement On October 19, 1982, public respondent Director Trajano issued the second
receipts in accordance with the five (5) segregated union funds (general fund, questioned order denying petitioners' Motion for Reconsideration.
educational funds, mutual aid fund, burial assistance fund and union building
fund) for which they maintained a distinct and separate bank accounts for Hence, this petition which We find meritorious for the following reasons:
each.
1. If herein union officers (also petitioners) were guilty of the alleged acts
D. The Union's constitution and by-laws is not ratified by the general imputed against them, said public respondent pursuant to Article 242 of the
membership hence, illegal. (pp. 27-28, Rollo) New Labor Code and in the light of Our ruling in Duyag vs. Inciong, 98
SCRA 522, should have meted out the appropriate penalty on them, i.e., to
Based on the foregoing revelations, private respondents filed with the expel them from the Union, as prayed for, and not call for a referendum to
Regional Office No. IV-A, Quezon City, Ministry of Labor and Employment, decide the issue;
a petition docketed as R04-ALRD-M- 9-35-81, for the expulsion of the union
officers on the ground that they committed gross violation of the Labor Code, 2. The alleged falsification and misrepresentation of herein union officers
specifically paragraphs (a), (b), (g), (h), (j) and (k) of Article 242; and, the were not supported by substantial evidence. The fact that they disbursed the
constitution and by-laws of the Union, particularly the provisions of Sections amount of P1,278.00 from Union funds and later on was disallowed for
6 and 7 thereof. failure to attach supporting papers thereon did not of itself constitute
falsification and/or misrepresentation. The expenditures appeared to have
In their Answer, the union officers denied the imputation and argued that the been made in good faith and the amount spent for the purpose mentioned in
disallowed expenditures were made in good faith; that the same conduced to the report, if concurred in or accepted by the members, are reasonable; and
the benefit of the members; and, that they are willing to reimburse the same
from their own personal funds. They likewise asserted that they should not
3. The repudiation of both private respondents to the highly sensitive position
of auditor at the October 4, 1982 election, is a convincing manifestation and
demonstration of the union membership's faith in the herein officers'
leadership on one hand and a clear condonation of an act they had allegedly
committed.

By and large, the holding of the referendum in question has become moot
and academic. This is in line with Our ruling in Pascual vs. Provincial Board
of Nueva Ecija, 106 Phil. 471, which We quote:

The Court should never remove a public officer for acts done prior to his
present term of office. To do otherwise would be to deprive the people of
their right to elect their officers. When the people have elected a man to
office, it must be assumed that they did this with knowledge of his life and
character, and that they disregarded or forgave Ms faults or misconduct, if he
had been guilty of any. It is not for the court, by reason of such faults or
misconduct to practically overrule the will of the people.

ACCORDINGLY, the resolution and order, dated August 13 and October 19,
1982, respectively, of public respondent Director Cresenciano B. Trajano of
the Bureau of Labor Relations, Ministry of Labor, Manila in BLR Case No.
A-0100-82 (RO4-A-LRD-M-9-35-81) are SET ASIDE and, the petition for
expulsion of herein union officers in R04-A-LRD-M-9-35-81 is hereby
DISMISSED for having been rendered moot and academic by the election of
herein union officers in the general membership meeting/election held on
October 4, 1982.

SO ORDERED.
MONTAÑO vs. VERCELES At the time Atty. Verceles lodged his opposition in the floor before the
G.R. No. 168583 July 26, 2010 holding of the election, we, the Comelec unanimously made the decision that
DEL CASTILLO, J.: Atty. Montaño and others are disqualified and barred from running for any
position in the election of the Federation, in view of pertinent provisions of
The Federation/Union’s Constitution and By-Laws govern the relationship the FFW Constitution.
between and among its members. They are akin to ordinary contracts in that
their provisions have obligatory force upon the federation/ union and its Our decision which we repeated several times as final was however further
member. What has been expressly stipulated therein shall be strictly binding deliberated upon by the body, which then gave the go signal for Atty.
on both. Montaño’s candidacy notwithstanding our decision barring him from
running and despite the fact that several delegates took the floor [stating] that
By this Petition for Review on Certiorari,1 petitioner Atty. Allan S. Montaño the convention body is not a constitutional convention body and as such
(Atty. Montaño) assails the Decision2 dated May 28, 2004 and Resolution3 could not qualify to amend the FFW’s present constitution to allow Atty.
dated June 28, 2005 of the Court of Appeals (CA) in CA-G.R. SP No. 71731, Montaño to run.
which declared as null and void his election as the National Vice-President
of Federation of Free Workers (FFW), thereby reversing the May 8, 2002 We would like to reiterate what we stated during the plenary session that our
Decision4 of the Bureau of Labor Relations (BLR) in BLR-O-TR-66-7-13- decision was final in view of the cited pertinent provisions of the FFW
01. Constitution and we submit that the decision of the convention body in
allowing Atty. Montaño’s candidacy is not valid in view of the fact that it
Factual Antecedents runs counter to the FFW Constitution and the body at that time was not acting
as a Constitutional Convention body empowered to amend the FFW
Atty. Montaño worked as legal assistant of FFW Legal Center on October 1, Constitution on the spot.
1994.5 Subsequently, he joined the union of rank-and-file employees, the
FFW Staff Association, and eventually became the employees’ union Our having conducted the election does not depart from the fact that we did
president in July 1997. In November 1998, he was likewise designated not change our decision disqualifying candidates such as Atty. Allan S.
officer-in-charge of FFW Legal Center.6 Montaño, and others from running. The National Convention as a co-equal
constitutional body of the Comelec was not given the license nor the
During the 21st National Convention and Election of National Officers of authority to violate the Constitution. It therefore, cannot reverse the final
FFW, Atty. Montaño was nominated for the position of National Vice- decision of the Comelec with regard to the candidacy of Atty. Allan Montaño
President. In a letter dated May 25, 2001,7 however, the Commission on and other disqualified candidates.16
Election (FFW COMELEC), informed him that he is not qualified for the
position as his candidacy violates the 1998 FFW Constitution and By-Laws, The BLR, in its Order dated August 20, 2001,17 did not give due course to
particularly Section 76 of Article XIX8 and Section 25 (a) of Article VIII,9 Atty. Montaño’s Motion to Dismiss but ordered the latter to submit his
both in Chapter II thereof. Atty. Montaño thus filed an Urgent Motion for answer to the petition pursuant to the rules. The parties thereafter submitted
Reconsideration10 praying that his name be included in the official list of their respective pleadings and position papers.
candidates.
On May 8, 2002, the BLR rendered a Decision18 dismissing the petition for
Election ensued on May 26-27, 2001 in the National Convention held at lack of merit. While it upheld its jurisdiction over the intra-union dispute case
Subic International Hotel, Olongapo City. Despite the pending motion for and affirmed, as well, Atty. Verceles’ legal personality to institute the action
reconsideration with the FFW COMELEC, and strong opposition and protest as president of an affiliate union of FFW, the BLR ruled that there were no
of respondent Atty. Ernesto C. Verceles (Atty. Verceles), a delegate to the grounds to hold Atty. Montaño unqualified to run for National Vice-
convention and president of University of the East Employees’ Association President of FFW. It held that the applicable provision in the FFW
(UEEA-FFW) which is an affiliate union of FFW, the convention delegates Constitution and By-Laws to determine whether one is qualified to run for
allowed Atty. Montaño’s candidacy. He emerged victorious and was office is not Section 76 of Article XIX19 but Section 26 of Article VIII20
proclaimed as the National Vice-President. thereof. The BLR opined that there was sufficient compliance with the
requirements laid down by this applicable provision and, besides, the
On May 28, 2001, through a letter11 to the Chairman of FFW COMELEC, convention delegates unanimously decided that Atty. Montaño was qualified
Atty. Verceles reiterated his protest over Atty. Montaño’s candidacy which to run for the position of National Vice-President.
he manifested during the plenary session before the holding of the election
in the Convention. On June 18, 2001, Atty. Verceles sent a follow-up letter12 Atty. Verceles filed a Motion for Reconsideration but it was denied by the
to the President of FFW requesting for immediate action on his protest. BLR.

Proceedings before the Bureau of Labor Relations Proceedings before the Court of Appeals

On July 13, 2001, Atty. Verceles, as President of UEEA-FFW and officer of Atty. Verceles thus elevated the matter to the CA via a petition for
the Governing Board of FFW, filed before the BLR a petition13 for the certiorari,21 arguing that the Convention had no authority under the FFW
nullification of the election of Atty. Montaño as FFW National Vice- Constitution and By-Laws to overrule and set aside the FFW COMELEC’s
President. He alleged that, as already ruled by the FFW COMELEC, Atty. Decision rendered pursuant to the latter’s power to screen candidates.
Montaño is not qualified to run for the position because Section 76 of Article
XIX of the FFW Constitution and By-Laws prohibits federation employees On May 28, 2004, the CA set aside the BLR’s Decision. While it agreed that
from sitting in its Governing Board. Claiming that Atty. Montaño’s jurisdiction was properly lodged with the BLR, that Atty. Verceles has legal
premature assumption of duties and formal induction as vice-president will standing to institute the petition, and that the applicable provision of FFW
cause serious damage, Atty. Verceles likewise prayed for injunctive relief.14 Constitution and By-Laws is Section 26 of Article VIII and not Section 76 of
Article XIX, the CA however ruled that Atty. Montaño did not possess the
Atty. Montaño filed his Comment with Motion to Dismiss15 on the grounds qualification requirement under paragraph (d) of Section 26 that candidates
that the Regional Director of the Department of Labor and Employment must be an officer or member of a legitimate labor organization. According
(DOLE) and not the BLR has jurisdiction over the case; that the filing of the to the CA, since Atty. Montaño, as legal assistant employed by FFW, is
petition was premature due to the pending and unresolved protest before the considered as confidential employee, consequently, he is ineligible to join
FFW COMELEC; and that, Atty. Verceles has no legal standing to initiate FFW Staff Association, the rank-and-file union of FFW. The CA, thus,
the petition not being the real party in interest. granted the petition and nullified the election of Atty. Montaño as FFW
National Vice-President.
Meanwhile, on July 16, 2001, the FFW COMELEC sent a letter to FFW
National President, Bro. Ramon J. Jabar, in reference to the election protest Atty. Montaño moved for reconsideration claiming that the CA seriously
filed before it by Atty. Verceles. In this correspondence, which was used by erred in granting Atty. Verceles’ petition on the ground that FFW Staff
Atty. Verceles as an additional annex to his petition before the BLR, the FFW Association, of which he is an officer and member, is not a legitimate labor
COMELEC intimated its firm stand that Atty. Montaño’s candidacy organization. He asserted that the legitimacy of the union was never raised
contravenes the FFW’s Constitution, by stating: as an issue. Besides, the declaration of the CA that FFW Staff Association is
not a legitimate labor organization amounts to a collateral attack upon its
legal personality, which is proscribed by law. Atty. Montaño also reiterated VERCELES HAS A CAUSE OF ACTION, DESPITE THE PENDENCY
his allegations of lack of jurisdiction and lack of cause of action due to a OF HIS PROTEST BEFORE FFW’S COMELEC AT THE TIME HE
pending protest. In addition, he claimed violation of the mandatory FILED HIS PETITION A QUO; AND ASSUMING FINALLY, THAT
requirement on certification against forum shopping and mootness of the case HEREIN RESPONDENT ATTY. VERCELES BE EXCUSED IN
due to the appointment of Atty. Verceles as Commissioner of the National DISREGARDING THE MANDATORY REQUIREMENT ON
Labor Relations Commission (NLRC), thereby divesting himself of interest CERTIFICATION AGAINST FORUM SHOPPING WHICH WAS
in any matters relating to his affiliation with FFW. TIMELY OBJECTED TO, THE COURT OF APPEALS COMMITTED
GRAVE ABUSE OF DISCRETION, AMOUNTING TO LACK AND/OR
Believing that it will be prejudiced by the CA Decision since its legal EXCESS OF JURISDICTION, IN NOT ORDERING THE DISMISSAL OF
existence was put at stake, the FFW Staff Association, through its president, THE CASE FOR HAVING BEEN RENDERED MOOT AND ACADEMIC
Danilo A. Laserna, sought intervention. BY A SUPERVENING EVENT –THAT WAS, WHEN HEREIN
RESPONDENT ATTY. VERCELES SOUGHT APPOINTMENT AND
On June 28, 2005, the CA issued a Resolution22 denying both Atty. WAS APPOINTED AS COMMISSIONER OF THE NATIONAL LABOR
Montaño’s motion for reconsideration23 and FFW Staff Association’s RELATIONS COMMISSION (NLRC), THUS, DIVESTING HIMSELF
motion for intervention/clarification.24 WITH ANY INTEREST WITH MATTERS RELATING TO HIS FORMER
MEMBERSHIP AND AFFILIATION WITH THE FEDERATION OF
Issues FREE WORKERS (FFW), HENCE, HE IS NO LONGER A REAL PARTY
IN INTEREST, AS HE DOES NOT STAND TO BE INJURED OR
Hence, this petition anchored on the following grounds: BENEFITED BY THE JUDGMENT IN THE INSTANT CASE.25

I. Atty. Montaño contends that the CA gravely erred in upholding the


jurisdiction of the BLR; in not declaring as premature the petition in view of
THE COURT OF APPEALS COMMITTED GRAVE ABUSE OF the pending protest before FFW COMELEC; in not finding that the petition
DISCRETION, AMOUNTING TO LACK AND/OR EXCESS OF violated the rule on non-forum shopping; in not dismissing the case for being
JURISDICTION, IN RENDERING THE ASSAILED DECISION, IN moot in view of the appointment of Atty. Verceles as NLRC Commissioner;
THAT: and in granting the petition to annul his election as FFW National Vice-
President on the ground that FFW Staff Association is not a legitimate labor
A.) THE SOLE GROUND USED AND/OR INVOKED IN GRANTING organization.
THE PETITION A QUO WAS NOT EVEN RAISED AND/OR INVOKED
BY PETITIONER; Our Ruling

B.) THE DECLARATION THAT "FFW STAFF ASSOCIATION IS NOT The petition is devoid of merit.
A LEGITIMATE LABOR ORGANIZATION", WITHOUT GIVING SAID
ORGANIZATION A ‘DAY IN COURT’ AMOUNTS TO A The BLR has jurisdiction over intra-union disputes involving a federation.
COLLATERAL ATTACK PROSCRIBED UNDER THE LAW; AND
We find no merit in petitioner’s claim that under Section 6 of Rule
C.) THE COURT OF APPEALS FAILED AND/OR REFUSED TO PASS
UPON OTHER LEGAL ISSUES WHICH HAD BEEN TIMELY RAISED, XV26 in relation to Section 1 of Rule XIV27 of Book V of the Omnibus
SPECIFICALLY ON THE PREMATURITY OF THE COMPLAINT AND Rules Implementing the Labor Code, it is the Regional Director of the DOLE
THE LACK OF CERTIFICATION AGAINST FORUM SHOPPING OF and not the BLR who has jurisdiction over election protests.
THE PETITION A QUO.
Section 226 of the Labor Code28 clearly provides that the BLR and the
II. Regional Directors of DOLE have concurrent jurisdiction over inter-union
and intra-union disputes. Such disputes include the conduct or nullification
THE COURT OF APPEALS ERRED IN UPHOLDING THE EXERCISE of election of union and workers’ association officers.29 There is, thus, no
OF JURISDICTION BY HEREIN RESPONDENT BUREAU AND IN doubt as to the BLR’s jurisdiction over the instant dispute involving member-
NOT ORDERING THE DISMISSAL OF THE CASE, DESPITE EXPRESS unions of a federation arising from disagreement over the provisions of the
PROVISION OF LAW GRANTING SAID JURISDICTION OVER CASES federation’s constitution and by-laws.
INVOLVING PROTESTS AND PETITIONS FOR ANNULMENT OF
RESULTS OF ELECTIONS TO THE REGIONAL DIRECTORS OF THE We agree with BLR’s observation that:
DEPARTMENT OF LABOR AND EMPLOYMENT.
Rule XVI lays down the decentralized intra-union dispute settlement
III. mechanism. Section 1 states that any complaint in this regard ‘shall be filed
in the Regional Office where the union is domiciled.’ The concept of
IN THE ALTERNATIVE, THE COURT OF APPEALS LIKEWISE domicile in labor relations regulation is equivalent to the place where the
ERRED IN NOT ORDERING THE DISMISSAL OF THE PETITION A union seeks to operate or has established a geographical presence for
QUO, IN THAT: purposes of collective bargaining or for dealing with employers concerning
terms and conditions of employment.
A.) THE FILING OF THE PETITION FOR NULLIFICATION OF THE
RESULT OF ELECTION IS PREMATURE, IN VIEW OF PENDENCY OF The matter of venue becomes problematic when the intra-union dispute
HEREIN RESPONDENT ATTY. VERCELES’ PROTEST BEFORE THE involves a federation, because the geographical presence of a federation may
COMMISSION ON ELECTION OF THE FEDERATION OF FREE encompass more than one administrative region. Pursuant to its authority
WORKERS (FFW COMELEC) AT THE TIME OF THE FILING OF THE under Article 226, this Bureau exercises original jurisdiction over intra-union
SAID PETITION, HENCE, HE HAS NO CAUSE OF ACTION; AND disputes involving federations. It is well-settled that FFW, having local
unions all over the country, operates in more than one administrative region.
B.) HEREIN RESPONDENT ATTY. VERCELES HAS VIOLATED Therefore, this Bureau maintains original and exclusive jurisdiction over
SECTION 5, RULE 7 OF THE 1997 RULES ON CIVIL PROCEDURE, AS disputes arising from any violation of or disagreement over any provision of
HIS PETITION A QUO HAS NO CERTIFICATION AGAINST FORUM its constitution and by-laws.30
SHOPPING, WHICH IS A MANDATORY REQUIREMENT. IT IS ALSO
IN UTTER DISREGARD AND IN GROSS VIOLATION OF SUPREME The petition to annul Atty. Montaño’s election as VP was not prematurely
COURT CIRCULAR NO. 04-94. filed.

IV. There is likewise no merit to petitioner’s argument that the petition should
have been immediately dismissed due to a pending and unresolved protest
FINALLY, ASSUMING ARGUENDO THAT HEREIN RESPONDENT before the FFW COMELEC pursuant to Section 6, Rule XV, Book V of the
BUREAU ACTED WITH JURISDICTION OVER THE CASE; AND Omnibus Rules Implementing the Labor Code.31
ASSUMING FURTHER THAT HEREIN RESPONDENT ATTY.
It is true that under the Implementing Rules, redress must first be sought unqualified for the position for failing to meet the requirements set forth
within the organization itself in accordance with its constitution and by-laws. therein.
However, this requirement is not absolute but yields to exception under
varying circumstances.32 In the case at bench, Atty. Verceles made his We find that both the BLR and CA erred in their findings.
protest over Atty. Montaño’s candidacy during the plenary session before the
holding of the election proceedings. The FFW COMELEC, notwithstanding To begin with, FFW COMELEC is vested with authority and power, under
its reservation and despite objections from certain convention delegates, the FFW Constitution and By-Laws, to screen candidates and determine their
allowed Atty. Montaño’s candidacy and proclaimed him winner for the qualifications and eligibility to run in the election and to adopt and
position. Under the rules, the committee on election shall endeavor to settle promulgate rules concerning the conduct of elections.39 Under the Rules
or resolve all protests during or immediately after the close of election Implementing the Labor Code, the Committee shall have the power to
proceedings and any protest left unresolved shall be resolved by the prescribe rules on the qualification and eligibility of candidates and such
committee within five days after the close of the election proceedings.33 A other rules as may facilitate the orderly conduct of elections.40 The
day or two after the election, Atty. Verceles made his written/formal protest Committee is also regarded as the final arbiter of all election protests.41 From
over Atty. Montaño’s candidacy/proclamation with the FFW COMELEC. the foregoing, FFW COMELEC, undeniably, has sufficient authority to
He exhausted the remedies under the constitution and by-laws to have his adopt its own interpretation of the explicit provisions of the federation’s
protest acted upon by the proper forum and even asked for a formal hearing constitution and by-laws and unless it is shown to have committed grave
on the matter. Still, the FFW COMELEC failed to timely act thereon. Thus, abuse of discretion, its decision and ruling will not be interfered with. The
Atty. Verceles had no other recourse but to take the next available remedy to FFW Constitution and By-laws are clear that no member of the Governing
protect the interest of the union he represents as well as the whole federation, Board shall at the same time perform functions of the rank-and-file staff. The
especially so that Atty. Montaño, immediately after being proclaimed, BLR erred in disregarding this clear provision. The FFW COMELEC’s
already assumed and started to perform the duties of the position. ruling which considered Atty. Montaño’s candidacy in violation of the FFW
Consequently, Atty. Verceles properly sought redress from the BLR so that Constitution is therefore correct.
the right to due process will not be violated. To insist on the contrary is to
render the exhaustion of remedies within the union as illusory and vain.34 We, thus, concur with the CA that Atty. Montaño is not qualified to run for
the position but not for failure to meet the requirement specified under
The allegation regarding certification against forum shopping was belatedly Section 26 (d) of Article VIII of FFW Constitution and By-Laws. We note
raised. that the CA’s declaration of the illegitimate status of FFW Staff Association
is proscribed by law, owing to the preclusion of collateral attack.42 We
Atty. Montaño accuses Atty. Verceles of violating the rules on forum nonetheless resolve to affirm the CA’s finding that Atty. Montaño is
shopping. We note however that this issue was only raised for the first time disqualified to run for the position of National Vice-President in view of the
in Atty. Montaño’s motion for reconsideration of the Decision of the CA, proscription in the FFW Constitution and By-Laws on federation employees
hence, the same deserves no merit. It is settled that new issues cannot be from sitting in its Governing Board. Accordingly, the election of Atty.
raised for the first time on appeal or on motion for reconsideration.35 While Montaño as FFW Vice-President is null and void.
this allegation is related to the ground of forum shopping alleged by Atty.
Montaño at the early stage of the proceedings, the latter, as a ground for the WHEREFORE, the petition is DENIED. The assailed May 28, 2004
dismissal of actions, is separate and distinct from the failure to submit a Decision of the Court of Appeals in CA-G.R. SP No. 71731 nullifying the
proper certificate against forum shopping.36 election of Atty. Allan S. Montaño as FFW National Vice-President and the
June 28, 2005 Resolution denying the Motion for Reconsideration are
There is necessity to resolve the case despite the issues having become moot. AFFIRMED.

During the pendency of this case, the challenged term of office held and SO ORDERED.
served by Atty. Montaño expired in 2006, thereby rendering the issues of the
case moot. In addition, Atty. Verceles’ appointment in 2003 as NLRC
Commissioner rendered the case moot as such supervening event divested
him of any interest in and affiliation with the federation in accordance with
Article 213 of the Labor Code. However, in a number of cases,37 we still
delved into the merits notwithstanding supervening events that would
ordinarily render the case moot, if the issues are capable of repetition, yet
evading review, as in this case.

As manifested by Atty. Verceles, Atty. Montaño ran and won as FFW


National President after his challenged term as FFW National Vice-President
had expired. It must be stated at this juncture that the legitimacy of Atty.
Montaño’s leadership as National President is beyond our jurisdiction and is
not in issue in the instant case. The only issue for our resolution is petitioner’s
qualification to run as FFW National Vice-President during the May 26-27,
2001 elections. We find it necessary and imperative to resolve this issue not
only to prevent further repetition but also to clear any doubtful interpretation
and application of the provisions of FFW Constitution & By-laws in order to
ensure credible future elections in the interest and welfare of affiliate unions
of FFW.

Atty. Montaño is not qualified to run as FFW National Vice-President in


view of the prohibition established in Section 76, Article XIX of the 1998
FFW Constitution and By-Laws.1awph!1

Section 76, Article XIX of the FFW Constitution and By-laws provides that
no member of the Governing Board shall at the same time be an employee in
the staff of the federation. There is no dispute that Atty. Montaño, at the time
of his nomination and election for the position in the Governing Board, is the
head of FFW Legal Center and the President of FFW Staff Association. Even
after he was elected, albeit challenged, he continued to perform his functions
as staff member of FFW and no evidence was presented to show that he
tendered his resignation.38 On this basis, the FFW COMELEC disqualified
Atty. Montaño. The BLR, however, overturned FFW COMELEC’s ruling
and held that the applicable provision is Section 26 of Article VIII. The CA
subsequently affirmed this ruling of the BLR but held Atty. Montaño
HALILI vs. COURT OF INDUSTRIAL RELATIONS and quitclaim Halili Enterprises, Halili Transit, Fortunato F. Halili his estate,
G.R. No. L-24864 April 30, l985 heirs and successors by reason of CIR Case No. 1099-V, it being their
MAKASIAR, J.: intention that they be absolutely, completely and finally absolved and
released from any and all liability in said case, including attorneys' liens the
Before Us for resolution is the urgent motion to cite Atty. Benjamin C. transfer of the property and payment of the amount hereinabove stated
Pineda, Ricardo Capuno and Manila Bank (Cubao Branch) in contempt for constituting for all intents and purposes a full, final and complete settlement
the alleged continued failure of aforenamed parties to comply with the and satisfaction of the award in CIR Case No. 1099-V and all incidents
temporary mandatory restraining order issued by this Court on September 1, thereto.
1983 and with the resolution dated September 13, 1983 which again directed
Atty. Pineda and union administrator Capuno to comply with the aforesaid 4. The UNION and its undersigned officers hereby warrant that the UNION
mandatory restraining order and which ordered the Manila Bank to transfer is a duly registered labor organization and that in a special meeting called for
the funds allocated for the workers to the NLRC (p. 376, L-24864, rec.; p. the purpose they were duly authorized on December 22, 1974, by all the
301, L027773 rec.). members- claimants in CIR Case No. 1099-V to sign this Memorandum of
Agreement with Release and Quitclaim which was unanimously approved
The issuance of the temporary mandatory restraining order stemmed from and ratified by said members-claimants as evidenced by a Resolution dated
the questioned orders of September 23, 1982 and February 9, 1983 issued by December 22, 1974, a copy of which is attached hereto and made a part
Labor Arbiter Raymundo Valenzuela in Case No. 1099-V before the NLRC hereof as Annex "B", and hereby jointly and severally hold the estate and
which orders respectively allowed the sale of the property awarded to satisfy heirs of Fortunato F. Halili free and harness from, and undertake to indemnify
or answer for the claims of the union members in these four cases and them for, any and all liability for any claims by members of the UNION, their
authorized the distribution of the proceeds of the purchase. heirs, assigns and agents relating to CIR Case No. 1099-V or attorneys' liens
in connection therewith (69 SCRA 509-510).
For a better appreciation of the aforesaid motion for contempt, We must
recall certain prefatory facts which the Solicitor General has so aptly summed On January 6, 1975, pursuant to the Agreement, the administratrix of the
up. Thus: estate of Fortunato F, Halili executed a Deed of Conveyance of Real
Property, transferring the aforementioned parcel of land to the Halili Bus and
The above-entitled cases involve disputes regarding claims for overtime of Conductors Union (PTGWO) in trust for the members of the union claimants.
more than five hundred bus drivers and conductors of Halili Transit. The parcel of land was eventually registered in the name of the Union on
Litigation initially commenced with the filing of a complaint for overtime February 14, 1975. Hence, on February 10, 1976, the contending parties
with the defunct Court of Industrial Relations on August 20, 1958 docketed moved for the dismissal of G.R. No. L-30110 and G.R. No. L-38655, which
as CIR Case No. 1099-V. The disputes were eventually settled when the this Honorable Court granted on February 27, 1976 (69 SCRA 505). The two
contending parties reached an Agreement on December 23, 1974, the other cases, G.R. No. L-24864 and G.R. No. L- 27773, were previously
pertinent portions of which are as follows: disposed of on February 26, 1968 and December 28, 1970, respectively (22
SCRA 785. and 36 SCRA 522).
WHEREAS, in the face of this strong urging on the part of the Supreme Court
Justices upon the parties to put an immediate end to this case by amicable On August 9, 1982, the Union, through Atty. Benjamin C. Pineda, filed an
settlement, the parties repeatedly came to conference, conscientiously urgent motion with the Ministry of Labor and Employment (MOLE)
explored all avenues of settlement, and finally arrived at the tentative requesting for authority to sell and dispose of the property. The motion was
agreement (tentative because of the condition that the same be sanctioned by granted in an order dated September 23, 1982. A prospective buyer, the
the court in the estate case) whereby the Administratrix would transfer to the Manila Memorial Park Cemetery, inc. expressed its misgivings on the
employees title to that tract of land, covered by TCT No. 36389, containing authority of the Union to sell the property in view of sec. 66 of PD 1529
an area of approximately 33,952 square meters, situated in the Barrio of San which requires no less than an order from a court of competent jurisdiction
Bartolome, Municipality of Caloocan, Province of Rizal, and pay in addition as authority to sell property in trust. So, Atty. Pineda filed a motion with the
the cash amount of P25,000.00 in full and final satisfaction of all the claims Supreme Court on December 1, 1982 requesting for authority to sell the
and causes of action of all of the employees against the estate of Fortunato F. property, This Honorable Court, however, merely noted the motion in a
Halili subject of CIR Case No. 1099-V. resolution dated December 8, 1982.

xxx xxx xxx Nevertheless, Atty. Pineda, without authority from the Supreme Court but
relying on the earlier authority given him by the Ministry of Labor, filed
NOW, THEREFORE, for and in consideration of the foregoing and of the another urgent motion with the latter, praying that the Union be authorized
covenants, stipulations and undertakings hereinafter contained, the parties to sell the lot to the Manila Memorial Park Cemetery, Inc. and to make
have agreed as follows: arrangements with it such that payment will be advanced for the real estate
taxes inclusive of penalties, attorney's lien which is equivalent to a thirty-five
l. The UNION, its officers and members-claimants relative to CIR Case No. percent (35%) of the total purchase price, and home developer's fee of
1099-V, shall withdraw and dismiss with prejudice Case No. 1099-V filed P69,000.00. Apparently, the prospective purchaser had decided to withdraw
by the UNION in behalf of its members-claimants before the Court of its objection regarding the Union's authority to sell. In an Order dated
Industrial Relations and all its incidents thereto. February 9, 1983, Labor Arbiter Raymundo R. Valenzuela granted the
motion. So, the sale was finally consummated on June 7, 1983, resulting in
2. The ESTATE shall deliver or cause to be delivered, to the UNION the the execution of an escrow agreement on June 8, 1983 wherein the purchase
following: price was deposited under escrow with the Manila Bank-Cubao Branch. The
Bank then released the amounts due the claimants in accordance with the
(a) Deed of Transfer of a parcel of land situated in Barrio San Bartolome, escrow agreement" (pp. 352- 356, L-24864 rec.).
Caloocan City, containing an area of THIRTY-THREE THOUSAND NINE
HUNDRED FIFTY-TWO (33,952) Square Meters, more or less, and covered The dispositive portion in L-24864 is re-stated hereunder:
by Transfer Certificate of Title No. 35389 of the Registry of Deeds of Rizal,
to be made, upon authority and approval granted by the Court of First of WHEREFORE, the appealed order and resolution en banc are hereby
Rizal, Branch IV, at Quezon City, in Proc. No. Q-10852 in the name of the affirmed and the Court of Industrial Relations is hereby enjoined to make a
Halili Bus Drivers & Conductors Union (PTGWO), free from any and all judicial determination of the union membership of the claimants, while the
liens encumbrances, and any and all claims whatsoever. Examining Division of said court shall proceed with its computation of the
compensable hours of work rendered by, and the corresponding
(b) Negotiable Check for TWENTY-FIVE THOUSAND (P25,000.00) compensation payable to, the drivers and conductors admitted by both parties
PESOS in the name of Domingo D. Cabading, President of the UNION. to be union members since October 1, 1956 and those contended by the union
to be such members but disputed by the employer. No costs. So ordered (p.
3. The transfer of the above-described parcel of land and receipt of the 186, L-24864 rec.).
amount of P25,000.00 constitute the full and final satisfaction of the claims
and award in said CIR Case No. 1099-V, as well as any and all attorney's When Atty. Jose C. Espinas (herein movant and alleged original counsel for
liens in said case, for and in consideration of which the UNION members- the Union) learned of the sale and apportionment of the proceeds from past
claimants in CIR Case No. 1099-V by these present now and forever release Union president Amado Lopez, he requested Labor Arbiter Raymundo
Valenzuela to allow him to look into the records of Case No. 1099-V. The agents to deposit with the NLRC 6% alleged union expenses paid to the
latter, however, told him that the records of the aforecited case were missing. Union or similarly deposited to its account; and ordered the NLRC and
Thereupon, Atty. Espinas requested Director Pascual Reyes of the NLRC to Manila Bank, Cubao Branch, or their agents or persons in their stead not to
locate the records (p. 356, L24864 rec.). allow withdrawals of amounts deposited in the name of Atty. Benjamin C.
Pineda and/or the Union or any of its officers (P. 235, L-24864; p. 188, L-
Hence, Atty. Espinas filed the urgent motion with prayer for a temporary 27773 rec.).
mandatory restraining order on August 26, 1983 and the supplement thereto
on August 29, 1983 (pp. 215, 227, L-24864 rec.). On September 6, 1983, respondent Union, thru Atty. Pineda, filed its
comment, in compliance with the resolution of September 1, 1983, on the
On August 30, 1983, the records of Case No. 1099-V were finally found and urgent motion and the supplement thereto both filed by counsel Espinas,
Atty. Espinas was dully informed of the development, alleging therein that the subject matter sought to be enjoined or mandated by
the restraining order ceased to exist rendering the same moot and academic,
The above two motions question the legality of the orders dated September and thus praying for the dismissal of the said motion and the supplement
23, 1982 and February 9, 1983 issued by Labor Arbiter Raymundo thereto (p. 237, L-24864 rec.; p. 191, L-27773 rec.).
Valenzuela in Case No. 1099-V before the NLRC which authorized the sale
of the awarded property and the distribution of the proceeds from such On September 7, 1983, Atty. Pedro Lopez, an original associate of Atty.
purchase. Espinas, filed his motion for leave to intervene, with the submission that the
lawyers involved should only divide 20% fees as per the workers' contract
Movants Union and counsel Espinas upon filing of the motions urgently pray and the rest refunded by Atty. Pineda and the alleged "union officers" for
of thisourt to: redistribution to the members (p. 265, L-24864, rec.; p. 219. L-27773 rec.).

1. Require Atty. Benjamin C. Pineda to deposit with the NLRC the amount Atty. Espinas, in behalf of the workers, filed a manifestation and motion to
of P712,992.00 paid to him or deposited to his account at Manila Bank, require Atty, Pineda and the union to comply with the temporary mandatory
Cubao Branch,allegedly representing 35% attorney's fees on the sale of restraining order on September 9, 1983, with prayer that the Manila Bank be
33,952 square meters of the lot registered in the name of the Union; ordered to transfer the funds allocated for the workers to the NLRC, which
should be instructed to pay the workers upon proper Identification (without
2. Require the Halili Drivers and Conductors Union through Domingo prejudice to additional shares) or to mail such amounts by money order or
Cabading or any of his representatives to deposit with the NIRC the 6% manager's check to the workers' addresses as furnished to the NLRC (p. 274,
alleged union expenses paid to them or similarly deposited to their account; L-24864, rec.; p. 231, L-27773 rec.).

3. Implead with leave of court this Manila Bank Cubao Branch to require the On September 12, 1983, petitioner filed a manifestation in compliance with
said bank to prevent further withdrawals of amount deposited in the name of the resolution of September 2, 1983 stating, among other things, that its
Atty. Pineda and/or the Halili Drivers and Conductors Union or any of its liability had been completely extinguished with the approval of the
officers and to turn over any remaining deposits to the NLRC for proper Memorandum of Agreement with Release and Quitclaim in L-38655 and L-
disposition; 30110; that said agreement operated as an absolute and complete release of
petitioner from any liability to the Union; and that petitioner had not been
4. Should Atty. Pineda and the Union officers have already withdrawn the given any notice of any proceedings respecting cases subsequent to the
deposits or parts thereof, require them to post a bond in the equivalent promulgation of the decisions aforestated (p. 281, L-24864, rec.; p. 237, L-
amounts of 35% (attorney's fee), 6% (union expenses), and 5% (broker's fee) 27773 rec.).
respectively of the total proceeds of the sale of the property, solidarity (p.
219, L-24864 rec.; p. 160, L-27773 rec.). Counsel Espinas (for the workers involved) filed his reply to comments of
respondent Union on September 14, 1983 praying for this Court to:
Likewise, and after due consideration of the merits, movants prayed that—
1. nullify the order of February 9, 1983 issued by Arbiter Raymundo
1. the order of Arbiter Valenzuela dated February 9, 983 be nullified insofar Valenzuela in CIR Case No. 1099-V and others connected therewith
as it allows Atty. Pineda 35% attorney's fees; regarding the distribution of proceeds of the sale of the land belonging to the
members-claimants for lack of due process and for being contrary to law;
2. the NLRC be directed to locate the records of Case No. 1099-V or
reconstitute the same and thereafter to equitably dispose 20% as fees to all 2. nullify the 35% attorney's fees of Atty. Benjamin Pineda as illegal and
lawyers who participated in the proceedings and any excess amounts to be unconscionable and in disregard of other lawyers in the case;
again distributed to the workers; and
3. require reimbursement to the members-from the Union P101,856.00
3. these cases be remanded to the NLRC with instructions as above-stated allocated without their consent as Union expenses; P101,856 unreceipted
and that the proper penalty be imposed on those involved and who have acted brokers' fees less P4,020.40 expenses for the transfer of title; to refund the 1
fraudulently and illegally (p. 220, L-24864 rec.; p. 165, L-27773 rec.). % of the net proceeds, P9,596.18, for named claimants; and to secure a refund
of P308,000.00 from the P712,992.00 fees of Atty. Pineda (the excess of 20%
The succeeding pleadings and developments which are common to all these fees for all lawyers);
cases are now presented chronologically.
4. subject the balance of P404,992.00 of the remainder of Atty. Pineda's 35%
On August 29, 1983, Atty. Espinas, for himself and members of the fees for distribution among the three lawyers as may be determined by the
respondent Union, filed a supplement to urgent motion stating that the NLRC; and
prayers in the urgent motion of August 26, 1983 are reiterated and praying
for the nullification of Arbiter Valenzuela's order not only on the award of 5. should this Court so decides, fix the fees (p. 285, L- 24864 rec.; p. 240, L-
attorney's fees but also on the allowance of payment of "union obligations" 27773 rec.).
not previously authorized nor approved by the NLRC (p. 227, L-24864, rec.;
p. 176, L-27773 rec.). On September 13, 1983, the Solicitor General filed his comment on the
urgent motion and the supplement thereto dated August 25, 1983 and August
In its resolution dated September 1, 1983, this Court impleaded the Manila 29, 1983, respectively with the recommendations that (1) the orders of
Bank, Cubao Branch as party respondent and directed the issuance of a Arbiter Valenzuela dated September 23, 1982 and February 9, 1983 be
temporary mandatory restraining order (p. 234, L-24864 rec. & p. 187, L- nullified for having been issued without due process; (2) the case must be
27773 rec.). This Court correspondingly issued a temporary mandatory remanded to the NLRC for further proceedings; and (3) the temporary
restraining order on the same date which enjoined Atty. Benjamin C. Pineda restraining order issued by this Court on September 1, 1983 be maintained,
or his agents or any person acting in his stead to deposit with the NLRC the pending final resolution by the NLRC (p. 351, L-24864 rec.).
amount of P712,992.00 paid to him or deposited in his account at Manila
Bank, Cubao Branch allegedly representing 35% attorney's fees on the sale The Solicitor General, on October 6, 1983, filed his manifestation and motion
of 33,952 square meters of the lot registered in the name of Halili Drivers in lieu of comment on the motion of Atty. Pedro Lopez for leave to intervene
and Conductors Union; directed the Union thru Domingo Cabading or his in L-24864 and L-27773 (p. 360, L-24864 rec.; p. 289, L-27773 rec.).
On October 6, 1983, counsel Espinas filed his comment on the intervention On December 20, 1983, Mr. de Pedro and Atty. Espinas, for the workers
of Atty. Pedro Lopez wherein he offers no objection to the latter's involved, filed their rejoinder to the comment of Atty. Pineda and Mr.
intervention and states that said counsel is also entitled to attorney's fees in Capuno reiterating therein their plea to declare Atty. Pineda and Mr. Capuno
accordance with his participation (p. 364, L-24864 rec.; p. 292, L-27773 in contempt of court and to mete out the proper penalty (p. 328, L-27773
rec.). rec.).

Atty. Pineda filed his comment and manifestation on October 7, 1983, in The Manila Banking Corporation filed its compliance with the Court
compliance with the resolution of September 13, 1983, alleging therein that resolution of November 22, 1983 on February 3, 1984, praying that its report
as per Retainer's Contract dated January 1, 1967, he handled Case No. 1099- to the NLRC on the amount of withdrawals be considered as sufficient
V before the Court of Industrial Relations alone. On the mandatory compliance with the said resolution (p. 343, L-27773 rec.).
restraining order, Atty. Pineda claims that as of October 4, 1983, he had a
balance of P2,022.70 in his account with the Manila Bank (p. 370, L-24864 Atty. Espinas filed his comment and motion on March 15, 1984, stating
rec.; p. 295, L-27773 rec.). among other things that as per report of the Manila Bank to the NLRC, Atty.
Pineda has not yet complied with the said order. He thus moved that Atty.
In its resolution dated October 18, 1983, this Court (1) set, aside as null and Pineda be required to post a bond on the undeposited balance in the amounts
void the orders of September 23, 1982 and February 9, 1983 of Arbiter of P710,969.30 and that Mr. Capuno be also required to post a bond before
Raymundo R. Valenzuela; (2) allowed the intervention of Atty. Pedro Lopez; the NLRC on the undeposited balance of P52,236.04 during the pendency of
(3) directed the Manila Bank (Cubao Branch), Atty. Benjamin Pineda, and the motion for contempt (p. 373, L-27773 rec.).
the Halili Drivers and Conductors Union through Domingo Cabading or any
of his representatives, to comply with the temporary mandatory restraining On April 4, 1984, Mr. Sergio de Pedro filed his reply to the aforesaid
order issued on September 1, 1983 and the resolution dated September 13, comment of the Union administrator and Atty. Pineda stating therein that
1983, within ten [10] days from receipt thereof; and (4) remanded these cases there are still questions to be resolved on the merits before the NLRC and
to the NLRC for further proceedings (p. 374, L-24864 rec.; p. 299, L-27773 hence, prays that Arbiter Antonio Tirona be required to continue hearing the
rec.). merits of the case pending in the said Commission (p. 377, L-27773 rec.).

The day before or on October 17, 1983, Sergio de Pedro, as representative of Before We resolve the motion for contempt, certain crucial facts which have
the workers and assisted by Atty. Espinas, thus fided the urgent motion to surfaced and which precipitated Our issuance of the resolution of October
cite Atty. Pineda, Ricardo Capuilo and Manila Bank (Cubao Branch) in 18, 1983 declaring the two questioned orders of Arbiter Valenzuela as null
contempt, alleging therein that after two letters dated October 6 and October and void, must be retraced.
l4, l983 to the NLRC which inquired as to whether or not compliant, with the
restraining order had been made, the Commission certified that as of October Then Union President Amado Lopez, in a letter dated August 21, 1958,
14, 1983, no deposits had been effected by the parties so (directed (p. 376, informed J.C. Espinas and Associates that the general membership of the said
L-24864 rec.; p. 301, L-27773 rec.). Union had authorized a 20% contingent fee for the law firm based on
whatever amount would be awarded the Union (p. 267, L-24864 rec.).
In its manifestation and motion filed on November 2, 1983, respondent
Manila Banking Corporation (Rustan-Cubao Branch), in compliance with Atty. Jose C. Espinas, the original counsel, established the award of 897
this Court's resolution of September 13, 1983, stated that it transmitted or workers' claim in the main cases before the defunct CIR and the Supreme
paid to the NLRC the amount of P417,380.64 under Cashier's Check No. Court. In L-24864, the Notice of Judgment of this Court dated February 26,
34084190 for the account of the Union and P2,022.70 under Cashier's Check 1968 was served on Messrs. J.C. Espinas & Associates (p. 188, L-24864
No. 34084191 for the account of Atty. Pineda and thus prayed therein that rec.). In L-27773, the Notice of Judgment dated December 29, 1970 was sent
the aforesaid transmittals be deemed as sufficient compliance with the to Atty. B.C. Pineda & Associates under same address-716 Puyat Bldg., Suit
aforecited resolution and that the urgent motion to cite respondents in 404 at Escolta, Manila (p. 147, L-27773 rec.) Note that this is the same
contempt dated October 17, 1983 be considered moot and academic (p. 390, address of Atty. J.C. Espinas & Associates.
L-24864 rec.).
When Atty. 'Pineda appeared for the Union in these cases, still an associate
On November 8, 1983, respondent Atty. Pineda filed his manifestation and of the law firm, his appearance carried the firm name B.C. Pineda and
motion in lieu of comment in compliance with this Court's resolution of Associates," giving the impression that he was the principal lawyer in these
October 20, 1983, stating that he and respondent Union thereby adopt the cases.
aforecited manifestation and motion of respondent Manila Banking
Corporation and thus prayed that since they have complied with this Court's Atty. Pineda joined the law firm of Atty. Espinas in 1965 when these cases
resolution of September 13, 1983, the urgent motion to cite them for were pending resolution. He always held office in the firm's place at Puyat
contempt be considered moot and academic (p. 394, L-24864 rec.; p. 310, L- Building, Escolta until 1974, except in 1966 to 1967 when he transferred to
27773 rec.). the Lakas ng Manggagawa Offices. During this one-year stint at the latter
office, Atty. Pineda continued handling the case with the arrangement that he
On November 10, 1983, respondent Manila Banking Corporation filed would report the developments to the Espinas firm. When he rejoined the law
another manifestation and motion in lieu of commence, by way of firm in 1968, he continued working on these cases and using the Puyat
compliance with the Court's resolution of October 20, 1983 with prayer that Building office as his address in the pleadings.
its previous manifestation and motion dated October 28, 1983 and filed on
November 2, 1983 be considered as sufficient compliance with the resolution When Atty. Pineda rejoined the Espinas firm in 1968, he did not reveal to his
of September 13, 1983 which would render the urgent motion to cite partners (he was made the most senior partner) that he had a retainer's
respondents in contempt moot and academic (p. 396, L-24864 rec. p. 312, L- contract entered into on January 1, 1967 which allegedly took effect in 1966.
27773 rec.). He stayed with the law firm until 1974 and still did not divulge the 1967
retainer's contract. Only the officers of the Union knew of the contract.
On the foregoing manifestations and motions, representative Sergio de
Pedro, with the assistance of Atty. Espinas, filed a comment on November The alleged retainer's contract between Atty. Pineda and the Union appears
16,1983 wherein he alleged that out of the P2,037,120.00 purchase price, anomalous and even illegal as well as unethical considering that-
only Pl,940,127.29 was deposited with the Manila Bank; that Atty. Pineda
has yet to return the balance of P710,969,30; and that the Union has still to 1. The contract was executed only between Atty. Pineda and the officers of
account for P111,452.18 (p. 399, L- 24864 rec.; p. 315, L-27773 rec.). the Union chosen by about 125 members only. It was not a contract with the
general membership, Only 14% of the total membership of 897 was
December 14, 1983, respondent Union filed its reply to Mr. de Pedro's above represented. This violates Article 242 (d) of the Labor Code which provides:
unsigned comment therein stating among other things that the alleged
missing amount of P96.992.71 was used for the payment of outstanding real The members shall determine by secret ballot, after due deliberation, any
estate taxes on real property of said Union covered by TCT No. 205755 and question of major policy affecting the entire membership of the organization,
that the amount of P2,022.70 only was remitted by Manila Bank to the NLRC unless the nature of the organization or force majeure renders such secret
for the account of Atty. Pineda (p. 323, L-27773 rec.) ballot impractical, in which case the board of directors of the organization
may make the decision in behalf of the general membership (emphasis Court was the proper body which had the power to grant such authority. He
supplied). could not and did not even wait for such valid authority but instead previously
obtained the same from the labor arbiter whom he knew was not empowered
2. The contingent fee of 30% for those who were still working with Halili to so authorize. Under Article 224 (a) of the Labor Code, only final decisions
Transit and the 45% fee for those who were no longer working worked to the or awards of the NLRC, the Labor Arbiter, or compulsory or voluntary
prejudice of the latter group who should and were entitled to more benefits. arbitrators may be implemented or may be the subject of implementing
Thus, too, when the alleged retainer's contract was executed in 1967, the orders by aforenamed body or officers.
Halili Transit had already stopped operations in Metro Manila. By then, Atty.
Pineda knew that all the workers would be out of work which would mean When Atty. Espinas discovered the sale of the property, he went to Arbiter
that the 45% contingent fee would apply to all. Valenzuela to look into the transaction who told him that the records of CIR
Case No. 1099-V were missing. It took director Pascual Reyes of the NLRC
3. The contract which retroactively took effect on January 1, 1966, was to locate the records.
executed when Atty. Espinas was still handling the appeal of Halili Transit
in the main case before the Supreme Court. Atty. Pineda would have but did The 45% attorney's lien on the award of those union members who were no
not substitute himself in place of Atty. Espinas or the law firm on the basis longer working and the 30% lien on the benefits of those who were still
of such contract. working as provided for in the alleged retainer's contract are very exorbitant
and unconscionable in view of Section 11, Rule VIII of Book III which
4. When Atty. Pineda filed his motion for approval of his attorney's lien with explicitly provides:
Arbiter Valenzuela on February 8, 1983, he did not attach the retainer's
contract. Sec. 11. Attorney's fees—Attorney's fees on any judicial or administrative
proceedings for the recovery of wages shall not exceed 10% of the amount
5. The retainer's contract was not even notarized (p. 248, L-24864 rec.). awarded. The fees may be deducted from the total amount due the winning
party.
The Manila Memorial Park Cemetery, Inc., as the prospective buyer, initially
expresses its misgivings over the authority of the Union to sell subject The amount of P101,856.00 which Atty. Pineda donated to the Union and
property conformably with Section 66 of P.D. No. 1529, which requires an which actually corresponds to 5% of the total 35% attorney's fees taken from
order from a court of competent jurisdiction authorizing the sale of a property the proceeds (p. 263, L-24864, rec.) appears improper since it amounts to a
in trust. The pertinent portion of Section 66 provides: rebate or commission. This amount was subsequently treated as union
miscellaneous operating expenses without the consent of the general
No instruments which transfers or mortgages or in any way deals with membership.
registered land in trust shall be registered, unless the enabling power thereto
is expressly conferred in the trust instrument, or unless a final judgment or Thus, in the case of Amalgamated Laborers' Association vs. Court of
order of a court of competent jurisdiction has construed the instrument in Industrial Relations (L-23467, 22 SCRA 1267 [March 27, 1968]), We
favor of the power, in which case a certified copy of such judgment or order declared:
may be registered.
We strike down the alleged oral agreement that the union president should
The decision of aforenamed purchaser to stop questioning the Union's share in the attorney's fees. Canon 34 of Legal Ethics condemns this
authority to sell and the expeditious manner by which Arbiter Valenzuela arrangement in terms clear and explicit. It says: 'No division of fees for legal
granted Atty. Pineda's motion for such authority to sell the property make the services is proper, except with another lawyer, based upon a division of
entire transaction dubious and irregular. service or responsibility.' The union president is not the attorney for the
laborers. He may seek compensation only as such president. An agreement
Thus, without notice to the other lawyers and parties, Atty. Pineda whereby a union president is allowed to share in attorney's fees is immoral.
commenced the proceeds before the NLRC with the filing of a motion and Such a contract we emphatically reject. It cannot be justified.
manifestation on August 9, 1982 with Arbiter Valenzuela of the NLRC
Office of the Labor Ministry wherein he asked for authority to sell the A contingent fee contract specifying the percentage of recovery an attorney
property. On September 23, 1983 or just over a month, Arbiter Valenzuela is to receive in a suit 'should be reasonable under all the circumstances of the
approved the motion per order of the same date. Notably, only Atty. Pineda case, including the risk and uncertainty of the compensation, but should
and the lawyers of the purchaser were informed of such order. always be subject to the supervision of a court, as to its reasonableness.
(emphasis supplied).
On February 4, 1983, again without notice to Atty. Espinas and Atty. Lopez,
Atty. Pineda filed a motion with Arbiter Valenzuela wherein he asked for A deeper scrutiny of the pleadings in L-24864 notably indicates a fraudulent
authority to distribute the proceeds of the sale of the property. This or deceitful pattern in the actuations of Atty. Pineda. Thus, in his motion for
distribution would include his attorney's fee which was allegedly the subject execution of judgment filed on September 18, 1965 in this case, he signed
of a retainer contract entered into between him and the alleged Union for and in behalf of "J.C. Espinas & Associates" (p. 323, rec.). In his
officers, On February 9, 1983, or barely five days from the day the motion manifestation dated December 10, 1968, he signed as "B.C. Pineda," lone
was filed, Arbiter Valenzuela, without informing the other lawyers and counsel for petitioner (p. 327, rec.); and yet, he carried the address of Espinas
relying exclusively on the unverified motion of Atty. Pineda (the records of & Associates at 716 G. Puyat Building, Escolta.
the case were not on hand), approved the said motion which authorized the
appointment. However, in the October 29, 1968 resolution of this Court, a copy thereof
was served on "Messrs. J.C. Espinas, B.C Pineda, J.J. dela Rosa &
This Court, as earlier stated, nullified said orders dated September 23, 1982 Associates" at Puyat Building, Escolta (p. 324, rec.). In the notice of
and February 9, 1983 of Labor Arbiter Valenzuela as violative of the due judgment dated December 29, 1970, this Court addressed the said pleading
process clause. It is a settled rule that in administrative proceedings, or cases to "Attys. B.C. Pineda & Associates with the same Puyat Building address
coming before administrative tribunals exercising quasi-judicial powers, due (p. 325, rec.). Notably also, then Union President Amado Lopez addressed
process requires not only notice and hearing, but also the consideration by his letter dated August 21, 1958 to J.C. Espinas & Associates" wherein he
the administrative tribunal of the evidence presented; the existence of informed the latter that the general membership of the Union had authorized
evidence to support the decision; its substantiality a decision based thereon them a 20%, contingent fee on whatever award would be given the workers
or at least contained in the record and disclosed to the parties; such decision (p. 267, rec.).
by the administrative tribunal resting on its own independent consideration
of the law and facts of the controversy; and such decision acquainting the The Manila Banking Corporation (Cubao Branch) has manifested that it
parties with the various issued involved and the reasons therefore (Ang Tibay turned over to the NLRC the amount of P417,380.64 for the Union's account,
vs. Court, 69 Phil. 635, cited on p. 84, Philippine Constitutional Law, which appears to be the balance of P950,021.76 corresponding to the net
Fernando, 1984 ed.) proceeds for distribution to the workers after deducting P525,480.40, the
total payments to claimants. The amount of P417,380.64 appears lacking,
Significantly Atty. Pineda's act of filing a motion with this Court on since accurately computed, the balance should be P424,541,36.
December 1, 1982 praying for authority to sell was by itself an admission on
his part that he did not possess the authority to sell the property and that this
However, the Union has yet to account for P101,856.00, the 5% donation or nature of an interlocutory order, or, if made after final decree, as remedial in
share from Atty. Pineda's attorney's fee of 35%. nature, and may be reviewed only on appeal from the final decree, or in such
other mode as is appropriate to the review of judgments in civil cases. ... The
For the account of Atty. Pineda, the Manila Banking Corporation has question of whether the contempt committed is civil or criminal, does not
remitted to the NLRC the amount of P2,022.70 only. This means that Atty. affect the jurisdiction or the power of a court to punish the same. ... (58 Phil.
Pineda is still accountable for the amount of P710,969.30. He is directed to 271, 272).
return the amount of P712,992.00 representing the 35% attorney's fees he
unlawfully received. For civil contempt, Section 7, Rule 71 of the Revised Rules of Court
explicitly provides:
In view of Our resolution of October 18, 1983, which set aside as null and
void the questioned orders dated September 23, 1982 and February 9, 1983 Sec. 7, Rule 71. Imprisonment until order obeyed. When the contempt
issued by Arbiter Raymundo Valenzuela, the sale of the Union property and consists in the omission to do an act which is yet in the power of the accused
the distribution of the proceeds therefrom had been effected without authority to perform, he may be imprisoned by order of a superior court until he
and, therefore, illegal Consequently. Atty. Pineda and Arbiter Valenzuela performs it.
become liable for their unauthorized acts,
Thus, in the case of Harden vs. Director of Prisons (L-2349, 81 Phil. 741
Atty. Pineda should be cited for indirect contempt under paragraphs (b), (c) [Oct. 22, 1948]), where petitioner was confined in prison for contempt of
and (d) of Section 3, Rule 71 of the Revised Rules of Court, The said court, this Court, in denying the petition and resolving the question of
paragraphs read thus: petitioner's indefinite confinement, had the occasion to apply and clarify the
aforequoted provision in the following tenor:
Sec. 3. indirect contempts to be punished after charge and hearing.—
The penalty complained of is neither cruel unjust nor excessive. In Ex-parte
xxx xxx xxx Kemmler 136 U.S. 436, the United States Supreme Court said that
'punishments are cruel when they involve torture or a lingering death, but the
(b) Disobedience of or resistance to a lawful writ, process, order, judgment, punishment of death is not cruel, within the meaning of that word as used in
or company court, or injunction granted by a court or judge, including the act the constitution. It implies there something inhuman and barbarous,
of a person who, after being dispossessed or ejected from any real property something more than the extinguishment of life.
by the judgment or process of any court of competent jurisdiction, enters or
attempts or induces another to enter into or upon such real property, for the The punishment meted out to the petitioner is not excessive. It is suitable and
purpose of executing acts of ownership or possession, or in any manner adapted to its objective; and it accords with section 7, Rule 64 of the Rules
disturbs the possession given to the person adjudged to be entitled thereto; of Court which provides that "when the contempt consists in the omission to
do an act which is yet in the power of the accused to perform, he may be
(c) Any abuse of or any interference with the process or proceedings of a imprisoned by order of a superior court until he performs it."
court not constituting direct contempt under section 1 of this rule;
If the term of imprisonment in this case is indefinite and might last through
(d) Any improper conduct tending, directly or indirectly to impede, obstruct, the natural life of the petitioner, yet by the terms of the sentence the way is
or degrade the administration of justice. left open for him to avoid serving any part of it by complying with the orders
of the court, and in this manner put an end to his incarceration. In these
Contempt of court is a defiance of the authority, justice or dignity of the circumstances, the judgment cannot be said to be excessive or unjust. (Davis
court; such conduct as tends to bring the authority and administration of the vs. Murphy [1947], 188 P., 229- 231.) As stated in a more recent case (De
law into disrespect or to interfere with or prejudice parties litigant or their Wees [1948], 210 S.W., 2d, 145-147), 'to order that one be imprisoned for an
witnesses during litigation (12 Am. jur. 389, cited in 14 SCRA 813). indefinite period in a civil contempt is purely a remedial measure. Its purpose
is to coerce the contemner to do an act within his or her power to perform.
Contempt of court is defined as a disobedience to the court by acting in He must have the means by which he may purge himself of the contempt .
opposition to its authority, justice and dignity. It signifies not only a willful The latter decision cites Staley vs. South Jersey Realty Co., 83 N.J. Eq., 300,
disregard or disobedience of the court's orders, but such conduct as tends to 90 A., 1042, 1043, in which the theory is expressed in this language:
bring the authority of 'the court and the administration of law into disrepute
or in some manner to impede the due administration of justice (17 C.J.S. 4). In a "civil contempt" the proceeding is remedial, it is a step in the case the
object of which is to coerce one party for the benefit of the other party to do
This Court has thus repeatedly declared that the power to punish for contempt or to refrain from doing some act specified in the order of the court. Hence,
is inherent in all courts and is essential to the preservation of order in judicial if imprisonment be ordered, it is remedial in purpose and coercive in
proceedings and to the enforcement of judgments, orders, and mandates of character, and to that end must relate to something to be done by the
the court, and consequently, to the due administration of justice (Slade defendant by the doing of which he may discharge himself. As quaintly
Perkins vs. Director of Prisons, 58 Phil. 271; In re Kelly, 35 Phil. 944; expressed, the imprisoned man carries the keys to his prison in his own
Commissioner of Immigration vs. Cloribel, 20 SCRA 1241; Montalban vs. pocket (pp. 747-748).
Canonoy, 38 SCRA 1).
Likewise. American courts had long enunciated these rulings:
In the matter of exercising the power to punish contempts, this Court
enunciated in the Slade Perkins case that "the exercise of the power to punish The commitment of one found in contempt of a court order only until the
contempts has a twofold aspect, namely (1) the proper punishment of the contemnor shall have purged himself of such contempt by complying with
guilty party for his disrespect to the court or its order; and (2) to compel his the order is a decisive characteristic of civil contempt. Maggio v. Zeitz, 333
performance of some act or duty required of him by the court which he US 56, 92 L. ed. 476, 68 S Ct 401.
refuses to perform. Due to this twofold aspect of the exercise of the power to
punish them, contempts are classified as civil or criminal. A civil contempt Civil or quasi-criminal contempt is contemplated by a statute providing that
is the failure to do something ordered to be done by a court or a judge for the if any person refused to obey or perform any rule, order, or judgment of court,
benefit of the opposing party therein; and a criminal contempt, is conduct such court shall have power to fine and imprison such person until the rule,
directed against the authority and dignity of a court or of a judge, as in order, or judgment shall be complied with. Evans v. Evans, 193 Miss 468, 9
unlawfully assailing or discrediting the authority or dignity of the court or So 2d. 641. (17 Am. Jur. 2d.)
judge, or in doing a duly forbidden act. Where the punishment imposed,
whether against a party to a suit or a stranger, is wholly or primarily to protect The reason for the inherent power of courts to punish for contempt is that
or vindicate the dignity and power of the court, either by fine payable to the respect of the courts guarantees the stability of the judicial institution.
government or by imprisonment, or both, it is deemed a judgment in a Without such guarantee said institution would be resting on a very shaky
criminal case. Where the punishment is by fine directed to be paid to a party foundation (Salcedo vs. Hernandez, 61 Phil. 724; Cornejo vs. Tan, 85 Phil.
in the nature of damages for the wrong inflicted, or by imprisonment as a 722),
coercive measure to enforce the performance of some act for the benefit of
the party or in aid of the final judgment or decree rendered in his behalf, the Likewise, Atty. Pineda should be subject to disbarment proceedings under
contempt judgment will, if made before final decree, be treated as in the Section 27 of Rule 138 of the Revised Rules of Court which provides:
2. The alleged 5% donation of Atty. Pineda to the Union taken from the 35%
Sec. 27. Attorneys removed or suspended by Supreme Court on what attorneys' fees was given to and received by then President Domingo
grounds.—A member of the bar may be removed or suspended from his Cabading alone, who thereafter left for the United States.
office as attorney by the Supreme Court for any deceit, malpractice, or other
gross misconduct in such office, grossly immoral conduct, or by reason of 3. The 1% allocated for unknown claimants or those not previously listed in
his conviction of a crime involving moral turpitude, or for any violation of the amount of P9,596.18 can easily be accounted for by the Union before the
the oath which he is required to take before admission to practice, or for a NLRC.
willful disobedience of any lawful order of a superior court, or for corrupt or
willfully appearing as an attorney for a party to a case without authority so In the same motion, Mr. Capuno clarifies that with regard to attorneys' fees,
to do. The practice of soliciting cases at law for the purpose of gain, either Atty. Pineda made the Union officers believe that he would be the one to pay
personally or through paid agents or brokers, constitutes malpractice. the fees of Attys. Espinas and Lopez for which reason, the 35% increased
fees was approved by the Union's board in good faith. The Union likewise
The Court may suspend or disbar a lawyer for any conduct on his part confirms that Atty. Pineda came into the picture only when he was assigned
showing his unfitness for the confidence and trust which characterize the by Atty. Espinas in, 1965 to execute the CIR decision which, thru Atty.
attorney and client relations, and the practice of law before the courts, or Espinas handling, was upheld by this Court in L-24864 in 1968. The Union
showing such a lack of personal honesty or of good moral character as to officers were aware that Atty. Espinas was the principal counsel even after
render him unworthy of public confidence (7 C.J.S. 733). Atty. Pineda's assignment. They also knew of the original contract for 20%
attorney's fees which was increased to 35% by Atty. Pineda upon the
It is a well-settled rule that the statutory grounds for disbarment or suspension arrangement that with the increase, he would answer for the payment of
are not to be taken as a limitation on the general power of the courts in this Attys. Espinas and Lopez' fees and for necessary representation expenses (p.
respect. The inherent powers of the court over its officers cannot be restricted 450, L-24864 rec.).
(In re Pelaez, 44 Phil. 567).
Acting on the aforesaid motion, this Court in its resolution of August 28,
Finally, Atty. Pineda could be prosecuted for betrayal of trust by an attorney 1964, dropped the Union and its officers from the within contempt charge (p.
under Article 209 of the Revised Penal Code. Said article provides: 455, L-24864 rec.).

Art. 209. Betrayal of must by an attorney or solicitor. Revelation of secrets.— WHEREFORE, ATTY. BENJAMIN PINEDA IS HEREBY FOUND
In addition of the proper administrative action , the penalty of prision GUILTY OF INDIRECT CONTEMPT OF COURT FOR WHICH HE IS
correccional in its minimum period, or a fine ranging from 200 to 1,000 HEREBY SENTENCED TO IMPRISONMENT IN THE MANILA CITY
pesos, or both shall be imposed upon any attorney-at-law or solicitor JAIL UNTIL THE ORDERS OF THIS COURT DATED SEPTEMBER 1
(procurador judicial) who, by any malicious breach of professional duty or AND SEPTEMBER 13, 1983 ARE COMPLIED WITH.
inexcusable negligence or ignorance, shall prejudice his client, or reveal any
of the secrets of the latter learned by him in his professional capacity ATTY. BENJAMIN PINEDA IS ALSO DIRECTED TO SHOW CAUSE
(emphasis supplied). WHY HE SHOULD NOT BE DISBARRED UNDER RULE 138 OF THE
REVISED RULES OF COURT.
The aforequoted criminal sanction for unprofessional conduct of an attorney
is without prejudice to proper administrative action, such as disbarment or LET COPIES OF THIS RESOLUTION AND THE RESOLUTION OF
suspension of attorneys (p. 503, Criminal Law Annotated, Padilla, 1972 Ed.). OCTOBER 18, 1983 BE FURNISHED THE MINISTRY OF LABOR AND
THE TANODBAYAN FOR APPROPRIATE ACTION.
Labor Arbiter Raymundo Valenzuela should be made to answer for having
acted without or beyond his authority in proper administrative charges. He SO ORDERED.
could also be prosecuted before the Tanodbayan under the provisions of the
Anti-Graft Law. Independently of his liabilities as a government officer, he
could be the subject of disbarment proceedings under Section 27, Rule 138
of the Revised Rules of Court.

Atty. Benjamin Pineda could also be held liable under Section 4(b) of R.A.
No. 3019 (Anti-Graft and Corrupt Practices Act) which makes it unlawful for
any person knowingly to induce or cause any public official to commit any
of the offenses defined in Section 3 of said act. Section 3 enumerates the
corrupt practices which public officers may be prosecuted for. Atty. Pineda
knowingly induced or caused Labor Arbiter Valenzuela to issue the
questioned orders without or beyond the latter's authority and to which orders
the former was not entitled, considering that he was not the sole and proper
representative.

The Manila Banking Corporation (Cubao Branch) per manifestation and


motion dated October 28, 1983 and reiterated on November 10, 1983, had
transmitted to the NLRC the remaining balance of P417,380.64 and
P2,022.70 for the account of the Union and Atty. Pineda, respectively. This
turnover of the aforecited amounts is a sufficient compliance with Our
restraining order and resolution of September 13, 1983 and hence, the Manila
Banking Corporation can no longer be liable for contempt of court.

Very recently, on August 23, 1984, respondent Union, thru Acting


Administrator Ricardo Capuno, filed its motion to drop Halili Bus Drivers
and Conductors Union from the contempt charge in view of these reasons:

1. The Manila Bank has already turned over to the NLRC the amount of
P59,716.14 which represents the remaining balance of 5% earmarked for
Union expenses incurred in the case aside from the amounts deposited in
escrow for the workers. The amount of P42,140.00 was spent legitimately by
the Union for administration purposes relative to the subject property. The
Union asserts that it is ready and willing to account for all expenses and
withdrawals from the bank before the NLRC.
PACIFIC BANKING CORPORATION vs. CLAVE Presidential Executive Assistant Clave should have noticed that article 111
G.R. No. L-56965 | 1984-03-07 refers to a proceeding for the recovery of wages and not to CBA negotiations.
AQUINO, J.: The two are different or distinct proceedings. Not satisfied with the
clarificatory resolution, Clave issued a third resolution wherein he held that
This case is about the legality of deducting from the monetary benefits it is the legal obligation of the bank to turn over to the union treasurer ten
awarded in a collective bargaining agreement the attorney's fees of the lawyer percent of the award as Saavedra's fees.
who assisted the union president in negotiating the agreement. It also
involves the jurisdiction of the Office of the President of the Philippines to Finally, in a fourth resolution dated April 13, 1981 Deputy Presidential
order such deduction. Executive Assistant Joaquin T. Venus, Jr. ordered the bank to pay the union
treasurer the said attorney's fees less the amounts corresponding to the
Since January, 1979, there had been negotiations between the Pacific protesting employees. He held that the following article 222 of the Labor
Banking Corporation and the Pacific Banking Corporation Employees Code, as amended by Presidential Decree No. 1691, effective May 1, 1980
Organization (PABECO) for a collective bargaining agreement for 1979 to (before the formalization of the CBA award) had no retroactive effect to the
1981. Because of a deadlock, the Minister of Labor assumed jurisdiction over case:
the controversy. On July 10, 1979, the Deputy Minister rendered a decision
directing the parties to execute a CBA in accordance with the terms and "ART. 222. Appearances and Fees. - . . . (b) No attorney's fees, negotiation
conditions set forth in his decision (pp. 16-01, Rollo). fees or similar charges of any kind arising from any collective bargaining
negotiations or conclusion of the collective agreement shall be imposed on
The union was represented in the negotiations by its president, Paula S. Paug, any individual member of the contracting union: Provided, however, that
allegedly assisted as consultant by Jose P. Umali, Jr., the president of the attorney's fees may be charged against union funds in an amount to be agreed
National Union of Bank Employees (NUBE) with which it was formerly upon by the parties. Any contract, agreement or arrangement of any sort to
affiliated (p. 209, Rollo). Lawyer Juanito M. Saavedra's earliest recorded the contrary shall be null and void." (271, PD 442; 220, PD 626; 222, PD
participation in the case was on July 15 and 27, 1979 when he filed a motion 1691).
for reconsideration and a supplemental motion. No action was taken on said
motions (p. 121, Rollo). The bank assailed in this Court the said resolutions by means of certiorari.
On February 5, 1982, the NUBE and thirteen employees of the bank,
The parties appealed to the Office of the President of the Philippines. The members of the PABECO, (p. 202, Rollo) intervened in this case and prayed
CBA negotiations were resumed. The union president took part in the second that the said resolutions be declared void and that said sum of P345,000 be
phase of the negotiations. Saavedra filed a memorandum. He claimed he paid directly to the employees or union members (p. 214, Rollo).
exerted much effort to expedite the decision. The Office of the President
issued on March 18, 1980 a resolution directing the parties to execute a CBA We hold that, under the circumstances, the Office of the President had no
containing the terms and conditions of employment embodied in the jurisdiction to make an adjudication on Saavedra's attorney's fees. The case
resolution. was appealed with respect to the CBA terms and conditions, not with respect
to attorney's fees. Although the fees were a mere incident, nevertheless, the
The CBA was ultimately finalized on June 3, 1980. Monetary benefits of jurisdiction to fix the same and to order the payment thereof was outside the
more than fourteen million pesos were involved in the three-year CBA, pale of Clave's appellate jurisdiction. He was right in adopting a hands-off
according to the bank's counsel. attitude in his first resolution and holding that the payment of the fees was a
question between the lawyer and the union.
Even before the formalization of the CBA on June 3, 1980, Saavedra on
March 24, 1980 filed in the case his notice of attorney 's lien. On May 20, Moreover, the case is covered squarely by the mandatory and explicit
1980, the bank's vice-president in a reply to the letter of the union president prescription of article 222 which is another guarantee intended to protect the
stated that he had serious doubts about paying the attorney's fees (pp. 144- employee against unwarranted practices that would diminish his
145, Rollo). compensation without his knowledge and consent. (See National Power
Corporation Supervisors' Union vs. National Power Corporation, L-28805,
The union officials requested the bank to withhold around P345,000 out of August 10, 1981, 106 SCRA 556). Other provisions of the Labor Code
the total benefits as ten percent attorney's fees of Saavedra. At first, the bank animated by the same intention are the following:
interposed no objection to the request in the interest of harmonious labor-
management relations (p. 145, Rollo). In theory, the actual ten percent "ART. 242. Rights and conditions of membership in a labor organization. -
attorney's fees may amount to more than one million pesos (p. 281, Rollo). The following are the rights and conditions of membership in a labor
organization:
For nearly a year, the Office of the President in four resolutions wrestled with "(n) No special assessment or other extraordinary fees may be levied upon
the propriety of Saavedra's ten percent attorney's fees. In a resolution dated the members of a labor organization unless authorized by a written resolution
May 29, 1980, Presidential Executive Assistant Jacobo C. Clave refused to of a majority of all the members at a general membership meeting duly called
intervene in the matter. He ruled that the payment of attorney's fees was a for the purpose. The secretary of the organization shall record the minutes of
question that should be settled by the union and its lawyer themselves (p. 24, the meeting including the list of all members present, the votes cast, the
Rollo). purpose of the special assessment or fees and the recipient of such assessment
or fees. The record shall be attested to by the president;.
Then, he "clarified" that ruling in a second resolution wherein he directed
that the attorney's fees may be deducted from the total benefits and paid to "(o) Other than for mandatory activities under the Code, no special
Saavedra in accordance with the following provisions of the Labor Code: assessment, attorney's fees, negotiation fees or any other extraordinary fees
may be checked off from any amount due an employee without an individual
"Art. 111. Attorney's fees. - (a) In cases of unlawful withholding of wages written authorization duly signed by the employee. The authorization should
the culpable party may be assessed attorney's fees equivalent to ten percent specifically state the amount, purpose and beneficiary of the deduction; and.
of the amount of wages recovered.
"(b) It shall be unlawful for any person to demand or accept, in any judicial . . . (288, PD 442; 291, PD 570-A; 240, PD 626; 241, PD 850).
or administrative proceedings for the recovery of wages, attorney's fees
which exceed ten percent of the amount of wages recovered." There is no doubt that lawyer Saavedra is entitled to the payment of his fees
but article 222 ordains that union funds should be used for that purpose. The
Article 161 is implemented in Rule VIII, Book III of the Implementing Rules amount of P345,000 does not constitute union funds. It is money of the
and Regulations as follows: employees. The union, not the employees, is obligated to Saavedra.

"Sec. 11. Attorney's fees. - Attorney's fees in any judicial or administrative WHEREFORE, the petition is granted. The resolutions dated August 12 and
proceedings for the recovery of wages shall not exceed 10% of the amount December 15, 1980 and April 13, 1981 are reversed and set aside. The
awarded. The fees may be deducted from the total amount due the winning questioned amount of about P345,000, with its increments, if any, should be
party." paid by the bank directly to its employees. No costs.

SO ORDERED
KAISAHAN AT KAPATIRAN NG MGA MANGGAGAWA AT
KAWANI SA MWC-EAST ZONE UNION vs. MANILA WATER 31. Attorney's fees â€" 10% to be deducted from AA and CBA
COMPANY, INC. receivables.
G.R. No. 174179 | 2011-11-16 32. All other issues are considered withdrawn.19
BRION, J.: In their Opposition, the petitioners argued that the MOA only covered the
payment of their share in the contracted attorney's fees, but did not include
the attorney's fees awarded by the NLRC. To support their claim, the
We resolve the petition for review on certiorari1 filed by the petitioners, petitioners submitted Borela's affidavit which relevantly stated:
Kaisahan at Kapatiran ng mga Manggagawa at Kawani sa MWC-East Zone 2. On December 19, 2003, in settlement of the notice of Strike for CBA
Union (Union) and Eduardo Borela, assailing the decision2 and the Deadlock, Manila Water Company, Inc. and the Union entered into an
resolution3 of the Court of Appeals (CA) in CA-G.R. SP No. 83654.4 Agreement settling the deadlock issued (sic) of the CBA negotiation
including [the] payment of the AA and the mode of payment thereof.
3. Considering that the AA payment was included in the Agreement, the
The Factual Antecedents Union representation deemed it wise, for practical reason, to authorize the
The background facts are not disputed and are summarized below. company to immediately deduct from the benefits that will be received by
The Union is the duly-recognized bargaining agent of the rank-and-file the member/employees the 10% attorney's fees in conformity with our
employees of the respondent Manila Water Company, Inc. (Company) while contract with our counsel.
Borela is the Union President.5 On February 21, 1997, the Metropolitan 4. The 10% attorney's fees paid by the members/employees is separate and
Waterworks and Sewerage System (MWSS) entered into a Concession distinct from the obligation of the company to pay the 10% awarded
Agreement (Agreement) with the Company to privatize the operations of the attorney's fees which we also gave to our counsel as part of our contingent
MWSS.6 Article 6.1.3 of the Agreement provides that "the Concessionaire fee agreement.
shall grant [its] employees benefits no less favorable than those granted to 5. There was no agreement that we are going to shoulder the entire attorney's
MWSS employees at the time of [their] separation from MWSS."7 Among fees as this would cost us 20% of the amount we would recover. There was
the benefits enjoyed by the employees of the MWSS were the amelioration also no agreement that the 10% attorney's fees in the MOA represents the
allowance (AA) and the cost-of-living allowance (COLA) granted in August entire attorney's cost because the said payment represents only our
1979, pursuant to Letter of Implementation No. 97 issued by the Office of compliance of our share in the attorney's fees in conformity with our contract.
the President.8 Likewise, we did not waive the awarded 10% attorney's fees because the
The payment of the AA and the COLA was discontinued pursuant to same belongs to our counsel and not to us and beyond our authority.20
Republic Act No. 6758, otherwise known as the "Salary Standardization (emphasis ours)
Law," which integrated the allowances into the standardized salary.9 The NLRC subsequently denied both parties' Motions for Partial
Nonetheless, in 2001, the Union demanded from the Company the payment Reconsideration,21 prompting the Company to elevate the case to the CA via
of the AA and the COLA during the renegotiation of the parties' Collective a petition for certiorari under Rule 65 of the Rules of Court. It charged the
Bargaining Agreement (CBA).10 The Company initially turned down this NLRC of grave abuse of discretion in sustaining the award of attorney's fees
demand, however, it subsequently agreed to an amendment of the CBA on on the grounds that: (1) it is contrary to the MOA22 concerning the payment
the matter, which provides: of attorney's fees; (2) there was no finding of unlawful withholding of wages
The Company shall implement the payment of the Amelioration Allowance or bad faith on the part of the Company; and (3) the attorney's fees awarded
and Cost of Living [A]llowance retroactive August 1, 1997 should the are unconscionable.
MWSS decide to pay its employees and all its former employees or upon
award of a favorable order by the MWSS Regulatory Office or upon receipt
of [a] final court judgment.11 The CA Decision
In its Decision promulgated on March 6, 2006,23 the CA modified the
Thereafter, the Company integrated the AA into the monthly payroll of all its assailed NLRC rulings by deleting "[t]he order for respondent MWCI to pay
employees beginning August 1, 2002, payment of the AA and the COLA attorney's fees equivalent to 10% of the total judgment awards." The CA
after an appropriation was made and approved by the MWSS Board of recognized the binding effect of the MOA between the Company and the
Trustees. The Company, however, did not subsequently include the COLA Union; it stressed that any further award of attorney's fees is unfounded
since the Commission on Audit disapproved its payment because the considering that it did not find anything in the Agreement that is contrary to
Company had no funds to cover this benefit.12 law, morals, good customs, public policy or public order.
In resolving the issue, the CA cited our ruling in Traders Royal Bank
As a result, the Union and Borela filed on April 15, 2003 a complaint against Employees Union-Independent v. NLRC,24 where we distinguished between
the Company for payment of the AA, COLA, moral and exemplary damages, the two commonly accepted concepts of attorney's fees â€" the ordinary
legal interest, and attorney's fees before the National Labor Relations and the extraordinary. We held in that case that under its ordinary concept,
Commission (NLRC).13 attorney's fees are the reasonable compensation paid to a lawyer by his client
for legal services rendered. On the other hand, we ruled that in its
extraordinary concept, attorney's fees represent an indemnity for damages
The Compulsory Arbitration Rulings ordered by the court to be paid by the losing party in a litigation based on
what the law provides; it is payable to the client not to the lawyer, unless
In his decision of August 20, 2003, Labor Arbiter Aliman D. Mangandog there is an agreement to the contrary.
(LA) ruled in favor of the petitioners and ordered the payment of their AA The CA noted that the fees at issue in this case fall under the extraordinary
and COLA, six percent (6%) interest of the total amount awarded, and ten concept â€" the NLRC having ordered the Company, as losing party, to
percent (10%) attorney's fees.14 pay the Union and its members ten percent (10%) attorney's fees. It found the
award without basis under Article 111 of the Labor Code which provides that
On appeal by the Company, the NLRC affirmed with modification the LA's attorney's fees equivalent to ten percent (10%) of the amount of wages
decision.15 It set aside the award of the COLA benefits because the claim recovered may be assessed only in cases of unlawful withholding of wages.
was not proven and established, but ordered the Company to pay the The CA ruled that the facts of the case do not indicate any unlawful
petitioners their accrued AA of about P107,300,000.00 in lump sum and to withholding of wages or bad faith attributable to the Company. It also held
continue paying the AA starting August 1, 2002. It also upheld the award of that the additional grant of 10% attorney's fees violates Article 111 of the
10% attorney's fees to the petitioners. Labor Code considering that the MOA between the parties already ensured
the payment of 10% attorney's fees, deductible from the AA and CBA
In its Motion for Partial Reconsideration of the NLRC's December 19, 2003 receivables of the Union's members. The CA thus adjudged the NLRC
decision, the Company pointed out that the award of ten percent (10%) decision awarding attorney's fees to have been rendered with grave abuse of
attorney's fees to the petitioners is already provided for in their December 19, discretion.
2003 Memorandum of Agreement (MOA) which mandated that attorney's The Union and Borela moved for reconsideration, but the CA denied the
fees shall be deducted from the AA and CBA receivables.16 This motion in its resolution of August 15, 2006.25 Hence, the present petition.
compromise agreement, concluded between the parties in connection with a
notice of strike filed by the Union in 2003,17 provides among others that:18
The Petition
NLRC if the same are not supported by substantial evidence. The Court has
The petitioners seek a reversal of the CA rulings on the sole ground that the not hesitated to affirm the appellate court's reversals of the decisions of labor
appellate court committed a reversible error in reviewing the factual findings tribunals if they are not supported by substantial evidence. 31 (italics and
of the NLRC and in substituting its own findings â€" an action that is not emphasis supplied; citation omitted)
allowed under Rule 65 of the Rules of Court. They question the CA's re-
evaluation of the evidence, particularly the MOA, and its conclusion that
there was no unlawful withholding of wages or bad faith attributable to the As discussed below, our review of the records and of the CA decision shows
Company, thereby contradicting the factual findings of the NLRC. They also that the CA erred in ruling that the NLRC gravely abused its discretion in
submit that a petition for certiorari under Rule 65 is confined only to issues awarding the petitioners ten percent (10%) attorney's fees without basis in
of jurisdiction or grave abuse of discretion, and does not include the review fact and in law. Corollary to the above-cited rule is the basic approach in the
of the NLRC's evaluation of the evidence and its factual findings.26 Rule 45 review of Rule 65 decisions of the CA in labor cases which we
The petitioners argue that in the present case, all the parties' arguments and articulated in Montoya v. Transmed Manila Corporation32 as a guide and
evidence relating to the award of attorney's fees were carefully studied and reminder to the CA. We laid down that:
weighed by the NLRC. As a result, the NLRC gave credence to Borela's
affidavit claiming that the attorney's fees paid by the Union's members are
separate and distinct from the attorney's fees awarded by the NLRC. The In a Rule 45 review, we consider the correctness of the assailed CA decision,
petitioners stress that whether the NLRC is correct in giving credence to in contrast with the review for jurisdictional error that we undertake under
Borela's affidavit is a question that the CA cannot act upon in a petition for Rule 65. Furthermore, Rule 45 limits us to the review of questions of law
certiorari unless grave abuse of discretion can be shown.27 raised against the assailed CA decision. In ruling for legal correctness, we
have to view the CA decision in the same context that the petition for
certiorari it ruled upon was presented to it; we have to examine the CA
The Case for the Company decision from the prism of whether it correctly determined the presence or
In its Memorandum filed on September 7, 2007,28 the Company argues that absence of grave abuse of discretion in the NLRC decision before it, not on
the correctness of the NLRC's interpretation of the provision of the MOA, the basis of whether the NLRC decision on the merits of the case was correct.
the reasonableness of the attorney's fees in question, and the application or In other words, we have to be keenly aware that the CA undertook a Rule 65
interpretation of a provision of the Labor Code on the matter are questions of review, not a review on appeal, of the NLRC decision challenged before it.
law which the CA validly inquired into in the certiorari proceedings. It argues This is the approach that should be basic in a Rule 45 review of a CA ruling
that the CA correctly ruled that the NLRC acted with grave abuse of in a labor case. In question form, the question to ask is: Did the CA correctly
discretion when it affirmed the LA's award of attorney's fees despite the determine whether the NLRC committed grave abuse of discretion in ruling
absence of a finding of any unlawful withholding of wages or bad faith on on the case?33 (italics and emphases supplied)
the part of the Company. It finally contends that the Union's demand, together
with the NLRC award, is unconscionable as it represents 20% of the amount
due or about P21.4 million. In the present case, we are therefore tasked to determine whether the CA
correctly ruled that the NLRC committed grave abuse of discretion in
awarding 10% attorney's fees to the petitioners.
Issues
The core issues posed for our resolution are: (1) whether the CA can review
the factual findings of the NLRC in a Rule 65 petition; and (2) whether the On the Award of Attorney's Fees
NLRC gravely abused its discretion in awarding ten percent (10%) attorney's
fees to the petitioners.
Article 111 of the Labor Code, as amended, governs the grant of attorney's
fees in labor cases:
The Court's Ruling

Art. 111. Attorney's fees.- (a) In cases of unlawful withholding of wages, the
We find the petition and its arguments meritorious. culpable party may be assessed attorney's fees equivalent to ten percent of
the amount of wages recovered.

On the CA's Review of the NLRC's Factual Findings


(b) It shall be unlawful for any person to demand or accept, in any judicial or
administrative proceedings for the recovery of wages, attorney's fees which
We agree with the petitioners that as a rule, the CA cannot undertake a re- exceed ten percent of the amount of wages recovered.
assessment of the evidence presented in the case in certiorari proceedings
under Rule 65 of the Rules of Court.29 However, the rule admits of
exceptions. In Mercado v. AMA Computer College-Parañaque City, Inc.,30 Section 8, Rule VIII, Book III of its Implementing Rules also provides, viz.:
we held that the CA may examine the factual findings of the NLRC to
determine whether or not its conclusions are supported by substantial
evidence, whose absence justifies a finding of grave abuse of discretion. We Section 8. Attorney's fees. â€" Attorney's fees in any judicial or
ruled: administrative proceedings for the recovery of wages shall not exceed 10%
of the amount awarded. The fees may be deducted from the total amount due
the winning party.
We agree with the petitioners that, as a rule in certiorari proceedings under
Rule 65 of the Rules of Court, the CA does not assess and weigh each piece
of evidence introduced in the case. The CA only examines the factual We explained in PCL Shipping Philippines, Inc. v. National Labor Relations
findings of the NLRC to determine whether or not the conclusions are Commission34 that there are two commonly accepted concepts of attorney's
supported by substantial evidence whose absence points to grave abuse of fees â€" the ordinary and extraordinary. In its ordinary concept, an
discretion amounting to lack or excess of jurisdiction. In the recent case of attorney's fee is the reasonable compensation paid to a lawyer by his client
Protacio v. Laya Mananghaya & Co., we emphasized that: for the legal services the former renders; compensation is paid for the cost
and/or results of legal services per agreement or as may be assessed. In its
extraordinary concept, attorney's fees are deemed indemnity for damages
As a general rule, in certiorari proceedings under Rule 65 of the Rules of ordered by the court to be paid by the losing party to the winning party. The
Court, the appellate court does not assess and weigh the sufficiency of instances when these may be awarded are enumerated in Article 2208 of the
evidence upon which the Labor Arbiter and the NLRC based their Civil Code, specifically in its paragraph 7 on actions for recovery of wages,
conclusion. The query in this proceeding is limited to the determination of and is payable not to the lawyer but to the client, unless the client and his
whether or not the NLRC acted without or in excess of its jurisdiction or with lawyer have agreed that the award shall accrue to the lawyer as additional or
grave abuse of discretion in rendering its decision. However, as an exception, part of compensation.35
the appellate court may examine and measure the factual findings of the
fixing an amount lower than the ten percent (10%) ceiling prescribed by the
We also held in PCL Shipping that Article 111 of the Labor Code, as article when circumstances warrant it.42 (emphases ours; citation omitted)
amended, contemplates the extraordinary concept of attorney's fees and that
Article 111 is an exception to the declared policy of strict construction in the
award of attorney's fees. Although an express finding of facts and law is still In the present case, the ten percent (10%) attorney's fees awarded by the
necessary to prove the merit of the award, there need not be any showing that NLRC on the basis of Article 111 of the Labor Code accrue to the Union's
the employer acted maliciously or in bad faith when it withheld the wages. members as indemnity for damages and not to the Union's counsel as
In carrying out and interpreting the Labor Code's provisions and compensation for his legal services, unless, they agreed that the award shall
implementing regulations, the employee's welfare should be the primary and be given to their counsel as additional or part of his compensation; in this
paramount consideration. This kind of interpretation gives meaning and case the Union bound itself to pay 10% attorney's fees to its counsel under
substance to the liberal and compassionate spirit of the law as embodied in the MOA and also gave up the attorney's fees awarded to the Union's
Article 4 of the Labor Code (which provides that "[a]ll doubts in the members in favor of their counsel. This is supported by Borela's affidavit
implementation and interpretation of the provisions of [the Labor Code], which stated that "[t]he 10% attorney's fees paid by the members/employees
including its implementing rules and regulations, shall be resolved in favor is separate and distinct from the obligation of the company to pay the 10%
of labor") and Article 1702 of the Civil Code (which provides that "[i]n case awarded attorney's fees which we also gave to our counsel as part of our
of doubt, all labor legislation and all labor contracts shall be construed in contingent fee agreement."43 The limit to this agreement is that the
favor of the safety and decent living for the laborer").36 indemnity for damages imposed by the NLRC on the losing party (i.e., the
Company) cannot exceed ten percent (10%).

We similarly so ruled in RTG Construction, Inc. v. Facto37 and in Ortiz v.


San Miguel Corporation.38 In RTG Construction, we specifically stated: Properly viewed from this perspective, the award cannot be taken to mean an
additional grant of attorney's fees, in violation of the ten percent (10%) limit
under Article 111 of the Labor Code since it rests on an entirely different
Settled is the rule that in actions for recovery of wages, or where an employee legal obligation than the one contracted under the MOA. Simply stated, the
was forced to litigate and, thus, incur expenses to protect his rights and attorney's fees contracted under the MOA do not refer to the amount of
interests, a monetary award by way of attorney's fees is justifiable under attorney's fees awarded by the NLRC; the MOA provision on attorney's fees
Article 111 of the Labor Code; Section 8, Rule VIII, Book III of its does not have any bearing at all to the attorney's fees awarded by the NLRC
Implementing Rules; and paragraph 7, Article 2208 of the Civil Code. The under Article 111 of the Labor Code. Based on these considerations, it is
award of attorney's fees is proper, and there need not be any showing that the clear that the CA erred in ruling that the LA's award of attorney's fees violated
employer acted maliciously or in bad faith when it withheld the wages. There the maximum limit of ten percent (10%) fixed by Article 111 of the Labor
need only be a showing that the lawful wages were not paid accordingly.39 Code.1á¢wphi1
(emphasis ours)

Under this interpretation, the Company's argument that the attorney's fees are
In PCL Shipping, we found the award of attorney's fees due and appropriate unconscionable as they represent 20% of the amount due or about P21.4
since the respondent therein incurred legal expenses after he was forced to million is more apparent than real. Since the attorney's fees awarded by the
file an action for recovery of his lawful wages and other benefits to protect LA pertained to the Union's members as indemnity for damages, it was
his rights.40 From this perspective and the above precedents, we conclude totally within their right to waive the amount and give it to their counsel as
that the CA erred in ruling that a finding of the employer's malice or bad faith part of their contingent fee agreement. Beyond the limit fixed by Article 111
in withholding wages must precede an award of attorney's fees under Article of the Labor Code, such as between the lawyer and the client, the attorney's
111 of the Labor Code. To reiterate, a plain showing that the lawful wages fees may exceed ten percent (10%) on the basis of quantum meruit, as in the
were not paid without justification is sufficient. present case.44

In the present case, we find it undisputed that the union members are entitled WHEREFORE, premises considered, the petition is hereby GRANTED. The
to their AA benefits and that these benefits were not paid by the Company. assailed decision dated March 6, 2006 and the resolution dated August 15,
That the Company had no funds is not a defense as this was not an 2006 of the Court of Appeals in CA-G.R. SP No. 83654 are REVERSED and
insuperable cause that was cited and properly invoked. As a consequence, SET ASIDE. The Labor Arbiter's award of attorney's fees equivalent to ten
the union members represented by the Union were compelled to litigate and percent (10%) of the total judgment award is hereby REINSTATED.
incur legal expenses. On these bases, we find no difficulty in upholding the
NLRC's award of ten percent (10%) attorney's fees.
No pronouncement as to costs.

The more significant issue in this case is the effect of the MOA provision that
attorney's fees shall be deducted from the AA and CBA receivables. In this SO ORDERED.
regard, the CA held that the additional grant of 10% attorney's fees by the
NLRC violates Article 111 of the Labor Code, considering that the MOA
between the parties already ensured the payment of 10% attorney's fees
deductible from the AA and CBA receivables of the Union's members. In
addition, the Company also argues that the Union's demand, together with
the NLRC award, is unconscionable as it represents 20% of the amount due
or about P21.4 million.

In Traders Royal Bank Employees Union-Independent v. NLRC,41 we


expounded on the concept of attorney's fees in the context of Article 111 of
the Labor Code, as follows:

In the first place, the fees mentioned here are the extraordinary attorney's fees
recoverable as indemnity for damages sustained by and payable to the
prevailing part[y]. In the second place, the ten percent (10%) attorney's fees
provided for in Article 111 of the Labor Code and Section 11, Rule VIII,
Book III of the Implementing Rules is the maximum of the award that may
thus be granted. Article 111 thus fixes only the limit on the amount of
attorney's fees the victorious party may recover in any judicial or
administrative proceedings and it does not even prevent the NLRC from
GABRIEL vs. THE HONORABLE SECRETARY OF LABOR AND that the workers through their union should be made to shoulder the expenses
EMPLOYMENT incurred for the attorney's services. Accordingly, the reimbursement should
G.R. No. 115949 | 2000-03-16 be charged to the union's general fund/account.8 [Id. at 22-24.]
QUISUMBING, J.:
Hence, the present petition seeking to partially annul the above-cited order
Before us is a special civil action for certiorari seeking to reverse partially of the public respondent for being allegedly tainted with grave abuse of
the Order1 [Rollo, pp. 22-24.] of public respondent dated June 3, 1994, in discretion amounting to lack of jurisdiction.
Case No. OS-MA-A-8-170-92, which ruled that the workers through their
union should be made to shoulder the expenses incurred for the professional The sole issue for consideration is, did the public respondent act with grave
services of a lawyer in connection with the collective bargaining negotiations abuse of discretion in issuing the challenged order?
and that the reimbursement for the deductions from the workers should be
charged to the union's general fund or account. Petitioners argue that the General Membership Resolution authorizing the
bank to check-off attorney's fee from the first lump sum payment of the
The records show the following factual antecedents: benefits to the employees under the new CBA satisfies the legal requirements
for such assessment.9 [Id. at 18-19.] Private respondents, on the other hand,
Petitioners comprise the Executive Board of the SolidBank Union, the duly claim that the check-off provision in question is illegal because it was never
recognized collective bargaining agent for the rank and file employees of submitted for approval at a general membership meeting called for the
Solid Bank Corporation. Private respondents are members of said union. purpose and that it failed to meet the formalities mandated by the Labor
Code.10 [Id. at 496-499.]
Sometime in October 1991, the union's Executive Board decided to retain
anew the service of Atty. Ignacio P. Lacsina (now deceased) as union counsel In check-off, the employer, on agreement with the Union, or on prior
in connection with the negotiations for a new Collective Bargaining authorization from employees, deducts union dues or agency fees from the
Agreement (CBA). Accordingly, on October 19, 1991, the board called a latter's wages and remits them directly to the union.11 [Holy Cross of Davao
general membership meeting for the purpose. At the said meeting, the College, Inc. vs. Joaquin, 263 SCRA 358-359 (1996).] It assures continuous
majority of all union members approved and signed a resolution confirming funding for the labor organization. As this Court has acknowledged, the
the decision of the executive board to engage the services of Atty. Lacsina as system of check-off is primarily for the benefit of the union and only
union counsel. indirectly for the individual employees.12 [Ibid.]

As approved, the resolution provided that ten percent (10%) of the total The pertinent legal provisions on check-offs are found in Article 222 (b) and
economic benefits that may be secured through the negotiations be given to Article 241 (o) of the Labor Code.
Atty. Lacsina as attorney's fees. It also contained an authorization for
SolidBank Corporation to check-off said attorney's fees from the first lump Article 222 (b) states:
sum payment of benefits to the employees under the new CBA and to turn
over said amount to Atty. Lacsina and/or his duly authorized representative.2 "No attorney's fees, negotiation fees or similar charges of any kind arising
[Id. at 25.] from any collective bargaining negotiations or conclusions of the collective
agreement shall be imposed on any individual member of the contracting
The new CBA was signed on February 21, 1992. The bank then, on request union: Provided, however, that attorney's fees may be charged against union
of the union, made payroll deductions for attorney's fees from the CBA funds in an amount to be agreed upon by the parties. Any contract, agreement
benefits paid to the union members in accordance with the abovementioned or arrangement of any sort to the contrary shall be null and void."
resolution.
Article 241 (o) provides:
On October 2, 1992, private respondents instituted a complaint against the
petitioners and the union counsel before the Department of Labor and "Other than for mandatory activities under the Code, no special assessment,
Employment (DOLE) for illegal deduction of attorney's fees as well as for attorney's fees, negotiation fees or any other extraordinary fees may be
quantification of the benefits in the 1992 CBA.3 [Id. at 26-29.] Petitioners, checked off from any amount due to an employee without an individual
in response, moved for the dismissal of the complaint citing litis pendentia, written authorization duly signed by the employee. The authorization should
forum shopping and failure to state a cause of action as their grounds.4 [Id. specifically state the amount, purpose and beneficiary of the deduction."
at 30-36.]
Article 241 has three (3) requisites for the validity of the special assessment
On April 22, 1993, Med-Arbiter Paterno Adap of the DOLE- NCR issued the for union's incidental expenses, attorney's fees and representation expenses.
following Order: These are: 1) authorization by a written resolution of the majority of all the
members at the general membership meeting called for the purpose; (2)
"WHEREFORE, premises considered, the Respondents Union Officers and secretary's record of the minutes of the meeting; and (3) individual written
Counsel are hereby directed to immediately return or refund to the authorization for check off duly signed by the employees concerned.
Complainants the illegally deducted amount of attorney's fees from the
package of benefits due herein complainants under the aforesaid new CBA. Clearly, attorney's fees may not be deducted or checked off from any amount
due to an employee without his written consent.
"Furthermore, Complainants are directed to pay five percent (5%) of the total
amount to be refunded or returned by the Respondent Union Officers and After a thorough review of the records, we find that the General Membership
Counsel to them in favor of Atty. Armando D. Morales, as attorney's fees, in Resolution of October 19, 1991 of the SolidBank Union did not satisfy the
accordance with Section II, Rule VIII of Book II (sic) of the Omnibus Rules requirements laid down by law and jurisprudence for the validity of the ten
Implementing the Labor Code."5 [Id. at 11, 201.] percent (10%) special assessment for union's incidental expenses, attorney's
fees and representation expenses. There were no individual written check off
On appeal, the Secretary of Labor rendered a Resolution6 [Id. at 201-209.] authorizations by the employees concerned and so the assessment cannot be
dated December 27, 1993, stating: legally deducted by their employer.

"WHEREFORE, the appeal of respondents Evangeline Gabriel, et. al., is Even as early as February 1990, in the case of Palacol vs. Ferrer-Calleja13
hereby partially granted and the Order of the Med-Arbiter dated 22 April [182 SCRA 710-711 (1990).] we said that the express consent of employees
1993 is hereby modified as follows: (1) that the ordered refund shall be is required, and this consent must be obtained in accordance with the steps
limited to those union members who have not signified their conformity to outlined by law, which must be followed to the letter. No shortcuts are
the check-off of attorney's fees; and (2) the directive on the payment of 5% allowed. In Stellar Industrial Services, Inc. vs. NLRC14 [252 SCRA 323, 325
attorney's fees should be deleted for lack of basis. (1996).] we reiterated that a written individual authorization duly signed by
the employee concerned is a condition sine qua non for such deduction.
SO ORDERED."7 [Id. at 209.]
These pronouncements are also in accord with the recent ruling of this Court
On Motion for Reconsideration, public respondent affirmed the said Order in the case of ABS-CBN Supervisors Employees Union Members vs. ABS-
with modification that the union's counsel be dropped as a party litigant and
CBN Broadcasting Corporation, et. al.,15 [G.R. No. 106518, March 11,
1999, p. 15.15 which provides:

"Premises studiedly considered, we are of the irresistible conclusion and, so


find that the ruling in BPIEU-ALU vs. NLRC that (1) the prohibition against
attorney's fees in Article 222, paragraph (b) of the Labor Code applies only
when the payment of attorney's fees is effected through forced contributions
from the workers; and (2) that no deduction must be take from the workers
who did not sign the check-off authorization, applies to the case under
consideration."

We likewise ruled in Bank of the Philippine Island Employees Union-


Association Labor Union (BPIEU-ALU) vs. NLRC,16 [171 SCRA 556, 569
(1989).]

"- the afore-cited provision (Article 222 (b) of the Labor Code) as prohibiting
the payment of attorney's fees only when it is effected through forced
contributions from workers from their own funds as distinguished from the
union funds. The purpose of the provision is to prevent imposition on the
workers of the duty to individually contribute their respective shares in the
fee to be paid the attorney for his services on behalf of the union in its
negotiations with management. The obligation to pay the attorney's fees
belongs to the union and cannot be shunted to the workers as their direct
responsibility. Neither the lawyer nor the union itself may require the
individual worker to assume the obligation to pay attorney's fees from their
own pockets. So categorical is this intent that the law makes it clear that any
agreement to the contrary shall be null and void ab initio."

From all the foregoing, we are of the considered view that public respondent
did not act with grave abuse of discretion in ruling that the workers through
their union should be made to shoulder the expenses incurred for the services
of a lawyer. And accordingly the reimbursement should be charged to the
union's general fund or account. No deduction can be made from the salaries
of the concerned employees other than those mandated by law.

WHEREFORE, the petition is DENIED. The assailed Order dated June 3,


1994, of respondent Secretary of Labor signed by Undersecretary Bienvenido
E. Laguesma is AFFIRMED. No pronouncement as to costs.

SO ORDERED.
MARIÑO, JR. VS. GAMILLA 2.3. In the case of a covered faculty member whose compensation is
G.R. No. 149763 | 2009-07-07 computed on a basis other than lecture unit per month, he shall receive salary
CHICO-NAZARIO, J.: increases that are equivalent to those provided in paragraphs 2.1 and 2.2
hereof, with the amount of salary increases being arrived at by using the usual
Assailed in this Petition for Review on Certiorari,[1] under Rule 45 of the method of computing the said faculty member's basic pay;
Rules of Court, are (1) the Decision[2] dated 16 March 2001 of the Court of
Appeals in CA-G.R. SP No. 60657, dismissing petitioners' Petition for 3.0. The UNIVERSITY shall likewise restore to the faculty members the
Certiorari under Rule 65 of the Rules of Court; and (2) the Resolution[3] amounts corresponding to the deductions in salary that were taken from the
dated 30 August 2001 of the appellate court in the same case denying pay checks in the second half of June, 1989 and in the first half of July, 1989,
petitioners' Motion for Reconsideration. provided that said deductions in salary relate to the union activities that were
held in the aforestated payroll periods, and provided further that the amounts
I involved shall be taken from the P42 Million (sic) economic package.

FACTS 4.0. A portion of the P42,000,000.00 economic package amounting to


P2,000,000.00 shall be used to satisfy all obligations that remained
The Petition at bar arose from the following factual and procedural outstanding and unpaid in the May 17, 1986 Collective Bargaining
antecedents. Agreement.

(1) Case No. NCR-OD-M-9412-022 5.0. Any unspent balance of the aggregate of P42,000,000.00 as of October
15, 1992, shall, within two weeks, be remitted to the Union[:]
At the time when the numerous controversies in the instant case first came
about, petitioners Atty. Eduardo J. Mariño, Jr., Ma. Melvyn P. Alamis, 5.1. The unspent balance mentioned in paragraph 5.0 inclusive of earnings
Norma P. Collantes, and Fernando Pedrosa were among the executive but exclusive of check-offs, shall be used for the salary increases herein
officers and directors (collectively called the Mariño Group) of the granted up to May 31, 1993, for increases in hospitalization, educational and
University of Sto. Tomas Faculty Union (USTFU), a labor union duly retirement benefits, and for other economic benefits.
organized and registered under the laws of the Republic of the Philippines
and the bargaining representative of the faculty members of the University 6.0. The benefits herein granted constitute the entire and complete package
of Santo Tomas (UST).[4] of economic benefits granted by the UNIVERSITY to the covered faculty
members for the balance of the term of the existing collective bargaining
Respondents Gil Y. Gamilla, Rene Luis Tadle, Norma S. Calaguas, Ma. agreement.
Lourdes C. Medina, Edna B. Sanchez, Remedios Garcia, Mafel Ysrael, Zaida
Gamilla, and Aurora Domingo were UST professors and USTFU members. 7.0. It is clearly understood and agreed upon that the aggregate sum of P42
million is chargeable against the share of the faculty members in the
The 1986 Collective Bargaining Agreement (CBA) between UST and incremental proceeds of tuition fees collected and still to be collected;
USTFU expired on 31 May 1988. Thereafter, bargaining negotiations ensued Provided, however, that he (sic) commitment of the UNIVERSITY to pay
between UST and the Mariño Group, which represented USTFU. As the the aggregate sum of P42 million shall subsist even if the said amount
parties were not able to reach an agreement despite their earnest efforts, a exceeds the proportionate share that may accrue to the faculty members in
bargaining deadlock was declared and USTFU filed a notice of strike. the tuition fee increases that the UNIVERSITY may be authorized to collect
Subsequently, then Secretary of the Department of Labor and Employment in School-Year 1992-1993, and, Provided, finally, that the covered faculty
(DOLE) Franklin Drilon assumed jurisdiction over the dispute, which was members shall still be entitled to their proportionate share in any
docketed as NCMB-NCR-NS-02-117-89. The DOLE Secretary issued an undistributed portion of the incremental proceeds of the tuition fee increases
Order on 19 October 1990, laying the terms and conditions for a new CBA in School-Year 1992-1993, and incremental proceeds are, by law and
between the UST and USTFU. In accordance with said Order, the UST and pertinent Department of Education Culture and Sports (DECS) regulations,
USTFU entered into a CBA in 1991, which was to be effective for the period required to be allotted for the payment of salaries, wages, allowances and
of 1 June 1988 to 31 May 1993 (hereinafter 1988-1993 CBA). In keeping other benefits of teaching and non-teaching personnel for the UNIVERSITY.
with Article 253-A[5] of the Labor Code, as amended, the economic
provisions of the 1988-1993 CBA were subject to renegotiation for the fourth 8.0. With this Agreement, the parties confirm that[:]
and fifth years.
8.1. the University has complied with the requirements of the law relative to
Accordingly, on 10 September 1992, UST and USTFU executed a the release and distribution of the incremental proceeds of tuition fee
Memorandum of Agreement (MOA),[6] whereby UST faculty members increases as these incremental proceeds pertain to the faculty share in the
belonging to the collective bargaining unit were granted additional economic tuition fee increase collected during the School-Year 1991-1992; and,
benefits for the fourth and fifth years of the 1988-1993 CBA, specifically,
the period from 1 June 1992 up to 31 May 1993. The relevant portions of the 8.2. the economic benefits herein granted constitute the full and complete
MOA read: financial obligation of the UNIVERSITY to the members of its faculty for
the period June 1, 1991 to May 31, 1993, pursuant to the provisions of the
existing Collective Bargaining Agreement.
MEMORANDUM OF AGREEMENT
9.0. Subject to the provisions of law, and without reducing the amounts of
xxxx salary increases granted under paragraphs 2.0, 2.1, 2.2 and 2.3[,] the UNION
shall have the right to a pro-rata lump sum check-off of all sums of money
1.0. The University hereby grants additional benefits to Faculty Members due and payable to it from the package of economic benefits granted under
belonging to the collective bargaining unit as defined in Article I, Section 1 this Agreement, provided that there is an authorization of a majority of the
of the Collective Bargaining Agreement entered into between the parties members of the UNION and provided, further, that the P42 million economic
herein over and above the benefits now enjoyed by the said faculty members, package herein granted shall not in any way be exceeded.
which additional benefits shall amount in the aggregate to P42,000,000.00[.]
10.0. This Agreement shall be effective for a period of two (2) years, starting
2.0. Under this Agreement the University shall grant salary increases, to wit: June 1, 1991 and ending on May 31, 1993, provided, however, that if for any
reason no new collective bargaining agreement is entered into at the
2.1. THIRTY (P30.00) PESOS per lecture unit per month to covered faculty expiration date hereof, this Agreement, together with the March 18, 1991
members retroactive to June 1, 1991; Collective Bargaining Agreement, shall remain in full force and effect until
such time as a new collective bargaining agreement shall have been executed
2.2. Additional THIRTY (P30.00) PESOS per lecture unit per month on top by the parties.
of the salary increase granted in [paragraph] 2.1 hereof to the said faculty
members effective June 1, 1992; xxxx

UNIVERSITY OF SANTO TOMAS


UST FACULTY UNION On 15 December 1994, respondents[10] filed with the Med-Arbiter, DOLE-
BY: National Capital Region (NCR), a Complaint for the expulsion of the Mariño
BY: Group as USTFU officers and directors, which was docketed as Case No.
(signed) NCR-OD-M-9412-022.[11] Respondents alleged in their Complaint that the
(signed) Mariño Group violated the rights and conditions of membership in USTFU,
FR. TERESO M. CAMPILLO, JR., O.P. particularly by: 1) investing the unspent balance of the P42 million economic
ATTY. EDUARDO J. MARINO, JR. benefits package given by UST without prior approval of the general
Treasurer membership; 2) simultaneously holding electionsviva voce; 3) ratifying the
President CBA involving the P42 million economic benefits package; and 4) approving
the attorney's/agency fees worth P4.2 million in the form of check-off.
Attested by[:] Respondents prayed that the Mariño Group be declared jointly and severally
liable for refunding all collected attorney's/agency fees from individual
(signed) members of USTFU and the collective bargaining unit; and that, after due
REV. FR. ROLANDO DELA ROSA, O.P. (Emphasis ours.) hearing, the Mariño group be expelled as USTFU officers and directors.

(2) Case No. NCR-OD-M-9510-028


On 12 September 1992, the majority of USTFU members signed individual
instruments of ratification,[7] which purportedly signified their consent to On 16 December 1994, UST and USTFU, represented by the Mariño Group,
the economic benefits granted under the MOA. Said instruments uniformly entered into a new CBA, effective 1 June 1993 to 31 May 1998 (1993-1998
recited: CBA). This new CBA was registered with the DOLE on 20 February 1995.

Respondents[12] filed with the Med-Arbiter, DOLE-NCR, on 18 October


RATIFICATION OF THE UST-USTFU MEMORANDUM OF 1995, another Complaint against the Mariño Group for violation of the rights
AGREEMENT DATED SEPTEMBER 10, 1992 GRANTING A and conditions of union membership, which was docketed as Case No. NCR-
PACKAGE OF THE P42 MILLION FACULTY BENEFITS WITH OD-M-9510-028.[13] The Complaint primarily sought to invalidate certain
PROVISION FOR CHECK-OFF. provisions of the 1993-1998 CBA negotiated by the Mariño Group for
USTFU and the registration of said CBA with the DOLE.
September 12, 1992
Date (3) Case No. NCR-OD-M-9610-001

TO WHOM IT MAY CONCERN: On 24 September 1996, petitioner Norma Collantes, as USTFU Secretary-
General, posted notices in some faculty rooms at UST, informing the union
I, the undersigned UST faculty member, aware that the law requires members of a general assembly to be held on 5 October 1996. Part of the
ratification and that without ratification by majority of all faculty members agenda for said date was the election of new USTFU officers. The following
belonging to the collective bargaining unit, the Memorandum of Agreement day, 25 September 1996, respondents wrote a letter[14] to the USTFU
between the University of Santo Tomas and the UST Faculty Union (or Committee on Elections, urging the latter to re-schedule the elections to
USTFU) dated September 10, 1992 may be questioned and all the faculty ensure a free, clean, honest, and orderly election and to afford the union
benefits granted therein may be cancelled, do hereby ratify the said members the time to prepare themselves for the same. The USTFU
agreement. Committee on Elections failed to act positively on respondents' letter, and
neither did they adopt and promulgate the rules and regulations for the
Under the Agreement, the University shall pay P42 million over a period of conduct of the scheduled election.
two (2) years from June 1, 1991 up to May 31, 1992.
Thus, on 1 October 1996, respondents[15] filed with the Med-Arbiter,
In consideration of the efforts of the UST Faculty Union as the faculty DOLE-NCR, an Urgent Ex-PartePetition/Complaint, which was docketed as
members' sole and exclusive collective bargaining representative in Case No. NCR-OD-M-9610-001.[16] Respondents alleged in their
obtaining the said P42 million package of economic benefits, a check-off of Petition/Complaint that the general membership meeting called by the
ten percent thereof covering union dues, and special assessment for Labor USTFU Board of Directors on 5 October 1996, the agenda of which included
Education Fund and attorney's fees from USTFU members and agency fee the election of union officers, was in violation of the provisions of the
from non-members for the period of the Agreement is hereby authorized to Constitution and By-Laws of USTFU. Respondents prayed that the DOLE
be made in one lump sum effective immediately, provided that two per cent supervise the conduct of the USTFU elections, and that they be awarded
(sic) shall be for [the] administration of the Agreement and the balance of attorney's fees.
eight per cent (sic) shall be for attorney's fees to be donated, as pledged by
the USTFU lawyer to the Philippine Foundation for the Advancement of the On 4 October 1996, the Med-Arbiter DOLE-NCR, issued a Temporary
Teaching Profession, Inc. whose principal purpose is the advancement of the Restraining Order (TRO) enjoining the holding of the USTFU elections
teaching profession and teacher's welfare, and provided further that the scheduled the next day.
deductions shall not be taken from my individual monthly salary but from
the total package of P42 million due under the Agreement. (4) Case No. NCR-OD-M-9610-016

Also on 4 October 1996, the UST Secretary General headed a general faculty
_________________________ assembly attended by USTFU members, as well as USTFU non-members,
Signature of Faculty Member (Emphasis ours.) but who were members of the collective bargaining unit. During said
assembly, respondents were among the elected officers of USTFU
(collectively referred to as the Gamilla Group). Petitioners filed with the
USTFU, through its President, petitioner Atty. Mariño, wrote a letter[8] dated Med-Arbiter, DOLE-NCR, a Petition seeking injunctive reliefs and the
1 October 1992 to the UST Treasurer requesting the release to the union of nullification of the results of the 4 October 1994 election. The Petition was
the sum of P4.2 million, which was 10% of the P42 million economic benefits docketed as Case No. NCR-OD-M-9610-016.
package granted by the MOA to faculty members belonging to the collective
bargaining unit. The P4.2 million was sought by USTFU in consideration of In a Decision dated 11 February 1997 in Case No. NCR-OD-M-9610-016,
its efforts in obtaining the said P42 million economic benefits package. UST the Med-Arbiter DOLE-NCR, nullified the election of the Gamilla Group as
remitted the sum of P4.2 million to USTFU on 9 October 1992.[9] USTFU officers on 4 October 1996 for having been conducted in violation
of the Constitution and By-Laws of the union. This ruling of the Med-Arbiter
After deducting from the P42 million economic benefits package the P4.2 was affirmed on appeal by the Bureau of Labor Relations (BLR) in a
million check-off to USTFU, the amounts owed to UST, and the salary Resolution issued on 15 August 1997. Respondents were, thus, prompted to
increases and bonuses of the covered faculty members, a net amount of file a Petition forCertiorari before this Court, docketed as G.R. No. 131235.
P6,389,145.04 remained. The remaining amount was distributed to the
faculty members on 18 November 1994.
While G.R. No. 131235 was pending, the term of office of the Gamilla Group WHEREFORE, premises considered, judgment is hereby rendered:
as USTFU officers expired on 4 October 1999. The Gamilla Group then
scheduled the next election of USTFU officers on 14 January 2000. a) Expelling [the Mariño Group] from their positions as officers of USTFU,
and hereby order them under pain of contempt, to cease and desist from
On 16 November 1999, the Court promulgated its Decision in G.R. No. performing acts as such officers;
131235, affirming the BLR Resolution dated 15 August 1997 which ruled
that the purported election of USTFU officers held on 4 October 1996 was b) Ordering [the Mariño Group] to jointly and severally refund to USTFU
void for violating the Constitution and By-Laws of the union.[17] the amount of P4.2 M checked-off as attorney's fees from the P42 M
economic package;
(5) Case No. NCR-OD-M-9611-009
c) Ordering [the Mariño Group] to account for:
On 15 November 1996, respondents[18] filed before the Med-Arbiter,
DOLE-NCR, a fourth Complaint/Petition against the Mariño Group, as well c.1. P2.0 M paid to USTFU in satisfaction of the remaining obligation of the
as the Philippine Foundation for the Advancement of the Teaching University under the 1986 CBA;
Profession, Inc., Security Bank Corporation, and Bank of the Philippine
Islands, which was docketed as Case No. NCR-OD-M-9611- c.2. P7.0 M as consideration of the Compromise Agreement entered into by
009.[19]Respondents claimed in their latest Complaint/Petition that they USTFU involving certain labor cases;
were the legitimate USTFU officers, having been elected on 4 October 1996.
They prayed for an order directing the Mariño Group to cease and desist from c.3. Interest/earnings of the P9,766,570.01 balance of the P42 M
using the name of USTFU and from performing acts for and on behalf of the invested/deposited by [the Mariño Group] with the PCI Capital Corporation.
USTFU and the rest of the members of the collective bargaining unit.
d) Ordering conduct of election of Union officers under the supervision of
DOLE Department Order No. 9 took effect on 21 June 1997, amending the this Department.[25]
Rules Implementing Book V of the Labor Code, as amended. Thereunder,
jurisdiction over the complaints for any violation of the union constitution Petitioners interposed an appeal[26] before the BLR, which was docketed as
and by-laws and the conditions of union membership was vested in the BLR-A-TR-52-25-10-99.
Regional Director of the DOLE.[20] Pursuant to said Department Order, all
four Petitions/Complaints filed by respondents against the Mariño Group, In the meantime, the election of USTFU officers was held as scheduled on
particularly, Case No. NCR-OD-M-9412-022, Case No. NCR-OD-M-9510- 14 January 2000,[27] in which the Gamilla Group claimed victory.[28] On 3
028, Case No. NCR-OD-M-9610-001, and Case No. NCR-OD-M-9611-009 March 2000, the Gamilla group, as the new USTFU officers, entered into a
were consolidated and indorsed to the Office of the Regional Director of the Memorandum of Agreement[29] with the UST, which provided for the
DOLE-NCR. economic benefits to be granted to the faculty members of the UST for the
years 1999-2001. Said Agreement was ratified by the USTFU members on 9
On 27 May 1999, the DOLE-NCR Regional Director rendered a March 2000.
Decision[21] in the consolidated cases in respondents' favor.
On the same day, 9 March 2000, the BLR promulgated its Decision[30] in
In Case No. NCR-OD-M-9412-022 and Case No. NCR-OD-M-9510-028, BLR-A-TR-52-25-10-99, the fallo of which provides:
the DOLE-NCR Regional Director adjudged the Mariño Group, as the
executive officers of USTFU, guilty of violating the provisions of the WHEREFORE, the appeal is GRANTED IN PART. Accordingly, the
USTFU Constitution and By-laws by failing to collect union dues and to decision appealed from is hereby MODIFIED to the effect that appellant
conduct a general assembly every three months. The DOLE-NCR Regional USTFU officers are hereby ordered to return to the general membership the
Director also ruled that the Mariño Group violated Article 241(c)[22] and amount of P4.2 million they have collected by way of attorney's fees.
(l)[23] of the Labor Code when they did not submit a list of union officers to
the DOLE; when they did not submit/provide DOLE and the USTFU Let the entire records of this case be remanded to the Regional Office of
members with copies of the audited financial statements of the union; and origin for the immediate conduct of election of officers of USTFU. The
when they invested in a bank, without prior consent of USTFU members, the election shall be held under the control and supervision of the Regional
sum of P9,766,570.01, which formed part of the P42 million economic Office, in accordance with Section 1 (b), Rule XV of Department Order No.
benefits package. 9, unless the parties mutually agree to a different procedure consistent with
ensuring integrity and fairness in the electoral exercise.
Additionally, the DOLE-NCR Regional Director declared that the check-off
of P4.2 million collected by the Mariño Group, as negotiation fees, was The BLR found no basis for the order of the DOLE-NCR Regional Director
invalid. According to the MOA executed on 10 September 1992 by UST and to the Mariño Group to account for the amounts of P2 million and P7 million
USTFU, the P42 million economic benefits package was chargeable against supposedly paid by UST to USTFU. The BLR clarified that UST paid
the share of the faculty members in the incremental proceeds of tuition fees USTFU a lump sum of P7 million. The P2 million of this lump sum was the
collected and still to be collected. Under Republic Act No. 6728,[24] 70% of payment by UST of its outstanding obligations to USTFU under the 1986
the tuition fee increases should be allotted to academic and non-academic CBA. This amount was subsequently donated by USTFU members to the
personnel. Given that the records were silent as to how much of the P42 Philippine Foundation for the Advancement of the Teaching Profession, Inc.
million economic benefits package was obtained through negotiations and The remaining P5 million of the lump sum was the consideration for the
how much was from the statutory allotment of 70% of the tuition fee settlement of an illegal dismissal case between UST and the Mariño Group.
increases, the DOLE-NCR Regional Director held that the entire amount was Hence, the P5 million legally belonged to the Mariño Group, and there was
within the statutory allotment, which could not be the subject of negotiation no need to make it account for the same. As to the interest earnings of the
and, thus, could not be burdened by negotiation fees. sum of P9,766,570.01 that was invested by the Mariño Group in a bank, the
BLR ruled that the same was included in the amount of P6,389,145.04 that
The DOLE-NCR Regional Director further found that the principal subject was distributed to the faculty members on 18 November 1994.
of Case No. NCR-OD-M-9610-001 (i.e., violation by the Mariño Group of
the provisions on election of officers in the Labor Code and the USTFU The BLR, however, agreed in the finding of the DOLE-NCR Regional
Constitution and By-Laws) had been superseded by the central event in Case Director that the P42 million economic benefits package was sourced from
No. NCR-OD-M-9611-009 (i.e., the subsequent election of another set of the faculty members' share in the tuition fee increases under Republic Act
USTFU officers consisting of the Gamilla Group). While there were two sets No. 6728. Under said law, 70% of tuition fee increases shall go to the
of USTFU officers vying for legitimacy, the eventual ruling of the DOLE- payment of salaries, wages, allowances, and other benefits of teaching and
NCR Regional Director, for the expulsion of the Mariño Group from their non-teaching personnel. As was held in the decision[31] and subsequent
positions as USTFU officers, practically extinguished Case No. NCR-OD- resolution[32] of the Supreme Court in Cebu Institute of Technology v. Ople,
M-9611-009. the law has already provided for the minimum percentage of tuition fee
increases to be allotted for teachers and other school personnel. This
The decretal portion of the 27 May 1999 Decision of the DOLE-NCR allotment is mandatory and cannot be diminished, although it may be
Regional Director reads: increased by collective bargaining. It follows that only the amount beyond
that mandated by law shall be subject to negotiation fees and attorney's fees
for the simple reason that it was only this amount that the school employees Moreover, [Section 5 of Rule X of] the CBL of the Union provides that:
had to bargain for.
Section 5. Special assessments or other extraordinary fees such as for
The BLR further reasoned that the P4.2 million collected by the Mariño payment of attorney's fees shall be made only upon such a resolution duly
Group was in the nature of attorney's fees or negotiation fees and, therefore, ratified by the general membership by secret balloting. x x x.
fell under the general prohibition against such fees in Article 222(b)[33] of
the Labor Code, as amended. Also, the exception to charging against union Also, Article 241(n)[37] of the Labor Code, as amended, provides that no
funds was not applicable because the P42 million economic benefits package special assessment shall be levied upon the members of the union unless
under the 10 September 1992 MOA was not union fund, as the same was authorized by a written resolution of a majority of all the members at a
intended not for the union coffers, but for the members of the entire general membership meeting duly called for the purpose[.]
bargaining unit. The fact that the P4.2 million check-off was approved by the
majority of USTFU members was immaterial in view of the clear command xxxx
of Article 222(b) that any contract, agreement, or arrangement of any sort,
contrary to the prohibition contained therein, shall be null and void. In "ABS-CBN Supervisors-Employees Union Members versus ABS-CBN
Broadcasting Corporation, 304 SCRA 489", our Supreme Court declared that
Lastly, as to the alleged failure of the Mariño Group to perform some of its Article 241(n) of the Labor Code, as amended, speaks of three (3) requisites,
duties, the BLR held that the change of USTFU officers can best be decided, to wit: (1) authorization by a written resolution of the majority of all members
not by outright expulsion, but by the general membership through the actual at the general membership meeting called for the purpose; (2) secretary's
conduct of elections. record of the minutes of the meeting; and (3) individual written authorization
for check-off duly signed by the employee concerned.
Petitioners' Motion for Partial Reconsideration[34] of the foregoing Decision
was denied by the BLR in a Resolution[35]dated 13 June 2000. Contrary to the provisions of Articles 222(b) and 241(n) of the Labor Code,
as amended, and Section 5, Rule X of [the] CBL of the Union, no resolution
Aggrieved once again, petitioners filed with the Court of Appeals a Petition ratified by the general membership of [the] USTFU through secret balloting
for Certiorari[36] under Rule 65 of the Rules of Court, which was docketed which embodied the award of attorney's fees was submitted. Instead, the
as CA-G.R. SP No. 60657. In a Resolution dated 26 September 2000, the Petitioners submitted copies of the form for the ratification of the MOA and
Court of Appeals directed respondents to file their Comment; and, in order the check-off for attorney's fees.
not to render moot and academic the issues in the Petition, enjoined
respondents and all those acting for and on their behalf from enforcing, xxxx
implementing, and effecting the BLR Decision dated 9 March 2000.
The aforementioned "ratification with check-off" form embodied the: (a)
On 16 March 2001, the Court of Appeals rendered its Decision in CA-G.R. ratification of the MOA; (b) check-off of union dues; and (c) check-off of a
SP No. 60657, favoring respondents. special assessment, i.e., attorney's fees and labor education fund. x x x.
Patently, the CBL was not complied with.
According to the Court of Appeals, the BLR did not commit grave abuse of
discretion, amounting to lack or excess of jurisdiction, in ruling that the P42 Worse, the check-off for union dues and attorney's fees were included in the
million economic benefits package was merely the share of the faculty ratification of the MOA. The members were thus placed in a situation where,
members in the tuition fee increases pursuant to Republic Act No. 6728. The upon ratification of the MOA, not only the check-off of union dues and
appellate court explained: special assessment for labor education fund but also the payment of attorney's
fees were (sic) authorized.[38]
It is too plain to see that the 60% of the proceeds is to be allocated specifically
for increase in salaries or wages of the members of the faculty and all other In like manner, the Court of Appeals found no grave abuse of discretion,
employees of the school concerned. Under Section 5(2) of Republic Act amounting to lack or excess of jurisdiction, on the part of the BLR in ordering
6728, the amount had been increased to 70% of the tuition fee increases the conduct of elections under the control and supervision of the DOLE-
which was specifically allocated to the payment of salaries, wages, NCR. Said the appellate court:
allowances and other benefits of teaching and non-teaching personnel of the
school[,] except administrators who are principal stockholders of the school We agree with the Petitioners that the elections of officers of the Union,
and to cover increases as provided for in the collective bargaining agreements before the Decision of the [BLR], had been unfettered by any intervention of
existing or in force at the time the law became effective[.] the DOLE. However, We agree with the Decision of the [BLR] for two (2)
specific reasons, namely: (a) the parties are given an opportunity to first agree
xxxx on a different procedure to ensure the integrity and fairness of the electoral
exercise, before the DOLE, may supervise the election[.]
It is too plain to see, too, that under the "Memorandum of Agreement"
between UST and the Union, x x x, the P42,000,000.00 economic package xxxx
granted by the UST to the Union was in compliance with the mandates of the
law and pertinent Department of Education, Culture and Sports regulation Under Article IX of the CBL, the Board of Officers of the Union shall create
(sic) required to be allotted following the payment of salaries, wages, a Committee on Elections, Comelec for brevity, composed of a chairman and
allowances and other benefits of teaching and non-teaching personnel of the two (2) members appointed by the Board of Officers[.]
University[.]
xxxx
xxxx
It, however, appears that the term of office of the Petitioners had already
Whether or not UST implemented the mandate of Republic Act 6728 expired in September of 1996. In fact, an election of officers was scheduled
voluntarily or through the efforts and prodding of the Union does not and on October 6, 1996. However, on October 4, 1996, [respondents] and the
cannot change or alter a whit the nature of the economic package or the members of the faculty of UST, both union member and non-union member,
purpose or purposes of the allocation of the said amount. For, if we elected [respondents] as the new officers of the USTFU. The same was,
acquiesced to and sustained Petitioners' stance, we will thereby be leaving however, (sic) nullified by the Supreme Court, on November 16, 1999.
the compliance by the private educational institutions of the mandate of However, as the term of office of the [respondents] had expired, on October
Republic Act 6728 at the will, mercy, whims and caprices of the Union and 4, 1999, there is nothing to nullify anymore. By virtue of an election, held on
the private educational institution. This cannot and should not come to pass. January 14, 2000, the [respondents] were elected as the new officers of the
Union, which election was not contested by the Petitioners or any other group
With our foregoing findings and disquisitions, We thus agree with the [BLR] in the union.
that the aforesaid amount of P42,000,000.00 should not answer for any
attorney's fees claimed by the Petitioners. x x x. xxxx

xxxx We are thus faced with a situation where one set of officers claim to be the
legitimate and incumbent officers of the Union, pursuant to the CBL of the
Union, and another set of officers who claim to have been elected by the to conduct election of union officers under the control and supervision of the
members of the faculty of the Union thru an election alleged to have been DOLE-NCR Regional Director.
supervised by the DOLE which situation partakes of and is akin to the nature
of an intra-union dispute[.] x x x. II

Undeniably, the CBL gives the Board of Officers the right to create and RULING
appoint members of the Comelec. However, the CBL has no application to a
situation where there are two (2) sets of officers, one set claiming to be the (1) The P42 million economic benefits package
legitimate incumbent officers holding over to their positions who have not
exercised their powers and functions therefor and another claiming to have Petitioners argue that the P42 million economic benefits package granted to
been elected in an election supervised by the DOLE and, at the same time, the covered faculty members were additional benefits, which resulted from a
exercising the powers and functions appended to their positions. In such a long and arduous process of negotiations between the Mariño Group and
case, the BLR, which has jurisdiction over the intra-union dispute, can UST. The BLR and the Court of Appeals were in error for considering the
validly order the immediate conduct of election of officers, otherwise, said amount as purely sourced from the allocation by UST of 70% percent of
internecine disputes and blame-throwing will derail an orderly and fair the incremental proceeds of tuition fee increases, in accordance with
election. Indeed, Section 1(b), [Rule XV], Book V of the Implementing Rules Republic Act No. 6728. Said law was improperly applied as a general law
and Regulations of the Labor Code, as amended, by Department Order No. that decrees the allocation by all private schools of 70% of their tuition fee
09, Series of 1997,[39] provides that, in the absence of any agreement among increases to the payment of salaries, wages, allowances and other benefits of
the members or any provision in the constitution and by-laws of the labor their teaching & non-teaching personnel. It is clear from the title of the law
organization, in an election ordered by the Regional Director, the chairman itself that it only covers government assistance to students and teachers in
of the committee shall be a representative of the Labor Relations Division of private education. Section 5 of Republic Act No. 6728 unequivocally limits
the Regional Office[.][40] the scope of the law to tuition fee supplements and subsidies extended by the
Government to students in private high schools. Thus, the petitioners
Ultimately, the Court of Appeals decreed: maintain that Republic Act No. 6728 has no application to the MOA executed
on 10 September 1992 between UST and USTFU, through the efforts of the
IN THE LIGHT OF ALL THE FOREGOING, the Petition is denied due Mariño Group.
course and is hereby DISMISSED.[41]
The Court disagrees with petitioners' stance.
Petitioners moved for reconsideration[42] of the Decision dated 16 March
2001 of the Court of Appeals, but it was denied by the said court in its The provisions of Republic Act No. 6728 were not arbitrarily applied by the
Resolution[43] dated 30 August 2001. DOLE-NCR Regional Director, the BLR, or the Court of Appeals to the P42
million economic benefits package granted by UST to USTFU, considering
Petitioners elevated the case to this Court via the instant Petition, invoking that the parties themselves stipulated in Section 7 of the MOA they signed
the following assignment of errors: on 10 September 1992 that:

I. 7.0. It is clearly understood and agreed upon that the aggregate sum of P42
million is chargeable against the share of the faculty members in the
WHETHER OR NOT THE HONORABLE COURT OF APPEALS incremental proceeds of tuition fees collected and still to be collected[;]
COMMITTED SERIOUS ERROR AND GRAVELY ABUSED ITS Provided, however, that he (sic) commitment of the UNIVERSITY to pay
DISCRETION WHEN IT UPHELD THE APPLICATION BY THE the aggregate sum of P42 million shall subsist even if the said amount
HONORABLE DIRECTOR OF THE BUREAU OF LABOR RELATIONS exceeds the proportionate share that may accrue to the faculty members in
OF THE PROVISIONS OF REPUBLIC ACT NO. 6728 TO THE P42 the tuition fee increases that the UNIVERSITY may be authorized to collect
MILLION CBA PACKAGE OF ECONOMIC BENEFITS OBTAINED BY in School-Year 1992-1993, and, Provided, finally, that the covered faculty
THE UST FACULTY UNION FROM THE UNIVERSITY OF SANTO members shall still be entitled to their proportionate share in any
TOMAS. undistributed portion of the incremental proceeds of the tuition fee increases
in School-Year 1992-1993, and which incremental proceeds are, by law and
II. pertinent Department of Education Culture and Sports (DECS) regulations,
required to be allotted for the payment of salaries, wages, allowances and
WHETHER OR NOT THE HONORABLE COURT OF APPEALS other benefits of teaching and non-teaching personnel for the
COMMITTED SERIOUS ERROR AND GRAVELY ABUSED ITS UNIVERSITY.[44] (Emphases supplied.)
DISCRETION WHEN IT DISALLOWED THE LUMP-SUM CHECK-OFF
AMOUNTING TO P4.2 MILLION BY RULING THAT THE P42 The "law" in the aforequoted Section 7 of the MOA can only refer to
MILLION CBA ECONOMIC PACKAGE OBTAINED BY THE UST Republic Act No. 6728, otherwise known as the "Government Assistance to
FACULTY UNION WAS MERELY AN ALLOCATION OF THE Students and Teachers in Private Education Act." Republic Act No. 6728 was
SEVENTY PER CENT (70%) OF THE TUITION INCREASES enacted in view of the declared policy of the State, in conformity with the
AUTHORIZED BY LAW AND THE DEPARTMENT OF EDUCATION, mandate of the Constitution, to promote and make quality education
CULTURE AND SPORTS. accessible to all Filipino citizens, as well as the recognition of the State of
the complementary roles of public and private educational institutions in the
III. educational system and the invaluable contribution that the private schools
have made and will make to education.[45] The said statute primarily grants
WHETHER OR NOT THE HONORABLE COURT OF APPEALS various forms of financial aid to private educational institutions such as
COMMITTED SERIOUS ERROR AND GRAVELY ABUSED ITS tuition fee supplements, assistance funds, and scholarship grants.[46]
DISCRETION WHEN IT DISREGARDED THE PROVISIONS ON
ELECTION OF UNION OFFICERS IN THE CONSTITUTION AND BY- One such form of financial aid is provided under Section 5 of Republic Act
LAWS OF THE UST FACULTY UNION AND INSTEAD UPHELD THE No. 6728, which states:
DIRECTIVE OF THE HONORABLE DIRECTOR OF THE BUREAU OF
LABOR RELATIONS TO CONDUCT THE ELECTION OF UNION SEC. 5. Tuition Fee Supplement for Student in Private High School. -
OFFICERS UNDER THE CONTROL AND SUPERVISION OF THE
REGIONAL DIRECTOR FOR THE NATIONAL CAPITAL REGION OF (1) Financial assistance for tuition for students in private high schools shall
THE DEPARTMENT OF LABOR AND EMPLOYMENT. be provided by the government through a voucher system in the following
manner:
Essentially, in order to arrive at a final disposition of the instant case, this
Court is tasked to determine the following: (1) the nature of the P42 million (a) For students enrolled in schools charging less than one thousand five
economic benefits package granted by UST to USTFU; (2) the legality of the hundred pesos (P1,500) per year in tuition and other fees during school year
10% check-off collected by the Mariño Group from the P42 million 1988-89 or such amount in subsequent years as may be determined from time
economic benefits package; and (3) the validity of the BLR order for USTFU to time by the State Assistance Council: The Government shall provide them
with a voucher equal to two hundred ninety pesos P290.00: Provided, That
the student pays in the 1989-1990 school year, tuition and other fees equal to came from the 70% allotment from tuition fee increases of UST. Preceding
the tuition and other fees paid during the preceding academic year: Provided, from this presumption, any deduction from the P42 million economic
further, That the Government shall reimburse the vouchers from the schools benefits package, such as the P4.2 million claimed by the Mariño Group as
concerned within sixty (60) days from the close of the registration period: attorney's/agency fees, should not be allowed, because it would ultimately
Provided, furthermore, That the student's family resides in the same city or result in the reduction of the statutorily mandated 70% allotment from the
province in which the high school is located unless the student has been tuition fee increases of UST.
enrolled in that school during the previous academic year.
The other reasons for disallowing the P4.2 million attorney's/agency fees
(b) For students enrolled in schools charging above one thousand five collected by the Mariño Group from the P42 million economic benefits
hundred pesos (P1,500) per year in tuition and other fees during the school package are discussed in the immediately succeeding paragraphs.
year 1988-1989 or such amount in subsequent years as may be determined
from time to time by the State Assistance Council, no assistance for tuition (2) The P4.2 Million Check-off
fees shall be granted by the Government:Provided, however, That the schools
concerned may raise their tuition fee subject to Section 10 hereof. Petitioners contend that the P4.2 million check-off, from the P42 million
economic benefits package, was lawfully made since the requirements of
(2) Assistance under paragraph (1), subparagraphs (a) and (b) shall be Article 222(b) of the Labor Code, as amended, were complied with by the
granted and tuition fees under subparagraph (c) may be increased, on the Mariño Group. The individual paychecks of the covered faculty employees
condition that seventy percent (70%) of the amount subsidized, allotted for were not reduced and the P4.2 million deducted from the P42 million
tuition fee or of the tuition fee increases shall go to the payment of salaries, economic benefits package became union funds, which were then used to pay
wages, allowances and other benefits of teaching and non-teaching personnel attorney's fees, negotiation fees, and similar charges arising from the CBA.
except administrators who are principal stockholders of the school, and may In addition, the P4.2 million constituted a special assessment upon the
be used to cover increases as provided for in the collective bargaining USTFU members, the requirements for which were properly observed. The
agreements existing or in force at the time when this Act is approved and special assessment was authorized in writing by the general membership of
made effective:Provided, That government subsidies are not used directly for USTFU during a meeting in which it was included as an item in the agenda.
salaries of teachers of nonsecular subjects. At least twenty percent (20%) Petitioners fault the Court of Appeals for disregarding the authorization of
shall go to the improvement or modernization of buildings, equipment, the special assessment by USTFU members. There is no law that prohibits
libraries, laboratories, gymnasia and similar facilities and to the payment of the insertion of a written authorization for the special assessment in the same
other costs of operation. For this purpose, schools shall maintain a separate instrument for the ratification of the 10 September 1992 MOA. Neither is
record of accounts for all assistance received from the government, any there a law prescribing a particular form that needs to be accomplished for
tuition fee increase, and the detailed disposition and use thereof, which the authorization of the special assessment. The faculty members who signed
record shall be made available for periodic inspection as may be determined the ratification of the MOA, which included the authorization for the special
by the State Assistance Council, during business hours, by the faculty, the assessment, have high educational attainment, and there is ample reason to
non-teaching personnel, students of the school concerned, and Department believe that they affixed their signatures thereto with full comprehension of
of Education, Culture and Sports and other concerned government agencies. what they were doing.
(Emphases ours.)
Again, the Court is not persuaded.
Although Section 5 of Republic Act No. 6728 does speak of government
assistance to students in private high schools, it is not limited to the same. The pertinent legal provisions on a check-off are found in Articles 222(b)
Contrary to petitioners' puerile claim, Section 5 likewise grants an and 241(n) and (o) of the Labor Code, as amended.
unmistakable authority to private high schools to increase their tuition fees,
subject to the condition that seventy (70%) percent of the tuition fee increases Article 222(b) states:
shall go to the payment of the salaries, wages, allowances, and other benefits
of their teaching and non-teaching personnel. The said allocation may also (b) No attorney's fees, negotiation fees or similar charges of any kind arising
be used to cover increases in the salaries, wages, allowances, and other from any collective bargaining negotiations or conclusion of the collective
benefits of school employees as provided for in the CBAs existing or in force agreement shall be imposed on any individual member of the contracting
at the time when Republic Act No. 6728 was approved and made effective. union: Provided, however, that attorney's fees may be charged against unions
funds in an amount to be agreed upon by the parties. Any contract, agreement
Contrary to petitioners' argument, the right of private schools to increase their or arrangement of any sort to the contrary shall be null and void.
tuition fee -- with their corresponding obligation to allocate 70% of said
increase to the payment of the salaries, wages, allowances, and other benefits Article 241(n) reads:
of their employees -- is not limited to private high schools. Section 9[47] of
Republic Act No. 6728, on "Further Assistance to Students in Private (n) No special assessment or other extraordinary fees may be levied upon the
Colleges and Universities," is crystal clear in providing that: members of a labor organization unless authorized by a written resolution of
a majority of all the members at a general membership meeting duly called
d) Government assistance and tuition increases as described in this Section for the purpose. The secretary of the organization shall record the minutes of
shall be governed by the same conditions as provided under Section 5 (2). the meeting including the list of all members present, the votes cast, the
purpose of the special assessment or fees and the recipient of such assessment
Indeed, a private educational institution under Republic Act No. 6728 still or fees. The record shall be attested to by the president.
has the discretion on the disposition of 70% of the tuition fee increase. It
enjoys the privilege of determining how much increase in salaries to grant And Article 241(o) provides:
and the kind and amount of allowances and other benefits to give. The only
precondition is that 70% percent of the incremental tuition fee increase goes (o) Other than for mandatory activities under the Code, no special
to the payment of salaries, wages, allowances and other benefits of teaching assessments, attorney's fees, negotiation fees or any other extraordinary fees
and non-teaching personnel.[48] may be checked off from any amount due to an employee without an
individual written authorization duly signed by the employee. The
In this case, UST and USTFU stipulated in their 10 September 1992 MOA authorization should specifically state the amount, purpose and beneficiary
that the P42 million economic benefits package granted by UST to the of the deduction.
members of the collective bargaining unit represented by USTFU, was
chargeable against the 70% allotment from the proceeds of the tuition fee Article 222(b) of the Labor Code, as amended, prohibits the payment of
increases collected and still to be collected by UST. As observed by the attorney's fees only when it is effected through forced contributions from the
DOLE-NCR Regional Director, and affirmed by both the BLR and the Court employees from their own funds as distinguished from union funds.[49]
of Appeals, there is no showing that any portion of the P42 million economic Hence, the general rule is that attorney's fees, negotiation fees, and other
benefits package was derived from sources other than the 70% allotment similar charges may only be collected from union funds, not from the
from tuition fee increases of UST. amounts that pertain to individual union members. As an exception to the
general rule, special assessments or other extraordinary fees may be levied
Given the lack of evidence to the contrary, it can be conclusively presumed upon or checked off from any amount due an employee for as long as there
that the entire P42 million economic benefits package extended to USTFU is proper authorization by the employee.
or her consent to the ratification of the MOA from his or her authorization of
A check-off is a process or device whereby the employer, on agreement with the check-off of union dues and special assessments. As it were, the
the Union, recognized as the proper bargaining representative, or on prior ratification of the MOA carried with it the automatic authorization of the
authorization from the employees, deducts union dues or agency fees from check-off of union dues and special assessments in favor of the union. Such
the latter's wages and remits them directly to the Union. Its desirability in a a situation militated against the legitimacy of the authorization for the P4.2
labor organization is quite evident. The Union is assured thereby of million check-off by a majority of USTFU membership. Although the law
continuous funding. As this Court has acknowledged, the system of check- does not prescribe a particular form for the written authorization for the levy
off is primarily for the benefit of the Union and, only indirectly, for the or check-off of special assessments, the authorization must, at the very least,
individual employees.[50] embody the genuine consent of the union member.

The Court finds that, in the instant case, the P42 million economic benefits The failure of the Mariño Group to strictly comply with the requirements set
package granted by UST did not constitute union funds from whence the P4.2 forth by the Labor Code, as amended, and the USTFU Constitution and By-
million could have been validly deducted as attorney's fees. The P42 million Laws, invalidates the questioned special assessment. Substantial compliance
economic benefits package was not intended for the USTFU coffers, but for is not enough in view of the fact that the special assessment will diminish the
all the members of the bargaining unit USTFU represented, whether compensation of the union members. Their express consent is required, and
members or non-members of the union. A close reading of the terms of the this consent must be obtained in accordance with the steps outlined by law,
MOA reveals that after the satisfaction of the outstanding obligations of UST which must be followed to the letter. No shortcuts are allowed.[52]
under the 1986 CBA, the balance of the P42 million was to be distributed to
the covered faculty members of the collective bargaining unit in the form of Viewed in this light, the Court does not hesitate to declare as illegal the
salary increases, returns on paycheck deductions; and increases in check-off of P4.2 million, from the P42 million economic benefits package,
hospitalization, educational, and retirement benefits, and other economic for union dues and special assessments for the Labor Education Fund and
benefits. The deduction of the P4.2 million, as alleged attorney's/agency fees, attorney's fees. Said amount rightfully belongs to and should be returned by
from the P42 million economic benefits package effectively decreased the petitioners to the intended beneficiaries thereof, i.e., members of the
share from said package accruing to each member of the collective collective bargaining unit, whether or not members of USTFU. This directive
bargaining unit. is without prejudice to the right of petitioners to seek reimbursement from
the other USTFU officers and directors, who were part of the Mariño Group,
Petitioners' line of argument - that the amount of P4.2 million became union and who were equally responsible for the illegal check-off of the aforesaid
funds after its deduction from the P42 million economic benefits package amount.
and, thus, could already be used to pay attorney's fees, negotiation fees, or
similar charges from the CBA - is absurd. Petitioners' reasoning is evidently (3) Election of new officers
flawed since the attorney's fees may only be paid from union funds; yet the
amount to be used in paying for the same does not become union funds until Having been overtaken by subsequent events, the Court need no longer pass
it is actually deducted as attorney's fees from the benefits awarded to the upon the issue of the validity of the order of BLR for USTFU to conduct its
employees. It is just a roundabout argument. What the law requires is that the long overdue election of union officers, under the control and supervision of
funds be already deemed union funds even before the attorney's fees are the DOLE-NCR Regional Director.
deducted or paid therefrom; it does not become union funds after the
deduction or payment. To rule otherwise will also render the general The BLR issued such an order since USTFU then had two groups, namely,
prohibition stated in Article 222(b) nugatory, because all that the union needs the Mariño Group and the Gamilla Group, each claiming to be the legitimate
to do is to deduct from the total benefits awarded to the employees the officers of USTFU.
amount intended for attorney's fees and, thus, "convert" the latter to union
funds, which could then be used to pay for the said attorney's fees. The DOLE-NCR Regional Director, in his Decision dated 27 May 1999,
decreed that the Mariño Group be expelled from their positions as USTFU
The Court further determines that the requisites for a valid levy and check- officers. But then, the BLR, in its Decision promulgated on 9 March 2000,
off of special assessments, laid down by Article 241(n) and (o), respectively, declared that the change of officers could best be decided, not by expulsion,
of the Labor Code, as amended, have not been complied with in the case at but by the general membership of the union through the conduct of election,
bar. To recall, these requisites are: (1) an authorization by a written resolution under the control and supervision of the DOLE-NCR Regional Director. In
of the majority of all the union members at the general membership meeting its assailed Decision dated 16 March 2001, the Court of Appeals agreed with
duly called for the purpose; (2) secretary's record of the minutes of the the BLR judgment in its ruling that the conduct of an election, under the
meeting; and (3) individual written authorization for check-off duly signed control and supervision of the DOLE-NCR Regional Director, is necessary
by the employee concerned.[51] to settle the question of who, as between the officers of the Mariño Group
and of the Gamilla Group, are the legitimate officers of the USTFU.
Additionally, Section 5, Rule X of the USTFU Constitution and By-Laws
mandates that: The Court points out, however, that neither the Decision of the BLR nor of
the Court of Appeals took into account the fact that an election of USTFU
Section 5. Special assessments or other extraordinary fees such as for officers was already conducted on 14 January 2000, which was won by the
payment of attorney's fees shall be made only upon a resolution duly ratified Gamilla Group. There is nothing in the records to show that the said election
by the general membership by secret balloting. was contested or made the subject of litigation. The Gamilla Group had
exercised their powers as USTFU officers during their elected term. Since
In an attempt to comply with the foregoing requirements, the Mariño Group the term of union officers under the USTFU Constitution and By-Laws was
caused the majority of the general membership of USTFU to individually only for three years, then the term of the Gamilla Group already expired in
sign a document, which embodied the ratification of the MOA between UST 2003. It is already beyond the jurisdiction of this Court, in the present
and USTFU, dated 10 September 1992, as well as the authorization for the Petition, to still look into the subsequent elections of union officers held after
check-off of P4.2 million, from the P42 million economic benefits package, 2003.
as payment for attorney's fees. As held by the Court of Appeals, however, the
said documents constitute unsatisfactory compliance with the requisites set The election of the Gamilla Group as union officers in 2000 should have
forth in the Labor Code, as amended, and in the USTFU Constitution and already been recognized by the BLR and the Court of Appeals. The order for
By-Laws, even though individually signed by a majority of USTFU USTFU to conduct another election was only a superfluity. The issue of who
members. between the officers of the Mariño Group and of the Gamilla Group are the
legitimate USTFU officers has been rendered moot by the succeeding events
The inclusion of the authorization for a check-off of union dues and special in the case.
assessments for the Labor Education Fund and attorney's fees, in the same
document for the ratification of the 10 September 1992 MOA granting the WHEREFORE, premises considered, the Petition for Review under Rule 45
P42 million economic benefits package, necessarily vitiated the consent of of the Rules of Court is hereby DENIED. The Decision dated 16 March 2001
USTFU members. For sure, it is fairly reasonable to assume that no and the Resolution dated 30 August 2001 of the Court of Appeals in CA-
individual member of USTFU would casually turn down the substantial and G.R. SP No. 60657, are hereby AFFIRMED WITH MODIFICATIONS.
lucrative award of P42 million in economic benefits under the MOA. Petitioners are hereby ORDERED to reimburse, jointly and severally, to the
However, there was no way for any individual union member to separate his faculty members of the University of Sto. Tomas, belonging to the collective
bargaining unit, the amount of P4.2 million checked-off as union dues and
special assessments for the Labor Education Fund and attorney's fees, with
legal interest of 6% per annum from 15 December 1994, until the finality of
this decision. The order for the conduct of election for the officers of the
University of Sto. Tomas Faculty Union, under the control and supervision
of the Regional Director of the Department of Labor and Employment-
National Capital Region, is hereby DELETED. No costs.

SO ORDERED.
VENGCO vs. TRAJANO Timbungco filed a motion for reconsideration of the above-quoted decision
G.R. No. 74453 | 1989-05-05 while Vengco, et al. filed their opposition to the said motion.
MEDIALDEA, J.:
On May 23, 1983, Officer-in-Charge Victoriano R. Calaycay issued an Order
This is a petition for certiorari which seeks to annul: (1) the Order of which held, thus:
respondent Director of the Bureau of Labor Relations dated May 23, 1983 in
BLR Case No. A-0179-82 entitled "Ambrocio Vengco, et al. vs. Emmanuel "Wherefore, premises considered, our resolution dated 29 December 1982 is
Timbungco" setting aside the decision dated December 29, 1982; and (2) the hereby set aside. However, an audit examination of the Books of Account of
Order dated April 2, 1986 denying the motion for reconsideration of the Kapisanan ng Manggagawa sa Associated Anglo-American Tobacco
Order dated May 23, 1983. Corporation (FOITAF) is hereby ordered.

The antecedent facts are as follows: SO RESOLVED." (p. 62, Rollo)

Sometime in the latter part of 1981, the Management of the Anglo-American Vengco, et al, sought reconsideration of the aforementioned order. They
Tobacco Corporation and the Kapisanan ng Manggagawa sa Anglo- contended that the examination of the books of accounts of the union is
American Tobacco Corporation. (FOI-TAF) entered into a compromise irrelevant considering that the issue involved in the case does not consist of
agreement whereby the company will pay to the union members the sum of union funds but back pay received by the union members from the company.
P150,000.00 for their claims arising from the unpaid emergency cost of Likewise, they pointed out that Timbungco did not give the money to the
living allowance (ECOLA) and other benefits which were the subject of their union treasurer and consequently, the amount was not entered in the records
compliant before the Ministry of Labor. Respondent Emmanuel Timbungco of the union.
(Timbungco, for short) who is the union president received the money which
was paid in installments. Thereafter, he distributed the amount among the On April 2, 1986, Trajano issued an order which affirmed the resolution of
union members. Petitioners Ambrocio Vengco, Ramon Moises, Rafael May 23, 1983 and denied the motion for reconsideration for lack of merit. (p.
Wagas and 80 others (Vengco, et al., for short) who are union members noted 58, Rollo)
that Timbungco was not authorized by the union workers to get the money;
and that ten percent (10%) of the P150,000.00 had been deducted to pay for Hence, the present recourse by Vengco, et al.
attorney's fees without their written authorization in violation of Article
242(o) of the Labor Code. So, they demanded from Timbungco an The issues raised in this case are as follows:
accounting of how the P150,000.00 was distributed to the members.
Timbungco did not give in to their demand. Thus Vengco, et al. filed a (1) Whether or not Timbungco is guilty of illegally deducting 10% attorneys'
complaint with the Ministry of Labor praying for: "(1) the expulsion of fees from petitioners' backwages; and
Emmanuel Timbungco as president of the union for violation of (the) union
constitution and by-laws and the rights and conditions of union members (2) Whether or not Trajano gravely abused his discretion amounting to lack
under the Labor Code; (2) an order to require Timbungco to render an of jurisdiction in ordering examination of union books instead of affirming
accounting of how the P150,000.00 was distributed; and (3) an order to his previous Order expelling Timbungco from the union and ordering him to
require private respondent to publish in the bulletin board the list of the render an accounting of P150,000.00 received by him. (p. 151, Rollo)
members and the corresponding amount they each received from the
P150,000.00." (Memorandum for Petitioners, p. 150, Rollo). In the resolution of June 4, 1986, We required the respondents to comment
on the petition.
In his answer with counterclaim, Timbungco alleged among others, that he
was authorized by a resolution signed by the majority of the union members In his comment, Timbungco reiterates the defenses he raised in his answer to
to receive and distribute the P150,000.00 among the workers; that the the complaint filed against him before the Med-Arbiter. In addition, he
computation of the benefits was based on the payroll of the company; that claims that he already filed an accounting report on the P150,000.00 with the
the ten percent (10%) attorney's fees was in relation to the claim of the local Bureau of Labor Relations which enumerated the names of the workers and
union for payment of emergency cost of living allowance before the Ministry the corresponding amounts they received with their respective signatures
of Labor which is totally distinct and separate from the negotiation of the opposite their names, the sub-total of the amount of benefits received per
CBA; and that the ten percent (10%) deduction was in accordance with department and the grand total of the amount distributed duly certified by the
Section II, Rule No. VIII, Book No. III of the Rules and Regulations Union Treasurer and Secretary and duly noted by Timbungco as Union
implementing the Labor Code and therefore, no authorization from the union President. (p. 73, Rollo)
members is required.
The Solicitor General, in his comment, agrees with Vengco, et al. and
On July 19, 1982, Med-Arbiter Willie B. Rodriguez issued an Order recommends that the petition be given due course. (p. 100, Rollo)
dismissing the complaint for lack of merit. (p. 33, Rollo)
Timbungco filed a reply to the aforesaid comment of the Solicitor General
Vengco, et al. appealed the aforesaid order to the Bureau of Labor Relations. which restates the arguments raised in his comment. (p. 121, Rollo)

On December 29, 1982, respondent Director of the Bureau of Labor The petition is meritorious.
Relations Cresenciano B. Trajano (Trajano, for short) rendered a decision,
the dispositive portion of which states: Article 241(o) of the Labor Code provides:

"Wherefore, premises considered, the instant appeal is hereby granted and "ART. 241. Rights and conditions of membership in a labor organization.
the Med-Arbiter's Order dated 19 July 1982 hereby set aside. Accordingly, The following are the rights and conditions of membership in a labor
respondent Emmanuel Timbungco is hereby ordered to render a full organization.
accounting of the One Hundred Fifty Thousand Pesos (P150,000.00) he
received from the management of Anglo-American Tobacco Corporation in xxx xxx xxx
behalf of the members of the Kapisanan ng mga Manggagawa sa Associated
Anglo-American Tobacco Corporation (FOITAF) and to publish in the "(o) Other than for mandatory activities under the Code, no special
union's bulletin board the list of all recipient union members and the assessment, attorney's fees, negotiation fees or any other extraordinary fees
respective amounts they have received, within ten (10) days from receipt may be checked off from any amount due an employee without an individual
hereof. Further, respondent is hereby expelled as president of the Kapisanan written authorization duly signed by an employee. The authorization should
ng Manggagawa sa Anglo American Tobacco Corporation (FOITAF). specifically state the amount, purpose and beneficiary of the deduction.
Lastly, the counterclaim interposed by the respondent's counsel, Atty.
Benjamin Sebastian is hereby ordered dismissed. xxx xxx xxx

So decided." (p. 50, Rollo.) It is very clear from the above-quoted provision that attorney's fees may not
be deducted or checked off from any amount due to an employee without his
written consent except for mandatory activities under the Code. A mandatory
activity has been defined as a judicial process of settling dispute laid down In view of the foregoing, We hold that the Orders dated May 23, 1983 and
by the law. (Carlos P. Galvadores, et al. vs. Cresenciano B. Trajano, Director April 2, 1986 were issued with grave abuse of discretion. The herein
of the Bureau of Labor Relations, et al., G.R. No. L-70067, September 15, controversy involves the propriety of the 10% deduction from the fringe
1986, 144 SCRA 138). In the instant case, the amicable settlement entered benefits of the union workers which they received from the management in
into by the management and the union can not be considered as a mandatory settlement of their claims. Such issue does not touch on union dues or funds.
activity under the Code. It is true that the union filed a claim for emergency Besides, the sum of P150,000.00 was not entered into the records of the
cost of living allowance and other benefits before the Ministry of Labor. But Union since, as earlier stated, the money was not turned over by Timbungco
this case never reached its conclusion in view of the parties' agreement. It is to the Union Treasurer. Consequently, the said Orders have no basis.
not also shown from the records that Atty. Benjamin Sebastian was
instrumental in forging the said agreement on behalf of the union members. ACCORDINGLY, the petition is granted. The assailed Orders dated May 23,
1983 of Officer-in-Charge Victoriano R. Calaycay of the Bureau of Labor
Timbungco maintains that the "Kapasiyahan" gave him the authority to make Relations, and April 2, 1986 of respondent Director Cresenciano B. Trajano
the deduction. This contention is unfounded. Contrary to his claim, the of the same Bureau are REVERSED and SET ASIDE and the latter's decision
undated "Kapasiyahan" or resolution did not confer upon him the power to dated December 29, 1982 is hereby reinstated. No costs.
deduct 10% of the P150,000.00 despite the alleged approval of the majority
of the union workers. A reading of the said resolution (p. 75, Rollo) yields SO ORDERED.
the same conclusion arrived at by Trajano who declared it defective. We
quote with approval Trajano's findings on this point:

"Further, a cursory examination of the alleged resolution shows that it is quite


defective. Not only that it is not dated but also that, with the exception of the
first page, the remaining pages were not captioned and did not state the very
purpose for which it was prepared. Thus, the alleged signatories were not
properly apprised thereof. There is, therefore, truth in complainant's
contention that they never authorized, more so, they had no knowledge of the
deduction of 10% attorney's fees until it was actually effected. Consequently,
the deduction was not valid." (p. 45, Rollo)

Moreover, the law is explicit. It requires the individual written authorization


of each employee concerned, to make the deduction of attorney's fees valid.

Likewise, We find that the other "Kapasiyahan" dated September 18, 1981
submitted by Timbungco belied his claim that he was authorized by the union
workers to receive the sum of P150,000.00 on their behalf. The pertinent
portion of the said "Kapasiyahan" provides:

"3. Na sa dahilang hindi bigla ang pagbabayad sa nasabing "CLAIM" bukod


pa sa marami kaming naghati-hati sa nasabing halaga ipinapasiya naming na
kusang-loob na kunin ang aming bahagi sa aming kapisanan sa unang linggo
ng Disyembre, 1981 at ito'y ipinaalam namin sa Pangulo ng Kapisanan na si
Ginoong Emmanuel Timbungco." (p. 47, Rollo)

The above-quoted statement merely indicated the intention of the workers to


get their claim on the first week of December, 1981 and to inform Timbungco
of their intention. Clearly, this statement can not be construed to confer upon
Timbungco the authority to receive the fringe benefits for the workers.
Absent such authority, Timbungco should not have kept the money to himself
but should have turned it over to the Union Treasurer. He, therefore,
exceeded his authority as President of the Union.

Moreover, Book III, Rule VIII, Section II of the Implementing Rules cited
by Timbungco which dispenses with the required written authorization from
the employees concerned does not apply in this case. This provision
envisions a situation where there is a judicial or administrative proceedings
for recovery of wages. Upon termination of the proceedings, the law allows
a deduction for attorney's fees of 10% from the total amount due to a winning
party. In the herein case, the fringe benefits received by the union members
consist of back payments of their unpaid emergency cost of living allowances
which are totally distinct from their wages. Allowances are benefits over and
above the basic salaries of the employees (University of Pangasinan Faculty
Union vs. University of Pangasinan, G.R. No. L-63122, February 20, 1984,
127 SCRA 691). We have held that such allowances are excluded from the
concept of salaries or wages (Cebu Institute of Technology (CIT) vs. Ople,
G.R. No. L-58870, December 18, 1987, 156 SCRA 629). In addition, the
payment of the fringe benefits were effected through an amicable settlement
and not in an administrative proceeding.

The submission by Timbungco of an accounting report on the distribution of


P150,000.00 is of no moment in the face of our findings that the deduction
of 10% for attorney's fees is illegal and void for failure to comply with the
requirements of the law.

Considering the aforestated violations of Timbungco, there can be no


question that he should bear the consequences of his acts. We find that the
penalty of expulsion from the union presidency imposed upon Timbungco is
justified.
GALVADORES vs. TRAJANO
G.R. No. 70067 September 15, 1986 On March 22, 1984, the Union filed a Manifestation to the effect that about
MELENCIO-HERRERA, J.: 6,067 members of the Union ratified the October 29, 1983 resolution of the
legislative council in a plebiscite called for that purpose. On the basis thereof,
Petitioner employees of the Philippine Long Distance Telephone Company Counsel moved for the payment of his legal fees under the September 7, 1983
(PLDT) and members of respondent Free Telephone Workers Union, now contract.
the Manggagawa ng Komunikasyon sa Pilipinas (simply referred to
hereinafter as the Union), question the legality of the check-off for attorney's Petitioners questioned the plebiscite on the ground that Question No. 2,
fees amounting to P1M, more or less, of respondent Atty. Jose C. Espinas which reads:
(hereinafter referred to as "Respondent Counsel") from the monetary benefits
awarded to PLDT employees in a deadlocked collective bargaining Question No. 2. Do you approve of the use of P1 million (P500,000.00 to be
agreement negotiations between the PLDT and the Union. withdrawn from PECCI and another P500,000.00 from IBAA) from our CBA
negotiation fund together with the attorney's fees (P1 million) that was
The case stemmed from the following facts: collected and to be loaned to the MKP/FTWU as our counterpart of the seed
money to start the housing program as agreed by the PLDT management and
Respondent Counsel has been the legal counsel of respondent Union since our union panel and included in the award of the MOLE?
1964. For his services, he was hired on a case to case contingent fee basis.
On September 7, 1983, he received a letter from the Union President reading: was misleading and deceptive as it assumed that there was no dispute
regarding the deduction of attorney's fees from the monetary benefits
The Free Telephone Workers Union once again request you to appear as awarded to PLDT employees.
counsel in the on going labor dispute at PLDT. In consideration of your
services therein, the union binds itself to compensate you for your fees and On February 18, 1985, respondent Director of the Bureau of Labor Relations
expenses therein on a contingent basis. The amount shall be 10% of any dismissed petitioners' complaint for lack of merit reasoning that "the outcome
improvement, with retroactive effect, of the PLDT's last offer to the deadlock of the plebiscite negates any further question on the right of the union counsel
in CBA negotiations which we know will result in a compulsory arbitration. to collect the amount of P115 from each of the employees involved."
A supporting board resolution will later confirm the letter. 1
It is this Decision that is assailed by petitioners principally on the ground that
PLDT's "last offer" referred to on the wage increases was: P230 for the first the individual written authorization of an the employees must first be
year of the proposed CBA; P100 for the second year; and P90 for the third obtained before any assessment can be made against the monetary benefits
year. 2 awarded to them pursuant to Article 242(o) of the Labor Code; and that
assuming that Respondent Counsel is entitled to attorney's fees, the same
On September 9, 1983, the Minister of Labor and Employment assumed should be taken from Union funds.
jurisdiction over all unresolved issues in the bargaining deadlock between
PLDT and the Union and proceeded to resolve the same by compulsory In their Comment, respondents Union and Counsel argue that compulsory
arbitration. arbitration is a "mandatory activity" and an exception to Article 242(o) of the
Labor Code, and that the Union members approved the questioned deduction
On October 23, 1983, the Minister of Labor awarded across-the-board wage in the plebiscite of January, 1984, under the condition that P lM of the same
increases of P 330/month effective November 9, 1982; P155/month effective would be made available for the Union's housing project.
November 9, 1983, and P155/month effective November 9, 1984, in addition
to the Christmas bonus of 1/2 month pay per employee effective December, In his Comment, the Solicitor General agrees with petitioners that the issue
1983, and other fringe benefits. As will be noted, there were improvements presented is squarely covered by Article 222(b) of the Labor Code, as
obtained from PLDT's "last offer." amended by P.D. No. 1691 so that attorney's fees, if legally payable, can only
be charged against Union funds.
On October 29, 1983, the Executive Board of the Union passed a resolution
requesting PLDT to deduct P115.00 per employee for the legal services The Court resolved to give due course.
extended to the Union by respondent Counsel.
Article 222(b) of the Labor Code provides:
On November 2, 1983, petitioners initially numbering 600 and finally 5,258,
filed a letter-complaint before the MOLE through their authorized Article 222. Appearance and Fees.
representative, petitioner Carlos Galvadores assailing the imposition of
P130.00 (later corrected to P155.00) per employee as attorney's fees of xxx xxx xxx
respondents counsel. Annexed to the complaint were the written statements
of the employee authorizing Galvadores to act for and in their behalf. (b) No attorney's fees, negotiation fees or similar charges of any kind arising
Petitioners took the position that the attorney's fees of respondent counsel from any collective bargaining negotiations or conclusion of the collective
were not only unreasonable but also violative of Article 242(o) of the Labor bargaining agreement shall be imposed on any individual member of the
Code; and that he deductions cannot given legal effect by a mere Board contracting union; Provided, however, that attorney's fees may be charged
resolution but needs the ratification by the general membership of the Union. against union funds in an amount to be agreed upon by the parties. Any
contract, agreement or arrangement of any sort to the contrary shall be null
Respondents Union and Counsel, on the other hand, proferred the argument and void.
that the attorney s fees being exacted pertained to his services during
compulsory arbitration proceedings and cannot be considered as negotiation While Article 242 of the same Code reads:
fees or attorney's fees within the context of Article 242(o) of the Labor Code
and that contrary to petitioners' claim that Respondent Counsel surfaced only Art. 242. Rights and conditions of membership in a labor organization. The
as lawyer of the Union when the employees themselves engaged in mass following are the rights and conditions of membership in a labor
action to force a solution to the deadlock in their negotiations, he appeared organization:
continuously from September 8, 1983 until the decision in the case was
rendered on October 23, 1983. Petitioners proposed a solution offering to pay xxx xxx xxx
P10.00 per employee, but Respondent Counsel refused.
(o) Other than for mandatory activities under the Code, no special
In the meantime, on November 4, 1983, PLDT filed notice that assessment assessment, attorney's fees, negotiation fees or any other extraordinary fees
had been withheld from the differential pay due petitioners but that the same may be checked off "from any amount due an employee without individual
would not be turned over to the Union without prior MOLE authority so as written authorization duly signed by the employee. The authorization should
not to involve management in the intra-union disagreement. specifically state the amount, purpose and beneficiary of the deduction.

February 13, 1984, the Minister of Labor referred the dispute to the Bureau The Omnibus Rules Implementing the Labor Code also provide that
of Labor Relations for being intra-union nature. Several hearings were held deductions from wages of the employees may only be made by the employer
by that Bureau. in cases authorized by law, including deductions for insurance premiums
advanced by the employer on behalf of the employees as well as union dues
where the right to check-off is authorized in writing by the individual
employee himself. 3

The provisions are clear. No check-offs from any amounts due employees
may be effected without individual written authorizations duly signed by the
employee specifically stating the amount, purpose and beneficiary of the
deduction. The required individual authorizations in this case are wanting. In
fact, petitioner employees are vigorously objecting. The question asked in
the plebiscite, besides not being explicit, assumed that there was no dispute
relative to attorney's fees.

Contrary to respondent Union's and Counsel's stand, the benefits awarded to


PLDT employees still formed part of the collective bargaining negotiations
although placed already under compulsory arbitration. This is not the
"mandatory activity" under the Code which dispenses with individual written
authorizations for check-offs, notwithstanding its "compulsory" nature. It is
a judicial process of settling disputes laid down by law. Besides, Article
222(b) does not except a CBA, later placed under compulsory arbitration,
from the ambit of its prohibition. The cardinal principle should be borne in
mind that employees are protected by law from unwarranted practices that
diminish their compensation without their knowledge and consent. 4

ACCORDINGLY, the assailed Decision of February 18, 1985 rendered by


respondent Director of the Bureau of Labor Relations, is hereby SET ASIDE.
The attorney's fees herein involved may be charged against Union funds
pursuant to Article 222(b) of the Labor Code, as may be agreed upon between
them.

SO ORDERED.
CONTINENTAL CEMENT CORPORATION LABOR UNION (NLU)
vs. CONTINENTAL CEMENT CORPORATION Nonetheless, some 110 striking workers did not return to work.
G.R. No. 51544 | 1990-08-30 Consequently, on July 26, 1976, private respondent filed with the
GANCAYCO, J.: Department of Labor reports on the dismissal of those who failed to comply
with the return-to-work order with copies of the reports furnished workers
This petition addresses the question of dismissal from the service of the affected.
officers and the suspension of some members of petitioner union by the
public respondent and the National Labor Relations Commission (NLRC). On July 29, 1976, the president of petitioner and 7 other officers requested
admission to work but were informed that their employment had been
On April 21, 1975, the NLRC issued an arbitration award in NLRC Cases terminated by the company.
No. 2406 and No. 3053 resolving certain demands of the petitioner respecting
the working terms and conditions that should be observed in the After due hearing, on March 10, 1977, the NLRC rendered judgment, the
establishment of private respondent. However, due to disagreement on the dispositive portion of which reads as follows:
interpretation of the provisions of the award concerning vacation, sick leaves
and standardization of wages, compliance therewith was delayed. In order to "WHEREFORE, this Commission hereby orders:
compel private respondent to immediately implement the award, petitioner
staged a strike on October 25, 1975. It was, however, lifted after the private 1. That the Union officers, together with its Board of Directors, namely:
respondent agreed to pay the disputed employees' leaves during the period Rosauro Ancheta, Lauro Bartolome, Alejandro Bernabe, Ireneo Bernabe,
July 1, 1974 to June 30, 1975 in three installments, that is, 50% on December Romulo Buluran, Fortunato Mendoza, Rodolfo L. Santiago, Guillermo
20, 1975, 25% on February 25, 1976 and 25% on March 15, 1976. Roque, Dioscoro Sergio, Manuel Sobrenilla, Sergio Panel, Antonio
Mendoza; Isaias Mendoza, Abundio Cruz, Edwin Hugo and Aquilino
Meanwhile, private respondent sought clarification from the labor arbiter on Sarmiento, be considered, as they are hereby considered, separated from the
whether a group of 91 workers who were unable to complete 300 days of service of the Company, and from their positions as officers of the Union, as
work within a 12-month period was entitled to proportionate payment of of 12 July 1976;
vacation and sick leave benefits. On March 19, 1976, the labor arbiter ruled
that the award required private respondent to make proportionate payments 2. That the Union members who participated in the strike be considered, as
in favor of the workers in question. This ruling was appealed by private they are hereby considered, under suspension, by way of penalty, for the
respondent but duration of their absence from work in the Company as a consequence of
on April 7, 1976 petitioner filed a notice of strike against private respondent their strike;
for its refusal to make the proportionate payments mentioned. Petitioner
carried out its threatened strike on May 16, 1976. The strike was settled on 3. That the Union members referred to in the immediately preceding
May 22, 1976 with private respondent agreeing inter alia, to pay the 91 paragraph return to their work in the Company, and that the latter take them
workers concerned P25,000.00 for "humanitarian reasons." Private back, at the same terms and conditions of employment obtaining before 12
respondent, however, reserved the right to seek clarification of its obligations July 1976, within five (5) days from receipt of this Decision;
under the NLRC award. Payment was made on May 25, 1976.
4. That the Company pay within ten (10) days from receipt of this Decision
The obligation of private respondent to pay the employees their vacation and the vacation and sick leave benefits of qualified employees for the period
sick leaves for the period July 1, 1975 to June 30, 1976 developed into a new from 1 July 1975 to 30 June 1976, as well as the 13th month pay and
issue between the parties. Prior to the payment becoming due, private increased minimum wages, unless the Company has been duly exempted
respondent negotiated with petitioner for a staggered form of payment as from the payment of the same;
before due to its financial difficulties and planned shutdown of the plant in
July. Petitioner at first insisted that its members be paid full; however, it 5. That the Company comply immediately and fully with the terms and
subsequently agreed to installment payments but gave warning on July 11, conditions of the Award of Voluntary Arbitrator Francisco Fuentes dated 12
1976, a Sunday, to the private respondent that payment of 50% of the benefits September 1974 (NLRC Case Nos. 2406 and 3053);
should be made not later than July 12, 1976 and the remaining 50%., not later
than the end of the month. Private respondent requested an extension up to 6. That the Union and Company incorporate in their collective bargaining
July 13, 1976 within which to consider the counter-proposal but this was agreement all includible terms and conditions in the Award referred to in
rejected by petitioner. paragraph 5 of this dispositive part; and

Petitioner staged a strike in the early of July 12, 1976, picketing the entrance 7. That, it appearing that the Union is officially affiliated with the National
of the premises of private respondent. Among the officers and members of Labor Union (NLU) and since the officers of the Union are, by virtue hereof,
petitioner who were identified on the picket line were Rosauro Ancheta, no longer holding their positions as such, the NLU temporarily handle the
Lauro Bartolome, Abundio Cruz, Edwin Hugo, Manuel Sobrenilla, Antonio affairs of the Union, in a trusteeship capacity, until the Union shall have
Mendoza, Alfredo Urtula, Lorenzo Hormadal, Nicolas Sobrenilla, Dioscoro reorganized in accordance with its Constitution and By-Laws, or in the
Sergio, Velasco Reyes, Fortunato Mendoza, Floro Villano, Lauro Francisco, absence of applicable internal rules, in accordance with the will of the
Salva Nelson, Antonio Cruz, Augusto Lopez and Francisco Sarmiento. Other majority of its members, but not more than three (3) months from the
workers at the roadblocks were not positively identified. promulgation of this Decision." 2

On July 13, 1976, the Minister of Labor issued an order thru the Director of On July 22, 1977, the petitioner appealed the above decision of the NLRC to
the Bureau of Labor Relations, directing the striking workers to resume work the Minister of Labor but the latter affirmed it on March 6, 1979. 3 A motion
under the terms and conditions prevailing prior to the work stoppage. 1 The for reconsideration filed by the Union was denied by the Minister of Labor
order was served on the parties in the afternoon of the same date. on August 1, 1979. 4

Nevertheless, on July 14, 1976, only 11 out of the total work force of about Hence, this special civil action for certiorari, wherein the issues raised are (1)
120 workers in one shift reported for work and were admitted by the whether or not the strike staged by petitioner on June 12, 1976 until its lifting
company. was illegal; and (2) in the affirmative, whether or not the penalties meted out
by the NLRC to the Union officers and the members are warranted by the
On July 15, 1976, petitioner filed a motion for reconsideration of the return- circumstances and the law.
to-work order or its suspension pending compliance by private respondent
with the 1975 NLRC award in favor of petitioner. Picketing was resumed Presidential Decree No. 823, as amended, provides:
despite the presence of military personnel who were called to assist in the
implementation of the return-to-work order. "Sec. 1. It is the policy of the State to encourage trade unionism and free
collective bargaining within the framework of compulsory and voluntary
On July 23, 1976, the Minister of Labor certified the dispute between the arbitration. Therefore, all forms of strikes, picketing and lockouts are hereby
parties to the NLRC for compulsory arbitration in NLRC Certified Case No. strictly prohibited in vital industries . . . ."
039. Under the Labor Code, this certification had the effect of automatically
enjoining any strike by the Union or lockout by the private respondent. Letter of Instruction No. 368 of the President provides:
Furthermore, petitioner could have applied with the Bureau of Labor
"For the guidance of workers and employers, some of whom have been led Relations for a writ of execution to enforce the award that was already final
into filing notices of strikes and lockouts even in vital industries, you are and executory.
hereby instructed to consider the following as vital industries and companies
or firms under PD 823 as amended: As to the second issue, petitioner assails as too harsh the suspension meted
out by the NLRC to its members.
xxx xxx xxx
The strikers in question did not only violate the no-strike policy of the state
2. Companies or firms engaged in the manufacture or processing of the in regard to vital industries; instead, they repeatedly defied the orders of the
following essential commodities: Director of Labor Relations and the Minister of Labor for them to return to
work. Their dismissal was recommended by the labor arbiter. However, out
xxx xxx xxx of compassion, the NLRC and the Minister of Labor only suspended them.

B. Cement." Petitioner then contends that the separation from work of the officers of the
union is quite severe. The officers had the duty to guide their members to
Private respondent was engaged in the manufacture of cement which is no respect the law. Instead, they urged them to violate the law and defy the duly
doubt a vital industry in which a strike or lockout is prohibited under the constituted authorities. Their responsibility is greater than that of the
foregoing aforestated decree. And even assuming that private respondent was members. Their dismissal from the service is a just penalty for their unlawful
not engaged in a vital industry, the strike that was staged by petitioner was acts.
nonetheless illegal. It was not in connection with any unresolved economic
issue in collective bargaining which is the only ground for which a lawful It is within the power of the NLRC to order the removal of the officers of
strike can be held. petitioner. This is provided for in the labor law.

Section 7 of the Rules and Regulations implementing Presidential Decree "Art. 242. 5 Rights and conditions of membership in a labor organization.
No. 823, as amended, provides:
The following are the rights and conditions of membership in a labor
"Section 7. Requirements for strikes and lockouts. Strikes and lockouts may organization:
be declared only upon compliance with the following requirements:
xxx xxx xxx
(a) Ground for strike A strike may be declared by a legitimate labor
organization which is the recognized bargaining agent of the appropriate (p) It shall be the duty of any labor organization and its officers to inform its
bargaining unit in connection with unresolved economic issues in collective members on provisions of the constitution and by-laws, collective bargaining
bargaining in non-vital industries. agreement, the prevailing labor relations system and all their rights and
obligations under existing labor laws. For this purpose, registered labor
(b) Ground for lockout A lockout may be declared only by an employer in organizations may assess reasonable dues to finance labor relations seminars
connection with unresolved economic issues in collective bargaining in non- and other labor education activities.
vital industries.
Any violation of the above rights and conditions of membership shall be a
xxx xxx xxx ground for cancellation of union registration or expulsion of an officer from
office, which ever is appropriate. At least 30 per cent of all the members of
(d) Unauthorized strikes or lockouts Notices of strikes or lockouts involving a union or any member or members specifically concerned may report such
industries, establishments or issues not covered or authorized under violation to the Bureau. The Bureau shall have the power to hear and decide
paragraphs (a) and (b) shall be dismissed, and the party filing the notice of any reported violation and to mete out the appropriate penalty."
strike or lockout shall be advised thereof."
The officers of petitioner misinformed the members and led them into staging
The issue between the petitioner and the private respondent at the time of the an illegal strike. If the NLRC is to attain the objective of the Labor Code to
strike concerned merely the implementation of an arbitration award of the ensure a stable but dynamic and just industrial peace 6 the removal of
NLRC. The petitioner had a remedy by applying for a writ of execution to undesirable labor leaders must be effected.
enforce that award. Its resort to a strike was without lawful basis.
WHEREFORE, the petition is DISMISSED as it has not been shown that the
Moreover, under Section 1 of Presidential Decree No. 823, there is a public respondent committed any grave abuse of discretion in rendering the
requirement of notice, as follows: orders dated March 6, 1979 and August 1, 1979 affirming the decision of the
NLRC dated March 10, 1977.
"However, any legitimate labor union may strike and any employer may
lockout in establishments not covered by General Order No. 5 only on SO ORDERED.
grounds of unresolved economic issues in collective bargaining, in which
case the union or the employer shall file a notice with the Bureau of Labor
Relations at least 30 days before the intended strike or lockout. . . ."

Petitioner claims that it filed a notice of strike on April 7, 1976. That notice
was in connection with a dispute that had been settled by the Memorandum
Agreement between the parties dated May 22, 1976. A notice of strike is
intended to enable the Bureau of Labor Relations to try to settle the dispute
amicably. The strike on July 12, 1976 denied the Bureau this opportunity.

Petitioner invokes the right to strike as a measure of self-defense as it had


been driven to the wall by the unjust refusal of private respondent to comply
with the NLRC award.

The non-compliance by the private respondent with the said award did not
threaten the existence of petitioner or that of its members. The dispute did
not concern the right of the Union to organize nor the employees' right to
work. It merely involved the non-payment of the vacation and sick leaves of
the employees for the past years' services.
DUYAG vs. INCIONG purpose of the special assessment or fees and the recipient of such
G.R. No. L-47775 | 1980-07-05 assessments or fees. The record shall be attested to by the president;
AQUINO, J.:
"(o) Other than for mandatory activities under the Code, no special
This case is about the removal of private respondents as union officers due assessments, attorney's fees, negotiation fees or any other extraordinary fees
to alleged irregularities and anomalies in the administration of the affairs of may be checked off from any amount due to an employee without an
the union. individual written authorization duly signed by the employee. The
authorization should specifically state the amount, purpose and beneficiary
On January 14, 1977, the five petitioners, who are arrastre checkers of E. of the deduction; and.
Razon, Inc. in the South Harbor, Port Area, Manila as well as bona fide
members of the Associated Port Checkers and Workers Union, filed with Xxx - xxx - xxx
Regional Office No. 4 of the Department of Labor a complaint containing
several charges against the four private respondents, who, respectively, are The foregoing legal provisions apply squarely to the unauthorized deductions
the president (for more than twenty years), treasurer, vice-president and from the wages of the arrastre checkers.
auditor of the union.
For such unauthorized collection of union dues, the responsibility of
The record reveals the following facts, some of which are admitted or not respondent Ricardo R. Manalad, as union president, is not denied.
denied by the private respondents, while the other facts are supported by
substantial evidence which is summarized in the decisions of the med-arbiter Withholding of union members' share in the profits amounting to P18,640.09.
and the - E. Razon, Inc., the arrastre operator, paid to the union on December 18,
1973 the sum of P25,684.61 as its share of the profits (profit-share) for the
Director of Labor Relations: period from May to October, 1973. Instead of distributing the whole amount
to the union members, the respondents paid to them only P19,974 and
Unauthorized increases in union dues. - For arrastre checkers, the monthly retained the balance of P5,710.61 which had not been accounted for.
union dues amount to ten pesos, as fixed in section 2(b), article VI of the
union's constitution and by laws approved on September 5, 1969. The Labor Arbiter found that other amounts were withheld by the
respondents from the union's profit-shares for subsequent periods. The total
The monthly union dues were increased by two pesos in the resolution of amount withheld is P18,640.09 or P18,570.63, as shown in page 8 of private
September 1, 1970 and by five pesos in the resolution of March 14, 1972. respondents' memorandum.
However, those two resolutions are void because they were not approved by
three-fourths of all the members of the board of directors, as required in With specific reference to the profit-share amounting to P22,559.50 paid by
article VII of the union's constitution and by-laws, dealing with amendments. E. Razon, Inc. for the period from November, 1973 to February, 1974, the
respondents deposited the amount in the account of the union's Cooperative
For March, April and May, 1973, the respondents without the benefit of any Credit Union of which respondent Manalad was also the president. Later, the
board resolution caused to be collected an additional one peso, thus respondents withdrew the said amount, distributed among the union members
increasing the union dues to eighteen pesos. the sum of P20,848 and withheld the balance of P1,711.50, which respondent
Manalad and the union treasurer, respondent Honorato K. Leaño,
For April and May, 1975, the respondents caused to be collected monthly appropriated as follows:
union dues amounting to nineteen pesos or another increase of one peso.
Manalad - Filipinas Bank and Trust Com-pany,
And for the first semester of 1976, a deduction of eight pesos and fifty Manila Hilton Branch Check No. 352966 dated March 22, 1975, drawn to
centavos was made from the mid-year bonus without any board resolution cash P1,000.00
authorizing such deduction. In prior years, no deduction for union dues was
made from the mid-year bonus. Leaño - Filipinas Bank and Trust Company,
Manila Hilton Branch Check No. 352967 dated March 22, 1975, drawn to
The med-arbiter concluded that the increases in union dues and the deduction cash 559.50
from the mid-year bonus are void because the same were collected in
contravention of the constitution and by-laws. Leaño - Filipinas Bank and Trust Company,
Manila Hilton Branch Check No. 352968 dated March 22, 1975, drawn to
Moreover, their collection was not covered by any check-off authorization cash 152.00
nor evidenced by any receipt and was in contravention of the Labor Code. --------------
The amounts collected were not duly accounted for. The Labor Code TOTAL P 1,711.50
provides:
The med-arbiter found that the modus operandi resorted to by the
"ART. 242. Rights and conditions of membership in a labor organization. - respondents with respect to the profit-share amounting to P22,559.50 was
The following are the rights and conditions of membership in a labor followed by them as to the deductions from the profit-shares for the other
organization: periods.

xxx - x - xxx He surmised that the union officers must have deducted a considerable
amount from the profit-shares because they started that practice in 1966 when
"(g) No officer, agent or member of a labor organization shall collect any E. Razon, Inc. and Guacods Marine Terminals, Inc. commenced the profit-
fees, dues, or other contributions in its behalf or make any disbursement of share program.
its money or funds unless he is duly authorized pursuant to its constitution
and by-laws;" However, during the pendency of the case in this Court, the private
respondents submitted a resolution dated November 25, 1977 wherein more
"(h) Every payment of fees, dues or other contributions by a member shall be than ninety percent of the union members allegedly ratified the deductions
evidenced by a receipt signed by the officer or agent making the collection from the mid-year bonus and profit-shares and authorized future deductions
and entered into the record of the organization to be kept and maintained for (pp. 921 and 1615-6, Rollo).
the purpose;
Although the said resolution rendered this aspect of the case moot, it cannot
xxx - x - xxx obliterate the violations of the constitution and by-laws and the Labor Code
already committed by respondents Manalad and Leaño. The deduction of
"(n) No special assessment or other extraordinary fees may be levied upon union dues from the mid-year bonus and the withholding of part of the profit-
the members of a labor organization unless authorized by a written resolution shares were illegal and improper at the time they were made.
of a majority of all the members at a general membership meeting duly called
for the purpose. The secretary of the organization shall record the minutes of Disbursements exceeding P500 which were not authorized by the board of
the meeting including the list of all members present, the votes cast, the directors. - Section 4(d), article IV of the union's constitution and by-laws
provides that the board of directors may "authorize and approve all Thus, on December 17, 1975 and March 29, June 9 and August 31, 1976,
disbursements from union fund where the amount involved is more than Manalad approved payments by the arrastre checkers' union to the other
P500 and without that authorization or approval in due form, no such union of the sums of P1,000, P250 and P1,250.
disbursements will be allowed by the Treasurer".
Conflict of interest on the part of Manalad. - Respondent Manalad organized
Respondent Manalad made the following disbursements of union funds in an a family corporation known as the Comet Integrated Stevedoring Services,
amount exceeding P500 without the requisite authorization of the board of Inc. whose rank-and-file employees are also members of the arrastre
directors: checkers' union. Thus, Manalad has functioned in the dual capacity of labor
leader and employer, not to mention the fact that he is also an officer of
Evidence Date Amount Disbursed another labor union, PTCCEA.

Annex S March 26, 1969 P1,400.00 As head of the arrastre checkers' union, he issued customs passes for the
Annex T June 1, 1970 1,000.00 checkers of his family-owned stevedoring firm to facilitate their rendition of
Annexes U to W July 13, August 6 services to some shipping companies.
and Sept. 24, 1971 3,111.40
Annexes Y, X, Z March 5 and 30 The complainants contend that such a situation has involved Manalad in a
and Z-1 and AA April 10, May 18, conflict of interest: if he favors his stevedoring firm, he is bound to jeopardize
to CC Aug. 30, Sept. 20 the interests of the arrastre checkers' union of which he is the president.
and Dec. 31, 1973 7,028.00
Annex DD Dec. 6, 1974 1,000.00 Under these facts, the med-arbiter in his decision of August 29, 1977 ordered
Annex R June 12, 1976 900.00 the removal of the private respondents as officers of the union and directed
them to reimburse to the members thereof the amounts illegally collected
Respondents Manalad and Leaño, also without prior board authorization, from them.
withdrew on twenty-three occasions union funds in the aggregate sum of
P43,026.80 deposited in Savings Account No. 5953 of the Manila Hilton The private respondents appealed to the Director of Labor Relations who in
Branch of the Filipinas Bank and Trust Company (Annexes GG to GG-22). his decision of November 9, 1977 reversed the decision of the med-arbiter.

The sum of P3.500 was paid to respondent Amparo pursuant to a resolution The Director held that resort to intra-union remedies is not necessary and that
dated July 12, 1971 which was approved by only six members of the board the five complainants have the right and personality to institute the
of directors, instead of fourteen members, as required in the constitution and proceeding for the removal of the respondents, to recover the amounts
by-laws of the union. illegally collected or withheld from them and to question illegal
disbursements and expenditure of union funds.
Maladministration of welfare fund. - Respondent Manalad allowed the
application of the funds of the union's Welfare Plan to the following However, the Director ruled that the power to remove the union officers rests
extraneous purposes: in the members and that the Bureau of Labor Relations generally has nothing
to do with the tenure of union officers which "is a political question".
1. On March 31, and April 6 and 14, 1973, the sum of P5,000 was taken from
the Pacific Memorial Plan collections and loaned to the union's Cooperative The Director further ruled that his office has jurisdiction to look into the
Credit Union, Inc. charge of illegal disbursements of union funds. He directed the Labor
Organization Division of the Bureau to examine the books of account and
2. On October 7, 1973, the sum of P1,500 was loaned to the same cooperative financial records of the union and to submit a report on such examination.
for organizational expenses.
The motions for reconsideration filed by the parties were denied by the
3. On August 7, 1971, the sum of P200 was taken from the welfare fund for Undersecretary of Labor in his resolution of January 25, 1978 (he was then
advance representation expenses of Manalad. Acting Director of Labor Relations). He ruled that the expulsion of union
officers is the prerogative of the members of the union.
4. On December 18, 1971, the sum of P1,600 was taken from the welfare
fund to cover cash advances to Marcelino Melegrito to be repaid upon the That decision of the Director is assailed in these special civil actions of
release of his credit union loan on March 8, 1973. certiorari and prohibition filed on February 10, 1978. The petitioners pray
that the four union officers be expelled.
According to the complainants, those disbursements were not authorized by
the board of directors. The case has been simplified by the admission of the private respondents in
page 13 of their memorandum that the Bureau of Labor Relations has
Respondents Manalad, Amparo and Puerto approved the payment of unquestionably the power to remove erring union officers under the last
retirement benefits amounting to (1) P3,500 to Miguel de Leon on June 21, paragraph of Article 242 of the Labor Code.
1976; (2) P7,000 to Eduardo Topacio on July 30, 1976 and (3) P7,000 to
Roberto Victoria on August 4, 1976. That paragraph provides that any violation of the rights and conditions of
union membership, as enumerated in paragraphs (a) to (p) of Article 242,
According to the complainants, the three employees did not deserve "shall be a ground for cancellation of union registration or expulsion of
retirement benefits because they had been dismissed for prolonged absences officer from office, whichever is appropriate. At least thirty percent (30%) of
and they had ceased to be members of the Welfare Plan. all the members of a union or any member or members specially concerned
may report such violation to the Bureau (of Labor Relations). The Bureau
Membership in another union. - Respondents Manalad, Amparo and Puerto shall have the power to hear and decide any reported violation to mete the
are also officers of the Philippine Technical Clerical Commercial Employees appropriate penalty".
Association, another labor union.
Nevertheless, the private respondents qualify their admission with the
Their membership in the latter union is manifestly violative of section 9, opinion that the Bureau of Labor Relations should remove the guilty union
article III of the constitution and by-laws of the arrastre checkers' union officers only when the members could not do so under the union's
which provides that an elected officer shall be deemed disqualified if he constitution and by-laws and that the removal should be subject to review by
becomes a member of another organization. the Minister of Labor.

In this connection, the complainants presented evidence to prove that because The Office of the Solicitor General, as amicus curiae, has taken the
of that interlocking stewardship of the arrastre checkers' union and the other unqualified stand that the Bureau is empowered to expel from the union any
union, the respondents improperly channeled to the latter funds of the arrastre officer found guilty of violating any of the rights and conditions of union
checkers' union. membership specified in article 242.
In this appeal, the Director of Labor Relations maintains his view that the
power of removal belongs to the union members, since the power to choose
the officers belongs to them, and that the med-arbiter and the Director should
simply assist the union members in enforcing its constitution and by-laws.

We hold that the Labor Arbiter did not err in removing the respondents as
union officers. The membership of Manalad and Puerto in another union is a
sufficient ground for their removal under the constitution and by-laws of the
union. In Manalad's case, his organization of a family-owned corporation
competing with the union headed by him renders it untenable that he should
remain as union president.

We hold further that Manalad, Puerto and Leaño violated the rights and
conditions of membership in the union within the meaning of article 242.
Hence, on that ground their expulsion from office is also justified.

The petitioners are entitled to the refund of the union dues illegally collected
from them. The union should make the proper refund.

The Director of Labor Relations erred in holding that, as a matter of policy,


the tenure of union officers, being a "political question", is, generally, a
matter outside his Bureau's jurisdiction and should be passed upon by the
union members themselves.

After hearing and even without submitting the matter to the union members,
erring union officials may be removed by the Director of Labor Relations as
clearly provided in article 242.

The Director should apply the law and not make policy considerations prevail
over its clear intent and meaning "The majority of the laws need no
interpretation or construction. They require only application, and if there
were more application and less construction, there would be more stability in
the law, and more people would know what the law is." (Lizarraga Hermanos
vs. Yap Tico, 24 Phil. 504, 513).

The labor officials should not hesitate to enforce strictly the law and
regulations governing trade unions even if that course of action would curtail
the so-called union autonomy and freedom from government interference.

For the protection of union members and in order that the affairs of the union
may be administered honestly, labor officials should be vigilant and watchful
in monitoring and checking the administration of union affairs.

Laxity, permissiveness, neglect and apathy in supervising and regulating the


activities of union officials would result in corruption and oppression.
Internal safeguards within the union can easily be ignored or swept aside by
abusive, arrogant and unscrupulous union officials to the prejudice of the
members.

It is necessary and desirable that the Bureau of Labor Relations and the
Ministry of Labor should exercise close and constant supervision over labor
unions, particularly the handling of their funds, so as to forestall abuses and
venalities.

Hence, the Director acted correctly in ordering an examination of the books


and records of the union. The examination should include a verification of
the charge that the petty loans extended by the union to its members were
usurious and that the fee for the issuance of checks is unwarranted since the
loans were made in cash.

WHEREFORE, (1) that portion of the decision of the med-arbiter, removing


respondents Manalad, Leaño and Puerto as union officers, is affirmed.
(Respondent Amparo is no longer an officer of the union.)

(2) We also affirm that portion of the decision of the Director of Labor
Relations, directing the Bureau's Labor Organization Division to examine the
books of accounts and records of the Associated Port Checkers and Workers
Union and to submit a report on such examination within a reasonable time.

(3) We declare that the five petitioners are entitled to a refund of the union
dues illegally collected from them. The Director of Labor Relations is
ordered to require the union to make the refund within twenty days from
notice to his counsel of the entry of judgment in this case.
Costs against the private respondents.

SO ORDERED.
PHILIPPINE SKYLANDERS, INC. vs. NATIONAL LABOR PSI, its president Mariles C. Romulo, and its personnel manager Dakila
RELATIONS COMMISSION moved for the dismissal of the complaint on the ground that the issue of
G.R. No. 127374 | 2002-01-31 disaffiliation was an inter-union conflict which lay beyond the jurisdiction of
BELLOSILLO, J.: the Labor Arbiter. On the other hand, PSEA-NCW took the cudgels for its
officers who were being sued in their capacities as former officers of PSEA-
This is a petition for certiorari[1] seeking to set aside the 31 July 1996 PAFLU and asserted that since PSEA was no longer affiliated with PAFLU,
Decision[2] of the National Labor Relations Commission affirming the 30 Ayroso or PAFLU for that matter had no personality to file the instant
June 1995 Decision of the Labor Arbiter holding petitioners Philippine complaint. In support of this assertion, PSEA-NCW submitted in evidence a
Skylanders, Inc., Mariles C. Romulo[3] and Francisco Dakila as well as the Katunayan signed by 111 out of 120 rank and file employees of PSI
elected officers of the Philippine Skylanders Employees and Workers disauthorizing Ayroso or PAFLU from instituting any action in their
Association-PAFLU[4] guilty of unfair labor practice and ordering them to behalf.[9]
pay private respondent Philippine Association of Free Labor Union (PAFLU)
September[5] P150,000.00 as damages. Petitioners likewise seek the reversal In a Decision rendered on 30 June 1995 the Labor Arbiter declared PSEA's
of the 31 October 1996 Resolution of the NLRC denying their Motion for disaffiliation from PAFLU invalid and held PSI, PSEA-PAFLU and their
Reconsideration. respective officers guilty of unfair labor practice. The Decision explained
that despite PSEA-PAFLU's status as the sole and exclusive bargaining agent
In November 1993 the Philippine Skylanders Employees Association of PSI's rank and file employees, the company knowingly sanctioned and
(PSEA), a local labor union affiliated with the Philippine Association of Free confederated with Dakila in actively assisting a rival union. This, according
Labor Unions (PAFLU) September (PAFLU), won in the certification to the Labor Arbiter, was a classic case of interference for which PSI could
election conducted among the rank and file employees of Philippine be held responsible. As PSEA-NCW's personality was not accorded
Skylanders, Inc. (PSI). Its rival union, Philippine Skylanders Employees recognition, its collective bargaining agreement with PSI was struck down
Association-WATU (PSEA-WATU) immediately protested the result of the for being invalid. Ayroso's legal personality to file the complaint was
election before the Secretary of Labor. sustained on the ratiocination that under the Labor Code no petition
questioning the majority status of the incumbent bargaining agent shall be
Several months later, pending settlement of the controversy, PSEA sent entertained outside of the sixty (60)-day period immediately before the
PAFLU a notice of disaffiliation citing as reason PAFLU's supposed expiry date of such five (5)-year term of the collective bargaining agreement
deliberate and habitual dereliction of duty toward its members. Attached to that the parties may enter into. Accordingly, judgment was rendered ordering
the notice was a copy of the resolution adopted and signed by the officers PSI, PSEA-PAFLU and their officers to pay PAFLU P150,000.00 in
and members of PSEA authorizing their local union to disaffiliate from its damages.[10]
mother federation.
PSI, PSEA and their respective officers appealed to the National Labor
PSEA subsequently affiliated itself with the National Congress of Workers Relations Commission (NLRC). But the NLRC upheld the Decision of the
(NCW), changed its name to Philippine Skylanders Employees Association Labor Arbiter and conjectured that since an election protest questioning
- National Congress of Workers (PSEA-NCW), and to maintain continuity PSEA-PAFLU's certification as the sole and exclusive bargaining agent was
within the organization, allowed the former officers of PSEA-PAFLU to pending resolution before the Secretary of Labor, PSEA could not validly
continue occupying their positions as elected officers in the newly-forged separate from PAFLU, join another national federation and subsequently
PSEA-NCW. enter into a collective bargaining agreement with its employer-company.[11]

On 17 March 1994 PSEA-NCW entered into a collective bargaining Petitioners separately moved for reconsideration but both motions were
agreement with PSI which was immediately registered with the Department denied. Hence, these petitions for certiorari filed by PSI and PSEA-NCW
of Labor and Employment. together with their respective officers pleading for a reversal of the NLRC's
Decision which they claimed to have been rendered in excess of jurisdiction.
Meanwhile, apparently oblivious to PSEA's shift of allegiance, PAFLU In due time, both petitions were consolidated.
Secretary General Serafin Ayroso wrote Mariles C. Romulo requesting a
copy of PSI's audited financial statement. Ayroso explained that with the In these petitions, petitioner PSEA together with its officers argued that by
dismissal of PSEA-WATU's election protest the time was ripe for the parties virtue of their disaffiliation PAFLU as a mere agent had no authority to
to enter into a collective bargaining agreement. represent them before any proceedings. They further asserted that being an
independent labor union PSEA may freely serve the interest of all its
On 30 July 1994 PSI through its personnel manager Francisco Dakila denied members and readily disaffiliate from its mother federation when
the request citing as reason PSEA's disaffiliation from PAFLU and its circumstances so warrant. This right, they averred, was consistent with the
subsequent affiliation with NCW. constitutional guarantee of freedom of association.[12]

Agitated by PSI's recognition of PSEA-NCW, PAFLU through Serafin For their part, petitioners PSI, Romulo and Dakila alleged that their decision
Ayroso filed a complaint for unfair labor practice against PSI, its president to bargain collectively with PSEA-NCW was actuated, to a large extent, by
Mariles Romulo and personnel manager Francisco Dakila. PAFLU alleged PAFLU's behavior. Having heard no objections or protestations from
that aside from PSI's refusal to bargain collectively with its workers, the PAFLU relative to PSEA's disaffiliation, they reckoned that PSEA's
company through its president and personnel manager, was also liable for subsequent association with NSW was done bona fide.[13]
interfering with its employees' union activities.[6]
The Solicitor General filed a Manifestation in Lieu of Comment
Two (2) days later or on 6 October 1994 Ayroso filed another complaint in recommending that both petitions be granted. In his Manifestation, the
behalf of PAFLU for unfair labor practice against Francisco Dakila. Through Solicitor General argued against the Labor Arbiter's assumption of
Ayroso PAFLU claimed that Dakila was present in PSEA's organizational jurisdiction citing the following as reasons: first, there was no employer-
meeting thereby confirming his illicit participation in union activities. employee relationship between complainant Ayroso and PSI over which the
Ayroso added that the members of the local union had unwittingly fallen into Labor Arbiter could rightfully assert his jurisdiction; second, since the case
the manipulative machinations of PSI and were lured into endorsing a involved a dispute between PAFLU as mother federation and PSEA as local
collective bargaining agreement which was detrimental to their interests.[7] union, the controversy fell within the jurisdiction of the Bureau of Labor
The two (2) complaints were thereafter consolidated. Relations; and lastly, the relationship of principal-agent between PAFLU and
PSEA had been severed by the local union through the lawful exercise of its
On 1 February 1995 PAFLU amended its complaint by including the elected right of disaffiliation.[14]
officers of PSEA-PAFLU as additional party respondents. PAFLU averred
that the local officers of PSEA-PAFLU, namely Macario Cabanias, Pepito Stripped of non-essentials, the fundamental issue tapers down to the
Rodillas, Sharon Castillo, Danilo Carbonel, Manuel Eda, Rolando Felix, legitimacy of PSEA's disaffiliation. To be more precise, may PSEA, which
Jocelyn Fronda, Ricardo Lumba, Joseph Mirasol, Nerisa Mortel, Teofilo is an independent and separate local union, validly disaffiliate from PAFLU
Quirong, Leonardo Reyes, Manuel Cadiente, and Herminia Riosa, were pending the settlement of an election protest questioning its status as the sole
equally guilty of unfair labor practice since they brazenly allowed themselves and exclusive bargaining agent of PSI's rank and file employees?
to be manipulated and influenced by petitioner Francisco Dakila.[8]
At the outset, let it be noted that the issue of disaffiliation is an inter-union respondent Philippine Association of Free Labor Unions (PAFLU)
conflict the jurisdiction of which properly lies with the Bureau of Labor September (now UNIFIED PAFLU) as well as the Resolution of 31 October
Relations (BLR) and not with the Labor Arbiter.[15] Nonetheless, with due 1996 denying reconsideration is REVERSED and SET ASIDE. No costs.
recognition of this fact, we deem it proper to settle the controversy at this
instance since to remand the case to the BLR would only mean intolerable SO ORDERED.
delay for the parties.

The right of a local union to disaffiliate from its mother federation is not a
novel thesis unillumined by case law. In the landmark case of Liberty Cotton
Mills Workers Union vs. Liberty Cotton Mills, Inc.[16] we upheld the right
of local unions to separate from their mother federation on the ground that as
separate and voluntary associations, local unions do not owe their creation
and existence to the national federation to which they are affiliated but,
instead, to the will of their members. The sole essence of affiliation is to
increase, by collective action, the common bargaining power of local unions
for the effective enhancement and protection of their interests. Admittedly,
there are times when without succor and support local unions may find it
hard, unaided by other support groups, to secure justice for themselves.

Yet the local unions remain the basic units of association, free to serve their
own interests subject to the restraints imposed by the constitution and by-
laws of the national federation, and free also to renounce the affiliation upon
the terms laid down in the agreement which brought such affiliation into
existence.

Such dictum has been punctiliously followed since then.[17]

Upon an application of the aforecited principle to the issue at hand, the


impropriety of the questioned Decisions becomes clearly apparent. There is
nothing shown in the records nor is it claimed by PAFLU that the local union
was expressly forbidden to disaffiliate from the federation nor were there any
conditions imposed for a valid breakaway. As such, the pendency of an
election protest involving both the mother federation and the local union did
not constitute a bar to a valid disaffiliation. Neither was it disputed by
PAFLU that 111 signatories out of the 120 members of the local union, or an
equivalent of 92.5% of the total union membership supported the claim of
disaffiliation and had in fact disauthorized PAFLU from instituting any
complaint in their behalf. Surely, this is not a case where one (1) or two (2)
members of the local union decided to disaffiliate from the mother federation,
but it is a case where almost all local union members decided to disaffiliate.

It was entirely reasonable then for PSI to enter into a collective bargaining
agreement with PSEA-NCW. As PSEA had validly severed itself from
PAFLU, there would be no restrictions which could validly hinder it from
subsequently affiliating with NCW and entering into a collective bargaining
agreement in behalf of its members.

There is a further consideration that likewise argues for the granting of the
petitions. It stands unchallenged that PAFLU instituted the complaint for
unfair labor practice against the wishes of workers whose interests it was
supposedly protecting. The mere act of disaffiliation did not divest PSEA of
its own personality; neither did it give PAFLU the license to act
independently of the local union. Recreant to its mission, PAFLU cannot
simply ignore the demands of the local chapter and decide for its welfare.
PAFLU might have forgotten that as an agent it could only act in
representation of and in accordance with the interests of the local union. The
complaint then for unfair labor practice lodged by PAFLU against PSI, PSEA
and their respective officers, having been filed by a party which has no legal
personality to institute the complaint, should have been dismissed at the first
instance for failure to state a cause of action.

Policy considerations dictate that in weighing the claims of a local union as


against those of a national federation, those of the former must be preferred.
Parenthetically though, the desires of the mother federation to protect its
locals are not altogether to be shunned. It will however be to err greatly
against the Constitution if the desires of the federation would be favored over
those of its members. That, at any rate, is the policy of the law. For if it were
otherwise, instead of protection, there would be disregard and neglect of the
lowly workingmen.

WHEREFORE, the petitions of Philippine Skylanders, Inc. and of Philippine


Skylanders and Workers Association-NCW, together with their respective
officers, are GRANTED. The Decision of the National Labor Relations
Commission of 31 July 1996 affirming the Decision of the Labor Arbiter of
30 June 1995 holding petitioners Philippine Skylanders and Workers
Association-NCW, Philippine Skylanders, Inc. and their respective officers,
guilty of unfair labor practice and ordering them to pay damages to private
ERGONOMIC SYSTEMS PHILIPPINES, INC. VS. ENAJE
G.R. No. 195163 | 2017-12-13 Then, from 26 February 2002 to 2 March 2002, 10 union members, namely
MARTIRES, J.: Jaime Bentuco, Marina Cacao, Carlito Dela Cerna, Christopher Masagca,
Christopher Palomares, Rolando Patotoy, Aser Pesado, Jr., Leonilo Ricafort,
This is a petition for review on certiorari assailing the Decision,[1] dated 21 Felix Sanchez and Francis Santua did not report for work without official
September 2010, and Resolution,[2] dated 14 January 2011, of the Court of leave. The union members were required to submit their explanation why
Appeals (CA), in CA-G.R. SP No. 102802, which affirmed with modification they should not be sanctioned for their refusal to submit DPRs and
the decision,[3] dated 31 October 2007, and resolution,[4] dated 21 abandonment of work, but they either refused to receive the notices or
December 2007, of the National Labor Relations Commission (NLRC) in received them under protest. Further, they did not submit their explanation
NLRC NCR No. RAB IV-01-16813-03-L. The NLRC, in turn, affirmed the as required. Subsequently, for refusal to submit DPRs and for abandonment,
decision,[5] dated 31 January 2005, of Labor Arbiter Generoso V. Santos respondents-union members were issued letters of termination.[13] On 27
(LA) in NLRC NCR No. RAB IV-01-16813-03-L, a case for illegal dismissal January 2003, the respondents filed a complaint for illegal dismissal and
and unfair labor practice. unfair labor practice against ESPI, Phillip C. Ng, and Ma. Lourminda O. Ng
(petitioners).[14]
THE FACTS
The Labor Arbiter's Ruling
Respondents were union officers and members of Ergonomic System
Employees Union-Workers Alliance Trade Unions (local union). On 29 In a decision, dated 31 January 2005, the LA held that the local union was
October 1999, the local union entered into a Collective Bargaining the real party in interest and the Federation was merely an agent in the CBA;
Agreement (CBA)[6]with petitioner Ergonomic Systems Philippines, Inc. thus, the union officers and members who caused the implied disaffiliation
(ESPI),[7] which was valid for five (5) years or until October 2004. The local did not violate the union security clause. Consequently, their dismissal was
union, which was affiliated with Workers Alliance Trade Unions-Trade unwarranted. Nevertheless, the LA ruled that since ESPI effected the
Union Congress of the Philippines (Federation), was not independently dismissal in response to the Federation's demand which appeared to be
registered. Thus, on 15 November 2001, before the CBA expired, the union justified by a reading of the union security clause, it would be unjust to hold
officers secured the independent registration of the local union with the ESPI liable for the normal consequences of illegal dismissal.
Regional Office of the Department of Labor and Employment (DOLE). Later
on, the union officers were charged before the Federation and investigated The LA further opined that there was no ground for the dismissal of the union
for attending and participating in other union's seminars and activities using members because the refusal to submit DPRs and failure to report for work
union leaves without the knowledge and consent of the Federation and ESPI were meant to protest the dismissal of their officers, not to sever employer-
as well as in initiating and conspiring in the disaffiliation before the freedom employee relationship. He added that neither ESPI nor the respondents were
period.[8] at fault for they were merely protecting their respective interests. In sum, the
LA ordered all the respondents to return to work but without back wages.
On 10 January 2002, the Federation rendered a decision[9] finding The fallo reads:
respondents-union officers Emerito C. Enaje, Benedicto P. Abello, Alex M.
Malaylay, Francisco G. Encabo, Jr., Rico Samson, Rowena Betitio, Felipe N. WHEREFORE, premises considered, judgment is hereby rendered ordering
Custosa, Jaime A. Juatan, Leovino Mulintapang, Nelson L. Onte, Emiliano the complainants to report back to their former jobs within ten (10) days from
P. Rone, and Rolieto Llamado guilty of disloyalty. They were penalized with receipt of this Decision and the respondent company is in turn directed to
immediate expulsion from the Federation.[10] accept them back but without back wages. In the event however, that this is
no longer possible, the respondent company is ordered to pay the
On 11 January 2002, the Federation furnished ESPI with a copy of its complainants their separation pay computed at one-half (1/2) month salary
decision against respondents-union officers and recommended the for every year of service, a fraction of at least six (6) months to be considered
termination of their employment by invoking Sections 2 and 3, Article 2 of as one (1) whole year. The respondent is likewise ordered to pay
the CBA.[11] complainants attorney's fees equivalent to ten (10%) percent of the total
thereof as attorney's fees.
ESPI notified respondents-union officers of the Federation's demand and
gave them 48 hours to explain. Except for Nelson Onte, Emiliano Rone, and All other claims are dismissed for lack of merit.
Rico Samson, the rest of the officers refused to receive the notices.
Thereafter, on 20 February 2002, respondents-union officers were issued SO ORDERED.[15]
letters of termination, which they again refused to receive. On 26 February
2002, ESPI submitted to the DOLE a list of the dismissed employees. On the Unconvinced, petitioners and respondents appealed before the NLRC.
same day, the local union filed a notice of strike with the National
Conciliation and Mediation Board (NCMB).[12] The NLRC Ruling

From 21 February to 23 February 2002, the local union staged a series of In a decision, dated 31 October 2007, the NLRC affirmed the ruling of the
noise barrage and "slow down" activities. Meanwhile, on 22 February 2002, LA. It adjudged that the dismissal of the union officers was effected only in
40 union members identified as: Amorpio Adriano, Jimmy Alcantara, response to the demand of the Federation and to comply with the union
Bernardo Antoni, Herminito Bedrijo, Romeo Belarmino, Yolanda Canopin, security clause under the CBA. The NLRC concluded that since there was no
Almelito Cuabo, Ricardo Del Pilar, Elmer Desquitado, Winefredo disloyalty to the union, but only disaffiliation from the Federation which was
Desquitado, Demetrio Diaz, Erick Ecraela, Quintero Enriquez, Crisanto a mere agent in the CBA, the cause for the respondents' dismissal was non-
Fernandez, Rommel Flores, Nelson Frias, Pedrito Geron, Dominador existent. It disposed the case in this wise:
Guimaldo, Ambrosio Henarez, Terencio Henares, Albert Lachica, Alberto
Lorenzo, Joel Malaylay, Susan Malbas, Rolando Manaril, Teddy Montible, WHEREFORE, premises considered, the appeals separately filed by
Fernando Ofaldo, Ronie Olivay, Raul Pagolong, Lorenzo Raniego, Amado complainants and respondents from the Decision of Labor Arbiter Generoso
Samson-Ty, Roel Soriano, Jonathan Sualibio, Esteban Sumicao, Joseph V. Santos dated January 31, 2005 are both DISMISSED for lack of merit.
Tabaday, Epifanio Tabarez, Regie Toting, Reynaldo Toting, Norman
Valenzuela and Rolando Yonson refused to submit their Daily Production The appeal filed by complainants from the Order dated January 4, 2007 is
Reports (DPRs). likewise DISMISSED for lack of merit.

On 26 February 2002, 28 union members namely Dioscoro Balajadia, Nerry The assailed Orders are hereby AFFIRMED.
Balinas, Noel Balmeo, Arnaldo Castro, Geroncio Dela Cueva, Alberto
Gapasin, Julius Genova, Loreto Gracilla, Roberto Ingiente, Jr., Roque Joven, SO ORDERED.[16]
Paterno Linogo, Isagani Masangka, Angelito Montilla, Pecifico Nigparanon,
Salvador Nobe, Manuel Oavenga, Reynaldo Ortiz, Romeo Quintana, Jemard Undeterred, petitioners and respondents moved for reconsideration. Their
Remotin, Reynaldo Roblesa, Samuel Rosales, Roberto Santos, Ronaldo motions, however, were denied by the NLRC in a resolution, dated 21
Santos, Rocky Talolong, Emilio Tonga, Bernardo Valdez, Dante Velasco and December 2007.
Rene Vicente abandoned their work and held a picket line outside the
premises of ESPI. The CA Ruling
be answered is whether the Federation may invoke the union security clause
In its decision, dated 21 September 2010, the CA affirmed with modification in the CBA.
the NLRC ruling. It held that ESPI and the respondents acted in good faith
when the former dismissed the latter and when the latter, in tum, staged a "Union security is a generic term, which is applied to and comprehends
strike without complying with the legal requirements. The CA, however, 'closed shop,' 'union shop,' 'maintenance of membership,' or any other form
pronounced that the concept of separation pay as an alternative to of agreement which imposes upon employees the obligation to acquire or
reinstatement holds true only in cases wherein there is illegal dismissal, a fact retain union membership as a condition affecting employment. There is
which does not exist in this case. The dispositive portion reads: union shop when all new regular employees are required to join the union
within a certain period as a condition for their continued employment. There
WHEREFORE, the instant petition is PARTIALLY GRANTED. The is maintenance of membership shop when employees, who are union
Decision of the Labor Arbiter, as sustained by the National Labor Relations members as of the effective date of the agreement, or who thereafter become
Commission, reverting the employer-employee position of the parties to the members, must maintain union membership as a condition for continued
status quo ante is AFFIRMED, with MODIFICATION, in that the provision employment until they are promoted or transferred out of the bargaining unit,
on the award of separation pay in lieu of reinstatement is deleted. or the agreement is terminated. A closed shop, on the other hand, may be
defined as an enterprise in which, by agreement between the employer and
SO ORDERED.[17] his employees or their representatives, no person may be employed in any or
certain agreed departments of the enterprise unless he or she is, becomes,
Aggrieved, petitioners and respondents moved for reconsideration but the and, for the duration of the agreement, remains a member in good standing
same was denied by the CA in a resolution, dated 14 January 2011. of a union entirely comprised of or of which the employees in interest are a
part."[21]
Hence, this petition.
Before an employer terminates an employee pursuant to the union security
ISSUES clause, it needs to determine and prove that: (1) the union security clause is
applicable; (2) the union is requesting the enforcement of the union security
I. WHETHER THE FEDERATION MAY INVOKE THE UNION provision in the CBA; and (3) there is sufficient evidence to support the
SECURITY CLAUSE IN DEMANDING THE RESPONDENTS' decision of the union to expel the employee from the union.[22]
DISMISSAL;
In this case, the primordial requisite, i.e., the union is requesting the
II. WHETHER THE STRIKE CONDUCTED BY THE RESPONDENTS enforcement of the union security provision in the CBA, is clearly lacking.
COMPLIED WITH THE LEGAL REQUIREMENTS; Under the Labor Code, a chartered local union acquires legal personality
through the charter certificate issued by a duly registered federation or
III.WHETHER THE RESPONDENTS' DISMISSAL FROM national union and reported to the Regional Office.[23] "A local union does
EMPLOYMENT WAS VALID. not owe its existence to the federation with which it is affiliated. It is a
separate and distinct voluntary association owing its creation to the will of
The petitioners argue that the respondents failed to comply with two (2) of its members. Mere affiliation does not divest the local union of its own
the procedural requirements for a valid strike, i.e., taking of a strike vote and personality, neither does it give the mother federation the license to act
observance of the seven-day period after submission of the strike vote report; independently of the local union. It only gives rise to a contract of agency,
that mere participation of union officers in the illegal strike is a ground for where the former acts in representation of the latter. Hence, local unions are
termination of employment; that the union members committed illegal acts considered principals while the federation is deemed to be merely their
during the strike which warranted their dismissal, i.e., obstruction of the free agent."[24]
ingress to and egress from ESPI's premises and commission of acts of
violence, coercion or intimidation; that the respondents are not entitled to The union security clause in the CBA between ESPI and the local union
reinstatement or separation pay because they were validly dismissed from provides:
employment; that the union members who unjustly refused to submit their
DPRs and abandoned their work were rightfully terminated because their acts SECTION 1. Union Shop. All regular, permanent employees covered by this
constituted serious misconduct or willful disobedience of lawful orders; and Agreement who are members of the UNION as of the date of effectivity of
that reinstatement is no longer possible because the industrial building owned this Agreement as well as any employees who shall subsequently become
by Ergo Contracts Philippines, Inc. was totally destroyed by fire on 6 members of the UNION during the lifetime of this Agreement or any
February 2005.[18] extension, thereof, shall as a condition of continued employment, maintain
their membership in the UNION during the term of this Agreement or any
In their comment,[19] the respondents counter that they were not legally extension thereof.
terminated because the grounds relied upon by the petitioners were non-
existent; that as ruled by the NLRC, they merely disaffiliated from the xxxx
Federation but they were not disloyal to the local union; that reinstatement is
not physically impossible because it was the industrial building owned by SECTION 3. The COMPANY shall terminate the services of any concerned
Ergo Contracts Philippines, Inc. that was gutted down by fire, not that of employee when so requested by the UNION for any of the following reasons:
ESPI; that even if the manufacturing plant of ESPI was indeed destroyed by
fire, the petitioners have other offices around the country where the a. Voluntary Resignation from the Union during the term of this Agreement
respondents may be reinstated; and that having failed to comply with the or any extension thereof;
order to reinstate them and having ceased operations, the petitioners must be
ordered to pay their separation pay. b. Non-payment of membership fee, regular monthly dues, mutual aid benefit
and other assessments submitted by the UNION to the COMPANY;
In their reply,[20] the petitioners aver that the respondents violated the union
security clause under the CBA; that their termination was effected in c. Violation of the UNION Constitution and Bylaws. The UNION shall
response to the Federation's demand to dismiss them; that they did not furnish the COMPANY a copy of their Constitution and Bylaws and any
comply with the requisites of a valid strike; that they refused to submit their amendment thereafter.
DPRs and abandoned their work; and that the award of separation pay had
no basis because the respondents had been legally dismissed from their d. Joining of another Union whose interest is adverse to the UNION,
employment. AWATU, during the lifetime of this Agreement.

THE COURT'S RULING e. Other acts which are inimical to the interests of the UNION and
AWATU.[25]
Only the local union may invoke the union security clause in the CBA.
There is no doubt that the union referred to in the foregoing provisions is the
The controversy between ESPI and the respondents originated from the Ergonomic Systems Employees Union or the local union as provided in
Federation's act of expelling the union officers and demanding their dismissal Article I of the CBA.[26] A perusal of the CBA shows that the local union,
from ESPI. Thus, to arrive at a proper resolution of this case, one question to not the Federation, was recognized as the sole and exclusive collective
bargaining agent for all its workers and employees in all matters concerning illegal acts during a strike may be declared to have lost his employment
wages, hours of work, and other terms and conditions of employment. status: Provided, That mere participation of a worker in a lawful strike shall
Consequently, only the union may invoke the union security clause in case not constitute sufficient ground for termination of his employment, even if a
any of its members commits a violation thereof. Even assuming that the union replacement had been hired by the employer during such lawful strike.
officers were disloyal to the Federation and committed acts inimical to its
interest, such circumstance did not give the Federation the prerogative to In the determination of the consequences of illegal strikes, the law makes a
demand the union officers' dismissal pursuant to the union security clause distinction between union members and union officers. The services of an
which, in the first place, only the union may rightfully invoke. Certainly, it ordinary union member cannot be terminated for mere participation in an
does not give the Federation the privilege to act independently of the local illegal strike; proof must be adduced showing that he or she committed illegal
union. At most, what the Federation could do is to refuse to recognize the acts during the strike. A union officer, on the other hand, may be dismissed,
local union as its affiliate and revoke the charter certificate it issued to the not only when he actually commits an illegal act during a strike, but also if
latter. In fact, even if the local union itself disaffiliated from the Federation, he knowingly participates in an illegal strike.[38]
the latter still has no right to demand the dismissal from employment of the
union officers and members because concomitant to the union's prerogative In the present case, respondents-union officers stand to be dismissed as they
to affiliate with a federation is its right to disaffiliate therefrom which the conducted a strike despite knowledge that a strike vote had not yet been
Court explained in Philippine Skylanders, Inc. v. NLRC,[27] viz: approved by majority of the union and the corresponding strike vote report
had not been submitted to the NCMB.
The right of a local union to disaffiliate from its mother federation is not a
novel thesis unillumined by case law. In the landmark case of Liberty Cotton With respect to respondents-union members, the petitioners merely alleged
Mills Workers Union vs. Liberty Cotton Mills, Inc., we upheld the right of that they committed illegal acts during the strike such as obstruction of
local unions to separate from their mother federation on the ground that as ingress to and egress from the premises of ESPI and execution of acts of
separate and voluntary associations, local unions do not owe their creation violence and intimidation. There is, however, a dearth of evidence to prove
and existence to the national federation to which they are affiliated but, such claims. Hence, there is no basis to dismiss respondents-union members
instead, to the will of their members. The sole essence of affiliation is to from employment on the ground that they committed illegal acts during the
increase, by collective action, the common bargaining power of local unions strike.
for the effective enhancement and protection of their interests. Admittedly,
there are times when without succor and support local unions may find it Dismissed respondents-union members are not entitled to back wages.
hard, unaided by other support groups, to secure justice for themselves.
While it is true that the award of back wages is a legal consequence of a
Yet the local unions remain the basic units of association, free to serve their finding of illegal dismissal, in G & S Transport Corporation v. Infante,[39]
own interests subject to the restraints imposed by the constitution and bylaws the Court pronounced that the dismissed workers are entitled only to
of the national federation, and free also to renounce the affiliation upon the reinstatement considering that they did not render work for the employer
terms laid down in the agreement which brought such affiliation into during the strike, viz:
existence.[28]
With respect to back wages, the principle of a "fair day's wage for a fair day's
In sum, the Federation could not demand the dismissal from employment of labor" remains as the basic factor in determining the award thereof. If there
the union officers on the basis of the union security clause found in the CBA is no work performed by the employee there can be no wage or pay unless,
between ESPI and the local union. of course, the laborer was able, willing and ready to work but was illegally
locked out, suspended or dismissed or otherwise illegally prevented from
A strike is deemed illegal for failure to take a strike vote and to submit a working. While it was found that respondents expressed their intention to
report thereon to the NCMB. report back to work, the latter exception cannot apply in this case. In
Philippine Marine Officers' Guild v. Compañia Maritima, as affirmed in
A strike is the most powerful weapon of workers in their struggle with Philippine Diamond Hotel and Resort v. Manila Diamond Hotel Employees
management in the course of setting their terms and conditions of Union, the Court stressed that for this exception to apply, it is required that
employment. As such, it either breathes life to or destroys the union and its the strike be legal, a situation that does not obtain in the case at bar.[40]
members.[29] (emphases supplied)

Procedurally, for a strike to be valid, it must comply with Article 278[30] of Thus, in the case at bar, respondents-union members' reinstatement without
the Labor Code, which requires that: (a) a notice of strike be filed with the back wages suffices for the appropriate relief. Fairness and justice dictate that
NCMB 30 days before the intended date thereof, or 15 days in case of unfair back wages be denied the employees who participated in the illegal concerted
labor practice; (b) a strike vote be approved by a majority of the total union activities to the great detriment of the employer.[41]
membership in the bargaining unit concerned, obtained by secret ballot in a
meeting called for that purpose; and (c) a notice be given to the NCMB of Nevertheless, separation pay is made an alternative relief in lieu of
the results of the voting at least seven days before the intended strike. These reinstatement in certain circumstances, like: (a) when reinstatement can no
requirements are mandatory, and the union's failure to comply renders the longer be effected in view of the passage of a long period of time or because
strike illegal.[31] of the realities of the situation; (b) reinstatement is inimical to the employer's
interest; (c) reinstatement is no longer feasible; (d) reinstatement does not
The union filed a notice of strike on 20 February 2002.[32] The strike serve the best interests of the parties involved; (e) the employer is prejudiced
commenced on 21 February 2002.[33] The strike vote was taken on 2 April by the workers' continued employment; (f) facts that make execution unjust
2002[34] and the report thereon was submitted to the NCMB on 4 April or inequitable have supervened; or (g) strained relations between the
2002.[35] Indeed, the first requisite or the cooling-off period need not be employer and employee.[42]
observed when the ground relied upon for the conduct of strike is union-
-busting.[36] Nevertheless, the second and third requirements are still Given the lapse of considerable time from the occurrence of the strike, the
mandatory. In this case, it is apparent that the union conducted a strike Court rules that the award of separation pay of one (1) month salary for each
without seeking a strike vote and without submitting a report thereon to the year of service, in lieu of reinstatement, is in order. This relief strikes a
DOLE. Thus, the strike which commenced on 21 February 2002 was illegal. balance between the respondents-union members who may not have known
that they were participating in an illegal strike but who, nevertheless, have
Liabilities of union officers and members rendered service to the company for years prior to the illegal strike which
caused a rift in their relations, and the employer who definitely suffered
Article 279(a)[37] of the Labor Code provides: losses on account of respondents-union members' failure to report to work
during the illegal strike.
Art. 279. Prohibited activities. - (a) x x x
WHEREFORE, the petition is PARTIALLY GRANTED. The 21 September
xxxx 2010 Decision and 14 January 2011 Resolution of the Court of Appeals in
CA-G.R. SP No. 102802 are AFFIRMED with MODIFICATION in that
Any union officer who knowingly participates in an illegal strike and any petitioners are hereby ORDERED to pay each of the above-named individual
worker or union officer who knowingly participates in the commission of respondents, except union officers who are hereby declared validly
dismissed, separation pay equivalent to one (1) month salary for every year
of service. Whatever sums already received from petitioners under any
release, waiver or quitclaim shall be deducted from the total separation pay
due to each of them.

SO ORDERED.
COASTAL SUBIC BAY TERMINAL, INC. VS. DEPARTMENT OF
LABOR and EMPLOYMENT 1. COASTAL SUBIC BAY TERMINAL, INC. SUPERVISORY
G.R. No. 157117 | 2006-11-20 EMPLOYEES UNION-APSOTEU; and
QUISUMBING, J.:
2. NO UNION.
For review on certiorari is the Court of Appeals' Decision[1] dated August
31, 2001, in CA-G.R. SP No. 54128 and the Resolution[2] dated February 5, The latest payroll of the employer, including its payrolls for the last three
2003, denying petitioner's motion for reconsideration. The Court of Appeals months immediately preceding the issuance of this decision, shall be the basis
had affirmed the Decision[3] dated March 15, 1999 of the Secretary of the for determining the qualified list of voters.
Department of Labor and Employment (DOLE) reversing the Mediator
Arbiter's dismissal of private respondents' petitions for certification election. SO DECIDED.[6]

The facts are as follows: The motion for reconsideration was also denied.[7]

On July 8, 1998, private respondents Coastal Subic Bay Terminal, Inc. Rank- On appeal, the Court of Appeals affirmed the decision of the Secretary.[8] It
and-File Union (CSBTI-RFU) and Coastal Subic Bay Terminal, Inc. held that there was no grave abuse of discretion on the part of the Secretary;
Supervisory Union (CSBTI-SU) filed separate petitions for certification its findings are supported by evidence on record; and thus should be accorded
election before Med-Arbiter Eladio de Jesus of the Regional Office No. III. with respect and finality.[9]
The rank-and-file union insists that it is a legitimate labor organization
having been issued a charter certificate by the Associated Labor Union The motion for reconsideration was likewise denied.[10] Hence, the instant
(ALU), and the supervisory union by the Associated Professional, petition by the company anchored on the following grounds:
Supervisory, Office and Technical Employees Union (APSOTEU). Private
respondents also alleged that the establishment in which they sought to I
operate was unorganized.
THE HONORABLE COURT OF APPEALS ERRED IN RELYING ON
Petitioner Coastal Subic Bay Terminal, Inc. (CSBTI) opposed both petitions THE "1989 REVISED RULES AND REGULATIONS IMPLEMENTING
for certification election alleging that the rank-and-file union and supervisory RA 6715" AS BASIS TO RECOGNIZE PRIVATE RESPONDENT
union were not legitimate labor organizations, and that the proposed APSOTEU'S REGISTRATION BY THE DOLE REGIONAL DIRECTOR.
bargaining units were not particularly described.
II
Without ruling on the legitimacy of the respondent unions, the Med-Arbiter
dismissed, without prejudice to refiling, both petitions which had been THE HONORABLE COURT OF APPEALS ERRED WHEN IT
consolidated. The Med-Arbiter held that the ALU and APSOTEU are one AFFIRMED PUBLIC RESPONDENT'S APPLICATION OF THE
and the same federation having a common set of officers. Thus, the PRINCIPLE OF STARE DECISIS TO HASTILY DISPOSE OF THE
supervisory and the rank-and-file unions were in effect affiliated with only LEGAL PERSONALITY ISSUE OF APSOTEU.
one federation.[4]
III
The Med-Arbiter ruled as follows:
THE HONORABLE COURT OF APPEALS DID NOT DECIDE IN
Viewed in the light of all the foregoing, this Office finds the simultaneous ACCORD WITH LAW AND JURISPRUDENCE WHEN IT AFFIRMED
filing of the instant petitions to be invalid and unwarranted. Consequently, PUBLIC RESPONDENT'S APPLICATION OF THE "UNION
this Office has no recourse but to dismiss both petitions without prejudice to AUTONOMY" THEORY.
the refiling of either.
IV
WHEREFORE, PREMISES CONSIDERED, let the instant petitions be, as
they are hereby DISMISSED. IN AFFIRMING PUBLIC RESPONDENT'S FINDING THAT PRIVATE
RESPONDENTS ARE "SEPARATE FEDERATIONS," THE
SO ORDERED.[5] HONORABLE COURT OF APPEALS:

Both parties appealed to the Secretary of Labor and Employment, who (1) IGNORED JURISPRUDENCE RECOGNIZING THE BINDING
reversed the decision of the Med-Arbiter. The Secretary thru Undersecretary NATURE OF A MED-ARBITER'S FACTUAL FINDINGS; AND
R. Baldoz, ruled that CSBTI-SU and CSBTI-RFU have separate legal
personalities to file their separate petitions for certification election. The (2) DISREGARDED EVIDENCE ON RECORD OF "ILLEGAL
Secretary held that APSOTEU is a legitimate labor organization because it COMMINGLING."[11]
was properly registered pursuant to the 1989 Revised Rules and Regulations
implementing Republic Act No. 6715, the rule applicable at the time of its Plainly, the issues are (1) Can the supervisory and the rank-and-file unions
registration. It further ruled that ALU and APSOTEU are separate and file separate petitions for certification election?; (2) Was the Secretary's
distinct labor unions having separate certificates of registration from the decision based on stare decisis correct?; and (3) Were private respondents
DOLE. They also have different sets of locals. The Secretary declared engaged in commingling?
CSBTI-RFU and CSBTI-SU as legitimate labor organizations having been
chartered respectively by ALU and APSOTEU after submitting all the The issue on the status of the supervisory union CSBTI-SU depends on the
requirements with the Bureau of Labor Relations (BLR). Accordingly, the status of APSOTEU, its mother federation.
Secretary ordered the holding of separate certification election, viz:
Petitioner argues that APSOTEU improperly secured its registration from the
WHEREFORE, the decision of the Med-Arbiter, Regional Office No. III is DOLE Regional Director and not from the BLR; that it is the BLR that is
hereby REVERSED. Let separate certification elections be conducted authorized to process applications and issue certificates of registration in
immediately among the appropriate employees of CSBTI, after the usual pre- accordance with our ruling in Phil. Association of Free Labor Unions v.
election conference, with the following choices: Secretary of Labor;[12] that the certificates of registration issued by the
DOLE Regional Director pursuant to the rules are questionable, and possibly
I. For all rank and file employees of CSBTI: even void ab initio for being ultra vires; and that the Court of Appeals erred
when it ruled that the law applicable at the time of APSOTEU's registration
1. COASTAL SUBIC BAY TERMINAL, INC. RANK-AND-FILE UNION- was the 1989 Revised Implementing Rules and Regulations of Rep. Act No.
ALU-TUCP; and 6715.

2. NO UNION. Petitioner insists that APSOTEU lacks legal personality, and its chartered
affiliate CSBTI-SU cannot attain the status of a legitimate labor organization
II. For all supervisory employees of CSBTI: to file a petition for certification election. It relies on Villar v. Inciong,[13]
where we held therein that Amigo Employees Union was not a duly Thus, APSOTEU is a legitimate labor organization and has authority to issue
registered independent union absent any record of its registration with the charter to its affiliates.[22] It may issue a local charter certificate to CSBTI-
Bureau. SU and correspondingly, CSBTI-SU is legitimate.

Pertinent is Article 235[14] of the Labor Code which provides that Are ALU, a rank-and-file union and APSOTEU, a supervisory union one and
applications for registration shall be acted upon by the Bureau. "Bureau" as the same because of the commonalities between them? Are they
defined under the Labor Code means the BLR and/or the Labor Relations commingled?
Division in the Regional Offices of the Department of Labor.[15] Further,
Section 2, Rule II, Book V of the 1989 Revised Implementing Rules of the The petitioner contends that applying by analogy, the doctrine of piercing the
Labor Code (Implementing Rules) provides that: veil of corporate fiction, APSOTEU and ALU are the same federation.
Private respondents disagree.
Section 2. Where to file application; procedure - Any national labor
organization or labor federation or local union may file an application for First, as earlier discoursed, once a labor union attains the status of a legitimate
registration with the Bureau or the Regional Office where the applicant's labor organization, it continues as such until its certificate of registration is
principal offices is located. The Bureau or the Regional Office shall cancelled or revoked in an independent action for cancellation.[23] In
immediately process and approve or deny the application. In case of addition, the legal personality of a labor organization cannot be collaterally
approval, the Bureau or the Regional Office shall issue the registration attacked.[24] Thus, when the personality of the labor organization is
certificate within thirty (30) calendar days from receipt of the application, questioned in the same manner the veil of corporate fiction is pierced, the
together with all the requirements for registration as hereinafter provided. action partakes the nature of a collateral attack. Hence, in the absence of any
[16] independent action for cancellation of registration against either APSOTEU
or ALU, and unless and until their registrations are cancelled, each continues
The Implementing Rules specifically Section 1, Rule III of Book V, as to possess a separate legal personality. The CSBTI-RFU and CSBTI-SU are
amended by Department Order No. 9, thus: therefore affiliated with distinct and separate federations, despite the
commonalities of APSOTEU and ALU.
SECTION 1. Where to file applications. - The application for registration of
any federation, national or industry union or trade union center shall be filed Under the rules implementing the Labor Code, a chartered local union
with the Bureau. Where the application is filed with the Regional Office, the acquires legal personality through the charter certificate issued by a duly
same shall be immediately forwarded to the Bureau within forty-eight (48) registered federation or national union, and reported to the Regional Office
hours from filing thereof, together with all the documents supporting the in accordance with the rules implementing the Labor Code.[25] A local union
registration. does not owe its existence to the federation with which it is affiliated. It is a
separate and distinct voluntary association owing its creation to the will of
The applications for registration of an independent union shall be filed with its members. Mere affiliation does not divest the local union of its own
and acted upon by the Regional Office where the applicant's principal office personality, neither does it give the mother federation the license to act
is located .... independently of the local union. It only gives rise to a contract of agency,
where the former acts in representation of the latter.[26] Hence, local unions
xxxx are considered principals while the federation is deemed to be merely their
agent.[27] As such principals, the unions are entitled to exercise the rights
The DOLE issued Department Order No. 40-03, which took effect on March and privileges of a legitimate labor organization, including the right to seek
15, 2003, further amending Book V of the above implementing rules. The certification as the sole and exclusive bargaining agent in the appropriate
new implementing rules explicitly provide that applications for registration employer unit.
of labor organizations shall be filed either with the Regional Office or with
the BLR.[17] A word of caution though, under Article 245 of the Labor Code,[28]
supervisory employees are not eligible for membership in a labor union of
Even after the amendments, the rules did not divest the Regional Office and rank-and-file employees. The supervisory employees are allowed to form
the BLR of their jurisdiction over applications for registration by labor their own union but they are not allowed to join the rank-and-file union
organizations. The amendments to the implementing rules merely specified because of potential conflicts of interest.[29] Further, to avoid a situation
that when the application was filed with the Regional Office, the application where supervisors would merge with the rank-and-file or where the
would be acted upon by the BLR. supervisors' labor union would represent conflicting interests, a local
supervisors' union should not be allowed to affiliate with the national
The records in this case showed that APSOTEU was registered on March 1, federation of unions of rank-and-file employees where that federation
1991. Accordingly, the law applicable at that time was Section 2, Rule II, actively participates in the union activity within the company.[30] Thus, the
Book V of the Implementing Rules, and not Department Order No. 9 which limitation is not confined to a case of supervisors wanting to join a rank-and-
took effect only on June 21, 1997. Thus, considering further that APSOTEU's file union. The prohibition extends to a supervisors' local union applying for
principal office is located in Diliman, Quezon City, and its registration was membership in a national federation the members of which include local
filed with the NCR Regional Office, the certificate of registration is valid. unions of rank-and-file employees.[31] In De La Salle University Medical
Center and College of Medicine v. Laguesma, we reiterated the rule that for
The petitioner misapplied Villar v. Inciong.[18] In said case, there was no the prohibition to apply, it is not enough that the supervisory union and the
record in the BLR that Amigo Employees Union was registered.[19] rank-and-file union are affiliated with a single federation. In addition, the
supervisors must have direct authority over the rank-and-file employees.[32]
Did the Court of Appeals err in its application of stare decisis when it upheld
the Secretary's ruling that APSOTEU is a legitimate labor organization and In the instant case, the national federations that exist as separate entities to
its personality cannot be assailed unless in an independent action for which the rank-and-file and supervisory unions are separately affiliated with,
cancellation of registration certificate?[20] do have a common set of officers. In addition, APSOTEU, the supervisory
federation, actively participates in the CSBTI-SU while ALU, the rank-and-
We think not. file federation, actively participates in the CSBTI-RFU, giving occasion to
possible conflicts of interest among the common officers of the federation of
Section 5, Rule V, Book V of the Implementing Rules states: rank-and-file and the federation of supervisory unions. For as long as they
are affiliated with the APSOTEU and ALU, the supervisory and rank-and-
Section 5. Effect of registration - The labor organization or workers' file unions both do not meet the criteria to attain the status of legitimate labor
association shall be deemed registered and vested with legal personality on organizations, and thus could not separately petition for certification
the date of issuance of its certificate of registration. Such legal personality elections.
cannot thereafter be subject to collateral attack, but maybe questioned only
in an independent petition for cancellation in accordance with these The purpose of affiliation of the local unions into a common enterprise is to
Rules.[21] increase the collective bargaining power in respect of the terms and
conditions of labor.[33] When there is commingling of officers of a rank-
and-file union with a supervisory union, the constitutional policy on labor is
circumvented. Labor organizations should ensure the freedom of employees
to organize themselves for the purpose of leveling the bargaining process but
also to ensure the freedom of workingmen and to keep open the corridor of
opportunity to enable them to do it for themselves.

WHEREFORE, the petition is GRANTED. The Court of Appeals' Decision


dated August 31, 2001, in CA-G.R. SP No. 54128 and the Resolution dated
February 5, 2003 are SET ASIDE. The decision of the Med-Arbiter is hereby
AFFIRMED.

SO ORDERED.
SAN MIGUEL CORPORATION EMPLOYEES UNION-PHILIPPINE members for purposes of collective bargaining agreement. On this basis,
TRANSPORT AND GENERAL WORKERS ORGANIZATION PDMP can charter or create a local, in accordance with the provisions of
(SMCEU-PTGWO) VS. SAN MIGUEL PACKAGING PRODUCTS Department Order No. 9.
EMPLOYEES UNION-PAMBANSANG DIWA NG
MANGGAGAWANG PILIPINO (SMPEOPLEEU-PDMP) WHEREFORE, the appeal is hereby GRANTED. Accordingly, the decision
G.R. No. 171153 | 2007-09-12 of the Regional Director dated July 14, 2000, canceling the registration of
CHICO-NAZARIO, J.: appellant San Miguel Packaging Products Employees Union-Pambansang
Diwa ng Manggagawang Pilipino (SMPPEU-PDMP) is REVERSED and
In this Petition for Review on Certiorari under Rule 45 of the Revised Rules SET ASIDE. Appellant shall hereby remain in the roster of legitimate labor
of Court, petitioner SAN MIGUEL CORPORATION EMPLOYEES organizations.[14]
UNION-PHILIPPINE TRANSPORT AND GENERAL WORKERS
ORGANIZATION (SMCEU-PTGWO) prays that this Court reverse and set
aside the (a) Decision[2] dated 9 March 2005 of the Court of Appeals in CA- While the BLR agreed with the findings of the DOLE Regional Director
G.R. SP No. 66200, affirming the Decision[3] dated 19 February 2001 of the dismissing the allegations of fraud and misrepresentation, and in upholding
Bureau of Labor Relations (BLR) of the Department of Labor and that PDMP can directly create a local or a chapter, it reversed the Regional
Employment (DOLE) which upheld the Certificate of Registration of Director's ruling that the 20% membership is a requirement for respondent to
respondent SAN MIGUEL PACKAGING PRODUCTS EMPLOYEES attain legal personality as a labor organization. Petitioner thereafter filed a
UNION-PAMBANSANG DIWA NG MANGGAGAWANG PILIPINO Motion for Reconsideration with the BLR. In a Resolution rendered on 19
(SMPPEU-PDMP); and (b) the Resolution[4] dated 16 January 2006 of the June 2001 in BLR-A-C-64-05-9-00 (NCR-OD-9908-007-IRD), the BLR
Court of Appeals in the same case, denying petitioner's Motion for denied the Motion for Reconsideration and affirmed its Decision dated 19
Reconsideration of the aforementioned Decision. February 2001.[15]

The following are the antecedent facts: Invoking the power of the appellate court to review decisions of quasi-
judicial agencies, petitioner filed with the Court of Appeals a Petition for
Petitioner is the incumbent bargaining agent for the bargaining unit Certiorari under Rule 65 of the 1997 Rules of Civil Procedure docketed as
comprised of the regular monthly-paid rank and file employees of the three CA-G.R. SP No. 66200. The Court of Appeals, in a Decision dated 9 March
divisions of San Miguel Corporation (SMC), namely, the San Miguel 2005, dismissed the petition and affirmed the Decision of the BLR, ruling as
Corporate Staff Unit (SMCSU), San Miguel Brewing Philippines (SMBP), follows:
and the San Miguel Packaging Products (SMPP), in all offices and plants of
SMC, including the Metal Closure and Lithography Plant in Laguna. It had
been the certified bargaining agent for 20 years - from 1987 to 1997. In Department Order No. 9, a registered federation or national union may
directly create a local by submitting to the BLR copies of the charter
Respondent is registered as a chapter of Pambansang Diwa ng certificate, the local's constitution and by-laws, the principal office address
Manggagawang Pilipino (PDMP). PDMP issued Charter Certificate No. 112 of the local, and the names of its officers and their addresses. Upon
to respondent on 15 June 1999.[5] In compliance with registration complying with the documentary requirements, the local shall be issued a
requirements, respondent submitted the requisite documents to the BLR for certificate and included in the roster of legitimate labor organizations. The
the purpose of acquiring legal personality.[6] Upon submission of its charter [herein respondent] is an affiliate of a registered federation PDMP, having
certificate and other documents, respondent was issued Certificate of been issued a charter certificate. Under the rules we have reviewed, there is
Creation of Local or Chapter PDMP-01 by the BLR on 6 July 1999.[7] no need for SMPPEU to show a membership of 20% of the employees of the
Thereafter, respondent filed with the Med-Arbiter of the DOLE Regional bargaining unit in order to be recognized as a legitimate labor union.
Officer in the National Capital Region (DOLE-NCR), three separate petitions
for certification election to represent SMPP, SMCSU, and SMBP.[8] All xxxx
three petitions were dismissed, on the ground that the separate petitions
fragmented a single bargaining unit.[9] In view of the foregoing, the assailed decision and resolution of the BLR are
AFFIRMED, and the petition is DISMISSED.[16]
On 17 August 1999, petitioner filed with the DOLE-NCR a petition seeking
the cancellation of respondent's registration and its dropping from the rolls
of legitimate labor organizations. In its petition, petitioner accused Subsequently, in a Resolution dated 16 January 2006, the Court of Appeals
respondent of committing fraud and falsification, and non-compliance with denied petitioner's Motion for Reconsideration of the aforementioned
registration requirements in obtaining its certificate of registration. It raised Decision.
allegations that respondent violated Articles 239(a), (b) and (c)[10] and
234(c)[11] of the Labor Code. Moreover, petitioner claimed that PDMP is Hence, this Petition for Certiorari under Rule 45 of the Revised Rules of
not a legitimate labor organization, but a trade union center, hence, it cannot Court where petitioner raises the sole issue of:
directly create a local or chapter. The petition was docketed as Case No.
NCR-OD-9908-007-IRD.[12]
WHETHER OR NOT THE HONORABLE COURT OF APPEALS
On 14 July 2000, DOLE-NCR Regional Director Maximo B. Lim issued an COMMITTED REVERSIBLE ERROR IN RULING THAT PRIVATE
Order dismissing the allegations of fraud and misrepresentation, and RESPONDENT IS NOT REQUIRED TO SUBMIT THE NUMBER OF
irregularity in the submission of documents by respondent. Regional Director EMPLOYEES AND NAMES OF ALL ITS MEMBERS COMPRISING AT
Lim further ruled that respondent is allowed to directly create a local or LEAST 20% OF THE EMPLOYEES IN THE BARGAINING UNIT
chapter. However, he found that respondent did not comply with the 20% WHERE IT SEEKS TO OPERATE.
membership requirement and, thus, ordered the cancellation of its certificate
of registration and removal from the rolls of legitimate labor
organizations.[13] Respondent appealed to the BLR. In a Decision dated 19 The present petition questions the legal personality of respondent as a
February 2001, it declared: legitimate labor organization.

Petitioner posits that respondent is required to submit a list of members


As a chartered local union, appellant is not required to submit the number of comprising at least 20% of the employees in the bargaining unit before it may
employees and names of all its members comprising at least 20% of the acquire legitimacy, citing Article 234(c) of the Labor Code which stipulates
employees in the bargaining unit where it seeks to operate. Thus, the that any applicant labor organization, association or group of unions or
revocation of its registration based on non-compliance with the 20% workers shall acquire legal personality and shall be entitled to the rights and
membership requirement does not have any basis in the rules. privileges granted by law to legitimate labor organizations upon issuance of
the certificate of registration based on the following requirements:
Further, although PDMP is considered as a trade union center, it is a holder
of Registration Certificate No. FED-11558-LC issued by the BLR on 14
February 1991, which bestowed upon it the status of a legitimate labor a. Fifty pesos (P50.00) registration fee;
organization with all the rights and privileges to act as representative of its
b. The names of its officers, their addresses, the principal address of the labor
organization, the minutes of the organizational meetings and the list of the The applicable Implementing Rules enunciates a two-fold procedure for the
workers who participated in such meetings; creation of a chapter or a local. The first involves the affiliation of an
independent union with a federation or national union or industry union. The
c. The names of all its members comprising at least twenty percent (20%) of second, finding application in the instant petition, involves the direct creation
all the employees in the bargaining unit where it seeks to operate; of a local or a chapter through the process of chartering.[27]

d. If the applicant union has been in existence for one or more years, copies A duly registered federation or national union may directly create a local or
of its annual financial reports; and chapter by submitting to the DOLE Regional Office or to the BLR two copies
of the following:
e. Four (4) copies of the constitution and by-laws of the applicant union,
minutes of its adoption or ratification and the list of the members who
participated in it.[17] (a) A charter certificate issued by the federation or national union indicating
the creation or establishment of the local/chapter;

Petitioner also insists that the 20% requirement for registration of respondent (b) The names of the local/chapter's officers, their addresses, and the
must be based not on the number of employees of a single division, but in all principal office of the local/chapter; and
three divisions of the company in all the offices and plants of SMC since they
are all part of one bargaining unit. Petitioner refers to Section 1, Article 1 of (c) The local/chapter's constitution and by-laws; Provided, That where the
the Collective Bargaining Agreement (CBA),[18] quoted hereunder: local/chapter's constitution and by-laws is the same as that of the federation
or national union, this fact shall be indicated accordingly.

ARTICLE 1 All the foregoing supporting requirements shall be certified under oath by
SCOPE the Secretary or the Treasurer of the local/chapter and attested to by its
President.[28]

Section 1. Appropriate Bargaining Unit. The appropriate bargaining unit


covered by this Agreement consists of all regular rank and file employees The Implementing Rules stipulate that a local or chapter may be directly
paid on the basis of fixed salary per month and employed by the COMPANY created by a federation or national union. A duly constituted local or chapter
in its Corporate Staff Units (CSU), San Miguel Brewing Products (SMBP) created in accordance with the foregoing shall acquire legal personality from
and San Miguel Packaging Products (SMPP) and in different operations the date of filing of the complete documents with the BLR.[29] The issuance
existing in the City of Manila and suburbs, including Metal Closure and of the certificate of registration by the BLR or the DOLE Regional Office is
Lithography Plant located at Canlubang, Laguna subject to the provisions of not the operative act that vests legal personality upon a local or a chapter
Article XV of this Agreement provided however, that if during the term of under Department Order No. 9. Such legal personality is acquired from the
this Agreement, a plant within the territory covered by this Agreement is filing of the complete documentary requirements enumerated in Section 1,
transferred outside but within a radius of fifty (50) kilometers from the Rizal Rule VI.[30]
Monument, Rizal Park, Metro Manila, the employees in the transferred plant
shall remain in the bargaining unit covered by this Agreement. (Emphasis Petitioner insists that Section 3 of the Implementing Rules, as amended by
supplied.) Department Order No. 9, violated Article 234 of the Labor Code when it
provided for less stringent requirements for the creation of a chapter or local.
This Court disagrees.
Petitioner thus maintains that respondent, in any case, failed to meet this 20%
membership requirement since it based its membership on the number of Article 234 of the Labor Code provides that an independent labor
employees of a single division only, namely, the SMPP. organization acquires legitimacy only upon its registration with the BLR:

There is merit in petitioner's contentions.


Any applicant labor organization, association or group of unions or workers
A legitimate labor organization[19] is defined as "any labor organization shall acquire legal personality and shall be entitled to the rights and privileges
duly registered with the Department of Labor and Employment, and includes granted by law to legitimate labor organizations upon issuance of the
any branch or local thereof."[20] The mandate of the Labor Code is to ensure certificate of registration based on the following requirements:
strict compliance with the requirements on registration because a legitimate
labor organization is entitled to specific rights under the Labor Code,[21] and
are involved in activities directly affecting matters of public interest. (a) Fifty pesos (P50.00) registration fee;
Registration requirements are intended to afford a measure of protection to
unsuspecting employees who may be lured into joining unscrupulous or fly- (b) The names of its officers, their addresses, the principal address of the
by-night unions whose sole purpose is to control union funds or use the labor labor organization, the minutes of the organizational meetings and the list of
organization for illegitimate ends.[22] Legitimate labor organizations have the workers who participated in such meetings;
exclusive rights under the law which cannot be exercised by non-legitimate
unions, one of which is the right to be certified as the exclusive (c) The names of all its members comprising at least twenty percent (20%)
representative[23] of all the employees in an appropriate collective of all the employees in the bargaining unit where it seeks to operate;
bargaining unit for purposes of collective bargaining.[24] The acquisition of
rights by any union or labor organization, particularly the right to file a (d) If the applicant union has been in existence for one or more years, copies
petition for certification election, first and foremost, depends on whether or of its annual financial reports; and
not the labor organization has attained the status of a legitimate labor
organization.[25] (e) Four (4) copies of the constitution and by-laws of the applicant union,
minutes of its adoption or ratification, and the list of the members who
A perusal of the records reveals that respondent is registered with the BLR participated in it. (Italics supplied.)
as a "local" or "chapter" of PDMP and was issued Charter Certificate No.
112 on 15 June 1999. Hence, respondent was directly chartered by PDMP.
It is emphasized that the foregoing pertains to the registration of an
The procedure for registration of a local or chapter of a labor organization is independent labor organization, association or group of unions or workers.
provided in Book V of the Implementing Rules of the Labor Code, as
amended by Department Order No. 9 which took effect on 21 June 1997, and However, the creation of a branch, local or chapter is treated differently. This
again by Department Order No. 40 dated 17 February 2003. The Court, in the landmark case of Progressive Development Corporation v.
Implementing Rules as amended by D.O. No. 9 should govern the resolution Secretary, Department of Labor and Employment,[31] declared that when an
of the petition at bar since respondent's petition for certification election was unregistered union becomes a branch, local or chapter, some of the
filed with the BLR in 1999; and that of petitioner on 17 August 1999.[26] aforementioned requirements for registration are no longer necessary or
compulsory. Whereas an applicant for registration of an independent union PDMP was registered as a trade union center and issued Registration
is mandated to submit, among other things, the number of employees and Certificate No. FED-11558-LC by the BLR on 14 February 1991. Until the
names of all its members comprising at least 20% of the employees in the certificate of registration of PDMP is cancelled, its legal personality as a
bargaining unit where it seeks to operate, as provided under Article 234 of legitimate labor organization subsists. Once a union acquires legitimate
the Labor Code and Section 2 of Rule III, Book V of the Implementing Rules, status as a labor organization, it continues to be recognized as such until its
the same is no longer required of a branch, local or chapter.[32] The intent certificate of registration is cancelled or revoked in an independent action for
of the law in imposing less requirements in the case of a branch or local of a cancellation.[41] It bears to emphasize that what is being directly challenged
registered federation or national union is to encourage the affiliation of a is the personality of respondent as a legitimate labor organization and not that
local union with a federation or national union in order to increase the local of PDMP. This being a collateral attack, this Court is without jurisdiction to
union's bargaining powers respecting terms and conditions of labor.[33] entertain questions indirectly impugning the legitimacy of PDMP.

Subsequently, in Pagpalain Haulers, Inc. v. Trajano[34] where the validity of Corollarily, PDMP is granted all the rights and privileges appurtenant to a
Department Order No. 9 was directly put in issue, this Court was unequivocal legitimate labor organization,[42] and continues to be recognized as such
in finding that there is no inconsistency between the Labor Code and until its certificate of registration is successfully impugned and thereafter
Department Order No. 9. cancelled or revoked in an independent action for cancellation.

As to petitioner's claims that respondent obtained its Certificate of We now proceed to the contention that PDMP cannot directly create a local
Registration through fraud and misrepresentation, this Court finds that the or a chapter, it being a trade union center.
imputations are not impressed with merit. In the instant case, proof to declare
that respondent committed fraud and misrepresentation remains wanting. This Court reverses the finding of the appellate court and BLR on this
This Court had, indeed, on several occasions, pronounced that registration ground, and rules that PDMP cannot directly create a local or chapter.
based on false and fraudulent statements and documents confer no legitimacy
upon a labor organization irregularly recognized, which, at best, holds on to After an exhaustive study of the governing labor law provisions, both
a mere scrap of paper. Under such circumstances, the labor organization, not statutory and regulatory,[43] we find no legal justification to support the
being a legitimate labor organization, acquires no rights.[35] conclusion that a trade union center is allowed to directly create a local or
chapter through chartering. Apropos, we take this occasion to reiterate the
This Court emphasizes, however, that a direct challenge to the legitimacy of first and fundamental duty of this Court, which is to apply the law. The
a labor organization based on fraud and misrepresentation in securing its solemn power and duty of the Court to interpret and apply the law does not
certificate of registration is a serious allegation which deserves careful include the power to correct by reading into the law what is not written
scrutiny. Allegations thereof should be compounded with supporting therein.[44]
circumstances and evidence. The records of the case are devoid of such
evidence. Furthermore, this Court is not a trier of facts, and this doctrine Presidential Decree No. 442, better known as the Labor Code, was enacted
applies with greater force in labor cases. Findings of fact of administrative in 1972. Being a legislation on social justice,[45] the provisions of the Labor
agencies and quasi-judicial bodies, such as the BLR, which have acquired Code and the Implementing Rules have been subject to several amendments,
expertise because their jurisdiction is confined to specific matters, are and they continue to evolve, considering that labor plays a major role as a
generally accorded not only great respect but even finality.[36] socio-economic force. The Labor Code was first amended by Republic Act
No. 6715, and recently, by Republic Act No. 9481. Incidentally, the term
Still, petitioner postulates that respondent was not validly and legitimately trade union center was never mentioned under Presidential Decree No. 442,
created, for PDMP cannot create a local or chapter as it is not a legitimate even as it was amended by Republic Act No. 6715. The term trade union
labor organization, it being a trade union center. center was first adopted in the Implementing Rules, under Department Order
No. 9.
Petitioner's argument creates a predicament as it hinges on the legitimacy of
PDMP as a labor organization. Firstly, this line of reasoning attempts to Culling from its definition as provided by Department Order No. 9, a trade
predicate that a trade union center is not a legitimate labor organization. In union center is any group of registered national unions or federations
the process, the legitimacy of PDMP is being impugned, albeit indirectly. organized for the mutual aid and protection of its members; for assisting such
Secondly, the same contention premises that a trade union center cannot members in collective bargaining; or for participating in the formulation of
directly create a local or chapter through the process of chartering. social and employment policies, standards, and programs, and is duly
registered with the DOLE in accordance with Rule III, Section 2 of the
Anent the foregoing, as has been held in a long line of cases, the legal Implementing Rules.[46] The same rule provides that the application for
personality of a legitimate labor organization, such as PDMP, cannot be registration of an industry or trade union center shall be supported by the
subject to a collateral attack. The law is very clear on this matter. Article 212 following:
(h) of the Labor Code, as amended, defines a legitimate labor
organization[37] as "any labor organization duly registered with the DOLE,
and includes any branch or local thereof."[38] On the other hand, a trade (a) The list of its member organizations and their respective presidents and,
union center is any group of registered national unions or federations in the case of an industry union, the industry where the union seeks to
organized for the mutual aid and protection of its members; for assisting such operate;
members in collective bargaining; or for participating in the formulation of
social and employment policies, standards, and programs, and is duly (b) The resolution of membership of each member organization, approved by
registered with the DOLE in accordance with Rule III, Section 2 of the the Board of Directors of such union;
Implementing Rules.[39]
(c) The name and principal address of the applicant, the names of its officers
The Implementing Rules stipulate that a labor organization shall be deemed and their addresses, the minutes of its organizational meeting/s, and the list
registered and vested with legal personality on the date of issuance of its of member organizations and their representatives who attended such
certificate of registration. Once a certificate of registration is issued to a meeting/s; and
union, its legal personality cannot be subject to collateral attack.[40] It may
be questioned only in an independent petition for cancellation in accordance (d) A copy of its constitution and by-laws and minutes of its ratification by a
with Section 5 of Rule V, Book V of the Implementing Rules. The majority of the presidents of the member organizations, provided that where
aforementioned provision is enunciated in the following: the ratification was done simultaneously with the organizational meeting, it
shall be sufficient that the fact of ratification be included in the minutes of
the organizational meeting.[47]
Sec. 5. Effect of registration. The labor organization or workers' association
shall be deemed registered and vested with legal personality on the date of
issuance of its certificate of registration. Such legal personality cannot Evidently, while a "national union" or "federation" is a labor organization
thereafter be subject to collateral attack, but may be questioned only in an with at least ten locals or chapters or affiliates, each of which must be a duly
independent petition for cancellation in accordance with these Rules. certified or recognized collective bargaining agent;[48] a trade union center,
on the other hand, is composed of a group of registered national unions or
federations.[49]
ART. 234-A. Chartering and Creation of a Local Chapter. - A duly registered
The Implementing Rules, as amended by Department Order No. 9, provide federation or national union may directly create a local chapter by issuing a
that "a duly registered federation or national union" may directly create a charter certificate indicating the establishment of the local chapter. The
local or chapter. The provision reads: chapter shall acquire legal personality only for purposes of filing a petition
for certification election from the date it was issued a charter certificate.

Section 1. Chartering and creation of a local/chapter. - A duly registered The chapter shall be entitled to all other rights and privileges of a legitimate
federation or national union may directly create a local/chapter by submitting labor organization only upon the submission of the following documents in
to the Regional Office or to the Bureau two (2) copies of the following: addition to its charter certificate:

(a) A charter certificate issued by the federation or national union indicating (a) The names of the chapter's officers, their addresses, and the principal
the creation or establishment of the local/chapter; office of the chapter; and

(b) The names of the local/chapter's officers, their addresses, and the (b) The chapter's constitution and by-laws: Provided, That where the
principal office of the local/chapter; and chapter's constitution and by-laws are the same as that of the federation or
the national union, this fact shall be indicated accordingly.
(c) The local/chapter's constitution and by-laws; provided that where the
local/chapter's constitution and by-laws is the same as that of the federation The additional supporting requirements shall be certified under oath by the
or national union, this fact shall be indicated accordingly. secretary or treasurer of the chapter and attested by its president. (Emphasis
ours.)
All the foregoing supporting requirements shall be certified under oath by
the Secretary or the Treasurer of the local/chapter and attested to by its
President.[50] Article 234 now includes the term trade union center, but interestingly, the
provision indicating the procedure for chartering or creating a local or
chapter, namely Article 234-A, still makes no mention of a "trade union
Department Order No. 9 mentions two labor organizations either of which is center."
allowed to directly create a local or chapter through chartering - a duly
registered federation or a national union. Department Order No. 9 defines a Also worth emphasizing is that even in the most recent amendment of the
"chartered local" as a labor organization in the private sector operating at the implementing rules,[54] there was no mention of a trade union center as
enterprise level that acquired legal personality through a charter certificate, being among the labor organizations allowed to charter.
issued by a duly registered federation or national union and reported to the
Regional Office in accordance with Rule III, Section 2-E of these Rules.[51] This Court deems it proper to apply the Latin maxim expressio unius est
exclusio alterius. Under this maxim of statutory interpretation, the expression
Republic Act No. 9481 or "An Act Strengthening the Workers' Constitutional of one thing is the exclusion of another. When certain persons or things are
Right to Self-Organization, Amending for the Purpose Presidential Decree specified in a law, contract, or will, an intention to exclude all others from its
No. 442, As Amended, Otherwise Known as the Labor Code of the operation may be inferred. If a statute specifies one exception to a general
Philippines" lapsed[52] into law on 25 May 2007 and became effective on rule or assumes to specify the effects of a certain provision, other exceptions
14 June 2007.[53] This law further amends the Labor Code provisions on or effects are excluded.[55] Where the terms are expressly limited to certain
Labor Relations. matters, it may not, by interpretation or construction, be extended to other
matters.[56] Such is the case here. If its intent were otherwise, the law could
Pertinent amendments read as follows: have so easily and conveniently included "trade union centers" in identifying
the labor organizations allowed to charter a chapter or local. Anything that is
not included in the enumeration is excluded therefrom, and a meaning that
SECTION 1. Article 234 of Presidential Decree No. 442, as amended, does not appear nor is intended or reflected in the very language of the statute
otherwise known as the Labor Code of the Philippines, is hereby further cannot be placed therein.[57] The rule is restrictive in the sense that it
amended to read as follows: proceeds from the premise that the legislating body would not have made
specific enumerations in a statute if it had the intention not to restrict its
meaning and confine its terms to those expressly mentioned.[58] Expressium
ART. 234. Requirements of Registration. - A federation, national union or facit cessare tacitum.[59] What is expressed puts an end to what is implied.
industry or trade union center or an independent union shall acquire legal Casus omissus pro omisso habendus est. A person, object or thing omitted
personality and shall be entitled to the rights and privileges granted by law to must have been omitted intentionally.
legitimate labor organizations upon issuance of the certificate of registration
based on the following requirements: Therefore, since under the pertinent status and applicable implementing
rules, the power granted to labor organizations to directly create a chapter or
(a) Fifty pesos (P50.00) registration fee; local through chartering is given to a federation or national union, then a trade
union center is without authority to charter directly.
(b) The names of its officers, their addresses, the principal address of the
labor organization, the minutes of the organizational meetings and the list of The ruling of this Court in the instant case is not a departure from the policy
the workers who participated in such meetings; of the law to foster the free and voluntary organization of a strong and united
labor movement,[60] and thus assure the rights of workers to self-
(c) In case the applicant is an independent union, the names of all itsmembers organization.[61] The mandate of the Labor Code in ensuring strict
comprising at least twenty percent (20%) of all the employees in the compliance with the procedural requirements for registration is not without
bargaining unit where it seeks to operate; reason. It has been observed that the formation of a local or chapter becomes
a handy tool for the circumvention of union registration requirements. Absent
(d) If the applicant union has been in existence for one or more years, copies the institution of safeguards, it becomes a convenient device for a small group
of its annual financial reports; and of employees to foist a not-so-desirable federation or union on unsuspecting
co-workers and pare the need for wholehearted voluntariness, which is basic
(e) Four copies of the constitution and by-laws of the applicant union, to free unionism.[62] As a legitimate labor organization is entitled to specific
minutes of its adoption or ratification, and the list of the members who rights under the Labor Code and involved in activities directly affecting
participated in it. public interest, it is necessary that the law afford utmost protection to the
parties affected.[63] However, as this Court has enunciated in Progressive
Development Corporation v. Secretary of Department of Labor and
SECTION 2. A new provision is hereby inserted into the Labor Code as Employment, it is not this Court's function to augment the requirements
Article 234-A to read as follows: prescribed by law. Our only recourse, as previously discussed, is to exact
strict compliance with what the law provides as requisites for local or chapter
formation.[64]
In sum, although PDMP as a trade union center is a legitimate labor
organization, it has no power to directly create a local or chapter. Thus,
SMPPEU-PDMP cannot be created under the more lenient requirements for
chartering, but must have complied with the more stringent rules for creation
and registration of an independent union, including the 20% membership
requirement.

WHEREFORE, the instant Petition is GRANTED. The Decision dated 09


March 2005 of the Court of Appeals in CA-GR SP No. 66200 is REVERSED
and SET ASIDE. The Certificate of Registration of San Miguel Packaging
Products Employees Union-Pambansang Diwa ng Manggagawang Pilipino
is ORDERED CANCELLED, and SMPPEU-PDMP DROPPED from the
rolls of legitimate labor organizations.

Costs against petitioner.

SO ORDERED.
FILIPINO PIPE AND FOUNDRY CORPORATION vs. NATIONAL Further, respondent NLU is hereby directed to pay the attorney's fees
LABOR RELATIONS COMMISSION equivalent to 10% of the actual damages, or the amount of ONE HUNDRED
G.R. No. 115180 | 1999-11-16 THOUSAND PESOS (P100, 000.00).
PURISIMA, J.:
For lack of showing that respondent Lerum acted in his personal capacity, he
At bar is a Petition for Certiorari under Rule 65 of the Revised Rules of Court is hereby ABSOLVED from any liability.
seeking to annul and set aside the Decision1 [Annex "A", Rollo, pp. 36-49.]
of the National Labor Relations Commission,2 [Composed of Pursuant to the Agreement, the complaint against all the other individual
Commissioners: Rogelio I. Rayala (Ponente); Edna Bonto-Perez (Presiding respondents are hereby DISMISSED.
Commissioner; and Domingo Zapanta.] dated September 29, 1993, in NLRC
NCR CA No. 003806-92, which reversed the Decision3 [Annex "S", Rollo, SO ORDERED."9 [Annex "S", Rollo, pp. 183-184.]
pp. 176-184.] of the Labor Arbiter,4 [Benigno C. Villarente, Jr.] dated
August 31, 1992, in NLRC Case No. 4-1309-86, disposing thus: Therefrom, both parties appealed to the NLRC which on September 29, 1993,
rendered the assailed decision. Dissatisfied therewith, the petitioner company
'WHEREFORE, premises considered, the appeal of complainant corporation found its way to this Court via the present petition; theorizing that:
is hereby dismissed for lack of merit; the appeal of Atty. Lerum and NLU is
hereby granted, and the Decision dated August 31, 1992 is hereby annulled I
and set side, and a new judgment is hereby entered declaring the complaint PUBLIC RESPONDENT NATIONAL LABOR RELATIONS
below dismissed for lack of merit insofar as respondent NLU and Atty. COMMISSION ERRED IN LAW, CAPRICIOUSLY AND
Lerum are concerned. WHIMSICALLY DISREGARDED THE EVIDENCE SUBMITTED IN
THE CASE AND GRAVELY ABUSED ITS DISCRETION AMOUNTING
SO ORDERED."5 [Annex "A", Rollo, p. 48.] TO LACK AND/OR EXCESS OF JURISDICTION WHEN IT HELD
THAT PRIVATE RESPONDENTS NATIONAL LABOR UNION (NLU)-
The antecedent facts can be culled as follows: TUCP AND ATTY. EULOGIO LERUM ARE NOT PRIMARILY
RESPONSIBLE AND, THEREFORE, NOT LIABLE FOR DAMAGES
On February 10, 1986, respondent National Labor Union-Trade Union SUFFERED BY PETITIONER ON ACCOUNT OF THE ILLEGAL
Congress of the Philippines (NLU-TUCP), a national federation of labor STRIKE THEY HAD DIRECTLY AIDED, ASSISTED, ABETTED AND
unions, filed with the then Ministry of Labor and Employment, in behalf of PARTICIPATED IN.
its local chapter, the Filipino Pipe Workers Union-National Labor Union
(FPWU-NLU, hereinafter referred to as Union), a notice of strike signed by II
its national president, Atty. Eulogio R. Lerum, against the petitioner, Filipino PUBLIC RESPONDENT NATIONAL LABOR RELATIONS
Pipe and Foundry Corporation, alleging as grounds therefor union busting COMMISSION GRAVELY ABUSED ITS DISCRETION AMOUNTING
and non-implementation of the Collective Bargaining Agreement.6 [Rollo, TO LACK AND/OR EXCESS OF JURISDICTION AND ACTED
p. 64.] CAPRICIOUSLY AND WHIMSICALLY IN TOTAL DISREGARD OF
THE EVIDENCE PRESENTED IN THE CASE WHEN IT HELD THAT
The initial conciliation conference was set on February 24, 1986 but due to PRIVATE RESPONDENTS MERELY ASSISTED THE LOCAL
lack of notice thereof to petitioner company, as well as the failure of FPWU- CHAPTER AND ITS MEMBERS IN STAGING A STRIKE AGAINST
NLU to furnish the latter a copy of the notice of strike, the initial conciliation PETITIONER AND THAT SUCH ASSISTANCE WAS NOT THE CAUSE
conference was re-set to March 3, 1986. NOR WAS IT AN INDESPENSABLE ELEMENT OF THE STRIKE.

In the early morning of March 3, 1986, however, without waiting for the III
outcome of the conciliation conference scheduled on said date, the FPWU- PUBLIC RESPONDENT NATIONAL LABOR RELATIONS
NLU staged the strike in question which lasted until June 13, 1986, when a COMMISSION GRAVELY ERRED IN LAW AND GRAVELY ABUSED
return to work agreement was reached by the union and petitioner company.7 ITS DISCRETION AMOUNTING TO LACK AND/OR EXCESS OF
[Rollo, p. 178.] JURISDICTION WHEN IT CONCLUDED THAT PETITIONER LOST
ITS CAUSE OF ACTION AGAINST PRIVATE RESPONDENTS AFTER
On April 8, 1986, petitioner company interposed before the Arbitration THE LOCAL UNION HIRED A NEW COUNSEL AND PETITIONER
Branch of the then Ministry of Labor and Employment, a petition to declare MOVED FOR PARTIAL DISMISSAL OF ITS COMPLAINT AGAINST
the strike illegal with prayer for damages against FPWU-NLU, NLU-TUCP THE STRIKING WORKERS INASMUCH AS PRIVATE
and its national president, Atty. Eulogio Lerum. RESPONDENTS ARE MERE THIRD PARTIES.10 [Petition, Rollo, p. 11.]

On December 23, 1988, petitioner company moved for the partial dismissal Rule XXII, Book V, of the Rules Implementing the Labor Code, provides:
of the Complaint against forty-three (43) officers and members of FPWU-
NLU, but maintained the action against the NLU-TUCP and Atty. Eulogio "Section 1. Grounds for strike and lockout. - A strike or lockout may be
Lerum.8 [Rollo, p. 39.] declared in cases of bargaining deadlocks and unfair labor practices.
Violations of collective bargaining agreements, except flagrant and/or
On August 31, 1992, the Labor Arbiter came out with a decision for petitioner malicious refusal to comply with its economic provisions, shall not be
company, ruling as follows: considered unfair labor practice and shall not be strikeable. No strike or
lockout may be declared on grounds involving inter-union and intra-union
"WHEREFORE, judgment is hereby rendered declaring that the strike staged disputes or on issues brought to voluntary or compulsory arbitration."
by respondents from March 3, 1986 to June 13, 1986 was ILLEGAL.
Accordingly and in conformity with the Return-to-Work Agreement, xxx.....xxx.....xxx
respondent National Labor Union-TUCP is hereby directed to pay the
complainant company the following: "Section 3. - Notice of strike or lockout.- In cases of bargaining deadlocks, a
notice of strike or lockout shall be filed with the regional branch of the Board
a) Actual damages in the form of loss of revenue during the duration of the at least thirty (30) days before the intended date thereof, a copy of said notice
strike which lasted for 100 days or in the amount of ONE MILLION PESOS having been served on the other party concerned. x x x"
(P1,000,000.00);
xxx.....xxx.....xxx
b) Damages to the good business standing and commercial credit of the
company in the amount of THREE HUNDRED FIFTY THOUSAND "Section 6. Conciliation. - Upon receipt of the notice, the regional branch of
PESOS (P350,000.00); and the Board shall exert all efforts at mediation and conciliation to enable the
parties to settle the dispute amicably. The regional branch of the Board may,
c) Exemplary damages to deter others similarly inclined from committing upon consultation, recommend to the parties to submit the dispute to
similar acts and to serve as an example for the public good, in the amount of voluntary arbitration.
TWO HUNDRED FIFTY THOUSAND PESOS (P250,000.00).
During the proceedings, the parties shall not do any act which may disrupt or President .....-.....Virgilio Bernal
impede the early settlement of the dispute. They are obliged as part of the Vice-Pres......-.....Ramon Alborte
duty to bargain collectively in good faith, to participate fully and promptly in Secretary.....-.....Ernesto Ballesteros
the conciliation meetings called by the regional branch of the board. The Treasure.....-.....Arsenio Agustin
regional branch of the Board shall have the power to issue subpoenas Auditor.....-.....Genaro Gabule
requiring the attendance of the parties to the meetings. xxx"
Board Members:
Applying the aforecited provision of law in point to the case under 1. Eduardo Cenina..........4. Felimon Simborio
consideration, the Court is of the finding and conclusion that the strike staged 2. Dante Canete.............5. Joseph Olazo
by FPWU-NLU was illegal for want of any legal basis. Contrary to the 3. Reynaldo Adelante .....6. Virgilio Elnar
grounds advanced by the union in the notice of strike, it turned out during the
March 3, 1986 conciliation conference that the purpose of the strike was to Shop Stewards:
pressure the petitioner company to: 1. Pablito Fajardo
2. Ruperto Manlangit
1) include in the salary of the strikers the P3.00 wage increase11 [Based on 3. Ruben Bongaos
Wage Order Nos. 2 and 3 (See Filipino Pipe Workers Union (NLU) vs.
Batario, Jr., 163, SCRA 789, p. 797)] effective March 1, 1986. We have given them full authority to deal with you on all matters covered by
our authority as sole collective bargaining representative of your rank and
2) compute their backwages covering the period from December 1, 1980 to file workers.
February 28, 1986, including vacation leave and sick leave.
Very truly yours,
A thorough sifting of the pertinent records discloses that the alleged union
busting was not substantiated and the supposed non-implementation of the (Sgd)
collective bargaining agreement was groundless because the demands of
FPWU-NLU, at the time the notice of strike was filed and at the time the EULOGIO R. LERUM
union actually struck, were the subject of a pending application for a writ of National President"14
execution filed by the union in Case No. AB-7933-80 (NCR-CA-8-674-80), [Rollo, p. 246.]
which application was granted on April 4, 1986 by the Labor Arbiter.12
[Filipino Pipe Workers Union (NLU) vs. Batario, Jr., 163, SCRA 789, p. In Progressive Development Corporation vs. Secretary, Department of Labor
792.] Verily, the strike staged by FPWU-NLU was baseless since it was still and Employment,15 [G. R. No. 96425, February 4, 1992, 205 SCRA 802,]
pre-mature then for the union to insist on the implementation of the adverted the Court explained the nature of the relationship between a mother
provision of the collective bargaining agreement, which was the subject of a union/federation and a local union, thus:
pending writ of execution.
"At this juncture, it is important to clarify the relationship between the mother
Then too, the failure of the union to serve petitioner company a copy of the union and the local union. In the case of Liberty Cotton Mills Workers Union
notice of strike is a clear violation of Section 3 of the aforestated Rules. The v. Liberty Cotton Mills, Inc., 66 SCRA 512 [1975], the Court held that the
constitutional precepts of due process mandate that the other party be notified mother union, acting for and in behalf of its affiliate, had the status of an
of the adverse action of the opposing party. So also, the same Section agent while the local union remained the basic unit of the association, free to
provides for a mandatory thirty (30) day cooling-off period which the union serve the common interest of all its members subject only to the restraints
ignored when it struck on March 3, 1986, before the 30th day from the time imposed by the constitution and by-laws of the association. x x x"16 [Id., pp.
the notice of strike was filed on February 10, 1986. 814-815.]

What is more, the same strike blatantly disregarded the prohibition on the The same is true even if the local union is not a legitimate labor organization.
doing of any act which may impede or disrupt the conciliation proceedings, Conformably, in the abovecited case the Court ruled that the mother
when the union staged the strike in the early morning of March 3, 1986, the federation was a mere agent and the local chapter/union was the principal,
very same day the conciliation conference was scheduled by the former notwithstanding the failure of the local union to comply with the procedural
Ministry of Labor. requirements that would make it a legitimate labor organization.

In light of the foregoing, it is beyond cavil that subject strike staged by the Evidently, in the case under scrutiny, whether or not FPWU, the local
union was illegal. chapter, complied with the procedural requirements that would make it a
legitimate labor organization is immaterial. It would not affect its status as
Anent the responsibility for the damages allegedly sustained by petitioner the principal and basic unit of the association. The requirement laid down in
company on account of the illegal strike, the latter theorized that the liability the Progressive Development case, that the local union must be a legitimate
therefor should be borne by NLU-TUCP and its national president, Atty. labor organization, pertains to the conditions before a union may file a
Eulogio Lerum, for having directly participated in aiding and abetting the petition for certification election and to be certified as sole and exclusive
illegal strike. It is argued that FPWU-NLU is a mere agent of respondent bargaining agent. In the present case, there is no dispute that FPWU-NLU is
NLU-TUCP, because FPWU-NLU, which was formed by respondent NLU- the sole and exclusive bargaining representative of the rank and file
TUCP is not registered as a local unit or chapter but directly affiliated with employees of petitioner company. The union's status as a legitimate labor
the latter and therefore, could not have acted on its own. Otherwise stated, organization is therefore of no moment in the resolution of the controversy
petitioner is of the view that FPWU-NLU, a local union, cannot act as the here. As the local union, it is considered as the principal; the entity which
principal of respondent NLU-TUCP, a mother federation, because it is not a staged the illegal strike and the one responsible for the resulting damages
legitimate labor organization.13 [Petition, Rollo, pp. 19-20.] In support of allegedly sustained by petitioner company.
this stance, petitioner cited the following letter of Atty. Lerum to the
company, to wit: Furthermore, the petitioner company is now estopped from reneging on the
recognition it extended to the FPUW-NLU as the bargaining representative
"NATIONAL LABOR UNION of its rank and file workers, by belatedly attacking its status which petitioner
company had voluntarily recognized. It should be noted that even as early as
An Affiliate of the Trade Union of the Philippines 1981, when the collective bargaining agreement sought to be implemented
3199 Ramon Magsaysay Blvd., Manila, Philippines by the union was entered into, the latter was already the bargaining
Tel. 61-42-65 representative of the employees concerned. It is not, therefore, true that it was
March 29, 1983 respondent NLU-TUCP which formed FPWU. At most, the entry into the
picture of the private respondent on March 23, 1983, merely affirmed the
Dear Sirs: status of FPWU as the recognized bargaining representative of the rank and
file employees of petitioner company.
Please be informed that we have formed a local union in your company and
the officers thereof are the following: Evidently, direct and primary responsibility for the damages allegedly caused
by the illegal strike sued upon fall on the local union FPWU, being the
principal, and not on respondent NLU-TUCP, a mere agent of FPWU-NLU
which assisted the latter in filing the notice of strike. Being just an agent, the
notice of strike filed by Atty. Eulogio Lerum, the national president of NLU-
TUCP, is deemed to have been filed by its principal, the FPWU-NLU.
Having thus dismissed the claim for damages against the principal, FPWU-
NLU, the action for damages against its agent, respondent NLU-TUCP, and
Atty. Lerum, has no more leg to stand on and should also be dismissed.

Premises studiedly considered, the Court is of the ineluctable conclusion, and


so holds, that the National Labor Relations Commission did not act with
grave abuse of discretion in reversing the Decision of the Labor Arbiter in
NLRC CASE No. 4-1309-86.

WHEREFORE, for lack of merit, the Petition is DISMISSED, and the


Decision of the National Labor Relations Commission in NLRC NCR CA
No. 003806-92 AFFIRMED. No pronouncement as to costs.

SO ORDERED.
SUGBUANON RURAL BANK, INC. vs. LAGUESMA
G.R. No. 116194 | 2000-02-02 On December 22, 1993, petitioner proceeded to file a petition with the DOLE
QUISUMBING, J.: Regional Office seeking the cancellation of the respondent union's
registration. It averred that the APSOTEU-TUCP members were actually
In this special civil action for certiorari and prohibition, petitioner seeks the managerial employees who were prohibited by law from joining or
annulment of the April 27, 1994 Resolution of the Department of Labor and organizing unions.
Employment, affirming the order of the Med-Arbiter, dated December 9,
1993, which denied petitioner's motion to dismiss respondent union's petition On April 22, 1994, respondent DOLE Undersecretary denied SRBI's appeal
for certification election. Esmmis for lack of merit. He ruled that APSOTEU- TUCP was a legitimate labor
organization. As such, it was fully entitled to all the rights and privileges
Petitioner Sugbuanon Rural Bank, Inc., (SRBI, for brevity) is a duly- granted by law to a legitimate labor organization, including the right to file a
registered banking institution with principal office in Cebu City and a branch petition for certification election. He also held that until and unless a final
in Mandaue City. Private respondent SRBI-Association of Professional, order is issued canceling APSOTEU- TUCP's registration certificate, it had
Supervisory, Office, and Technical Employees Union (APSOTEU) is a the legal right to represent its members for collective bargaining purposes.
legitimate labor organization affiliated with the Trade Unions Congress of Furthermore, the question of whether the APSOTEU- TUCP members
the Philippines (TUCP). should be considered as managerial or confidential employees should not be
addressed in the proceedings involving a petition for certification election
On October 8, 1993, the DOLE Regional Office in Cebu City granted but best threshed out in other appropriate proceedings.
Certificate of Registration No. R0700-9310-UR-0064 to APSOTEU- TUCP,
hereafter referred to as the union. On May 25, 1994, SRBI moved for reconsideration of the Undersecretary's
decision which was denied on July 7, 1994. The Med- Arbiter scheduled the
On October 26, 1993, the union filed a petition for certification election of holding of certification elections on August 12, 1994.
the supervisory employees of SRBI. It alleged, among others, that: (1)
APSOTEU-TUCP was a labor organization duly-registered with the Labor Hence the instant petition grounded on the following assignments of error:
Department; (2) SRBI employed 5 or more supervisory employees; (3) a
majority of these employees supported the petition; (4) there was no existing I
collective bargaining agreement (CBA) between any union and SRBI; and
(5) no certification election had been held in SRBI during the past 12 months RESPONDENT UNDERSECRETARY LAGUESMA ACTED WITH
prior to the petition. GRAVE ABUSE OF DISCRETION AND PALPABLY ERRED:

On October 28, 1993, the Med-Arbiter gave due course to the petition. The A. IN HOLDING THAT ART. 257 OF THE LABOR CODE REQUIRES
pre-certification election conference between SRBI and APSOTEU- TUCP THE MED-ARBITER TO CONDUCT A CERTIFICATION ELECTION IN
was set for November 15, 1993. ANY UNORGANIZED ESTABLISHMENT EVEN WHEN THE
PETITIONING UNION DOES NOT POSSESS THE QUALIFICATION
On November 12, 1993, SRBI filed a motion to dismiss the union's petition. FOR AN APPROPRIATE BARGAINING AGENT; AND
It sought to prevent the holding of a certification election on two grounds:
First, that the members of APSOTEU-TUCP were in fact managerial or B. IN REFUSING TO ASSUME JURISDICTION OVER THE
confidential employees. Thus, following the doctrine in Philips Industrial PETITIONER'S APPEAL AND TO DISMISS THE RESPONDENT
Development Corporation v. National Labor Relations Commission,1 [210 UNION'S PETITION FOR CERTIFICATION ELECTION.
SCRA 339 (1992)] they were disqualified from forming, joining, or assisting
any labor organization. Petitioner attached the job descriptions of the II
employees concerned to its motion. Second, the Association of Labor
Unions-Trade Unions Congress of the Philippines or ALU-TUCP was RESPONDENT UNDERSECRETARY LAGUESMA ACTED WITH
representing the union. Since ALU- TUCP also sought to represent the rank- GRAVE ABUSE OF DISCRETION AND PALPABLY ERRED IN
and-file employees of SRBI, there was a violation of the principle of DENYING THE PETITIONER'S APPEAL DESPITE THE FACT THAT:
separation of unions enunciated in Atlas Lithographic Services, Inc. v.
Laguesma.2 [205 SCRA 12 (1992)] A. THE ALLEGED MEMBERS OF RESPONDENT UNION ARE
MANAGERIAL EMPLOYEES WHO ARE LEGALLY DISQUALIFIED
The union filed its opposition to the motion to dismiss on December 1, 1993. FROM JOINING ANY LABOR ORGANIZATION.
It argued that its members were not managerial employees but merely
supervisory employees. The members attached their affidavits describing the B. AT THE VERY LEAST, THE ALLEGED MEMBERS OF
nature of their respective duties. The union pointed out that Article 245 of RESPONDENT UNION ARE OCCUPYING HIGHLY CONFIDENTIAL
the Labor Code expressly allowed supervisory employees to form, join, or POSITIONS IN PETITIONER AND, THUS, THE LEGAL
assist their own unions. DISQUALIFICATION OF MANAGERIAL EMPLOYEES EQUALLY
APPLY TO THEM. Msesm
On December 9, 1993, the Med-Arbiter denied petitioner's motion to dismiss.
He scheduled the inclusion-exclusion proceedings in preparation for the III
certification election on December 16, 1993.
IN ANY EVENT, THE CONCLUSIONS REACHED IN THE SUBJECT
SRBI appealed the Med-Arbiter's decision to the Secretary of Labor and RESOLUTIONS ARE CONTRARY TO LAW AND ARE
Employment. The appeal was denied for lack of merit. The certification DIAMETRICALLY OPPOSED TO RESPONDENT UNION'S
election was ordered. RECORDED ADMISSIONS AND REPRESENTATIONS.

On June 16, 1994, the Med-Arbiter scheduled the holding of the certification Considering petitioner's assigned errors, we find two core issues for
election for June 29, 1994. His order identified the following SRBI personnel immediate resolution:
as the voting supervisory employees in the election: the Cashier of the Main
office, the Cashier of the Mandaue Branch, the Accountant of the Mandaue (1) Whether or not the members of the respondent union are managerial
Branch, and the Acting Chief of the Loans Department. employees and/or highly-placed confidential employees, hence prohibited by
law from joining labor organizations and engaging in union activities?
On June 17, 1994, SRBI filed with the Med-Arbiter an urgent motion to
suspend proceedings The Med-Arbiter denied the same on June 21, 1994. (2) Whether or not the Med-Arbiter may validly order the holding of a
SRBI then filed a motion for reconsideration. Two days later, the Med- certification election upon the filing of a petition for certification election by
Arbiter cancelled the certification election scheduled for June 29, 1994 in a registered union, despite the petitioner's appeal pending before the DOLE
order to address the motion for reconsideration. Secretary against the issuance of the union's registration?

The Med-Arbiter later denied petitioner's motion for reconsideration. SRBI The other issues based on the assigned errors could be resolved easily after
appealed the order of denial to the DOLE Secretary on December 16, 1993. the core issues are settled.
Respecting the first issue, Article 212 (m) of the Labor Code defines the the doctrine of necessary implication, the disqualification of managerial
terms "managerial employee" and "supervisory employees" as follows: employees equally applies to confidential employees.12 [Golden Farms, Inc.
v. Ferrer-Calleja, 175 SCRA 471, 477 (1989); Bulletin Publishing Co. Inc.
"Art. 212. Definitions- vs. Hon. Augusto Sanchez, etc., 144 SCRA 628, 634 (1986)] The
confidential-employee rule justifies exclusion of confidential employees
xxx because in the normal course of their duties they become aware of
management policies relating to labor relations.13 [San Miguel Corp.
(m) 'Managerial employee' is one who is vested with powers or prerogatives Supervisors and Exempt Employees Union v. Laguesma, supra, 375.] It must
to lay down and execute management policies and/or hire, transfer, suspend, be stressed, however, that when the employee does not have access to
lay-off, recall, discharge, assign or discipline employees. Supervisory confidential labor relations information, there is no legal prohibition against
employees are those who, in the interest of the employer, effectively confidential employees from forming, assisting, or joining a union.14 [Id. at
recommend such managerial actions if the exercise of such authority is not 376; National Association of Trade Unions-Republic Planters Bank
merely routinary or clerical in nature but requires the use of independent Supervisor Chapter v. Torres, 239 SCRA 546, 560 (1994)]
judgment. All employees not falling within any of the above definitions are
considered rank-and-file employees for purposes of this Book (Italic Petitioner contends that it has only 5 officers running its day-to-day affairs.
supplied)." They assist in confidential capacities and have complete access to the bank's
confidential data. They form the core of the bank's management team.
Petitioner submitted detailed job descriptions to support its contention that Petitioner explains that:
the union members are managerial employees and/or confidential employees
proscribed from engaging in labor activities.3 [Records, pp. 80-82.] "...Specifically: (1) the Head or the Loans Department initially approves the
Petitioner vehemently argues that the functions and responsibilities of the loan applications before they are passed on to the Board for confirmation. As
employees involved constitute the "very core of the bank's business, lending such, no loan application is even considered by the Board and approved by
of money to clients and borrowers, evaluating their capacity to pay, petitioner without his stamp of approval based upon his interview of the
approving the loan and its amount, scheduling the terms of repayment, and applicant and determination of his (applicant's) credit standing and financial
endorsing delinquent accounts to counsel for collection."4 [Id. at 16.] Hence, capacity. The same holds true with respect to renewals or restructuring of
they must be deemed managerial employees. Petitioner cites Tabacalera loan accounts. He himself determines what account should be collected,
Insurance Co. v. National Labor Relations Commission,5 [152 SCRA 667 whether extrajudicially or judicially, and settles the problem or complaints
(1987)] and Panday v. National Labor Relations Commission,6 [209 SCRA of borrowers regarding their accounts;
122 (1992)] to sustain its submission. In Tabacalera, we sustained the
classification of a credit and collection supervisor by management as a "(2) the Cashier is one of the approving officers and authorized signatories
managerial/supervisory personnel. But in that case, the credit and collection of petitioner. He approves the opening of accounts, withdrawals and
supervisor "had the power to recommend the hiring and appointment of his encashment, and acceptance of check deposits, He deals with other banks
subordinates, as well as the power to recommend any promotion and/or and, in the absence of the regular Manager, manages the entire office or
increase."7 [152 SCRA 667, 674 (1987)] For this reason he was deemed to branch and approves disbursements of funds for expenses; and
be a managerial employee. In the present case, however, petitioner failed to
show that the employees in question were vested with similar powers. At best "(3) the Accountant, who heads the Accounting Department, is also one of
they only had recommendatory powers subject to evaluation, review, and the authorized signatories of petitioner and, in the absence of the Manager or
final decision by the bank's management. The job description forms Cashier, acts as substitute approving officer and assumes the management of
submitted by petitioner clearly show that the union members in question may the entire office. She handles the financial reports and reviews the
not transfer, suspend, lay-off, recall, discharge, assign, or discipline debit/credit tickets submitted by the other departments."15 [Rollo, pp. 13-
employees. Moreover, the forms also do not show that the Cashiers, 14.]
Accountants, and Acting Chiefs of the loans Department formulate and
execute management policies which are normally expected of management Petitioner's explanation, however, does not state who among the employees
officers. has access to information specifically relating to its labor relations policies.
Even Cashier Patricia Maluya, who serves as the secretary of the bank's
Petitioner's reliance on Panday is equally misplaced. There, we held that a Board of Directors may not be so classified. True, the board of directors is
branch accountant is a managerial employee because the said employee had responsible for corporate policies, the exercise of corporate powers, and the
managerial powers, similar to the supervisor in Tabacalera. Their powers general management of the business and affairs of the corporation. As
included recommending the hiring and appointment of his subordinates, as secretary of the bank's governing body, Patricia Maluya serves the bank's
the power to recommend any promotion and/or increase.8 [Supra; 209 SCRA management, but could not be deemed to have access to confidential
122, 126 (1992)] information specifically relating to SRBI's labor relations policies, absent a
clear showing on this matter. Thus, while petitioner's explanation confirms
Here, we find that that the Cashiers, Accountant, and Acting Chief of the the regular duties of the concerned employees, it shows nothing about any
Loans Department of the petitioner did not possess managerial powers and duties specifically connected to labor relations.
duties. We are, therefore, constrained to conclude that they are not
managerial employees. As to the second issue. One of the rights of a legitimate labor organization
under Article 242(b) of the Labor Code is the right to be certified as the
Now may the said bank personnel be deemed confidential employees? exclusive representative of all employees in an appropriate bargaining unit
Confidential employees are those who (1) assist or act in a confidential for purposes of collective bargaining. Having complied with the
capacity, in regard (2) to persons who formulate, determine, and effectuate requirements of Art. 234, it is our view that respondent union is a legitimate
management policies [specifically in the field of labor relations].9 [San labor union. Article 257 of the Labor Code mandates that a certification
Miguel Corp. Supervisors and Exempt Employees Union v. Laguesma, 277 election shall automatically be conducted by the Med-Arbiter upon the filing
SCRA 370, 374 (1997) citing Westinghouse Electric Corp. v. NLRB (CAS) of a petition by a legitimate labor organization.16 [Emphasis supplied.
398 F2d. 689; Ladish Co., 178 NLRB 90 (1969), B.F. Goodrich Co., 115 Furusawa Rubber Philippines Inc. v. Secretary of Labor and Employment,
NLRB 722.] The two criteria are cumulative, and both must be met if an 282 SCRA 635, 641 (1997); California Manufacturing Corp. v. Laguesma,
employee is to be considered a confidential employee-that is, the confidential 209 SCRA 606, 610-611 (1992)] Nothing is said therein that prohibits such
relationship must exist between the employee and his superior officer; and automatic conduct of the certification election if the management appeals on
that officer must handle the prescribed responsibilities relating to labor the issue of the validity of the union's registration. On this score, petitioner's
relations.10 [Supra.] appeal was correctly dismissed.

Article 245 of the Labor Code11 [ART 245. Ineligibility of managerial Petitioner argues that giving due course to respondent union's petition for
employees to join any labor organization; right of supervisory employees. - certification election would violate the separation of unions doctrine.17
Managerial employees are not eligible to join, assist or form any labor [Atlas Lithographic Services, Inc. v. Bienvenido Laguesma, et al, 205 SCRA
organization. Supervisory employees shall not be eligible for membership in 12 (1992)] Note that the petition was filed by APSOTEU- TUCP, a legitimate
a labor organization of the rank-and-file employees but may join, assist or labor organization. It was not, filed by ALU. Nor was it filed by TUCP,
form separate labor organizations of their own.] does not directly, prohibit which is a national labor federation of with which respondent union is
confidential employees from engaging in union activities. However, under affiliated. Petitioner says that respondent union is a mere alter ego of ALU.
The records show nothing to this effect. What the records instead reveal is
that respondent union was initially assisted by ALU during its preliminary
stages of organization. A local union maintains its separate personality
despite affiliation with a larger national federation.18 [Pambansang
Kapatiran Ng Mga Anak Pawis sa Formey Plastic National Workers
Brotherhood v. Secretary of Labor, 253 SCRA 96, 103 (1995)] Petitioner
alleges that ALU seeks to represent both respondent union and the rank-and-
file union. Again, we find nothing in the records to support this bare
assertion.

The law frowns on a union where the membership is composed of both


supervisors and rank-and-file employees, for fear that conflicts of interest
may arise in the areas of discipline, collective bargaining, and strikes.19
[Philippine Phosphate Corporation v. Torres, 231 SCRA 335, 342 (1994)]
However, in the present case, none of the members of the respondent union
came from the rank-and-file employees of the bank.

Taking into account the circumstances in this case, it is our view that
respondent Undersecretary committed no reversible error nor grave abuse of
discretion when he found the order of the Med-Arbiter scheduling a
certification election in order. The list of employees eligible to vote in said
certification election was also found in order, for none was specifically
disqualified from membership.

WHEREFORE, the instant petition is hereby DISMISSED. No


pronouncement as to costs.

SO ORDERED.
HOLY CHILD CATHOLIC SCHOOL vs. HON. PATRICIA STO. On August 10, 2002, Med-Arbiter Agatha Ann L. Daquigan denied the
TOMAS petition for certification election on the ground that the unit which private
G.R. No. 179146 | 2013-07-23 respondent sought to represent is inappropriate. She resolved:
PERALTA, J.:
A certification election proceeding directly involves two (2) issues namely:
Assailed in this petition for review on certiorari under Rule 45 of the Rules (a) the proper composition and constituency of the bargaining unit; and (b)
of Civil Procedure are the April 18, 2007 Decision 1 and July 31, 2007 the validity of majority representation claims. It is therefore incumbent upon
Resolution2 of the Court of Appeals in CA-G.R. SP No. 76175, which the Med-Arbiter to rule on the appropriateness of the bargaining unit once its
affirmed the December 27, 2002 Decision3 and February 13, 2003 composition and constituency is questioned.
Resolution4 of the Secretary of the Department of Labor and Employment
(SOLE) that set aside the August 10, 2002 Decision5 of the Med-Arbiter Section 1 (q), Rule I, Book V of the Omnibus Rules defines a “bargaining
denying private respondent’s petition for certification election. unit” as a group of employees sharing mutual interests within a given
employer unit comprised of all or less than all of the entire body of employees
The factual antecedents are as follows: in the employer unit or any specific occupational or geographical grouping
within such employer unit. This definition has provided the “community or
On May 31, 2002, a petition for certification election was filed by private mutuality of interest” test as the standard in determining the constituency of
respondent Pinag-Isang Tinig at Lakas ng Anakpawis – Holy Child Catholic a collective bargaining unit. This is so because the basic test of an asserted
School Teachers and Employees Labor Union (HCCS-TELUPIGLAS), bargaining unit’s acceptability is whether or not it is fundamentally the
alleging that: PIGLAS is a legitimate labor organization duly registered with combination which will best assure to all employees the exercise of their
the Department of Labor and Employment (DOLE) representing HCCS- collective bargaining rights. The application of this test may either result in
TELU-PIGLAS; HCCS is a private educational institution duly registered the formation of an employer unit or in the fragmentation of an employer
and operating under Philippine laws; there are approximately one hundred unit.
twenty (120) teachers and employees comprising the proposed appropriate
bargaining unit; and HCCS is unorganized, there is no collective bargaining In the case at bar, the employees of [petitioner], may, as already suggested,
agreement or a duly certified bargaining agent or a labor organization quite easily be categorized into (2) general classes[:] one, the teaching staff;
certified as the sole and exclusive bargaining agent of the proposed and two, the non-teaching-staff. Not much reflection is needed to perceive
bargaining unit within one year prior to the filing of the petition.6 Among the that the community or mutuality of interest is wanting between the teaching
documents attached to the petition were the certificate of affiliation with and the non-teaching staff. It would seem obvious that the teaching staff
Pinag-Isang Tinig at Lakas ng Anakpawis Kristiyanong Alyansa ng would find very little in common with the nonteaching staff as regards
Makabayang Obrero (PIGLAS-KAMAO) issued by the Bureau of Labor responsibilities and function, working conditions, compensation rates, social
Relations (BLR), charter certificate issued by PIGLASKAMAO, and life and interests, skills and intellectual pursuits, etc. These are plain and
certificate of registration of HCCS-TELU as a legitimate labor organization patent realities which cannot be ignored. These dictate the separation of these
issued by the DOLE.7 two categories of employees for purposes of collective bargaining.
(University of the Philippines vs. Ferrer-Calleja, 211 SCRA 451)19
In its Comment8 and Position Paper,9 petitioner HCCS consistently noted
that it is a parochial school with a total of 156 employees as of June 28, 2002, Private respondent appealed before the SOLE, who, on December 27, 2002,
broken down as follows: ninety-eight (98) teaching personnel, twenty-five ruled against the dismissal of the petition and directed the conduct of two
(25) non-teaching academic employees, and thirty-three (33) non-teaching separate certification elections for the teaching and the non-teaching
non-academic workers. It averred that of the employees who signed to personnel, thus:
support the petition, fourteen (14) already resigned and six (6) signed twice.
Petitioner raised that members of private respondent do not belong to the We agree with the Med-Arbiter that there are differences in the nature of
same class; it is not only a mixture of managerial, supervisory, and rank-and- work, hours and conditions of work and salary determination between the
file employees – as three (3) are vice-principals, one (1) is a department teaching and non-teaching personnel of [petitioner]. These differences were
head/supervisor, and eleven (11) are coordinators – but also a combination pointed out by [petitioner] in its position paper. We do not, however, agree
of teaching and non-teaching personnel – as twenty-seven (27) are non- with the Med-Arbiter that these differences are substantial enough to warrant
teaching personnel. It insisted that, for not being in accord with Article 24510 the dismissal of the petition. First, as pointed out by [private respondent],
of the Labor Code, private respondent is an illegitimate labor organization “inappropriateness of the bargaining unit sought to be represented is not a
lacking in personality to file a petition for certification election, as held in ground for the dismissal of the petition[.”] In fact, in the cited case of
Toyota Motor Philippines Corporation v. Toyota Motor Philippines University of the Philippines v. Ferrer-Calleja, supra, the Supreme Court did
Corporation Labor Union;11 and an inappropriate bargaining unit for want not order the dismissal of the petition but ordered the conduct of a
of community or mutuality of interest, as ruled in Dunlop Slazenger (Phils.), certification election, limiting the same among the non-academic personnel
Inc. v. Secretary of Labor and Employment12 and De La Salle University of the University of the Philippines.
Medical Center and College of Medicine v. Laguesma.13
It will be recalled that in the U.P. case, there were two contending unions,
Private respondent, however, countered that petitioner failed to substantiate the Organization of Non-Academic Personnel of U.P. (ONAPUP) and All
its claim that some of the employees included in the petition for certification U.P. Workers Union composed of both academic and nonacademic personnel
election holds managerial and supervisory positions.14 Assuming it to be of U.P. ONAPUP sought the conduct of certification election among the
true, it argued that Section 11 (II),15 Rule XI of DOLE Department Order rank-and-file non-academic personnel only while the all U.P. Workers Union
(D.O.) No. 9, Series of 1997, provided for specific instances in which a sought the conduct of certification election among all of U.P.’s rank-and-file
petition filed by a legitimate organization shall be dismissed by the Med- employees covering academic and nonacademic personnel. While the
Arbiter and that “mixture of employees” is not one of those enumerated. Supreme Court ordered a separate bargaining unit for the U.P. academic
Private respondent pointed out that questions pertaining to qualifications of personnel, the Court, however, did not order them to organize a separate labor
employees may be threshed out in the inclusionexclusion proceedings prior organization among themselves. The All U.P. Workers Union was not
to the conduct of the certification election, pursuant to Section 2,16 Rule XII directed to divest itself of its academic personnel members and in fact, we
of D.O. No. 9. Lastly, similar to the ruling in In Re: Globe Machine and take administrative notice that the All U.P. Workers Union continue to exist
Stamping Company,17 it contended that the will of petitioner’s employees with a combined membership of U.P. academic and non-academic personnel
should be respected as they had manifested their desire to be represented by although separate bargaining agreements is sought for the two bargaining
only one bargaining unit. To back up the formation of a single employer unit, units. Corollary, [private respondent] can continue to exist as a legitimate
private respondent asserted that even if the teachers may receive additional labor organization with the combined teaching and non-teaching personnel
pay for an advisory class and for holding additional loads, petitioner’s in its membership and representing both classes of employees in separate
academic and non-academic personnel have similar working conditions. It bargaining negotiations and agreements.
cited Laguna College v. Court of Industrial Relations,18 as well as the case
of a union in West Negros College in Bacolod City, which allegedly WHEREFORE, the Decision of the Med-Arbiter dated 10 August 2002 is
represented both academic and non-academic employees. hereby REVERSED and SET ASIDE. In lieu thereof, a new order is hereby
issued directing the conduct of two certification elections, one among the
non-teaching personnel of Holy Child Catholic School, and the other, among
the teaching personnel of the same school, subject to the usual pre-election
conferences and inclusion-exclusion proceedings, with the following THE HONORABLE COURT OF APPEALS ERRED IN HOLDING THAT
choices: THE RULING IN THE CASE OF TOYOTA MOTOR PHILIPPINES
CORPORATION VS. TOYOTA MOTOR PHILIPPINES CORPORATION
A. Certification Election Among [Petitioner]’s Teaching Personnel: LABOR UNION (268 SCRA 573) DOES NOT APPLY IN THE CASE AT
BAR DESPITE THE [COMMINGLING] OF BOTH SUPERVISORY OR
1. Holy Child Catholic School Teachers and Employees Labor Union; and MANAGERIAL AND RANK-AND-FILE EMPLOYEES IN THE
RESPONDENT UNION;
2. No Union.
II
B. Certification Election Among [Petitioner]’s Non-Teaching Personnel: THE HONORABLE COURT OF APPEALS ERRED IN ITS
CONFLICTING RULING ALLOWING THE CONDUCT OF
1. Holy Child Catholic School Teachers and Employees Labor Union; and CERTIFICATION ELECTION BY UPHOLDING THAT THE
RESPONDENT UNION REPRESENTED A BARGAINING UNIT
2. No Union. DESPITE ITS OWN FINDINGS THAT THERE IS NO MUTUALITY OF
INTEREST BETWEEN THE MEMBERS OF RESPONDENT UNION
[Petitioner] is hereby directed to submit to the Regional Office of origin APPLYING THE TEST LAID DOWN IN THE CASE OF UNIVERSITY
within ten (10) days from receipt of this Decision, a certified separate list of OF THE PHILIPPINES VS. FERRER-CALLEJA (211 SCRA 451).37
its teaching and non-teaching personnel or when necessary a separate copy
of their payroll for the last three (3) months prior to the issuance of this We deny.
Decision.20
Petitioner claims that the CA contradicted the very definition of managerial
Petitioner filed a motion for reconsideration21 which, per Resolution dated and supervisory employees under existing law and jurisprudence when it did
February 13, 2003, was denied. Consequently, petitioner filed before the CA not classify the vice-principals, department head, and coordinators as
a Petition for Certiorari with Prayer for Temporary Restraining Order and managerial or supervisory employees merely because the policies and
Preliminary Injunction.22 The CA resolved to defer action on the prayer for guidelines they formulate are still subject to the review and evaluation of the
TRO pending the filing of private respondent’s Comment.23 Later, private principal or executive officers of petitioner. It points out that the duties of the
respondent and petitioner filed their Comment24 and Reply,25 respectively. vice-principals, department head, and coordinators include the evaluation
and assessment of the effectiveness and capability of the teachers under
On July 23, 2003, petitioner filed a motion for immediate issuance of a TRO, them; that such evaluation and assessment is independently made without the
alleging that Hon. Helen F. Dacanay of the Industrial Relations Division of participation of the higher Administration of petitioner; that the fact that their
the DOLE was set to implement the SOLE Decision when it received a recommendation undergoes the approval of the higher Administration does
summons and was directed to submit a certified list of teaching and non- not take away the independent nature of their judgment; and that it would be
teaching personnel for the last three months prior to the issuance of the difficult for the vice-principals, department head, and coordinators to
assailed Decision.26 Acting thereon, on August 5, 2003, the CA issued the objectively assess and evaluate the performances of teachers under them if
TRO and ordered private respondent to show cause why the writ of they would be allowed to be members of the same labor union.
preliminary injunction should not be granted.27 Subsequently, a
Manifestation and Motion28 was filed by private respondent, stating that it On the other hand, aside from reiterating its previous submissions, private
repleads by reference the arguments raised in its Comment and that it prays respondent cites Sections 9 and 1238 of Republic Act (R.A.) No.
for the immediate lifting of the TRO and the denial of the preliminary
injunction. The CA, however, denied the manifestation and motion on 9481 to buttress its contention that petitioner has no standing to oppose the
November 21, 200329 and, upon motion of petitioner,30 granted the petition for certification election. On the basis of the statutory provisions, it
preliminary injunction on April 21, 2005.31 Thereafter, both parties filed reasons that an employer is not a party-in-interest in a certification election;
their respective Memorandum.32 thus, petitioner does not have the requisite right to protect even by way of
restraining order or injunction.
On April 18, 2007, the CA eventually dismissed the petition. As to the
purported commingling of managerial, supervisory, and rank-and-file First off, We cannot agree with private respondent’s invocation of R.A. No.
employees in private respondent’s membership, it held that the Toyota ruling 9481. Said law took effect only on June 14, 2007; hence, its applicability is
is inapplicable because the vice-principals, department head, and limited to labor representation cases filed on or after said date.39 Instead, the
coordinators are neither supervisory nor managerial employees. It reasoned: law and rules in force at the time private respondent filed its petition for
certification election on May 31, 2002 are R.A. No. 6715, which amended
x x x While it may be true that they wield power over other subordinate Book V of Presidential Decree (P.D.) No. 442 (the Labor Code), as amended,
employees of the petitioner, it must be stressed[,] however[,] that their and the Rules and Regulations Implementing R.A. No. 6715, as amended by
functions are not confined with policy-determining such as hiring, firing, and D.O. No. 9, which was dated May 1, 1997 but took effect on June 21, 1997.40
disciplining of employees, salaries, teaching/working hours, other monetary
and non-monetary benefits, and other terms and conditions of employment. However, note must be taken that even without the express provision of
Further, while they may formulate policies or guidelines, nonetheless, such Section 12 of RA No. 9481, the “Bystander Rule” is already well entrenched
is merely recommendatory in nature, and still subject to review and in this jurisdiction. It has been consistently held in a number of cases that a
evaluation by the higher executives, i.e., the principals or executive officers certification election is the sole concern of the workers, except when the
of the petitioner. It cannot also be denied that in institutions like the employer itself has to file the petition pursuant to Article 259 of the Labor
petitioner, company policies have already been preformulated by the higher Code, as amended, but even after such filing its role in the certification
executives and all that the mentioned employees have to do is carry out these process ceases and becomes merely a bystander.41 The employer clearly
company policies and standards. Such being the case, it is crystal clear that lacks the personality to dispute the election and has no right to interfere at all
there is no improper [commingling] of members in the private respondent therein.42 This is so since any uncalled-for concern on the part of the
union as to preclude its petition for certification of (sic) election.33 employer may give rise to the suspicion that it is batting for a company
union.43 Indeed, the demand of the law and policy for an employer to take a
Anent the alleged mixture of teaching and non-teaching personnel, the CA strict, hands-off stance in certification elections is based on the rationale that
agreed with petitioner that the nature of the former’s work does not coincide the employees’ bargaining representative should be chosen free from any
with that of the latter. Nevertheless, it ruled that the SOLE did not commit extraneous influence of the management; that, to be effective, the bargaining
grave abuse of discretion in not dismissing the petition for certification representative must owe its loyalty to the employees alone and to no other.44
election, since it directed the conduct of two separate certification elections
based on Our ruling in University of the Philippines v. Ferrer-Calleja.34 Now, going back to petitioner’s contention, the issue of whether a petition
for certification election is dismissible on the ground that the labor
A motion for reconsideration35 was filed by petitioner, but the CA denied organization’s membership allegedly consists of supervisory and rank-
the same;36 hence, this petition assigning the alleged errors as follows: andfile employees is actually not a novel one. In the 2008 case of Republic
v. Kawashima Textile Mfg., Philippines, Inc.,45 wherein the
I. employercompany moved to dismiss the petition for certification election on
the ground inter alia that the union membership is a mixture of rank-and-file
and supervisory employees, this Court had conscientiously discussed the Art. 245. Ineligibility of managerial employees to join any labor
applicability of Toyota and Dunlop in the context of R.A. No. 6715 and D.O. organization; right of supervisory employees. Managerial employees are not
No. 9, viz.: eligible to join, assist or form any labor organization. Supervisory employees
shall not be eligible for membership in a labor organization of the rank-and-
It was in R.A. No. 875, under Section 3, that such questioned mingling was file employees but may join, assist or form separate labor organizations of
first prohibited, to wit: their own (Emphasis supplied)

Sec. 3. Employees' right to self-organization. - Employees shall have the right Unfortunately, just like R.A. No. 875, R.A. No. 6715 omitted specifying the
to self-organization and to form, join or assist labor organizations of their exact effect any violation of the prohibition would bring about on the
own choosing for the purpose of collective bargaining through legitimacy of a labor organization.
representatives of their own choosing and to engage in concerted activities
for the purpose of collective bargaining and other mutual aid or protection. It was the Rules and Regulations Implementing R.A. No. 6715 (1989
Individuals employed as supervisors shall not be eligible for membership in Amended Omnibus Rules) which supplied the deficiency by introducing the
a labor organization of employees under their supervision but may form following amendment to Rule II (Registration of Unions):
separate organizations of their own. (Emphasis supplied)
Sec. 1. Who may join unions. - x x x Supervisory employees and security
Nothing in R.A. No. 875, however, tells of how the questioned mingling can guards shall not be eligible for membership in a labor organization of the
affect the legitimacy of the labor organization. Under Section 15, the only rank-and-file employees but may join, assist or form separate labor
instance when a labor organization loses its legitimacy is when it violates its organizations of their own; Provided, that those supervisory employees who
duty to bargain collectively; but there is no word on whether such mingling are included in an existing rank-and-file bargaining unit, upon the effectivity
would also result in loss of legitimacy. Thus, when the issue of whether the of Republic Act No. 6715, shall remain in that unit x x x. (Emphasis supplied)
membership of two supervisory employees impairs the legitimacy of a rank-
and-file labor organization came before the Court En Banc in Lopez v. and Rule V (Representation Cases and Internal-Union Conflicts) of the
Chronicle Publication Employees Association, the majority pronounced: Omnibus Rules, viz.;

It may be observed that nothing is said of the effect of such ineligibility upon Sec. 1. Where to file. - A petition for certification election may be filed with
the union itself or on the status of the other qualified members thereof should the Regional Office which has jurisdiction over the principal office of the
such prohibition be disregarded. Considering that the law is specific where it employer. The petition shall be in writing and under oath.
intends to divest a legitimate labor union of any of the rights and privileges
granted to it by law, the absence of any provision on the effect of the Sec. 2. Who may file. - Any legitimate labor organization or the employer,
disqualification of one of its organizers upon the legality of the union, may when requested to bargain collectively, may file the petition. The petition,
be construed to confine the effect of such ineligibility only upon the when filed by a legitimate labor organization, shall contain, among others:
membership of the supervisor. In other words, the invalidity of membership
of one of the organizers does not make the union illegal, where the xxxx
requirements of the law for the organization thereof are, nevertheless,
satisfied and met. (Emphasis supplied) (c) description of the bargaining unit which shall be the employer unit unless
circumstances otherwise require; and provided further, that the appropriate
Then the Labor Code was enacted in 1974 without reproducing Sec. 3 of R.A. bargaining unit of the rank-and-file employees shall not include supervisory
No. 875. The provision in the Labor Code closest to Sec. 3 is Article 290, employees and/or security guards. (Emphasis supplied)
which is deafeningly silent on the prohibition against supervisory employees
mingling with rank-and-file employees in one labor organization. Even the By that provision, any questioned mingling will prevent an otherwise
Omnibus Rules Implementing Book V of the Labor Code (Omnibus Rules) legitimate and duly registered labor organization from exercising its right to
merely provides in Section 11, Rule II, thus: file a petition for certification election.

Sec. 11. Supervisory unions and unions of security guards to cease operation. Thus, when the issue of the effect of mingling was brought to the fore in
- All existing supervisory unions and unions of security guards shall, upon Toyota, the Court, citing Article 245 of the Labor Code, as amended by R.A.
the effectivity of the Code, cease to operate as such and their registration No. 6715, held:
certificates shall be deemed automatically cancelled. However, existing
collective agreements with such unions, the life of which extends beyond the Clearly, based on this provision, a labor organization composed of both rank-
date of effectivity of the Code shall be respected until their expiry date insofar and-file and supervisory employees is no labor organization at all. It cannot,
as the economic benefits granted therein are concerned. for any guise or purpose, be a legitimate labor organization. Not being one,
an organization which carries a mixture of rank-and-file and supervisory
Members of supervisory unions who do not fall within the definition of employees cannot possess any of the rights of a legitimate labor organization,
managerial employees shall become eligible to join or assist the rank and file including the right to file a petition for certification election for the purpose
organization. The determination of who are managerial employees and who of collective bargaining. It becomes necessary, therefore, anterior to the
are not shall be the subject of negotiation between representatives of granting of an order allowing a certification election, to inquire into the
supervisory union and the employer. If no agreement s reached between the composition of any labor organization whenever the status of the labor
parties, either or both of them may bring the issue to the nearest Regional organization is challenged on the basis of Article 245 of the Labor Code.
Office for determination. (Emphasis supplied)
xxxx
The obvious repeal of the last clause of Sec. 3, R.A. No. 875 prompted the
Court to declare in Bulletin v. Sanchez that supervisory employees who do In the case at bar, as respondent union's membership list contains the names
not fall under the category of managerial employees may join or assist in the of at least twenty-seven (27) supervisory employees in Level Five positions,
formation of a labor organization for rank-and-file employees, but they may the union could not, prior to purging itself of its supervisory employee
not form their own labor organization. members, attain the status of a legitimate labor organization. Not being one,
it cannot possess the requisite personality to file a petition for certification
While amending certain provisions of Book V of the Labor Code, E.O. No. election. (Emphasis supplied)
111 and its implementing rules continued to recognize the right of
supervisory employees, who do not fall under the category of managerial In Dunlop, in which the labor organization that filed a petition for
employees, to join a rank- and-file labor organization. certification election was one for supervisory employees, but in which the
membership included rank-and-file employees, the Court reiterated that such
Effective 1989, R.A. No. 6715 restored the prohibition against the questioned labor organization had no legal right to file a certification election to
mingling in one labor organization, viz.: represent a bargaining unit composed of supervisors for as long as it counted
rank-and-file employees among its members.
Sec. 18. Article 245 of the same Code, as amended, is hereby further
amended to read as follows:
It should be emphasized that the petitions for certification election involved Dunlop no longer hold sway in the present altered state of the law and the
in Toyota and Dunlop were filed on November 26, 1992 and September 15, rules. 46
1995, respectively; hence, the 1989 Rules was applied in both cases.
When a similar issue confronted this Court close to three years later, the
But then, on June 21, 1997, the 1989 Amended Omnibus Rules was further above ruling was substantially quoted in Samahang Manggagawa sa Charter
amended by Department Order No. 9, series of 1997 (1997 Amended Chemical Solidarity of Unions in the Philippines for Empowerment and
Omnibus Rules). Specifically, the requirement under Sec. 2(c) of the 1989 Reforms (SMCC-Super) v. Charter Chemical and Coating Corporation. 47
Amended Omnibus Rules - that the petition for certification election indicate In unequivocal terms, We reiterated that the alleged inclusion of supervisory
that the bargaining unit of rank-and-file employees has not been mingled employees in a labor organization seeking to represent the bargaining unit of
with supervisory employees - was removed. Instead, what the 1997 Amended rank-and-file employees does not divest it of its status as a legitimate labor
Omnibus Rules requires is a plain description of the bargaining unit, thus: organization. 48

Rule XI Indeed, Toyota and Dunlop no longer hold true under the law and rules
Certification Elections governing the instant case. The petitions for certification election involved in
Toyota and Dunlop were filed on November 26, 1992 and September 15,
xxxx 1995, respectively; hence, the 1989 Rules and Regulations Implementing
R.A. No. 6715 (1989 Amended Omnibus Rules) was applied. In contrast,
Sec. 4. Forms and contents of petition. - The petition shall be in writing and D.O. No. 9 is applicable in the petition for certification election of private
under oath and shall contain, among others, the following: x x x (c) The respondent as it was filed on May 31, 2002.
description of the bargaining unit.”
Following the doctrine laid down in Kawashima and SMCC-Super, it must
In Pagpalain Haulers, Inc. v. Trajano, the Court had occasion to uphold the be stressed that petitioner cannot collaterally attack the legitimacy of private
validity of the 1997 Amended Omnibus Rules, although the specific respondent by praying for the dismissal of the petition for certification
provision involved therein was only Sec. 1, Rule VI, to wit: election:

Sec. 1. Chartering and creation of a local/chapter.- A duly registered Except when it is requested to bargain collectively, an employer is a mere
federation or national union may directly create a local/chapter by submitting bystander to any petition for certification election; such proceeding is non-
to the Regional Office or to the Bureau two (2) copies of the following: a) a adversarial and merely investigative, for the purpose thereof is to determine
charter certificate issued by the federation or national union indicating the which organization will represent the employees in their collective
creation or establishment of the local/chapter; (b) the names of the bargaining with the employer. The choice of their representative is the
local/chapter's officers, their addresses, and the principal office of the exclusive concern of the employees; the employer cannot have any partisan
local/chapter; and (c) the local/ chapter's constitution and by-laws; provided interest therein; it cannot interfere with, much less oppose, the process by
that where the local/chapter's constitution and bylaws is the same as that of filing a motion to dismiss or an appeal from it; not even a mere allegation
the federation or national union, this fact shall be indicated accordingly. that some employees participating in a petition for certification election are
actually managerial employees will lend an employer legal personality to
All the foregoing supporting requirements shall be certified under oath by block the certification election. The employer's only right in the proceeding
the Secretary or the Treasurer of the local/chapter and attested to by its is to be notified or informed thereof.
President.
The amendments to the Labor Code and its implementing rules have
which does not require that, for its creation and registration, a local or chapter buttressed that policy even more. 49
submit a list of its members.
Further, the determination of whether union membership comprises
Then came Tagaytay Highlands Int'l. Golf Club, Inc. v. Tagaytay Highlands managerial and/or supervisory employees is a factual issue that is best left
Employees Union-PTGWO in which the core issue was whether mingling for resolution in the inclusion-exclusion proceedings, which has not yet
affects the legitimacy of a labor organization and its right to file a petition for happened in this case so still premature to pass upon. We could only
certification election. This time, given the altered legal milieu, the Court emphasize the rule that factual findings of labor officials, who are deemed to
abandoned the view in Toyota and Dunlop and reverted to its pronouncement have acquired expertise in matters within their jurisdiction, are generally
in Lopez that while there is a prohibition against the mingling of supervisory accorded not only with respect but even finality by the courts when supported
and rank-and-file employees in one labor organization, the Labor Code does by substantial evidence. 50 Also, the jurisdiction of this Court in cases
not provide for the effects thereof. Thus, the Court held that after a labor brought before it from the CA via Rule 45 is generally limited to reviewing
organization has been registered, it may exercise all the rights and privileges errors of law or jurisdiction. The findings of fact of the CA are conclusive
of a legitimate labor organization. Any mingling between supervisory and and binding. Except in certain recognized instances, 51 We do not entertain
rank-and-file employees in its membership cannot affect its legitimacy for factual issues as it is not Our function to analyze or weigh evidence all over
that is not among the grounds for cancellation of its registration, unless such again; the evaluation of facts is best left to the lower courts and administrative
mingling was brought about by misrepresentation, false statement or fraud agencies/quasi-judicial bodies which are better equipped for the task. 52
under Article 239 of the Labor Code.
Turning now to the second and last issue, petitioner argues that, in view of
In San Miguel Corp. (Mandaue Packaging Products Plants) v. Mandaue the improper mixture of teaching and non-teaching personnel in private
Packing Products Plants-San Miguel Packaging Products-San Miguel Corp. respondent due to the absence of mutuality of interest among its members,
Monthlies Rank-and-File Union-FFW, the Court explained that since the the petition for certification election should have been dismissed on the
1997 Amended Omnibus Rules does not require a local or chapter to provide ground that private respondent is not qualified to file such petition for its
a list of its members, it would be improper for the DOLE to deny recognition failure to qualify as a legitimate labor organization, the basic qualification of
to said local or chapter on account of any question pertaining to its individual which is the representation of an appropriate bargaining unit.
members.
We disagree.
More to the point is Air Philippines Corporation v. Bureau of Labor
Relations, which involved a petition for cancellation of union registration The concepts of a union and of a legitimate labor organization are different
filed by the employer in 1999 against a rank-and-file labor organization on from, but related to, the concept of a bargaining unit:
the ground of mixed membership: the Court therein reiterated its ruling in
Tagaytay Highlands that the inclusion in a union of disqualified employees Article 212(g) of the Labor Code defines a labor organization as "any union
is not among the grounds for cancellation, unless such inclusion is due to or association of employees which exists in whole or in part for the purpose
misrepresentation, false statement or fraud under the circumstances of collective bargaining or of dealing with employers concerning terms and
enumerated in Sections (a) and (c) of Article 239 of the Labor Code. conditions of employment." Upon compliance with all the documentary
requirements, the Regional Office or Bureau shall issue in favor of the
All said, while the latest issuance is R.A. No. 9481, the 1997 Amended applicant labor organization a certificate indicating that it is included in the
Omnibus Rules, as interpreted by the Court in Tagaytay Highlands, San roster of legitimate labor organizations. Any applicant labor organization
Miguel and Air Philippines, had already set the tone for it. Toyota and shall acquire legal personality and shall be entitled to the rights and privileges
granted by law to legitimate labor organizations upon issuance of the In the same manner, the teaching and non-teaching personnel of [petitioner]
certificate of registration. 53 school must form separate bargaining units. Thus, the order for the conduct
of two separate certification elections, one involving teaching personnel and
In case of alleged inclusion of disqualified employees in a union, the proper the other involving non-teaching personnel. It should be stressed that in the
procedure for an employer like petitioner is to directly file a petition for subject petition, [private respondent] union sought the conduct of a
cancellation of the union’s certificate of registration due to certification election among all the rank-and-file personnel of [petitioner]
misrepresentation, false statement or fraud under the circumstances school. Since the decision of the Supreme Court in the U.P. case prohibits us
enumerated in Article 239 of the Labor Code, as amended.54 To reiterate, from commingling teaching and non-teaching personnel in one bargaining
private respondent, having been validly issued a certificate of registration, unit, they have to be separated into two separate bargaining units with two
should be considered as having acquired juridical personality which may not separate certification elections to determine whether the employees in the
be attacked collaterally. respective bargaining units desired to be represented by [private respondent].
In the U.P. case, only one certification election among the non-academic
On the other hand, a bargaining unit has been defined as a "group of personnel was ordered, because ONAPUP sought to represent that bargaining
employees of a given employer, comprised of all or less than all of the entire unit only. No petition for certification election among the academic
body of employees, which the collective interests of all the employees, personnel was instituted by All U.P. Workers Union in the said case; thus,
consistent with equity to the employer, indicated to be best suited to serve no certification election pertaining to its intended bargaining unit was
reciprocal rights and duties of the parties under the collective bargaining ordered by the Court. 58
provisions of the law."55 In determining the proper collective bargaining unit
and what unit would be appropriate to be the collective bargaining agency, Indeed, the purpose of a certification election is precisely to ascertain the
the Court, in the seminal case of Democratic Labor Association v. Cebu majority of the employees’ choice of an appropriate bargaining unit – to be
Stevedoring Company, Inc., 56 mentioned several factors that should be or not to be represented by a labor organization and, if in the affirmative case,
considered, to wit: (1) will of employees (Globe Doctrine); (2) affinity and by which one. 59
unity of employees' interest, such as substantial similarity of work and duties,
or similarity of compensation and working conditions; (3) prior collective At this point, it is not amiss to stress once more that, as a rule, only questions
bargaining history; and (4) employment status, such as temporary, seasonal of law may be raised in a Rule 45 petition. In Montoya v. Transmed Manila
and probationary employees. We stressed, however, that the test of the Corporation, 60 the Court discussed the particular parameters of a Rule 45
grouping is community or mutuality of interest, because “the basic test of an appeal from the CA’s Rule 65 decision on a labor case, as follows:
asserted bargaining unit's acceptability is whether or not it is fundamentally
the combination which will best assure to all employees the exercise of their
collective bargaining rights." 57 x x x In a Rule 45 review, we consider the correctness of the assailed CA
decision, in contrast with the review for jurisdictional error that we undertake
As the SOLE correctly observed, petitioner failed to comprehend the full under Rule 65. Furthermore, Rule 45 limits us to the review of questions of
import of Our ruling in U.P. It suffices to quote with approval the apt law raised against the assailed CA decision. In ruling for legal correctness,
disposition of the SOLE when she denied petitioner’s motion for we have to view the CA decision in the same context that the petition for
reconsideration: certiorari it ruled upon was presented to it; we have to examine the CA
decision from the prism of whether it correctly determined the presence or
[Petitioner] likewise claimed that we erred in interpreting the decision of the absence of grave abuse of discretion in the NLRC decision before it, not on
Supreme Court in U.P. v. Ferrer-Calleja, supra. According to [petitioner], the the basis of whether the NLRC decision on the merits of the case was correct.
Supreme Court stated that the non-academic rank-and-file employees of the In other words, we have to be keenly aware that the CA undertook a Rule 65
University of the Philippines shall constitute a bargaining unit to the review, not a review on appeal, of the NLRC decision challenged before it.
exclusion of the academic employees of the institution. Hence, [petitioner] This is the approach that should be basic in a Rule 45 review of a CA ruling
argues, it sought the creation of separate bargaining units, namely: (1) in a labor case. In question form, the question to ask is: Did the CA correctly
[petitioner]’s teaching personnel to the exclusion of non-teaching personnel; determine whether the NLRC committed grave abuse of discretion in ruling
and (2) [petitioner]’s non-teaching personnel to the exclusion of teaching on the case? 61
personnel.
Our review is, therefore, limited to the determination of whether the CA
[Petitioner] appears to have confused the concepts of membership in a correctly resolved the presence or absence of grave abuse of discretion in the
bargaining unit and membership in a union. In emphasizing the phrase “to decision of the SOLE, not on the basis of whether the latter's decision on the
the exclusion of academic employees” stated in U.P. v. Ferrer-Calleja, merits of the case was strictly correct. Whether the CA committed grave
[petitioner] believed that the petitioning union could not admit academic abuse of discretion is not what is ruled upon but whether it correctly
employees of the university to its membership. But such was not the intention determined the existence or want of grave abuse of discretion on the part of
of the Supreme Court. the SOLE.

A bargaining unit is a group of employees sought to be represented by a WHEREFORE, the petition is DENIED. The April 18, 2007 Decision and
petitioning union. Such employees need not be members of a union seeking July 31, 2007, Resolution of the Court of Appeals in CA-G.R. SP No. 76175,
the conduct of a certification election. A union certified as an exclusive which affirmed the December 27, 2002 Decision of the Secretary of the
bargaining agent represents not only its members but also other employees Department of Labor and Employment that set aside the August 10, 2002
who are not union members. As pointed out in our assailed Decision, there Decision of the Med-Arbiter denying private respondent's petition for
were two contending unions in the U.P. case, namely[,] the Organization of certification election are hereby AFFIRMED.
Non-Academic Personnel of U.P. (ONAPUP) and the All U.P. Worker’s
Union composed of both U.P. academic and non-academic personnel. SO ORDERED.
ONAPUP sought the conduct of a certification election among the rank-and-
file non-academic personnel only, while the All U.P. Workers Union
intended to cover all U.P. rank-and-file employees, involving both academic
and non-academic personnel.

The Supreme Court ordered the “non-academic rank-and-file employees of


U.P. to constitute a bargaining unit to the exclusion of the academic
employees of the institution”, but did not order them to organize a separate
labor organization. In the U.P. case, the Supreme Court did not dismiss the
petition and affirmed the order for the conduct of a certification election
among the non-academic personnel of U.P., without
prejudice to the right of the academic personnel to constitute a separate
bargaining unit for themselves and for the All U.P. Workers Union to
institute a petition for certification election.
DE LA SALLE UNIVERSITY MEDICAL CENTER AND COLLEGE FFW. They are, for all intents and purposes, separate with each other and
OF MEDICINE vs. LAGUESMA their affiliation with FFW would not make them members of the same labor
G.R. No. 102084 | 1998-08-12 union. This must be the case because it is settled that the locals are considered
MENDOZA, J: the basic unit or principal with the labor federation assuming the role of an
agent. The mere fact, therefore, that they are represented by or under the same
Petitioner De La Salle University Medical Center and College of Medicine agent is of no moment. They are still considered separate with each other. 3
(DLSUMCCM) is a hospital and medical school at Dasmariñas, Cavite.
Private respondent Federation of Free Workers-De La Salle University On July 30, 1991, petitioner DLSUMCCM appealed to the Secretary of
Medical Center and College of Medicine Supervisory Union Chapter (FFW- Labor and Employment, citing substantially the same arguments it had raised
DLSUMCCMSUC), on the other hand, is a labor organization composed of before the med-arbiter. However, its appeal was dismissed. In his resolution,
the supervisory employees of petitioner DLSUMCCM. dated August 30, 1991, respondent Undersecretary of Labor and
Employment Bienvenido E. Laguesma found the evidence presented by
On April 17, 1991, the Federation of Free Workers (FFW), a national petitioner DLSUMCCM concerning the alleged managerial status of several
federation of labor unions, issued a certificate to private respondent FFW- employees to be insufficient, He also held that, following the ruling of this
DLSUMCCMSUC recognizing it as a local chapter. On the same day, it filed Court in Adamson & Adamson, Inc. v. CIR, 4 unions formed independently
on behalf of private respondent FFW-DLSUMCCMSUC a petition for by supervisory and rank-and-file employees of a company may legally
certification election among the supervisory employees of petitioner affiliate with the same national federation.
DLSUMCCM. Its petition was opposed by petitioner DLSUMCCM on the
grounds that several employees who signed the petition for certification Petitioner moved for a reconsideration but its motion was denied. In his order
election were managerial employees and that the FFW-DLSUMCCMSUC dated September 19, 1991, respondent Laguesma stated:
was composed of both supervisory and rank-and-file employees in the
company. 1 We reviewed the records once more, and find that the issues and arguments
adduced by movant have been squarely passed upon in the Resolution sought
In its reply dated May 22, 1991, private respondent FFW-DLSUMCCMSUC to be reconsidered. Accordingly, we find no legal justification to alter, much
denied petitioner's allegations. It contended that - less set aside, the aforesaid resolution. Perforce, the motion for
reconsideration must fail.
2. Herein petition seeks for the holding of a certification election among the
supervisory employees of herein respondent. It does not intend to include WHEREFORE, the instant motion for reconsideration is hereby denied for
managerial employees. lack of merit and the resolution of this office dated 30 August 1991
STANDS.
xxx xxx xxx
No further motions of a similar nature shall hereinafter be entertained. 5
6. It is not true that supervisory employees are joining the rank-and-file
employees' union. While it is true that both regular rank-and-file employees Hence, this petition for certiorari.
and supervisory employees of herein respondent have affiliated with FFW,
yet there are two separate unions organized by FFW. The supervisory Petitioner DLSUMCCM contends that respondent Laguesma gravely abused
employees have a separate charter certificate issued by FFW. 2 his discretion. While it does not anymore insist that several of those who
joined the petition for certification election are holding managerial positions
On July 5, 1991, respondent Rolando S. de la Cruz, med-arbiter of the in the company, petitioner nonetheless pursues the question whether unions
Department of Labor and Employment Regional Office No. IV, issued an formed independently by supervisory and rank-and-file employees of a
order granting respondent union's petition for certification election. He said: company may validly affiliate with the same national federation. With
respect to this question, it argues:
. . . [petitioner] . . . claims that based on the job descriptions which will be
presented at the hearing, the covered employees who are considered THE PUBLIC RESPONDENT, HONORABLE BIENVENIDO E.
managers occupy the positions of purchasing officers, personnel officers, LAGUESMA, UNDERSECRETARY OF LABOR AND EMPLOYMENT,
property officers, cashiers, heads of various sections and the like. IN A CAPRICIOUS, ARBITRARY AND WHIMSICAL EXERCISE OF
POWER ERRED AND COMMITTED GRAVE ABUSE OF DISCRETION
[Petitioner] also argues that assuming that some of the employees concerned AMOUNTING TO ACTING WITHOUT OR IN EXCESS OF
are not managerial but mere supervisory employees, the Federation of Free JURISDICTION WHEN HE DENIED THE PETITIONER'S APPEAL
Workers (FFW) cannot extend a charter certificate to this group of employees AND ORDERED THE HOLDING OF A CERTIFICATION ELECTION
without violating the express provision of Article 245 which provides that AMONG THE MEMBERS OF THE SUPERVISORY UNION
"supervisory employees shall not be eligible for membership in a labor EMPLOYED IN PETITIONER'S COMPANY DESPITE THE FACT THAT
organization of the rank-and-file employees but may join, assist or form SAID SUPERVISORY UNION WAS AFFILIATED WITH THE
separate labor organizations of their own" because the FFW had similarly FEDERATION OF FREE WORKERS TO WHICH THE RANK-AND-
issued a charter certificate to its rank-and-file employees. FILE EMPLOYEES OF THE SAME COMPANY ARE LIKEWISE
AFFILIATED, CONTRARY TO THE EXPRESS PROVISIONS OF
xxx xxx xxx ARTICLE 245 OF THE LABOR CODE, AS AMENDED. 6

In its position paper, [petitioner] stated that most, if not all, of the employees The contention has no merit.
listed in . . . the petition are considered managerial employees, thereby
admitting that it has supervisory employees who are undoubtedly qualified Supervisory employees have the right to self-organization as do other classes
to join or form a labor organization of their own. The record likewise shows of employees save only managerial ones. The Constitution states that "the
that [petitioner] promised to present the job descriptions of the concerned right of the people, including those employed in the public and private
employees during the hearing but failed to do so. Thus, this office has no sectors, to form unions, associations or societies for purposes not contrary to
basis in determining at this point in time who among them are considered law, shall not be abridged." 7 As we recently held in United Pepsi-Cola
managerial or supervisory employees. At any rate, there is now no question Supervisory Union v. Laguesma, 8 the framers of the Constitution intended,
that [petitioner] has in its employ supervisory employees who are qualified by this provision, to restore the right of supervisory employees to self-
to join or form a labor union. Consequently, this office is left with no organization which had been withdrawn from them during the period of
alternative but to order the holding of certification election pursuant to martial law. Thus:
Article 257 of the Labor Code, as amended, which mandates the holding of
certification election if a petition is filed by a legitimate labor organization Commissioner Lerum sought to amend the draft of what was later to become
involving an unorganized establishment, as in the case of herein respondent. Art. III, § 8 of the present Constitution:

As to the allegation of [petitioner] that the act of the supervisory employees xxx xxx xxx
in affiliating with FFW to whom the rank-and-file employees are also
affiliated is violative of Article 245 of the Labor Code, suffice it to state that MR. LERUM . . . Also, we have unions of supervisory employees and of
the two groups are considered separate bargaining units and local chapters of security guards. But what is tragic about this is that after the 1973
Constitution was approved and in spite of an express recognition of the right power in respect of the terms and conditions of labor. Yet the locals remained
to organize in P.D. No. 442, known as the Labor Code, the right of the basic units of association, free to serve their own and the common interest
government workers, supervisory employees and security guards to form of all, . . . and free also to renounce the affiliation for mutual welfare upon
unions was abolished. the terms laid down in the agreement which brought it to existence. 16

xxx xxx xxx The questions in this case, therefore, are whether the rank-and-file employees
of petitioner DLSUMCCM who compose a labor union are directly under the
We are afraid that without any corresponding provision covering the private supervisory employees whose own union is affiliated with the same national
sector, the security guards, the supervisory employees . . . will still be federation (Federation of Free Workers) and whether such national
excluded and that is the purpose of this amendment. federation is actively involved in union activities in the company so as to
make the two unions in the same company, in reality, just one union.
xxx xxx xxx
Although private respondent FFW-DLSUMCCMSUC and another union
In sum, Lerum's proposal to amend Art. III, § 8 of the draft Constitution composed of rank-and-file employees of petitioner DLSUMCCM are indeed
by including labor unions in the guarantee of organizational right should be affiliated with the same national federation, the FFW, petitioner
taken in the context of statements that his aim was the removal of the DLSUMCCM has not presented any evidence showing that the rank-and-file
statutory ban against security guards and supervisory employees joining employees composing the other union are directly under the authority of the
labor organizations. The approval by the Constitutional Commission of his supervisory employees. As held in Adamson & Adamson, Inc. v. CIR, 17 the
proposal can only mean, therefore, that the Commission intended the fact that the two groups of workers are employed by the same company and
absolute right to organize of government workers, supervisory employees, the fact that they are affiliated with a common national federation are not
and security guards to be constitutionally guaranteed. 9 sufficient to justify the conclusion that their organizations are actually just
one. Their immediate professional relationship must be established. To
Conformably with the constitutional mandate, Art. 245 of the Labor Code borrow the language of Adamson & Adamson, Inc. v. CIR: 18
now provides for the right of supervisory employees to self-organization,
subject to the limitation that they cannot join an organization of rank-and-file We find without merit the contention of petitioner that if affiliation will be
employees: allowed, only one union will in fact represent both supervisors and rank-and-
file employees of the petitioner; that there would be an indirect affiliation of
Supervisory employees shall not be eligible for membership in a labor supervisors and rank-and-file employees with one labor organization; that
organization of the rank-and-file employees but may join, assist or form there would be a merging of the two bargaining units; and that the respondent
separate labor organizations of their own. union will lose its independence because it becomes an alter ego of the
federation. 19
The reason for the segregation of supervisory and rank-and-file employees
of a company with respect to the exercise of the right to self-organization is Mention has already been made of the fact that the petition for certification
the difference in their interests. Supervisory employees are more closely election in this case was filed by the FFW on behalf of the local union. This
identified with the employer than with the rank-and-file employees. If circumstance, while showing active involvement by the FFW in union
supervisory and rank-and-file employees in a company are allowed to form activities at the company, is by itself insufficient to justify a finding of
a single union, the conflicting interests of these groups impair their violation of Art. 245 since there is no proof that the supervisors who compose
relationship and adversely affect discipline, collective bargaining, and the local union have direct authority over the rank-and-file employees
strikes. 10 These consequences can obtain not only in cases where composing the other local union which is also affiliated with the FFW. This
supervisory and rank-and-file employees in the same company belong to a fact differentiates the case from Atlas Lithographic Services, Inc. v.
single union but also where unions formed independently by supervisory and Laguesma, 20 in which, in addition to the fact that the petition for
rank-and-file employees of a company are allowed to affiliate with the same certification election had been filed by the national federation, it was shown
national federation. Consequently, this Court has held in Atlas Lithographic that the rank-and-file employees were directly under the supervisors
Services Inc. v. Laguesma 11 that - organized by the same federation.

To avoid a situation where supervisors would merge with the rank-and-file It follows that respondent labor officials did not gravely abuse their
or where the supervisors' labor organization would represent conflicting discretion.
interests, then a local supervisors' union should not be allowed to affiliate
with a national federation of unions of rank-and-file employees where that WHEREFORE, the petition is DISMISSED.
federation actively participates in union activities in the company.
SO ORDERED.
As we explained in that case, however, such a situation would obtain only
where two conditions concur: First, the rank-and-file employees are directly
under the authority of supervisory employees. 12 Second, the national
federation is actively involved in union activities in the company. 13 Indeed,
it is the presence of these two conditions which distinguished Atlas
Lithographic Services, Inc. v. Laguesma from Adamson & Adamson, Inc. v.
CIR 14 where a different conclusion was reached.

The affiliation of two local unions in a company with the same national
federation is not by itself a negation of their independence since in relation
to the employer, the local unions are considered as the principals, while the
federation is deemed to be merely their agent. This conclusion is in accord
with the policy that any limitation on the exercise by employees of the right
to self-organization guaranteed in the Constitution must be construed strictly.
Workers should be allowed the practice of this freedom to the extent
recognized in the fundamental law. As held in Liberty Cotton Mills Workers
Union v. Liberty Cotton Mills, Inc.: 15

The locals are separate and distinct units primarily designed to secure and
maintain an equality of bargaining power between the employer and their
employee members in the economic struggle for the fruits of the joint
productive effort of labor and capital; and the association of locals into the
national union . . . was in furtherance of the same end. These associations are
consensual entities capable of entering into such legal relations with their
members. The essential purpose was the affiliation of the local unions into a
common enterprise to increase by collective action the common bargaining
PHILIPPINE LABOR ALLIANCE COUNCIL (PLAC) vs. BUREAU From the very petition with its annexes, it is undisputed that there was a
OF LABOR RELATIONS finding in the challenged order by respondent Bureau of Labor Relations of
G.R. No. L-41288 | 1977-01-31 the non-ratification by the majority of the employees of the certified
FERNANDO, J: collective bargaining agreement, thus calling for its decertification. It is also
noteworthy that in the comment of respondent labor union, considered as its
It would be to frustrate the hopes that inspired the present Labor Code 1 to answer, the allegation that there was such a ratification was specifically
minimize judicial participation in the solution of employer-employee denied. It cannot be taken as having proven. There is nothing in the
disputes resort to the courts would remain unabated. Nevertheless, in view of exhaustive memorandum of petitioner either that would justify the
the certiorari jurisdiction of this Tribunal, 2 a grave abuse of discretion may imputation that respondent Bureau, in ordering decertification of the
be alleged as a grievance thus calling for remedial action. So petitioner collective bargaining agreement with petitioner to be followed by a
Philippine Labor Alliance Council did hope to achieve in this certiorari and certification election, committed a transgression of the present Labor Code,
prohibition proceeding against respondent Bureau of Labor Relations. 3 It much less one of such grievous character as to taint its actuation with a
would indict an order 4 for a certification election by respondent Bureau as jurisdictional infirmity. It is quite apparent therefore that with due
tainted by a jurisdictional infirmity in view of what is contended to be an recognition of the ability and scholarship evident in the pleadings of Attorney
existing duly certified collective bargaining contract between it and private Fortunato Gupit, Jr. for the petitioner, the attempt to invoke our certiorari
respondent Orion Manila, Inc., the employer. It would thus ignore the jurisdiction cannot succeed. 20 So it was noted at the outset.
withdrawal in the same order of such certification based on a finding that
there was a failure on the part of the majority of the employees in the 1.It is indisputable that the present controversy would not have arisen if there
bargaining unit to ratify the collective contract, renewed nine months before were no mass disaffiliation from petitioning union. Such a phenomenon is
the termination of the previous agreement. Apparently, the difficulty nothing new in the Philippine labor movement. 21 Nor is it open to any legal
confronting it was due to the disaffiliation of many of its members. The order objection. It is implicit in the freedom of association explicitly ordained by
complained of recognized that there was such a sentiment on the part of the Constitution. 22 There is then the incontrovertible right of any individual
sizable number of employees in the collective bargaining unit, thus making to join an organization of his choice. That option belongs to him. A
patent the desirability of conducting a certification election. That was the workingman is not to be denied that liberty. 23 He may be, as a matter of
method to determine the exclusive bargaining representative followed even fact, more in need of it if the institution of collective bargaining as an aspect
under the previous labor legislation. 5 It would thus appear rather obvious of industrial democracy is to succeed. No obstacle that may possible thwart
that the attempt to impute arbitrariness to respondent Bureau cannot be the desirable objective of militancy in labor's struggle for better terms and
attended with success. The petition must be dismissed. conditions is then to be placed on his way. Once the fact of disaffiliation has
been demonstrated beyond doubt, as in this case, a certification election is
It was a detailed narration of facts set forth in the petition, starting with the the most expeditious way of determining which labor organization is to be
allegation that there was a renewal of the collective bargaining agreement the exclusive bargaining representative. It is as simple as that. There is
with a union shop clause on March 9, 1974 between petitioner union and relevance to this excerpt from a recent decision, Philippine Association of
respondent company to last for another period of three (3) years Free Labor Unions v. Bureau of Labor Relations: 24 "Petitioner thus appears
incorporating therein new economic benefits to expire on December 31, to be woefully lacking in awareness of the significance of a certification
1977. 6 The claim was that at that time it was the only bargaining agent of election for the collective bargaining process. It is the fairest and most
the respondent company unchallenged by any labor organization. 7 Then effective way of determining which labor organization can truly represent the
came the assertion that on May 27 1974, with due notice to all the members working force. It is a fundamental postulate that the will of the majority, if
of the petitioner union, and with more than 1,500 of them present, such given expression in an honest election with freedom on the part of the voters
collective bargaining agreement was ratified by a unanimous vote. 8 It was to make their choice, is controlling. No better device can assure the institution
then so certified by the former National Labor Relations Commission on June of industrial democracy with the two parties to a business enterprise,
4, 1974. 9 It was further alleged that at the time of such certification, there management and labor, establishing a regime of self-rule. As was pointed out
was no pending request for union recognition by any other labor organization by Chief Justice Castro in Rivera v. San Miguel Brewery Corporation, Inc.,
with management. 10 Thereafter, on June 20, 1974, respondent Federation of 'a collective bargaining agreement is the law of the plant.' To the same effect
Free Workers, setting forth that its members represent more than 60% out of is this explicit pronouncement in Mactan Workers Union v. Aboitiz: 'The
1,500 members, more or less, rank-and-file employees of respondent terms and conditions of a collective bargaining contract constitute the law
company, sought a certification election. 11 Petitioner union, as could be between the parties.' What could be aptly stressed then, as was done in
expected, opposed such a move as in its view the collective bargaining Compania Maritima v. Compania Maritima Labor Union, is 'the primacy to
agreement entered into with the respondent company had been certified. 12 which the decision reached by the employees themselves is entitled.' Further,
It was sustained, the Secretary of Labor to whom an appeal was taken it was therein stated: 'That is in the soundest tradition of industrial
concurring with the former National Labor Relations Commission affirming democracy. For collective bargaining implies that instead of a unilateral
the dismissal of such petition for certification, on the ground of the existence imposition by management, the terms and conditions of employment should
of a certified collective bargaining agreement. 13 That did not end the be the subject of negotiation between it and labor. Thus the two parties
dispute, as respondent Federation on January 15, 1975, filed a complaint with indispensable to the economy are supposed to take care of their respective
the respondent Bureau of Labor Relations, the present Labor Code having interests. Moreover, the very notion of industrial self-rule negates the
become effective, alleging that some employees, numbering 848 in all, in a assumption that what is good for either party should be left to the will of the
resolution attached to the complaint disaffiliated from petitioner union and other. On the contrary, there is an awareness that labor can be trusted to
affiliated with it, characterizing the certified agreement as having been promote its welfare through the bargaining process. To it then must be left
entered into allegedly to thwart such disaffiliation and seeking a declaration the choice of its agent for such purpose.' To paraphrase an observation of the
of the nullity thereof. 14 After both petitioner union and respondent recently-retired Chief Justice Makalintal in Seno v. Mendoza, it is essential
Federation of Free Workers had filed their pleadings, 15 the Med-Arbiter, on that there be an agreement to govern the relations between labor marked by
March 20, 1975, dismissed the complaint. 16 There was a motion for confusion, with resulting breaches of the law by either party. There is, it
reconsideration, then an opposition. 17 On April 8, 1975, respondent Bureau would appear, a decidedly unsympathetic approach to the institution of
of Labor Relations issued an order setting aside the certification of the collective bargaining at war with what has so often and so consistently
collective bargaining agreement and ordering a certification election within decided by this Tribunal." 25
20 days from receipt of the order, upon the following declaration: "In the
instant case, it is not disputed that the collective bargaining agreement 2.A different conclusion could have been reached had there been no
certified by the National Labor Relations Commission was not ratified by the decertification. The contract-bar rule could then be invoked by petitioner. It
majority of the employees within the bargaining unit. This is defective. It is is, as pointed out by Justice Fernandez in Confederation of Citizens Labor
blatant non-observance of the basic requirement necessary to certification. . Unions v. National Labor Relations Commission, 26 "a principle in labor law
. . With respect to the complaint of the confirmation of disaffiliation of the that a collective bargaining agreement of reasonable duration is, in the
members of respondent Philippine Labor Alliance Council, the same should interest of the stability of industrial relations, a bar to certification elections."
be resolved in the most expedient and simple method of determining the 27 Even then, as was pointed out in the just-cited Philippine Association of
exclusive bargaining representative the holding of a certification election." Free Labor Unions decision, it "is not to be applied with rigidity. . . . The
18 There was a motion for reconsideration as well as a verified urgent petition element of flexibility in its operation cannot be ignored." 28 In this
filed with the Secretary of Labor by respondent Company, but the order was controversy, however, such a principle is not applicable. The collective
affirmed on July 31, 1975, the motion to consider being denied. 19 bargaining agreement entered into by petitioner with management on March
9, 1974 was decertified in the challenged order of April 8, 1975. 29 The
power to decertify by respondent Bureau is not disputed. It was the exercise
thereof that is now assailed. If done arbitrarily, there is valid ground for
complaint. The due process clause is a guarantee against any actuation of that
sort. It stands for fairness and justice. That standard was not ignored. It
suffices to read the petition to disprove any allegation of such failing, whether
in its procedural or substantive aspect. Petitioner was heard by respondent
Bureau before the order of decertification was issued on April 8, 1975. The
denial of its motion for reconsideration came also after it had an opportunity
to present its side. Procedural due process was thus observed. Nor was there
any denial of substantive due process in the sense of such decertification
being an act of arbitrariness and caprice.

In the order of April 8, 1975, it was specifically pointed out: "In the instant
case, it is not disputed that the collective bargaining agreement certified by
the National Labor Relations Commission was not ratified by the majority of
the employees within the bargaining unit. This is defective. It is blatant non-
observance of the basic requirement necessary to certification. To allow it to
remain uncorrected would allow circumvention of what the law specifically
ordained. We cannot countenance irregularities of the highest order to exist
in our very own eyes to be perpetuated. With respect to the complaint of the
confirmation of disaffiliation of the members of respondent Philippine Labor
Alliance Council, the same should be resolved in the most expedient and
simple method of determining the exclusive bargaining representative the
holding of a certification election." 30 In the order denying the motion for
reconsideration dated July 31, 1975, it was first noted: "On January 20, 1975,
FFW and 848 Orion employees filed with the Bureau a petition for the
annulment of the 1974 collective bargaining agreement and for the
confirmation of the disaffiliation of the 848 employees from PLAC and their
affiliation with FFW. The petition alleged among other, that the new
agreement was concluded about ten months before the expiry date of the old
purposely to defeat the right of the covered employees to choose their
bargaining representative at the proper time appointed by law. It appears,
indeed, that there was no urgency for the premature renegotiations
considering that the new agreement provides for a 50-centavo salary increase
effective yet on January 1, 1976." 31 Then, there was further clarification of
the decision reached as to the holding of a certification election being the
appropriate mode of solving the dispute: "With the decertification of the
collective agreement, the representation issue comes back to the fore.
Petitioner wants this resolved by ruling on the affiliation and disaffiliation of
the union. The Bureau holds, however, that certification election can better
resolve the issue. Parenthetically, it should be stated that a certification
election can still be held even if the collective agreement were certified,
considering the peculiar facts of the case. Good policy and equity demand
that when an agreement is renegotiated before the appointed 60-day period,
its certification must still give way to any representation issue that may be
raised within 60-day period so that the right of employees to choose a
bargaining unit agent and the right of unions to be chosen shall be preserved."
32

3.There is, finally, another insuperable obstacle to the success of this petition.
There is no need for a citation of authorities to show how well-settled and
firmly-rooted is the doctrine of the well-nigh conclusive respect for the
findings of facts of administrative tribunals, leaving to the judiciary, in the
ultimate analysis, this Tribunal, to set forth the correct legal norm applicable
to the controversy. With specific reference to the agencies at present dealing
with labor relations, there is this excerpt from Justice Aquino's opinion in
Antipolo Highway Lines, Inc. v. Inciong: 33 "A dispassionate scrutiny of the
proceedings in the NLRC does not sustain petitioners' view that they were
denied due process and that the NLRC committed a grave abuse of discretion.
(See Maglasang v. Ople, L-38813, April 29, 1975 per Justice Fernando). We
found no justification for setting aside the factual findings of the NLRC,
which like those of any other administrative agency, are generally binding on
the courts (Timbancaya v. Vicente, 62 O.G. 9424, 9 SCRA 852)." 34

WHEREFORE, this petition for certiorari and prohibition is dismissed. The


restraining order issued by this Court in its resolution of September 8, 1975
is hereby lifted. No costs.
MALAYANG SAMAHAN NG MGA MANGGAGAWA SA M. On March 21, 1987, a Petition for Impeachment was filed with the national
GREENFIELD (MSMG-UWP) vs. RAMOS federation ULGWP by the defeated candidates in the aforementioned
G.R. No. 113907 | 2000-02-28 election.
PURISIMA, J.:
On June 16, 1987, the federation conducted an audit of the local union funds.
At bar is a Petition for Certiorari under Rule 65 of the Revised Rules of Court The investigation did not yield any unfavorable result and the local union
to annul the decision of the National Labor Relations Commission in an officers were cleared of the charges of anomaly in the custody, handling and
unfair labor practice case instituted by a local union against its employer disposition of the union funds.
company and the officers of its national federation.
The 14 defeated candidates filed a Petition for Impeachment/Expulsion of
The petitioner, Malayang Samahan ng mga Manggagawa sa M. Greenfield, the local union officers with the DOLE NCR on November 5, 1987, docketed
Inc., (B) (MSMG), hereinafter referred to as the "local union", is an affiliate as NCR-OD-M-11-780-87. However, the same was dismissed on March 2,
of the private respondent, United Lumber and General Workers of the 1988, by Med-Arbiter Renato Parungo for failure to substantiate the charges
Philippines (ULGWP), referred to as the "federation". The collective and to present evidence in support of the allegations.
bargaining agreement between MSMG and M. Greenfield, Inc. names the
parties as follows: On April 17, 1988, the local union held a general membership meeting at the
Caruncho Complex in Pasig. Several union members failed to attend the
"This agreement made and entered into by and between: meeting, prompting the Executive Board to create a committee tasked to
investigate the non-attendance of several union members in the said
M. GREENFIELD, INC. (B) a corporation duly organized in accordance assembly, pursuant to Sections 4 and 5, Article V of the Constitution and By-
with the laws of the Republic of the Philippines with office address at Km. Laws of the union, which read:
14, Merville Road, Parañaque, Metro Manila, represented in this act by its
General manager, Mr. Carlos T. Javelosa, hereinafter referred to as the "Seksyon 4. Ang mga kinukusang hindi pagdalo o hindi paglahok sa lahat ng
Company; hakbangin ng unyon ng sinumang kasapi o pinuno ay maaaring maging sanhi
ng pagtitiwalag o pagpapataw ng multa ng hindi hihigit sa P50.00 sa bawat
-and- araw na nagkulang.

MALAYANG SAMAHAN NG MGA MANGGAGAWA SA M. Seksyon 5. Ang sinumang dadalo na aalis ng hindi pa natatapos ang pulong
GREENFIELD (B) (MSMG)/UNITED LUMBER AND GENERAL ay ituturing na pagliban at maparusahan ito ng alinsunod sa Article V,
WORKERS OF THE PHILIPPINES (ULGWP), a legitimate labor Seksyong 4 ng Saligang Batas na ito. Sino mang kasapi o pisyales na mahuli
organization with address at Suite 404, Trinity Building, T.M. Kalaw Street, and dating sa takdang oras ng di lalampas sa isang oras ay magmumulta ng
Manila, represented in this act by a Negotiating Committee headed by its P25.00 at babawasin sa sahod sa pamamagitan ng salary deduction at higit sa
National President, Mr. Godofredo Paceno, Sr., referred to in this Agreement isang oras ng pagdating ng huli ay ituturing na pagliban.3 [Rollo, p. 34]
as the UNION."1 [Rollo, p. 29]
On June 27, 1988, the local union wrote respondent company a letter
The CBA includes, among others, the following pertinent provisions: requesting it to deduct the union fines from the wages/salaries of those union
members who failed to attend the general membership meeting. A portion of
Article II-Union Security the said letter stated:

Section 1. Coverage and Scope. All employees who are covered by this "xxx xxx xxx
Agreement and presently members of the UNION shall remain members of
the UNION for the duration of this Agreement as a condition precedent to In connection with Section 4 Article II of our existing Collective Bargaining
continued employment with the COMPANY. Agreement, please deduct the amount of P50.00 from each of the union
members named in said annexes on the payroll of July 2-8, 1988 as fine for
xxxxxx their failure to attend said general membership meeting."4 [Rollo, p.35]

xxxxxx In a Memorandum dated July 3, 1988, the Secretary General of the national
federation, Godofredo Paceño, Jr. disapproved the resolution of the local
Section 4. Dismissal. Any such employee mentioned in Section 2 hereof, who union imposing the P50.00 fine. The union officers protested such action by
fails to maintain his membership in the UNION for non-payment of UNION the Federation in a Reply dated July 4, 1988.
dues, for resignation and for violation of UNION's Constitution and By-Laws
and any new employee as defined in Section 2 of this Article shall upon On July 11, 1988, the Federation wrote respondent company a letter advising
written notice of such failure to join or to maintain membership in the the latter not to deduct the fifty-peso fine from the salaries of the union
UNION and upon written recommendation to the COMPANY by the members requesting that:
UNION, be dismissed from the employment by the COMPANY; provided,
however, that the UNION shall hold the COMPANY free and blameless from " x x x any and all future representations by MSMG affecting a number of
any and all liabilities that may arise should the dismissed employee question, members be first cleared from the federation before corresponding action by
in any manner, his dismissal; provided, further that the matter of the the Company."5 [Ibid., p.40]
employee's dismissal under this Article may be submitted as a grievance
under Article XIII and, provided, finally, that no such written The following day, respondent company sent a reply to petitioner union's
recommendation shall be made upon the COMPANY nor shall COMPANY request in a letter, stating that it cannot deduct fines from the employees'
be compelled to act upon any such recommendation within the period of sixty salary without going against certain laws. The company suggested that the
(60) days prior to the expiry date of this Agreement conformably to law." union refer the matter to the proper government office for resolution in order
to avoid placing the company in the middle of the issue.
Article IX
The imposition of P50.00 fine became the subject of bitter disagreement
Section 4. Program Fund - The Company shall provide the amount of P10, between the Federation and the local union culminating in the latter's
000.00 a month for a continuing labor education program which shall be declaration of general autonomy from the former through Resolution No. 10
remitted to the Federation x x x."2 [Ibid, p. 30-31, p. 823-824] passed by the local executive board and ratified by the general membership
On September 12, 1986, a local union election was held under the auspices on July 16, 1988.
of the ULGWP wherein the herein petitioner, Beda Magdalena Villanueva,
and the other union officers were proclaimed as winners. Minutes of the said In retaliation, the national federation asked respondent company to stop the
election were duly filed with the Bureau of Labor Relations on September remittance of the local union's share in the education funds effective August
29, 1986. 1988. This was objected to by the local union which demanded that the
education fund be remitted to it in full.
The company was thus constrained to file a Complaint for Interpleader with
a Petition for Declaratory Relief with the Med-Arbitration Branch of the "Effective today, November 21, 1988, you are hereby expelled from
Department of Labor and Employment, docketed as Case No. OD-M-8-435- UNITED LUMBER AND GENERAL WORKERS OF THE PHILIPPINES
88. This was resolved on October 28, 1988, by Med-Arbiter Anastacio Bactin (ULGWP) for committing acts of disloyalty and/or acts inimical to the
in an Order, disposing thus: interest and violative to the Constitution and by-laws of your federation.

"WHEREFORE, premises considered, it is hereby ordered: You failed and/or refused to offer an explanation inspite of the time granted
to you.
1. That the United Lumber and General Workers of the Philippines
(ULGWP) through its local union officers shall administer the collective Since you are no longer a member of good standing, ULGWP is constrained
bargaining agreement (CBA). to recommend for your termination from your employment, and provided in
Article II Section 4, known as UNION SECURITY, in the Collective
2. That petitioner company shall remit the P10,000.00 monthly labor Bargaining agreement."9 [Ibid., 1523-1524]
education program fund to the ULGWP subject to the condition that it shall
use the said amount for its intended purpose. On the same day, the federation advised respondent company of the
expulsion of the 30 union officers and demanded their separation from
3. That the Treasurer of the MSMG shall be authorized to collect from the employment pursuant to the Union Security Clause in their collective
356 union members the amount of P50.00 as penalty for their failure to attend bargaining agreement. This demand was reiterated twice, through letters
the general membership assembly on April 17, 1988. dated February 21 and March 4, 1989, respectively, to respondent company.

However, if the MSMG Officers could present the individual written Thereafter, the Federation filed a Notice of Strike with the National
authorizations of the 356 union members, then the company is obliged to Conciliation and Mediation Board to compel the company to effect the
deduct from the salaries of the 356 union members the P50.00 fine."6 [Rollo, immediate termination of the expelled union officers.
p.47]
On March 7, 1989, under the pressure of a threatened strike, respondent
On appeal, Director Pura-Ferrer Calleja issued a Resolution dated February company terminated the 30 union officers from employment, serving them
7, 1989, which modified in part the earlier disposition, to wit: identical copies of the termination letter reproduced below:

"WHEREFORE, premises considered, the appealed portion is hereby We received a demand letter dated 21 November 1988 from the United
modified to the extent that the company should remit the amount of five Lumber and General Workers of the Philippines (ULGWP) demanding for
thousand pesos (P5,000.00) of the P10,000.00 monthly labor education your dismissal from employment pursuant to the provisions of Article II,
program fund to ULGWP and the other P5,000.00 to MSMG, both unions to Section 4 of the existing Collective Bargaining Agreement (CBA). In the said
use the same for its intended purpose."7 [Ibid, p. 48] demand letter, ULGWP informed us that as of November 21, 1988, you were
expelled from the said federation "for committing acts of disloyalty and/or
Meanwhile, on September 2, 1988, several local unions (Top Form, M. acts inimical to the interest of ULGWP and violative to its Constitution and
Greenfield, Grosby, Triumph International, General Milling, and Vander By-laws particularly Article V, Section 6, 9, and 12, Article XIII, Section 8."
Hons chapters) filed a Petition for Audit and Examination of the federation
and education funds of ULGWP which was granted by Med-Arbiter Rasidali In subsequent letters dated 21 February and 4 March 1989, the ULGWP
Abdullah on December 25, 1988 in an Order which directed the audit and reiterated its demand for your dismissal, pointing out that notwithstanding
examination of the books of account of ULGWP. your expulsion from the federation, you have continued in your employment
with the company in violation of Sec. 1 and 4 of Article II of our CBA, and
On September 30, 1988, the officials of ULGWP called a Special National of existing provisions of law.
Executive Board Meeting at Nasipit, Agusan del Norte where a Resolution
was passed placing the MSMG under trusteeship and appointing respondent In view thereof, we are left with no alternative but to comply with the
Cesar Clarete as administrator. provisions of the Union Security Clause of our CBA. Accordingly, we hereby
serve notice upon you that we are dismissing you from your employment
On October 27, 1988, the said administrator wrote the respondent company with M. Greenfield, Inc., pursuant to Sections 1 and 4, Article II of the CBA
informing the latter of its designation of a certain Alfredo Kalingking as local effective immediately."10 [Rollo, p. 58-59.]
union president and "disauthorizing" the incumbent union officers from
representing the employees. This action by the national federation was On that same day, the expelled union officers assigned in the first shift were
protested by the petitioners in a letter to respondent company dated physically or bodily brought out of the company premises by the company's
November 11, 1988. security guards. Likewise, those assigned to the second shift were not
allowed to report for work. This provoked some of the members of the local
On November 13, 1988, the petitioner union officers received identical letters union to demonstrate their protest for the dismissal of the said union officers.
from the administrator requiring them to explain within 72 hours why they Some union members left their work posts and walked out of the company
should not be removed from their office and expelled from union premises.
membership.
On the other hand, the Federation, having achieved its objective, withdrew
On November 26, 1988, petitioners replied: the Notice of Strike filed with the NCMB.

(a) Questioning the validity of the alleged National Executive Board On March 8, 1989, the petitioners filed a Notice of Strike with the NCMB,
Resolution placing their union under trusteeship; DOLE, Manila, docketed as Case No. NCMB-NCR-NS-03-216-89, alleging
the following grounds for the strike:
(b) Justifying the action of their union in declaring a general autonomy from
ULGWP due to the latter's inability to give proper educational, (a) Discrimination
organizational and legal services to its affiliates and the pendency of the audit
of the federation funds; (b) Interference in union activities

(c) Advising that their union did not commit any act of disloyalty as it has (c) Mass dismissal of union officers and shop stewards
remained an affiliate of ULGWP;
(d) Threats, coercion and intimidation
(d) Giving ULGWP a period of five (5) days to cease and desist from further
committing acts of coercion, intimidation and harrassment.8 [Rollo, p. 1522- (e) Union busting
1523.]
The following day, March 9, 1989, a strike vote referendum was conducted
However, as early as November 21, 1988, the officers were expelled from and out of 2, 103 union members who cast their votes, 2,086 members voted
the ULGWP. The termination letter read: to declare a strike.
Thereafter, the company transferred its administration and account/client
On March 10, 1989, the thirty (30) dismissed union officers filed an urgent servicing department at AFP-RSBS Industrial Park in Taguig, Metro Manila.
petition, docketed as Case No. NCMB-NCR-NS-03-216-89, with the Offfice For failure to find a suitable place in Metro Manila for relocation of its
of the Secretary of the Department of Labor and Employment praying for the factory and manufacturing operations, the company was constrained to move
suspension of the effects of their termination from employment. However, the said departments to Tacloban, Leyte. Hence, on April 16, 1990,
the petition was dismissed by then Secretary Franklin Drilon on April 11, respondent company accordingly notified its employees of a temporary
1989, the pertinent portion of which stated as follows: shutdown. in operations. Employees who were interested in relocating to
Tacloban were advised to enlist on or before April 23, 1990.
"At this point in time, it is clear that the dispute at M. Greenfield is purely an
intra-union matter. No mass lay-off is evident as the terminations have been The complaint for unfair labor practice was assigned to Labor Arbiter
limited to those allegedly leading the secessionist group leaving MSMG- Manuel Asuncion but was thereafter reassigned to Labor Arbiter Cresencio
ULGWP to form a union under the KMU. xxx Ramos when respondents moved to inhibit him from acting on the case.

xxx xxx xxx On December 15, 1992, finding the termination to be valid in compliance
with the union security clause of the collective bargaining agreement, Labor
WHEREFORE, finding no sufficient jurisdiction to warrant the exercise of Arbiter Cresencio Ramos dismissed the complaint.
our extraordinary authority under Article 277 (b) of the Labor Code, as
amended, the instant Petition is hereby DISMISSED for lack of merit. Petitioners then appealed to the NLRC. During its pendency, Commissioner
Romeo Putong retired from the service, leaving only two commissioners,
SO ORDERED."11 [Rollo, p. 937.] Commissioner Vicente Veloso III and Hon. Chairman Bartolome Carale in
the First Division. When Commissioner Veloso inhibited himself from the
On March 13 and 14, 1989, a total of 78 union shop stewards were placed case, Commissioner Joaquin Tanodra of the Third Division was temporarily
under preventive suspension by respondent company. This prompted the designated to sit in the First Division for the proper disposition of the case.
union members to again stage a walk-out and resulted in the official
declaration of strike at around 3:30 in the afternoon of March 14, 1989. The The First Division affirmed the Labor Arbiter's disposition. With the denial
strike was attended with violence, force and intimidation on both sides of their motion for reconsideration on January 28, 1994, petitioners elevated
resulting to physical injuries to several employees, both striking and non- the case to this Court, attributing grave abuse of discretion to public
striking, and damage to company properties. respondent NLRC in:

The employees who participated in the strike and allegedly figured in the I. UPHOLDING THE DISMISSAL OF THE UNION OFFICERS BY
violent incident were placed under preventive suspension by respondent RESPONDENT COMPANY AS VALID;
company. The company also sent return-to-work notices to the home
addresses of the striking employees thrice successively, on March 27, April II. HOLDING THAT THE STRIKE STAGED BYTHE PETITIONERS AS
8 and April 31, 1989, respectively. However, respondent company admitted ILLEGAL;
that only 261 employees were eventually accepted back to work. Those who
did not respond to the return-to-work notice were sent termination letters III. HOLDING THAT THE PETITIONER EMPLOYEES WERE DEEMED
dated May 17, 1989, reproduced below: TO HAVE ABANDONED THEIR WORK AND HENCE, VALIDLY
DISMISSED BY RESPONDENT COMPANY; AND
M. Greenfield Inc., (B)
Km. 14, Merville Rd., Parañaque, M.M. IV. NOT FINDING RESPONDENT COMPANY AND RESPONDENT
May 17, 1989 FEDERATION OFFICERS GUILTY OF ACTS OF UNFAIR LABOR
PRACTICE.
xxx
Notwithstanding the several issues raised by the petitioners and respondents
On March 14, 1989, without justifiable cause and without due notice, you in the voluminous pleadings presented before the NLRC and this Court, they
left your work assignment at the prejudice of the Company's operations. On revolve around and proceed from the issue of whether or not respondent
March 27, April 11, and April 21, 1989, we sent you notices to report to the company was justified in dismissing petitioner employees merely upon the
Company. Inspite of your receipt of said notices, we have not heard from you labor federation's demand for the enforcement of the union security clause
up to this date. embodied in their collective bargaining agreement.

Accordingly, for your failure to report, it is construed that you have Before delving into the main issue, the procedural flaw pointed out by the
effectively abandoned your employment and the Company is, therefore, petitioners should first be resolved.
constrained to dismiss you for said cause.
Petitioners contend that the decision rendered by the First Division of the
Very truly yours, NLRC is not valid because Commissioner Tanodra, who is from the Third
Division, did not have any lawful authority to sit, much less write the
M. GREENFIELD, INC., (B) ponencia, on a case pending before the First Division. It is claimed that a
commissioner from one division of the NLRC cannot be assigned or
By: temporarily designated to another division because each division is assigned
a particular territorial jurisdiction. Thus, the decision rendered did not have
WENZEL STEPHEN LIGOT any legal effect at all for being irregularly issued.

Asst. HRD Manager"12 [Rollo, p. 837.] Petitioners' argument is misplaced. Article 213 of the Labor Code in
enumerating the powers of the Chairman of the National Labor Relations
On August 7, 1989, the petitioners filed a verified complaint with the Commission provides that:
Arbitration Branch, National Capital Region, DOLE, Manila, docketed as
Case No. NCR-00-09-04199-89, charging private respondents of unfair labor "The concurrence of two (2) Commissioners of a division shall be necessary
practice which consists of union busting, illegal dismissal, illegal suspension, for the pronouncement of a judgment or resolution. Whenever the required
interference in union activities, discrimination, threats, intimidation, membership in a division is not complete and the concurrence of two (2)
coercion, violence, and oppresion. commissioners to arrive at a judgment or resolution cannot be obtained, the
Chairman shall designate such number of additional Commissioners from the
After the filing of the complaint, the lease contracts on the respondent other divisions as may be necessary."
company's office and factory at Merville Subdivision, Parañaque expired and
were not renewed. Upon demand of the owners of the premises, the company It must be remembered that during the pendency of the case in the First
was compelled to vacate its office and factory. Division of the NLRC, one of the three commissioners, Commissioner
Romeo Putong, retired, leaving Chairman Bartolome Carale and
Commissioner Vicente Veloso III. Subsequently, Commissioner Veloso
inhibited himself from the case because the counsel for the petitioners was done hastily and summarily. The company acts in bad faith in dismissing a
his former classmate in law school. The First Division was thus left with only worker without giving him the benefit of a hearing.
one commissioner. Since the law requires the concurrence of two
commisioners to arrive at a judgment or resolution, the Commission was "The power to dismiss is a normal prerogative of the employer. However,
constrained to temporarily designate a commissioner from another division this is not without limitation. The employer is bound to exercise caution in
to complete the First Division. There is nothing irregular at all in such a terminating the services of his employees especially so when it is made upon
temporary designation for the law empowers the Chairman to make the request of a labor union pursuant to the Collective Bargaining Agreement,
temporary assignments whenever the required concurrence is not met. The xxx. Dismissals must not be arbitrary and capricious. Due process must be
law does not say that a commissioner from the first division cannot be observed in dismissing an employee because it affects not only his position
temporarily assigned to the second or third division to fill the gap or vice but also his means of livelihood. Employers should respect and protect the
versa. The territorial divisions do not confer exclusive jurisdiction to each rights of their employees, which include the right to labor."
division and are merely designed for administrative efficiency.
In the case under scrutiny, petitioner union officers were expelled by the
Going into the merits of the case, the court finds that the Complaint for unfair federation for allegedly commiting acts of disloyalty and/or inimical to the
labor practice filed by the petitioners against respondent company which interest of ULGWP and in violation of its Constitution and By-laws. Upon
charges union busting, illegal dismissal, illegal suspension, interference in demand of the federation, the company terminated the petitioners without
union activities, discrimination, threats, intimidation, coercion, violence, and conducting a separate and independent investigation. Respondent company
oppression actually proceeds from one main issue which is the termination did not inquire into the cause of the expulsion and whether or not the
of several employees by respondent company upon the demand of the labor federation had sufficient grounds to effect the same. Relying merely upon the
federation pursuant to the union security clause embodied in their collective federation's allegations, respondent company terminated petitioners from
bargaining agreement. employment when a separate inquiry could have revealed if the federation
had acted arbitrarily and capriciously in expelling the union officers.
Petitioners contend that their dismissal from work was effected in an Respondent company's allegation that petitioners were accorded due process
arbitrary, hasty, capricious and illegal manner because it was undertaken by is belied by the termination letters received by the petitioners which state that
the respondent company without any prior administrative investigation; that, the dismissal shall be immediately effective.
had respondent company conducted prior independent investigation it would
have found that their expulsion from the union was unlawful similarly for As held in the aforecited case of Cariño, "the right of an employee to be
lack of prior administrative investigation; that the federation cannot informed of the charges against him and to reasonable opportunity to present
recommend the dismissal of the union officers because it was not a principal his side in a controversy with either the company or his own union is not
party to the collective bargaining agreement between the company and the wiped away by a union security clause or a union shop clause in a collective
union; that public respondents acted with grave abuse of discretion when they bargaining agreement. An employee is entitled to be protected not only from
declared petitioners' dismissals as valid and the union strike as illegal and in a company which disregards his rights but also from his own union the
not declaring that respondents were guilty of unfair labor practice. leadership of which could yield to the temptation of swift and arbitrary
expulsion from membership and mere dismissal from his job."
Private respondents, on the other hand, maintain that the thirty dismissed
employees who were former officers of the federation have no cause of action While respondent company may validly dismiss the employees expelled by
against the company, the termination of their employment having been made the union for disloyalty under the union security clause of the collective
upon the demand of the federation pursuant to the union security clause of bargaining agreement upon the recommendation by the union, this dismissal
the CBA; the expelled officers of the local union were accorded due process should not be done hastily and summarily thereby eroding the employees'
of law prior to their expulsion from their federation; that the strike conducted right to due process, self-organization and security of tenure. The
by the petitioners was illegal for noncompliance with the requirements; that enforcement of union security clauses is authorized by law provided such
the employees who participated in the illegal strike and in the commission of enforcement is not characterized by arbitrariness, and always with due
violence thereof were validly terminated from work; that petitioners were process.16 [Sanyo Philippines Workers Union-PSSLU vs. Canizares, 211
deemed to have abandoned their employment when they did not respond to SCRA 361] Even on the assumption that the federation had valid grounds to
the three return to work notices sent to them; that petitioner labor union has expell the union officers, due process requires that these union officers be
no legal personality to file and prosecute the case for and on behalf of the accorded a separate hearing by respondent company.
individual employees as the right to do so is personal to the latter; and that,
the officers of respondent company cannot be liable because as mere In its decision, public respondent also declared that if complainants (herein
corporate officers, they acted within the scope of their authority. petitioners) have any recourse in law, their right of action is against the
federation and not against the company or its officers, relying on the findings
Public respondent, through the Labor Arbiter, ruled that the dismissed union of the Labor Secretary that the issue of expulsion of petitioner union officers
officers were validly and legally terminated because the dismissal was by the federation is a purely intra-union matter.
effected in compliance with the union security clause of the CBA which is
the law between the parties. And this was affimed by the Commission on Again, such a contention is untenable. While it is true that the issue of
appeal. Moreover, the Labor Arbiter declared that notwithstanding the lack expulsion of the local union officers is originally between the local union and
of a prior administrative investigation by respondent company, under the the federation, hence, intra-union in character, the issue was later on
union security clause provision in the CBA, the company cannot look into converted into a termination dispute when the company dismissed the
the legality or illegality of the recommendation to dismiss by the union nd petitioners from work without the benefit of a separate notice and hearing.
the obligation to dismiss is ministerial on the part of the company.13 As a matter of fact, the records reveal that the the termination was effective
[Decicion of the Labor Arbiter, p. 16 (p. 197 of Rollo)] on the same day that the the termination notice was served on the petitioners.

This ruling of the NLRC is erroneous. Although this Court has ruled that In the case of Liberty Cotton Mills Workers Union vs. Liberty Cotton Mills,
union security clauses embodied in the collective bargaining agreement may Inc.17 [90 SCRA 391], the Court held the company liable for the payment of
be validly enforced and that dismissals pursuant thereto may likewise be backwages for having acted in bad faith in effecting the dismissal of the
valid, this does not erode the fundamental requirement of due process. The employees.
reason behind the enforcement of union security clauses which is the sanctity
and inviolability of contracts14 [Tanduay Distillery Labor Union vvs. "xxx Bad faith on the part of the respondent company may be gleaned from
NLRC, 149 SCRA 470 citing Victoria's Milling Co., Inc. vs. Victoria's- the fact that the petitioner workers were dismissed hastily and summarily. At
Manapla Workers' Organization, 9 SCRA 154] cannot override one's right to best, it was guilty of a tortious act, for which it must assume solidary liability,
due process. since it apparently chose to summarily dismiss the workers at the union's
instance secure in the union's contractual undertaking that the union would
In the case of Cariño vs. National Labor Relations Commission,15 [G. R. No. hold it 'free from any liability' arising from such dismissal."
91086, 8 May 1990, 185 SCRA 177.] this Court pronounced that while the
company, under a maintenance of membership provision of the collective Thus, notwithstanding the fact that the dismissal was at the instance of the
bargaining agreement, is bound to dismiss any employee expelled by the federation and that it undertook to hold the company free from any liability
union for disloyalty upon its written request, this undertaking should not be resulting from such a dismissal, the company may still be held liable if it was
remiss in its duty to accord the would-be dismissed employees their right to
be heard on the matter. Thus, a local union which has affiliated itself with a federation is free to sever
such affiliation anytime and such disaffiliation cannot be considered
Anent petitioners contention that the federation was not a principal party to disloyalty. In the absence of specific provisions in the federation's
the collective bargaining agreement between the company and the union, constitution prohibiting disaffiliation or the declaration of autonomy of a
suffice it to say that the matter was already ruled upon in the Interpleader local union, a local may dissociate with its parent union.24 [Ferrer vs.
case filed by respondent company. Med-Arbiter Anastacio Bactin thus ruled: National Labor Relations Commission, 224 SCRA 410; People's Industrial
and Commercial Employees and Workers Organization (FFW) vs. People's
After a careful examination of the facts and evidences presented by the Industrial and Commercial Corp., 112 SCRA 440]
parties, this Officer hereby renders its decision as follows:
The evidence on hand does not show that there is such a provision in
1.) It appears on record that in the Collective Bargaining Agreement (CBA) ULGWP's constitution. Respondents' reliance upon Article V, Section 6, of
which took effect on July 1, 1986, the contracting parties are M. Greenfield, the federation's constitution is not right because said section, in fact, bolsters
Inc. (B) and Malayang Samahan ng Mga Manggagawa sa M. Greenfield, Inc. the petitioner union's claim of its right to declare autonomy:
(B) (MSMG)/United Lumber and General Workers of the Philippines
(ULGWP). However, MSMG was not yet a registered labor organization at Section 6. The autonomy of a local union affiliated with ULGWP shall be
the time of the signing of the CBA. Hence, the union referred to in the CBA respected insofar as it pertains to its internal affairs, except as provided
is the ULGWP."18 [Rollo, p. 199] elsewhere in this Constitution.

Likewise on appeal, Director Pura Ferrer-Calleja put the issue to rest as There is no disloyalty to speak of, neither is there any violation of the
follows: federation's constitution because there is nothing in the said constitution
which specifically prohibits disaffiliation or declaration of autonomy. Hence,
It is undisputed that ULGWP is the certified sole and exclusive collective there cannot be any valid dismissal because Article II, Section 4 of the union
bargaining agent of all the regular rank-and-file workers of the company, M. security clause in the CBA limits the dismissal to only three (3) grounds, to
Greenfield, Inc. (pages 31-32 of the records). wit: failure to maintain membership in the union (1) for non-payment of
union dues, (2) for resignation; and (3) for violation of the union's
It has been established also that the company and ULGWP signed a 3-year Constitution and By-Laws.
collective bargaining agreement effective July 1, 1986 up to June 30, 1989.19
[Ibid.] To support the finding of disloyalty, the Labor Arbiter gave weight to the fact
that on February 26, 1989, the petitioners declared as vacant all the
Although the issue of whether or not the federation had reasonable grounds responsible positions of ULGWP, filled these vacancies through an election
to expel the petitioner union officers is properly within the original and and filed a petition for the registration of UWP as a national federation. It
exclusive jurisdiction of the Bureau of Labor Relations, being an intra-union should be pointed out, however, that these occurred after the federation had
conflict, this Court deems it justifiable that such issue be nonetheless ruled already expelled the union officers. The expulsion was effective November
upon, as the Labor Arbiter did, for to remand the same to the Bureau of Labor 21, 1988. Therefore, the act of establishing a different federation, entirely
Relations would be to intolerably delay the case. separate from the federation which expelled them, is but a normal retaliatory
reaction to their expulsion.
The Labor Arbiter found that petitioner union officers were justifiably
expelled from the federation for committing acts of disloyalty when it With regard to the issue of the legality or illegality of the strike, the Labor
"undertook to disaffiliate from the federation by charging ULGWP with Arbiter held that the strike was illegal for the following reasons: (1) it was
failure to provide any legal, educational or organizational support to the based on an intra-union dispute which cannot properly be the subject of a
local. x x x and declared autonomy, wherein they prohibit the federation from strike, the right to strike being limited to cases of bargaining deadlocks and
interfering in any internal and external affairs of the local union."20 [Rollo, unfair labor practice (2) it was made in violation of the "no strike, no lock-
p. 200] out" clause in the CBA, and (3) it was attended with violence, force and
intimidation upon the persons of the company officials, other employees
It is well-settled that findings of facts of the NLRC are entitled to great reporting for work and third persons having legitimate business with the
respect and are generally binding on this Court, but it is equally well-settled company, resulting to serious physical injuries to several employees and
that the Court will not uphold erroneous conclusions of the NLRC as when damage to company property.
the Court finds insufficient or insubstantial evidence on record to support
those factual findings. The same holds true when it is perceived that far too On the submission that the strike was illegal for being grounded on a non-
much is concluded, inferred or deduced from the bare or incomplete facts strikeable issue, that is, the intra-union conflict between the federation and
appearing of record.21 [Bontia vs. NLRC, 255 SCRA 167] the local union, it bears reiterating that when respondent company dismissed
the union officers, the issue was transformed into a termination dispute and
In its decision, the Labor Arbiter declared that the act of disaffiliation and brought respondent company into the picture. Petitioners believed in good
declaration of autonomy by the local union was part of its "plan to take over faith that in dismissing them upon request by the federation, respondent
the respondent federation." This is purely conjecture and speculation on the company was guilty of unfair labor pratice in that it violated the petitioner's
part of public respondent, totally unsupported by the evidence. right to self-organization. The strike was staged to protest respondent
company's act of dismissing the union officers. Even if the allegations of
A local union has the right to disaffiliate from its mother union or declare its unfair labor practice are subsequently found out to be untrue, the presumption
autonomy. A local union, being a separate and voluntary association, is free of legality of the strike prevails.25 [Master Iron Labor Union vs. National
to serve the interests of all its members including the freedom to disaffiliate Labor Relation s Commission , 219 SCRA 47.]
or declare its autonomy from the federation to which it belongs when
circumstances warrant, in accordance with the constitutional guarantee of Another reason why the Labor Arbiter declared the strike illegal is due to the
freedom of association.22 [Volkschel vs. Bureau of Labor Relations, 137 existence of a no strike no lockout provision in the CBA. Again, such a ruling
SCRA 42] is erroneous. A no strike, no lock out provision can only be invoked when
the strike is economic in nature, i.e. to force wage or other concessions from
The purpose of affiliation by a local union with a mother union or a federation the employer which he is not required by law to grant.26 [Panay Electric
Company Inc. vs. NLRC, 248 SCRA 688; Peoples' Industrial and
"xxx is to increase by collective action the bargaining power in respect of the Commercial Employees and Workers Organization (FFW) vs. PIC Corp.,
terms and conditions of labor. Yet the locals remained the basic units of 112 SCRA 440; Consolidated Labor Association of the Philippines vs.
association, free to serve their own and the common interest of all, subject to Marsman and Co., Inc., 11 SCRA 589; Master Iron Labor Union vs. NLRC,
the restraints imposed by the Constitution and By-Laws of the Association, 219 SCRA 47; Phil. Metal Foundries Inc. vs. CIR, 90 SCRA 135;] Such a
and free also to renounce the affiliation for mutual welfare upon the terms provision cannot be used to assail the legality of a strike which is grounded
laid down in the agreement which brought it into existence."23 [Tropical Hut on unfair labor practice, as was the honest belief of herein petitioners. Again,
Employees' Union-CGW vs. Tropical Hut Food Market Inc., 181 SCRA 173; whether or not there was indeed unfair labor practice does not affect the
Adamson, Inc. vs. CIR, 127 SCRA 268; Liberty Cotton Mills Worker Union strike.
vs. Liberty Cotton Mills, Inc., 66 SCRA 512]
On the allegation of violence committed in the course of the strike, it must the sanction imposed for such violation enunciated in Wenphil Corporation
be remembered that the Labor Arbiter and the Commission found that "the vs. NLRC32 [170 SCRA 69 (1989)] has become an ineffective deterrent.
parties are agreed that there were violent incidents x x x resulting to injuries Thus, the Court recently promulgated a decision to reinforce and make more
to both sides, the union and management."27 [Decision of the Labor Arbiter, effective the requirement of notice and hearing, a procedure that must be
Rollo, p. 203] The evidence on record show that the violence cannot be observed before termination of employment can be legally effected.
attributed to the striking employees alone for the company itself employed
hired men to pacify the strikers. With violence committed on both sides, the In Ruben Serrano vs. NLRC and Isetann Department Store (G.R. No.
management and the employees, such violence cannot be a ground for 117040, January 27, 2000), the Court ruled that an employee who is
declaring the strike as illegal. dismissed, whether or not for just or authorized cause but without prior notice
of his termination, is entitled to full backwages from the time he was
With respect to the dismissal of individual petitioners, the Labor Arbiter terminated until the decision in his case becomes final, when the dismissal
declared that their refusal to heed respondent's recall to work notice is a clear was for cause; and in case the dismissal was without just or valid cause, the
indication that they were no longer interested in continuing their employment backwages shall be computed from the time of his dismissal until his actual
and is deemed abandonment. It is admitted that three return to work notices reinstatement. In the case at bar, where the requirement of notice and hearing
were sent by respondent company to the striking employees on March 27, was not complied with, the aforecited doctrine laid down in the Serrano case
April 11, and April 21, 1989 and that 261 employees who responded to the applies.
notice were admittted back to work.
WHEREFORE, the Petition is GRANTED; the decision of the National
However, jurisprudence holds that for abandonment of work to exist, it is Labor Relations Commission in case No. NCR-00-09-04199-89 is
essential (1) that the employee must have failed to report for work or must REVERSED and SET ASIDE; and the respondent company is hereby
have been absent without valid or justifiable reason; and (2) that there must ordered to immediately reinstate the petitioners to their respective positions.
have been a clear intention to sever the employer-employee relationship Should reinstatement be not feasible, respondent company shall pay
manifested by some overt acts.28 [Philippine Advertising Counselors, Inc. separation pay of one month salary for every year of service. Since petitioners
vs. National Labor Relations Commission, 263 SCRA 395; Balayan Colleges were terminated without the requisite written notice at least 30 days prior to
vs. National Labor Relations Commission, 255 SCRA 1.] Deliberate and their termination, following the recent ruling in the case of Ruben Serrano
unjustified refusal on the part of the employee to go back to his work post vs. National Labor Relations Commission and Isetann Department Store, the
amd resume his employment must be established. Absence must be respondent company is hereby ordered to pay full backwages to petitioner-
accompanied by overt acts unerringly pointing to the fact that the employee employees while the Federation is also ordered to pay full backwages to
simply does not want to work anymore.29 [Nueva Ecija I Electric petitioner-union officers who were dismissed upon its instigation. Since the
Cooperative, Inc. vs. Minister of Labor, 184 SCRA 25, 30.] And the burden dismissal of petitioners was without cause, backwages shall be computed
of proof to show that there was unjustified refusal to go back to work rests from the time the herein petitioner employees and union officers were
on the employer. dismissed until their actual reinstatement. Should reinstatement be not
feasible, their backwages shall be computed from the time petitioners were
In the present case, respondents failed to prove that there was a clear intention terminated until the finality of this decision. Costs against the respondent
on the part of the striking employees to sever their employer-employee company.
relationship. Although admittedly the company sent three return to work
notices to them, it has not been substantially proven that these notices were SO ORDERED.
actually sent and received by the employees. As a matter of fact, some
employees deny that they ever received such notices. Others alleged that they
were refused entry to the company premises by the security guards and were
advised to secure a clearance from ULGWP and to sign a waiver. Some
employees who responded to the notice were allegedly told to wait for further
notice from respondent company as there was lack of work.

Furthermore, this Court has ruled that an employee who took steps to protest
his lay-off cannot be said to have abandoned his work.30 [Bontia vs. National
Labor Relations Commission, 255 SCRA 167; Batangas Laguna Tayabas
Bus Company vs. NLRC, 212 SCRA 792; Jackson Building Condominium
Corporation vs. NLRC, 246 SCRA 329] The filing of a complaint for illegal
dismissal is inconsistent with the allegation of abandonment. In the case
under consideration, the petitioners did, in fact, file a complaint when they
were refused reinstatement by respondent company.

Anent public respondent's finding that there was no unfair labor practice on
the part of respondent company and federation officers, the Court sustains
the same. As earlier discussed, union security clauses in collective bargaining
agreements, if freely and voluntarily entered into, are valid and binding.
Corrolarily, dismissals pursuant to union security clauses are valid and legal
subject only to the requirement of due process, that is, notice and hearing
prior to dismissal. Thus, the dismissal of an employee by the company
pursuant to a labor union's demand in accordance with a union security
agreement does not constitute unfair labor practice.31 [Tanduay Distillery
Labor Union vs. NLRC, 149 SCRA 470;Seno vs. Mendoza, 21 SCRA 1124.]

However, the dismissal was invalidated in this case because of respondent


company's failure to accord petitioners with due process, that is, notice and
hearing prior to their termination. Also, said dismissal was invalidated
because the reason relied upon by respondent Federation was not valid.
Nonetheless, the dismissal still does not constitute unfair labor practice.

Lastly, the Court is of the opinion, and so holds, that respondent company
officials cannot be held personally liable for damages on account of the
employees' dismissal because the employer corporation has a personality
separate and distinct from its officers who merely acted as its agents.

It has come to the attention of this Court that the 30-day prior notice
requirement for the dismissal of employees has been repeatedly violated and
NATIONAL UNION OF BANK EMPLOYEES (NUBE) VS. Thereafter, on June 20, 2003, the Board of Directors of NUBE-PEC adopted
PHILNABANK EMPLOYEES ASSOCIATION (PEMA) a Resolution6 disaffiliating itself from NUBE. Cited as reasons were as
G.R. No. 174287 | 2013-08-12 follows:
PERALTA, J.:
XXXX
Assailed in this petition for review on certiorari under Rule 45 of the 1997
Rules of Civil Procedure are the May 22, 2006 Decision 1 and August 17, WHEREAS, in the long period of time that the Union has been affiliated with
2006 Resolution 2 of the Court of Appeals (CA) in CA-G.R. SP No. 84606, NUBE, the latter has miserably failed to extend and provide satisfactory
which reversed the May 27, 2004 Decision 3 of the Secretary of Labor and services and support to the former in the form of legal services, training
Employment acting as voluntary arbitrator, the dispositive portion of which assistance, educational seminars, and the like;
states:
WHEREAS, this failure by NUBE to provide adequate essential services and
WHEREFORE, in light of the foregoing findings, the Bank is hereby support to union members have caused the latter to be resentful to NUBE and
ORDERED to release all union dues withheld and to continue remitting to to demand for the Union’s disaffiliation from the former[;]
NUBE-PNB chapter the members’ obligations under the CBA, LESS the
amount corresponding to the number of non—union members including WHEREAS, just recently, NUBE displayed its lack of regard for the interests
those who participated in the unsuccessful withdrawal of membership from and aspirations of the union members by blocking the latter’s desire for the
their mother union. early commencement of CBA negotiations with the PNB management[;]

The parties are enjoined to faithfully comply with the above-mentioned WHEREAS, this strained relationship between NUBE and the Union is no
resolution. longer conducive to a fruitful partnership between them and could even
threaten industrial peace between the Union and the management of PNB.
With respect to the URGENT MOTION FOR INTERVENTION filed by
PEMA, the same is hereby denied without prejudice to the rights of its WHEREAS, under the circumstances, the current officers of the Union have
members to bring an action to protect such rights if deemed necessary at the no choice but to listen to the clamor of the overwhelming majority of union
opportune time. members for the Union to disaffiliate from NUBE. 7

SO ORDERED. 4 The duly notarized Resolution was signed by Edgardo B. Serrana (President),
Rico B. Roma (Vice-President), Rachel C. Latorre (Secretary), Valeriana S.
We state the facts. Garcia (Director/Acting Treasurer), Ruben C. Medrano (Director), and Verlo
C. Magtibay (Director). It is claimed that said Resolution was
Respondent Philippine National Bank (PNB) used to be a govemment-owned overwhelmingly ratified by about eighty-one percent (81%) of the total union
and controlled banking institution established under Public Act 2612, as membership.
amended by Executive Order No. 80 dated December 3, 1986 (otherwise
known as The 1986 Revised Charter of the Philippine National Bank). Its On June 25, 2003, NUBE-PEC filed a Manifestation and Motion 8 before the
rank-and-file employees, being government personnel, were represented for Med-Arbitration Unit of DOLE, praying that, in View of its independent
collective negotiation by the Philnabank Employees Association (PEMA), a registration as a labor union and disaffiliation from NUBE, its name as
public sector union. appearing in the official ballots of the certification election be changed to
“Philnabank Employees Association (PEMA)” or, in the alternative, both
In 1996, the Securities and Exchange Commission approved PNB’s new parties be allowed to use the name “PEMA” but with PEMA-FFW and
Articles of Incorporation and By-laws and its changed status as a private NUBE-PEC be denominated as “PEMA-Bustria Group” and “PEMA-
corporation. PEMA affiliated with petitioner National Union of Bank Serrana Group,” respectively.
Employees (NUBE), which is a labor federation composed of unions in the
banking industry, adopting the name NUBE-PNB Employees Chapter On the same date, PEMA sent a letter to the PNB management informing its
(NUBE-PEC). disaffiliation from NUBE and requesting to stop, effective immediately, the
check-off of the £15.00 due for NUBE. 9
Later, NUBE-PEC was certified as the sole and exclusive bargaining agent
of the PNB rank-and-file employees. A collective bargaining agreement Acting thereon, on July 4, 2003, PNB informed NUBE of PEMA’s letter and
(CBA) was subsequently signed between NUBE-PEC and PNB covering the its decision to continue the deduction of the P15.00 fees, but stop its
period of January 1, 1997 to December 31, 2001. remittance to NUBE effective July 2003. PNB also notified NUBE that the
amounts collected would be held in a trust account pending the resolution of
Pursuant to Article V on Check-off and Agency Fees of the CBA, PNB shall the issue on PEMA’s disaffiliation. 10
deduct the monthly membership fee and other assessments imposed by the
union from the salary of each union member, and agency fee (equivalent to On July 11, 2003, NUBE replied that: it remains as the exclusive bargaining
the monthly membership dues) from the salary of the rank-and-file representative of the PNB rank-and-file employees; by signing the
employees within the bargaining unit who are not union members. Moreover, Resolution (on disaffiliation), the chapter officers have abandoned NUBE-
during the effectivity of the CBA, NUBE, being the Federation union, agreed PEC and joined another union; in abandoning NUBE-PEC, the chapter
that PNB shall remit 1215.00 of the £65.00 union dues per month collected officers have abdicated their respective positions and resigned as such; in
by PNB from every employee, and that PNB shall directly credit the amount joining another union, the chapter officers committed an act of disloyalty to
to NUBE’s current account with PNB. 5 NUBE-PEC and the general membership; the circumstances clearly show
that there is an emergency in NUBE-PEC necessitating its placement under
Following the expiration of the CBA, the Philnabank Employees temporary trusteeship; and that PNB should cease and desist from dealing
Association-FFW (PEMA-FFW) filed on January 2, 2002 a petition for with Serrana, Roma, Latorre, Garcia, Medrano, and Magtibay, who are
certification election among the rank-and-file employees of PNB. The expelled from NUBE-PEC. 11 With regard to the issue of non-remittance of
petition sought the conduct of a certification election to be participated in by the union dues, NUBE enjoined PNB to comply with the union check-off
PEMA-FFW and NUBE-PEC. provision of the CBA; otherwise, it would elevate the matter to the grievance
machinery in accordance with the CBA.
While the petition for certification election was still pending, two significant
events transpired — the independent union registration of NUBE-PEC and Despite NUBE’s response, PNB stood firm on its decision. Alleging unfair
its disaffiliation with NUBE. labor practice (ULP) for non-implementation of the grievance machinery and
procedure, NUBE brought the matter to the National Conciliation and
With a legal personality derived only from a charter issued by NUBE, Mediation Board (NCMB) for preventive mediation. 12 In time, PNB and
NUBE-PEC, under the leadership of Mariano Soria, decided to apply for a NUBE agreed to refer the case to the Office of the DOLE Secretary for
separate registration with the Department of Labor and Employment voluntary arbitration. They executed a Submission Agreement on October
(DOLE). On March 25, 2002, it was registered as an independent labor 28, 2003. 13
organization under Registration Certificate No. NCR-UR-3-3790-2002.
Meantime, the DOLE denied PEMA’s motion to change its name in the members, the disaffiliation is unjustified and the disaffiliated minority group
official ballots. The certification election was finally held on October 17, has no authority to represent the employees of the bargaining unit. This is the
2003. The election yielded the following results: import of the principle laid down in [Volkschel Labor Union v. Bureau of
Labor Relations supra] and the inverse application of the Supreme Court
Number of eligible voters 3,742 decision in [Philippine Federation of Petroleum Workers v. CIR] regarding
Number of valid votes cast 2,993 entitlement to the check-off provision of the CBA.
Number of spoiled ballots 72
Total 3,065 As a necessary consequence to our finding that no valid disaffiliation took
place, the right of NUBE to represent its local chapter at the PNB, less those
Philnabank Employees Association-FFW 289 employees who are no longer members of the latter, is beyond reproach.
National Union of Bank Employees (NUBE)-
Philippine National Bank (PNB) Chapter 2,683 However, the Bank cannot be faulted for not releasing union dues to NUBE
No Union 21 at the time when representation status issue was still being threshed out by
Total 2,993 14 proper governmental authority. Prudence dictates the discontinuance of
remittance of union dues to NUBE under such circumstances was a
On April 28, 2004, PEMA filed before the voluntary arbitrator an Urgent legitimate exercise of management discretion apparently in order to protect
Motion for Intervention, 15 alleging that it stands to be substantially affected the Bank’s business interest. The suspension of the check-off provision of
by whatever judgment that may be issued, because one of the issues for the CBA, at the instance of the latter made in good faith, under the present
resolution is the validity of its disaffiliation from NUBE. It further claimed circumstances cannot give rise to a right of action. For having been exercised
that its presence is necessary so that a complete relief may be accorded to the without malice much less evil motive and for not causing actual loss to the
parties. Only NUBE opposed the motion, arguing that PEMA has no legal National Union of Bank Employees (NUBE), the same act of management
personality to intervene, as it is not a party to the existing CBA; and that [cannot] be penalized. 17
NUBE is the exclusive bargaining representative of the PNB rank-and-file
employees and, in dealing with a union other than NUBE, PNB is violating Aggrieved, PEMA filed before the CA a petition under Rule 43 of the Rules
the duty to bargain collectively, which is another form of ULP. 16 on Civil Procedure with prayer for the issuance of a temporary restraining
order (TRO) or writ of preliminary injunction (WPI). On November 2, 2004,
Barely a month after, DOLE Acting Secretary Manuel G. Imson denied the CA denied the application for WPL 18 PEMA’s motion for
PEMA’s motion for intervention and ordered PNB to release all union dues reconsideration was also denied on February 24, 2005, noting PNB’s
withheld and to continue remitting the same to NUBE. The May 27, 2004 manifestation that it would submit to the judgment of the CA as to which
Decision opined: party it should remit the funds collected from the employees. 19

Before we delve into the merits of the present dispute, it behooves [Us] to On June 21, 2005, however, petitioner again filed an Urgent Motion for the
discuss in passing the propriety of the MOTION FOR INTERVENTION Issuance of a TRO against the June 10, 2005 Resolution of DOLE Acting
filed by the Philnabank Employees Association (PEMA) on April 28, 2004, Secretary Imson, which ordered PNB to properly issue a check directly
the alleged [break-away] group of NUBE-PNB Chapter. payable to the order of NUBE covering the withheld funds from the trust
account. 20 Considering the different factual milieu, the CA resolved to grant
A cursory reading of the motion reveals a denial thereof is not prejudicial to the motion. 21
the individual rights of its members. They are protected by law.
Subsequent to the parties’ submission of memoranda, the CA promulgated
Coming now to the main issues of the case, suffice it to say that after an its May 22, 2006 Decision, declaring the validity of PEMA’s disaffiliation
evaluative review of the record of the case, taking into consideration the from NUBE and directing PNB to return to the employees concerned the
arguments and evidence adduced by both parties, We find that indeed no amounts deducted and held in trust for NUBE starting July 2003 and to stop
effective disaffiliation took place. further deductions in favor of NUBE. 22

It is well settled that [l]abor unions may disaffiliate from their mother As to the impropriety of denying PEMA’s motion for intervention, the CA
federations to form a local or independent union only during the 60-day noted:
freedom period immediately preceding the expiration of the CBA. [Tanduay
Distillery Labor Union v. National Labor Relations Commission, et al.] x x x Among the rights of the [PEMA] as an affiliate of a federation is to
However, such disaffiliation must be effected by a majority of the members disaffiliate from it. Any case in which this is an issue is then one in which
in the bargaining unit. (Volkschel Labor Union v. Bureau of Labor the union has a significant legal interest and as to which it must be heard,
Relations). irrespective of any residual rights of the members after a decision that might
deny a disaffiliation. It is a non—sequitur to make the intervention of the
Applying the foregoing jurisprudence to the case at bar, it is difficult to union in this case dependent on the question of whether its members can
believe that a justified disaffiliation took place. While the record apparently pursue their own agenda under the same constraints. 23
shows that attempts at disaffiliation occurred sometime in June of 2003 x x
x the latest result of a certification election dated 17 October 2003 mooted On the validity of PEMA’s disaffiliation, the CA ratiocinated:
such disaffiliation.
The power and freedom of a local union to disaffiliate from its mother union
Further, even if for the sake of argument an attempt at disaffiliation occurred, or federation is axiomatic. As Volkschel vs. Bureau of Labor Relations [137
the record is bereft of substantial evidence to support a finding of effective SCRA 42] recognizes, a local union is, after all, a separate and voluntary
disaffiliation. There might have been a mass withdrawal of the union association that under the constitutional guarantee of freedom of expression
members from the NUBE-PNB Chapter. The record shows, however, that is free to serve the interests of its members. Such right and freedom
only 289 out of 3,742 members shifted their allegiance from the mother invariably include the right to disaffiliate or declare its autonomy from the
union. Hence, they constituted a small minority for which reason they could federation or mother union to which it belongs, subject to reasonable
not have successfully severed the local union’s affiliation with NUBE. restrictions in the law or the federation’s constitution. [Malayang Samahan
ng mga Manggagawa sa M. Greenfield vs. Ramos, 326 SCRA 428]
Thus, since only a minority of the members wanted disaffiliation as shown
by the certification election, it can be inferred that the majority of the Without any restrictive covenant between the parties, [Volkschel Labor
members wanted the union to remain an affiliate of the NUBE. [Villar, et al. Union vs. Bureau of Labor Relations, supra, at 48,] it is instructive to look
v. Inciong, et al.]. There being no justified disaffiliation that took place, the into the state of the law on a union’s right to disaffiliate. The voluntary
bargaining agent’s right under the provision of the CBA on Check—Off is arbitrator alludes to a provision in PD 1391 allowing disaffiliation only
unaffected and still remained with the old NUBE-PNB Chapter. x x x within a 60-day period preceding the expiration of the CBA. In Alliance of
Nationalist and Genuine Labor Organization vs. Samahan ng mga
While it is true that the obligation of an employee to pay union dues is co— Manggagawang Nagkakaisa sa Manila Bay Spinning Mills, etc. [258 SCRA
te1minus with his affiliation [Philippine Federation of Petroleum Workers v. 371], however, the rule was not held to be iron-clad. Volkschel was cited to
CIR], it is equally tenable that when it is shown, as in this case, that the support a more flexible view that the right may be allowed as the
withdrawal from the mother union is not supported by majority of the circumstances warrant. In Associated Workers Union—PTGWO vs.
National Labor Relations Commission [188 SCRA 123], the right to is a finding grounded entirely on speculation, surmises and conjectures; (2)
disaffiliate was upheld before the onset of the freedom period when it became When the inference made is manifestly mistaken, absurd or impossible; (3)
apparent that there was a shift of allegiance on the part of the majority of the Where there is a grave abuse of discretion; (4) When the judgment is based
union members. on a misapprehension of facts; (5) When the findings of fact are conflicting;
(6) When the CA, in making its findings, went beyond the issues of the case
XXXX and the same is contrary to the admissions of both parties; (7) When the
findings are contrary to those of the trial court; (8) When the findings of fact
As the records show, a majority, indeed a vast majority, of the members of are conclusions without citation of specific evidence on which they are
the local union ratified the action of the board to disaffiliate. Our count of the based; (9) When the facts set forth in the petition as well as in the petitioner’s
members who approved the board action is, 2,638. If we divide this by the main and reply briefs are not disputed by the respondents; and (10) When the
number of eligible voters as per the certification election which is 3,742, the findings of fact of the CA are premised on the supposed absence of evidence
quotient is 70.5%, representing the proportion of the members in favor of and contradicted by the evidence on record. 28 The Court finds no cogent
disaffiliation. The [PEMA] says that the action was ratified by 81%. Either reason to apply these recognized exceptions.
way, the groundswell of support for the measure was overwhelming.
Even a second look at the records reveals that the arguments raised in the
The respondent NUBE has developed the ingenious theory that if the petition are bereft of merit.
disaffiliation was approved by a majority of the members, it was neutered by
the subsequent certification election in which NUBE-PNB Chapter was The right of the local union to exercise the right to disaffiliate from its mother
voted the sole and exclusive bargaining agent. It is argued that the effects of union is well settled in this jurisdiction. In MSMG- U WP v. Hon. Ramos,
this change must be upheld as the latest expression of the will of the 29 We held:
employees in the bargaining unit. The truth of the matter is that the names of
PEMA and NUBE—PNB Chapter are names of only one entity, the two sides A local union has the right to disaffiliate from its mother union or declare its
of the same coin. We have seen how NUBE-PNB Employees Chapter autonomy. A local union, being a separate and voluntary association, is free
evolved into PEMA and competed with Philnabank Employees Association- to serve the interests of all its members including the freedom to disaffiliate
FFW for supremacy in the certification election. To realize that it was PEMA or declare its autonomy from the federation which it belongs when
which entered into the contest, we need only to remind ourselves that PEMA circumstances warrant, in accordance with the constitutional guarantee of
was the one which filed a motion in the certification election case to have its freedom of association.
name PEMA put in the official ballot. DOLE insisted, however, in putting
the name NUBE—PNB Chapter in the ballots unaware of the implications of The purpose of affiliation by a local union with a mother union [or] a
this seemingly innocuous act. 24 federation

NUBE filed a motion for reconsideration, but it was denied; 25 hence, this "x x x is to increase by collective action the bargaining power in respect of
petition raising the following issues for resolution: the terms and conditions of labor. Yet the locals remained the basic units of
association, free to serve their own and the common interest of all, subject to
I. The Secretary of Labor acted without error and without grave abuse of the restraints imposed by the Constitution and By-Laws of the Association,
discretion in not giving due course to the urgent motion for intervention filed and free also to renounce the affiliation for mutual welfare upon the terms
by PEMA. laid down in the agreement which brought it into existence."

II. The Secretary of Labor acted without grave abuse of discretion and Thus, a local union which has affiliated itself with a federation is free to sever
without serious error in ruling that PEMA’s alleged disaffiliation was invalid. such affiliation anytime and such disaffiliation cannot be considered
disloyalty. In the absence of specific provisions in the federation's
III. The Secretary of Labor did not commit serious error in ordering the constitution prohibiting disaffiliation or the declaration of autonomy of a
release of the disputed union fees/dues to NUBE-PNB Chapter. local union, a local may dissociate with its parent union. 30

IV. There is no substantial basis for the issuance of a preliminary injunction Likewise, Philippine Skylanders, Inc. v. National Labor Relations
or temporary restraining order. Commission 31 restated:

V. Under the Rules of Court, the appeal/petition of PEMA should have been The right of a local union to disaffiliate from its mother federation is not a
dismissed. novel thesis unillumined by case law. In the landmark case of Liberty Cotton
Mills Workers Union vs. Liberty Cotton Mills, Inc., we upheld the right of
VI. PEMA and NUBE are not one and the same, and the denial by the local unions to separate from their mother federation on the ground that as
Secretary of Labor of the motion for intervention was proper. separate and voluntary associations, local unions do not owe their creation
and existence to the national federation to which they are affiliated but,
VII. NUBE-PNB Chapter, not PEMA, has been fighting for PNB rank-and— instead, to the will of their members. The sole essence of affiliation is to
file interests and rights since PNB’s privatization, which is further proof that increase, by collective action, the common bargaining power of local unions
NUBE-PNB Chapter and PEMA are not one and the same. for the effective enhancement and protection of their interests. Admittedly,
there are times when without succor and support local unions may find it
VIII. The alleged disaffiliation was not valid as proper procedure was not hard, unaided by other support groups, to secure justice for themselves.
followed.
Yet the local unions remain the basic units of association, free to serve their
IX. NUBE is entitled to check-off. 26 own interests subject to the restraints imposed by the constitution and by—
laws of the national federation, and free also to renounce the affiliation upon
Stripped of the non-essential, the issue ultimately boils down on whether the terms laid down in the agreement which brought such affiliation into
PEMA validly disaffiliated itself from NUBE, the resolution of which, in existence.
turn, inevitably affects the latter’s right to collect the union dues held in trust
by PNB. Such dictum has been punctiliously followed since then. 32

We deny the petition. And again, in Coastal Subic Bay Terminal, Inc. v. Department of Labor and
Employment — Office of the Secretary, 33 this Court opined:
Whether there was a valid disaffiliation is a factual issue. 27 It is elementary
that a question of fact is not appropriate for a petition for review on certiorari Under the rules implementing the Labor Code, a chartered local union
under Rule 45 of the Rules of Court. The parties may raise only questions of acquires legal personality through the charter certificate issued by a duly
law because the Supreme Court is not a trier of facts. As a general rule, We registered federation or national union, and reported to the Regional Office
are not duty-bound to analyze again and weigh the evidence introduced in in accordance with the rules implementing the Labor Code. A local union
and considered by the tribunals below. When supported by substantial does not owe its existence to the federation with which it is affiliated. It is a
evidence, the findings of fact of the CA are conclusive and binding on the separate and distinct voluntary association owing its creation to the will of
parties and are not reviewable by this Court, except: (1) When the conclusion its members. Mere affiliation does not divest the local union of its own
personality, neither does it give the mother federation the license to act Consequently, by PEMA’s valid disaffiliation from NUBE, the vinculum that
independently of the local union. It only gives rise to a contract of agency, previously bound the two entities was completely severed. As NUBE was
where the former acts in representation of the latter. Hence, local unions are divested of any and all power to act in representation of PEMA, any act
considered principals while the federation is deemed to be merely their agent. performed by the former that affects the interests and affairs of the latter,
As such principals, the unions are entitled to exercise the rights and privileges including the supposed expulsion of Serrana et al., is rendered without force
of a legitimate labor organization, including the right to seek certification as and effect.
the sole and exclusive bargaining agent in the appropriate employer unit. 34
Also, in effect, NUBE loses it right to collect all union dues held in its trust
Finally, the recent case of Cirtek Employees Labor Union-Federation of Free by PNB. The moment that PEMA separated from and left NUBE and exists
Workers v. Cirtek Electronics, Inc 35 ruled: as an independent labor organization with a certificate of registration, the
former is no longer obliged to pay dues and assessments to the latter;
x x x [A] local union may disaffiliate at any time from its mother federation, naturally, there would be no longer any reason or occasion for PNB to
absent any showing that the same is prohibited under its constitution or rule. continue making deductions. 42 As we said in Volkschel Labor Union v.
Such, however, does not result in it losing its legal personality altogether. Bureau of Labor Relations: 43
Verily, Anglo—KMU v. Samahan Ng Mga Manggagawang Nagkakaisa Sa
Manila Bar Spinning Mills At J.P. Coats enlightens: x x x In other words, ALUMETAL [NUBE in this case] is entitled to receive
the dues from respondent companies as long as petitioner union is affiliated
A local labor union is a separate and distinct unit primarily designed to secure with it and respondent companies are authorized by their employees
and maintain an equality of bargaining power between the employer and their (members of petitioner union) to deduct union dues. Without said affiliation,
employee—members. A local union does not owe its existence to the the employer has no link to the mother union. The obligation of an employee
federation with which it is affiliated. It is a separate and distinct voluntary to pay union dues is coterminous with his affiliation or membership. "The
association owing its creation to the will of its members. The mere act of employees‘ check-off authorization. even if declared irrevocable, is good
affiliation does not divest the local union of its own personality, neither does only as long as they remain members of the union concerned." A contract
it give the mother federation the license to act independently of the local between an employer and the parent organization as bargaining agent for the
union. It only gives rise to a contract of agency where the former acts in employees is terminated by the disaffiliation of the local of which the
representation of the latter. 36 employees are members. x x x 44

These and many more have consistently reiterated the earlier view that the On the other hand, it was entirely reasonable for PNB to enter into a CBA
right of the local members to withdraw from the federation and to form a new with PEMA as represented by Serrana et al. Since PEMA had validly
local union depends upon the provisions of the union's constitution, by-laws separated itself from NUBE, there would be no restrictions which could
and charter and, in the absence of enforceable provisions in the federation's validly hinder it from collectively bargaining with PNB.
constitution preventing disaffiliation of a local union, a local may sever its
relationship with its parent. 37 In the case at bar, there is nothing shown in WHEREFORE, the foregoing considered, the instant Petition is DENIED.
the records nor is it claimed by NUBE that PEMA was expressly forbidden The May 22, 2006 Decision and August 17, 2006 Resolution of the Court of
to disaffiliate from the federation nor were there any conditions imposed for Appeals in CA-G.R. SP No. 84606, which reversed the May 27, 2004
a valid breakaway. This being so, PEMA is not precluded to disaffiliate from Decision of the Secretary of Labor and Employment, are AFFIRMED.
NUBE after acquiring the status of an independent labor organization duly
registered before the DOLE. SO ORDERED.

Also, there is no merit on NUBE’s contention that PEMA’s disaffiliation is


invalid for non-observance of the procedure that union members should make
such determination through secret ballot and after due deliberation,
conformably with Article 241 (d) of the Labor Code, as amended. 38
Conspicuously, other than citing the opinion of a “recognized labor law
authority,” NUBE failed to quote a specific provision of the law or rule
mandating that a local union’s disaffiliation from a federation must comply
with Article 241 (d) in order to be valid and effective.

Granting, for argument’s sake, that Article 241 (d) is applicable, still, We
uphold PEMA’s disaffiliation from NUBE. First, non-compliance with the
procedure on disaffiliation, being premised on purely technical grounds
cannot rise above the employees’ fundamental right to self-organization and
to form and join labor organizations of their own choosing for the purpose of
collective bargaining. 39 Second, the Article nonetheless provides that when
the nature of the organization renders such secret ballot impractical, the union
officers may make the decision in behalf of the general membership. In this
case, NUBE did not even dare to contest PEMA’s representation that “PNB
employees, from where [PEMA] [derives] its membership, are scattered from
Aparri to Jolo, marming more than 300 branches in various towns and cities
of the country,” hence, “[to] gather the general membership of the union in a
general membership to vote through secret balloting is virtually impossible.”
40 It is understandable, therefore, why PEMA’s board of directors merely
opted to submit for ratification of the majority their resolution to disaffiliate
from NUBE. Third, and most importantly, NUBE did not dispute the
existence of the persons or their due execution of the document showing their
unequivocal support for the disaffiliation of PEMA from NUBE. Note must
be taken of the fact that the list of PEMA members (identifying themselves
as “PEMA-Serrana Group" 41) who agreed with the board resolution was
attached as Annex “H” of PEMA’s petition before the CA and covered pages
115 to 440 of the CA rollo. While fully displaying the employees’ printed
name, identification number, branch, position, and signature, the list was left
unchallenged by NUBE. No evidence was presented that the union members’
ratification was obtained by mistake or through fraud, force or intimidation.
Surely, this is not a case where one or two members of the local union
decided to disaffiliate from the mother federation, but one where more than
a majority of the local union members decided to disaffiliate.
ALLIANCE OF NATIONALIST AND GENUINE LABOR protection to labor and social justice provisions of the Constitution and the
ORGANIZATION (ANGLO-KMU) vs. SAMAHAN NG MGA labor laws, rules or regulations, we have always adopted the liberal approach
MANGGAGAWANG NAGKAKAISA SA MANILA BAY SPINNING which favors the exercise of labor rights. 2
MILLS AT J.P. COATS (SAMANA BAY)
G.R. No. 118562 | 1996-07-05 This Court is not ready to bend this principle to yield to a mere procedural
FRANCISCO, J.: defect, to wit: failure to observe certain procedural requirements for a valid
disaffiliation. Non-compliance with the procedure on disaffiliation, being
Petitioner Alliance of Nationalist and Genuine Labor Organization (ANGLO premised on purely technical grounds cannot rise above the fundamental
for brevity) is a duly registered labor organization while respondent union right of self-organization. 3
Samahan Ng Mga Manggagawang Nagkakaisa sa Manila Bay Spinning Mills
and J.P. Coats (SAMANA BAY for brevity) is its affiliate. In representation We quote, with approval, the findings of herein public respondent, that:
of SAMANA BAY, ANGLO entered and concluded a Collective Bargaining
Agreement (CBA) with Manila Bay Spinning Mills and J.P. Coats Manila . . . the resolution of the general membership ratifying the disaffiliation action
Bay, Inc. (hereinafter referred to as the corporations) on November 1, 1991. initiated by the Board, substantially satisfies the procedural requirements for
On December 4, 1993, the Executive Committee of SAMANA BAY decided disaffiliation. No doubt was raised on the support of the majority of the union
to disaffiliate from ANGLO in view of the latter's dereliction of its duty to members on the decision todisaffiliate. 4
promote and advance the welfare of SAMANA BAY and the alleged cases
of corruption involving the federation officers. Said disaffiliation was This, to our mind, is clearly supported by the evidence. ANGLO's alleged
unanimously confirmed by the members of SAMANA BAY. acts inimical to the interests of respondent union have not been sufficiently
rebutted. It is clear under the facts that respondent union's members have
On April 4, 1994, a petition to stop remittance of federation dues to ANGLO unanimously decided to disaffiliate from the mother federation and ANGLO
was filed by SAMANA BAY with the Bureau of Labor Relations on the has nothing to offer in dispute other than the law prohibiting the disaffiliation
ground that the corporations, despite having been furnished copies of the outside the freedom period.
union resolution relating to said disaffiliation, refused to honor the same.
ANGLO counter-acted by unseating all officers and board members of In the same wise, We find no ground for ruling against the validity of the
SAMANA BAY and appointing, in their stead, a new set of officers who disaffiliation in the light of recent jurisprudential rules.
were duly recognized by the corporations.
Although P.D. 1391 provides:
In its position paper, ANGLO contended that the disaffiliation was void
considering that a collective bargaining agreement is still existing and the Item No. 6. No petition for certification election, for intervention and
freedom period has not yet set in. The Med-Arbiter resolved that the disaffiliation shall be entertained or given due course except within the 60-
disaffiliation was void but upheld the illegality of the ouster of the officers day freedom period immediately preceeding the expiration of a collective
of SAMANA BAY. Both parties filed their respective appeals with the bargaining agreement,
Department of Labor and Employment. In a resolution dated September 23,
1994, herein public respondent modified the order and ruled in favor of said law is definitely not without exceptions. Settled is the rule that a local
respondent union, disposing as follows: union has the right to disaffiliate from its mother union when circumstances
warrant. 5 Generally, a labor union may disaffiliate from the mother union to
WHEREFORE, the appeal of respondent ANGLO is hereby denied for lack form a local or independent union only during the 60-day freedom period
of merit while the appeal of petitioners is hereby granted. Accordingly, the immediately preceding the expiration of the CBA. However, even before the
order of the Med-Arbiter is modified by: onset of the freedom period, disaffiliation may be carried out when there is a
shift of allegiance on the part of the majority of the members of the union. 6
1) declaring the disaffiliation of petitioner union from respondent ANGLO
as valid; Coming now to the second issue, ANGLO contends that individual private
respondents were validly ousted as they have ceased to be officers of the
2) directing respondent Manila Bay Spinning Mills, Inc. and J.P. Coats to incumbent union (ANGLO-KMU) at the time of disaffiliation. In order to fill
stop remitting to ANGLO federation dues and instead to remit the whole the vacuum, it was deemed proper to appoint the individual replacements so
amount of union dues to the treasurer of petitioner union; and as not to put in disarray the organizational structure and to prevent chaos and
confusion among the general membership and within the company.
3) enjoining ANGLO-KMU from interfering in the affairs of petitioner
union. The contention is bereft of merit. A local labor union is a separate and distinct
unit primarily designed to secure and maintain an equality of bargaining
SO ORDERED. 1 power between the employer and their employee-members. A local union
does not owe its existence to the federation with which it is affiliated. It is a
ANGLO filed a motion for reconsideration but the same was denied for lack separate and distinct voluntary association owing its creation to the will of
of merit. Hence, this petition for certiorari under Rule 65. its members. 7 The mere act of affiliation does not divest the local union of
its own personality, neither does it give the mother federation the license to
The petition calls upon us to resolve two issues, to wit: act independently of the local union. It only gives rise to a contract of agency
8 where the former acts in representation of the latter.
1) whether the disaffiliation was valid; and
By SAMANA BAY's disaffiliation from ANGLO, the vinculum that
2) whether petitioner can validly oust individual private respondents from previously bound the two entities was completely severed. ANGLO was
their positions. divested of any and all power to act in representation of SAMANA BAY.
Thus, any act performed by ANGLO affecting the interests and affairs of
We rule for the respondents. SAMANA BAY, including the ouster of herein individual private
respondent, is rendered without force and effect.
For clarity, we shall first consider the issue respecting the validity of the
disaffiliation. WHEREFORE, premises considered, the petition is hereby DISMISSED.

Petitioner ANGLO wants to impress on us that the disaffiliation was invalid


for two reasons, namely: that the procedural requirements for a valid
disaffiliation were not followed; and that it was made in violation of P.D
1391.

Anent the first ground, we reiterate the rule that all employees enjoy the right
to self-organization and to form and join labor organizations of their own
choosing for the purpose of collective bargaining. This is a fundamental right
of labor and derives its existence from the Constitution. In interpreting the
VOLKSCHEL LABOR UNION vs. BUREAU OF LABOR On January 25, 1977, respondent Bureau, through its Acting Director,
RELATIONS Francisco L. Estrella, REVERSED the Med-Arbiter's Resolution, and
G.R. No. L-45824 | 1985-06-19 declared that the Bureau recognized "the continued affiliation of Volkschel
CUEVAS, J.: Labor Union with the Associated Labor Union for Metal Workers." 2

Petition for certiorari to review the Resolutions dated January 25, 1977 and Petitioner appealed the Acting Director's Resolution to the Secretary of Labor
March 14, 1977 of the Bureau of Labor Relations. (now Minister of Labor and Employment) who, treating the appeal as a
Motion for Reconsideration, referred the same back to respondent Bureau.
On April 25, 1977, however, a Supplemental Petition was filed seeking the On March 14, 1977, the Bureau denied the appeal for lack of merit.
issuance of -
Hence, the instant petition.
(1) A preliminary mandatory injunction commanding respondents to return
to petitioner the union dues amounting to about P55,000.00 lawfully Meanwhile, on April 4, 1977, on motion of ALUMETAL, the then Acting
pertaining to it but illegally levied upon, collected and handed over by Secretary of Labor, Amado Gat Inciong, issued a writ of execution
respondent Bureau, acting through the NLRC sheriff, to respondent commanding the Sheriff of the National Labor Relations Commission "to
Associated Labor Union for Metalworkers, with the collusion of respondents enforce and execute the order of January 25, 1977, which has become final
DMG, Inc., Karbayan, Inc. and RTC Machineries, Inc.; and executory. 3 Pursuant thereto, the NLRC Sheriff enforced and
implemented the Order of January 25, 1977, as a result of which respondent
(2) A preliminary restraining order prohibiting respondents from making companies turned over and handed to respondent federation the union dues
further delivery to respondent Associated Labor Union for Metalworkers of and other assessments in accordance with the check-off provision of the
Union dues collected or to be collected through check-off from the wages of CBA.
petitioner's members by respondents, DMG, Inc., Karbayan, Inc., RTC
Machineries, Inc., and People's Car, Inc., under or by virtue of the questioned From the pleadings filed and arguments of counsel, the following issues
writ of execution issued by respondent Bureau, dated April 4, 1977. present themselves for this Court's resolution.

Petitioner was once affiliated with the Associated Labor Union for Metal I
Workers (ALUMETAL for short). On August 1, 1975, both unions, using the Is petitioner union's disaffiliation from respondent federation valid?.
name Volkschel Labor Union Associated Labor Union for Metal Workers,
jointly entered into a collective bargaining agreement with respondent II
companies. One of the subjects dealt with is the payment of union dues which Do respondent companies have the right to effect union dues collections
is provided for in Section 3, Article I, of the CBA, which reads: despite revocation by the employees of the check-off authorization? and

"Section 3. CHECK-OFF. - The COMPANY agrees to make payroll III


deductions not oftener than twice a month of UNION membership dues and Is respondent federation entitled to union dues payments from petitioner
such special assessments fees or fines as may be duly authorized by the union's members notwithstanding their disaffiliation from said federation?
UNION, provided that the same is covered by the individual check-off
authorization of the UNION members. All said deductions shall be promptly We resolve the first issue in the affirmative.
transmitted within five (5) days by the COMPANY to the UNION Treasurer.
The COMPANY shall prepare two (2) checks. One (1) check will be under The right of a local union to disaffiliate from its mother union is well-settled.
the name of the local union as their local fund including local special In previous cases, it has been repeatedly held that a local union, being a
assessment funds and the other check will be for the ALU Regional Office separate and voluntary association, is free to serve the interest of all its
regarding the remittance of the UNION dues deduction." members including the freedom to disaffiliate when circumstances warrant.
4 This right is consistent with the Constitutional guarantee of freedom of
On March 10, 1976, a majority of petitioner's members decided to disaffiliate association (Article IV, Section 7, Philippine Constitution).
from respondent federation in order to operate on its own as an independent
labor group pursuant to Article 241 (formerly Article 240) of the Labor Code Petitioner contends that the disaffiliation was not due to any opportunists
of the Philippines, the pertinent portion of which reads: motives on its part. Rather it was prompted by the federation's deliberate and
habitual dereliction of duties as mother federation towards petitioner union.
"Incumbent affiliates of existing federations or national unions may Employees' grievances were allegedly left unattended to by respondent
disaffiliate only for the purpose of joining a federation or national union in federation to the detriment of the employees' rights and interests.
the industry or region in which it properly belongs or for the purpose of
operating as an independent labor group." In reversing the Med-Arbiter's resolution, respondent Bureau declared: the
Department of Labor is set on a task to restructure the labor movement to the
Accordingly, a resolution was adopted and signed by petitioner's members end that the workers will unite themselves along industry lines. Carried to its
revoking their check-off authorization in favor of ALUMETAL and notices complete fruition, only one union for every industry will remain to bargain
thereof were served on ALUMETAL and respondent companies. collectively for the workers. The clear policy therefore even now is to conjoin
workers and worker groups, not to dismember them. 5 This policy is
Confronted with the predicament of whether or not to continue deducting commendable. However, we must not lose sight of the constitutional mandate
from employees' wages and remitting union dues to respondent ALUMETAL of protecting labor and the workers' right to self-organization. In the
which wrote respondent companies advising them to continue deducting implementation and interpretation of the provisions of the Labor Code and
union dues and remitting them to said federation, respondent companies its implementing regulations, the workingman's welfare should be the
sought the legal opinion of the respondent Bureau as regards the controversy primordial and paramount consideration. In the case at bar, it would go
between the two unions. On November 11, 1976, Med-Arbiter George A. against the spirit of the labor law to restrict petitioner's right to self-
Eduvalla of respondent Bureau rendered a Resolution which in effect found organization due to the existence of the CBA. We agree with the Med-
the disaffiliation legal but at the same time gave the opinion that petitioner's Arbiter's opinion that "A disaffiliation does not disturb the enforceability and
members should continue paying their dues to ALUMETAL in the concept administration of a collective agreement; it does not occasion a change of
of agency fees. 1 administrators of the contract nor even an amendment of the provisions
thereof." 6 But nowhere in the record does it appear that the contract entered
From the said Resolution of the Med-Arbiter both petitioner and respondent into by the petitioner and ALUMETAL prohibits the withdrawal of the
ALUMETAL appealed to the Director of respondent Bureau. Petitioner former from the latter.
contended that the Med-Arbiter's opinion to the effect that petitioner's
members remained obligated to pay dues to respondent ALUMETAL was This now brings us to the second issue. Under Section 3, Article I, of the
inconsistent with the dispositive finding that petitioner's disaffiliation from CBA, the obligation of the respondent companies to deduct and remit dues
ALUMETAL was valid. ALUMETAL, on the other hand, assailed the to ALUMETAL is conditioned on the individual check-off authorization of
Resolution in question asserting that the disaffiliation should have been petitioner's members. In other words, ALUMETAL is entitled to receive the
declared contrary to law. dues from respondent companies as long as petitioner union is affiliated with
it and respondent companies are authorized by their employees (members of
petitioner union) to deduct union dues. Without said affiliation, the employer
has no link to the mother union. The obligation of an employee to pay union
dues is coterminous with his affiliation or membership. "The employees'
check-off authorization, even if declared irrevocable, is good only as long as
they remain members of the union concerned." 7 A contract between an
employer and the parent organization as bargaining agent for the employees
is terminated by the disaffiliation of the local of which the employees are
members. 8 Respondent companies therefore were wrong in continuing the
check-off in favor of respondent federation since they were duly notified of
the disaffiliation and of petitioner's members having already rescinded their
check-off authorization.

With the view we take on those two issues, we find no necessity in dwelling
further on the last issue. Suffice it to state that respondent federation is not
entitled to union dues payments from petitioner's members. "A local union
which has validly withdrawn from its affiliation with the parent association
and which continues to represent the employees of an employer is entitled to
the check-off dues under a collective bargaining contract." 9

WHEREFORE, the Resolutions of the Bureau of Labor Relations of January


25, 1977 and March 14, 1977 are REVERSED and SET ASIDE. Respondent
ALUMETAL is ordered to return to petitioner union all the union dues
enforced and collected through the NLRC Sheriff by virtue of the writ of
execution dated April 4, 1977 issued by respondent Bureau.

No costs.

SO ORDERED.
DEL MONTE PHILIPPINES, INC. VS. SALDIVAR
G.R. No. 158620 | 2006-10-11 Only Del Monte interposed an appeal with the NLRC.[13] The NLRC
Tinga, J.: reversed the Labor Arbiter and ruled that all the complainants were validly
dismissed.[14] On review, the Court of Appeals ruled that only Timbal was
The main issue for resolution herein is whether there was sufficient cause for illegally dismissed.[15] At the same time, the appellate court found that Del
the dismissal of a rank-and-file employee effectuated through the Monte had failed to observe procedural due process in dismissing the co-
enforcement of a closed-shop provision in the Collective Bargaining employees, and thus ordered the company to pay P30,000.00 to each of the
Agreement (CBA) between the employer and the union. co-employees as penalties. The co-employees sought to file a Petition for
Review[16] with this Court assailing the ruling of the Court of Appeals
The operative facts are uncomplicated. affirming their dismissal, but the petition was denied because it was not
timely filed.[17]
The Associated Labor Union (ALU) is the exclusive bargaining agent of
plantation workers of petitioner Del Monte Philippines, Inc. (Del Monte) in On the other hand, Del Monte, through the instant petition, assails the Court
Bukidnon. Respondent Nena Timbal (Timbal), as a rank-and-file employee of Appeals decision insofar as it ruled that Timbal was illegally dismissed.
of Del Monte plantation in Bukidnon, is also a member of ALU. Del Monte Notably, Del Monte does not assail in this petition the award of P30,000.00
and ALU entered into a Collective Bargaining Agreement (CBA) with an to each of the co-employees, and the ruling of the Court of Appeals in that
effective term of five (5) years from 1 September 1988 to 31 August 1993.[1] regard should now be considered final.

Timbal, along with four other employees (collectively, co-employees), were The reason offered by the Court of Appeals in exculpating Timbal revolves
charged by ALU for disloyalty to the union, particularly for encouraging around the problematic relationship between her and Artajo, the complaining
defections to a rival union, the National Federation of Labor (NFL). The witness against her. As explained by the appellate court:
charge was contained in a Complaint dated 25 March 1993, which
specifically alleged, in relation to Timbal: "That on July 13, 1991 and the However, the NLRC should have considered in a different light the situation
period prior or after thereto, said Nena Timbal personally recruited other of petitioner Nena Timbal. Timbal asserted before the NLRC, and reiterates
bonafide members of the ALU to attend NFL seminars and has actually in this petition, that the statements of Gemma Artajo, ALU's sole witness
attended these seminars together with the other ALU members."[2] The against her, should not be given weight because Artajo had an ax[e] to grind
matter was referred to a body within the ALU organization, ominously at the time when she made the adverse statements against her. Respondents
named "Disloyalty Board." never disputed the claim of Timbal that in the two (2) collection suits initiated
by Timbal and her husband, Artajo testified for the defendant in the first case
The charge against Timbal was supported by an affidavit executed on 23 and she was even the defendant in the second case which was won by Timbal.
March 1993 by Gemma Artajo (Artajo), also an employee of Del Monte. We find it hard to believe that Timbal would so willingly render herself
Artajo alleged that she was personally informed by Timbal on 13 July 1991 vulnerable to expulsion from the Union by revealing to an estranged
that a seminar was to be conducted by the NFL on the following day. When colleague her desire to shift loyalty. The strained relationship between
Artajo demurred from attending, Timbal assured her that she would be given Timbal and Artajo renders doubtful the charge against the former that she
honorarium in the amount of P500.00 if she were to attend the NFL meeting attempted to recruit Artajo to join a rival union. Inasmuch as the respondents
and bring new recruits. Artajo admitted having attended the NFL meeting failed to justify the termination of Timbal's employment, We hold that her
together with her own recruits, including Paz Piquero (Piquero). Artajo stated reinstatement to her former position in accordance with the September 27,
that after the meeting she was given P500.00 by Timbal.[3] 1996 decision of the Labor Arbiter is appropriate.[18]

Timbal filed an Answer before the Disloyalty Board, denying the allegations The Labor Arbiter, in his favorable ruling to the dismissed employees, had
in the complaint and the averments in Artajo's Affidavit. She further alleged noted that "complainant Timbal['s] x x x accuser has an axe to grind against
that her husband, Modesto Timbal, had filed a complaint against Artajo for her for an unpaid debt so that her testimony cannot be given credit."[19] The
collection of a sum of money on 17 March 1993, or just six (6) days before NLRC, in reversing the Labor Arbiter, did not see it fit to mention the
Artajo executed her affidavit. She noted that the allegations against her were circumstances of the apparent feud between Timbal and Artajo, except in the
purportedly committed nearly two (2) years earlier, and that Artajo's act was course of narrating Timbal's allegations.
motivated by hate and revenge owing to the filing of the aforementioned civil
action.[4] However, in the present petition, Del Monte utilizes a new line of argument
in justifying Timbal's dismissal. While it does not refute the
Nevertheless, the ALU Disloyalty Board concluded that Timbal was guilty contemporaneous ill-will between Timbal and Artajo, it nonetheless alleges
of acts or conduct inimical to the interests of ALU, through a Resolution that there was a second witness, Paz Piquero, who testified against Timbal
dated 7 May 1993.[5] It found that the acts imputed to Timbal were partisan before the Disloyalty Board.[20] Piquero had allegedly corroborated Artajo's
activities, prohibited since the "freedom period" had not yet commenced as allegations and positively identified Timbal as among those present during
of that time. Thus, the Disloyalty Board recommended the expulsion of the seminar of the NFL conducted on 14 July 1992 and as having given her
Timbal from membership in ALU, and likewise her dismissal from Del transportation money after the seminar was finished. Del Monte asserts that
Monte in accordance with the Union Security Clause in the existing CBA Piquero was a disinterested witness against Timbal.[21]
between ALU and Del Monte. The Disloyalty Board also reached the same
conclusions as to the co-employees, expressed in separate resolutions also Del Monte also submits two (2) other grounds for review. It argues that the
recommending their expulsion from ALU.[6] decision of the Labor Arbiter, which awarded Timbal full backwages and
other allowances, was inconsistent with jurisprudence which held that an
On 21 May 1993, the Regional Vice President of ALU adopted the employer who acted in good faith in dismissing employees on the basis of a
recommendations of the Disloyalty Board and expelled Timbal[7] and her closed-shop provision is not liable to pay full backwages.[22] Finally, Del
co-employees from ALU.[8] The ALU National President affirmed the Monte asserts that it had, from the incipience of these proceedings
expulsion.[9] On 17 June 1993, Del Monte terminated Timbal and her co- consistently prayed that in the event that it were found with finality that the
employees effective 19 June 1993, noting that the termination was "upon dismissal of Timbal and the others is illegal, ALU should be made liable to
demand of [ALU] pursuant to Sections 4 and 5 of Article III of the current Del Monte pursuant to the CBA. The Court of Appeals is faulted for failing
Collective Bargaining Agreement."[10] to rule upon such claim.

Timbal and her co-employees filed separate complaints against Del Monte For her part, Timbal observes that Piquero's name was mentioned for the first
and/or its Personnel Manager Warfredo C. Balandra and ALU with the time in Del Monte's Motion for Partial Reconsideration of the decision of the
Regional Arbitration Branch (RAB) of the National Labor Relations Court of Appeals.[23] She claims that both Piquero and Artajo were not in
Commission (NLRC) for illegal dismissal, unfair labor practice and good terms with her after she had won a civil suit for the collection of a sum
damages.[11] The complaints were consolidated and heard before Labor of money against their immediate superior, one Virgie Condeza.[24]
Arbiter Irving Pedilla. The Labor Arbiter affirmed that all five (5) were
illegally dismissed and ordered Del Monte to reinstate complainants, The legality of Timbal's dismissal is obviously the key issue in this case. We
including Timbal, to their former positions and to pay their full backwages are particularly called upon to determine whether at this late stage, the Court
and other allowances, though the other claims and charges were dismissed may still give credence to the purported testimony of Piquero and justify
for want of basis.[12] Timbal's dismissal based on such testimony.
recognize is that all workers, whether union members or not, are "entitled to
It bears elaboration that Timbal's dismissal is not predicated on any of the security of tenure."[32] The guarantee of security of tenure itself is
just or authorized causes for dismissal under Book Six, Title I of the Labor implemented through legislation, which lays down the proper standards in
Code,[25] but on the union security clause in the CBA between Del Monte determining whether such right was violated.[33]
and ALU. Stipulations in the CBA authorizing the dismissal of employees
are of equal import as the statutory provisions on dismissal under the Labor Agabon v. NLRC[34] did qualify that constitutional due process or security
Code, since "[a] CBA is the law between the company and the union and of tenure did not shield from dismissal an employee found guilty of a just
compliance therewith is mandated by the express policy to give protection to cause for termination even if the employer failed to render the statutory
labor."[26] The CBA, which covers all regular hourly paid employees at the notice and hearing requirement. At the same time, it should be understood
pineapple plantation in Bukidnon,[27] stipulates that all present and that in the matter of determining whether cause exists for termination,
subsequent employees shall be required to become a member of ALU as a whether under Book Six, Title I of the Labor Code or under a valid CBA,
condition of continued employment. Sections 4 and 5, Article II of the CBA substantive due process must be observed as a means of ensuring that security
further state: of tenure is not infringed.

ARTICLE II Agabon observed that due process under the Labor Code comprised of two
aspects: "substantive, i.e., the valid and authorized causes of employment
Section 4. Loss of membership in the UNION shall not be a ground for termination under the Labor Code; and procedural, i.e., the manner of
dismissal by the Company except where loss of membership is due to: dismissal."[35] No serious dispute arose in Agabon over the observance of
substantive due process in that case, or with the conclusion that the
1. Voluntary resignation from [ALU] earlier than the expiry date of this petitioners therein were guilty of abandonment of work, one of the just causes
[CBA]; for dismissal under the Labor Code. The controversy in Agabon centered on
whether the failure to observe procedural due process, through the non-
2. Non-payment of duly approved and ratified union dues and fees; and observance of the two-notice rule, should lead to the invalidation of the
dismissals. The Court ruled, over the dissents of some Justices, that the
3. Disloyalty to [ALU] in accordance with its Constitution and By-Laws as failure by the employer to observe procedural due process did not invalidate
duly registered with the Department of Labor and Employment. the dismissals for just cause of the petitioners therein. However, Agabon did
not do away with the requirement of substantive due process, which is
Section 5. Upon request of [ALU], [Del Monte] shall dismiss from its service essentially the existence of just cause provided by law for a valid dismissal.
in accordance with law, any member of the bargaining unit who loses his Thus, Agabon cannot be invoked to validate a dismissal wherein substantive
membership in [ALU] pursuant to the provisions of the preceding section. due process, or the proper determination of just cause, was not observed.
[ALU] assumes full responsibility for any such termination and hereby
agrees to hold [Del Monte] free from any liability by judgment of a Even if the dismissal of an employee is conditioned not on the grounds for
competent authority for claims arising out of dismissals made upon demand termination under the Labor Code, but pursuant to the provisions of a CBA,
of [ALU], and [the] latter shall reimburse the former of such sums as it shall it still is necessary to observe substantive due process in order to validate the
have paid therefor. Such reimbursement shall be deducted from union dues dismissal. As applied to the Labor Code, adherence to substantive due
and agency fees until duly paid.[28] process is a requisite for a valid determination that just or authorized causes
existed to justify the dismissal.[36] As applied to the dismissals grounded on
The CBA obviously adopts a closed-shop policy which mandates, as a violations of the CBA, observance of substantial due process is indispensable
condition of employment, membership in the exclusive bargaining agent. A in establishing the presence of the cause or causes for dismissal as provided
"closed-shop" may be defined as an enterprise in which, by agreement for in the CBA.
between the employer and his employees or their representatives, no person
may be employed in any or certain agreed departments of the enterprise Substantive due process, as it applies to all forms of dismissals, encompasses
unless he or she is, becomes, and, for the duration of the agreement, remains the proper presentation and appreciation of evidence to establish that cause
a member in good standing of a union entirely comprised of or of which the under law exists for the dismissal of an employee. This holds true even if the
employees in interest are a part.[29] A CBA provision for a closed-shop is a dismissal is predicated on particular causes for dismissal established not by
valid form of union security and it is not a restriction on the right or freedom the Labor Code, but by the CBA. Further, in order that any CBA-mandated
of association guaranteed by the Constitution.[30] dismissal may receive the warrant of the courts and labor tribunals, the causes
for dismissal as provided for in the CBA must satisfy to the evidentiary
Timbal's expulsion from ALU was premised on the ground of disloyalty to threshold of the NLRC and the courts.
the union, which under Section 4(3), Article II of the CBA, also stands as a
ground for her dismissal from Del Monte. Indeed, Section 5, Article II of the It is necessary to emphasize these principles since the immutable truth under
CBA enjoins Del Monte to dismiss from employment those employees our constitutional and labor laws is that no employee can be dismissed
expelled from ALU for disloyalty, albeit with the qualification "in without cause. Agabon may have tempered the procedural due process
accordance with law." requirements if just cause for dismissal existed, but in no way did it eliminate
the existence of a legally prescribed cause as a requisite for any dismissal.
Article 279 of the Labor Code ordains that "in cases of regular employment, The fact that a CBA may provide for additional grounds for dismissal other
the employer shall not terminate the services of an employee except for a just than those established under the Labor Code does not detract from the
cause or when authorized by [Title I, Book Six of the Labor Code]." necessity to duly establish the existence of such grounds before the dismissal
Admittedly, the enforcement of a closed-shop or union security provision in may be validated. And even if the employer or, in this case, the collective
the CBA as a ground for termination finds no extension within any of the bargaining agent, is satisfied that cause has been established to warrant the
provisions under Title I, Book Six of the Labor Code. Yet jurisprudence has dismissal, such satisfaction will be of no consequence if, upon legal
consistently recognized, thus: "It is State policy to promote unionism to challenge, they are unable to establish before the NLRC or the courts the
enable workers to negotiate with management on an even playing field and presence of such causes.
with more persuasiveness than if they were to individually and separately
bargain with the employer. For this reason, the law has allowed stipulations In the matter at bar, the Labor Arbiter-the proximate trier of facts-and the
for 'union shop' and 'closed shop' as means of encouraging workers to join Court of Appeals both duly appreciated that the testimony of Artajo against
and support the union of their choice in the protection of their rights and Timbal could not be given credence, especially in proving Timbal's
interests vis-a-vis the employer."[31] disloyalty to ALU. This is due to the prior animosity between the two
engendered by the pending civil complaint filed by Timbal's husband against
It might be suggested that since Timbal was expelled from ALU on the Artajo. Considering that the civil complaint was filed just six (6) days prior
ground of disloyalty, Del Monte had no choice but to implement the CBA to the execution of Artajo's affidavit against Timbal, it would be plainly
provisions and cause her dismissal. Similarly, it might be posited that any injudicious to presume that Artajo possessed an unbiased state of mind as she
tribunal reviewing such dismissal is precluded from looking beyond the executed that affidavit. Such circumstance was considered by the Labor
provisions of the CBA in ascertaining whether such dismissal was valid. Yet Arbiter, and especially the Court of Appeals, as they rendered a favorable
deciding the problem from such a closed perspective would virtually ruling to Timbal. The NLRC may have decided against Artajo, but in doing
guarantee unmitigated discretion on the part of the union in terminating the so, it failed to provide any basis as to why Artajo's testimony should be
employment status of an individual employee. What the Constitution does believed, instead of disbelieved. No credible disputation was offered by the
NLRC to the claim that Artajo was biased against Timbal; hence, we should Timbal was engaged in recruitment of ALU members at [Del Monte] to
adjudge the findings of the Labor Arbiter and the Court of Appeals as more attend NFL seminars."[39]
cogent on that point.
The Disloyalty Board may have appreciated Piquero's testimony in its own
Before this Court, Del Monte does not even present any serious argument finding that Timbal was guilty, yet the said board cannot be considered as a
that Artajo's testimony against Timbal was free from prejudice. Instead, it wholly neutral or dispassionate tribunal since it was constituted by the very
posits that Piquero's alleged testimony against Timbal before the Disloyalty organization that stood as the offended party in the disloyalty charge.
Board should be given credence, and that taken with Artajo's testimony, Without impugning the integrity of ALU and the mechanisms it has
should sufficiently establish the ground of disloyalty for which Timbal employed for the internal discipline of its members, we nonetheless hold that
should be dismissed. in order that the dismissal of an employee may be validated by this Court, it
is necessary that the grounds for dismissal are justified by substantial
The Court sees the danger to jurisprudence and the rights of workers in evidence as duly appreciated by an impartial trier of facts.[40] The existence
acceding to Del Monte's position. The dismissal for cause of employees must of Piquero's testimony was appreciated only by the Disloyalty Board, but not
be justified by substantial evidence, as appreciated by an impartial trier of by any of the impartial tribunals which heard Timbal's case. The appreciation
facts. None of the trier of facts below-the Labor Arbiter, the NLRC and the of such testimony by the Disloyalty Board without any similar affirmation or
Court of Appeals-saw fit to accord credence to Piquero's testimony, even concurrence by the NLRC-RAB, the NLRC, or the Court of Appeals, cannot
assuming that such testimony was properly contained in the record. Even the satisfy the substantive due process requirement as a means of upholding
NLRC decision, which was adverse to Timbal, made no reference at all to Timbal's dismissal.
Piquero's alleged testimony.
All told, we see no error on the part of the Court of Appeals when it held that
Del Monte is able to point to only one instance wherein Piquero's name and Timbal was illegally dismissed.
testimony appears on the record. It appears that among the several
attachments to the position paper submitted by the ALU before the NLRC- We now turn to the second issue raised, whether the Labor Arbiter correctly
RAB was a copy of the raw stenographic notes transcribed, apparently on 17 awarded full backwages to Timbal.
April 1993, during a hearing before the Disloyalty Board. The transcription
is not wholly legible, but there appears to be references therein to the name Del Monte cites a jurisprudential rule that an employer who acted in good
"Paz Piquero," and her apparent testimony before the Disloyalty Board. We faith in dismissing employees on the basis of a closed- shop provision may
are unable to reproduce with accuracy, based on the handwritten not be penalized even if the dismissal were illegal. Such a doctrine is
stenographic notes, the contents of this seeming testimony of Piquero, admittedly supported by the early case of National Labor Union v. Zip
although Del Monte claims before this Court that Piquero had corroborated Venetian Blind[41] and the later decision in 1989 of Soriano v. Atienza,[42]
Artajo's claims during such testimony, "positively identified [Timbal's] wherein the Court affirmed the disallowance of backwages or "financial
presence in the NFL seminar on 14 July 1992," and "confirmed that Timbal assistance" in dismissals under the aforementioned circumstance.
gave Artajo P500.00 for recruiting participants in the NFL seminar."[37]
However, the Court now recognizes that this doctrine is inconsistent with
There are evident problems on our part, at this late stage, in appreciating these Article 279 of the Labor Code, as amended by Republic Act No. 6715, which
raw stenographic notes adverting to the purported testimony of Piquero, took effect just five (5) days after Soriano was promulgated. It is now
especially as a means of definitively concluding that Timbal was guilty of provided in the Labor Code that "[a]n employee who is unjustly dismissed
disloyalty. Certainly, these notes cannot be appreciated as entries in the from work shall be entitled to reinstatement without loss of seniority rights
official record, which are presumed prima facie evidence of the facts therein and other privileges and to his full backwages, inclusive of allowances, and
stated,[38] as such records can only be made by a public officer of the to his other benefits or their monetary equivalent computed from the time his
Philippines or by a person in the performance of a duty specially enjoined by compensation was withheld from him up to the time of his actual
law. These transcripts were not taken during a hearing conducted by any reinstatement." Thus, where reinstatement is adjudged, the award of
public office in the Philippines, but they were committed in the course of an backwages and other benefits continues beyond the date of the labor arbiter's
internal disciplinary mechanism devised by a privately organized labor decision ordering reinstatement and extends up to the time said order of
union. Unless the authenticity of these notes is duly proven before, and reinstatement is actually carried out.[43]
appreciated by the triers of fact, we cannot accord them any presumptive or
conclusive value. Rep. Act No. 6715 effectively mitigated previous jurisprudence which had
limited the extent to which illegally dismissed employees could claim for
Moreover, despite the fact that the apparent record of Piquero's testimony backwages. We explained in Ferrer v. NLRC:[44]
was appended to ALU's position paper, the position paper itself does not
make any reference to such testimony, or even to Piquero's name for that With the passage of Republic Act No. 6715 which took effect on March 21,
matter. The position paper observes that "[t]his testimony of [Artajo] was 1989, Article 279 of the Labor Code was amended to read as follows:
directly corroborated by her actual attendance on July 14, 1992 at the agreed
[venue]," but no mention is made that such testimony was also "directly Security of Tenure. - In cases of regular employment, the employer shall not
corroborated" by Piquero. Then again, it was only Artajo, and not Piquero, terminate the services of an employee except for a just cause or when
who executed an affidavit recounting the allegations against Timbal. authorized by this Title. An employee who is unjustly dismissed from work
shall be entitled to reinstatement without loss of seniority rights and other
Indeed, we are inclined to agree with Timbal's observation in her Comment privileges and to his full backwages, inclusive of allowances, and to his other
on the present petition that from the time the complaint was filed with the benefits or their monetary equivalent computed from the time his
NLRC-RAB, Piquero's name and testimony were invoked for the first time compensation was withheld from him up to the time of his actual
only in Del Monte's motion for reconsideration before the Court of Appeals. reinstatement.
Other than the handwritten reference made in the raw stenographic notes
attached to ALU's position paper before the NLRC-RAB, Piquero's name or and as implemented by Section 3, Rule 8 of the 1990 New Rules of Procedure
testimony was not mentioned either by ALU or Del Monte before any of the of the National Labor Relations Commission, it would seem that the Mercury
pleadings filed before the NLRC-RAB, the NLRC, and even with those Drug Rule (Mercury Drug Co., Inc. vs. Court of Industrial Relations, 56
submitted to the Court of Appeals prior to that court's decision. SCRA 694 [1974]) which limited the award of back wages of illegally
dismissed workers to three (3) years "without deduction or qualification" to
In order for the Court to be able to appreciate Piquero's testimony as basis obviate the need for further proceedings in the course of execution, is no
for finding Timbal guilty of disloyalty, it is necessary that the fact of such longer applicable.
testimony must have been duly established before the NLRC-RAB, the
NLRC, or at the very least, even before the Court of Appeals. It is only after A legally dismissed employee may now be paid his back wages, allowances,
the fact of such testimony has been established that the triers of fact can come and other benefits for the entire period he was out of work subject to the rule
to any conclusion as to the veracity of the allegations in the testimony. enunciated before the Mercury Drug Rule, which is that the employer may,
however, deduct any amount which the employee may have earned during
It should be mentioned that the Disloyalty Board, in its Resolution finding the period of his illegal termination (East Asiatic Company, Ltd. vs. Court of
Timbal guilty of disloyalty, did mention that Artajo's testimony "was Industrial Relations, 40 SCRA 521 [1971]). Computation of full back wages
corroborated by Paz Piquero who positively identified and testified that Nena and presentation of proof as to income earned elsewhere by the illegally
dismissed employee after his termination and before actual reinstatement limited only to those arising from statutes or contracts other than a Collective
should be ventilated in the execution proceedings before the Labor Arbiter Bargaining Agreement. The Voluntary Arbitrator or Panel of Voluntary
concordant with Section 3, Rule 8 of the 1990 New Rules of Procedure of Arbitrators will have original and exclusive jurisdiction over money claims
the National Labor Relations Commission. 'arising from the interpretation or implementation of the Collective
Bargaining Agreement and, those arising from the interpretation or
Inasmuch as we have ascertained in the text of this discourse that the OFC enforcement of company personnel policies', under Article 261."[51]
whimsically dismissed petitioners without proper hearing and has thus
opened OFC to a charge of unfair labor practice, it ineluctably follows that Our conclusion that the Labor Arbiter in the instant case could not properly
petitioners can receive their back wages computed from the moment their pass judgment on the cross-claim is further strengthened by the fact that Del
compensation was withheld after their dismissal in 1989 up to the date of Monte and ALU expressly recognized the jurisdiction of Voluntary
actual reinstatement. In such a scenario, the award of back wages can extend Arbitrators in the CBA. Section 2, Article XXXI of the CBA provides:
beyond the 3-year period fixed by the Mercury Drug Rule depending, of
course, on when the employer will reinstate the employees. Section 2. In the event a dispute arises concerning the application of, or
interpretation of this Agreement which cannot be settled pursuant to the
It may appear that Article 279 of the Labor Code, as amended by Republic [grievance procedure set forth in the] preceding Section, the dispute shall be
Act No. 6715, has made the employer bear a heavier burden than that submitted to an arbitrator agreed to by [Del Monte] and [ALU].
pronounced in the Mercury Drug Rule, but perhaps Republic Act No. 6715
was enacted precisely for the employer to realize that the employee must be Should the parties fail to agree on the arbitrator, the same shall be drawn by
immediately restored to his former position, and to impress the idea that lottery from a list of arbitrators furnished by the Bureau of Labor Relations
immediate reinstatement is tantamount to a cost-saving measure in terms of of the Department of Labor and Employment.
overhead expense plus incremental productivity to the company which lies
in the hands of the employer.[45] xxxx

The Labor Arbiter's ruling, which entitled Timbal to claim full backwages Thus, as the law indubitably precludes the Labor Arbiter from enforcing
and other allowances, "without qualifications and diminutions, computed money claims arising from the implementation of the CBA, the CBA herein
from the time [she was] illegally dismisse[d] up to the time [she] will be complementarily recognizes that it is the Voluntary Arbitrators which have
actually reinstated," conforms to Article 279 of the Labor Code. Hence, the jurisdiction to hear the claim. The Labor Arbiter correctly refused to exercise
Court of Appeals was correct in affirming the Labor Arbiter insofar as jurisdiction over Del Monte's cross-claim, and the Court of Appeals would
Timbal was concerned. have no basis had it acted differently. At the same time, even as we affirm
the award of backwages against Del Monte, our ruling should not operate to
Finally, we address the claim that the Court of Appeals erred when it did not prejudice in any way whatever causes of action Del Monte may have against
rule on Del Monte's claim for reimbursement against ALU. We do observe ALU, in accordance with the CBA.
that Section 5 of the CBA stipulated that "[ALU] assumes full responsibility
of any such termination [of any member of the bargaining unit who loses his WHEREFORE, the instant petition is DENIED. The assailed Decision of the
membership in ALU] and hereby agrees to hold [Del Monte] free from any Court of Appeals dated 26 August 2002 is AFFIRMED. Costs against
liability by judgment of a competent authority for claims arising out of petitioner.
dismissals made upon demand of [ALU], and latter shall reimburse the
former of such sums as it shall have paid therefore."[46] SO ORDERED.

This stipulation does present a cause of action in Del Monte's favor should it
be held financially liable for the dismissal of an employee by reason of
expulsion from ALU. Nothing in this decision should preclude the operation
of this provision in the CBA. At the same time, we are unable to agree with
Del Monte that the Court of Appeals, or this Court, can implement this
provision of the CBA and accordingly directly condemn ALU to answer for
the financial remuneration due Timbal.

Before the Labor Arbiter, Del Monte had presented its cross-claim against
ALU for reimbursement should it be made liable for illegal dismissal or
unfair labor practice, pursuant to the CBA. The Labor Arbiter had actually
passed upon this claim for reimbursement, stating that "[as] for the cross-
claims of respondent DMPI and Tabusuares against the respondent ALU-
TUCP, this Branch cannot validly entertain the same in the absence of
employer-employee relationship between the former and the latter."[47] We
have examined Article 217 of the Labor Code,[48] which sets forth the
original jurisdiction of the Labor Arbiters. Article 217(c) states:

Cases arising from the interpretation or implementation of collective


bargaining agreements and those arising from the interpretation or
enforcement of company personnel policies shall be disposed of by the Labor
Arbiter by referring the same to the grievance machinery and voluntary
arbitration as may be provided in said agreements. [Emphasis supplied.]

In contrast, Article 261 of the Labor Code indubitably vests on the Voluntary
Arbitrator or panel of Voluntary Arbitrators the "original and exclusive
jurisdiction to hear and decide all unresolved grievances arising from the
interpretation or implementation of the Collective Bargaining
Agreement."[49] Among those areas of conflict traditionally within the
jurisdiction of Voluntary Arbitrators are contract-interpretation and contract-
implementation,[50] the questions precisely involved in Del Monte's claim
seeking enforcement of the CBA provision mandating restitution by ALU
should the company be held financially liable for dismissals pursuant to the
union security clause.

In reconciling the grants of jurisdiction vested under Articles 261 and 217 of
the Labor Code, the Court has pronounced that "the original and exclusive
jurisdiction of the Labor Arbiter under Article 217(c) for money claims is
BANK OF THE PHILIPPINE ISLANDS vs. BPI EMPLOYEES strictly construed since it purportedly curtails the right of the absorbed
UNION-DAVAO CHAPTER-FEDERATION OF UNIONS IN BPI employees to abstain from joining labor organizations.[8]
UNIBANK
G.R. No. 164301 | 2011-10-19 Pursuant to our directive, the Union filed its Comment[9] on the Motion for
LEONARDO-DE CASTRO, J.: Reconsideration. In opposition to petitioner's arguments, the Union, in turn,
adverts to our discussion in the August 10, 2010 Decision regarding the
In the present incident, petitioner Bank of the Philippine Islands (BPI) moves voluntary nature of the merger between BPI and FEBTC, the lack of an
for reconsideration[1] of our Decision dated August 10, 2010, holding that express stipulation in the Articles of Merger regarding the transfer of
former employees of the Far East Bank and Trust Company (FEBTC) employment contracts to the surviving corporation, and the consensual nature
"absorbed" by BPI pursuant to the two banks' merger in 2000 were covered of employment contracts as valid bases for the conclusion that former
by the Union Shop Clause in the then existing collective bargaining FEBTC employees should be deemed new employees.[10] The Union
agreement (CBA)[2] of BPI with respondent BPI Employees Union-Davao argues that the creation of employment relations between former FEBTC
Chapter-Federation of Unions in BPI Unibank (the Union). employees and BPI (i.e., BPI's selection and engagement of former FEBTC
employees, its payment of their wages, power of dismissal and of control
To recall, the Union Shop Clause involved in this long standing controversy over the employees' conduct) occurred after the merger, or to be more
provided, thus: precise, after the Securities and Exchange Commission's (SEC) approval of
the merger.[11] The Union likewise points out that BPI failed to offer any
ARTICLE II counterargument to the Court's reasoning that:

xxxx The rationale for upholding the validity of union shop clauses in a CBA, even
if they impinge upon the individual employee's right or freedom of
Section 2. Union Shop - New employees falling within the bargaining unit association, is not to protect the union for the union's sake. Laws and
as defined in Article I of this Agreement, who may hereafter be regularly jurisprudence promote unionism and afford certain protections to the
employed by the Bank shall, within thirty (30) days after they become regular certified bargaining agent in a unionized company because a strong and
employees, join the Union as a condition of their continued employment. It effective union presumably benefits all employees in the bargaining unit
is understood that membership in good standing in the Union is a condition since such a union would be in a better position to demand improved benefits
of their continued employment with the Bank.[3] (Emphases supplied.) and conditions of work from the employer. x x x.

The bone of contention between the parties was whether or not the x x x Nonetheless, settled jurisprudence has already swung the balance in
"absorbed" FEBTC employees fell within the definition of "new employees" favor of unionism, in recognition that ultimately the individual employee will
under the Union Shop Clause, such that they may be required to join be benefited by that policy. In the hierarchy of constitutional values, this
respondent union and if they fail to do so, the Union may request BPI to Court has repeatedly held that the right to abstain from joining a labor
terminate their employment, as the Union in fact did in the present case. organization is subordinate to the policy of encouraging unionism as an
Needless to state, BPI refused to accede to the Union's request. Although instrument of social justice.[12]
BPI won the initial battle at the Voluntary Arbitrator level, BPI's position
was rejected by the Court of Appeals which ruled that the Voluntary While most of the arguments offered by BPI have already been thoroughly
Arbitrator's interpretation of the Union Shop Clause was at war with the spirit addressed in the August 10, 2010 Decision, we find that a qualification of
and rationale why the Labor Code allows the existence of such provision. On our ruling is in order only with respect to the interpretation of the provisions
review with this Court, we upheld the appellate court's ruling and disposed of the Articles of Merger and its implications on the former FEBTC
of the case as follows: employees' security of tenure.

WHEREFORE, the petition is hereby DENIED, and the Decision dated Taking a second look on this point, we have come to agree with Justice
September 30, 2003 of the Court of Appeals is AFFIRMED, subject to the Brion's view that it is more in keeping with the dictates of social justice and
thirty (30) day notice requirement imposed herein. Former FEBTC the State policy of according full protection to labor to deem employment
employees who opt not to become union members but who qualify for contracts as automatically assumed by the surviving corporation in a merger,
retirement shall receive their retirement benefits in accordance with law, the even in the absence of an express stipulation in the articles of merger or the
applicable retirement plan, or the CBA, as the case may be.[4] merger plan. In his dissenting opinion, Justice Brion reasoned that:

Notwithstanding our affirmation of the applicability of the Union Shop To my mind, due consideration of Section 80 of the Corporation Code, the
Clause to former FEBTC employees, for reasons already extensively constitutionally declared policies on work, labor and employment, and the
discussed in the August 10, 2010 Decision, even now BPI continues to specific FEBTC-BPI situation -- i.e., a merger with complete "body and soul"
protest the inclusion of said employees in the Union Shop Clause. transfer of all that FEBTC embodied and possessed and where both
participating banks were willing (albeit by deed, not by their written
In seeking the reversal of our August 10, 2010 Decision, petitioner insists agreement) to provide for the affected human resources by recognizing
that the parties to the CBA clearly intended to limit the application of the continuity of employment -- should point this Court to a declaration that in a
Union Shop Clause only to new employees who were hired as non-regular complete merger situation where there is total takeover by one corporation
employees but later attained regular status at some point after hiring. FEBTC over another and there is silence in the merger agreement on what the fate of
employees cannot be considered new employees as BPI merely stepped into the human resource complement shall be, the latter should not be left in legal
the shoes of FEBTC as an employer purely as a consequence of the limbo and should be properly provided for, by compelling the surviving
merger.[5] entity to absorb these employees. This is what Section 80 of the Corporation
Code commands, as the surviving corporation has the legal obligation to
Petitioner likewise relies heavily on the dissenting opinions of our respected assume all the obligations and liabilities of the merged constituent
colleagues, Associate Justices Antonio T. Carpio and Arturo D. Brion. From corporation.
both dissenting opinions, petitioner derives its contention that "the situation
of absorbed employees can be likened to old employees of BPI, insofar as Not to be forgotten is that the affected employees managed, operated and
their full tenure with FEBTC was recognized by BPI and their salaries were worked on the transferred assets and properties as their means of livelihood;
maintained and safeguarded from diminution" but such absorbed employees they constituted a basic component of their corporation during its existence.
"cannot and should not be treated in exactly the same way as old BPI In a merger and consolidation situation, they cannot be treated without
employees for there are substantial differences between them."[6] Although consideration of the applicable constitutional declarations and directives, or,
petitioner admits that there are similarities between absorbed and new worse, be simply disregarded. If they are so treated, it is up to this Court to
employees, they insist there are marked differences between them as well. read and interpret the law so that they are treated in accordance with the legal
Thus, adopting Justice Brion's stance, petitioner contends that the absorbed requirements of mergers and consolidation, read in light of the social justice,
FEBTC employees should be considered "a sui generis group of employees economic and social provisions of our Constitution. Hence, there is a need
whose classification will not be duplicated until BPI has another merger for the surviving corporation to take responsibility for the affected employees
where it would be the surviving corporation."[7] Apparently borrowing from and to absorb them into its workforce where no appropriate provision for the
Justice Carpio, petitioner propounds that the Union Shop Clause should be merged corporation's human resources component is made in the Merger
Plan.[13]
By upholding the automatic assumption of the non-surviving corporation's Indeed, there are differences between (a) new employees who are hired as
existing employment contracts by the surviving corporation in a merger, the probationary or temporary but later regularized, and (b) new employees who,
Court strengthens judicial protection of the right to security of tenure of by virtue of a merger, are absorbed from another company as regular and
employees affected by a merger and avoids confusion regarding the status of permanent from the beginning of their employment with the surviving
their various benefits which were among the chief objections of our corporation. It bears reiterating here that these differences are too
dissenting colleagues. However, nothing in this Resolution shall impair the insubstantial to warrant the exclusion of the absorbed employees from the
right of an employer to terminate the employment of the absorbed employees application of the Union Shop Clause. In the Decision, we noted that:
for a lawful or authorized cause or the right of such an employee to resign,
retire or otherwise sever his employment, whether before or after the merger, Verily, we agree with the Court of Appeals that there are no substantial
subject to existing contractual obligations. In this manner, Justice Brion's differences between a newly hired non-regular employee who was
theory of automatic assumption may be reconciled with the majority's regularized weeks or months after his hiring and a new employee who was
concerns with the successor employer's prerogative to choose its employees absorbed from another bank as a regular employee pursuant to a merger, for
and the prohibition against involuntary servitude. purposes of applying the Union Shop Clause. Both employees were
hired/employed only after the CBA was signed. At the time they are being
Notwithstanding this concession, we find no reason to reverse our previous required to join the Union, they are both already regular rank and file
pronouncement that the absorbed FEBTC employees are covered by the employees of BPI. They belong to the same bargaining unit being represented
Union Shop Clause. by the Union. They both enjoy benefits that the Union was able to secure for
them under the CBA. When they both entered the employ of BPI, the CBA
Even in our August 10, 2010 Decision, we already observed that the legal and the Union Shop Clause therein were already in effect and neither of them
fiction in the law on mergers (that the surviving corporation continues the had the opportunity to express their preference for unionism or not. We see
corporate existence of the non-surviving corporation) is mainly a tool to no cogent reason why the Union Shop Clause should not be applied equally
adjudicate the rights and obligations between and among the merged to these two types of new employees, for they are undeniably similarly
corporations and the persons that deal with them.[14] Such a legal fiction situated.[18]
cannot be unduly extended to an interpretation of a Union Shop Clause so as
to defeat its purpose under labor law. Hence, we stated in the Decision that: Again, it is worthwhile to highlight that a contrary interpretation of the Union
Shop Clause would dilute its efficacy and put the certified union that is
In any event, it is of no moment that the former FEBTC employees retained supposedly being protected thereby at the mercy of management. For if the
the regular status that they possessed while working for their former former FEBTC employees had no say in the merger of its former employer
employer upon their absorption by petitioner. This fact would not remove with another bank, as petitioner BPI repeatedly decries on their behalf, the
them from the scope of the phrase "new employees" as contemplated in the Union likewise could not prevent BPI from proceeding with the merger
Union Shop Clause of the CBA, contrary to petitioner's insistence that the which undisputedly affected the number of employees in the bargaining unit
term "new employees" only refers to those who are initially hired as non- that the Union represents and may negatively impact on the Union's majority
regular employees for possible regular employment. status. In this instance, we should be guided by the principle that courts must
place a practical and realistic construction upon a CBA, giving due
The Union Shop Clause in the CBA simply states that "new employees" who consideration to the context in which it is negotiated and purpose which it is
during the effectivity of the CBA "may be regularly employed" by the Bank intended to serve.[19]
must join the union within thirty (30) days from their regularization. There is
nothing in the said clause that limits its application to only new employees We now come to the question: Does our affirmance of our ruling that former
who possess non-regular status, meaning probationary status, at the start of FEBTC employees absorbed by BPI are covered by the Union Shop Clause
their employment. Petitioner likewise failed to point to any provision in the violate their right to security of tenure which we expressly upheld in this
CBA expressly excluding from the Union Shop Clause new employees who Resolution? We answer in the negative.
are "absorbed" as regular employees from the beginning of their
employment. What is indubitable from the Union Shop Clause is that upon In Rance v. National Labor Relations Commission,[20] we held that:
the effectivity of the CBA, petitioner's new regular employees (regardless of
the manner by which they became employees of BPI) are required to join the It is the policy of the state to assure the right of workers to "security of tenure"
Union as a condition of their continued employment.[15] (Article XIII, Sec. 3 of the New Constitution, Section 9, Article II of the 1973
Constitution). The guarantee is an act of social justice. When a person has no
Although by virtue of the merger BPI steps into the shoes of FEBTC as a property, his job may possibly be his only possession or means of livelihood.
successor employer as if the former had been the employer of the latter's Therefore, he should be protected against any arbitrary deprivation of his job.
employees from the beginning it must be emphasized that, in reality, the legal Article 280 of the Labor Code has construed security of tenure as meaning
consequences of the merger only occur at a specific date, i.e., upon its that "the employer shall not terminate the services of an employee except for
effectivity which is the date of approval of the merger by the SEC. Thus, we a just cause or when authorized by" the Code. x x x (Emphasis supplied.)
observed in the Decision that BPI and FEBTC stipulated in the Articles of
Merger that they will both continue their respective business operations until We have also previously held that the fundamental guarantee of security of
the SEC issues the certificate of merger and in the event no such certificate tenure and due process dictates that no worker shall be dismissed except for
is issued, they shall hold each other blameless for the non-consummation of a just and authorized cause provided by law and after due process is
the merger.[16]We likewise previously noted that BPI made its assignments observed.[21] Even as we now recognize the right to continuous, unbroken
of the former FEBTC employees effective on April 10, 2000, or after the employment of workers who are absorbed into a new company pursuant to a
SEC approved the merger.[17] In other words, the obligation of BPI to pay merger, it is but logical that their employment may be terminated for any
the salaries and benefits of the former FEBTC employees and its right of causes provided for under the law or in jurisprudence without violating their
discipline and control over them only arose with the effectivity of the merger. right to security of tenure. As Justice Carpio discussed in his dissenting
Concomitantly, the obligation of former FEBTC employees to render service opinion, it is well-settled that termination of employment by virtue of a union
to BPI and their right to receive benefits from the latter also arose upon the security clause embodied in a CBA is recognized in our jurisdiction.[22] In
effectivity of the merger. What is material is that all of these legal Del Monte Philippines, Inc. v. Saldivar,[23] we explained the rationale for
consequences of the merger took place during the life of an existing and valid this policy in this wise:
CBA between BPI and the Union wherein they have mutually consented to
include a Union Shop Clause. Article 279 of the Labor Code ordains that "in cases of regular employment,
the employer shall not terminate the services of an employee except for a just
From the plain, ordinary meaning of the terms of the Union Shop Clause, it cause or when authorized by [Title I, Book Six of the Labor Code]."
covers employees who (a) enter the employ of BPI during the term of the Admittedly, the enforcement of a closed-shop or union security provision in
CBA; (b) are part of the bargaining unit (defined in the CBA as comprised the CBA as a ground for termination finds no extension within any of the
of BPI's rank and file employees); and (c) become regular employees without provisions under Title I, Book Six of the Labor Code. Yet jurisprudence has
distinguishing as to the manner they acquire their regular status. consistently recognized, thus: "It is State policy to promote unionism to
Consequently, the number of such employees may adversely affect the enable workers to negotiate with management on an even playing field and
majority status of the Union and even its existence itself, as already amply with more persuasiveness than if they were to individually and separately
explained in the Decision. bargain with the employer. For this reason, the law has allowed stipulations
for 'union shop' and 'closed shop' as means of encouraging workers to join
and support the union of their choice in the protection of their rights and
interests vis-a-vis the employer."[24] (Emphasis supplied.)

Although it is accepted that non-compliance with a union security clause is a


valid ground for an employee's dismissal, jurisprudence dictates that such a
dismissal must still be done in accordance with due process. This much we
decreed in General Milling Corporation v. Casio,[25] to wit:

The Court reiterated in Malayang Samahan ng mga Manggagawa sa M.


Greenfield v. Ramos that:

While respondent company may validly dismiss the employees expelled by


the union for disloyalty under the union security clause of the collective
bargaining agreement upon the recommendation by the union, this dismissal
should not be done hastily and summarily thereby eroding the employees'
right to due process, self-organization and security of tenure. The
enforcement of union security clauses is authorized by law provided such
enforcement is not characterized by arbitrariness, and always with due
process. Even on the assumption that the federation had valid grounds to
expel the union officers, due process requires that these union officers be
accorded a separate hearing by respondent company.

The twin requirements of notice and hearing constitute the essential elements
of procedural due process. The law requires the employer to furnish the
employee sought to be dismissed with two written notices before termination
of employment can be legally effected:

(1) a written notice apprising the employee of the particular acts or omissions
for which his dismissal is sought in order to afford him an opportunity to be
heard and to defend himself with the assistance of counsel, if he desires, and

(2) a subsequent notice informing the employee of the employer's decision


to dismiss him. This procedure is mandatory and its absence taints the
dismissal with illegality.

Irrefragably, GMC cannot dispense with the requirements of notice and


hearing before dismissing Casio, et al. even when said dismissal is pursuant
to the closed shop provision in the CBA. The rights of an employee to be
informed of the charges against him and to reasonable opportunity to present
his side in a controversy with either the company or his own union are not
wiped away by a union security clause or a union shop clause in a collective
bargaining agreement. x x x[26] (Emphases supplied.)

In light of the foregoing, we find it appropriate to state that, apart from the
fresh thirty (30)-day period from notice of finality of the Decision given to
the affected FEBTC employees to join the Union before the latter can request
petitioner to terminate the former's employment, petitioner must still accord
said employees the twin requirements of notice and hearing on the possibility
that they may have other justifications for not joining the Union. Similar to
our August 10, 2010 Decision, we reiterate that our ruling presupposes there
has been no material change in the situation of the parties in the interim.

WHEREFORE, the Motion for Reconsideration is DENIED. The Decision


dated August 10, 2010 is AFFIRMED, subject to the qualifications that:

(a) Petitioner is deemed to have assumed the employment contracts of the


Far East Bank and Trust Company (FEBTC) employees upon effectivity of
the merger without break in the continuity of their employment, even without
express stipulation in the Articles of Merger; and

(b) Aside from the thirty (30) days, counted from notice of finality of the
August 10, 2010 Decision, given to former FEBTC employees to join the
respondent, said employees shall be accorded full procedural due process
before their employment may be terminated.

SO ORDERED.
UNITED POLYRESINS, INC. VS. PINUELA On January 7, 2008, respondent filed a complaint before the National
G.R. No. 209555 | 2017-07-31 Conciliation and Mediation Board (NCMB), claiming that petitioners refused
DEL CASTILLO, J.: to bargain collectively. During the scheduled conferences before the NCMB,
petitioners raised the issue of non-payment of the P300,000.00 owing to UPI
This Petition for Review on Certiorari[1] assails the December 11, 2012 and insisted on its payment; they also threatened to deduct the amount of
Decision[2] and October 10, 2013 Resolution[3] of the Court of Appeals P1,500.00 from the respective salaries of the union members.[11]
(CA) in CA-G.R. SP No. 115402 which set aside the June 11, 2011
Decision[4]of the National Labor Relations Commission (NLRC) in NLRC- Because of the recurring threat of failed CBA negotiations and salary
LAC Case No. 06- 001577-09. deductions as means of recovering the P300,000.00 loaned to the union,
union members began to demand the holding of a special election of union
Factual Antecedents officers. They likewise accused respondent and the other union officers of
mismanagement, unduly hanging on to their positions, and lack of
Petitioner United Polyresins, Inc. (UPI) is a registered domestic corporation accountability.[12]
doing business in San Pedro, Laguna, while petitioners Ernesto Uy Soon, Jr.
and Julito Uy Soon are its corporate officers. Thus, in March 2008, special elections were held, and a new union President
and set of officers were elected.[13]
Respondent Marcelino Pinuela was employed by UPI in 1987. He became a
member of the labor union, Polyresins Rank and File Association (PORFA), On March 29, 2008, the union's new set of officers conducted an
and was elected President thereof in May, 2005 and slated to serve until the investigation into the fact that the union had little or no funds remaining in
end of 2007. its bank account. Respondent attended the investigation, and admitted that
the union had no more funds as they were "utilized in the prosecution of cases
The collective bargaining agreement (CBA) then existing between UPI and during his incumbency."[14] He likewise failed to make a formal turnover of
PORFA provided that: documents to the new President. Respondent was required to surrender union
documents in his possession on the next scheduled meeting.[15]
Section 3. The Company shall grant to the Union the amount of Three
Hundred Thousand Pesos (P300,000.00) free of interest as the union's capital On April 8, 2008, another inquiry was held where respondent was present.
for establishing a cooperative to meet the needs of its members. Said loan The investigation centered on respondent's continued failure to account for
shall fall due and become payable at the same date that this Bargaining the union's bank accounts, documents, and deposits made during his
Agreement expires, to wit - December 31, 2007. In the event of non-payment, incumbency, and his failure to formally turn over union's papers to the new
all officers and members will be personally accountable. In case of additional officers. After the meeting, respondent and the new officers proceeded to the
funds, they can make a written request [addressed] to the President of the bank, where they discovered that the PORFA account had already been
company.[5] closed.[16]

The CBA likewise contained a union security clause which provided that On April 10, 2008, the new set of union officers issued a Resolution[17]
employees who cease to be PORFA members in good standing by reason of expelling respondent from PORFA for being guilty of the following
resignation or expulsion shall not be retained in the employ of UPI. violations:

Upon his assumption as union President, respondent wrote the former union 1. No annual financial statement.
President, Geoffrey Cielo (Cielo), to turn over the records, papers, documents 2. No listings or ledger of union member's [sic] emergency loans.
and financial statements of the union. Cielo surrendered the union's bank 3. Unposted cheques on the Union's passbook collected from union members
account documents, among others, which indicated that the union had an [sic] monthly dues.
available P78,723.60 cash balance. Cielo likewise submitted a Financial 4. Our union checking account at Security Bank were [sic] Zero
Report indicating that the union had P208,623.60 in cash and P159,500.00 in balance/closed account.
receivables. 5. No receipts/cash disbursement presented for the union operational [sic]
expenses.
Finding that the bank documents and Cielo's report did not match, and Cielo 6. Unable to return the P300,000.00 lent by the management free of interest.
unable to explain the discrepancies, the union's Executive Committee, which (Art. XXVII, Section 3 of our CBA).
was headed by respondent, resolved to hire a certified public accountant to 7. Unable to explain and present documents to support where the agency fees
conduct an audit of the union's finances. In a December 1, 2005 report, the and union dues collected from legitimate union members were used.[18]
accountant concluded that the union's finances, income, and disbursements
for the years 2003 and 2004 were not properly documented, recorded, and The officers held that these violations constituted an infringement of the
reported. He recommended that the union officers "take a seminar on basic union's Constitution, particularly Article XV, Section 1, paragraphs (e) and
bookkeeping and accounting;"[6] that the union adopt and/or install the (f) thereof, which specifically prohibit the misappropriation of union funds
necessary accounting and internal control systems; that the union prepare the and property and give ground for the impeachment and recall of union
proper financial statements; and that the officers take corrective measures in officers.[19]
financial management as an integral part of sound management.[7]
In an April 11, 2008 letter[20] to petitioners, PORFA communicated
Meanwhile, during respondent's term as PORFA President, it appeared that respondent's expulsion from the union.
UPI automatically deducted from the respective salaries of PORFA members
amounts representing union membership dues and loan payments. These On April 14, 2008, petitioners issued a letter of termination[21] to
amounts, which totalled P2,402,533.43, were then regularly turned over by respondent, to take effect immediately.
UPI to PORFA in the form of fifty eight (58) crossed checks, made payable
to PORFA.[8] These amounts were then deposited and credited to PORFA's Ruling of the Labor Arbiter
account.[9]
Respondent filed a complaint against petitioners before the Labor Arbiter for
On December 8, 2007, or several days before the P300,000.00 loan by UPI illegal dismissal, with monetary claims and damages, which was docketed as
to PORFA became due, petitioners, respondent, and the other union officers NLRC Case No. RAB-IV-08-27303-08-L. He claimed that his dismissal was
met to discuss the proposed new CBA. Thereat, petitioners told respondent effected in bad faith and without due process and was thus illegal. Petitioners
that until the P300,000.00 is returned, the former shall not discuss the countered that respondent's dismissal is valid under the union security clause
proposed CBA. Respondent explained that the union did not have the of the CBA; that his failure to return the P300,000.00 loan to the union due
finances and had only P78,723.60, which was the original amount turned to mismanagement/misappropriation constitutes just cause for his expulsion
over by Cielo to respondent when the latter assumed office as union from the union, as well as dismissal from employment; that he was accorded
President. Petitioners then told respondent and the other union officers that substantive and procedural due process; that the herein individual petitioners
if the amount is not returned, the same will be deducted from the salaries of may not be held liable for respondent's claims; and that accordingly, the case
the union members.[10] should be dismissed.
On April 20, 2009, the Labor Arbiter issued a Decision[22] dismissing What cannot escape from [sic] our attention and consideration are the
respondent's complaint on the finding that respondent was not illegally following:
terminated, thus:
(1) there was an obligation x x x to return the amount of P300,000.00 to the
While complainant, as then Union President, denies any misappropriation of respondent upon termination of the CBA on December 31, 2007,
union funds, it is undisputed that he failed to account for the missing union
funds and to return the P300,000.00 which the respondent company had lent (2) complainant, as the President of the Union at the time the loan was due
for the union's assistance upon the expiration of the CBA dated December and demandable, failed to account for said funds, and under the same
31, 2007. provision, was to be held personally accountable,

More importantly, in the investigation conducted by the newly elected (3) Pinuela actually participated x x x in the whole process of determining
officers of the union, it was uncovered that union funds were in fact accountability over the union funds,
personally used by the former officers of PORFA which includes
complainant. (4) denied knowledge over and receipt of the missing funds, despite his being
among those charged with its custody and safe-keep, as the Union President.
Thus, the union passed a resolution expelling complainant from the PORFA
union and the corresponding letter was sent to the respondent company It is also to be noted that the complainant as union president, could not
informing the latter of complainant's expulsion coupled with a explain nor comment on the fact that their union's bank account is already a
recommendation that complainant be terminated from employment pursuant closed account. Even if We assume and in fact complainant admitted that he
to the union security clause of the CBA. had custody of P78,723.60 as union funds as of June 3, 2005, still he could
not account the whereabouts of the said money. As a signatory to the said
Given the foregoing, we rule that complainant was validly dismissed since account, complainant cannot be considered as entirely faultless since he was
the respondent company merely did its obligation under the CBA by grossly negligent in the custody of the funds. There is substantial basis in
terminating the services of complainant who ceased to be a member in good complainant's dismissal thus, the award of backwages and 13th month pay
standing of the PORFA union by reason of expulsion. should be deleted. However, even if We find complainant's dismissal to be
valid, there is equally no evidence showing that he pocketed the missing
WHEREFORE, premises considered, judgment is hereby rendered funds of the union. In this regard since he had rendered a considerable
DISMISSING the instant complaint for lack of merit. number of years in the service (21 years) complainant may be awarded
separation pay at the rate of 1/2 month salary for every year of service (396
SO ORDERED.[23] x 13 x 21 years) from the inception of his employment till his dismissal in
the interest of justice and compassion since his infraction did not involve
Ruling of the National Labor Relations Commission serious misconduct.
Further, We also hold that while complainant's dismissal was valid pursuant
Respondent appealed before the NLRC, which initially overturned the Labor to the enforcement of the Union Security Clause, respondents however did
Arbiter in a December 8, 2009 Decision,[24] which decreed as follows: not comply with the requisite procedural due process. As held in the case of
Agabon vs. NLRC, x x x the Supreme Court held that where the dismissal is
WHEREFORE, the assailed Decision is hereby SET ASIDE and a NEW one for a cause recognized by the prevailing jurisprudence, the absence of the
is entered declaring the complainant-appellant's dismissal to be illegal. statutory due process should not nullify the dismissal or render it illegal x x
Respondents Union [sic] and respondent company are hereby declared x. Accordingly, for violating complainant's statutory rights, respondents
jointly and severally liable to pay complainant his full backwages from the should indemnify him the amount of P30,000.00 as nominal damages in
date he was dismissed until date instant [sic] and to pay his separation pay addition to his separation pay.
equivalent to one month salary per year of service computed as follows:
WHEREFORE, premises considered, respondents-appellees' Motion for
BACKWAGES Reconsideration is GRANTED, a new Decision is rendered finding
04/14/08 - 10/14/09 complainant's dismissal as valid. Respondents-appellees are however
P396 x 26 days x 18 mos. ordered to pay complainant the amounts of P108,108.00 and P30,000.00 as
P10,296.00 x 18 days separation pay and nominal damages.
=
P185,328.00 All other claims whether monetary or otherwise are hereby DISMISSED.

SO ORDERED.[26]

SEPARATION PAY Ruling of the Court of Appeals


P396.00 x 26 x 22yrs.
P10,296 x 22yrs. In a Petition for Certiorari[27] before the CA and docketed as CA-G.R. SP
= No. 115402, respondent sought to reverse the above NLRC Decision and
P226,512.00 reinstate its December 8, 2009 Decision, arguing that the Commission
gravely erred in concluding that he was personally accountable for the
missing funds, the closing of PORFA's bank account, and that he was grossly
negligent in the custody of the union funds. In their Comment,[28] petitioners
13th Month Pay countered that respondent's dismissal was attended by due process; that he is
P185,328.00 / 12 guilty of the infractions for which he was dismissed; and that his guilt had
= been proved by substantial evidence.
P15,444.00
On December 11, 2012, the CA issued the assailed Decision containing the
following pronouncement:

Grand Total Petitioner insists that he is innocent of the charges against him made by the
PORFA (the union), particularly the embezzlement of the union funds. He
P427,284.00 vehemently denied misappropriation of the same and that the PORFA Union
officers conspired with the Respondents in removing him as a member in
SO ORDERED.[25] good standing of the said union and his subsequent dismissal as employee
pursuant to the CBA's union security clause.
However, on motion for reconsideration, the NLRC issued its June 11, 2011
Decision, which held as follows: Respondents on the other hand, denied the Petitioner's allegation of
conspiracy and that in fact, there was a series of conferences conducted
jointly by the management and the union on the matter of lost union funds
and that the Petitioner was made aware of the charges against him before he Petitioners filed a Motion for Reconsideration,[31] which was denied by the
was terminated. They claim that the management participated in the CA in its October 10, 2013 Resolution. Hence, the instant Petition.
investigations and that it was shown that even if the Petitioner as president
of the union did not misappropriate the funds nevertheless he committed Issues
omission/gross negligence for which reason he was expelled therefrom. The
Respondents also claim that Petitioner was accorded procedural due process In a June 22, 2015 Resolution,[32] the Court resolved to give due course to
during the investigations. the Petition, which contains the following assignment of errors:

It is basic in labor jurisprudence that the burden of proof rests upon I.


management to show that the dismissal of its worker was based on a just
cause. When an employer exercises its power to terminate an employee by THE APPELLATE COURT ERRED IN RULING THAT THE CHARGES
enforcing the union security clause, it needs to determine and prove the OF MISAPPROPRIATION AGAINST THE RESPONDENT WERE
following: INSUFFICIENT (SUBSTANTIVE DUE PROCESS)

(1) the union security clause is applicable; II.


(2) the union is requesting for the enforcement of the union security provision
in the CBA; and THE APPELLATE COURT ERRED IN RULING THAT THE
(3) there is sufficient evidence to support the decision of the union to expel RESPONDENT WAS NOT PROPERLY INFORMED OF THE CHARGES
the employee from the union. AGAINST HIM (PROCEDURAL DUE PROCESS).

The dispute before Us does not raise any issue with respect to the first two III.
requisites; the issue being whether there was sufficient evidence to support
Petitioner's expulsion from PORFA. In arriving at any conclusion thereto, the THE APPELLATE COURT ERRED IN RULING THAT THE
Petitioner must first be accorded due process of law. x x x RESPONDENT IS ENTITLED TO SEPARATION PAY, BACKWAGES
FROM DISMISSAL TO THE FINALITY OF ITS DECISION, AND 13TH
xxxx MONTH PAY.[33]

On both questions of whether there exist[s] sufficient evidence to support Petitioners' Arguments
Petitioner's expulsion from the union (substantive due process), and whether
Petitioner was properly informed of the accusation against him and his Praying that the assailed CA dispositions be set aside and that respondent's
dismissal from employment (procedural due process), We answer in the case be dismissed instead, petitioners maintain in their Petition and
negative. Reply[34] that substantive and procedural due process were observed in
respondent's case; that respondent was apprised of the charges against him
An examination of the submitted evidence before the Labor Arbiter show and given the opportunity to refute them; that the evidence points to the
[sic] that the same are not enough to prove the alleged charges of conclusion that he misappropriated the union's funds and was unable to
misappropriation against the Petitioner and neither was he properly informed explain the dissipation thereof; that for what he has done, respondent violated
thereof. Article XV, Section 1, paragraphs (e) and (f) of the union's Constitution; that
respondent's dismissal on the basis of the union security clause in the CBA
xxxx was thus valid, based on substantial proof, and in accord with the
pronouncement in Cariño v. National Labor Relations Commission,[35]
On the other hand, the Petitioner have [sic] shown adequate explanation where the dismissal of an employee was upheld on the basis of the union
about the funds of the union that came to his possession. The Memorandum security and expulsion clauses contained in the CBA; and that since his
of Ramon M. Martinez, a Certified Public Accountant, show [sic] that he dismissal is valid, then he is not entitled to his monetary claims.
made an audit of the funds of the union during the previous administration
and that the actual funds the union had was merely P34,344.25 when Respondent's Arguments
Petitioner took over. This amount was not even shown to have been
misappropriated by the Petitioner. In his Comment,[36] respondent maintains that the CA did not err in finding
that the evidence against him was insufficient; that the CA was correct in
Compounding this want of substantive evidence is the lack of procedural due ruling that his right to procedural due process was violated when he was not
process that Petitioner was entitled to. As [has] been previously discussed, properly informed of the charges against him; and that for these reasons, he
the Petitioner was not given the proper first notice. Thereafter, despite such was illegally dismissed and thus entitled to his monetary claims.
lack of first notice, on the mere letter of the union that he was expelled
therefrom because of alleged causes, the Petitioner was dismissed from Our Ruling
employment by the Respondents in the termination letter dated 14 April 2008
on the sole basis of union security clause. Such action cannot be The Court denies the Petition.
countenanced. In the same Inguillo case, the Supreme Court also ruled:
Respondent's expulsion from PORFA is grounded on Article XV, Section 1,
'Thus, as held in that case, 'the right of an employee to be informed of the paragraphs (e) and (f) of the union's Constitution, which provides:
charges against him and to reasonable opportunity to present his side in a
controversy with either the company or his own Union is not wiped away by ARTICLE-XV
a Union Security Clause or a Union Shop Clause in a collective bargaining IMPEACHMENT AND RECALL
agreement. An employee is entitled to be protected not only from a company
which disregards his rights but also from his own Union, the leadership of Section 1. Any of the following shall be ground for the impeachment or recall
which could yield to the temptation of swift and arbitrary expulsion from of the union officers.
membership and mere dismissal from his job.'
a.
In sum, the NLRC gravely abused its discretion in reconsidering its earlier Committing or causing the commission directly or indirectly of acts against
Decision which is more in accord with the evidence on record. the interest and welfare of the union;
b.
WHEREFORE, the petition is hereby GRANTED. The assailed Decision Malicious attack against the union, its officers or against a fellow union
dated 11 June 2010[29] is hereby SET ASIDE. The Decision dated 8 officer or member;
December 2009 is REINSTATED with the MODIFICATION that the c.
backwages shall be recomputed from the date of Petitioner's dismissal to the Failure to comply with the obligation to turn over and return to union
finality of this Decision. treasurer within three (3) days unexpanded [sic] sum of money received from
the money funds to answer for an authorized union purpose;
SO ORDERED.[30] (Citations omitted) d.
Gross misconduct unbecoming of a union officer;
e usual terms and conditions under which membership or continuation of
Misappropriation of union funds and property. This is without prejudice to membership is made available to other members."[44]
the filing of an appropriate criminal or civil action against the responsible
officer/(s) by any interested party; On account of the foregoing disquisition, the other issues raised by the parties
f. need not be discussed.
Willful violation of any provision of the constitution or rules, regulations,
measures, resolution(s) and decision of the union.[37] (Emphasis supplied) WHEREFORE, for the foregoing reasons, the Petition is hereby DENIED.
The December 11, 2012 Decision and October 10, 2013 Resolution of the
However, these provisions refer to impeachment and recall of union officers, Court of Appeals in CA-G.R. SP No. 115402 are AFFIRMED.
and not expulsion from union membership. This is made clear by Section
2(e) of SO ORDERED.
the same Article XV, which provides that "(t)he union officers impeached
shall 'IPSO FACTO' to [sic] be considered resigned or ousted from office and
shall no longer be elected nor appointed to any position in the union." In
short, any officer found guilty of violating these provisions shall simply be
removed, impeached or recalled, from office, but not expelled or stripped of
union membership.

It was therefore error on the part of PORFA and petitioners to terminate


respondent's employment based on Article XV, Section 1, paragraphs (e) and
(f) of the union's Constitution. Such a ground does not constitute just cause
for termination.

A review of the PORFA Constitution itself reveals that the only provision
authorizing removal from the union is found in Article X, Section 6, that is,
on the ground of failure to pay union dues, special assessments, fines, and
other mandatory charges.[38] On the other hand, grounds for disqualification
from membership may be found in Article IV, which states that-

Section 3. The following are not eligible neither [sic] for membership nor to
election or appointment to any position in the union:

a. Subversive or persons who profess subversive ideas.


b. Persons who have been convicted of crime involving moral turpitude.
c. Persons who are not employees of the company.[39]

These provisions do not apply in respondent's case. Although he was


eventually charged with estafa,[40] a crime involving moral turpitude,[41]
still, he has not been convicted of the crime. For this reason, he may not be
disqualified as union member.

Thus, for what he is charged with, respondent may not be penalized with
expulsion from the union, since this is not authorized and provided for under
PORFA's Constitution.

Contrary to petitioners' claim, Cariño v. National Labor Relations


Commission is not applicable here. In that case, the employee was terminated
on the basis of existing suspension and expulsion provisions contained in the
CBA and rules on discipline found in the union's Constitution. There are no
such provisions in PORFA's Constitution; neither has it been shown that
there are similar stipulations in the parties' CBA.

The matter of respondent's alleged failure to return petitioners' P300,000.00


which was lent to PORFA is immaterial as well. It may not be used as a
ground to terminate respondent's employment; under the Labor Code, such a
contribution by petitioners to PORFA is illegal and constitutes unfair labor
practice.

ART. 248. Unfair labor practices of employers. - It shall be unlawful for an


employer to commit any of the following unfair labor practice:

xxxx

(d) To initiate, dominate, assist or otherwise interfere with the formation or


administration of any labor organization, including the giving of financial or
other support to it or its organizers or supporters;[42] (Emphasis supplied)

This could be an opportune time for the union to consider amending its
Constitution in order to provide for specific rules on the discipline of its
members, not just its officers. After all, it is given the right under the Labor
Code, "to prescribe its own rules with respect to the acquisition or retention
of membership."[43] But it may not insist on expelling respondent from
PORFA and assist in his dismissal from UPI without just cause, since it is an
unfair labor practice for a labor organization to "cause or attempt to cause an
employer to discriminate against an employee, including discrimination
against an employee with respect to whom membership in such organization
has been denied or to terminate an employee on any ground other than the
DEL PILAR ACADEMY vs. DEL PILAR ACADEMY EMPLOYEES employees. As regards the proposal to amend the provision on summer
UNION vacation leave with pay, DEL PILAR alleged that the proposal cannot be
G.R. No. 170112 | 2008-04-30 considered unfair for it was done to make the provision of the CBA
NACHURA, J.: conformable to the DECS' Manual of Regulations for Private Schools.[4]

Before this Court is a petition for review on certiorari assailing the July 19, On October 2, 1998, Labor Arbiter Nieves V. De Castro rendered a Decision,
2005 Decision[1] of the Court of Appeals (CA) in CA-G.R. SP. No. 86868, viz.:
and its September 28, 2005 Resolution[2] denying the motion for
reconsideration. Reviewing the records of this case and the law relative to the issues at hand,
we came to the conclusion that it was an error on [the] part of [DEL PILAR]
Following are the factual antecedents. not to have collected agency fee due other workers who are non-union
members but are included in the bargaining unit being represented by [the
Respondent Del Pilar Academy Employees Union (the UNION) is the UNION]. True enough as was correctly quoted by [the UNION] Art. 248, to
certified collective bargaining representative of teaching and non-teaching wit:
personnel of petitioner Del Pilar Academy (DEL PILAR), an educational
institution operating in Imus, Cavite. Employees of an appropriate collective bargaining unit who are not members
of the recognized collective bargaining agency may be assessed a reasonable
On September 15, 1994, the UNION and DEL PILAR entered into a fee equivalent to the dues and other fees paid by members of the recognized
Collective Bargaining Agreement (CBA)[3] granting salary increase and collective bargaining agreement: Provided, that the individual authorization
other benefits to the teaching and non-teaching staff. Among the salient required under Article [241], paragraph (o) of this Code shall not apply to the
provisions of the CBA are: non-members of the recognized collective bargaining agent.

ARTICLE V As it is, [DEL PILAR's] unwarranted fear re-individual dues [without]


authorization for non-union members has no basis in fact or in law. For
SALARY INCREASE receipt of CBA benefits brought about by the CBA negotiated with
[petitioners], they are duty bound to pay agency fees which may lawfully be
SECTION 1. Basic Pay - the ACADEMY and the UNION agreed to maintain deducted sans individual check-off authorization. Being [recipients] of said
the wage increase in absolute amount as programmed in the computation benefits, they should share and be made to pay the same considerations
prepared by the ACADEMY and dated 30 June 1994 initialed by the imposed upon the union members. [DEL PILAR], therefore, was in error in
members of the bargaining panel of both parties, taking into account refusing to deduct corresponding agency fees which lawfully belongs to the
increases in tuition fees, if any. union.

SECTION 2. The teaching load of teachers shall only be Twenty-Three (23) Anent the proposal to decrease the coverage of the 11th and 12th month
hours per week effective this school year and any excess thereon shall be vacation with pay, we do not believe that such was done in bad faith but
considered as overload with pay. rather in an honest attempt to make perfect procession following the DECS'
Manuals. Moreso, it is of judicial notice that in the course of negotiation,
SECTION 3. Overloadpay (sic) will be based on the Teachers' Basic Monthly almost all provisions are up for grabs, amendments or change. This is
Rate. something normal in the course of a negotiation and does not necessarily
connote bad faith as each every one (sic) has the right to negotiate reward or
SECTION 4. The ACADEMY agrees to grant longevity pay as follows: totally amend the provisions of the contract/agreement.
P100.00 for every 5 years of continuous service. The longevity shall be
integrated in the basic salary within three (3) years from the effectivity of this All told while there was error on [the] part of [DEL PILAR] for the first issue,
agreement. [it] came through in the second. But as it is, we do not believe that a finding
of unfair labor practice can be had considering the lack of evidence on record
ARTICLE VI that said acts were done to undermine the union or stifle the member's right
to self organization or that the [petitioners] were in bad faith. If at all, it's (sic)
VACATION LEAVE WITH PAY error may have been the result of a mistaken notion that individual check-off
authorization is needed for it to be able to validly and legally deduct
SECTION 1. Every faculty member who has rendered at least six (6) assessment especially after individual[s] concerned registered their
consecutive academic semester of service shall be entitled to the 11th month objection. On the other hand, it is not error to negotiate for a better term in
and 12th month pay as summer vacation leave with pay. They may, however, the CBA. So long as [the] parties will agree. It must be noted that a CBA is
be required to report [and] undergo briefings or seminars in connection with a contract between labor and management and is not simply a litany of
their teaching assignments for the ensuing school year. benefits for labor. Moreso, for unfair labor practice to prosper, there must be
a clear showing of acts aimed at stifling the worker's right to self-
SECTION 2. Non-teaching employees who shall have rendered at least one organization. Mere allegations and mistake notions would not suffice.
(1) year of service shall be entitled to fifteen days leave with pay.
ACCORDINGLY, premises considered, the charge of unfair labor practice
The UNION then assessed agency fees from non-union employees, and is hereby Dismissed for want of basis.
requested DEL PILAR to deduct said assessment from the employees'
salaries and wages. DEL PILAR, however, refused to effect deductions SO ORDERED.[5]
claiming that the non-union employees were not amenable to it.
On appeal, the National Labor Relations Commission (NLRC) affirmed the
In September 1997, the UNION negotiated for the renewal of the CBA. DEL Arbiter's ruling. In gist, it upheld the UNION's right to agency fee, but did
PILAR, however, refused to renew the same unless the provision regarding not consider DEL PILAR's failure to deduct the same an unfair labor
entitlement to two (2) months summer vacation leave with pay will be practice.[6]
amended by limiting the same to teachers, who have rendered at least three
(3) consecutive academic years of satisfactory service. The UNION objected The UNION's motion for reconsideration having been denied,[7] it then went
to the proposal claiming diminution of benefits. DEL PILAR refused to sign to the CA via certiorari. On July 19, 2005, the CA rendered the assailed
the CBA, resulting in a deadlock. The UNION requested DEL PILAR to decision, affirming with modification the resolutions of the NLRC. Like the
submit the case for voluntary arbitration, but the latter allegedly refused, Arbiter and the NLRC, the CA upheld the UNION's right to collect agency
prompting the UNION to file a case for unfair labor practice with the Labor fees from non-union employees, but did not adjudge DEL PILAR liable for
Arbiter against DEL PILAR; Eduardo Espejo, its president; and Eliseo unfair labor practice. However, it ordered DEL PILAR to deduct agency fees
Ocampo, Jr., chairman of the Board of Trustees. from the salaries of non-union employees.

Traversing the complaint, DEL PILAR denied committing unfair labor The dispositive portion of the CA Decision reads:
practices against the UNION. It justified the non-deduction of the agency
fees by the absence of individual check off authorization from the non-union
WHEREFORE, premises considered, the petition is PARTIALLY summer vacation leave with pay for two (2) months for teaching staff who
GRANTED. The assailed resolution of the NLRC dated April 30, 2004 is have rendered six (6) consecutive semesters of service. For the non-teaching
hereby MODIFIED. Private respondent Del Pilar Academy is ordered to personnel, the UNION worked for their entitlement to fifteen (15) days leave
deduct the agency fees from non-union members who are recipients of the with pay.[13] These provisions in the CBA surely benefited the non-union
collective bargaining agreement benefits. The agency fees shall be equivalent employees, justifying the collection of, and the UNION's entitlement to,
to the dues and other fees paid by the union members. agency fees.

SO ORDERED.[8] Accordingly, no requirement of written authorization from the non-union


employees is needed to effect a valid check off. Article 248(e) makes it
DEL PILAR filed a motion for reconsideration of the decision, but the CA explicit that Article 241, paragraph (o),[14] requiring written authorization is
denied the same on September 28, 2005.[9] inapplicable to non-union members, especially in this case where the non-
union employees receive several benefits under the CBA.
Before us, DEL PILAR impugns the CA Decision on the following grounds:
As explained by this Court in Holy Cross of Davao College, Inc. v. Hon.
I. IN PROMULGATING THE CHALLENGED DECISION AND Joaquin[15] viz.:
RESOLUTION, THE HON. COURT OF APPEALS DISREGARDED THE
FACT THAT THE ANNUAL INCREASE IN THE SALARIES OF THE The employee's acceptance of benefits resulting from a collective bargaining
EMPLOYEES WAS NOT A BENEFIT ARISING FROM A COLLECTIVE agreement justifies the deduction of agency fees from his pay and the union's
BARGAINING AGREEMENT, BUT WAS MANDATED BY THE entitlement thereto. In this aspect, the legal basis of the union's right to
DIRECTIVE OF A GOVERNMENTAL DEPARTMENT; and agency fees is neither contractual nor statutory, but quasi-contractual,
deriving from the established principle that non-union employees may not
II. CONSIDERING THE ANNUAL SALARY INCREASE OF NON- unjustly enrich themselves by benefiting from employment conditions
UNION MEMBERS WAS NOT A BENEFIT ARISING FROM THE CBA, negotiated by the bargaining union.
THEIR INDIVIDUAL WRITTEN AUTHORIZATIONS ARE STILL
REQUIRED TO ALLOW PETITIONER ACADEMY TO LEGALLY By this jurisprudential yardstick, this Court finds that the CA did not err in
DEDUCT THE SAME FROM THEIR RESPECTIVE SALARY.[10] upholding the UNION's right to collect agency fees.

The issue here boils down to whether or not the UNION is entitled to collect WHEREFORE, the petition is DENIED. The Decision and Resolution of the
agency fees from non-union members, and if so, whether an individual Court of Appeals in CA-G.R. SP No. 86868, are AFFIRMED.
written authorization is necessary for a valid check off.
SO ORDERED.
The collection of agency fees in an amount equivalent to union dues and fees,
from employees who are not union members, is recognized by Article 248(e)
of the Labor Code, thus:

Employees of an appropriate collective bargaining unit who are not members


of the recognized collective bargaining agent may be assessed reasonable
fees equivalent to the dues and other fees paid by the recognized collective
bargaining agent, if such non-union members accept the benefits under the
collective bargaining agreement. Provided, That the individual authorization
required under Article 241, paragraph (o) of this Code shall not apply to the
non-members of recognized collective bargaining agent.

When so stipulated in a collective bargaining agreement or authorized in


writing by the employees concerned, the Labor Code and its Implementing
Rules recognize it to be the duty of the employer to deduct the sum equivalent
to the amount of union dues, as agency fees, from the employees' wages for
direct remittance to the union. The system is referred to as check off.[11] No
requirement of written authorization from the non-union employees is
necessary if the non-union employees accept the benefits resulting from the
CBA.[12]

DEL PILAR admitted its failure to deduct the agency fees from the salaries
of non-union employees, but justifies the non-deduction by the absence of
individual written authorization. It posits that Article 248(e) is inapplicable
considering that its employees derived no benefits from the CBA. The annual
salary increase of its employee is a benefit mandated by law, and not derived
from the CBA. According to DEL PILAR, the Department of Education,
Culture and Sports (DECS) required all educational institutions to allocate at
least 70% of tuition fee increases for the salaries and other benefits of
teaching and non-teaching personnel; that even prior to the execution of the
CBA in September 1994, DEL PILAR was already granting annual salary
increases to its employees. Besides, the non-union employees objected to the
deduction; hence, a written authorization is indispensable to effect a valid
check off. DEL PILAR urges this Court to reverse the CA ruling insofar as
it ordered the deduction of agency fees from the salaries of non-union
employees, arguing that such conclusion proceeds from a misplaced premise
that the salary increase arose from the CBA.

The argument cannot be sustained.

Contrary to what DEL PILAR wants to portray, the grant of annual salary
increase is not the only provision in the CBA that benefited the non-union
employees. The UNION negotiated for other benefits, namely, limitations on
teaching assignments to 23 hours per week, additional compensation for
overload units or teaching assignments in excess of the 23 hour per week
limit, and payment of longevity pay. It also negotiated for entitlement to

También podría gustarte